Está en la página 1de 488

DEMO : Purchase from www.A-PDF.

com to remove the watermark

@~~@~@©0@
80~D0D~~
PRElENTACIÓN

Esta obra está orientada a todos los estudiantes Pre-universitarios de


ciencias e ingeniería, inclusive para todos aquellos de centros de estudios
secundarios que aspiren a mantenerse en un buen nivel académico.

La presente publicación contiene los Exámenes de Admisión toma-


dos en la UNIVERSIDAD NACIONAL DE INGENIERÍA; pero conside-
rando sólo los últimos, en los cuales se ha modificado la estructura de los
mismos. Estos cambios obedecen a que últimamente este centro de estu-
dios, ha puesto interés en que sus ingresantes tengan una preparación más
integral, razón por la cual ha introducido: el razonamiento verbal, el razona-
miento lógico matemático, y además le ha dado mayor importancia a la
cultura en general.

Esta obra considera los exámenes de admisión a partir del año 2001,
fecha en la cual las tres pruebas que se toman, en tres difrentes días, se
presentan como se indica acontinuación:

1.- Aptitud Académica y cultura general.

2.- Matemática: Aritmética, Álgebra, Geometría y Trigonometría.

3.- Física y Química.

En la elaboración de esta obra se ha tenido en cuenta el nivel académi-


co en que se encuentran la mayoría de los estudiantes, especialmente los
que egresan de los centros educativos secundarios; razón por la cual los
"problemas" o preguntas se desarrollan en forma simple, cuidando de
emplear conocimientos básicos o de fácil acceso para un alumno pre-
universitario en general. Con este aporte, esperamos complementar la bue-
na formación académica que necesita el estudiante para adquirir esa destre-
za y eficiencia necesaria que le permitirá ingresar a la universidad.
También debemos recordar al estudiante que una buena formación aca-
démica radica en tener una "teoría sólida" antes de empeazar a resolver un
"problema", pués ésto les ahorrará tiempo y energías, elementos valiosos
que un estudiante competitivo debe saber explotarlos.

Por último, tenemos que hacer resaltar el aporte del equipo intelectual
y técnico de esta empresa Editora, que permitió que se hiciera realidad la
presente publicación, con la cual estamos seguros estar aportando con la
. comunidad estudiosa.
ÍNDICE GENERAL

1. Aptitud Académica y cultura general. 1 - 332

2. Matemática: Aritmética, Álgebra, Geometría y Trigonometría. 1 - 301


. .

2. Física y Química 1 - 312


;

MATfMATICA
EXÁMENES DE ADMISiÓN
~- í\- -
'-/ ~IL_
UNIVERSIDAD NACIONAL DE
INGENIERíA

r
LIMA - PERU
CONTENIDO 2

Examen de Admisión 2001 - I 1.


Solucionario : 6

Examen de Admisión 2001 - 11 ..........................................................•...........•.... 16


Solucionario 20

Examen de Admisión 2002 - I .. 32


Solucionario 36

Examen de Admisión 2002 - 11 47


Solucionario 52

Examen de Admisión 2003 - I : 65


Solucionario ; 70

Examen de Admisión 2003 - 11 ....................•........•................•............................. 85


Solucionario 90

Examen de Admisión 2004 - I : 105


Solucionario 110

Examen de Admisión 2004 - 11 ....•....•.. : ..........•.............•................................... 127


Solucionario 132

Examen de Admisión 2005 - I 149


Solucionario 154

Examen de Admisión 2005 - 11 ............•............................................................ 171


Solucionario 176

Examen de Admisión 2006 - I .. 192


Solucionario 197

Examen de Admisión 2006 - 11 .............................•..•..•...............•..............•....... 213


Solucionario 218

Examen de Admisión 2007 - I 236


Solucionario 241

Examen de Admisión 2007 - 11 258


Solucionario 262

Examen de Admisión 2008 - I 280


Solucionario 285
MATEMÁTICA
__ U_N_I 2_0_0_1_-I_M_A_:r_EM_Á_T_IC_A_ ••••• =~El.J 1

MATEMÁTICA 6. Sea el conjunto A ={ XE I? / ~ EZ}

El elemento de A que se encuentre en la posición 50 es:


1. Si 20< p+q<30 y ,/ +l/ < 2,.2 .xlonde tr. q y r A) 2104 B) 2205 C) 2301
son números primos. Entonces P +q+r es igual a: D) 2402 E) 2403
A) 37 B) 35 C) 33 D) 30 E) 2R
7. Si A es el conjunto solución de la ecuación
2. Si el número am;;; es un cuadrado perfecto, enton- 2x2 + 2x - 3/,2 +x +3 = 3 , 'entonces la suma de los
ces la suma de los dígitos de dicho número es:
elementos de A es:
A) 12 B) 14 C) IR Dl22 E) 26
A) -3 B) -1 .ci i D) 3 E)4

3. Cada año se deposita mil nuevos soles en una cuen-


8. La grúlica de la función .f definida por:
ta bancaria que produce 5% de interés semestral y con
el mismoperio." J de capitalización. ¿Qué capital se f(X)=lx-21+lx-41 es:
trcudrá inrnedi. unente después de haberse efectuado
el tercer dcpóvuo?
Al3ó7-1 B) 4801 C) 3318
Dl6¡¡UI E) 3200

4. Un joyero tiene un lingote de oro de ley 0.<)00 que


pesa 1500 g. ¡,Qué cantidad de oro puro (en g) tendrá
que añadir al lingote para elevar su ley a 0.<)25?
A) 350 B) 500 C) 600 D)750 E) 300

5. Las notas de un examen de Aptitud Académica están


distribuidas en el siguiente histograma de frecuencias:

Número de
alumnos
14

Notas 9. Dada la ecuación 2x2 -r I./X + 30 = () Y XI' Xl sus


¡,Cuál es 'la nota promedio del examen? raíces. ¿Para qué valores de 111 se cumple la relación
A) 10,12 B) 11.08 C) 12,02 .\'1 _:1 '1

D) 13,12 El 14,06 "2 - '5"


A) Iml= 16 B) Iml= 10 C)lml= 14
D) Iml=8 E) Iml=20 14. En la suma combinatoria S == 2 + (n) (11-2. ')
donde nEN, n~3
10. Un agricultor quiere levantar una cerca alrededor
de un terreno rectangular que está ubicado en la ribera al simplificar se obtiene siempre:
de un TÍo. usando 1000 m de material, ¿cuál es el área A) Un número primo.
más grande que puede cercar. considerando que no va a B) Un cuadrado perfecto.
poner una cerca a lo largo del TÍo?
e) Un número impar.
D) Un número par.
E) Un múltiplo de 4.

A) 50 000
L.--J
nl • B) 62500 III C) 67 500 nl
15. Dada la función polinomial:

p(x)=x3
. deP(lOOOI).
-10000x2 -1 0002 .r+9999 . Calcule el valor

., 2 A) -3 B) -2 ci -1 D} O E} I
D) 100000 m- E) 125 000 m
16. Hay 12 puntos A. /3, ... en un plano dado, no en-
2 2 centrándose tres de éstos sobre la misma recta, en nin-
11. Dado el sistema: .r +4 y =25
{ x+2y=7 gún-caso.
Entonces el número de rectas que son determinadas por
los puntos, y el número de rectas que pasan por el
Si 2v>x • entonces el valor de Les:
. J puntoA. son respectivamente:

A) I B) 3/2 C)2 D) 8/3 E)3 A) 11 ; 6(, B) 66 ; 11 C') 11 : 55


D)55; II E) 9 ; 55
12. Dadas las matrices
17. Sea n>2 un número natural y al ,a2' ._.l/" ; los

c=[: ~] yD=[~ :]
rénninos de una progresión geométrica de razón 1/11.
¿Cuál es el mínimo valor de (11 para que:

Entonces se puede afirmar que e!I ')


D es: (al+a2+_·+a,,) sea un número natural?

A) 9[1 78] A) I B)1l C) (Il-I)!

D) (n-1r-1

18. i ..cierta base b un número N ricnc 1;. forma


r I !1 I(h} ; en la base b-I dicho mimero .iene la for-
13. El valor de la expresión: ma 15ABC(h-I) donde las 3 letras son dígnos.

Entonces el valor de b es:

es: A)6 B) 8 C) 10
D) I1 E) mayor que 11
A)-l E) 1
19. Sean x, y, z números naturales, donde

f +-f + I~ = 1,4375 .¿Cuántas ternas solución

(x, y. z) se obtienen, en las cuales z = 3?

A)5 8)4 C)3 D)2 E) I

20. Sea A.B = 53 361 el producto de dos números


enteros positivos donde A tiene dos cifras, B tiene tres
cifras y es divisible entre 3, entonces el valor de B, es:

A) 231 8) 539 C) 639


B)-I --
2
(e2
+b
<p
2
)

D) 693 E) 837

21. Dada la siguiente identidad trigonométrica

25. Sea ABeD un cuadrilátero, donde Be / / AD:


2(3X) 2(X)
cos T -sen '2 = Aco~2(~)+B sea P E Be , AP es bisectriz del ángulo BAD; suponga
----> ,
co.\) x - sen2 x . 2
también que De es bisectriz exterior del ángulo D
El valor de AB es: del triángulo ABD. Si BD - AB = 3, determine la lon-

A) -2 8) -1 C)O D) I E)2 gitud de FC,

A)3 8)6 C)9 O) 12 Ell5


22. En un triángulo isósceles, las medianas trazadas
de sus vértices de ángulos iguales se intersecan perpen- 26. En un triángulo obtusángulo ABe (obtuso en el,
dicularmente. Entonces el coseno de uno de los ángulos
AB = I/l Y Ae = 11. La circunferencia ex-inscrita
iguales es:
relativa al lado Be determina sobre este lado el punto
A) l.
3
C) J3 de tangencia P y la circunferencia inscrita es tangente a
2 dicho lado en el punto Q. Calcular PQ.

I I 'A) m+n 8) .!!!:..=!.!. C) 11/+21/


D) JIO E)2J3 4 2 4
D) 2m-3n E)m-n

23. El valor de la expresión (cos~ + isen~ r7


es:
27. En la figura
rico de 3xo-12°
L¡ // L¡ Y IJj // L4 ,el valor numé-
es:
A) i 8H C)-i D) i E)I+i

24. En la figura se tiene un ángulo central de medida


. <p radianes y arcos de longitudes by c respectivarncn-
te. Entonces el área de la' región sornbreada mide.
~ 4~~::::::::::i:":::;"i:.m:;[I==~U~N~I~2~00~1~-CI
~M~A~J~E~M~Á~J!IC~A~:J
A) 15° B) 16° C) 17° D) 18° E) 19°

28. En el triángulo ABC, DE AB tal que AD =9 Y


DB = 6. Se trazan los segmentos DFparalelo aBC
(F en AC) y DE que intercepta a BC en e .( enla
prolongación de AC), de modo que 4AF = 3FE. Si
De = 8, calcular Gii. B

A) 10 B) 6 C)8 D)7 E) 17
6 rr ( R + Rr + r2 I
B)3 R+r I
I

29. En el triángulo rectángulo la mediana relativa a


un cateto de longitud b se interseca perpendicularmen- rr(R+2Rr+r2J
te con la mediana relativa a la hipotenusa, Entoncesla C)"3 R+r D) 3~(R+r)2
_ longitud del otro cateto es:

e) 4,[3.J
E):3
2rr (3Rr - r
R+r
2
J

32. En la gráfica, si AC = 5, la suma de las coordcna-


das de C es:
30. En la figura mostrada: E = 1,ce / /QF, e
FM 6 y Y1
es el punto medio de MQ y el área de la región triángular

Lf
PQM es 100 m" . Hallar el área de la región sombreada
(en II?).
Q

A(1;2) B(4;2)
x

A)4 B) 10 C)8 D)6 E)9

33. El área total de una pirámide regular pcntagonal

A) 15 B) ill C) 150 es de45 11


2
y su área lateral 25 L? . El coseno del ángu-
7 7
lo diedro que forma una cara lateral con la base de la
D) 230 E) 300 pirámide es:
6 7
1
A) J3 B) .!
C) 12
31. En la figura los radios de las dos circunferencias 2 5
concéntricas miden R y r (R>r) . La diferencia de D) .1.
longitudes de los arcos BB' y AA' es 2; r y el ángulo 5
AGB' mide 1200. Calcular la suma de las longitudes de
los arcos AA' Y BB'.
::~~:==:r~=:::=,-
__ U_N_I_2_0_0_1_-_I_M_A_:r_E_M_A_-T_I_C_A_ •••••• J~::.::~=-~0
34. Se tiene un paralelepípedo rectangular, dondc las A) 161,56 B) 163,56 C) 165,56
dimensiones de las bases son 5 cm. y 8 cm. y su altura 12
D) 167,56 E) 169,56
cm. Un agujero que va desde la base superior hasta la
base inferior tiene la forma de un prisma triangular rec-
to, cuyas bases son triángulos equiláteros con aristas de 38. En la figura adjunta, la longitud del segmento AB
longitud 3 un. Calcular el área de la superficie total del es: .
sólido determinado.
e
A)500-2.fj B)500-%~ C)500-%fj
4 3

D)500-1fj E)500-~fj
2
4
35. Algirar un rectángulo de lados a y b alrededor del
lado b se obtiene un cilindro de 288 1t uJ de volumen y 2
A~~~llL--------~B
al girar el rectángulo alrededor del lado a, se obtiene un
cilindro de 384 1t u' de volumen. A) 2fj C)4J:.
Determine el área del rectángulo. D) 5J:.

A) 38 u"
39. El mayor valor que toma la función
D) 68u"
f(x )=ws2x+ J.I'en2x+2 es:
36. Se tiene una pirámide regular E - ABCD, con base
cuadrangular. Sea MNQP la sección determinada por A)2+ JiO B)6 C)3+JTO
un plano secante a la superficie lateral. Si EM = 3,
D) 1+ JiO E)5
EQ = 5 = 5 y EP = 6; entonces EN es igual a:

A) 2,72 B) 2,55 C) 3,11


40. En la identidad trigonométrica
D) 3,55 E) 4,11
2senx+ 3clisx=kclis(x-ex)
37. Un molinete de riego tiene un alcance de 12 m y determinar tan ex .
2
un ángulo de giro de 1350• Calcular 'el área (en 111. ) del
sector circular mojado por el molinete. A)_2_ B)1- C)~
Usar rr=3.14.
m 3
Jf3
E) Jf3
3
~~::~::::~~3~~U~N~I~20_0_1-_I_M_A_JE_M_A_·T_IC_A I~.::::::::::::~

SOLUCIONARIO
MATEMÁTICA Se pide: L cifras = 2 (a + c)
1. Datos: = 2 (11)
20 <p + q < 30 ... ( 1) = 22
? 2 Clave: O
pr+o 2
--2-=r ...(2)
3. Datos:
P , q y r: números primos ...(3) Interés: 5% semestral
De(l)y(3): capital: C = 'SI 1000 (Capitalización Semestral)
p, q y r pueden tomar los valores de: Primer depósito
1; 3; 5; 7: 11; 13; 17; 23; 29 6 meses 6 meses 6 meses 6 meses
si: P = 17 1\ q =7 ~~~~
C 1,05 C (1,05)2C (1,05)3C (1,05)4C
Segundo depósito
En (2):

~ r = 13 (es número primo) .


1--- l año ····;
~~
••.C
6 meses

(I,OS)C
6 meses

(I,OS)2C
Cumpliendo los valores de p, q y r con las tres condi-
ciones. Tercer depósito

p+q+r= 17+7+ 13
.
1 - - -- 2 años ·C
= 37 Finalmente, al haberse efectuado el tercer depósito el
Clave: A capital total será:

2. Por condición: aacc es cuadrado perfecto


CT =(1.0SyIC+(1.05)2C+C
Descomponiendo polinómicamente:
= C[(I.05)4 +(1.05)2 + IJ
a(/cc = 1OOGa+ Cc:
~ S/.IOOO[3.3IS]
=IOO(lla)+11c
=S/3318
= 11(IOOa+ c)
Clave: C
De dondi:: (1OOa+c) tiene que ser tl para que aazc sea
cuadrado perfecto 4. 'Ley inicial: Lo = 0,9

IUOa+c= 11 Por definición:

a+c = 11

Pero, (/,c~9 ~ u+c< IS = (0,9)( 1500 g)


~ a+c= 11 = 1350 s

¡¡:
Cuando agregamos x í: de oro la nueva leyes L, = 0,925
7. Operando:
L - ...,W",o:,-,-,m'-.+_x
1- WT +x

2
o 925 = 1350 + x 2( x + x + 3) - 3J x2 + x + 3 - <) =O ... (1)
Reemplazando:
, 1500+x
x= 500 Si: Jx 2
+X +3 =a =) a> O ... (2)

Se añadirá al lingote 500 g de oro


Luego en (1):
Clave: B
2c?-3a-9=0
5. Del enunciado construimos la siguiente tabla de da-
tos: (2a+3)(a-3)=O
x¡ f¡ xJ¡
Intervalos
[05;08) 6,5 6 39 =) a=3
[08; 11) 9,5 14 133
De (2): a> O =) a=3
[11; 14) 12,5 16 200
Luego en (1):
[14; 17) 15,5 10 155
[17; 20] 18,5 4 74 Jx 2
+X + 3 = 3
Total 'n=50 601 x2 +x - 6 = O
(x+3)(x-2)=0
Por definición:
=) x=-3 1\ x=2
Promedio =X=_L.._,·_,- "xl:
/1.
.. C.S.=A={-3;2}

. 601 finalmente la suma de los elementos de A es:


De datos: x=5ü=12,02
-3 + 2 =-1
Clave: C
Clave: B
6. A ={ X E IR / ~ E Z }
S·f(x) = Ix - 21 + Ix - 41
Por teoría de raíces: '¡;=¡;:o:O
Para x < 2:

f(x )=-(x- 2)-(x-4)=6- 2x

Para 2Sx<4':
=) .¡;=¡ = O, 1, 2, ..., 49 (número entero)
t t t t f(x)= (x-2)-(x-4)=2
Posición: 1 2 3 SO

Finalmente, el elemento deA que corresponde a la posi-


ción 50 será: {(x) = (x - 2) + (x - 4) = 2x - 6
.¡;=¡ =49
x<2
=) x=(49)2 +1=2402 =) f(x)=¡6~2X , 2Sx<4
Clave: D 2x - 6 , x;:O:4
Gráfica:
dS
-= 1000-4x=0
e/x

~ x= 250

En (1): 2 (250) + Y = 1000


~ y= 500
Clave: C
Luego: SllIax =125000 I/?

9. Datos: 2x2 +lI1x+30=0 ... (1) Clave: E

... (1)
... (2)
x+2y=7 ... (2)
De la ecuación (1) por propiedad de raíces:
7-x
De (2): y=--
. 2
In
XI +X2 =-2 ... (3)
Reemplazando el valor de y en (1):
30
xl·x1=T ... (4)

ry
De (2) Y (4) : XI =3 J\ XI =-3 [-7x+12=0

(x-3)(x-4)=0
Reemplazando en (3):
~ xl=3

1+5=-.!!! -3-5=-/11 reemplazando los valores de X en (2):


- 2 J\ 2
11I=-16 J\ m=16
Para xI =3 :
Clave: A
(3)+2.1'1 ==7 ~ )"1=2
10. Para x} =4 :
Río

l _x
Terreno

y
I Por condición:
Para: xl=3
x <2Y

YI=2

3 < 2 (2) (cumple)


• Perímetro: 2x + Y = 1000 ... (1)
• Área: S =xy ...(2)
Para: x1'=4 )'ry=l
.- 2
De (1) Y(2):
S=x(IOOO-2x) (no cumple)
2
=1000x-2x

Para que S sea máximo la derivada del área S respecto a Finalmente:


x debe ser igual a cero:
Clave: B

12.
::========[:U~N:I

e=[:
lMJitAi.'·"ii

~]
2~O~O~1~-I~M~A~:r~E~M~A~·T~I~C~A~J======.:@:flZ:&.i:
..

5,=--
- 1- r
(lo
=--=
2
'3
1--
3
1
1
:::'m.0

2
e =[: ~][: ~]=[~ ~] Luego en la expresión (*):

e] =_[12
~][: ~]=[~ ~] Clave: C

ne N 1I~3

8
e = [~ ~] n!
-,---,.--
(II-I)!
+ -'-----,~
(n-2)!2! (n-3)!2!
n(I1-1) (11-1)(11-2)
Además:
D=[~ :] = --2-

= 11
2 - 211 +
+ -'----:-2-~
1

D = [~ :][~ :] = [~ ~]
2

Finalmente, se obtiene un cuadrado perfecto.

D] = fl Clave: B
'-
O ~][~ :]=[~ ~]
15. Aplicando el método de factorización:

p( x) = x] - IOOOOx- 21 0002x + l)l)l)l)

= x] -IOOOI(x2+x-I)+x2 -x-2

= x] +x -x-IOOOI(x2 +x-I)-2
2

= x(x2 +x-I)-IOOOI(x2 +x-I)-2

= (x2 +x'-I)(x-IOOOI)-2
Clave: D
~ P(IOOOI)=-2
13 ..Ordenando sus términos
Clave: B
D =J._2+J._2.<-J._2+ ... ...(*) 16. Los 12puntos (¡I, 13, e ...) forman un dodec.igono.
2 3 4 ~ & 27
por lo tanto el número de rectas N 11 son determinadas
=[t+¡+t + ...] - [~+ %+~+ ...] por el número de diugonales totu les N [l mas los
'------y-----' • 11=12 lados:
SI: Serie geomérrica S2: Serie gométrica
de razón 1/2 de razón 1/3 NR=ND+11
11(11- 3)
1 =---+11
2
(lo '2
Donde: 5 = -¡-=-;:- = --1 = 1
I 12(12-3)
1--
2
= 2 +12
=(i(,
El número de rectas que pasan por el punto (A), es el I
1 1 1 1 I
1
número de diagonal es trazadas desde un vértice (N,,) I
1 1 2 3 I 5
más dos lados.
1 2 3 I 4 I
5
I
I
- - --
1 1 3 I 6
NR=N,,+2 I
1 3 I 6 I 10
=(n-3)+2 I - - --
1 1 I
4
=(12-3)+2 1 I
I
4 :_l.? __
=11 1 I 1
I
1 I
5
Clave: B
luego: N = 15(10)(10)5(h_l)
1 7. Progresión geornétrica: al ,a2' ....(111
De la base: (b-I»IO ~ b>11

Razón: ~ ,donde: 17 E IN ; 17 > :. b es mayor que 11


11
Clave: E

... (1)
Por teoría:

z=3
[ 1- (11"]
... (2)
{/I
J
X,y.ZE IN (3)

De (2) en (1

.2.+2:.+~ = 14175
2 4 16 .-

=_{_/I_[ + 11 11-2 + ... +1]


~ 2x+
J,.
\' = 5

ti
11-1
11
11-1
1
'\

Si Sil E IN mínimo: (/1 = 1/"-1


Clave: E
De (3):
1; 3
1

Obteniéndose 3 temas: (O; 5; 3) : (1; 3: 3) : (2: 1; 3)

18. Dato: N = 111 i(h) = 15AflC(I,_I) Clave: C

Llevando N = 1111(1.) a has e 10: 20. Datos: A.S = 53361 (1)


A =ab ... (2)
N = 1 X b4 + 1 X b3 + 1X b'l + 1 x b + 1 x l
B = x)'z = 3 ... (3)
=b4+b3+b2+b+1

Luego N debemos lIevarlo a la base (b - 1) por divisio- De (1): A.B=9xI12x72


nes sucesivas:
De (2) y (3): B = 3 = 9 xii x 7 - 693
A = 11 x 7 =77
Clave: 1)
21. Recordando:

• cos2 x - .I'en2
23. E = (COSI+i sen ;)77
x = cO.l'(2x) ---.,...... ---.,......
o 1
• cos2 x - .fen2 y = cos(x + y )cos(x- y) = ¡77
• cos2x=2cos2 x-I .
·4+1
=/

=i
Clave: D

24. Por teoría:


cos( 2x) cos x
cos(2x) 1 L2
S=---
2 e
=cos x

=2cos (f)-1 2 ...(2) En el gráfico:

Por comparación de (1) Y (2) :


A = 2 Y B=-I
=} A.B = (2)(-1) ~-2
Clave: A

22. Graficando de acuerdo al enunciado:


De donde: S = S OAB - S OMN
1 c2 1 b2
G: Baricentro
=2$-2$

Medianas: BH =~( ;b c
2 2
)

AN
Clave: A
CM
25. Graficando de acuerdo al enunciado:

A~~--~H~~~~C B a P x C

Propiedad: El baricentro G se ubica ~ BH del punto B

En el triángulo rectángulo BHC


Por pitágoras:

--BC= J (3a) 2
+a- ?
3

A 2<7
Del gráfico: t:J.ABP y t:J.DBC son isósceles,lue;go:
D

=aM =} m+n=a+x ... (ex)


Dato: m+n-a=3
=} CO.l'p = __(_l _ = __1_
[1M M De(a): (a+x)-a =3
=} x= 3
Clave: D
Clave: A
l' UN12001-1 MATEMÁTICA ~I
:.e.:

G~meZ

26. Graficando: Del gráfico:

MDF - MBC

3(1+ FC 3a
---=-

¿:; tl tlEDF - EGC


~
15
FC= 2a
9
... (rt.)

Al--- n _--.-1+-_ 8+..\ X

-¡;:;- = FC
Del gráfico:
De (a): 8+x x
.1I1=Il-a+x+b -¡;:;- = 2t;
~ 111 = 11+X + b-a ... (1)

.1Il+b=ll+a+x Clave: C
~ b - a = 11+ X - 11/ ... (2)
29. B
Reemplazando (2) en (1):
1I1=II+X+(I1+X-IIl)
~ X=II/-Il

Clave: E
27.

Por razones métricas en el ~ BAP

AG=h=JIIl(211l)=J2/11 ... (1)


Del gráfico: Ilx + 5x + 2x = 1800
x= 10 °
En el ~ AGP: '2)
(- _
)2 = 11/
2
+ 211/2
Luego: 3.\_12°=3(10°)_120
= 18" b
Clave: D ~ 11I= 2J1, ... (2)

28. Graficando de acuerdo al enunciado Enel ~AGB:

B Datos: AD = 9
DB = 6
De (1): x-? = ( JiIl1 )2 +(2111)2
AF= 3a

FE=4a ~ x= J6/11
De (2): = J6(~)
2" '
= ~J2
2
Clave: E
30. DalOS: • CC // QF Finalmente calculamos:

Q • e punto medio de QM
L + L •• 21t (:.rR - r2 )
• PF=a /\ FM =6a I 2 :. R +r

• SPQM = 1 (){)lIl
Clave: E

32. y

(7a)(2b) 2
Del gráfico: S PQM = 2 = 100m

... (*)

(3a)(b) x
SFGC =--2-
3 Del gráfico: x=4
= 2(ab)

De (*): = %C~O m 2
)
AC=5=j32+(y_2)2

150 25 = 32 +(y_2)2
=v : 2

=) y=6
Clave: C
Luego, la suma de las coordenadas del plinto C:
31. x+y=4+6=10
Clave: 8

33.

AA'=L¡
BE' = L2

... (1)

Del gráfico: ... (2)

De (1) Y (2) obtenemos:

Dato: 2 ... (1)


STOTAL = 45 u

2
SLATERAL = 25u ... (2)
0~=====;;~~;
..]__ U_N_I_2_00_1_-I_M_AT_E_M_A_·T_IC_A_.....IIª' ~~~;:;~=~;i::},H~z
De(l)y(2): SBASE =STOTAL -SLATERAL
Dato: V¡,=288n u"
=45u2 -25u2
Del gráfico: V¡,=1ta2 .h=288n /(" ... (1)
= 20u2

Del gráfico: SBIISE =~ I~ )=20

~ h=1i.. ... (3)


1

SBASE =5( }~l )=25

~ H=J!l ... (4)


I Dato: V;,=384n 11
3
... (2)

/¡ (8/1) 4 Del gráfico:


De (3) Y (4) : CO.l'Cl=-=-- =-
H (10/1) 5
nb2a = Y:,
Clave: B
De (*): nb2a = 384n u"
34.
M
3m
b2a = 384uJ ... (3)

I Multiplicando (1) y (3):


12m

(IV =(288)(384) ¡/,


1
............
L ....

5m
Finalmente se tiene:
8m
Del gráfico, el área total del sólido es: S =ab =48u2
Clave: B
2X(3Cm)2J3]
ST =2(8cmx 12cm)+2
[ 8ol/xScm 4
36.Graficando de acuerdo al enunciado:
E
+ 3( 12<11/ X 3 cm)2(Scmx 12 cm)

=5OOcm-
? 9J3
---CIII-
1

2
Clave: B

A~------------~
Superponiendo los triángulos EN? y MEQ:
Por teoría: tanl. A + 8) = tal/ A + l{tl/ 8
1-1(//1 Atan 8

/ana+wl/.(e+a)
Del gráfico: tan ( a + e + a) = -----:,:--~
I-/ana/a/l.(e+a)

9
l+~
x x
p x I_lx~
x x
x= 6J2

=) hsena(3-x)=3kmx.\·ene ... (1) ClllVe: E

39. f(x) = cosa + 3sen 2t + 2 ... (*)

Por propiedad se sabe que:


~ hsetHJ.=:'Ok 11I sene ... (2)

Dividiendo (1) entre (2): -J A2 + 82 s Asena+ 8cosa::; J A2 + 82

x En (*): {(x) = cos'Lx + 3.\'eIl2x+2


'III('X~
3-x=jQ x=2,7272
~12+32
Clave: A
=JI2+32 +2
37.
=JiO+2
Clave: A

40. Desarrollando y comparando:

2 sen x + 3 cos x = k CO,I' (x-a)

= k cosa CO.\' x + kseno. .\'{'I/X


"-v---' '--v--'
El área sombreada representa el área mojada por el 3 2
molinete
k cosa = 3

(3rc)(
2 ""4 12m
)2 k seno: =2
S=~= =16956m2 Dividiendo las dos últimas expresiones tenemos:
2 2 '
Clave: E k seno: 2
e kco,l'a = '3
38.
2
3 ~ lana= '3

Clave: B
4

2,
~:::::::::::::::J.1L q~
x ------1
MATEMÁTICA
1 • Un contratista dice que puede terminar, un tramo de
5. Si el conjunto A ={XE R.! Jx2
-I-~ ~O}
una autopista en "a" días si le proporcionan un cierto
tipo de máquinas; pero con "e" máquinas adicionales Entonces el conjunto R. -A está dado por:
de dicho tipo, puede hacer el trabajo en "b" días (a -
A)0 B) [-2,2) C)(-2;2)
b = 1). Si el rendimiento de las máquinas es el mismo,
entonces el número de días que empleará una máquina
para hacer el trabajo es: D)(-2;1) E)[ -2 , 1)

B) ab2e
E) (a + h)e 6. Sea f(x)=x2 +J,+ I una función definida para
x-

2. Si al número 1573 dado en base n , lo pasamos a la


base (n + 1), entonces la suma de sus cifras en la base
los x que cumplen la siguiente relación: Jx~ ·-1 <./3.
n +1 es: Hallar el intervalo donde varíaf{x)
A) 2n + I B)3 C)2
E) n+-I A)(-2;-I] B) [1 ; 2,25) C) [2 ;5]
D) n + 3

3. Una persona trata de formar un cubo de ladrillos D) [2; 5,25) E) [3; 5,25)
cuyas dimensiones (del ladrillo) son 20 cm, 15 cm y 8
cm. Entonces, el número de ladrillos que necesita para 7. Dos recipientes contienen vino. El primero tiene vinu
formar el cubo más pequeño (de manera que las aristas hasta la mitad y el segundo un tercio de su volumen. Se
de igual longitud sean paralelas) son: completan estos recipientes con agua, vertiéndosc las
A) 129 B) 143 el680 D) 2400 E) 720 mezclas a un tercer recipiente. Sabiendo que la capaci-
dad del segundo recipiente es el triple que el primero,
4. Si la suma de A nuevos soles se divide en dos partes, entonces el % de vino que contiene el tercer recipiente
de tal modo que al ser impuesto una de las partes al a% es:
(1 < a < 10) y la otra al (II + 2)% anual, ambas al mismo A) 37,0 B) 37,5 C) 38,0 D) 3R,S El VI ()
tiempo, producen igual interés. Entonces una de dichas
partes es: 8. La función polinomial P(x) = al" + hX2 - b + a, con
aE Z+ ,y tal que P( 1) < 4, tiene 2 raíces positivas
A (a + 1) Aa Aa iguales, entonces un valor de a - b es:
A) 2(a+2) B) 2(a+2) C) 2((1+ 1)
A)3 B)4 C)5 D)6 El7

A(a+2) Aa 9. De una muestra de números enteros, se tiene quc el


D)-- E) 2(,1--1)
2((1-1) )
mayor de ellos aparece 4 veces, y su frecuencia es 2Z 1
·. :A: ••
UJllU·tMM '¡¡:*~~I,-_U_N_I_2_0_0_1-_II_M_A_:r_E_M_Á_T_IC_A_~I~t;
G~m.;z
del total de números impares. Si el total de impares ex- 13. Una pareja, días antes de celebrar sus Bodas de
cede en 8 unidades del total de pares, entonces el núme- Plata, decide comprar un boleto de lotería de la forma
ro de datos de la muestra es:
al.caa , el cual es múltiplo de los años que llevan casa-
A) 26 B) 20 C) 25 0)18 E) 28 dos. Hallar la suma de cifras del mayor número que
cumpla esta condición.
A) 27 B)30 C) 33 O) 36 E) 39

tiende a:
14. Dada la matriz M=[2coi e sen2e]
A) 00 B) 1/4 C) 3/4 O) 1/2 E) 1 sen2e 2sel1~e

11. Sabiendo que ~ =f= ~ y además: Entonces


A) M
la matriz
B) 2M
M3 es igual a :
C) 3M O) 4M E) 8M

(A+a)(B+b)(D+d)
.= M3 , calcular: D 3fA!j
Vd +d 3f4
Vd2
A)M B)3,/M C)# 1 5. So> A =[: : :1 una matriz, entonces " ma-

triz A4'¡ está representada por:


12. Sean ¡;g: [1 ; 00)-) R. funciones definidas por:

[10 O

Entonces
aproximadamente:
la gráfica de la función composición go f es
A)
989
49 1
49 :l B)[ 4~
1080 49
I

:l
O O

A) y B, y C) [ 4~
1225
I
49 :l D) [ 4~·
1127 49
I

:1
O

C) y
x

D) Y
x
E) [ :,
1274 49
I

:l
.r:~,
,./ ,
16. En un anillo definido por 2 circunferencias
concéntricas C y C' de radios R y r, (R > r) se colocan 6

x 1 x circunferencias de radios R-r_ de manera que cada


2

~_~
__
,c
una de ellas es tangente a las 2 contiguas así como tam-

E) bién a C y C'. Entonces el valor de !l es:

.r:
/,
,¡ :
A)3 B) 5 C)2 0)4
r

E) 5/2

2 x
17. Se tiene dos aleaciones de plata y cobre de distin- 24. El ángulo El, en grados, que satisface la ecua-
ta ley; mezclando pesos iguales de ambas aleaciones se
obtiene una aleación de ley 0,865; y mezclando canti-
ción: 3J2cos(~)+JI+COSEl = -/6 pertenece al in-
dades de ambas aleaciones que tengan el mismo peso
de cobre se obtiene otra de ley 0,880. ¿Cuál es la ley tervalo:
primitiva de cada una de las aleaciones?
A) ElE(1800;2400) B)ElE(1200; 135°)
A) 0,98 : 0,89 B) 0,91 0,82
C) 0,92 0,91 O) 0,98 : 0,82 C) 8E(-3000:3000) O) ElE(900: 120"}
E) 0,93 0,91
E) ElE (240°; 270°)
18. Los valores enteros x e y son los lados de un
25. En la siguiente figura, halle el valor del segmento
2 EF, si DH = OH.
rectángulo. Si se cumple que a x+y< a:1 D

J.....x+ y < II +_1_ para a > O, hallar el rectángulo de


a2 a+1
mayor área. A
A) 2 u2 B) 3 u2 C) 4 u2 O) 5 u2 E) 6 L/2

19. SeaNun número cuadrado perfecto impar. Si


N + 23 es divisor de 136xR,siendo R primo, hallar el
C· .
menor número N que cumple lo anterior.
A)9 B) 25 C) 49 O) 81 E) 121 A) ~ B) JIR C) 4J1O
15
R E) R

20. Sean x, z, N enteros no negativos. La cantidad de 26. Una persona localizada en A observa directamen-
números N tales que 10 < N < 35, que no se pueden te al este y ve un OVNI con un ángulo de elevación de
expresar en la forma N = 5x + 8z es igual a: 45°. En el mismo instante otra persona localizada en B,
a I km directamente al oeste de A ve el mismo OVNI
A) I B)3 C)7 0)5 E) 9
con un ángulo de elevación de 30°. Determine la distan-
cia en km de la persona localizada en B al OVNI.
21. EL valor máximo que toma la función
A) 1,89 B) 2,22 C) 2,73
¡.(
x )=.3sen 2 x+4cos 2 X, XE R , es.. O) 2,91 E) 3,01

A)3 B)4 C)5 0)6 E)7 27. El valor numérico aproximado de:

. J2 (511:) ( Ti)
11:'\
22. Si 0<0.<1", O<~<~ , tano.=~ ,sen~= k E =--¡-tan

A) 1,06 B) 1,56
Ti -sell

C) 2,11 O) 2,19
es:

E) 2:;(,
entonces el valor de a.+2~ es: 28. Sea 0.= ángulo diedro inferior (ABC, BCD)

Entonces, tanci es igual a: B


A)~ B)~
3

23. Al calcular el valor de F=_I_-


senlO° casi 0°
J3
obtenemos:
A) I B)2 C)3 0)5 E) 4 A ·/'-- -Y
::~~~:=:;:W~~::=I
__ U_N_I_2_00_1_-_II_M_A_:r_E_M_A_'T_IC_A_ ••••• ====---~
0)4/2 E)~ O) I E).1L
2rr(II1-1) 1tm
29. Si a+p=90° ,entonces el valor de la longitud del
34. Con centro en los vértices de un cubo de lado a se
segmento Be es:
A)a trazan esferas de radio E... Si la esfera de radio R con-
2
tiene a estas 8 esferas y es tangente a cada una de ellas,
B)Jb2-a2
qb-a entonces el valor de ~ es:
O) a+b

E)Q..
4
A) Jf B) fi-l q.h O) J2-J E) f{
E 3 35. En la figura, el cubo tiene lado 1 y el punto P se
30. Se tiene dos circunferencias de radios R y r, R > r, escoge de manera que el triángulo SPH tenga área mí-
tangentes exteriormente. Si las tangentes comunes for- nima. El valor de esta área mínima es:

man un ángulo de 60°, entonces el valor ~ es:


s A)R
A) 1/4 B)1I2 qll3 0)2/5 E)J/5
31. Una pirámide tiene una base que es un cuadrado J
de lado I y su vértice se encuentra sobre una perpendi-
B) fi.
cular al plano que contiene al cuadrado y pasa por un
vértice del cuadrado. Si la altura de la pirámide es igual D
q J2-1
a J, el valor de su área lateral es igual a: J
O)fi
A)J2 C) J+J2
2
E)1'
O)2fi.-l E) 1+J2 36. Hallar el valor de verdad de los siguientes enun-
ciados:
32. Sea un triángulo equilátero de lado a , donde uno 1) La suma de las longitudes de dos lados opuestos
de sus lados está sobre el ejeXyun vértice se encuentra de un cuadrilátero convexo es menor que la suma
en el origen. Entonces el volumen generado por dicho de las longitudes de sus diagonales.
triángulo al girar alrededor del eje Yes:
I1) Todo cuadrilátero convexo, puede ser inscrito
rr r;:: 3 en una circunferencia (de tal forma que todos
B) -,,3rt
12 sus vértices pertenecen a la circunferencia).

1Il) Dadas dos rectas paralelas LI y Lz distintas, dos

puntos A, S en LI, dos puntos e, D en Lz y un


33. Se tiene dos poleas de igual diámetro, conectadas
por una faja de longitud igual a "m" veces ( me IN) la punto E en el segmento Ae( AotE . eotE) .
longitud de la circunferencia de una de las poleas. Ha-
llar el diámetro de las poleas, si se sabe que la longitud Si IAEI=IABI, y ICEI=ICD¡' entonces el
de la faja que no hace contacto con las poleas es 2 l.
ángulo SED es recto o es llano.

A)~ B)/+2 q_2_1_ A) VVV B) VVF q VFF O) VFV E) FFF


rr(m-I) 1tm 1t(m-J)
37. Dos circunferencias tangentes en el punto A dc 39. Tres puntos A. B Y C forman un triángulo
radios I y 2 respectivamente, son también tangentes a equilátcro. Considerando P un punto interior al /\13C
una recta en los puntos B y e. Hallar el radio de la cir-
tal que las alturas PD (del t!. CPB) , PE (del.'; /\1'8)
cunferencia inscrita en el triángulo ABe.
Y PF (del t!. APC) miden 1, 2 Y 3 respectivamente.

A)l B)2J3 C) 2J3 + J(, Calcular el área del triángulo cquilátcro.


3 3 3
A) 12J3 B) 36J3 e) 27
O) 2J3 +J6 -3/2 E) 2J3 + J6 - 3/2 + I
3 3 0)36 E) 15J3
38. Sea ABCD un cuadrado de lado L sobre los lados 40. Sea ABCD un cuadrado y AEF un triángulo
equilátcro inscrito en ABCD. Hallar el área dcl cuadrado
AB.y ,.ID se construyen triángulos equilárcros:
~ EAD y ~ FAB respectivamente. Calcular el área ABCD, sabiendo que el área dcl triángulo AEF es J3
del triángulo ~ EFA.
A)2 8)2+J3 C)3 O) 3+jj E) 4
2
B) [.2 C) [.2 O) l? E) L
8 (, 4 2

SOLUCIONARIO
MATEMÁTICA 2. Se tiene: 1573- n
Por descomposición polinómica pasamos a base diez:
1. Asumiendo que inicialmente se Ic proporciona "y"
, 1
máquinas, tenemos: 1573,,=11-+511-+711+3

Por divisiones sucesivas pasamos a base (11+ 1)


DÍAS MÁQUINAS
a y 113+5112+711+3
1 1
b y+c n + 11-
X I - 4112+711
Además: (/-f,=1 ... (a) 4/12+411 311+3 11+1
Como a mayor número de máquinas se demora menos 311+3 311+~ "
días, la relación es inversa, entonces: 311+3 - O
• ay = h(.!'+e) - O
Por lo tanto: 15731/ = 1200(1/+1)
y(a -17) = be
La suma de las cifras es: 1 +2+0+0 = 3
De (a) : y v bc ... (1)
'ClaH: B

• x =ay ...(2) 3_ Tenemos ladrillos de las siguientes dimensiones:

De (1): ="hc
Por lo tanto, una máquina empleará abc día,
Clave: D
( 20 CIII
~&cm
15 cm

¡¡
.~=:'-=.::e::::::1__
:=:=:"
Formemos el cubo más pequeño de manera que las aris-
U_N_12_0_0_1-_II_M_A_:T_EM_A_·T_IC_A_ •••• II===:,YD:' ':+~:'~:'~.=~*0
tas de igual longitud sean paralelas: Las partes son: ~ y A(a+2)
.. 2(a+l) 2(a+l)

Clave: C

5. Dado: A={XE R ¡Jx2 -I-Jlx-ll ~ o}


Valores que puede tomar la variable x.

.4--1 Tenemos:

Del gráfico observamos que "I" debe ser el mínimo co-


X2_1~0 =>_xE(--oo;-i]v[l;oo) ... (a)
mún múltiplo de 20 ; 1S Y 8 .
1= m.e.m. (20 cm; 1S cm; 8 cm) • Jx2-1 ~Jlx-ll
= 120 cm
De donde el número de ladrillos "n" se obtiene aSÍ:

Volumen del cubo


n
Volumen de un ladrillo

3
(120cm) (x+2)(x-I)~0 I\x(x-l)~O
(20cm)(15 cm)(S cm)
=> XE (--00;-2] v [1 ;00) ... ( ~)
= 720
Clave: E Intersectando (a) y (P) tenemos:

4. Se tiene un capital de: A nuevos soles. (a)n(p) =A= (--00; -2] v [1: 00)
Se divide en dos partes:
• A-x impuesto al a% anual. finalmente: R -A=(-2:1)
.... • x impuesto al (a + 2)% anual.
Por condición del problema estas partes producen igual Clave: D
interés, es decir:

a%(A-x)=(a+2)%x 6. Hallemos el DOI7I(f) de la relación:

aA
=> \'= -- Jx2-1<J3
. 2(a+ 1)
• x2 - I ~ O => X E (-00 ; -1] v [1 , 00)

Luego: A_X=.4-:(_(OA
. 2 a+l
)J • x2-1<3 => xE(-2;2]

" De donde el DOI1{f) está dado por la intersección de


A(a'f))
los conjuntos encontrados.
= 2(a +:~)
XE(-2;-I] v [1.2)
UN12001-11 MATEMÁTICA

Factorizando:
Además: /(x) = x2 +~+ 1 es una función par
x
b o l-b
(.r(x)=/(-x)), por lo tanto el Rall/(x) lo podemos
x=-1 -1 j-b b-l
encontrar analizando solamente el intervalo [1; 2) . b-j r-e O
Como la función es creciente y positiva.

~ /(x) = .((1)=3 p(x)=(x+l) [x2+(b-I)X+I-h]


nun,
~~
Hafz Por condición ésta debe
f(x)múx =f(2)=5,25 Negativa generar dos ratees
positivas iguales (.1""-u)

f(x) E [3; 5,25)


Clave: E
b = l ~ Genera: XI =-"2 = O
7. Del enunciado:
h=-3 ~Genera: XI =x2 =2
Segundo
Primer Recipiente Finalmente: a-b=I-(-3)=4
Recipiente Clave: B
Agua 4V Agua
V

V V'no 2V Vino 9. Del enunciado:


Números impares x+8
2V 6V
Números pares x
Número total de datos: 2x +8
Frecuencia absoluta del número mayor: 4
5V Agua
Tercer
1-------1 Recipiente Frecuencia relativa del número mayor: 2; I(.r+X)
3V Vino
Por definición de frecuencia relativa:
8V
Por tanto el % de vino que contiene el tercer recipien-
te es: ~ x=9;-21 ( -21 descartado)
. 31' ~x=9
%vil1o=-- "101"0=37,5%
. 3V +51
Luego el número total de datos:
Clave: B
2x+8=2(9)+8=26
8. Sea la función: p(x)=CL\ 1,.>:- -b+a Chn :A

Condicicón: P(I )<4


10. De la serie dada tenemos:
~ P(I)=a+h-h+a <4
S=.!..+.!..+....!..+...!...+
..+-I-.
~ ac Z
3 8 15 24 k2 _1
Por condición: aE Z I ; ~ a= I
I I I I
= -- +-- +--+ ..+-----+
Ix3 2x4 3x5 (k-l}(k+I)"
Obtem "use. p(x)=- ., h+1

"
= Hl-}) + H±- ¡)+ H}-i )+... De donde:

Si: g(J(x)) = y
+t(k~l- k~l)+···
=> . y= Jx2 -x
=i(l+})
_3

Graficando:
Clave: C
y
11. Datos:
g~
• ~=!!..=
a b
D =k
d ------r
=> A = ak; B = bk Y D = dk ... (1 )
1+'" x
-2-
• (A +a)(B+b)(D+d) = M3 Clave: A

De(l): =>a.b.d(k+l)3=M3

=> v;;¡;;j(k+l)=M ...(2)


abcaa=3 y 8
.
Se pide:
Además abcaa debe ser el máximo por condición del
problema, por lo tanto a, b Y e son máximos.

De (1): Si: abcaa=8 => amáx=8


Luego:

8bc88=3
De (2)
e
Clave: A => b+c=3 => blllax =9

12.Datos:' J(x)=x2-lxl; xe[l;oo)


:. abcaa = 89688
=>J(X)=X2-X ;x~1 La suma de sus cifras:

., g(x) =.¡; ; ú 1 8 + 9 +6 + 8 + 8 = 39
Clave: E
La composición de funciones:

(e- J)(x) = g(J(x)) = Jx2 -x 14. Dato del problema:

Domg» J={xEDomJ / J(x)eDomg} M = [2COS 6


2
sen2,6 ]
sen26 2sen-6
~~~=~=:;~;;;I __
~
U_N_I_20_0_1_-I_1 _M_A_JE_M_A_'T_I_C_A_ ~~~=.":'
=~;;-:~~i~ GOllleZ

Elevando al cuadrado:

!J2 _ 2cos 2 8 sen28 ][ 2eos 2 8


I -
[ sen28 2sen28 sen28 o o
n
A = [ n-II

~ 2sel/28 1 1+2+ ...+(n-l) n-I


1
m: :]
=
[ 4cos2

2sen2e
a
4sen"e+sen
o 2
e o
=2M
'-----v----'
4se,,20
-[ ,
1+2+ ,+n
1
n :]
Elevando al cubo: Si n = 49

o
M3= [4coie
2sen2e
2sen2e][2eoie
2
4sen e sen28
seI/2,e]
Zse¡ - 8
A49l~ 1225
1
49 :]
= 2[ 2cos e 2
sen2e ][ 2coi8
2
se1/2?8 ] Clave: C
sel12e 2sen 8 sen28 2scn-e
16. Graficando:
,\,f2

= 2[ 4c()s~e 2SCIl;e]
2sen28 4sen-S

=4 [2COS
sen2e
2
e =: J
2cos-8
~
Al
4M
Clave: D

15. Por dato del problema:

AO[: : :] Del gráfico, triángulo equilátcro OA"!\,

OM=MN
Elevando al cuadrado, al cubo, .. " a la n-ésima
r+R-r =R-r
o o
A2 = 1
[10 0] [ 21
o -
2

1
1
1 m: 1
1 1 1+2 2
1
:] => -=3
R
r
Clave: \
o o o
. A'O[ ; 0][
o - I
1+2
1
2 m: 1
1 1
3
1+2+3
1
3 :]
~dti~~ii~~::::::~~[::~U~N~12~O~01~-!"~M~A~J~E~M!Á~TI~C~A::1=::::::::::':H::MM=~
ZG¡;'m.Z .= ~
17. Tenemos: 18. Por condición:

Aleación de ley L 1 Aleación de ley L2

• L+y<II+_I_
a2 a+l

... (a)

2
Sabemos que: ( a + al2 ) ;:: 2 , \;/a '" O

Si queremos que x e y sean máximos entonces:

Hacemos (a 2 + a12) = 2 mínimo.

En (a) : 2x+2y<12
x+y<6

('
Plata
a
t Cobre
CJ~ ~I/ a [)
"""Pl:-at:-a-'::C-:obre
I
2
3
4
3 --
2
Área
o
S=(1)(4)=4
S=(1)(4)=6
• S=(I)(4)=6u2
u2
u2

" Aleación 11 4 1 • S=(I)(4)=4 u?

f,/ 2a(71 Por lo tanto:

Caso ni W¡+ W2 V
Ley = 0,880 Clave: E

Del caso 1: • L, + Ú]. = O 865 19. Del enunciado: N = k2


2 •
=> N = 1 ; 9 ; 25 ; 49 ; 81 ; 121 ; ...
=> L,+~=1,73 ... (a)
También: (N + 23) es divisor de 136R= 17x8x R
Del caso 11:· a = (1- L,)/tí = (1- l,z)W2 Para N = 1Y R = 3:

/tí L, + W2 ~ = O 880 (1+23)=24 es divisor de 136x3=40R


/tí+U2 ' Para N = 81 Y R = 13:

(81 + 23)= 104 es divisor de 136x 13= 1768

Por lo tanto, el menor número N en las alternativas


L,.~ =0,7462 ... (~) es 81 .
Clave: O
Resolviendo (a) y (~) : 20. Datos: • x, z , N pertenecen a los enteros no
negativos.
L, =0,91
• 10< N < 3 5 => 3 24 valores para N.
~ =0,82
Clave: B • N = 5x+8z
De donde: 10<5x+8z<35 F=_I_- J3
x= O ~ z = 2, 3, 4 ( 3 valores) 23. senl00 cos l O?
x= I ~ z = 1,,2, 3 (3 valores)
x= 2
x= 3
~
~
z = 1, 2, 3
z = O, 1, 2
(3
(3
valores)
valores)
_2
-
(1/se/!2 100 -
J3 /2 )
cos 100
x=4 ~ z=O, I (2 valores) , ("os}OO ,
x=5 ~ z=O, I ( 2 valores) 0
= 2 (COS60 cos 100-sen60° se/! I 0 0
)

x=6 ~ z=O ( I valor) senlO°cos l O?


'-----.r-----'
Existen 17 valores que se pueden expresar de la forma .!.sen20°
N = 5x + 8z, por lo tanto son 7 los números N que no 2
se pueden expresar de esta forma. = 2 cos70°
Clave: C I
2sen20°

=4 Clave: E
21. f(x)=3sen2x+4cos2x ; XER

24. En la ecuación:
= 3(sen2x+cos2 x)+cos2 x
~ 3J2 cos(e / 2) + ~I + cose =-16
1 ~~
negativo posiuvo
=3+ca"z x
~ cos(e/2)<O ; 1+'cose=2cos2(el2)
Si XE R ~ -1 ~cosx ~ I
Luego:
o~coi x~1
3J2 cos( e/2) + J2lco~ e/2)1 = -16
3~3+coix~4
3~f(x)~4
3J2 COSI+J2( -COSI) =-16
:. f(x) máximo es 4.
Clave: B
~ co{i) -f =

.. ~ = 360° k ± 150° ; k E Z
22. • tana=~ /\ O< a <1L
7 2
5 rny/]1 Para: k = O: e E (-300 0
; 300°)
~ a ~a=8° Clave: C
7

I
• senp = r:;:: /\ O <p < 1L
,,10 2

Se pide:

11:
En radianes:
4
Clave: O e
· :A: , "
QJW UN12001-11 MATEMÁTICA 27 '"
Glr.ñez

Del gráfico:
En el triángulo rectángulo AHO
R
sena= "2 =_1_ ... (1)
J5R J5
2
En el triángulo rectángulo AEB (J6 -J2)
Reemplazando: E= J2( J6+J2)_
~+_x_ 4. J6-J2 4
J2 2R2J2 3x
sena = ...(2)
3J2
4J2 R
4
De(l) y (2):
= 1,06
I 3x Clave: A
J5 = 4J2R
28.
::::) x= 4M R
15
Clave: C

26. Del enunciado.

B En el ..,¿j ANM: tan!!:..= J2


2 2
Del gráfico: tan30 = _1_ = 0 xf2
J3 Ikm+x/2
Piden:
::::)x=(J3-I)km
x=2,73km
Clave: C
- {~)
27. Piden: E = J24 tan (51t) - sen (~)12 - 1-( V;J
12

51t 1t 1t .. .
\2+\2 ="2 (Angula s complementarios)
Clave: B
Luego, podemos construir el siguiente triángulo rectán-
gulo:
29. 31. Graficando de acuerdo al enunciado.
M

~I--\
teorema de las tres
perpendiculares

Del gráfico: ¿jDOE = ¿jOMD Piden: Área lateral (SLAT)


~a+x=b
x=b-a
Del gráfico: SLAT =2 (SI +S2)
Clave: C

30.
=2( 1~1+ J~XI)
= 1+Ji.
Clave: E

32. y

I
En el triángulo rectángulo

sen30o=~=l
2r 2
O' QN

... (1)
/
-'-.",/"
.•
En el triángulo rectángulo OMN

sen30o=_R_ ... (2)


R+3r Del gráfico: _~= ~ ... (*)
Igualando ( 1) Y (2):
El volumen generado por el tJ. OMN al girar alrededor
~=_R_ del eje y: (V:r) .
2 R+3r
Por elTeorema de Pappus:

Vr = 2rrx(S,WAiN)

Clave: C
De (*):

rt r: 3
='4v3a

Clave: O
Finalmente:
a a
R ~(I+J3)

=J3-1
Clave: B

Sea la longitud total de la faja: L


• L = m (21tR) (Dato) ...(a) 35. B
Del gráfico:
• L=2/+21tR ... (~)
Igualando (a) y (~) :

m(21tR)=2/+21tR

R= __ I-
1t(m-l)

Luego: Diámetro = 2R

- 2(
-
I
1t(m-l)
)
Aplicando Pitágoras:
Clave: C BH =..[j ... (1)
El área del triángulo será mínima cuando la altura PO
34. Del enunciado:
sea mínima ( hmin ), que sucederá cuando AE y BH se
crucen perpendicularmente
hmin será la proyección perpendicular de PO sobre' la
base del cubo.
1-
hmin ="2AD
_.fi ... (2)
-2
El área mínima del triángulo BPH está dado por:

BHxhmin
SóBPH(min) = 2

Del gráfico: AB es diámetro de la esfera de radio R.


_ ..[j( 1)
DE (1) Y(2):
- 2
~ AB=2R=~+aJ3+~

...(a)
=H
Clave: A
.:.&.: .
UJIIU
G~iiléz

36. Analizando los enunciados: Segundo caso:


I. B
--~~~--~----------------LI

------------~~~~~D~--L2

A Del gráfico:
a='e
MBO: n<O+d} L.BED= 180° (Llano)
1'10CD: m c.c+b Por existenoia de triángulo
:. El enunciado es verdadero (V).
Sumando: n + 11I < (a + d) + (c+ b) Clave: D
AB+CD < BD+AC
37.
:. El enunciado es verdadero (V).

n.

"No todo cuadrilátero convexo, puede ser inscrito en


una circunferencia".
:. El enunciado es falso (F). Por propiedad:

LBAC=90° y BC=2J(2)(I)=2.fi
1lI.
Primer caso: Del gráfico: .L::l BAC -.L::l OHB

h c/2
~ sene= r;:=--?--
2,,2 -

~ h=
12-
2- e

Pitágoras en el .L::l BAC:

Del gráfico (por propiedad):


LBED=a+8
También: 1800=2a+28
~ a+8=90° 4!i 1\
~ c=--3- h= 216
3 ... (*)
L.BED=90° (Recto)
Cálculo del inradio (r) en el triángulo ABC, aplcandoel De la figura: Só ABC = SI +S2 +S3
Teorema de Poncelet:
Lh Lxi Lx2 Lx3
2r+2.fi = b+c -=--+--+--
2 2 2 2
b+c-.fi
=) r= 2

De (*):
_(~)+( if )-3.fi L=4.fi
- 3
Luego:
2../3 +../6 -3.fi
SÓABC=3L
3
Clave: D
= 3(4E)
38. F
= 12.fi'
Clave: A
40.

'-JDL----=-L----'C

Área del triángulo EAF. (SL\)


Dato: SÓAEF =.fi
S = AExFH
'" 2 2
L(L/2)
=) L E =.fi =) L=2
4
=--2-
t: L L.fi
= L2/4 Del gráfico: AC = a ,,2 ="2 + -2-
Clave: D

39.
(1 + E)
a=-¡¡-
Cálculo de la superficie del cuadrado:

S=a2

=ctr
=2+,[3
Clave: B
1----- L----~
~~::::::::::::~ __ ~U_N~I~2~00~2~-_I~M~A~J~E~M_Á_T_IC~A
__ ~~~~~~~~

~~~~~~

MATEMÁTICA

MATEMÁTICA PARTE 1
1. Sean los conjuntos E)(~;-+)
A={x=!.../r,sez, COI/ l<r<3 y 0<.I'<3}
s 5. Si las dos siguientes sumas están expresadas en una
base p.
B = {x e IR / 1 < x < 2} . Calcular A u 13
205" +
A13C" A+B+C=15p
2. Determinar el valor de verdad de las siguientes pro- 403"
posiciones:
1) Si todos los elementos de una fila (o columna) de
Entonces el producto A x B x e expresado en la base
p es igual a:
una matriz cuadrada es cero, entonces, su determi-
nante es cero. A)30 B)34 C) 36 0)42
11) Si dos filas (o columnas) no nulas de una matriz
cuadrada son iguales, entonces su determinante es 6. Con tres números enteros .tI =11 , x2=1I+2,
diferente de cero.
~ = 1I+ 4 se forman las seis posibles fracciones ~
I11)Si en una matriz cuadrada se intercambian dos fi- Xi

las (o columnas), entonces el determinante de la para k #i·


matriz resultante es igual al determinante de la Para que la suma de dichas fracciones sea un número
rnatnz original salvo el signo.
A) VVV B) VFV C) VVF D) FVV E) FFF
entero, será necesario que x, valga:

A) I B)2 C)3 0)4 E)6


3. Sean (tll:tl2:a,) y (/;I;/)2:b,) los tres primeros tér-
7. El número 100! que se obtiene del producto:
minas de una sucesión aritmética y geométrica respec-
IOO!= Ix2x3x ...
x99x I00 termina en 11 ceros. entonces
tivamente, tale, que: tl2 - "2 = I . - ", 1= 0,4 . 11 es igual a:

Si la razón aritmética es 2 y la razón geométrica es 1/2, A) 10 B) 11 C) 12 0)20 El 2.+


entonces el valor de "1 asociado al menor valor posible
8. Se dice que un cuarteto de números primos es "Ie:-
de (/1 es:
gal" si satisface las dos siguientes condiciones.
A)-4,8 B)-8 C)-11,2 D)-14,4 E)-17,6 a. La suma de los cuatro números es Igual a 70.
b. La suma de 3 de ellos es igual al producto de UIlOde
4. El conjunto A ={xe IR/a <-4 y 2-ax>JlI.i+x} los 3 por el otro número primo (no considerado
es igual a: entre los 3).
Entonces, el número de cuartetos "legales" es igual a:
A) (-oo;Oju[_..L: C)R D)3
I1 A)O B) I C)2 E)4
9. Sean los 4 números 15. En la tabla siguiente aparecen varios valores de
dos funciones f y g.
745X
p =2 ; lJ =3ó215
; r = 7:'729 ; t = 1724X(,

x 5 6 7 8
Su escritura en orden creciente es:
f(x) 8 7 6 5
A) p;q;r;t B) p;q;l;r C) p;l;q;r
g(x) 7 8 6 5
D) q;p;r;l E) q;p;t;r

10. Hallar el valor nominal de un pagaré negociado al . [C!,'+f)Of-2]<6)


Determine el valor de: !,'o!,'
2/3% mensual por 3 meses, sabiendo que la diferencia
entre el descuento comercial y el racional es de $2,00. A)O B) I C)2 D)3 E)4
A) $ 5 000
D) $ 5 300
B)$5100
E) $ 5 400
C) $ 5 200

16. Sea la inecuaci6n


a2(x-llas-x
<
(a 2X 1
- r con
aSx a4x+2
11 . Dos recipientes A y B contienen vino. El recipien-
te A contiene la mitad de su volumen y B contiene un O<a<1
tercio de su volumen. Luego, los recipientes se cornple-
Entonces el menor valor entero que satisface la
tan con agua vertiéndose la mezcla en un tercer reci-
inecuación es:
piente. Sabiendo que la capacidad de B es el doble que
A) -2 B) -1 C)O D) I E)2
la de A, entonces el porcentaje de vino que contiene la
tercera mezcla es:
17. Si <1 Y 22 son las raíces cuadradas del número
A) \~% B) 31~% complejo 2'" O, entonces el valor de (21 + 22)" es:
1

E) I:~% A)ZIZ2 B)21Z22 C)O D)I E)Z.1


D) 71~%

12. Cuatro atletas deben recorrer 800 metros planos 18. Del sistema:
en una competencia con relevos cada 200 metros, si las J'+1_2y = 11
velocidades de los primeros relevos fueron 20, 21, 23
y 1
mis ¿qué velocidad debe imprimir el cuarto relevo para J' +2 + =41
igualar el record establecido con un promedio de 20,3 Hallar lo!,' x
mis por equipo? A) 1/2 B) 2/3 C) 3/2 D)2 E)4
A) 17,20 mIs B) 17,39 mis C) 17,55 mis
D) 17,58 mis E) 17,87 mis 19. En un concurso, una dama debe adivinar el precio
de un cierto producto. El animadorle dice: El precio
13. Dada la inecuación ~< x-b· ; 0< b < a tiene tres dígitos enteros y dos decimales, los dignos
x+u x+b enteros pueden ser 1,2,3,7,8 Y los dígitos decimales
Su solución es la unión de dos intervalos, siendo uno de 6, 9 además el precio es mayor que 300. ¿De cuántas
ellos: maneras se puede dar el precio, si se permite la repitición
sólo de los dígitos I y 2?
A) (-00; -b) B)(-b;O)
A) 24 B)48 C)56 D) 84 E)l)2
D){-u;-b) E) (-a; 00)
20. La suma de las cifras del m.c.m. de:
14. Al simplificar
Y 212 -1 es:
9
2 -1
{An[(B-C')UCB-C)r} -{An[B-CC- A»)'" n B'}
A)37 B)36 C)35 D) 34 E) 33
se obtiene:

A)(AnB)' B)AuB C)0 D)B' E)AnB'


1) En todo triángulo acutángulo la altura es menor
que la semi-suma de los otros dos lados que parten
MATEMÁTICA PARTE 2 del mismo vértice.

21. En la siguiente figura: MC CB AB y 11) En todo triángulo, la altura es menor que la medi-
4 8 da de los otros tres lados del triángulo.
MC = MD. Calcular: IlII/ x 3
III)En todo triángulo acutángulo. la suma de las tres
alturas es mayor que la suma de los tres lados del
triángulo.

A)VVV B)VVF C)FFV DlVFV ElVFF

27. En un cuadrilátero ABCD, el punto P divide el


segmento AC en la razón 1/3, (Al' < pc). Si las áreas
de las regiones triangulares ABD y BDC miden 7011/2 y
A) ¡:f B) 22 C)~ D) 24 E) 12
4 7 3 5 9 30m2 respectivamente, entonces el área de la región
triangular PBD mide:
22. El valor de G = etan 24°c 1011 57°-cft¡n24°c/lIIl 33°
es: A) 45",2 B) 4411? C)4211/2 Dl401l? E)3'JII?

A)2 B) .5 C)-2 D)-I El I


28. Decir el valor de verdad (V o F) de las siguientes
afirmaciones:
23. Si R es el rango de la función I y
a. Tres rectas paralelas no coplanares determinan exac-
. SCII7x tamente tres planos (en el espacio)
.f (x) = CIJs6x + cos4x + ros 2x - 2.1'CII x entonces pode-
h. Tres planos perpendiculares dos a dos tienen un solo
mos afirmar: punto común.

A) Re(O:I) B) Re(-I;O) C) Re[O;I/2] c. Si LI Y L2 son dos rectas en el espacio que no son


paralelas ni se interceptan y PI Y P2 planos que lo
D) (-1: l)eR El (O;I/eR
contienen (LI e fj y '-:. e ,~)
entonces 'í n ''-2",0.

24. En la siguiente figura calcular el valor de o ,


A) VVF B) VFV Cl VFF r» VVV El FVV

? 7)
29. Si f ( tan- x + e tan- x = sec x + ese x
.¡ 4'

Hallar f(2) + f(3) .


~ a (3 9
f----- b'---i---2a-l-a·"¡ Al 20 B) 21 C) 22 D)23 El24

B) ~ Cl~ Dl~ E) 5rr


6 4 3 12 30. Se tiene un triángulo acuningu!o AUC en el que xc
trazan las alturas AH y CJ. Se unen H y./ COIl M punto
25.Si .l'ella=2.l"ell~y,·(I.1'~=3c(l.l'a.. Hallarel valor medio de AC; si el menor ángulo que forman 1;"
de "c(Js(a-~)". bisectrices del ¿ABC y del ¿HM./ mide 8 y el .ac. \
mide a, hallar la medida del ¿HA!'
A) -5/7 B) -3/7 C) 3/7 D) 5/7 E) 6/7
A)2a-8 B)38-a
26. Deducir el valor de verdad de las siguientes afir-
maciones: D)9+2a. E)28+rx
UN12002-1 MATEMÁTICA 11=======~0
31. En la figura O es el centro de la circunferencia, E)3.fi.
D) .fi.a =r»
AB es diámetro, 11I íjjj = 30", 11IM = 120", si CD= 211I
y EC= 10m, entonces AC es igual a:
36. Hallar el volumen de una esfera inscrita en un oc-
tavo de esfera cuyo volum~n es 2~-tt 11'.

A) 21~11:(.13 _1)11' B) 24011:(.13- 2)11'


A¡..:::=-----,,-,I----lB
C) 51;11: (3.13- 5)u' D) 21611:(3.13-1),,.'

E) 5 1 211:(.fi.- 1)u'
37; Sobre las rectas x+y-4=0 y x-y=O se en-
A) 2J37 +6.13 B)2l:,7+4.13 C)2h7+2.13 cuentran las diagonales de un rombo. Si uno de sus vér-
tices es el origen de coordenadas y la medida de una de
D)2J37-6.13 E)2J37-4.13
sus diagonales es igual a la medida del lado del rombo,
32. En un triángulo rectángulo se inscribe una circun- entonces el área del rombo es:
ferencia cuyo radio r es 1/6 de la longitud de la
hipotenusa. Luego, la longitud del segmento que une el
D)-29.13 E)!!Jl
3 .
incentro con el baricentro del triángulo dado es:

2
A) -r
3
B) 4-r 3
C)-r
5
D)f#-r
38. Dos caras de un triedro miden 45° cada uno y for-
man un diedro de 90°. Hallar el valor de la tercera cara.
A) 60° B)45° C)135° 0)120° E) 30"
33. Se tiene el tetraedro V-ABe triángulo rectángulo
en V tal que VA = VB = ve. Calcular el coseno del án- 39. Dos autos parten simultáneamente desde un punto
gulo diedro que forman las caras ABe y ABV. "P" en direcciones que forman un ángulo "8" uno a
5km / /¡ Y el otro a 12 km/h. Calcular el "ClI.I" 8" sabien-
A) fI
V"3
B) .13
3
E) J2
T
do que al cabo de I hora la distancia desde el punto "P"
al punto medio del segmento que separa ambos autos es
34. Se tiene un paralelepípedo rectangular recto de de 7 km.
lados 6a y 8 a y de altura 12a (a> O). Se hace un agu- A)5/8 B)7/16 C)3/80 0)9/40 E) 13/25
jero que tiene la forma de un prisma hexagonal recto
regular de lado 2a que va desde la base superior hasta
40. Para que valores de x E (O; 11:)se cumple:
la base inferior, entonces el área total de la superficie
del sólido que queda es:

A) 12(36+/3}? D) 12(48 - ./3)a2


B) 12(40 - .I3)a2 E) 12(48 + .I3)a2
A) (0;11:) D) (o: ~;)
C) 12(40+.I3)i
B) (O;~) E) (2311:;11:)

35. En el triángulo
(IIILB
rectángulo isósceles
= 90°) , los catetos miden a cm. Del vértice B le-
ABe
C) (O;T)
vantamos una perpendicular BD al plano del triángulo,
con BD = 2a cm . Determine la distancia del. punto D a
la hipotenusa AC.
SOLUCIONARIO
ten dos filas (o columnas) iguales entonces restan-
MATEMÁTICA PARTE 1 do una de la otra se pueden obtener una fila (o co-
1. Datos: lumna) de ceros donde estaríamos en el caso 1.

111) VERDADERO.- Se verifica mediante aplicación


A={x=~/r,SEZ, con l<r<3/\ 0<s<3} ... (1)
de propiedades.
B = {x E IR / l < x < 2} ... (2) Clave: B
De (1): Si re Z /\ 1< r < 3 ~ r=2 ... (3)
3. Datos: (1)
Si seZ /\ 0<s<3~ s=I;2 ... (4)

IlI3 -lJ:11= 0,4 ~ ll:1-lJ:1 = 0,4 ... (Ila)


Reemplazando (3) Y (4) en (1):

A = {1;2} I •
a, -I~ = -0.4 ... (1Ih)
2 Sucesión aritmética (Dato: r = 2):
De (2):

~ AuB=[1;2]
B =(1;2) ---L==L-

1
1

2
2
SA

~
=lll+r(l/)

III =lIl
a2 =al

a, =al
+2

+4
l
1/=0;1:2:3;.

... (a)

Clave: A Sucesión Geométrica (Dato: r = 1/2 ):

2. Analizando las proposiciones SG=bxr" ; 11=0;1;2;3;...

I) VERDADERO.-

diagonal. Así:
Para obtener la determinante de
una matriz se multiplican los elementos en forma

t~~""'.",aI2 .-til')"
~ ::~)
b
-

.
1 4
= '!l
2 ... (~)

f2t~ Q22 ...~23


De (a) y (~) en (1) tenemos:
a11~\ Q1t aJi b
----.>::;,·-·----:.~·0.:. al +2- 2l =0,4 ... (1)
al] -:, -,a12S\>~13,
.,., =M
-, ":-.,,-' De (a) y (~) en (lIa) tenemos:
a21 Q22 ....-, -, ai,]',
1'1=0,4
4,
III +4- .. (2)

M =(all X{/n xa" +(/21 xan Xli" +lI,1 xlII2 XlI23)- Resolviendo (1) Y (2): al = -5,6 1'1=-8
De «('1\' ~) en (I1b) tenemos:
(lI,1 xa22 X{/I, +llll xlIn XlI2, +lI21 Xlll2 Xli,,)

Observe quc si una lila (o columna) fueran ceros en- b


III +2--1- = -0,4 .. (3)
tonces todos los sumandos se convierten en ceros.
Para una matriz en general se puede verificar por bl
al +4- =- 0,4 .. (4)
el desarrollo de Laplace. 4
11) FALSO.- Por propiedad de matrices, las filas (o Resolviendo (3) Y (4): al=~7, ; b¡=-1I,2
columnas) pueden variar sus valores restando y su- Por lo tanto:
mando ,IS respectj, lores entre l s filas (o a",,,, = -7.2 Y hl=-11.2
colurnn l. Cons '1 ' 1 c-ta propiedad, si exis- Clave: C
(1)+(1'-1)+(1'-2) = 1'+5
4. De: A = {X E IR / a < -4 Y 2 -l/X> JlI.i + X}
~ 1'=7
Reemplazando el valor de P en ( I) Y (2) :
Se tiene:
B =(7)-1 =6
J(/)? + X 2: O /\ 2- 'IX >O /\ (2 _l/x)2 > l/} +x C=(7)-2=5
Se nos pide:
x(Xl/+I)2:0 /\ X>¿ /\ x2('?-l/)-(411+l)x+4>0
ABC = l x 6 x 5 = 30
~ '--y--' y

I 11 111 Pasando a base l' = 7 :


~ 30= 427
2 2
De (111): (a _l/)x -(4l/ + l)x+4 >O
Clave: D
2
Tenemos: II -u /\ 1\ = 24l/ +I 6. Dado: xl=a ; x2=a+2 ; x,=lI+4
Se forman y suman las 6 posibles fracciones de la 1'01'-
Por dato: a < - 4 ~,?-a>O /\1\<0
ma: xk; k,<¡
:.(a2-a)x2-(411+I)X+4>0 V XER xi

~ XE R. a a
S =--+--+--+--+--+--
0+2 0+2 a+4 lI+4
a+2 0+4 a a+4 l/ l/+2
Luego, de (1), (11) Y (111) tenemos:
S=6+~
1I(1I+4)
XE[O;-.!.]({ /\ XE[2;=)
a
/\ XE U?
Entonces para que S sea entero at« +4) es submúltiplo
de 24 donde a E Z .
~. XE[O;-±] Si: l/= I ~ S¡; Z

Finalmente: A ~[O; -±] Si: a=2'~ S=8EZ


~ xI =a=2
Clave: B
Clave: B
5. Dado:
7. 100! = I x 2 x 3 x 4 ... x 99 x 100. Los "ceros"
205,> ) aparecen cuando se multiplican los rruiltiplos de 5. con
ABC,>
... (1) uno de los números pares inferior inmediato.
403"
Ix2x3x4x ... xI0}
... (11) 4 x 5 x 10 200: 2 ceros.
Operando en (1) IIxl2xl3x 14x ... x20
Ira. columna: 5+C= 13,> 14 x 15 x 20 4 200: 2 ceros.
= 1'+3 ~ C= 1'-2 ... (1) 21 x 22 x 23 x 24 x ... x 30
Zra. columna: 24 x 25 x 30 18 000 : 3 ceros.
1+(0+ B) = lO"
31 x 32 x 33 x 34 x ... x 40
I+B=I' ~ B=I'-I ... (2)
34 x 35 x 40 47 600 : 2 ceros.
3ra. columna: 1+ (2 + A) = 4 41 x 42 x 43 x 44 x ... x 50 }
~ A=I ... (3) 44 x 45 x 50. = 99000: 3 ceros .
De (1), (2) Y (3) en (11): 51 x 52 x 53 x 54 x ... x 60
A+B+C=I'+S 54 x 55 x 60 In 200 : 2 ceros.
~=::4$#&¡::;:=,=j
~
UN12002-1 MATEMÁTICA ~~4JT~~
GomeZ

61 x 62 x 63 x 64 x ... x 70 10. Datos: De - DR =2 (dólares)


64 x 65 x 70 = 291200 2 ceros.
_ 2
71 x 72 x 73 x 74 x ... x 80 } r - 300 (mensual)
74 x 75 x 80 = 444 000 3 ceros.
1 = 3 meses
81 x 82 x 83 x 84 x ... x 90 }
84 x 85 x 90 = 642 600 : 2 ceros. Por teoría: De-D = VNx(rl)2
R 1 +rl
91 x 92 x 93 x 94 x ... xl 00 }
Reemplazando valores:
94 x 95 x 100 = 893 000 3 ceros.
TOTAL: 24 ceros VN(3~OX3r

8. Condiciones del problema:


Clave: E

~
$ 2

VN =
(1+
$ 5100
;~n
= ----,,-.0.-_---:-'_

a.b.c.d son primos ... (1)


... (2)
Clave: B
a +b+c+d =70
a+!J+c=axd ... (3)
11 . Dos recipientes:
De (2) en (3): (70-d)=l/xd
~ 70=d(l/+
'--y---'
1)
V' f=l2V
2 a = 34 (NO)
5 a = 13 (SI)
7 (/ = 9 (NO)
VI:~

f--
~I .-13 V
~VINO

Reemplazando estos datos en (2):


VINO

13+" + e+5
b + e
= 70
= 52
V/2+2V/J-7V/6
- -
'--r-'
5 + 47 (NO) ~ % Vino del tercerrecipiente
7%
= --.ll. x IO()%
]V
11+41 (SI)
70001
23 + 29 (SI) =11f /O

Luego, existen dos cuartetos legales: Clave: D


(13; I 1;41;5) Y (13; 23; 29; 5)
12. Graficando:
Clave: C
12
9. Dado los números:
ti

20 mis 21 mis
--------
23
1)

11//.1'
14
----------
v
l' = 2745X= (64)1243
I -
200 m
-
200 m ~
-
200 m
-1
200 m [
'1 = 3(,215= (243)1243

r = 73m = (343)1243 Dato: VI' = 20,3 11/ Is

1 = 1724X1=, (289)1243 Por cinemática:

Ordenando de mayor a menor tenemos: 1 = distancia


velocidad
(64)1243 < (243)1143 < (289)1243 < (343)1243 ti
vl'=---'-'----
1'<'1<I<r 'I +12 +1, +14

Clave: B
~=:::::::::::::::l:JU~N~IJ2~0202~-II~M~A~~!E~M~Á~T~IC~A~:t:::::::::::::::=~39
=
Glr.ñez

11 = {A -[ B-(C - A)'] n B'}

Reemplazando datos:
= {An[Br}
800m
203m/.\' Simplificando x:
200/11 200m 200m 200m
---+---+---+-- x=1 + 11
20m/.\' 2Im/.\' 23m/.\' v

=> v = 17,874 mrs ={AnBe}-{AnBe}


Clave: E = 0
Clave: C
13. Dado: x-a x=b 15. Datos:
x+a < x+b con O<b<a
x 5 6 7 8
x-a x=b
----- <O f(x) 8 7 6 5
x+a x+b
g(x) 7 8 6 5
2x(b-a) <o
(x+l/) (x+b)
Determinemos: M = [(~ +1')
¡.:(}~
"r - 2J (6)
Por la condición: h- a < O

=> x >0
= [K + f] (f (6» - 2
(x +l/) (x +b) ~(~(6»

(-) (+) 1 (-) (+)


De la tabla: f(6) = 7 g(6) = 8
1\

-Q -b O M = [~ + f] (7) - 2
~(8)
=> xE{-a;-h)v(O;~)
Se pide un intervalo: x E (-a ;-h) _ ~(7)+f(7)-2
- ~(8)
Clave: D
6+6-2
14. Propiedades: (B-é)=(BnC) -5-
=2
Si Bec[B-(C-A)]' => [B-(C-A»)"nB'=B' Clave: C
Simplificando el ejercicio:
16. Tenemos:

x = {A n[(B-é)veB- ClJ}-{A n[B-(C- A»)' n Be} (l2(X-I) X aS-x (a2X-1f


< con O< a < I
v v aSx (l4x+2
I Il
Simplificando:
1= {A n[(Bn C)v(Bn C)n
= {An[[BV(Bn C)]n[Cn(Bn Cm}
= {An[(Bv B)n(Bv C)]n[(Cv B)n(cvc)]f} Dado que: 0<11 < I

=> 3-4x > 2f -5x-2


={A n[Bn(BvC)]n[(CvB)n(U)Il'}
=> 2x2 - x - 5 <O
={A n[Bn[CvBIl'}

={A nB'}
19. CASO 1. Cuando la cifra de la parte entera sea
1-./41 1+./41
Puntos críticos: x=--4- 1\ x=--4- diferente:
300 < (/ be. 11/ 1/
.j, .j, .j, .j, .j,
(-) 1 (+)
3 1 1 6 6
1-.J4T -1 O 1+ .J4T
-4- -~- 7 .2 2 9
8 3 3
7
I-J4I.I+J4I) 3x4x3x2xl
x E ( --4-' --4-
Los números enteros comprendidos en éste intervalo se- Número de maneras: NI = 3 x 4 x 3 x 2 x 1= 72
rán: -1 yO. CASO 11. Cuando se repite la cifra de la parte entera:
El menor entero: x =-1 300<0 he. 111 1/
Clave: B .j, .j, .j, .j, .j,

3 1 1 6 6
17. Tenemos: <= II +bi 7 2 9
8
<1= J; ; <2 = -J; 3x2xlx2xl

si: J; =a+ J¡3 ~ <1=a+ JI3 ; <2 =-a- J¡3 Número de maneras: N2 = 3 x 2 x 1x 2 x l = 12
Finalmente el número total de maneras será:
Reemplazando en : «1 + Z2/ = 1
NT=NI+Nz=72+12=84
(ZI+<2)' =(a+ J¡3-a- J¡3)' Clave: 1)

=0
Clave: C 20. Si:
~ A=511 1\ 8=4095
18. Dado: ... (1)
Por propiedad:
}' +21"+1 =41 ... (2)
m.e.m (A;8)x M.C.D(A;8)= Ax B ... (a)
De (1): 2.1"= 3-'+ 1 _ 11
También:
Reemplazando en (2): }' +2(3-'+1-11)=41
M.C.D (29 _1;212 _1)=2M.C.D(9:12)_1
3' + 2 x (3)}' - 2 x l I = 41 . (13)

}' (1 + 2 x 3) = 41 + 22 =21_1=7

~ }' =3 2 En (a):

~ .\"=2
m.c.m(A;B)= Ax8
Reemplazando en (1): 2" = 3(2)+1_11 = 16 M.C.D (A:/J)

~ y=4 Sil x c.1095


7
1
Finalmente: [{}!{y x = [(}!{42 ="2 = 298935

Clave: A
:. ¿cifras (m.c.Ill.)=2+9+R. 9+3+)=36

Clave: B
l~~~~5~~~~~j
GlI'~z·
UN12002-1 MATEMÁTICA I-~~~~~~~'
:=) 2eos Gx sen x = sen'l x =sen S.x
MATEMÁTICA PARTE 2
:=) 2 cos 4 x sen x = sen 5 x - sen 3 x
21 . Datos: AM = 5k ; eB = 4k ; AB = 8k , :=) 2 cos 2 x sen x = seu 3 x - seu x
Reemplazando:

,( (sen7x-sen5x )+(sen5x-sel13x )+( sensx=senx )-,\'l'I17x

JEIJ~.
j x) 2\'(-'lIX

-senx
Zsenx

A 2k 3k 3k B

Del gráfico: x=a+p Luego: R1 ={-'H:=) R1 c(-I;O)


ralla+tallp Clave: B
:=) tall(x) = tall(a+p) = 1
=tana x tan
13 .. , (*)

4k 4
Del gráfico: talla=5[=S
Q. ,,-&
24~
4k 2 b
tan 13=67(= 3 20

a 2a 4a
Remplazando en (*): tan x I b I 2a-l-a-1

22 Se tiene: eos4a = ~ =J...


7 2a 2
Clave: C ~4ex=60°
rr
22. De: G = ctg24° ctg57° - ctg24° ctg33° Ct=ISOO
rr
= ctg24° (ctg57° - ctg33°) Clave: A
= ctg 24° (tg 33° - ctg 33°)
25. Datos: sen a.=Tsen 13 ..' (1)
= ctg24° (-2ctg 66°)
cos 13=3 cos ex .., (2)
= tg 66° (-2ctg 66°)
Se pide: cos(a- (3) = cosa eosp+sell a sen 13
= -2 ,
De (1) Y (2): = co,~-13 + 2se,,2 13
Clave: C j

23. En la función: 1 +5.1'(',,2 P


= 3 ... (3)
'( ) 6 4 2 senI x
.f x = eos x + eos x + eos x - 2sell x
También:
Donde: Zsen x *O :=) x * /l1t; ti EZ -.
De (1):
{(x) 2cos6xsellx+ 2eos 4xsellx+ 2eos2xsellx-sell7x
2setlx 2
De (2): eoia = eos 13
9
Transformando un producto a una diferencia,
2
2sellase/l13 = eos(a+p) - sell(a-p) Sumando: 2 cos p
=4se/l 13+- -
9
UN12002-1 MATEMÁTICA ~=.~::;:=~::::~.:-~
~=~
27. Graficando de acuerdo al enunciado: .
9=36.w? ~+I-.I'e'/ ~
? 8
~ .I'ell-~= 35
Reemplazando en (3):

('(Is(a-~)=
1+)
3
U~) 5
'7
Clave: D

26. AL-----------------~~D

1) VERDADERO: Trián- e Datos: S ARO = 70",2 (1)


gulo acutángulo. es
aquel que tiene sus án- (2)
gulos internos menores
que 90°. Del gráfico:
B'L---H.L..I..-----=>.A
Del gráfico: S ABO = SI + S2 + S3 + S4 }+ ... (3)

S BCO = 3S1 + 3S2 - S3 - S4 ... (4)


h < {/ (por ser cateto)
h <b (por el' cateto) S ABO + S BCO = 4S1 + 4S2
=>h«lI;b) 701// + 30m
2
= 4(SI + S2)

11) FALSO:La altura 110 de-


... (5)
pende de la longitud de
los lados.
Reemplazando (1) Y (5) en (3):

70m2 = 25m2 + S3 + S4
2
~ S3 + S4 = 45 11I

Finalmente: SpRI> = \', + S4 = 4)1II~


En el gráfico: h > ti, > III Clave: A
I1I)FALSO 28.
B
a) VERDADERO

A
b
e
Del gráfico: h¡,<a (por ser cateto)

h,.<h (por ser cateto)

h(/ <e (por ser cateto)

--:-)
h,¡+'JI +/¡¡ <ti te
Clave: E ~,L¿, L" determinan P." R¡, P1'


~~··~:::::'¡¡¡::,:i~'~:;;Z;~"
Gomr~ "i' ::!..::
~:':d=Z&:::~=('-~~~II[JU~N~I~2~O~O~2~-I:M~A~:r~E~M~Á~T~IC~A=I:;:
30. Graficando de acuerdo al enunciado:
b) VERDADERO
B

e) VERDADERO
"' __ 180"-{(P+x)+(p+a)] r a
'!' 2 - =90"-P-Í-T

S= 180"-{2x+2a] 90"-x-a
2 .
En el cuadrilatero ABNM, la suma de ángulos internos
igual a 360°:
(P + x) + el> + (180"+8) + (S + 2a) = ~6()"

$+ x)+(9<J'-P-~-T )+(1 8<J'+6)+ (9<J'+x-a+ 2a) = 36<J'


Clave: D ~x=26+a
Clave: E
2
29. Propiedades: .vec x = 1+ tw? x ... (1)
31.
c.fec2 x = 1 + ctan2 x ...(2)
'.D
Reemplazando (1) Y(2) en la función:

f(tan2 x+ctan2 x) = sec4 x+csec4 x

=(I+tan2 xf +(I+ctw? xt

4
= 2+tan x+~tan2 x+ctan' x) + ctan4 x
2
= (tan2 x+ctan2 x) +2(tan2 x+ctan2 x)

2 2
Si hacemos: a = tan x + ctan x

Tenemos: fea) = a2 + 2a
Calculando la expresión: Aplicando ley de cosenos en el Ó ADC :

f(2)+ f(3) = [(2l + 2 (2)J+ [(3l + 2 (3)J x2 = (12)2 + (2)2_2(1~)(2) cos3<J'

=23 = 148 - 24.13


Clave: A ~ x=2 J 37 - 6.13
Clave: D
32. Graficando según el enunciado. 33. Graficando: C
A

Propiedad:

BO=2r ; OM =r

==} BM =3r

Del gráfico: A

B~~~D~~--~----------~~C
9 (VMf =i/ - (a~J ==} VM = "~
r: inradio (radio de la circunferencia inscrita)
I : lncentro (intersección de bisectrices).
O : Bericentro (intersección de medianas).
(CM)2 = (llJ2)2 _(a~J ==} CM = "~

Del triángulo rectángulo CVB :


Por definición: Semiperimetro p = AC + r ... (1)
Área del triángulo rectángulo ABC, de dos formas:
COsa = VM = (1I~) =./3
S"'1'8C=pxr=
ACxBH
2 CM ("¿;) 3

6rx BH
==} (6r+r)r=--2-- Clave: B

==}BH = 7r ... (2)


3 34. Graficando:
En el triángulo rectángulo BID: a>O
9+a=45° ==} 9=4SO-a ... (3)
En el triángulo rectángulo BHC:
45°+ <1>
+ a = 90" ==} <1>
= 45°-a ...
(4) 12a
De (3) y (4) se concluye que:
9=<1> ... (S)
Luego en el triángulo rectángulo BHM: Sa
7r
BH "'1 7 Área del exágono regular =6 .Jia2
('os(29) = BM = 3r = '9 ... (6)
El área total pedida: Ar = Al + A2 ... (1)

Por trigonometría: (
'(1,.2
,9
1+ CO.l'2
2
e (De S) Al = Área total del paralelepípedo - Área de las bases del prisma
7
1+- 8 = [2(12a x8a)+ 2(6(/ x 12a) + 2(6(/ x 8l1)]- 2(6./3l12)
De (6): = __ 9 =_
2 9
=4~2(/-12J3a2 ,.. (2)
2.12
==} cos e = -:r- A2 = Área lateral del prisma,
Aplicando ley de cosenos en el Ó. BIO
= 6(2a x 12a) = l44i? .,. (3)

d = J(Bí)2 + (BG)1 -2(Bí)(BG)co.l'e Finalmente en (1):

Ar=(432a2-12./3" )+144,,2
= J(r.l2)2 +(2d -2(r.l2)(2r) cose
2
r./6 =12(48-J3)u
T Clave: D Clave: D
a===:',i"':"":':"-:':" ·:,:.~:·
...=[Ju~N~IJ2~o~o~2-II
JM~A~:r~E~M~A~·T~I~C~A~,J=====:
....:.=:::.~...~45""
35. D
~R=8Ll ... (2)
Desde la vista "1" se observa:

A B

o
Desde la vista "11" se observa:

a.fi R=r+r.fi
Del gráfico: = -2-
~r=T(/3-I)
En el triángulo rectángulo BDH:
= (K;,) (/3 _1)
x = (2a)2 + (a~r =4(/3-1)/1 ... (2)
3J'T"
2
Clave: E Volumen de la esfera de radio r :
V = 4n:,.'
36. Graficando según el enunciado: , 3
VISTA 1 De (2): = 4; [4(/3 -I)u]'
..••••••VISTAIl =512n:(~~ __ ) ,
3 .w.~ ) 11
Clave: e
37.
y-x=o } ~ P=(.r;.l') =(2;2)
y+x-4=ü

Dato: V=256n:,,' ... (*)


3
Por definición, volumen de la esfera de radio R:

V - 4 7r R'
totul -T
x
Por condición: V = 8'1 Vtotnl = '81 (4'3 n: R') ... (1)

Igualando (*) y (1):


La condición del problema:
..!.(~
8 3
n R') = 256n:,/
3 OA = L CD=l
Distancias que recorren en 1 hora:
Del gráfico: L= 2./2 csc3(f = 4./2
d5=VXI=_'Tx"kll/ 111= 5 km
Del gráfico: / = 212 c:sc6(f = 4 J6
3
Área del rombo de lado L: 1 = v x 1= 12 km xl/¡
{'2 T = 12 km

S L = 2( OA ~ EP) = OA x EP Aplicando Pitágoras en 105 triángulos rectángulos:

Triángulo PBH: (7 kll/)2 = ,,2 + (12kll/- x)2 ..,(1)


= [2(2.12)] x [Lco,dOO]
TriánguloPAD: (5klll)2=(2/l)2+(12kll/-2x)2 .. ,(2)
Reemplazando L : = [4J2]x [(4h)( 1)] Luego de (1) Y (2) : x = 87 km
16
= 16J3 (no existe clave)
En el triángulo rectángulo PAD :
Área del rombo de lado l :
12 klll - 2 (~7
¡¡;- klll )
'SI =2( CD~OP)=CDXOP cose= PA
PD
= 12klll-2x
5 k 11I 5klll

= [2/ (0,160°] x (2J2)

Clave: U
Reemplazando / :

16J3
-3- (aceptado)
40. Graficando para x E (O; 11:)

Clave: C , 2x
.1 (x), = (0,1' 3
38. Triedro A-BFC """ ""G

, 2.1'
.f(x)¡ =l'II,I' 3

Del gráfico: ó /\¡'-C (eljllilríl('ro)


D
;
/1':, 1
I
-,
: \
v-,
".
t'()
X 1. = ("(),\"?
2 x

-
--~~+--_nr--~o~~~n~~~~---~
'2 '2 2 X
Clave: A
Del gráfir» se observa que:

ClJI,2 ~ < ('(!I' 2x


, 2 " :1

2 x 2x
CIJ,I' "2- cos 3 <O
a

Clave: e
d'2 --------,
~~[J~~O~
MATEMÁTICA

MA'l'EMÁ'l'IOA PAB'l'E I 5. El mínimo entero m tal que: (x)' - 7 x + 9)' - 63)'" ten-
1. Una ecuación dá como solución una fracción ordinaria ga al menos 1998 términos es:
irreductible, de manera que el término del denominador
A)40 B)41 C)42 D) 43 E) 44
excede al numerador en 10 878. Halle la suma de Jos tér-
minos de la fracción sabiendo que reducida a decimal dá
una periódica mixta que tiene 3 cifras en la parte no perió- 6. Hallar un número de 4 cifras ahcd que sea divisible
dica y seis en la periódica.
A)18872 B)22872 C)23872
por 13 y tal que ea = 3(ac+2). Dar como respuesta

D)24872 E)25782 a+b+c+d


A)15 B)16 C)17 D) 18 E) 19
2. Dos recipientes A y B contienen vino. El recipiente A
está lleno hasta su mitad, el B en un tercio de su volumen.
Se completan las capacidades de A y B con agua, 7. En un partido de fútbol entre los equipos M y W, la
vertiéndose las mezclas a un tercer recipiente C. Sabiendo relación de hinchas al iniciar el encuentro, es como A es a
que la capacidad de B es el doble de A. Entonces el porcen- B (A> B) a favor del equipo W Sin embargo, luego de un
taje de vino que contiene la mezcla e es aproximadamen- gol del equipo M la relación inicial se invierte. Sabiendo
te: que el encuentro se inició con h espectadores, resulta que
el número de espectadores que se cambiaron al equipo M
A)36 % B)37 % C)38 % D)39 % E)40 % es:

3. Si la diferencia entre el descuento comercial y el des- Ah


cuento racional de un pagaré de $ 900 descontado 60 días A) A+B
antes de su vencimiento es de $ 0,09. Entonces el valor
aproximado de la tasa de descuento es: Bh ABh
D) A+B E) A2+B2
A)4% B)5 % C)6% D)7% E)8%

8. El siguiente producto está expresado en una cierta base


4. Dada la expresión: ~a2 + Jb = 1I +b b: (5) x (123456) = 606Y58 donde Yes un dígito, enton-
Determinar el valor de verdad de las afirmaciones: ces para el menor valor de b, la suma b + Y es:
l. No existen números enteros que satisfagan la expre- A)9 B)IO C)II D) 12 E) 13
sión.

n. Si b e {O;I}, entonces a < O 9. Sea la función f(x) = 4+3 / (4-J"'"X), definida en

el intervalo (260°; 360°]. Entonces los valores mínimo y


m. si»
I 7: O • entonces lb
a = 2h - "2b máximo de la función son, respectivamente:

A)FVV B) FFV C) FFF D) VVV E) VFV 37


A.)-I Y 5 B)-I YO C) 11 y 5
___ U_N_I_2_00_2_-I_I_M_A_J_E_M_Á_T_IC_A __ ••• I::~~~~~~~~ :.10:
~1l!Q
G~iñez

53 -b
11 y
D) 5 Y 7

..1.0- Dadas las siguientes


E)

inecuaciones:
5
A .[;
-n si ~Jel(A+-') = 12

(/ matriz identidad)
r - y < O x + 4 < 3y Y < x + 2, entonces

[",,~
los pares (x ; y) que satisfacen estas inecuaciones están
2,,]
representados por la región sombreada: Hallar el determinante de la matriz
b
A)-12 B) -10 C)IO 0)12 E) 16
A) B)
13. Sean las matrices

u = [~ ~
1 2
~l ~il
I
V =[ ~ ~
-1 O I

C) Q = aU + f3 V donde a, f3 E R

Los valores de a, f3 para los cuales existen los números


p, q tales que, simultáneamente se cumple,

E) son:
A) Solamente a f3 =O
B) Solamente a O' f3 arbitrario

C) Solamente f3 = O ; a arbitrario

11. Al resolver, en el conjunto de los números cornple- D) No existen tales números

Ü+i)Z-IV = -1-; E) a y f3 son arbitrarios


jos, el sistema: { 2iz + (1-;) IV = i
14.Si 2+ 14+26+:1~+ ... +x RI6

El valor de es: Entonces el valor de x es:


IV
A)110 B)122 C)134 D)146 E)I:iX
I i I ;
B) --- C) "2+6"
. 2 6 15. ¿Cuántos ancestros tenía usted hace 10 generucio-
nes?
I i
E) 6"-2 A)2046 B)2022 C) 1024 D) I022 E)I020

12. Sean u y b números enteros positivos pares: con es- 16. ¿De cuántas maneras :1argentinos. 4 peruanos, 4 chi-
tos números se forma la matriz lenos y 2 bolivianos pueden sentarse, ordenadamente, en
una mesa redonda de modo que los de la misma nacionali-
dad se sienten juntos?
Al 3456 B)6912 el 20736
D) 41472 E) 165888 MATEMÁTICA PARTE 2
21. Sea AI1CD un cuadrado de lado L; M es el punto
17. El rango de F(x) = 1;1[(X-1)2+2Ixl] es: medio del lado AD, E es un punto en el lado AI1. P es la
intersección de Mil con EC y F es tal que DF contiene a P.

Al R-[-I; 1] B) R-(-l;l) C) (O;oo) Sabiendo que IAEI = IEFI, calcular el valor de ¡FBI
Dl (-00;0) El (-1;00)
A) J2 L L ,n
e)- E)-L
18. Un avión realiza una maniobra a velocidad supersó- 2 3 3

nica. según la ~rayectoria: 2y2 - x2 = 48


22. En la siguiente figura:
Hallar la menor distancia de la trayectoria al punto (6 ; O)
A)9 B)8 e)7 D)6 E)5

19. Determinar el valor de verdad de las afirmaciones:

1 '
l. Si XE (-1;5) => -2 . 'i E (O;J)
x+.

11. Si x E [0;4) => ffifx+


6-X r:
-.-2--"x+I>O Si r 1u , R=3u
A
. x-I OBF = 60°. Entonces la medida del ángulo BDE es:
lll. SI x + 1 > x => x < - 3

A)7,5° B)IO° C)15° D)22,5" El30"


A)FVV B)FVF C)FFV D)FFF E)VVV

20. En una tabla de distribución de frecuencias con 6 23.En un trapezoide ABCD, AB=IJC. 11I8=900.
intervalos de igual amplitud, el valor míni mo es 500 y el
valor máximo es de 1700. Si la característica medida es el II/D = 45°. Se traza el segmento BH perpendicular a AD.
ingreso (en soles) de un grupo de trabajadores y se sabe
además que
Si AD = t , calcular IJH .

2 l' r,n
r, =} h H5 = 0,95 fi, = 10 A)-e
3
C)-
3
El- -
2
donde:
24. Se tiene un triángulo isósceles, cuyos lados de igual
.1; = frecuencia absoluta si mple
longitud miden b CII/. Para poder obtener un triángulo con
h, = frecuencia relativa simple la mayor área posible, el tercer lado debe tener una longi-
tud de:
H¡ = frecuencia relativa acumulada
¿Qué porcentaje de trabajadores ganan como mínimo 900 A),n b cm B) -b
,n CIII el ¡;, 1, 1'1/1
2
soles y como máximo 1300 soles?

Al 75 % B) 37,5 % C) 35 % D)bclII E)¡; "('111

D)30% E) 62,5 %
25. El mayor ángulo diedro qUI! forman dos caras adya-
centes de un octaedro regular de arista f I!S tal que su
coseno vale:
30. ¿Cuáles de las siguientes afirmaciones son verdade-
.fi ras (V) o falsas (F)?:
B)-2

,ÍJ I 1) tan (1283'¡')=-1


D)-3 E)--
3
11) sen(nTC)+.<ec(flTC)=(-IY' ' '<lilE Z
26. El radio R de la menor esfera hueca que contiene a ID) Si 3,Jsen(} . Jta;¡(} < O , entonces () pertenece al
cuatro esferas sólidas de radio r es igual a
tercer cuadrante.
A)FFV B)FVV C)VVV D)VFF E)VVF
A) .fi r

31. Si P = (x ; I - a) es un punto que pertenece a la


D) 2 r gráfica de la 'función seno, hallar:

A = (.<enx)(l - senx)(cosec x)
,27. La superficie total de un cubo es T. Entonces la dia-
gonal de dicho cubo es igual a: I
A)I-a C)- D)a E)a-I
a
.fiT
A).fi T B).fiT C)-2- D)2fi E),¡rr
32. Los extremos de la basede un triángulo son los pun-
tos A = (0;0) Y B = (3;0). Determinar la ordenada del
28. Una superficie S se obtiene por la rotación de un
trapecio isósceles alrededor del eje que contiene a su lado
mayor, Si dicho trapecio tiene un ángulo agudo de 60° y
vértice opuesto C =( -t; y ). de tal manera que la medida

bases de 4 cm y 12 cm de longitud. ¿Cuál es la medida del del ángulo CAB es igual al doble de la medida del ángulo
área en cm2 de S7 CBA.

A) 64 TC,ÍJ B) I 28TC,ÍJ C)32TC,ÍJ


JI5 JI5
B)2 C)--¡-
D) 112TC,ÍJ E)96TC,ÍJ

JI5 JI5
29. La figura muestra un montacarga con un tambor de D)-6- E)-8-

7TC
60 cm de diámetro, si el montacarga gira 4 radianes,

entonces la carga se eleva aproximadamente a una altura


33. La diferencia entre el valor máximo y el valor míni-
de: (tomar' TC = 3,1416) mo de la función f(x)=lsenxj+lcos.xj es aproximada-
mente igual a:
A)I,68 m A) 0,41 B) 0,42 C) 0,44 D) 0,46 E) 0,91
B)I,67 m
C)1,66 m . '
34. Si sec? x + cosec2 x =7 , hallar:
D)I,6.5 m
,E)I.63 m

A) 13 B)14 C)22 D)16 E)15

i
.::==::::::::::::=::=:::l~_U_N_1 2_0_0_2-_II_M_A_:T"_EM_ÁT_I_CA __ 1 :!

35. Sea ABCD un cuadrilátero y sea O el punto de inter- 38. En la siguiente figura: BM + MA = Be + CA .
sección de los diagonales AC y BD'- Si las áreas de las
La longitud de la semicircunferencia generada al tomar
regiones triangulares AOB, BOC y COD valen 1, 2, Y 4
como diámetro el segmento MB es:
m' respectivamente, el valor del área en m' de la región
triangular AOD es:
M
A)2 B)6 C)8 D)3 E)7

36. Hallar el módulo del complejo:


rL . A
CI_m----l
donde: z=cos9+isen9 y ge(n;3;)
A)tr(m-h)
A)tan9 B) cot9 C) 2wn9
2trhm rrhm
D) 2cot9 E) 4tan9 C) 2h+m D) 2(2h+m)

3rrhm
37. En la figura, hallar el área sombreada comprendida E) 2h+m
entre el triángulo ABC recto en B y la semicircunferencia,
sabiendo que el arco BT es de 1200•
39. Un cuadrado MNPQ cuyos lados miden
C
J2 - fi. u, está inscrito en una circunferencia. Calcular

la distancia del punto Q al punto medio del arco MN.

A) 0,5 u B)lu C) 1,5 u

D)fi.u

40. El punto de tangencia de la circunferenciainscrita en


un trapecio rectángulo, divide al mayor de los lados no
paralelos en dos segmentos que miden I m y 9 m respecti-
vamente. Luego la base mayor mide:
A) (3./3 -1t) e2
6
B) (2~ -·L2 A) 12m
~14m
B) 10m
ml6m
C) 13 m

C) (J'J +Tr) e2 D) (tr - J'J) e2


4 6

(tr+l)e2
E)-4-
i
t

SOLUCIONARIO
MATEMÁTICA PARTE I
1. Del enunciado: 7V
Ox-N=O % de vino en la mezcla = _6_ x 100%
:IV
N
~ x=/5 = 39%
Clave: D
Condiciones:
o
• Fracción ordinaria: 0,.10 ... (1) 3. Datos: VN = 900dólares
• Fracción irreductible: O y N son PESI ... (2) I = 60días

... (1)
~ ab c d ej lilll-abc N
• o.ab c d e f Ii 111 = 999999000 = /5 ... (3)
VNX/xr
2
Por definición: °c=---
• N-O = 10878 = 2x3x7 x37 ... (4) 36000

900x60xr
De (2) Y (4): D "# 2 ; 3 ; 7 ; 37 ... (5) 36000

3
De (3) Y (5): 999999000 = D = -r ... (a)
2
O = 53 X 11 x 13 = 17875
VNxlxr
Por definición: O,.
En (4) N = 17875-10878 36000+/ r
= 6997
900x60xr
~ N+D = 17875+6997
36000+60r
= 24872
Clave: D 900r
... (f3)
600+r
2. Graficando de acuerdo al enunciado.
Reemplazando (a) y (f3) en (1):

3 900r
-r - -- = 0,09
2 600+r
v
VINO X ~ r2-0,06r-36 = O
2

~ r=
0.06 ± J (-0.06i2 + 4(1)(36)

~ r = 6,03
.- El valor aproximado de la taza de descuento :ó''',
V¡ V '2V 7V
~O." T =6 Clave: C
_--'C_--
4. Resolviendo: ~ a2 +.jb = 1I +b Para c = 4 en (11): ;;¡ "= 48

(~ a2 + .jb r= r (a + b
En (1):
=> c=4 y d=8

a2 +,¡¡; = a2 + 2ab+b2 Ib48 = 13

=> b = 2
.jb -iJ2
=> a=--- Nos piden:
...(a)
2b :. a+b+c+d = 1+2+4+8 = 15
Analizando:
Para c =S en (11): ;;¡ =51
1) FALSO: b = I => a = O Si 3 a.b E Z
=> c=5 y d=1
I I
1I) FALSO: Si b =- => ,,=;->0 En (1): (No satisface)
4 8
11I) VERDADERO:
Para a = 2 en (11): cd = 66+3c ... (4)

La igualdad (4) solo se cumple si: e =9


Clave: B cti =93
=> c=9 y d=3
5. Factorizando la expresión:
M = (xy-7x+9y-63)'" En (1):

= [(y-7)(x+9»)'" => b = O

.. a+b+c+d = 2+0+9+3 = 14
= (y - 7)' (x + 9)'
Paraa=3en(lI): cd=66+3c ... (4)
El total de términos diferentes de M será: (m+ li
La igualdad (4) no cumple para ningun valor de de (".

Existen 2 respuestas; pero las claves se adaptan a:


Por dato: (1Il+li ~ 1998
a+b+c+d = 1+2+4+8 =15
=> In ~ 43.69899
Clave: A
Por lo tanto el valor mínimo entero: m = 44
7. Datos: Hinchas del equipo M : 11I
Clave: E
Hinchas del equipo W : 1/

6. Por condición: abcd = 13 •.. (1)


Total: m+1l = /¡ ... (1)
Inicialmente (antes del gol):
cd = 3 (liZ+2) (11)
o ••

Por condición: ti A
-;;¡=¡¡
= 3(10a +c+2)
tilA
=> 1/ =- ... (11)'
=> 10~cd ~99 ... (1)
B
Analizando: c = I ; 2 ; 3 ; 4 ; 5 ; 6 ;7 ; 8 ; 9
lilA

4 ~ IOa+c+2 ~33 ... (2) De (11) en (1): 1Il+-=h


B
Analizando: II = I ;2 ;3
hB
=>1Il=B+A ... (*)
Para a = 1 en (11): cd = 36 + 3c ... (3)

La igualdad (3) solo se cumple si: c = 4; 5


Despues del gol: 9. Dado: J(x) = 4+ _....:3"----~
4 _ 3sel1x

De donde: -1 :5 sen x :5 O

B B f (x): es máximo si sen. x es máximo


:) II-m- = x+x-
A A f (x): es mínimo si sen x es mínimo

De (11): mA)_m!!"=x+x--ª- .f . (270°) = 4+ 3(_1) = 5 3


( B .A A m/n 4-3 1 1
(A -B) .
:) x=m-- .f (360°) = 4+~ 5
B meL< 4_3\U}

Clave: E
=(..!:!L \(A-B)
De (*):
B+AJ' B 10. Para:

h(A-B)
= (A+B)
y
Clave: B

S.Analizando las cifras de primer orden:

------~~~-----x
.. (a)
606YS8(b) x+4 < 1y
y

30 = ;;;SQJ)

=mh+8
:) mb = 22 o: 2xll = Ix22 4/3
-""-•••
4..::::::.----+------- x
De donde: b = II ó b = 22
El menor valor de b es I I . y < x+2
Luego.. y

Pasando la multiplicación base 11:

194871(IO)x5(1{) =123456(1 1)x5(11) l.,.·'". X

974355 (IU) = 606058 (11) ... (f3) Intersectando,


I
los límites de las áreas
Comparando (a) y (~) . Y=O
x2 - y=O ... (1)

Finarmente: b+Y = (I 1)+ (O) x+4=3y ... (2)

~ II y=x+2 ... (3)


Clave: C
,-_U_N_I 2_0_02_-_II_M_A_:T_EM_A_'T_IC_A_ ••• I::::¡:: :===::::;;::::"==0
De (3) en (2): 12. a y b son enteros pares positivos
.r +4 = 3(x+2)
-b O

Reemplazando
=> x=-I ;

(x = - 1) en (I) :
y=1
1
1
.1 =[~
O
1
O
0]
()

(-J/-y=O Sumando las matrices:

=> y = I
lntersectando graficamente : A+1.[:
Punto común y

=[a~1
1
~
I b
~t]«1

Por condición: det(A + 1)= 12

-4 a+1 -b -(/
O 2 2 =12
Clave: A
I 1 b+1
11 . En el sistema:
(a+I)(2)(b+I)-2h-[-2a+2(a + 1)] = 12
(1 + i) z - IV = - 1 - i '" (1)
(/¡+I)(a) = 6
2iz+(I-i)IV = i '" (2)
Donde a y b son enteros positivos:

De (1): IV = (1 + i) (z + 1) '" (3) a=2 ; b=2


Luego:
De (3) en (2): 2iz+(I-i)[(1 +i)(z+ 1)]= i

2iz+2z+2=i B = [a lit]
b2 b
= [2 4]
4 2 "

-2+;
=> z== 2+2i
=> det (8) = (2)(2) - (4)(4)

Operando: '" (4) =-12


Clave: A
Reemplazando (4) en (3):

13. Del enunciado:


IV= (I+i)[(! i-t )+IJ
Q=aU+¡3V a;¡3eR
3i
2

+~[:J
()
2

3, I Q.a[i 4 O
:']
Finalmente:
.L 4t-4 2 1 -1 O
IV 3i
2
También: ",(1)
1+
2 6
Clave: C
5~;::. tLt-: :::1
u:at _'
UN12002-11 MATEMÁTICA Idlit"",
WR:·-i-#
la liS"
i.··'.#.ihifi'dé~

1 2 1][1] [1 o -1] [1] [1] s= (tll+XJ


2 1I

a[~ : ~ ~ + P, ~I ~ ~ ~, = p ~

o Por dato: 816 = 2+XJ


( -2- 1/

De igual forma:
~{l+]~ a = .E.
6
De (a):

~
816- _ (2+XJ(
(x+ 146) (x-134)
-- --X+IO)
2

= ()
12

~ x = 134 ; x =- 146 (dcscartudo )

Reemplazando
+J='E]
Q en (2):
... (2)

I 15_ ~
~ x = 134
Clave: C

[al: : :] +B[~, : :']E]=q[~J i\ 1ra generación = 21 ancestros

[
a~
1 2

o
1] [ 1]
+P
[1
~I
O

~
-1] [ 1]
~ ~I
[ 1]
=« ~\ ~~~*
/ \ ~
2da generación = 22 ancestros

~ p [ ~ ] = lj r i¡] ~ p= %
-2

De donde. (u) y (P) son arbitrarios.


l-l

Clave: E
~*~ •~ loma generación = 210 anccstros
= 1024 an, -tros

14_ La serie aritmética: Clave: C

2+ 14 + 26 + 3g + ... +x =816
16. Perrnutación circular de 4 grupos:
Razón: r = 12
1~=(4-1)!=6
Primer término: 01 = 2
Numero de tcrminos: 1/ Además cada grupo pueden perrnutarsc así:
~~~;';~~':;==~
__ U_N_I_2_0_02_-_II_M_A_:r_E_M_A_·T_IC ••A_ •••• 1=-r:::'T='"¡;;:::r0
- 3 argentinos 1:;=3!=6 d = J (x - 6f + (y - Or
- 4 peruanos ~ = 4! = 24
= J(x-6i +/
- 4 chilenos ~ = 41 = 24

- 2 bolivianos 12=2!=2 De (a):

Total de maneras:

= J 3(X-4; + 72

= 6 x 6 x 24 x 24 x 2

= 41472 Para que d sea mínimo: (x-4) = O

Clave: D

=> dmm
- J 3(0)+72
- ---
2
17. F(x) = I~I [(x-I)2+2Ixl]
=6
De donde: Clave: D

Ixl *- O => x > O Ó x < O


19. Analizando las afirmaciones:
Para x > O: Ixl = x
2
F (x» 1 l. Si X E (-I ; 5) => --E (0;1/
2x+5
Para x < O: Ixl = -x Veamos:
-1<x<5
F(x) = 3-(x-2T => F(x)<-I
Finalmente: 3 <2x + 5<15
F(X)E R-[-I •.1] 1 3
-<---<1 (VERDADERO)
Clave: A
5 2x + 5

18. De la ecuación:

2 y2_x2 = 48
11. Si XE [0;4) => P6-X_
x+2
.[;+1> O

2 x2 + 48
Y =--- ... (a)
=> P6-X x+2
> .[;-1
2
Graficando: f(x) >!i (x)

Analicemos cada función:

¡(x) =
P6-X
x+2
=> x-16
x+2 S; O

~ (-) ~
-2 16
(6;0) x
x E [-2; 16]
La distancia de un punto de la curva al punto (6; O) está
dada por:
~~==~:I __
U_N_I_2_0_0_2_-I_1
_M_AT_E_M_A_·T_I_C_A_....JI==~=:;::====::~~:~::
K (x) = .Jx - I =} x >O Por teorfa:

=} X E [0,00)
• 176 = H6 - Hó = 1.0 - 0.95 = O.OS
Considerando la condición inicial:
xE[O;4) =} DomfLx¡ = [0;4) Por definición: 11 = 1L =} 1/ = fr, =...!.Q.. = 200
h, /¡6 0.05
Dom K (x) = [0,4) h = /¡3 (n) = 0,25 (200) = 50
Si min f (x) > max g (x) será suficiente para considerar Por dato:
que la expresión es verdadera
I I
f~ = - f~ =} f~ = - (50) =25
2 - 2
minf(x~4) = JI(~)~~)=,fi
Trabajadores que ganan entre:
max g (x~4) = J4 -1 = I
T [900 ; 1300] = 50+25
Como min f (x) > max g (x) = 75
=} la expresión es ¡VERDADERA!
Total de trabajadores: 200
m si x-I
-- > x =} x <-3 75
x+3 % pedido =- x 100 %
200
x-I = 37,5 %
Veamos: ---x>O
x+3 Clave: B

(X+I)2 < O
x+3 MATEMÁTICA PARTE 2
21. Graficando según el enunciado:
~
-3 -1 -1 L-x
=} x < - 3 (VERDADERO)
Ar-=2~_~~-=_~Fr-_~x~~B
Clave: E

L
20. Construyen I i.ihla de distribución: "2
7{)'. -500
Ancho de clase: I - -- = 200
M

i INTERVALOS ¡; H¡
L
n
- "2
1 [500;700>
f--
2 [700;900>
3 [900;1100> 0,25 D'--'-U-----L-------'P-'--' ('
4 [1100;1300>

5 [1300; 1500> 0,95 Del gráfico: óPEF _ ÓPI r:

6 [1500; 1700> 10 L-x


TOIAL =} !.!!:= _2_ ... (IX)
n L
~~~'Z~========:(~U~N~I2~0~0~2i-ICI
~M~A~:r!E~M~Á~T!IC~A~J=========~59
Teorema de Menelao en el Ó. A F D Si BH=IIl+n =) PC=m+n

f.XlllxL=f.xnxx En el triángulo is6sceles CED


2 2
ED=CE=m
Finalmente:

L-x
AD AH+HE+ED
De (a): x= i.« 2
L l= n+(m+n)+1II

L =) 1.= m+n
x=- 2
3
Clave: C - J.
=) BH = n+1Il = "'2
22. Graficando según el enunciado: Clave: B

-/\, \
24. Graficando

Del gráfico: F 8 C = DO' B = 120° (O'D/lOF)

Por dato: DEIIAC El área de un triángulo isósceles

o fj D = a = 30° bxb
S = -- sen a
Clave: E 2

2
23. Graficando de acuerdo al enunciado b
="2 sena
B
Por propiedad: -1 $ senci $1

Para que S sea máximo:

un a = 1 => a 90°

Luego: x=Jb2+b2
=hh
------J. Clave: A

Ó.ABH =ó.BCP

Si AH = n =) BP = n
~·=:======:I__
~~
U_N_I 2_0_02_-1_1 _M_A_rE_M_A_-T_IC_A__ ~P.:
1:=
~~====~~UJ~ Gl'im.;z-
25. Graficando el octaedro regular que se caracteriza Los vértices del tetraedro regular ABCD son los centros de
porque sus ocho caras son triángulos equilateros. las 4 circunferencias sólidas de radio 1: inscritas en la cir-
cunferencia de radio R y centro "O".
E
Propiedad: ... (1)

En el triángulo rectángulo CHD :

- - = J (2r)-- (2r)2
CO+OH .rs 1

De (1): (30H)+OH =2r2Jf


~ OH = Í6 ... (2)

Del gráfico, por propiedad:

R = CO + r

F De (1): = (30H)+r
cos2a = 2cos2 a-I ... (*)
De (2): =[3(Í6)J+r
Del gráfico: En el triángulo rectángulo EOM

L
1
=r(I+Jf)
cosa = _2_
l~ Jj Clave: B
2
Reemplazando cosa en (*): 27. Graficando,

cos2a = {.k )-1 3


A
Clave: E
26.Del enunciado: I
1 ____
a /
/
/

/
/
a
/
/

a
Superficie total: T = 6112

~ l/= Jf6" ... (*)

Diagonal AB: d = aJj

De (*): = (~)x~
lIT
=-2-

Clave: C
UN12002-11 MATEMÁTICA 61

28. 30. Analizando las afirmaciones:

K = 8cm I.
8cm g
r = 4.J3 cm
4cm
31t] =-1
-:.:.:' 4:/5 SI!!: _-:..-:..-:..:.:.:.-:. h =4cl/1
tan
[
3201t+
4
4cm 4cm h tan 31t = _1 (VERDADERO)
4
----------------
11. un(II1t)+sec(n1t) = (_1)"

sen (1I1t) = O V It E Z

Superficie lateral: S L sec (11n) = (-1)" "lilE Z


La superficie total del sólido está dado por:
~ sen(11 1t) + sec (1I1t) = (_1)" VII E Z

S = 2SL_wnll + SL-álintl", (VERDADERO)

=2(lr r g) + 2lr r n 111. Si 9E III Cuadrante

= 2lr(4.J3 cm) (8cm)+2lr(4J3 cm)(4cm) (+) (+)

= 96lr,fj cm2
Clave: E

29.
senñ < O

LA
GJ (-) (-)
lan8 > O

~ Usel!8 x Jtalt8 <O (VERDADERO)


Clave: C

31. Por definición si:


P=(x;l-a)E f(x)=seltx
Dato: r = 30 cm = 0,3 m
~ sen x = 1-(/ ... (*)
La longitud de arco girado es igual a la altura elevada de la
En la ecuación dada:
carga.
L=(XXr
A = sen x O-serlx) (ese x)

7lr = sen x (I-seltx)(_I_)


= -x (0,3m) sen x
4
= I-senx seIlXi'O,
7 x 3.1416
----X 0,3 m De (*): = I-O-a)
4
=a
= 1,64934 m Clave: D
~ 1,65m
Clave: D
32. Graficando de acuerdo al enunciado: 34. Del enunciado:

y 2 2
sec x + ese x 7
C(I/2 ;y)
1 1
+ 7
co.\·2x sen2x
sen2x + cos2 x
7
cos2 x sen2 x

7 ... (*)
Del gráfico: 2 2
cos x sen x
y _ 2y Operando en la ecuación:
tanU=-- -- ... (1)
3-1 5
. 2
E = (sec 2 x + tan 2 x) (csc 2X + ctg 2 x )

(_1 (_1
/an2u=L
1
= 2y
.. (2) + sen ?
2
x) +cos
2
x)
2 ? ,.,
2' COS X CO,\'- x sen: x scn: x

( 1 + sen2 x) (1 + cos 2x )
Por trigonometría:
cos2 x· se,,2 x
2 2 2
?(2\'
- )
.:)
I+cos x+sen x+co,\'2
cos:
?
X' sen" x
?
x·sen x

De(l) y (2): 2y= (2.")2


1-- 1+ (1)+
2
cos x· sen x
?

~ ? ?
cos : x· sen" x
íI5
~ Y=VT =2( co.\,2x
1
sen2x
)+1
Clave: B
De (*): = 2(7)+1
33. Analizando gráficamente. =15

Icosxl Clave: E
ttx) = Isenxl+lco.l"xl
Isenxl
35. Graficando de acuerdo al enunciado:
~ ~~C

Del ~r:ífico: 11, )",,,, = Ji


Por propiedad: D
f (x)""" = 1
Del enunciado

f (x ),,,••.•- f (x )"'iu = .!2 -1


=0,41
Clave: A ~x =211,1

Clave: r\
FJ~!~Z::..e,i==,-_U_N_I_2_0_02_-_II_M_A_:r_E_M_A_·_T_IC_A_.....J 1=
36. Dato: z = CIISe + i .~elle Del gráfico por geometría:

2
=) Z = cos2e + i sen Tñ ... (*) Asomhrc;ktt =A trj¡ín!:!ulu - Ascmidn:ultl

En la ecuación:
ABxBC 1 2

iz+1 iz-I
=--2-- - '2rrr
W =--+--
iz-I iz+1

(iz+I)2 (iz-li
=-Z--+--2-
-z -1 -z-I = ~2./3-lrr(-LJ2
2 2 ./3
~2./3 d2
=-2---6-

_ 2[clls2e + i.l'en2e -1) Clave: A


De (*): - cos2e + i.l'en2e + 1
38.
_ 2 [(co.l'2e-l)+ isen2e) M
- (cos2e+I)+ i.l'en2e

2
_ 2 [-2sen e + i2senecose1
- 2
2ClJS e+i2seneClJSe

_ 2sena (-sene + icose)


- cosa (cosa + i sella)

= 21t1l1ai
=) IWI = 2 tan e
Clave: C
Por condición:
37.

=) r = "11I
.. , (*)
2(2" + 11I)

Por geometría se sabe la longitud del arco L:


L = ttr
"11I )
De~*): .=1t ( 2(211+",)
f
1thm
Del gráfico: r = e tan 30° = ./3
- 2(211+11I)

CB = l1K60° = e.;5 Clave: O


0m~!;M'iWé~~~!I __U_N_I_2_0_0_2_-_II_M_A_J_E_M_A_'T_I_C_A_ •••••• t===~==:¡wlli:':lli'*¡ill:~'~!I,~e';'I¡:
39. Graficando dc acuerdo al enunciado: 40. Graficando:

En el triángulo rectángulo CHD:

J (2r) 2 +(8111) 2 =1011I


=> r=3m
La base mayor:

Dato: l=h-.n u _._(1) = (1" + 11Il) + 8/11


= [(3/11 + Im)]+ 8m
Por trigonometría: cos 22,5 = J 2 + J2 (11)
~ 12m
Del gráfico, triángulo QOP: Clave: A

RJ2 =l

=> R=~
.fi

De (1):
_(F72) ... (*)
- J2
En el triángulo rectángulo HTO:

QH
2 =R ClJ\"
"- 22 ~o

=> QH =2 R cos 225

De (*) y (11):

=1
Clave: B
~~~~~~~~=~
L.""-==--
U_N_I_2_00_3_-I_M_A_T_E_M_A_'T_lr __ \ __ I~~~~~~~~~~~

~~[J~~~
MATEMÁTICA

MATEMÁTIOA PARTE 1 I ; I<!O


2. Sea MI) =
{ O;
, 1<0
1000
1
"3 o o Si definimos g(I)= h(/+2)-h(I-2),
1 1
o "2 "2
1. El valor de es: entonces se cumple que:
1
O O
"2
Al g(,):¡¡ 1<1
l<t<2
1 O o 1>2
3

¡¡
A)
O 1 (~)I"M) /:::;I
2: B) Se,) ~ 1<1 < 2
O O 1 / "2 2
2

B)
(*tW~) O

XXI
O el g(') ~ ¡¡ 1<1
1:::;/<2
I "22
o (!t 1
2

O O (~tXM) D) g('):{¡ -2<1<2


i

/"22
s: -2

,(,):¡¡
1
O O 1 <-2
"3
C) 1000 1000 E) -2:::;/<2
O -2-
2 /"22
O 1
O
2:
3. Hallar el número de raíces que tienen la ecuación
1000
O O
IlogJrll+.\" - 'í = O
~
D) (~)IO()fI 1000
A) I B) 2 C) 3 r» 4 El 5
O
2IOO()
ooo 4. La población de venados de una región esta dada
O O
(!r por la función V (1) = _14 + 21/ 2 + 100 . donde / es el
1000 tiempo en años. Entonces, el intervalo de tiempo, donde

,;1
O
3 ocurre la población máxima de venados es:
E) 1000
O -2- A) [O; j] B) [ I ; 2] C) L 2::3 1
O 1000 D) [3; 4] E) [4; 5]
O
2
ªª~~~~~~~~ U~N_12_0_0_3-_I
__AT_E_M_A_'T_IC_A
__ I~~~==~~=
9. La cantidad de cifras de los números A , B Y e son
5. Calcular el valor de K = Ja+C-5 : si la división: números consecutivos. Si el producto A·I H , e2 tiene
(l-(

por lo menos 125 cifras. entonces la cantidad máxima


X21 -ax+c
es exacta. de cifras que puede tener dicho producto es:
x2 -x+ 1
A) 130 B) 131 C) 132 D) 133 E) 134
A) 10 B) 8 C)2 D)6 E)4
10. Sean P •. q el menor y el mayor factor primo del
6. Para cumplir con el pedido de un lote de artículos de
exportación se trabajó durante 16 días de la siguiente número N = 1004006004001. Si q - P = 6. enton-
manera: El primer día trabajaron 9 obreros, el segundo ces la suma q + P vale:
13 obreros, el tercero 17 obreros y así sucesivamente.
A) 16 B) 20 C)32 D) 40 E) 52
Si todos los días se hubiese trabajado con 15 obreros,
20% menos eficientes; entonces el número de días en la 11. Juan invierte SI. 50000 a una tasa del 12% de
que se habría acabado el pedido, es: interés simple anual. Al cabo de 3 años, invierte la
A) 69 B) 63 C) 56 D) 52 E) 48 utilidad a una tasa del3% de interés simple mensual. Si
luego de transcurrido un tiempo "1" la utilidad de la
segunda inversión es e175% de la utilidad de la primera
7. La gráfica de la siguiente desigualdad: x2 + y2 < 2 es:
(en los 3 años), y si no ha retirado la ir • rsión inicial.
entonces el monto total asciende a (en SI.)
A) y B) Y
A) 98 000 B) 94 000 C) 93 000
D) 81 500 E) 80 500
x
-2 12 12. Si el promedio de 10 números de entre los 50
(cincuenta) primeros enteros positivos es 27,5 ; el pro-
medio de los 40 enteros positivos restantes es:
C) y
A) 20 B) 22 C) 23 D) 24 E) 25
,, ,
,
,
, X
13. Carlos debe almorzar pollo o pescado (o ambos)
-2, ,2 en su almuerzo de cada día del mes de marzo. Si en su
. ,, ,
, almuerzo durante 20 días hubo pollo y durante 25 días
hubo pescado, entonces, el número de días que almorzó
pollo y pescado es:
D) Y E) Y A) 18 B) 16 C) 15 D) 14 E) 13

,, ,
, x X 14. Dos amigas compran a y "papayas (a> h) res-
-12, , ,12 -2 2 pectivamente; en el camino se encuentran con un ami-
go y deciden compartir entre los tres las papayas, en
partes iguales. Si el amigopago "P" nuevos soles por
su parte, entonces la repartición del dinero entre las dos
8. Sean los números a y b tales que amigas es:

(a-b)P . 2bP
A)~. a+h
¿Cuántos pares ordenados (a ;") son soluciones?
2aP (a-h)P
A) 1 B) 2 C)3 D)4 E) 5
B) a+" ; --a+h
(b-2a)P (a-2b)P
€) =i:»: a+h
____ U_N_I_2_00_3_-I_M_A_~~E~M~A~'T~IC~A~'.~~~~~~~ <3>
(20-b)P . (2b-a)P MATEMATICA PARTE 2
O) a+b ' 0+17 20. En un exámen, un estudiante debe resolver 10
preguntas de las 13 dadas. S i tiene que contestar nece-
E) aP . bP
a+b ' a+b sariamente por lo menos 3 de entre las S primeras.
entonces el número de maneras en que puede elegir las
¡ O preguntas es:
1 5. Sea la sucesión S O ; SI; S 2 ; ... ; Sk : ..• donde
I A) 80 B)220 e) 276 O) 286 E) 316
So = 49 ; SI =7 •. S2 = J7 ..... Sk = 7k(k-l) ,
21. El número de elementos del conjunto
para k e 2. Entonces la suma de las cifras del pro-

ducto de todos los términos de la sucesión será igual a: F = {XE [0,21t]/ cos2x secx+ secx+ 1 = O} es:

A») B)4 e)s 0)6 E) 7 A) 1 B)2 e)3 0)4 E) S

22. Si Ig x+ctg x= 2, y
16. En la sucesión de números reales

202S+x2k ¡;;-----;-(It: 1/ X+t'I" IIX)


xk+1 ' 2x ; para k = 0,1.2
... . V'X ".r+Clg 11 x ' .'
k
~---_~~'l:-II-X+-(-'I~-'
-;
Se sabe que x 5 = 4,S ; entonces xI 05 será igual a:
E=Jeglx+clg"x .
A) 4,S B) 4,SS C) 4,SSS
Siendo 11 potencia de 2, entonces el valor de E ~ es:
O) 4,SSSS E) 4,SSSSSS
A)2 B)4 e)8 D) 16 E) 32
17. Si I zi 1=4, Arg [z(1 +i) J=1-' entonces el número
23. Resolver la siguiente ecuación trigonornérrica:
complejo Z en su forma polar es:
ctg 1 = Se/1,X+Clg x

A) !(2k + I)Jr B) ~(2k + I)Jr

e) i(2k + l)rr O) !(4k+l)Jr

E) !( 4k + 3)rr
18. Sea N el número de pares de números reales
24. Simplificar la siguiente expresión:
(x, y) que son soluciones de la ecuación:

Jy-x+Jx- y = x2 + y2

Entonces N es igual a:
A)O B) l C)2 0)3 E) 00

A) sell(6~;rr) B) cos(3X+Jr)
19. El número de raíces de la ecuación:

JI-9x2 =2xJI-9x2 es igual a: C) cosex¡;Jr) O) cos(6~;Jr)

A)O B) l e)2 0)3 E) 4


E) cos( 6x ; Jr)
l
:.s.:
UN12003-1 MATEMÁTICA
~!~
GomeZ

25. Los lados de un triángulo miden en metros J2 ,


.f6 y J8. Hallar la longitud de la menor altura.

A) J2111 B) .J6m C)¡¡;;;


2 2

D) ¡6-;;; El rs;;;

26. Una circunferencia es tangente a tres lados de un


paralelogramo. Si las alturas del paralelogramo miden
16 y 20 unidades. Calcular la longitud de la cuerda ence-
rrada por la circunferencia en el lado no tangente.
A) 41f:R B) ~nR C) 3nR
A) 12u B) 14u C) 16u D) 18u E)20u

D) .!Q1f:R E) lLnR
27. En el interior de un cuadrante de una circuntercn- 3 3
cia CI de radio R, se construye una semicircunferencia
31. En la figura mostrada. ASCD es un paralelograrno:
C2 ' cuyo diámetro es uno de los radios del cuadrante
M y N son puntos medios de A S Y A D respectiva-
de C I . Hallar el radio de otra circunferencia C 3 tan-
mente. Hallar el área de la región sornbrcada. si la
gente a CI ,a C 2 ya un radio del cuadrante.
medida del área del paralclograrno es 120 u 1 .
A)E 8)4
6 :> ~ ~~ --iS

E) !l
7

28. La suma de dos ángulos exteriores de un triángulo


miden 270 el lado mayor mide 48/11. Hallar la dis-
0
;

tancia del baricentro al ci, .uncentro. D

A)6/11 ai s .» C) 12/11 D) 16/11 E)20m A) 30,Ou2 Bl 27.5u2 C) 25.0u 2

29. Tres rectas se: uncrsccan dos a dos. ¿Cuántos D) 22,5u 2 E) 20,5//2
puntos del plano, determinado por dichas rectas,
equidistan de: las tres rectas? 32. Hallar la medida del área de la región comprendida
A) Uno R) Dos C) Tres entre la curva de ecuación: x2 + Y 2 + 4x - óv + '-1 = () .
D) Cuatro E) Cinco
y la circunferencia que pasa por el punto P (2,6), que
es concéntrica con la curva anterior.
30. En la figura se muestra una circunferencia de radio
R, y dos diámetros perpendiculares. Con centro en los
extremos de estos diámetros se trazan arcos de circun-
ferencia de radio R. Hallar el perímetro de la región
sornhrcada. El 257tu 2
UN12003-1 MATEMÁTICA

33. Si S es la suma de las medidas de los ángulos 37. IX Si es la medida de un ángulo agudo tal que
diedros de un tetracdro. entonces se puede afirmar que:
cos 1996° = +s e n o: Calcular el valor de:
A) S E (21t ; 31t) D) S E (41t 81t) E = ese 150: - U/J 150:

B) SE(1t; 41t) E) S E (21t 61t) A) I B) 1.5 C) 2 D) 2.5 El3

C) S E (21t ; 41t)
38. En la figura ABCD es un cuadrado cuyo lado mide

34. Se tiene un tetraedro regular ABCD cuya arista 8 cm. Si Ig8° es aproximadamente ~ . determine el

miden "(1" unidades, en la arista A D se ubica el punto valor de "r".


O. En la pirámideABCO, la longitud de la altura traza-
da desde el vértice O es igual a la longitud de OD.

Calcular OD.

A) a(J3 -.J2) D) a(J6+.J2)


B) a(J6-2) E) a(J6+2)
C) a(J3+.J2)
"-- .L.J D
35. Sea la pirámide S-ABC cuya altura cae en el cen-
A) C) 3<:111. D) 4<:11I. E) 5wl.
tro O de la circunferencia inscrita en su base. Si
39. Dada la íunciónj, definida por:
AB=1201lt;AC=IIII1I;BC=39m y
((x) = sellx+cosx
SA = 4Jl217 III El volumen del sólido en 111J es: . I-senx+cosx
Al 72 000 B) 72 400 C) 72 480 si k es un entero no negativo, entonces los puntos de
D) 72 640 E) 72 810 discontinuidad del son:

36. En la figura siguiente AB = BC = CD = r, don- A) HC4k + I)rr} D) {2rr k}

de r es la radio de la semicircunferencia de diámetro B) {(2k +1)rr }v {k 1t} E) {krr}


A D. Determinar

sólido generado
en función de "r" el volumen

al rotar la región sombreada alrededor


del
C) H( 4k + I )1t}u{( 2k + I )1t}

del segmento AD .
40. Hallar los valores x en el intervalo (O.rr) para
los cuales existej si:

f(x) = I
.JI +senx- 2co.\·2 x

1t 5 1t)
D A) [lj 23 ] D)(~
A 6
J rtr 3 2m' J
A) m· 6 C) . 51t] 51t)
"""2 B)
3 B) [~ ' 6
E) (~
3 ' 6
3 rtr J
D)~
3 E) lT C)(lj 1t
23 )
UN12003-1 MATEMÁTICA

SOLUCIONARIO
m
MATEMÁTICA PARTE I w O O 1
3
() o
A'OOl= O (t)m 999(t)m x o 1 1
3
l
o o
O O (t)WJ o o
2 2
.L
1. Sea A= O 1
2 2 l 2

O O
2 l ( \ 'ex'"
"3) O o

Cálculo de A 'I~X) O
(MC«¡ I<XXl{t)' 001
l
O O l
O O o O (t)'00l
"3 "3
A2= O
2l l
2
x O I
2 2
l
No existe clave:

O O
2
l
O O
2
l 2. Dada las funciones h (1) Y g (r):

... (1)
h( I)= { 1 ; t ~O
(t) 2 O O O ; 1<0 ... (2)

(tf 2(t)2
g(I);'" h(t + 2)-1z(1- 2) ... (3)
O
• Si I - 2 ~ O ~ t +2 ~ 4 >O

O (tf O ~ t ~ 2
Luego de (1) se deduce que:
o·· ti)

h(t-2)=1

(tf O O I
"3 O O h(t+2)=1
En (3):
A3 = O (ir 2(tf x o 2l "2l g(t)=I-1

o o (tr o o I 2 =0

• Si t +2 < O ~ t- 2 <4 <O

(tf O O => t <-2


Luego de (2) se deduce que:
o·· (11)

O (tf 3(tf h(t+2)=0

O (tf
O
En (3):
h(t-2)=0

g(I)=O-O
=0
UN12003-1 MATEMÁTICA lS:~§~~~:::;:
• En el intervalo -2 ~ t < 2 4. La población de venados está dado por:

Para: -4 ~ t - 2 < O v (1) = _t4 + 21t + 100


De (2): h( t - 2) = O Dando la forma:

Para: O~ t +2 < 4

De (1): h(/- 2) = I De donde V(t) será máximo cuando:

En (3):
g(I)=I-O

Finalmente:
=1
~ =ff
0 t <-2 =3,24
-2 st < 2 La población máxima de venados ocurre en el intervalo
g(/)
1
= ~
1~2

Clave: E
de tiempo:

tE [3; 4]

3. En la ecuación: Clave: D

1/"!i2Ixll + x2 -5 =O
5. Del enunciado:

Ilog2Ixll=5-x2 X21 - ax+ c


x2_x+1
Si hacemos:
Para aplicar el teorema del resto primero hacemos el
fl(X)=llog2Ixll artificio, multiplicamos al numerador y denominador
h(x)=5-x2 de la fracción por: x +I
Graficando:
21 + (x21 -ax+c)(x+ 1)
y x - {LX C - -'--0--;:-----,:-'---
x2-x+1 - (x2-x+I)(x+l'

(x3 f -ax+c(x+ 1) ...(*)

.f¿(x) = 5-x2 x3 +1

x3 + 1=0
x x3 =_1

Aplicando el teorema del resto en la división (*). e


igualando a cero (O).
Las dos funciones son simétricas con respecto al eje "y",
((_1)7 -ax+c)(x+ 1) = Ox(x+ 1)
se cortan en 4 puntos, por tanto la suma de las dos funcio-
nes: -1-ax+c=O

fl(x)+ h(x)=llog2Ixl/+x2 -5 =0 ~ -ax=O ~ a=O x;tO


Tendrán 4 raíces. ~ -1-O+c=O
Clave: D c=1
Luego en:
7. Ladesigualdad x2 + y2 < 2 representaaunsuperticie
K=~a+e-5 Para hallar la gráfica recurrimos a analizar la ecuación
a-e
x2+y2=2
= ~o+ 1-5
0-1 que expresado así:
=2
X 2 +y-= (~)2
,,2
1

Clave:C

6. Datos: representa la ecuación de la circunferencia de radio J2


Total de artículo a producir :T y
{2
arto
' Ios que pro d uce un dfla un o b rero: A = obr.
A rttcu

x
Primer casó: Eficiencia 100% (1.0)
-{2
Primer día : A=..!!.!:!.:..x90brxldia
obr.
-{2
Segundo día: A = art. x l 3 obr xl dia Para saber si la zona que representa la desigualdad es la
obr.
interna o externa a la circunferencia, es suficiente vcri-
A=JI..I:.l..:.xl70brxldia ficarlo con un valor, en este caso un interno:
Tercer día
obr. x=O; y=-D

x2+ l <2

16avodía A=JI..I:.l..:.x690brxldia (0)2 + (0)2 < 2


obr.
0<2 [es correcto!
Sumando los 16 días, tendremos el total de artículos T

Luego el gráfico de esta y


T = A i!!lxl dia(9+ 13+ 17 + ... +69) obr
obr desigualdad, considerando
{2
que la desigualdad (e) no , ,
= A artxdia[(9+ 69)1
2
6]
incluye los valores iguales ,, ,
t \
al radio, la gráfica seni: x
°
I I

=624A(artxdia) ... (1) -{2 '. ,'{2


,
, ,I

Segundo Caso: Etic rcncia 80% (0.8) -{2


Clave: O
Días trabajadas: f)
8. Para resolver la ecuación primero expresamos a
Total artículos: T = A(0.8)il!J.. x 150br x D
ohr ésta en el sistema de numeración decimal.

= 12A D art. ... (2)


U,I(311) + 0,b(12) = (2(4) + 0.Í(4»)( 0.1(3))
Igualando (1) Y (2):
= 624Aartxdill = 12ADart.
~+
10(311)
17(12)
10(12) -
-(2 (4)
+-1-1-1-)
30(4) 10(.1)

~ D=52dias
L + /; =(2+ 112 )(t)
Clave: O 12+ :'a!J= 25a
a(25-Jb) = 12 ... (*)
De esta expresión se deduce que "a" es divisor de 12 y La cantidad máxima de cifras que puede tener
b ::;8 , para que los números sean naturales ( J:1:l ), es A4 n' e 2 , de (*), será: '
decir:
En (*): n",,,x =9x+7
si a = 3 = 9(14)+ 7
=133
3(25-3b)=l2
~ b=7 (be J:1:l)
Clave: O
si a =6 10. Datos:
6(25-3b) = 12 N = 100 400 600 4001
Menor factor primo de N: p
~ b= 23 (bé J:1:l)
3 Mayor Factor primo de N : q
si a = 12 También se sabe que:
12(25 - 3b) = 12 q-p =6 ... (*)
~ b=8 (be N) Analizando ~I número N se deduce que
Luego concluimos que tenemos dos pares ordenados
o o o o o

(a ; b) de respuestas: (3 ; 7) Y (12 ; 8) N,,;2;3;4;5;6


Clave: B N =7 (mínimo)
~ 1'=7
9. Se dan los números A , 8 , e
Reemplazando en (*):
Por dato el número de cifras de cada número es conse-
cutivo, es decir: q-(7)= 6
El número de cifras de A : x ~ q=13
El número de cifras de 8: x + l Luego calculamos:
El número de cifras de e: x +2 p+q=7+13
= 20
El número de cifras de estos números estará dado por:

IOx-' ::; A < 10x Clave: B


10x:o; 8 < 10-<+' 11. Considerando la inversión en dos etapas:
1Ox+' :o; e < 1Ox+2 - Primera etapa:
Si elevamos a las potencias respectivas cada uno de los Capital inicial e, = SI. 50000
números A , 8 Y e, tendremos:
Taza o razón /", = 12 % anual
104x-4:o; A4 < 104.<
Tiempo :T = 3 años
L03x :o; 83 < 103x+3
Interés : "
102x+2 :o; e2 < l02x+4
Por definición
Multiplicado: 109.<-2:0; A4 83 e2 < 109X+7 ... (*)
e,xl7xT
Por dato del problema se sabe que el número mínimo de ',= 100

cifras" A 48 -c 2" es igual a 125. Aplicando propiedades: (S 1.50000)x (12--b-)X


ano
(3 años)

(9x-2)+1=125 100
= SI. 18000
~ x=14
- Segunda Etapa: Promedio de los 50 números:

Capital C2=/,=SI.18000 P. - 1+ 2+3+ ... +50


50 - 50
Tasa o razón /"2 = 3%
50(50+ 1)
Tiempo T2 =t 2
50
Además se sabe que: = 25.5
75 Por propiedad y reemplazando valores:
12 = 100/,
P¡oxIO+ P40x40 = ~'iO x50
= 1~0(SI.l8000) (27,5)(10)+ P40(40) = (25.5)(50)
= SI. 13500 P40 = 25

Cálculo del tiempo t :


Clave: E

13. Si A es el conjunto de días que Carlos comió pollo,


y B el conjunto de días que comió pescado, entonces
por datos del problema se tiene:

Días que comió pollo : 11 (A) = 20


100
~ 1= 25 meses Días que comió pescado : 11 (B) = 25
Días que comió pollo y pescado: n (A (l13) = x
Como no se retiró el capital inicial de SI. 50000 duran-
te los 25 meses éste siguió ganauo intereses al 12 % Total días que comió (Marzo) : I! (A u 13) = 31
anual, es decir:
Graficando:
1 - CXrXI
3 -J(j()

(SI.500oo)X(12~)x(Naii(JJ )
100
= SI. 12500

Finalmente, el monto total asciende a: Luego del gráfico:

11 (A + 13) = (20- x)+ (x)+ (25 - x)


M =C+/1+1!+l,
31 = 45-x
= S 1.50000 + S 1.18000 + SI. 1:1:i()O+ S 1.12500 x= 14
= SI.\l4()()() Clave: D

Clave: B 14. Datos:


A tiene : a papayas
12. Datos:
B tiene : b papayas
Promedio de los 10 números: P,o = 27,5 B pagó por su parte: P nuevos soles
Los primeros 50 números enteros positivos Por condición del problema a cada lino de los tres le
corresponde "N" partes iguales de las "a + b" papaya,.
l ; 2 : 3 : ... ; 49 ; 50
es decir:
N = a+h ... (1) 15. De la sucesión tenemos:
3
Considerando que B paga por su parte" P" nuevos soles, So =72
El costo unitario de papaya será: SI = 72-1
p=.f.. 1_1
ti N S2 =7 2

De (l):
=.....L 1 1
a+b S3 = 72-3
-3-
LL
S4 = 73 4
_ 3P
- a+b ... (2)

• La cantidad de papayas que aporta A para e será:


DA = a-N
Por lo tanto el producto de:
De (l): =a_a+b
3 n = SoXSI XS2 XS] XS4 x ....
2a -b 1 1 1 1 1
=-3- ... (3) = 72 x7x71-'i X7'i-T x7'i-¡ x ...
()

La cantidad de dinero que recibirá A por su aporte:


=7
4 -4+4-!+~-¡+
.
....
PA = DAxPU = 2401
De (2) y (3): = 2a-bx-.lL
3 a+b Finalmente la suma de las cifras de 2 401 es:

= 2a-b P S=2+4+0+1=7
...(4)
a+b Clave: E

• La cantidad de papayas que aporta B para e será: 16. Datos:


DB=b-N
Xs =4.5
De (1): =b_a+b
3 20.25+x¡
xk + 1= --;:--"- ... (*)
=2b-a 2xk
... (5)
3
Reemplazando xj en (*):
• La cantidad de dinero que recibe B por su aporte:

PB = DBxPU 20.25 + (4.5)2


x6 = 2(4.5)
= 2b-ax-.lL
De (2) y (5):
3 a+b =4,5

= 2b-a P Reemplazando x6 en (*)


... (6)
a+b
20.25+(4.5)2
La repartición del dinero por las dos amigas será
x7 = 2(4.5)
PA y PB de (4) y (6).
= 4.5

Clave: O
XI04 = 4.5
Reemplazando xlO4 en (*): De la ecuación: y - x ~. O ...( l )

x-y2:0 ...(2)
20,25 + (4,5)2
xlO5 = 2(4,5) De (1) y (2): x=y ...(3)
=4,5
Reemplazando (3) en la ecuación:
Clave: A
0+0 = x2 +(x)2
17. Sea el número complejo en su forma polar
=> x=O (única solución)
z = r( cos9+isen9) ... (*)
La cantidad N de pares de números reales que se pueden
Donde: El módulo: Izl = r formar son:

El Argumento: A rg( z ) = 9 (x;y)=(O;O)

Datos: => N =l
Clave: B
... ( 1)
19. De la Ecuación:
Arg[z(1 +i)] =1 ...(2)
~1-9x2 =2x~I-9x2 ... (*)
De (1) obtenemos: Los valores que puede asumir x esta dado por la expre-
sión subradical:
Izil =4
Izllil=4 1-9x 2:02

Izlxl =4 9x2 -1 sO

Por propiedad:
Izl =4
Iz I= 4 ... (3)
x
2
-t ~O

(x+t)(x-t) sO
De (2) obtenemos:

rg[z(I+i)]= Í => XE [-t: tJ ... (1)

Arg(~)+ Arg(1 +i) = Í Resolviendo la ecuación (*)

1l =1l ~1_9x2 =2x~I-9x2


Arg(z) +
4 2
~1_9x2 -2x~I-9x2 =0
Arg(z)= ¡ ...(4)
~1-9x2 (1- 2x) = O
Luego en (*) , de (3) y (4): Analizando el primer factor:

~1-9x2=O

Clave: A
1-9x 2 =0
x2 _2- =0
9
18. Resolviendo la ecuación en los reales:
(x+t)(x-t) =O
UN12003-1 MATEMÁTICA 1:==~2ªª~~
=> xI =-t MATEMÁTICA PARTE 2
I
=> x2 =)
21 . En el conjunto:
El segundo factor:
F = {XE [O,21t]/cos2x secx+ secx+ 1= O}
1- 2x = O
XE [O,21t] ... (*)
[No válida por (l)!
En la expresión:
Finalmente, la ecuación tiene dos raíces:
COJ2x Jecx+Jecx+ 1= O

(co.I·2 x- .l"e/12x)Jecx+ .\"ecx+ I = O

Clave:C 2
[COJ x-(I-cOJ2 x)}ecx+Jecx+ 1= O

2
20. Escoger un grupo de k objetos de otro mayor 11 sin (2coJ x-I )Jecx+ Jecx+ I = O
importar el orden, es combinar este grupo de objetos.
2cOJ2 x Jecx- Jecx+ Jecx+ 1= O
El números total de formas de hacerla esta dado por:
2cOJ2 x(_l_)+ 1= O
e"= __ Il_!_ COJX
k (Il-k)!k! 2co.a+ 1= O
En este problema, la condición es que de las 10 pregun-
tas a resolver. se tomen 3 de las 5 primeras. Siendo un ~"01'.. ,
y =_12
total de 13 prccuntas.

ler. Caso: :1 preguntas de las 5, y 7 de las 8 restantes


=> x = 231t+ 2k1t ; 4; + 2k1t

NI=ejxe~
k = 1; 2; 3; ...
Por condición ("):
2do. Caso: 5 preguntas de las 5, y 6 de las 8 restantes
x= 21t . 41t
N2 =e~xc~ 3 ' 3
3er. Caso: 5 preguntas de las 5, y 5 de las 8 restantes Finalmente, el conjunto F tiene 2 elementos

N2=e~xe~
Finalmente, el número total de combinaciones posibles sería:
Clave: B
N=NI+N2+NJ

8 5 8 8 22. Si tg x+ctg x= 2 ." (1)


= (eS3 xe )+(e
7 4 xe )+(d
6 ) xe )S
¡-lglI X+«:I,::II .r
= (1Oxt!)+ (5x28)+(1 x56) r-------F
E = Jtg" x + ctg " X ... (2)
=276
'Clave:C De (1):
tgx+_I_=2
tg x
tg2 x- 2tgx+ 1= O
(lgx-I)2=O

tg x » I} ... (3)
ctgx= I
Reemplazando (3) en (2): 25. Se dan los lados del triángulo:
a=.J2
b=J6
c=,¡s

Aplicando la propiedad de Pitágoras a los lados:


Finalmente:

8=8 [cumple!
Clave:B
Se trata de un triángulo rectángulo:

23. En la ecuación:

ctg1=senx+clgx x # krr.

ctg x+cscx = sen.x+ ctg x


ese.\" = sen x ~
l· c=V8 ·1-
-I-=senx
sen x La altura menor corresponde a la del lado mayor. La
_1_-sellx=O que se obtiene aplicando propiedades de relaciones mé-
sen x tricas en los triángulos rectángulos.
sen2x-I=O c.h = ab
senx = ±I ,¡s_h= .J2_J6
De donde:
h = .J2_J6'
x = 2I( 2k + 1) k E Z ,¡s
2
J(,
Clave: A
2
24. Reduciendo mediante identidades uigonométricas Clave: B

- 26. Graficando según el enunciado:

Clave: D
UN12003-1 MATEMÁTICA

MNPQ: Paralelogramo. 28_ Graficando:


A.B.e: Puntos de Tangencia.

El triángulo OSR es triángulo isósceles, entonces la


cuerda:

RS = 8 u + 8 u = J 6 u
Clave: C. e

27. Graficando según el enunciado:


Dato: a+ 13= 2700

Propiedad:

a+13+9= 3600
270 + 9 = 3600
0

9=900
Graficando con estos datos:

Por propiedad los puntos: 01 ; T Y 02 son colineales, A 1_. 24_m _


luego:
Baricentro: G
En el triángulo S0201 :
Circunscentro: °
°2S = J(1+xf -(1- t x
Por propiedad en un triángulo
l. El baricentro se encuentra
rectángulo
sobre
se cumple:
el punto medio
= .,¡¡¡¡; de la hipotenusa (O)

2. AO=OB
En el triángulo 002S :

-2 -2 -2
Además en todo triángulo el baricentro divide a las •
002 = OS2 +OS2 medianas en dos partes, cuya relación en sus dimensio-
nes es de 2 al.
(R_x)2 =x2 + (rz¡¡;)2
Aplicando la propiedad (2):
R2-2Rx+x2 =x2+2Rx
AO=OB
x=.B...
24m = x+2x
4
x=8m
Clave: C
La distancia del baricentro (C) al circunceruro (O) es
de 8 lit.
Clave: B
29. Si intersectamos dos a dos las tres rectas obten- El área sombreada tiene cuatro partes iguales, entonces
dremos el siguiente gráfico: primero sumamos una de ellas:

." (1)

Del gráfico:

Por Simetría: L; = L; =}R ... (2)

- 1t
f..-:¡ =Ó·R ... (3)

Reemplazando (2) y (3) en (1):

P¡ =(}R)+(1R)+(~R)
Los puntos que equidistan de las rectas son los tres ex-
centros y el incentro del triángulo que se forma:

Luego al perímetro p¡ lo multiplicamos por cuatro (4):

2p=4(P¡)

=4e:R)
= .!im.
R
3
Clave: O

31. En el gráfico trazamos:

NE//DC

Luego tenemos 4 puntos del plano que equidistan de las


tres rectas. NF-----~

Clave: O

30. Analizando el Gnifico:


DL-----------------~
Dato:

AOABCD = 120 u
Del gráfico:

AP=!J
TI
"3 PC=3h
~-~~-.....::...L....~
Los segmentos A P Y PC guardan una relación de l a
3. Por propiedad las áreas de los triángulos AM P Y ¡J¡\lIC
también se guardaran la misma relación, es decir:

A"AMP=S

A"PMC =3S
Además se observa que área del triángulo AMe es la
cuarta parte del área del paralelogramo:
El área comprendida entre e 1 y e2 :
A - AOABCD A = nr}: -1tI.?
AAMC - 4 2
= n( 5 u t -n(3
?
u)
120 u
= 167tu2
4
=30 u Clave:C
Del gráfico: 33. Por propiedad se sabe que en todo triedro se cum-
AAAMC =4S ple:
30 u =4S
~ 3S =22,5 u

Clave: O
7t < a + 13+ 8 < 3n

32. Ordenando la ecuación que representa la curva el

el: x2+ y2+4x-6y+4=O En el tetraedro:


(x+2)2+(y_3)2 =32 Q

La curva eI se trata de una circunferencia de radio


/"1 = 3( u) y COIl centro en (-2 ; 3). La otra circunfe-
rencia e 2 • debe tener el mismo centro puesto que son
cuncéntricas.

Además se sabe que (2; 6) E e2


Graficando: N
y M

P
En el ángulo Q 7t < a + 13+ e < 3n
En el ángulo M n < a + 8 + A < 31t
En el ángulo P 1t < 8 + A + Y < 31t
x
En el ángulo N 1t < 13+ (i + y < 3n

Sumando: 41t < 2a + 213+ 28 + 28 + 2A.+ 2y < I 2n


Cálculo del radio de la circunf~rencia e2 : 2n < a + 13+ 8 + (i + A.+ Y < 61t

'2 =J[2-(-2)f+(6-3)2 27t < S < 67t

= .14 +3
2 2
~ SE (2n; 67t)
=5 (11) Clave: E
:.:
UNI2003-1 MATEMÁTICA
~.~
GomeZ

34. Graficando el tetraedro regular, cuyas aristas de- Graficando:


ben ser iguales: s
D

AL_--;?'-~;:::::--/B
B
A

C Altura de la Pirámide: h

Área de la Base : A,,(~ABC)


Tetraedro regular:
Inradio de la Base :r
AC = BC = CD = DA = a
Semi perímetro de la base:
Por dato:

OH =OD=x P = a+b+c
2
Del gráfico: 120m+39m+ 111 /Il
OA =a-x 2

Por propiedad: = 13511I ... (1)

DG =h = a-/6 Cálculo de A" por el teorema de Herón:


3
De (1):
Por semejanza de triángulos:

óAHO-óAGD A" =-fp(p-a)(p-b)(p-c)


DG DA
=JI35m(135111-120m)(135m-39m)(135111-1111II)
OH = OA
aJó a = 216011I2 ... (2)
-3-=~
Además por definición:
~ x=a(Jó-2)
A" = px r

Clave: B De (1) ) \.2): 2160/112 = 135111xr


35. Datos:
~ r = 1611I ... (3)
Base: lado: AB =-a = 120m Por Propiedades:
lado: BC=b=39m
AD=p-BC
lado: AC = C = I11 m
= 135m-39m
Arista lateral: SA = 4Ji2i7 =96111 ... (4)
Cálculo de A O , triángulo rectángulo AOO : Cálcu lo de Ir

2
AO=A0 +r2 h=~r2_(~t
De (3) Y (4): = (96m)2 + (16m)2
.. (2)
=9472m2 ... (S)

En el triángulo rectángulo AOS Volumen generada por SI:

A02=AS2_h2 Vcono = t(7th2)(Í)


De (5): 9472m2 =(4.J1217m)2 _h2

h2 = 194721112-9472m2
= [1(ÍJ3f](Í)
~ h= 100m .... (6) 1tr3
=-8- ... (3)
Cálculo del Volumen de la pirámide:
Volumen generado por S2 :
De (2) Y (6):
V = A"xh
3
Vcilindro = (7th2 )r
(2160m2)x(IOOm)
- 3 = 7t(~J3t r
= 72000m-3
=lnr3 ...(4)
Clave: A 4
36. En la figura: Volumen de la esfera de radio 1:
B
r V 4 nr3
R,'
1
esfera = 3 ... (S)
1
1 Reemplazando '(3) , (4) Y (6) en (1):
52 " 1
V - 4 m .3 2[1t/.3 ] 3 3
1
solido -3 - 8" -¡nr
,L---~WL----~/----~~----~D
m.3
Al girar la semicircunferencia de radio R sobre el eje =-3-
se AO genera una esfera de radio R. Clave: O
Al girar las superficie S, el eje A O genera dos conos. Al

girar S 2 genera un cilindro: 37. Dato: cos 1996 ° = =sentx ... (1)

El volumen del sólido pedido estará dado por: Re~uciendo al primer cuadrante c;os(1996°)

Vsolido = Yesrem - 2Vcono - Vcilindro ... (1) c.:os(1996°) = c;os(Sx3600 + 196°)


En el gráfico: = c;os(196°)

= c;os(1800 + 16°)

=-c;os(16°) ... (11)

Igualando [ y 11:
-sena. = -c;o.d 6
senc: = sen74°
:.s.:
0~;;~;;;;;~=-
_.....;U_N...;12;;.;O.;.03;;..- ..1 M...;A;.;.T;..;;E_M_Á_T...;IC...;A--il g¡1lQ
GO'iñez

~ a ==.74°
39. En la función: F(x)== Senx+cosx
~ 15a == I l l 0° l-sel1x+cosx
Los puntos de discontinuidad de la función F , son los
Cálculo de E: E == cscl5a - senl5a valores que adquiere la variable x, que hacen indetermi-
== cscl IOo-senl 10° nada la expresión, es decir si:
== ese 30° - sen30° I-senx+cosx == O
== 2 -0,5 UI1X-COSX == I
== 1,5
hsell(x-¡)== I
Clave: B

38. Dato:
sen(x-¡)== 1
Formas generales:
Ig8° == 1 ... (1)
7 x-lI == 'Ikt: + lE. ~ x == (4k + 1)1I. (1)
4 4 2 ...

En el gráfico trazamos EG y F H x _lE. == Zkt: + 31t ~ X == (2k + 1)1t (2)


4 4 ...
Convenientemente. AF es diagonal del cuadrado De (1) Y (2), F(x) es discontinuo en:
AGFH, entonces:
XE {(4k+I)-i}U{(2k+I)1t}
m4:GFA == 45°
Clave: C
B e
40. En la función:

x
x
E
F(x)==
.J 1+ sen x=I Lcos"
?
x
; XE (0;1t)
... (1)
F
Para que F (x) exista debe cumplirse:

1+ senx= 2cos2 x> O


8-x Resolviendo:

(2senx-I)(senx+ 1) > O

De (1): (senx+I»O
D
Del gráfico: ~ 2sellx-1 >0

6==53°-45° senx > ~


== 8° ... (2)
En la circunferencia trigonométrica
Además:
f(x)=senx
16== __ x__
g 8 cm+x

Ig80 == __ x__
De (2):
8cm-x

De (1):
I x 11

7 8 cm+x o 1L 7t
6 '2
~ x == I cm
Luego: XE (~ ; 561t)
Clave: A
Clave: D
UN12003-11 MATEMÁTICA I=::::;'~

MATEMATlGA .r(F2)X2 + 3f(8)


Si ( )
g x = x+fUU(23)))
1. En una caja eI hay 5 bolas negras y en otra caja
Entonces j(g(4)) ~s igual a:
e 2 hay 8 bolas blancas. Se escogen 3 bolas de e I y se
A) O B) I C) 11 D).!l E) 3
colocan en e2. Luego, al azar, se escogen 3 bolas de 7 5
e2 y se las coloca en el' Después de este procedi- 5. La población de peces en un estanque aumenta a
razón lid 20% anual. Al final del segundo año se tiene
miento, sea b el número de bolas blancas-en eI y n el
una población de P2 peces. Al final del tercer año, la
número de bolas negras en e2 . Entonces:
población P3 se ajusta a la siguiente proporción
A)b = n- 2 D)b = n-I
P2 = P3 . Si la población inicial Po fue 200 peces.
B)b=n+2 E)b = n 3 4,5
entonces P3 es:
C)b = n +l
A) 330 Bl 360 q 420 D) 430 E) 432
2. El conjunto de soluciones del siguiente sistema:
6. Tres peatones cruzan un puente en 10 minutos, 4
x2 + y2 = r2 minutos y 6 minutos respectivamente, dando pasos de
{ x-y=r distinta longitud. Si estas longitudes son efectuadas en
tiempo iguales, dar la razón de la longitud de cada paso
Para r > O , es:
del peatón más veloz con la suma de las longitudes de
A) 0 los otros dos peatones.
B) Un conjunto unitario.
C) Un conjunto con dos elementos. qU D) JJi El .!Ji
16 15 13
D) Un conjunto con tres elementos.
E) Un conjunto con cuatro elementos. 7. Hallar la suma de los elementos del conjunto

3. Sean las matrices {(-~)a (a+l) ; tal que "a es entero pOSitiVO}

A =[:~] y B =[: :] tales que A)1148 B)1224 C)1248 D)1272 E)1278

8. Un número n es múltiplo de 3. Entonces podemos


AB=[~ ~]
afirmar que el residuo de dividir:
Entonces el valor de a + b + e + des:
231l+5 + 2511+4 + 25
A) -1 B) O q -2 D) 1 E) 2

entre 7 es:
4. Sea la función J: (1 ; 00) -7 N , tal que J(x) es el
A) 6 B) 5 C)4 D)3 El2
número de primos menores o iguales a x.
9. La suma:
14. Un poblado de América del Sur tenía problemas
5= -0,12 + 0,23 -0,34 +0,45 -0,56 +0,67 con el agua potable, por contener altos niveles de arsé-
expresada como una fracción de números en base 8, es nico, por lo cual, a un instituto de protección ambiental
igual a: se le encargó investigar y proporcionar un tratamiento
que removiera la mayor cantidad de arsénico del agua.
1018 En la tabla se representan los resultados obtenidos para
A)0,2318 B)~
e) 644 el porcentaje de remoción de arsénico en 60 muestras
4208 8
de aguas tratadas con cloruro de aluminio.
D) 145R E) 1458
4208. 6448 % de remoción Números de muestras

[50 - 60) 9
10. Hallar el menor entero positivo 11 tal que las 73 [60 - 80) 16
fracciones
[80 - 90) 15
19 20 21 . 91 [90 - 95) 20
11+21 ' 11+22 ' n+23'" 'n+93
Proporcione el número aproximado de muestras en el
sean todas irreducibles.
que se ha obtenido al menos el 75% de remoción de
A) 93 B) 95 C) 97 D) 10.1 E) 103 arsénico.

11. Con cierto tipo de papel se elabora un libro de 1000 A) 39 B) 25 C) 21 D) 16 E) 12

páginas cuyo espesor (sin contar las tapas) es de 1 UI1.


Suponga que una hoja muy grande de dicho tipo de 15. Sea 1.. < 1< -1 donde {[ y b son números reales,
(/ h
papel es doblada en 2 (con lo cual su área se reduce a la
entonces dadas las proposiciones
mitad); es vuelta a doblar, y así sucesivamente es dobla-
da 50 veces, adquiriendo un espesor total E.
1) (a+I)" > (b+I)2
El valor aproximado de E es:

A)2c/JI B) 10m

D) 5x 105 km E)2.25xI07 km
Sor ciertas:
12. Se-desea construir un ferrocarril sobre una montaña.
Desde el pie hasta la cima, se necesita haccrlo subir 600 A) I y 11 B)lIylll e) 1 y 111
metros. ¿En cuánto aumentaría el trayecto a recorrer si se D) 1, 11 Y 111 E) Solo 11
requiere reducir la pendiente de 4% al 2%? (en km).
A) 9 B) II e) 13 D) 15 E)17 16. Se desea fabricar una caja de base cuadrada y sin
tapa, con una hoja cuadrada de plata pura de lado x,
13. Tres jovenes A, B y e
deciden realizar UI1 viaje a
cortando cuadrados de lado Jl. en cada esquina y do-
Tacna. NN tacneño, se une al grupo pero no tiene dine-
blando los lados. El rango en que debe estar x para que,
ro para viajar. A, ¡¡ Y e hacen un "pozo común" para
numé: ,,,dllenle, el volumen sea mayor que el área total
pagar el pasaje de lox cuatro que cuesta SI. 400, A aporta
de la caja es:
SI. 180, B aporta SI. 120 Y e aporta SI. 100. En tacna
el papá de NN obsequia a los 3 jóvenes 20 botellas de
miel de abejas para que se lo distribuyan proporcional- A) (o; 2fI(t~D) D) (21'; 2{(~~ :))
mente a lo aportado por cada uno al viaje de su hijo.
Entonces la cantidad de botellas que les toca A, B y e, B) (2e;oo! E) 21(i..±l) ; 00)
respectivamente, es: \ F-I

16,4,0 B)I::>~.O e)10,7,} e) (O;2e)


PI '1,6,5 E) <) -1
UN12003-11 MATEMÁTICA I:t..ª~~~~~:i~
17. Indique gráficamente todos los puntos del plano
E)
que verifican las relaciones:

kl:'>1 Y Izl:,>I
donde z = x+ iy

21. La figura muestra una disposición de equilibrio.


Determinar la distancia en metros entre el bloque y el
punto fijo.

A) 10,2

B) 12,4

el 14,2
D) 16,9

E) 18,7

Punto
18. Las soluciones reales de la ecuación fijo

log5( x2 - 20x) = 3 son:


22. Hallar el valor de E = fmax - fmill ; si
A) no existen B) únicamente x = 25
e) únicamente x =5 D)x¡=5;x2=25 J(x) = 2cosx(c:osx-senx)-1 ; XE [~ ; 5~J
E)x¡ =-5 ; x2 =25
A)-2J2 B) -1 C) 2 D)2J2 Ell
19. Se tiene cuatro números, tales que, los tres primeros
están en progresión geométrica y los tres últimos en pro-
gresión aritmética de razón seis; siendo el primer número 23. En la figura mostrada calcular /((11 <p .
igual al cuarto. La suma de los cuatro números es:
A) 22 B) 18 C) 14 D) 16 E) 20
A)l
2
3 B) 2
20. La gráfica de la desigualdad 1xl + 1yl < 4 es:
el 1-

A) A. A.'
~ x B) ~
2
2

VV 24. Dada la ecuación z 2 + (a + ib)z


E)

+ e + id = O
1
6

C)

J D)+2
-=4', ,
,
, '
',x
,,<4 -2 2
x
donde a, b, e, d son reales. Esta ecuación tiene una raíz
real, si sus coeficientes satisfacen la relación:

A) abd=c2+d2b D) abd=b2+d2c
, ,, -2 B) abd=d2+c2b E)abd=d2+b2c
-41
e) abd=b2+c2d
25. El valor de verdad de las siguientes proposiciones es: bisectriz.t Si HD= I u y DE=2u,hallarlalongituddel
1) Toda línea recta separa al plano que la contiene en segmento AH
dos conjuntos convexos.
A) 5 u B) 4 ti e) 3 ti D) 2 u E) 1 ti
11) Si le quitamos un punto a un plano, el conjunto
resultante es convexo. 29. Hallar la longitud del lado de un polígono regular
IlI) Toda poligonal no convexa que gira 3600 alrede- de 24 lados en función del radio R de la circunferencia
dor de uno de sus extremos y en el plano que la circunscrita a dicho polígono.
contiene, determina siempre una región convexa.

A) VVV B) VFV C) FVV D) VVF E) VFF


A) R~2 - ¡;;.;J3 D) R~2+~

B) R~ 2 + ..r;:;.J3
26. En la figura mostrada el punto ° es el ortocentro
E) R~2~2-J)

e 1es el incentro del !:!.ABC.Hallar la relación entre e) R~2-~2-J)


a.« y~. B 30. En la figura mostrada se tiene una

\ semicircunferencia de centro ° y otras tres

semicircunferencias del mismo radio -J6 y centros

0, 01 Y 02 _ El área de la región sornbreuda es:

AL.:...----!...----~C

A) ~ = 2a.- 8 D) 13 = a. + e
4
B) P=2(a.-8) E) ~ = a. - 8

e) p= a.; e
27. En la siguiente figura, si .JR + J;. = 10, enton-
ces BD + DE es: A) (37t- J)) cII/2 D) (47t+3J3)cII/2
A B) (57t+ J3) 01/2 E) (87t-J3)C/1/2
e) (77t-3J3)CII/2
31. Dada la ecuación de la par.ibo!a:

y 2 _ 4Y - 8x + 44 = O, entonces la suma de las coor-


denadn- d,' toco de la parábola es:

A)7 B) 8 e) 9 r» 1 () E¡ 11

32. En la figura mostrada se tiene un hexaedro regular


A)95u B)96ue)97u D)98u E)IOOu
en el que se han trazado los segmentos A e . B f) Y

28. En un triángulo ABCse traza la bisectriz A E que MN ,talqueMyNsonpuntosmediosde A13 y r:F


intercepta aliado BC en "D",luego desde los vértices respectivamente. Entonces podemos afirmar que la
B, C se trazan las perpendiculares BH , CE a dicha suma de los ángulos que forman al cruzarse en el espacio
UN12003-fI MATEMÁTICA 1=' ::;::::==0
¡

el volumen generado al rotar la región cuadrada en tor-


AG, BD Y MN tomados dos a dos, es igual a:
no del eje L, sea el mayor posible. (ver figura).
A) 15°
D) 600

C
A) 1200 B) 150 0
e) 2200 D) 1800 E) 1350

33. Una pirámide regular tiene como base un triángu-


lo equilátero dit lado" .e ". Si las caras laterales de la 38. En un cono circular recto se inscriben dos esferas
pirámide son perpendiculares entre sí, la altura de la tangentes exteriormente entre sí, cuyos radios son de
pirámide mide: - 3 cm y 5 cm respectivamente. La altura de dicho cono es:
A) 26 cm B)25 cm C)24cm
A) 1.
2
B) -k C) ~ D) 53 E) 1...
4 D) 23 cm E) 22 cm

39. Del gráfico: y


34. Tres planos tangentes a una misma esfera de radio
"R" son ortogonales entre sí y se intersectan en un punto
P. La distancia del punto P al centro de la esfera es:

x
A) .J3 R B) 2R C) J2R D) 3R

35. Se tiene un prisma hexagonal regular ABCDEF -


A' B' C' D' E' F' cuyos lados de la base y la altura miden Determinar:
a=b + sen a + sen. b
3 sen -3-
2a (a> O) , luego por las aristas AB y D' E' pasa un
K = ----"....,....-----
plano formando el sólido ABD 'E 'A 'B 'D ', Entonces el
6 cos a'6b +cos a+cos b
volumen del sólido formado es:

1.J3 a3 3 1 1 1 I I
A)
2
B) 2J3a A) -2 B) :-}' C) -4 D) 2' E)}'

D) 4.J3 a3 E) ª-.J3a3
3 40. Si f (x) =l- senl x] ' indicar verdadero (V) ó
falso (F) para las siguientes proposiciones:
36. La relación entre el volumen de un tronco de pirá-
mideregularcuadrangular,de áreas 4a 2 y 16a 2 (a> O) .1) f(x) es creciente en (~; 3;)
Yel volumen de una esfera inscrita es:

A)!l B).2. C) -ª- D) i. E) ~


I1) f(x) es decreciente en (_3; ;- t)
7t 7t 7t 7t 7
III) f(x) tiene como rango [0;2]

37. Determinar la medida del ángulo a de modo que A)VFF B)VFV C)VVF D)VVV E)FVV
MAftMA'l'ICA 3. Sean los matrices:
t. Del enunciado:
el e2
A=G ~] B=[: !] ... (1)

'-I:] G AB=[~ ~]
Ejecutamos AB de (1):
... (2)

""G
Jero: []JN
xN
-L
o
18B~YB I
AB=[:
a+h
~][: !]
= [ a+3c
b+d]
b+sd ... (3)

x +y 3N-xN
YB· lgual.indo los elementos de (2) y (3):
De donde: a+c=1 ... (4)

x+y=3 ... (1) h+d=O ... (5)

... (2)
a+3c=O ... (ó)

h+3d=1 ... (7)


e2: 3-x=n De (6) Y (4):
x=3-n ... (3)
a+3c = O
Reemplazando (2) Y (3) en (1): ((+e=1

(3-11)+&=3 2e ~-I ~ c=-- 1


2
&=11 De (7) Y (5):
Clave: E
b+3d = 1
2. Del sistema: b+d =0
x2+y2=r2 ... (1) 2d=1 ~
x-y=r ... (2) Reemplazando (4) y (5):
Donde: r > O
a+(-!) =I ~ a=l.
De (2) y =x - r ... (3) 2
De (3) en (1): x2 +(x_r)2 = r2 &+(!)=O ~
!J =_.L
2
2x2 -2rx = O Luego:
x(x-r)=O
~ xI =0

~ x2 = r
Clave: D
Clave: C
&I~-::;;; •. :W
G~nlri'.1 ¡Wiil&iiMLM_1
t··k·lHit"",* ~..
UN12003-11 MATEMÁTICA 1;
.
:,mm"::;;~1
K' -

4. La función f se define aSÍ: Primer año:

f(x) = {Cantidadde numeros primos n] n:S;x; XE (I;oo)} fj = Po + I~~PO = 1,2'Po .. , (2)


Veamos los primeros números primos:
Segundo año:
2 ; 3 ; 5 ; 7 ; 1I ; 13 ; 17 ; 19 ; 23 ; 29 ; ... ... (1)
20
P2 = fj + lOOP¡ = 1,2 fj
En la expresión:

f( J2)x2 + 3f(8)
... (2)
De (2): = 1,2(1,2 Po)

g(x) = x+f(f(f(23))) = l,44Po ... (3)


Considerando (1), debemos calcular:
Reemplazando (3) en (1):
f(J2) =O

f(8) = 4 1,44Po _ p]
-3-- 4,5
f(23) =8 ~ f(8) =4 ~ f(4) =2

~+(~l.
1,44( 200) p]
3 = 4,5
p] = 432
2 Clave: E
6. Sean A , B Y e los tres peatones.:

Reemplazando estos valores en (2): Por condición:

2
A con longitud de paso a tarda 10 mino
( )_(0)x +3(4)
B con longitud de paso b tarda 4 mino
g x - x+(2)
e con longitud de la paso e tarda 6 mino
=--'.L
x+2
Observe que el peatón que dé el pasó más largo emplerá
menos tiempo, lo contrario al peatón que dé el paso
12
~ g(4)= (4)+2 menos largo empleará mas tiempo. De esto se deduce
que las distancias de cada paso de los peatones son
=2 inversamente a sus tiempos que emplean en cruzar el
Finalmente: presente, es decir.
De (1): f(g(4» = f(2) lIxI0=bx4=cx6
=1 axlO = bx4 = cx6
Clave: B 60 60 60
a b c ... (*)
5. Datos: 6=T5=iQ
Razón de aumento de los dos primeros años: 20%
El peatón más veloz será el que empleo menos tiempo,
Población inicial de peces: Po = 200
es decir B que tarda 4 minutos:
Población de peces al finalizar el 2do. año P2 Por propiedad:
Población de peces al final el 3er. año p]
De (*): 2-=lI+c
15 6+LO
Por condición del problema:
_b_=12
P2 _ P1 a+c 16
3-4,5 ...(1)
Con los primeros años los peces aumentan con una
Clave:C
razón de 20%.
9. En la suma:
7. La expresión (1) a (a + 1) es un entero positivo,
S =-0,1(2) +0,2(3) -0,3(4) +0,4(5) -0,5(6) +0,6(7)
entonces "a" es un entero positivo.
Como la expresión esta en base lO, se cumple: =-C~)(2) +C~)(3) -C~)(4)+(16)(5) -CSO)(6)+C~)(7)
a < 10 •. a E Z ... (1) = -.1.+ 1_1+1_1+ 2.
2 3 456 7
Q. < 10 ~ a = 2'4'6'8 -210+ 280-315+336-350+360
2. ' , ,
420
El conjunto de números que se pueden formar son:

A= (1~(a+I)/a=2;4;6;81
= {123; 245 ;367 ;489}
Clave: E
La suma de estos elementos:
10. Las fracciones
s = 123 + 245 + 367 + 489
19 . 20 . 21. 91
= 1224 11+21 ' 11+22 ' 23 '· .. 11+93
Clave: B
En general se pueden expresarse así:

8. Si un número "11" es múltiplo de 3, entonces: 18+k k = 1; 2; 3; ...73


11+20+k '
11= 3k ; k eZ ... (*)
Donde: 18+ k Y 11+ 20+ k deben ser PESI
Si dividimos a la expresión entre 7 obtendremos:

2311
+5 + 2'"+4 + 2' = ex7 + R Propiedad: I! + 20 + k - (18 + k ) = 11+ 2
Donde: (18+k) y (11+2) sonPESI
23".25 + 251/.24+ 25 = 7+ R
Es decir:
De (*): 19 Y 11 + 2 son PESI

20 Y II'¡' 2 son PESI


21 y 11 + 2 son PESI

91 Y 11+ 2 son PESI

~ 19x20x2Ix ...91 y 11+2 son PESI


Luego 11 + 2 es número primo mayor que 91.

o o
7+IO=7+R ~ 11+ 2 = 97
o o 11 = 95
7+3=7+R
Clave' B
3=R
~ R=3
Clave: O
UNI2003-11 MATEMÁTICA 1:::::· "'===;:0
11. Dato: Del gráfico:
19~ = J!...
Espesor de 1000 (páginas): E = l cm. = 10 -5 km. d2
ler, dobles: 2 hojas = 2' hojas De (3): 002 = 600111
, d
2do. dobles: 2 x 2 hojas = 22 hojas 2
=> d2 = 30 0001/1 ... (4)
3er. dobles: 2 x 2 x 2 hojas = 2 J hojas
Nota: Para ángulos muy pequeños, en un triángulo
SOavo.dobles: 2x2x2x ... x2 hojas = 250 hojas rectángulo, la hipotenusa se puede considerar
igual al cateto adyacente.
Aplicando regla de tres:

SOO hojas ~ 1 cm. I ~b si a~O => c=(/


a
250. hojas ~ x cm
Tomando en cuenta el criterio anterior como e y ~
50
2 hojas x l cm son pequeños, entonces:
=> x = --;:;;~''-:--:--
SOO hojas T, =d ,
T2 = d2
=2,2SxlO'2cm
Luego:
= 2,2Sx 107 km
T2 - T, = 30 000 m - IS 000 m
Clave: E = IS 0001/1
=ISkm.

12. Dato: Altura de montaña: h = 600 m. Clave: O

• Pendiente del 4% tg e = 0,04 ... (1) 13. Aporte de los tres jóvenes:

Pasaje Para NN Total


Joven A 100 80 180
Joven B 100 20 120
Joven e 100 O 100

Del gráfico: El aporte para el pasaje de "NN" es solamente de parte


tge =J!... del joven A y B, en la proporción:
d,
... (1)
De (1): 004 = 600m
, d,
Las 20 botellas de miel de abeja también deben repartirse
=> d , = IS 000 m ... (2) solamente entre los jovenes A y B; es decir:

' .. (2)
• Pendiente del 2% : tg~ = 0,02 ... (3)

Por condición del problema, la repartición de botellas


de miel deben ser proporcionales al aporte del dine,:Ó
para el pasaje de NN, es decir en la proporción (1):
bA _ bB
-¡--T
0===:====J__U_N_I_20_0_3_-_II_M_A_T_E_M_A_'_T_IC_A_I=~7i:~=:r::=::: e: 1

Propiedad de proporciones:
bA bB bA +bB 15. Si . (1)
-¡-=[=4+I
=) a.b < O ... (2)
=) b = 4(bA +bB) Invirtiendo en (1) Y tomando encuenta (2):
A
5
O>a>b>-I
De (2): _ 4(20)
--5- -1<b<a<O .. (3)
Luego las alternativas:
=16
1) (a+I)2 > (b+I)2 es ¡Verdadera! (V)
bB_bA+bB
=) T---5- De (3): -1<b<a<O
20 -1+1 <b+1 <a+1 <0+1
5 O<b+l<a+I<1
=4

Clave: A
II) a2 > bJ es [Verdadero! (V)
14. El número aproximado de muestras en donde se
De (3): -1</)<a<O
ha obtenido al menos 75% de remoción de Arsénico
será (Tabla): =) bJ -7 (-)

No. de muestras ~ 75% =) b2 -7 (+)


% de remoción Muestras Luego: b2 > bJ
(90-95) 20 I1I) aJ-bJ>O es Verdadero (V)

(80-90) 15 De (3): -1</)<a<O


(60-80) x ;0 <x<16 173 < {l3
0«,,-/)3
Total de muestras: T = 20 + 15 + x ... (*)
a3 _/)3 > O
Interpelando las 16 muestras en el intervalo (60 -80) Clave: D

15 5
/r ~A~ -,\~ 16. Del enunciado:

I I
60 75 80 1-' --r--x~ --1--' 2-C 1

x
~
I I :e, e' ,: L~
o 16
,
. -[", ':::L _
Luego:
.:!: = l.§.
5 20
e
e 1
x - 2C
x

x=4

Finalmente en (*): e
----T:.' , ~-----
e j
T = 20 + 15 + (4) , ,
= 39 'e,
,
e:,
Clave: A h , ,
La caja formada será: Graficando:

e x

x-u
El área total de la caja:

A = (X-U)2 +4[(x-U)e]
11. Operando en la expresión:
El volumen y área total de la caja serán:
Izl::; 1 ... (4)
V=(X-2e)2xe Por definición:
Por condición del problema:
z=x+iy
V>A
[z] =~x2 + l ... (5)
(x - 2f.l
e> (x - ul + 4[(x - u)e]
(x- u)e > (x- U)+ 4[e] Reemplazando (5) en (4):
xe-2e2 >x+u
U(f+ 1) ~x2+ i sI
x> f-I 2
x + l::;1 ... (6)

~ XE
(2f.ce
(-1
+ 1) . 00)
'
Graficando:
y
Clave: E

17. La gráfica de:


x
lell::; I
'-v---'
-1

(1)

I. Operando en la expresión: -1

le z 1::; I •... (1)


Finalmente, la gráfica de la expresión dada se obtiene
Por definición: Z
e =eX(cos6+isen6) intersectando los gráficos de 1 y 11.

y
~ lezl=IeX(co,\"6+iseIl6~
= eXlcos 6 + i sen el
= eX ~( cos 6)2 + (sell 6)2
x
= eX.JI -1
x ... (2)
=e
-1
Reemplazando (2) en (1) :

eX::; I Clave: D
eX::; e" ~ x::;O ... (3)
UNI2003-11 MATEMÁTICA ~:i. ==" :.1':' ====w:~~...
18. En la ecuación: Gráfica: y
4,
[ogs(x2 -20x) = 3 ,,
,,
Por definición: , x
x2 -20x = S'
x2 -20x-J25 =O
(x+5)(x- 25) = O -4

~ x+5=O • Si x <O
. En (1): -x+lyl <O
~ x-25 =0
Clave: E
x<lyl-4
Gráfica:
19.De los cuatro números, los tres primeros forman
una progresión geornétrica y los tres últimos forman
una progresión aritmética. Además el primero número
y el último son iguales:
Progresión geométrica
,---------A---.
x+12 ; x ; x+r ; x+2r
'--v------'"
Progresión aritmética

Dato: r =6
Por propiedad, en una progresión geornétrica se cum- Finalmente. uniendo los dos gráficas:
ple que el producto de los medios es igual al producto de
los extremos: y
(x+ 2 r)(x+ r) = x2 4
,,
x2 + 3rx + 2,.2 = x2 ,,, ,,,
2r ,, ,,
x=-3 x

2(6)
r=6:
3
=-4
Los cuatro números serán:
x=8 Clave: E
x+6=-4
x+ 12 = 2 21. La disposición de equilibrio queda así:
x=8
A
La suma:

s = (8)+ (-4) + (2)+ (8) = 14


Clave: C

20. En la desigualdad:
Ixl+lyl < 4 ... (1)
• Si x 2: () :
En t l ) x+lyl<4 p
x<-1 -1,.1
Por la ley de cosenos: Luego considerando (2):
y
BP = ~ AB2 + Ap2 - 2(AB)(AP) cos 30°

= (IOm)2+(20m)2-2(IOm)(20m~

= 10-/5-2J3 m
x
= l2.4m
Clave: B

22. Dato: XE [1 ; 5gn:] ... (1)

Propiedades a aplicarse:
-h:'> cos(2x+¡):'> O

• cos2a=cos2a-sen2a (Pl) ::! :'>vlzcos(2x+¡):'> 9


f(x)l1lin f(x)",,,x
• un 2a = 2sena cosa (P2)
• cos( 2a+ ¡)= cos z«. cos¡ - sen2a sen¡ Finalmente: E = f(x)max-f(x)mill
=0-(-1)
= cos2a( -k )-sen2a( -k) =1
Clave: E
= -k(cos2a-sen2a) ... (P3) 23. Completando el gráfico:
En la función:
f(x) = 2cosx(coJX - senx)-I
= 2cos2 x- Zcos x senx-l
= 2cos2 x - Zcos x senx - (sell2 x + cos2 x)

= (FOS
2
x;;1 sen2 ~ J-[ ~cos ;;ten~ 1
= cos2x -
'-v--'
sen'Lx
l.
P3
Del gráfico:
= J2 cos(2x+¡) ... (2)
tg$ = 4 ... (1)
6
Operando en (1):
tga=~ ...(2) •
~ < x < 5n: d
2 - - g
Ig($+a) =.1 ... (3)
n: < 2x < 5n: d
- - 4 Por trigonometría:
5n: < 2x+~ < 3n:
4 - 4 - 2 Ig($+a)= Ig$+/¡;a
l-tg$xlga
.Q.+J....
Entonces: cose:)=--k De (1) y (2): 6 el

cose;) =o
1-(%)(*)
... (4)
UN12003-11MATEMÁTICA I=~::===::::::=;¡~
--===-- z
Igualando (3) y (4):
2
3 d +6
d =sr
~ d=±3
Reemplazando +3 en (1):
Observe que por lo menos un segmento de red a
Igcj> = (3) une a dos puntos del conjunto que no está conte-
6 nido en el plano. Falso (F).
I
. Hl) "Toda poligonal no convexa que gira 360 alrede-
0

2
Clave: A dor de uno de sus extremos y en el plano que la
contiene, determina siempre una región convexa".
24. En la ecuación:

z2 +(a+ib)z+c:+id =0
a , b , c: , d son constantes reales
z es una variable real
i es el número imaginario Al girar 360 la poligonal alrededor de uno de sus
0

Operando la ecuación: extremos genera un círculo, una región poligonal.


Verdadero (V).
z2+az+ibz+c:+id =0 Clave: B
z2+az+c:+i(bz+d) =0
'----v-----' '--v-' 26. Completando el gráfico:
Real Imaginario
Como la expresión está igualada a cero (O), debe cumplirse:
Parte imaginaria: b: + d = O

z = -¡;d ...(*)
Parte. Real: Z2+ClZ+C=0
De(*): (-t/ +a(-t)+c:=O
A~~----~----~~~B
d -abd +b2c: = O
2

¡} o : Ortocentro, concurren las alturas.


1 : Incentro, concurren las bisectrices.
Clave: E Por propiedad:
2a+2b+2c: = 180 0

25. En las proposiciones: a+b+c = 90 0 ... (*)

1) "Toda línea recta separa al plano que la contiene en Por propiedad de triángulos rectángulos:
dos conjuntos convexos". I:!.AOB: (a+a)+2h=90° ... (1)

I:!.APB: (p+h)+2a =90 0


... (2)

I:!.CRB: (6+c)+2h=90° ... (3)

(á+P+ 6)+ (a +b + c)+ 2(2b + a) = 270 0

De (*): a+p+6+(900)+2(2h+a) = 27{)O


Los semi planos son conjuntos convexos: Verda-
a+p+6+2(2h +a) = 180°
dero (V)
De (1): (x+p+8+2(90-a) = 18{}O
11) "Si le quitamos un punto a un plano el conjunto
resultante es convexo". p=a-8
Clave: E
UN12003-11 MATEMÁTICA 1;:::::::::::==0
27. Dato: JR + J;. = 10 El triángulo CHD es semejante al triángulo BED

A ... (1)

También se tiene que el triángulo AHC es semejante


al triángulo AEB

BE = d+HD+DE
CH el
BE = d+lu+2u
CH el
d +3/1
--l-'- ... (2)
Del gráfico:
Igualando (1) Y (2):
FB = R - r ... (1)
d +3/1 2
Como C E // H D ,entonces: -(-'-=1
d = 3u
HD=CE ... (2)
Clave:C
En el triángulo rectángulo FCE
29. El lado de un polígono regular de 24 lados subtiendc
CE = -1 FE2 - FC2
una arco de:
- 360°
De (1): =J(R+r)2-(R+r)2 AB=24""

=2JF& = 15°
Graficando:
Finalmente del gráfico:

~[
BD+ DE = (BH +CE)+ DE

=R+2JF&+r

= (JR+J;.)2 o
= (10)2
= 100 B
Clave: E
28. Del enunciado: o : Centro de la circunferencia.
R : Radio de la circunferencia.
El lado AB se puede calcular por la ley de cosenos, para
lo cual necesitamos conocer "coseno 15"".

300) 1+ CIII" 30°


C~=-~~--------~~B (
cO.l"T = 2

~I+ J3
co.\·15°= ~

= -I2+J3 ... (1)


2
Del gráfico: CH // EB Cálculo del lado A 8:
r.; 1O~O~J4:!I:.==m:&:,.:..:':r:w:';¡.:',,:••:m:,'":;I__ U_N_1 2_0_0_3_-_II_M_A_T_E_M_A_'
T_IC_A I'='@. Wu:::;:.:=!:m~~~~
Finalmente, reemplazando (2) y (3) en (1):
AS = ~ R2 + R2 - 2R . R cos 15°
= R.,j2 - 2 cos 15° ASombreada = (121t)-(5rr+3../3)
= 7rr-313 (cm)
De (1):
=R~2-{~) Clave: C

= R~2-.,j2+../3
31.En una parábola, por definición:
Clave: A
y2 = 4Px ... (1)

30. El área de la región sombreada se obtiene: I Y


I
I

. .ASombreada = ASemicircllJo- A x ... (1) I

I P
x

En la ecuación del problema:


Datos: r =,/6 (cm)
l-4y-8x+44 =O
R = 2r = 2,/6 (cm)
... (2)
(;'_2)2 =8(x-5)
Cálculo del área del semicírculo:
'y
rrR2
ASemicircllJo = 2

P
2~--~~==~----
... (2)
Cálcúlo del área Ax :

Comparando (1) Y (2):


4P = 8 =} P = 2
Luego las coordenadas del foco:
F = (5+2; 2)
= (7 ; 2)
La suma de las coordenadas:
S=7+2=9
Clave: C

32. En el gráfico:

Á----:.r----"F
Luego:
Ax = 2AJ + A2 + 2A)

= 2(21t) + (n) + 2( 3f!) e


= 51t+ 3../3 ... (3)
.=:===&:¡¡¡:d=&!ll'
G~~Z == im:~=. =:=~U~N~I~2@0~03~-li"~M~A~T~E~M~Á~T~IC~A~):=======:~101 <'\

Observamos que: Del gráfico:


AG// DF MN// BF AP =..fi f
... (1)
2 .
Luego podemos trasladar estos segmentos, así:
Por condición el triángulo ABC es equilátero, entonces:
AH =.J3 f
2 . ... (2)

El punto G es baricentro del triángulo ABC,luego:


AG = GH
2 I
Por propiedad de proporciones:
A '-- __ ---,.Y AG = GH = AG+GH
2 I 2+ I
Como los tres segmentos son diagonales de cuadrados => AG = 2(AG + GH)
de las mismas dimensiones, se habrá formado un trián- 3
gulo equilátero, 2(AH)
GN =-3-

De (2):
_2(1 f)
- 3
.J3.
=)e ... (3)
A B Cálculo de h en el triángulo rectángulo APG .
Entonces:
Ángulo for mado por AG y MN: 60° h=~Ap2_AG2

Ángulo formado por AG y AM: 60°


Ángulo formado por AM y MN: 60°
La suma total será:

Clave: B
Clave: O
33. Del enunciado:
P 34. Graficando según el enunciado:

A~~---~--~~C
Las caras laterales de la pirámide son triángulos rectán-
gulo isósceles.
P P' P"

~~~
A B C A'
En el triángulo rectángulo APC
Vplisma = base x altura
PC=JAP2+AC2
= [~(A'E'X AA')JX(AS)
=JR2+R2
De (*): =[~(2aJ3x2a)JX(2a)
=.fiR ... (1)
= 4J3a
3

En el triángulo rectángulo PCO Clave: D

PO = J PC2 +C02
36. Del enunciado:
=~(FR)2 +R2
=RJ3
Clave: A

35. Del enunciado:

En el cuadrilátero ASCO, aplicando semejanza de


triárngulos:

Como la base es un exágono regular, entonces el trián-


gulo O'O'E' es un triángulo equilátero.
En el triángulo rectángulo O' O'T'

h= J(2a)2 _a2
=aJ3
Del gráfico tambien se deduce:
A'E' = 2h
= 2(aJ3) .!!:.-=Ji.
=2J3a ... (*)
R 2a
R2 = 2a2
El sólido pedido es el prisma recto cuya base es el R = a.fi ... (1)
triángulo A E'A' :
~::::;:t¡:~:w:.:
..::::.~:*:%:>t::Jl~JN~I~2~O~03~-!II~M~A~~!E~M~Á~T~IC~A~J::::::::::::::::~~:
103
Del gráfico:
VGenerado = A X (21tR)
h=2R ... (2)
En el problema:
El volumen de la esfera:

Vesfem = 1 1tR3

De (1): =~1t( -Í2 a)3

=~-Í21ta3 ... (3)


3 e
El volumen de un tronco de pirámide regular está dado por:

... (4) D
En el triángulo AOP
Donde: h es la altura del tronco de pirámide.
Al es el área de la base del tronco de pirámide. R= :hsen(4S0 + a.) ... (1)

A2 es el área menor paralela a Al'


El volumen generado será:
En nuestro problema, el volumen del tronco de pirámi-
de regular: V Generado =A X 21tR
De (4): De (1): =d2x2{Jisen(4S0+a.)]

I _
\ "'1I1eo P -
(2Ri:(
-:3 L 16 a 2) + (4a 2) + V'( 16 a 2X 4a 2)J = tcd 3 sell( 45° + a.) ... (2)

Por definición todo sen a. :


De(l)
~(Ji (/)[
=- -3- 20a
2 r;:-¡J
+V64a' -1$sen$1
S6-Í2 a3 Luego el V Generado será máximo si:
=-3- ...
(5)
De (2):

Finalmente de (5) y (3) obtenemos: sen(4S0+a.)= I


=> 4S+a. = 90°
S6-Í2 a3
Vtronco P _ --3--
a. = 45°
Vesfem - 8-Í21ta3
Clave:C
-3-,- 38. Del enunciado: B
=2 Datos: r = 3cm
1t
Clave: O R=Scm

37. Teorema de Pappus


- Gulding: Al girar una
área A alrededor de un eje
e, se genera un volumen
V que es igual al producto
del valor A por el valor de
la longitud de la circunfe-
rencia que genera el cen-
tro de gravedad (c. Gl de
e
dicha área. Es decir:
Por semejanza de triángulos:
40. Para resolver las tres proposiciones deben hallar
I1S08 -I1TQ8 el rango de la función.
~ SO =08 J(x) = I-senlx!
TQ Q8
R R+2r+h Analizando senlxl :
-;: r+h
Scm Scm+2(3cm)+h
3cm 3cm+h
h=9cm
x
Cálculo de la altura 8 P
8P=2R+2r+h
= 2(S cm) + 2(3 cm)+9cm
=2Scm
Clave: B Analizando -sen\x\

39. En el gráfico:
y

x
x

b+ (-a) = 180° Analizando 1- senjxl


a=b = -180° ... (1)
y
Propiedades: sen(180 + e) = -sene} ... (2)
cos(180 + e) = -cose
En la expresión dada:

3
3sen( a b)+ sen. a+ senb
K=---7~~--------
6cos( a (jb )+cosa +cosb x

3sen( ~) + sell a + sen(180° + a) En las proposiciones:


De (1):
.6cos( -~80)+cos a+cos(1800+a)
1) J(x) es decreciente en (I; 3;); verdadero (V)
3sen(-{)0)+sen a +(-sen a)
De (2):
= 6cos(-30)+ cosa + (-cosa)
1I) J(x) es decrecienteen (_3; ;-I); Verdadero(V)
_{-~)+O IlO J (x) tiene con rango [O; 2]; Verdadero (V)

- ,1)+0 Clave: O

I
2
Clave: A
I
.1
I
cione la suma de las cifras de la cantidad que hubiese
MATEMÁTICA recibido la persona de menos edad, si la repartición hu-
1. Los números a, b . e, d satisfacen las ecuaciones: biera sido directamente proporcional a sus edades.
A) 10 B)II C) 12 D) 13 E) 14.
abed(!!). + deba(!!) = 20496
7. El producto de un número por" a " es 448 y por
d-c=b-a=2
" b " es 336. Calcule el producto de este número por ei
Entonces el valor de a + b + e + des: mayor número capícua de 3 cifras que se pueden for-
A) 16 B) 20 C) 24 D) 28 E) 32 mar con" a " y " b ''.

A) 46 508 B) 47609 C) 48 608


2. La suma de las razones geométricas que se pueden
formar con dos cantidades es 14. D) 49 610 E) 50 620

Calcule la relación entre la media geométrica; la media 8. En una obra se observa que faltando 54 días para su
armónica de esas dos cantidades. culminación fueron despedidos 10 obreros: pero a 12
A) 2,5 B) 2,0 C) 1,0 D) 0,5 E) 0,25 días para la culminación debe contratarse x obreros
para cumplir con el plazo estipulado.
3. Encuentre el menor producto positivo de a y b ta- Determinar la suma de cifras de x.
les que a +b , a2 +b2 ,sean enteros, pero a4 + b4 A)6 B)7 C)8 D)9 EllO
no lo sea, donde a.b E (O; 00)
9. En el gráfico se presenta la distribución del nú-
A) 0,25 B) 0,3 C) 0,6 D) 0,45 E) 0,5 mero de pacientes atendidos diariarncnbe en un Cen-
tro de Salud de la Zona Norte de Lima. 'La muestra
fue de 50 días de atención.
4. El número de fracciones equivalentes a ~;;3 ,cuyo Número
de días
producto de sus términos sea de cuatro cifras, es:
12 - - - - - - - - - - - - - - - -~
A) 14 B) 15 C) 16 D) 17 E) 18
10 - - - - - - - - - - - - r- - - r--
5. Un representante de electrodomésticos gana el 7%
de comisión por ventas a domicilio ¿Cuál será el monto 8 - - - - - - - - -r--
que recibirá por comisión, si ejecutada la cobranza y
6
deducida dicha comisión, entrega a la casa comercial la - -~ - ,.....
suma de 13 300 nuevos soles? 4
A) I 001 B)931 C)996 D) 870 E) 780
2

6. La repartición de una herencia fue inversamente pro-


porcional a las edades de tres personas, el reparto fue 35 36 37 38 39 40
de: 29400; 16 800: ~<) 200 respectivamente. Propor- Número de pacientes atendidos
:&:
J__ U_N_I _20_0_4_-I_M_AT_E_M_A_' T_I_C_A....I ~m~
GomeZ

Determine la validez de las afirmaciones siguientes: 13. Determinar el valor de verdad de las afirmaciones:
1.En el 20% de los días el Centro de Salud atendió a
1. Si xI = x2 => f(xl) = f(x2), para toda fun-
lo más 39 pacientes.
ción l.
II.En el 90% de los días el Centro de Salud ha atendi-
do un mínimo de 36 pacientes.
.II. Si f(x)=~4 ; XE [-2 ;4) => f es una
1ll.En más del 50% de los días el Centro de Salud ax-
atendió al menos 38 pacientes.
función sobreyectiva sobre x E [-2; 2)
A) FV V B) V FF C)FVF IlI. Toda función. impar es univalente.
O)FFV E)V V F A)V V V B) V VF C) FV F
O)FFV E) VFF
10. Sean a, b , c: y d cuatro números reales positi-
vos tal que a - b = c - d Y a < c: . Decir la verdad o 14. Hallar la suma A de números complejos.
falsedad de las siguientes afirmaciones.
A = (1+i)+ (2+ ¡2)+ (3+ ¡')+ (4+ ¡4)+ ...+ (4" + ¡4")
a c:
I. ¡; <J, si a <b
A)n(2n+l) B)2n(4n+l) C)O
c: a O) n(4n+l) E) 2n( 4n -1)
Il. J<¡;' si c c d I

15. Resolver el siguiente sistema de ecuaciones:

2x2+5xy-18y2=O

A)FFV B) FV V C) FV F xy + y2 -12 =O
O) VFV E) V FF
A) (-4; 2),(-2; 4) B) (-4; - 2),( -2; 4)
11. Qué cantidad es necesaria aumentar a las raíces C) (4;2),(-4;-2) 0)(4;2),(-2;4)
de la ecuación:
E) (4;-2),(-4;-2)

16. Determinar el conjunto de valores del número real


l
Para que las cantidades resultantes sean iguales en mag- r tal que la función f(x) = (rx2 - 2rx+lr , esté de-
nitud pero de signos opuestos.
fin ida en [O; l] .
A) a-b B)~ C) a+b
ab a-b ab A) (-8; ol B) (O; +00) C) [O ; 1)
O)~ E) b-a
a+b ab O) (-00; 1) E) [1 ; + 00)

12. Dada la siguiente ecuación: 17. Sea (a,,) la sucesión cuyo término general es :

(/" =~Il+I-V;;

Hallar x, sabiendo que" n " es cualquier entero positi- Entonces podemos afirmar que:
vo y log es el logaritmo en base 10. A) u" diverge a 00
A)6 B)3 C)4 0)2 B) u" converge a 11

C) ll" converge a I


UN12004-1 MATEMÁTICA I
D) Gil converge aO Podemos decir que:

E) Gil diverge a -00


Al abc=xy(c+d)
B) a b c= x yd
18. De 6 números positivos y 5 números negativos se
escogen 4 números al azar y se multiplican. Entonces, C) a b d v=x yc
el número de maneras en que el producto resultará po-
sitivo es: D) abd=xy(c+d)

A) 45 B) 170 C) 330 D) 480 E) I 080 E) acd=xy(c+d)

19. Sean las matrices: 23. El cuadrilátero PQRS está inscrito en una cir-

cunferencia, siendo el lado PS su diámetro. Sea T el


B=[: ~] punto de intersección de las prolongaciones de los la~

Tal que A B = BA , calcular el valor de (a + c) dos PQ y RS ; si PQ = 7 u, RS = 4u y TR = 6u .

A) 1/4 B) 1/2 C)I D)2 E) 3 Entonces el valor de QR es:

20. Los números A,B y D son proporcionales a a, b A) m u B) 51 u C) m u


Y d respectivamente. Si se cumple que a +b = 5 ;
D) .J35 lL E) J37 u
b +d =3 Y A +B+D = 2 abd , siendo abd
múltiplo de 3. Hallar B . 24. En un cuadrado se inscribe un octógono regular.

A) 102 B) 107 C) 123 D) 214 E) 321 La razón del perímetro del cuadrado a la del octógono
es igual a:

21. En la igura, ABC es un triángulo equilátero y la


í

A) J2 B) 2~ C) 1+ J2
medida del ángulo a es de 100°. Calcular la medida 2

del ángulo p. D) 2J2 + I E) 3J2 - 2


3
B

25. Un hexágono ABCDEF tiene sus ángulos con-


gruentes y sus lados son tales que:

AS = CD = EF =a y BC = DE = FA = b , sien-
2
do a> b . Calcular el área en u de dicho hexágono.

A) (a2
+4~b+b2)J3

A) 40° B) 20" C) 30° D) 15° E) 10° B) (a2 +4;b+b2)J3


22. pe la gráfica
C) (a;bfJ3

D) (a2-4~b+b2)J3

2
El (a -4~b+b2)-./6
A
26. En todo polígono regular convexo ABCDEF ... , 2a u . Luego se toma el punto RE P, de forma que la
se verifica que: proyección del segmento RQ sobre el plano nos da el

A) AC2 + AB2 = AD - BC
segmento R H de longitud a ti . Si la medida del ángulo

B) AC2 - AB2 = AD - BC
AHR es t y el perímetro del triángulo ARQ mide

3J3 a u . Hallar AQ .
C) AC2xAB2 =ADxBC

D) AC2+AB2=ADxBC
A) 31 au B) 2[3 a u C) 51 a u

E) AC2 - AB2 = ADx BC


D) 5J3 a u
2
27.A=(a,b), B=(a,-b), C=(-a,-b),
32. En un cono circular recto está inscrita una esfera,
D = ( -a, b ) son los vértices de un rectángulo.
cuya área de la superficie esférica es igual al área de la
base del cono. ¿En qué relación están el área de la su-
Si P=(x,y) cumple que DP=6u, CP=7u y
perficie lateral del cono limitado por la circunferencia
B P = 5 u , entonces el valor de A P es de tangencia con la superficie esférica y la superficie
lateral del cono?
A) 15 u . B) 2J3 u C) 3u
A) 9/25 B) 5/21 C) 3/7 D) 4/25 E) 4/21
D) 4u E) 3J2/1

33. Dos ángulos centrales de una circunferencia cum-


28. En un ángulo diedro. las distancias de un punto inte- plen lo siguiente:

4J2 u, 4 u y 8 ti res-
rior a las caras y a la arista miden 1) Son suplementarios.
pectivamente. Calcule la medida del ángulo diedro. 2) La diferencia de los arcos que subtienden es 2("11/.

A) 65° B) 70° C) 75° D) 80° E) 85° 3) La razón entre la medida de los ángulos es 4/rr .

Halle (en cm) la longitud del radio de la circunferencia.


29. Se tiene una esfera de centro O y radio 5 u . Un
2(4+rc) 3(4+rc) 7(4+ re)
plano ~corta a la esfera en una circunferencia C y la dis-
A) re(4-re) Bl re(4-re) C) 2re(4-re)
tancia de O a Pes 4 ti . Entonces el volumen del cono con
vértice O y base el círculo limitado por C es igual a: 4(4+rr) 9(4+rr)
D) rr(4-rr) E) 2rr(4-rr)
A) 8re u3 B) 12IT u·1
34. Dada la función f ,definida por
D) 16re u3 E) l Sn 1I.\

f(x) = tan x+ col x+ .,Jsell x + .,Jcos x .


30. En un rombo ABCO. M es punto medio de CD y
Halle -:1 dominio de f .
la diagonal BDcortaa AM en un punto R .SI RM =5t1
A) (krt; kt: + nl2) \:/k E Z
Y la medida del ángulo DRM es 53°; hallar BD.
B)(2kn;2kn+nl2) \:/kEZ
A) 18 u B) 30u C) 35u D) 36u E) 40u
C) (2kn + re ; Zkt: + 3nl2): \:/k E Z
31 . Sea P un plano y E una recta paralela a dicho
D) (2kn - nl2 ; 2kre) \:/k E Z
plai . Se toman los punto Q E .e y A E (j' Je forma
que la pro)<" ,llÍn del scru.cnto A Q sobre el plano mide
E) (kn ; kn + nl2); \:/k E Z
UN12004-1 MATEMÁTICA I 7

35. Si a es un ángulo agudo, halle el valor M ,donde


39. Si 2cose+i(2sene+I)=
M = cOl(a/4)-tan(a/4)
csca+col a
. [3cos $+ i (3 sen $+ 1)]( COS} + i sel1~n
A) ~ B).J2 e) 2 D) 2,5 E) 4
Entonces, el valor de cos( $+ f) es:
36. La siguiente figura es un cuadrado, donde Q es el
punto medio del lado AB ,determine csc e. A) -1 B) J3 C) I D) J3 E) J5
2

40. En la figura mostrada, halle el área de la región


triangular OQP .

y
(O; 1)

A)2 C)3 D)4 E)2J5 (1;0) X

37. Determine el rango de la función definida por:

((x)= 31t xc O
. . Iarc tg xl- 21arcclg xl

A) (-4; -3) B) (-I;O) C) (-2; -1) sene cose sene cose


A) B)
4 8
D) (-3;-2) E) (2;3)
sene cose sene cose
C) D)
16 2
38. Indique una solución general para la ecuación:
4cosxcos2xcos3x= I E) -sene cose

A) ktc ± ¡; "i/ k E Z

B) k1t+lI "i/k E Z
-2

C) k1t± 1 "i/k E Z

D) kit ± ~ "i/kE Z

E) kt: ± ~ "i/k E Z
~.,_:rmrnl.
~~
~:==:V:IWf:i:Y:}it:'~:'="IL__ UN_I_2_0_04_-_1
Mt~._
_M_AT_E_M_A_' T_IC_A
__ 1MI~;;;;;;;;;;=:;~!~~

MATEMÁTICA Luego:

, • De la ecuación: MG.(a;b) = M
MH.(a;b) 2
abcd (lI) + dcba{l/) = 20496 1+1.
a b
~ 20496= 14443(11) _ a+b ... ( 1)
-2M
Luego:
abed(il) Por condición del problema:

deba(II) l!:.+!!..=14
b a
1444\11)
2 b2
~=14
Observando la última columna (izquierda) a +d > II ; ba
pues las unidades de orden superior que se acumulan, a2 +b2 = 14ab
en una suma de dos sumandos a lo más es uno (1). a2+2ab+b2 = 16ab
Tomando en cuenta las consideraciones anteriores: (a+b)2 = 16ab
En la primera columna (derecha):
a+b=4M
d+a=13(II)=lxll+3=14 ... (1) a+b =2
~ 2M ...(2)
En la segunda columna:

1(11)+c+b=14{1I) Comparando (1) Y (2), concluimos que:

= Ixll +4 Me.(a;b) =2
=15 MH.(a;b)
~c+b=15-1(II) Clave: B
= 14 ... (2) 3. Si a; b e {O; 00) . Se debe hallar" a x b •. mínimo,
Finalmente, de (1) Y (:?)
de tal forma que:
(ti + (f) 1- (e + 11 = I 4 + I 4
a+b e Z (1)
{/+h+c+d=2i->
a2+b2eZ (2)
Clave: D
Nota: Las ecuacioncs ,1 - e =b- a =2 j;-'¡o es ne-
4
a +b 4
",Z .. \.\)

cesario!
Operando con la expresión de (2):

2. Sean los 'a y b los números, entonces por dato del a2 +b2 = a2 + 2ab+¡,2 - 2ah
~
problema tenemos: z
De(2)
La media geométrica: MG.(a;b)=M 2
= (a + b) - 2ah
'-.--' ... (4)
'--.r--'
- 2 ~
La media .rrrnónica MH.(o:h)=-- Z

1+1 0e(Ij ncccsariarncmc

a b

¡¡
Operando con la expresión de (3): k =7;8;9;...
;21

4 4 (,)2
a +b = {/- + (2)2
b
La variable k adquiere 15 valores con 1", ,'1I:1k orie i-
nan 15 fracciones equivalentes.

=(a2)2 +2(a2)(b2)+(b2)2 _2(a2)(b2) Clave: B

=(a2+b2)2 -(2ab)x(ab) ...(5) 5. Datos:


=r:
de(2)
:r: Porcentaje que gana el representante: 7%
de (4) Se entrega a la casa comercial : C = 13,300
Observe que en (4) "2ab" tiene que ser un número ente- El dinero "C" que entrega el representante a la casa co-
ro necesariamente para que se cumpla la igualdad. mercial representa el 93% del total, pues el 7% se que-
da con él.
En la expresión (5) para que esta expresión no sea nú-
mero entero la expresión que está a su costado multi- Aplicando regla de tres directa:
plicando debe tener la forma.
93% ~ S/.l3300
ab=! ... (6) 7% ~ x
2

~ (2ab)(1)= ~ 7%xSI.13300
aheZ ~ x= 93%
Luego de (6) podemos afirmar: = SI. I 001

abmin = t = 0,5 k = I El monto de la Comisión del representante será SI. I 00 l.

Clave: E Clave: A
4. Reduciendo la fracción a una función equivalente 6. Sean A ; B Y C las edades de las personas, entonces
irreductible. la repartición de la herencia fue:
-ªl.. 3
Persona de edad A : 24 400
203 7
Persona de edad B : 16 800
De esta fracción (t) se podrán formar infinitas' frac-
Persona de edad C : 39 200
ciones equivalentes, así: Total herencia : 85400 ... (1)
3x2 3x3 . Con la repartición se dio en forma inversamente pro-
7x2 7x3 ' porcional a sus edades, se cumple:
_ 3xk 29400xA = 16800xB = 39200xC
-7xk ; donde k = I , 2 , 3 , ... ... (2)

Por condición del problema se nos pide que el producto m.c.m.(29400; 16800; 39200) = 117600
se sus términos sea de cuatro cifras, es decir: Dividiendo el m.e.m. en (2):
(3 x k )( 7 x k ) = abcd 29400 x A 16800xB 39200xC
21k = abcd 117600 117600 117600

Es decir: .A=!i=S2 ... (3)


4 7 3
1000 $ 21k 2 :59999
Se ha obtenido una relación entre las tres edades de las per-
47,619$ k2 $476,143 sonas:
6,9 :5 k :521.82 Si la repartición de la herencia se hiciera en proporción
~ 7:5 k :5 21 directa a sus edades, a cada una de las personas le hu-
biera tocado x, y , z respectivamente, es decir: El trabajo que hace los x obreros durante los 12 días:

~=l.=~ Tx =xx(12 días)


4 7 3
El trabajo que dejan de hacer los 10 obreros durante
Propiedad de proporciones: los S4 días es compensado con los x trabajadores du-
x y Z x+ y+z rante los 12 días, es decir.
¡-=7"=3"= 4+7+3 ... (4)
TIO = Tx
De (1) se puede deducir que:
(10 obr)x(S4 días) = xx(12 dias)
x+y+z=8S400 ... (S) x = 4S días
Entonces de (S) en (4):
La suma de las cifras de x:
l= 1..= ~= 85400 S=4+S=9
4 7 3 14
Clave: O
La persona de menor edad:

1. = 8S4OO 9. En el gráfico se puede observar que sumando el


3 14 valor de cada altura de las barras se obtiene las sumas
z = 18300 de días atendidos (SO días).
Finalmente, las sumas de las cifras de z: Debajo de cada barra se indica el número de pacientes
atendidos durante los días respectivos:
1+8+3+0+0= 12
De las afirmaciones:
Clave: B
l. "El 20% de los días el Centro de Salud ntendió a
7. Sea n el número, entonces por condición del pro- lo más 39 pacientes".
blema:
na = 448 ... (1) El 20% de SO días = l~~ X SO di as = 10 d ias
nb = 336 ... (2)
En las barras se observa que en un periodo de 10
De (1) Y (2) se deduce días se atendió a 40 pacientes (altura barra), se-
na » nb gún el enunciado debieron atender no mas de 39
~ a>b pacientes, por tanto la afirmación es:

El mayor número capícua que se puede formar con a y b: ¡FALSA!.

C = ab a 11. "En el 90% de los 'días el Centro de Salud ha aten-


Finalmente: dido un mínimo de 36 pacientes.
/1.xC=lIx(aba) 9 0
90% de SO días = 1 0 0xSO dias = 40 di as
2
=1I(axI0 +bxI0+a)
En las barras del gráfico se observa que en 40
=1I(IOla+IOh) días,tomando las 5 primeras barras suman:
~ IOI(na)+ 10(nb) s = 3S +36+37 + 39 = 143
De (1) y (2): = 101(448)+ 10(336) La cantidad de 143 pacientes supera la cantidad
= 48 608 mínima de 36, por tanto la afirmación es:
¡VERDADERA!
Clave: C
111. "En más del SO% de los días el Centro de Salud
8. El trabajo que dejan de hacer los 10 obreros durante los atendió al menos a 38 pacientes".
54 días: Más del SO% de las días puede ser 90%

T1o=IO (//. ,54 di as ... (1) 90% de SO días = 40 días


Del análisis anterior (11) se concluyó que en 40
días se atendió a 143 pacientes, se superó las expecta-
tivas del enunciado (38 pacientes), entonces
estaafirrnación es: e a
b<-;¡
¡VERDADERA! al < ab
Clave: A cd +ab < O
De (1): e(e+b-a)-ab <O
10. Del enunciado: a; b ; e y d E IR + e2+eb-ca-ab <O
Además se tiene las condiciones: e(e-a)+b(e-a) <O
(e-a)(e+ b) < O ...(3)
d=c+b-a
a-b=c-d ~ { b=a-c+d (1)
... Por condición del problema:
a«c ~ a-e<O ... (11) De (2): c-a>O
Analizando los tres casos: Los números e y b son reales positivos
a e ~ c+b s-t)
1) b<-;¡ :
Luego, llegamos a una conclución:
Si: a <b ~ a- b<O ... (1)
(e-a)(e+b) <O
De (11) Y (1): '-v-''-v-' FALSA
(+) (+)
(a -e)(a -b) > O
Clave: E
a" =ab=ca+cb > O
a(a -b-e)+eb >O 11. La ecuación de segundo grado de la forma:
-a(e+b-a)+eb >O
Ax2 + Bx+c = O ... (1)
De (1): -a(d)+eb>O El valor de sus raices se obtinen:
cb » a d
-B+JB2-4AC ... (11)
a
-<-
e XI = 2A
b d
VERDADERO -B-JB2-4AC ... (111)
x2 = 2A
e a
10 -;¡<¡;: Sumando (11)y (I11)obtenemos:

Si: c<d ~ e- d <O ... (2) B


XI +x2 =-1\ ... (IV)

De (11) y (2): Dada la ecuación:


(a-e)(é-d»O
(Q _l!..)x2 + 2(a+b )x+ Q.+ t_1 =O
... (1)
ac+ad _e2 -al> O b 'a b a

a(a-e+d)-ad >O Comparando (1) y (1) se puede deducir que:


De (1): e(b)-ad > O
cb » ad B = 2({/+") ... (2)
e a
->- La condición del problema nos dice que al sumar una
d h
cantidad "N" a cada raíz de la ecuación, entonces estas
FALSA son iguales en magnitud; pero de signos opuestos, es
decir:
UN12004-1 MATEMÁTICA li-- _.;¡::====::~¡¡;~"
xl+N=-(x2+N)
Tomando en cuenta esta condición ( x > I ),la raíl. xI.
-2N = xI + x2
se descarta, quedando como única raíz x2 = 3
De (3): -2N =_Ji
A Clave: B
2N =!L
A 13. Analizando cada caso:
I. VERDADERO:
De (4):
2N = 2(a+b)
s!:'_Q. Del enunciado:
b a Si: XI = x2 .. _(1)
(a+b)
N =-,--,
a- -b-
... (2)

ab
Considerando que: f (xI) = f ( xI)
(a + b)ab
(a + b)(a - b) De (1):
=..E!L Se obtiene la relación (2), entonces se cumple para
a-b toda función.
Clave: B 11. FALSO:

12.
IO/O}PI Sea: f(x) = _3_ ; XE [2 ; 4)
[og(2x-I)"+log(x-l) =1l ... (I) ax-4
Analizando para que valores dex no está definida
log(2x-lr +log(x-Ir =n la función, es decir si:

n lo g (2x - l ) + 11 lo g (x - l ) = Il ax - 4 = O => x = .:!:


a
log (2x - 1) + lo g (x -1) =I
log(2x-I)(x-I)=1 (E n x = a4 ,-
se crea una asíntota vertica 1)
Por propiedad: Si: a >0:
1)( 1) 10
(2x - x -

2x2 - 3x - 9 = O
=
=> DOlnf(x)=[-2 ~)v(~ ;4)
(2x+ 3)(x - 3) = O Graficando:

f(x)
=> 2x+3=0

xI = -3/2

=> x-3=0

x2 = 3

De (1) se deduce que 11 es un número positivo, entonces x


las expresiones:
2x-I>0 /\ x-I>O
x>l. /\ x>1
2
=> x> I ... (2)
~ Rall¡(x)=(-00;¡(-2)]u(f(4);00) y

Para la función f(x) = ~4 XE [-2 ; 2)


ax-

Donde a > O , se tiene:

Dom f(x) = [-2 ; ;) u(~ ; 2/ x


Graficando: Observe que r, * Xz pero: f(xl) = f(Xz) =y
Por tanto ¡(x) no es univalente.
f(x) I
I

I
Clave: E
I
I
I 14. Para resolver este ejercicio debemos tener en
f(2) ----~--. cuenta que:
I ,

, 1: (1 + k)k
S=I+2+3+ ... + k=--- 2
-2 f(4) -----~---~-..
1:
-.---\->--- I ~

2 4 x + ¡2 + i3 + i4 =i+(-I)+(-¡)+I =0
¡5 + i6+ ¡7+ ¡8=i+(-I)+(-i)+'1=0
i9 + ilO +ill +i12 = i+(-I)+(-i)+ 1= O

En la suma:

A = (1+i)+ (2+ i2)+ (3+ i')+ (4+ i4)+ ...+ (4/1+ i4")
~ ROIlf(x) =(-00; f(-2)] u(f(4) ; 00) = (1+2+3+4+ ... +411)+(i+i2 +i' +i4 + ... +i4")

= [[1 + (4~)](411)]+ [i + (-1)+ (-i)+1 + ... + 1]


Observe que elRan ¡(x) definida en

XE [-2 ; 4), es diferente que Ran ¡(x) defini- + 4n)]+ [O]


= [211(1

da en x E [-2 ; 2} , entonces se puede afirmar que = 211(1+ 4n)


a la función ¡(x) no es sobreyectiva.
Clave: B

Nota: Para a < O no es necesario analizar la fun- 15. En el sistema de ccuaciones:

ción puesto que ya se demostró al menos que para 2x2+5xy-ISy2=O ... (1)

a > O no es sobreyectiva. xy+ y2 -12 = O ... (2)

Operando en (1):
11I. FALSO:
2x2 +5xy-ISl =O
Una fu~ción ¡(x) es univalente o inyectiva cuan-
(2x+9y)(x-2y) =O
do para x¡ del dominio de una función le corres- ~'-v-'
1 11

ponde un solo valor de y¡ del rango y viceversa. De (1): 2x+9y =O

La función ¡(x) = Ixl es impar


... (3)
UN12004-1 MATEMÁTICA I=~===='''''':'w;:
Reemplazando (3) en (2): De (1):
O<x::;1
(-~ y )y + l-12 =O
-I<x-I::;O
y=±2H (y é IR) O::; (x_I)2 <1

De (11): x- 2)' = O -1::; (x_I)2 -1 < O

~ x=2y ... (4) 1 <-1


(x_I)2 _1 -
Reemplazando (4) en (2):
1 > 1 ... (3)
(2y)y+l-12=0 (x_I)2 _1 -

31 =12 De (2): r"?1 ... (4)


y=±2
Para llegar a la expresión (4) se partió de la premisa de
YI =2 Y2 =-2 que r no debería tener estos valores, es decir:

Reemplazando YI = 2 en (4): ~ ré[l; (0)


De lo anterior podemos afirmar que r puede tomar el
XI=2(2)=~
complemto de los vol ores obtenidos anteriormente:
Reemplazando Y2 = -2 en (4): rE (_00 ; 1)
X2 =2(-2)=-~ Clave:D
Finalmente el conjunto solución: 17. Para s.rhcr a donde converge el término a 11 • de-
bemos hacer que 1/ sea lo bastante grande (1/ ~ 00) .
C.S. = {(xI: -"1) . (x~ : Y2)}
={(4 ;2) ..(-4; -2)} Si aplicamos límites (1/ ~ 00) a la expresión:

Clave: C
... (1)

16. En la función: Nos dará una indeterminación, razón por la cual prime-
ro cambiaremos su torma aprovechando la siguiente
f(x)=-o _1_- identidad algebraica:
r.\--2rx+1
Donde: x E (O ; 1 J ... (1) (x - y)( x2 + xy + i) = x' - /

En (1):
Los valores de r que hacen indeterminada la función,
se obtiene haciendo el denominador de la función f (x) [(~I1+I/ +~II+1.lf,;'+(v,;-n
igual a cero, así: all=(~I/+I-lf,;')? .'
. (~II+lr +~I1+Llf,;'+($,r
r.r2 -2u'+1 =0
Despejando la variable r: (~11+ 1 )3 - (lf,;')'
rx2 - 2rx + 1 = O
r(x2 - 2x) =-1
(~II+ 1 f +~II+ Lv,;-+ (lf,;')2
r[(x-I)2 -IJ =-1 1

1
r=- ---- ... (2)
(1"_1)2_1
Aplicando límites: ~ 2a-c=2a+3

l c=-3
lini an = lini 2 ,
n-+- n-+- (~n+ 1) +~n+ \.V;;+(V;;r
~ -3=-a+1
lima =..L a=4
11 co
n-+-
=0 Se pide calcular:
a+c =4+(-3)= I
Clave: O
Clave: C
18. Si se dispone de 6 números positi vos y 5 números
negativos, para obtener 4 números cuyo producto sea 20. Del enunciado:
positivo se presentan 3 casos:
A=!l=D ... (1)
Caso 1: Seleccionar 4 números positivos: a b el
C6- 6! _6x5x4!_15 a+b=5 ... (2)
4 - (6-4)!4! - 2!4! -
b+d =3 ... (3)
Caso 2: Seleccionar 4 números negativos:
A+B+D=2xabel ... (4)
CS _ 5! _ 5 x 4! _ 5 _ o
4 - (5-4)!4! - I !4! - ... (5)
abc = 3
Caso 3: Seleccionar 2 números positivos y 2 números Por propiedad de múltiplos de números, en (5):
negativos: _ o o
6 'i 6! 5! abc=S ~ a+b+tl s=Y
C2 xCi = (6_2)!2!x (5-2)!2! = 150 Considerando además (2) y (3):
El total de formas de seleccionar 4 números, tal que su a=3 b=2 d= I
producto se positivo: Reemplazando los valores de a, b Y elen (4):
T=cg+c~+qxci A+ B+ D = 2x(321) = 642 ...(6)
=15+5+150
Por propiedad de proporciones iguales:
=170
A+B+D
Clave: B a+b+d

19. Se dan las matrices: De (6):

También se tiene que:


B A+B+D
Del enunciado: Ax B = B x A b a+b+e1
1- = 107

[~ -I]x[a
I e
-I]=[a
5 e
-:] ~ B=214
Clave: O
2a-c -3]=[2a+3 -a+l]
[ 3a + e 8 2c + 15 - e+5 21. Datos:
Por propiedad, si dos matrices son iguales, entonces a = 100 0

todos sus elementos entre si son iguales:


El triángulo ASC es equilátero.
~=.========[JU~N~I~2~O~04~-II }M~A~J~E~M~A~·T~I~C~A~=_=,,:;g:.
Por propiedades:
B
===:S:,,:..
Del enunciado.
==.
••.•..
------..;S

Por el teorema de las secantes:


Cálculo de ángulo p, en el triángulo BDQ,
(PQ+QT)xQT = (SR+RT)x RT
p + 60 0
+ a = 180
(7 u+QT)xQT = (4u +6u)x6u
P + 60 + (100
0 0
) = 180
(7 u)(QT)+(QT)(QT) = 60u
P = 200

~ QT2 +(7 u)QT -60u =O


Clave: B
(QT + 12u)(QT -5u) = O

22. En la Gráfica: ~ QT + 12u =0


QT = -12 u (descartado)

~ QT-5u=0
... (1)
QT=5u

En el triángulo rectángulo PRT.


6u I
cose = 12u = 2 ... (2)

Cálculo de QR, del triángulo QRT. Por el teorema de


ED es secante al triángulo ASe. cosenos:

Por el teorema de Menelde:


QR = ~(QT)2 + (RTl- 2(QT)(RT) cose
a.b.c = x.y(c +d)
De(l) y (2): = (5 u)2 + (6 u)2 - 2(5 u)(6 u )(~)
Clave: A
=~6Iu2_30u2
23. Datos:
=J3Iu
PQ=7u
Clave: B
RS=4u
TR=6u
JM~A~:r~E~M~Á~T~I~C~AC):::===="=~
~;[=~u~N~I~2~o~o~4~-II
~~~ez~' "~":":Í':tw=¡¡¡':N:q:'1t:¡:::::lf:""::.::.~:0:"":*,.:!lP:..r

24. Del enunciado: 25. Si sus ángulos del hexágono son congruentes, en-
x tonces este está inscrito dentro de un triángulo equiláte-
ro, así:
G

x x

Los 4 triángulos formados en cada vértice del cuadro Observe que:


son triángulos rectángulos iguales de a = 45° , luego:
ó'BGC ;: ó'EDH ;: t1IAF ... (1)

cosa =1. Para hallar el área del hexágono ABCDEF, primero cal-
x
culamos el área del triángulo equilátero IGH y luego
restamos el área de los tres triángulos equiláteros igua-
les. De (1):

A ABCDEF = A AJGH - 3A 6BGC ... (2)

... (1) Por definición el área de un triasgulo equilátero:

Del gráfico, perímetro de cuadro ABCD: A= e2134 ... (*)


De (1):
Del gráfico, área del triángulo equilatero IGH:
PABCD =4(y+x+ y)
213
=4[( 1x )+x+( 1x)] AMGH =(a+2b) "'4

=4(.J2+I)x ... (2)


2
= (a +4ab+4b
2
)1 .... (3)

Calculo del área del triángulo equilatero BGC:


Perímetro del octágono regular:
o J:~
A6BGC = b--;f ... (4)
PEFGH/JKL = 8x ... (3)

Finalmente de (2) y (3):

PABCD

PEFGHIJKL

Clave: C Clave: A
UNI 2004-1 MATEMÁTICA I==:=::~!~~~
26. Graficando el polígono regular: Teorema de Marlen: Para cualquier punto P situado
dentro de un rectángulo se cumple:

~
A F En el problema, calculo de AP:

(CP)2 +(AP)2 = (DP)2 +(BP)2

G (7u)+(AP)2 =(6u)2+(5ul
(APl = 12
Por ser un polígono regular se cumple:
AB = BC = CD = DE = ... ... (1) ~ AP=±2.J3
AC=BD=DF=.... ...(2) AP = 2.J3 (-2.J3 se descarta)
Teorema de Ptolomeo: En todo cuadrilátero inscrito
Clave: B
en una circunferencia, el producto de sus diagonales es
igual a la suma del producto de sus lados opuestos.
28. Según el enunciado:
Del gráfico; cuadrilátero ABCD inscrito en la circunfe-
rencia.
D

Considerando (1) Y (2) para aplicar el Teorema de


Ptolomeo: Por el teorema de las tres perpendiculares:

(AC)(BD) = (AB)(CD)+ (BC)(AD) BP.lAB Y CP.lAC


(AC)(AC) = (AB)(AB)+ (BC)(AD) Luego:
(AC)2 + (AB)2 = (AD)(BC) Ángulo diedro = a +8 ... (*)
En el triángulo rectángulo ABP
Clave: E
sena = 4J2 u = J2
Su 2
27. Del anunciado tenemos:
~ a=45° ... (1)
y
En el triángulo rectángulo APC:

.I'en8=~=.!.
8u 2
~~ ..' (2)
~ 8 = 30°
1
x Finalmente en (*) de (1) Y (2):

Ángulo diedro = a +8 = 45° + 30° = 75°


CC-a;-b) B(a;-b) Clave: C


UN12004-1 MATEMÁTICA Xi

29. Del enunciado y trazado algunos líneas auxiliares El punto R es el baricentro del triángulo ACD, entonces:
Circunferencia C RD =1. .. , (1)
• PR I
AR _ 2
• RM-'
AR ='1
5u I
AR=IOu .. , (2)

En el triángulo rectángulo APR:

cos 53° = ~~

Datos:
De (2)=-
Radio de la esfera: R = 5u
Altura del cono : h = 4u ~ PR=6u .., (3)

Reemplazando (3) en (1):


En el triángulo rectángulo OPB,
PB = r = ~r-(0-B-)-:-2
--(-O-P""")
2 RD 2 ~ RD= 12u
6;=1 .., (4)

=~ Cálculo de BD:
BD = 2(PD)
=J(5u)2_(4u)2
= 2(PR+RD)
= 3u
De (3) y (4): =2(6u+12u)
El volumen del cono: = 36u

Clave: O

=t[(nr2}h] 31 .Graficando:

=H(n(3u)2)(4u)]
= 12Jtu3

Clave: B

30. Del enunciado,


B
Datos:
RM=5 u
A
Ángulo: DRM = 53°

Dato: Perímetro /!;AQR : P = 3~ (/II .. , (1)

A'=:----+---~C Calculo de AR en el triángulo AH R:

AR = ~AH2 +HR2 -2(AH)(HR)w,\'}

= (2a u)2 +(0 u/ -2(2au)(a u)(~)


D =~Oll .., (2)
~- -="\\wWttt~1 UN12004-1 MATEMÁTICA I:,::¡;. 1_'00.©%WtiM1WMWiJ!!':"*"~i~
~ ~~'<i. rt- ~:izm:Mit;im.~'jm:::tJ.XK$t@t_~G8iñez

Perímetro del triángulo AQR, por definición: Por condición del problema:
P=AQ+QR+AR Abase = Aesfcra

De (1) y (2): 3/3 au = AQ+ QR+/3 lIU rrR 2 = 4rrtl 2

=> QR=2/3au-AQ ... (3) rI =!l ... (1)


2
Aplicando Pitágoras en: En el triángulo SOP:
!lA Q H: h = J AQ2 - ( 2 a ti ) 2 ... (4)
-2 __ I
R

ts 9 =¡¡- 2 ... (*)


!lHQR: h=JAQ2-(au)2 ... (5)
Por definición:
Igualando (4) y (5):
Ttg 9
rg 29=--,-
~AQ2_(2a¡¡)2 =JAQ2-(au)2 1-lg-9

AQ2 = QR2 + 3a2¡¡2


De (*):
_ 2(t)
De (3): AQ-, = ( 2",3r: all-AQ )2 +3a 211 2 - I-(tf
AQ = 5/3 _4
4 -3
Clave: C => 29 = 53° ... (2)

32_ Del enunciado:


v Por propiedad de ángulos:
SP 1. O' P Y SA 1. AO

=> <l: AOO' = -c ASP


De (2): => <l:AOO'=P=29=53° ... (3)

Cálculo de r, en el triángulo rectángulo AOO' :

sen p=L
ti

De(l)y(3): i = l!: => r = l.R (4)


5 R 5

Cálculo de la gcncrauíz G R del cono de radio R.

\'ec28 = SV
. R

pV IV - " < P >/ ) De (3): ~= SV


3 R

=> 5
SV="3R=GR ... (5)

Radio de la base del cono: R


r Cálculo del área lateral del cono de radio R.
Radio de la esfera inscrita: d
At-(R) = rrRGII
Del gráfico:
De (5):
=rrR(~R)
Área del base eJel cono total: Abase = rrR 2

Área de la superficie esférica: Acsfera = 4rrtl 2 =2rrR2 ... (6)


3

¡ ~
I
UN12004-1 MATEMÁTICA l'
Cálculo de la generatriz del cono de radio r: Reemplazando (III) en (1):

ese 2B = A.Y ~B+B = rr


r rr
1t2 (IV)
5 _ AV ~ B=4+1t
o ••

De (4):
3- lR Reemplazando (11)en (111):
5

~
2
AV=3R=GR ... (7) a=i( 4:1t)
Cálculo del área lateral del cono de radio r: ~
4+rr O" (V)
AL(r) = m'Gr
Cálculo del radio de la circunferencia en (11),aplicando
=rr(~R)(~R) (IV) y (V):
= ..i. R2
rr ...(8)
R= 2elll
2
15
(4~1t)-( 4: 1t )
Finalmente, de (6) y (8):
2(4+1t)
4 2
AL(r) 15rrR 4 = 1t(4-1t)
AL.(R) = ?"rrR2 = 25 Clave: A
3
Clave: O 34. En la función:

33. Graficando según el enunciado, donde los ángu- o~(x) = tg x+ctg xO+Jsenx +Jcos x
los los daremos en radianes, puesto que tenemos longi- sen.x cosx ~ ~
= --+--+'l/sen. x + 'l/eosx
tudes de arcos: cosx senx ~
'(i) (11)

De (1): cos x -# O ... (1)

.I'ellx-#O ... (2)


De (11): sen x ~ O ... (3)

cos x ~ O ... (4)


De (1) Y (4):
cos x> O
De (2) Y(3):
Por definición, la longitud de un arco de circunferencia sen x » O
se obtiene multiplicando la medida del ángulo (radianes)
y
por el radio.
1
Por la condición (1) del problema se tiene:
a + e = 1t ... (1)
Por la condición (2) del problema se tiene: a > e
aR-BR = 2 cm
x
R= a-B
2cm ... (1I)
-1

Por la condición (3) del problema se tiene:

ea 4
1t
~ a=iB
1t
... (11I) -1
~~I_.....;;.U ..N..I_20..0..4_-I_M_AT_E_M_A_'T_I_C_A_
~ X E Iel'. cuadrante. covS!
=_-_2
\-ellS!.
- 2
Generalizando: =clg % (3)

XE (2rr.k 2rr.k+i) Reemplazando las identidades de (2) y (3) en (1):

Clave: B

35. En la expresión:
(1)
~ Clave: C
clgS!:.-lgS!
M = 4 4 a<2!: 36. Én el gráfico:
csca+clg a __
. (1)
~ 2
(11) A~ ~i- a__ -"

1)
ex
clg--Ig- ex = ---Ig-
I ex
4 4 IgS! 4
4 2a
l_tg2S!.
___ 4
Igs:!:.
4

=7---
-
[1 -Ig z4n 1
2/gS!.
D

Del gráfico: 8 = 2a ___ (1)


4
En el tritingulo rectángulo QBC
= 2(cI 2(%))
g
1 a=~=-'- __
. (2)
s 2a 2
--- (2) _ Por identidades trigonométricas:

21ga
tg2a=--?-
I-Ig-a

2(1-)
De (2): /g2ex=---
I-(if
4
"3
De (1): Ig 2a=/g 8=1.
3 4L1 3
Luego:

csc8 =%
Clave: B
UN12004-1 MATEMÁTICA I;==~~~~~::;~
38. Para resolver la ecuación aplicaremos las Generalizando:
identidades:
2x = 2k7t + 27t
o.-~ o.+~ - 3
• cos o. + cos ~ = 2 =»: co,I'- -
2
... (1)
x = k7t+~
-3

CO,~S =
~1+COS2S
2 Clave: C

... (2) 39.EPor definición de números complejos:

Luego en la ecuación: eio. = cosa + i s en a


4 cos x.cos 2x. cos 3x = I
En la ecuación:
2(2 cos x. cos 2x )cos 3x = I
~ Zcos S+i(2senS+ 1) = [:3cos$+i(3sen$+ O{ea,I'} +i s('//}]
(1)

2( CO.I·3x + cos x )co,~ 3x = I ¡!!


2(cosS+ i senS)+ i = [3cos$+ i(3sen$+ 1)](' 3
(2cos 2 3x) + (2cos x. cos 3x) = I ~
'---.,-------' ~ 2,)0
(2) (1)
¡!!.
(1 + cos 6x)+ (2cosx. cos3x) = I 2e+i = [3cos$+ i(3sen$ + 1)]1'J
,lt
cos 6x+ Zcosx cos3x = O
~
(1)
e -'") (2e;9 + i) = 3cos$+ i(3sen$+ 1)

('(Js6x+ (cos4x+ cos2x) =O ;(9-"') -;~


21' 3 +ie
'-v------' - =3cos$+i(3sl'I/$+I)
. ,
\cos6x+ cos2x)+ cos4x =O
~ (1) (11)
(1)
Resolviendo (1):
(2cos2x. cos4x)+cos4x=0
cos4x(2cos2x+ 1) = O
1= 2[eas( S-})+i sen( S-})]+ i[cos( -~)+ i sel/( -~)]
.l'el/(S-~)]+{i+{ -1)]
~ 2cos2x+I=O
= 2[cos(S-~)+i
cos'I:x = -t 2XE 11 e
= 2cos(S-~)+i 2sefl(s-~)+ii+(-I{ -1)
y
= 2COf(e-~)+
, 3
J3
2
+i[2fell(e-~)+l]
' 3 2

Igualando las partes reales e imaginarias de (1) y (11):

131\2 • Parte real (R).


x
~
1
2cos(S-})+ 1 = 3cos$

2co.\'( S -~) = 3cos$ -1 .(1)

2x = 27t • Parte imaginaria (i):


3
2sen(S-~)+t= 3sen$+ I

2,fell(S-~)= 3sen$+t ... (2)


~~========:' I__ -¿:,~::tzm:
U;.;.;N.;.I,;;;2,;,O,;,O.;.4-.;.I_M;.;;.;.A,;.;:T.;;E;;;.M;;;.A;.;'.;.T.;.IC;.A;.;....¡I::

Elevando al cuadrado ambos miembros de (1) Y (2) Y Por semejanza de triángulos:


luego surnándolos:
IBPI IABI
[2c;o,ie-})r +[2fene-}r =( XlI.I'4>-1J +(3sen4>+~f IOPI = IQOI
1. = !co.\"e¡
4 = 2(J3ClIS4>+sen4» I X

.)3cos4>-sencjl=2 leo.\" e¡
~ x=-2- ... (1)
{cos~.coscjl- selt~sen4>] = 2
Por estar en el 11 cuadrante:
2c;o.,{4>+~) = 2
Icosel = -CO.l" O

co·i4>+~)=1 En (1):

Clave: C ~ x=--2-
ClI.'·O

40. Como la circunferencia es de radio igual a unidad Las coordenadas de Q seran:


(1), las coordenadas del triángulo serán:
... (2)
y
Calculo de la altura h:
(O; 1) h = sen O-O
(-eosO;senO = sen O ... (3)
R

X (SCII O)
(1 ;0) x

Clave: A
~ M_A_~E_M_Á_T_IC_A~I~~~§§~i~

~~~~t!J~~
MATEMÁTICA"

A) Sólo III B) Sólo I y II C) Sólo I y III


MATEMÁTICA D) Sólo 1 E) Sólo Ir y III

1. El número AABB es un cuadrado perfecto y la


6. Sean los números a , b Y r enteros. Al dividir
raíz correspondiente es un número de la forma XX .
(a + b) entre b, se obtiene como cociente 3r y como
Calcule A + B + X . resto r. Si a > 15r y b es primo menor a 10. Enton-
A) 16 B) 17 C) 18 D) 19 E) 20 ces b es ígual a:
A)1 B) 2 C) 3 D) 5 E) 7
2. Se tiene 2 pagarés uno a 8% de descuento anual
pagadero en 45 días y el otro al 5% de descuento
anual pagadero en 72 días. Si el valor actual de los 7. Determinar el número de puntos de A n B , si
A y B están dados por:
2 pagarés suma SI. 8 500, entonces la suma de sus
valores nominales es: A = {(x, Y)E R2/ Ixl+lyl S 4}
A) 8 585,9 B) 8 590,8 C) 8 875,0
D) 9444,4 E) 10119,0 B = {(x, Y)E R2/ Ixl-Iyl <: 4}
A) Un punto D) Ocho puntos
3. El número mam(s) expresado en base "a" es
B) Dos puntos E) Infinitos puntos
x3x . Indique cuántas cifras tiene en el sistema
C) Cuatro puntos
binario.
A)4 B) 5 C) 6 D) 8 E) 10 o
8. Sea el número R = n 00 ... 0 = 11+ 6 , halle la
~
4. Para construir un puente de 600 m se ha con- 11 veces
tratado 30 obreros para trabajar 12 días en jorna-
suma de las cifras del número (R - 4) .
das de 10 horas. Pero una nueva decisión técnica
exige que el puente sea de 900 m, para ello se con- A) 40 B) 42 C)44 D)46 E)48
trata 6 obreros más.
¿En cuántos días se construirá el puente con los 36 9. Se tiene 7 cajas que contienen bolitas que pe-
san exactamente 20 gramos. Se obtiene informa-
obreros en jornadas de 6 horas diarias?
ción que, por error, una de las cajas contiene bolitas
A) 15 B) 20 C) 25 D) 30 E) 35 que pesan 19,7 gramos. Con una sola pesada, se
desea descubrir la caja con las bolitas más livia-
5. Sea U(N) la última cifra del entero no negativo N. nas. Procedemos de la siguiente forma: se enu-
Si x = U(A + B) , entonces de las expresiones: meran las cajas, del 1 al 7. Luego se toman k
bolitas de la k-ésima caja, obteniéndose en total
1. x = U(A) + U(B) 28 bolitas. Si el peso de las 28 bolitas fue de 558,2
II. x = U(A + U(B)) gramos, entonces el número de la caja con las
bolitas más livianas es:
III. x = U(U(A) + U(B))
A) 1 B) 2 C)4 D) 6 E) 7
Son correctas:
~~;;::=;=ii=::!~~;:::;1 UN12004-11 MATEMÁTICA 1¡:;~~::2:!~;::ir::::¡;¡.li#ii@;W.
entonces se puede decir que:
10. De las acciones compradas por un financista
el 40% son acciones A y su cotización de compra A) s; converge a O
fue de SI. 5,50; el 45% son acciones B y su cotiza-
ción de compra fue de SI. 12,00; yel 15% son ac- B) S/1 converge a 1
ciones Cy su cotización de compra fue de SI. 16,00. C) S/1 converge a 2
Si la cotización de estas acciones han variado de
manera que las acciones A se han incrementado D) S" converge a n
en 80%, las acciones B se han incrementado en 25%
y las acciones C se han incrementado en 12,5%; E) S" diverge
entonces la cotización promedio (en %) de sus ac-
ciones se ha incrementado en: 1 5. Determine la base a tal que
A) 28 B) 34,1 C) 39 D)45,1 E)77,1 log" m = -21
11. En una exposición en el Museo de Arte de
1 1 1
París, se van a colocar en línea 3 cuadros de Picasso, A) 243 B) 81 C) 27
4 cuadros de Rembrandt y 2 de Van Gogh. ¿De cuán-
tas maneras pueden ser ubicados los cuadros, de 1 1
modo que los de Rembrandt se encuentren siem- D) 9 E) 3
pre juntos?
16. Sea X un conjunto no vacio y R e P( X) un
A) 288 B) 1 728 C) 2880
subconjunto no vacío del conjunto potencia de X. R
D)17280 E) 36288 es un anillo de conjuntos sí para cualquier par de
elementos A y B en R se cumple:
12. El producto de las raíces reales de la ecuación
A v B E R Y A \ B E R.
~ x2 + 3x + 6 - 3x = x2 + 4 Si R es un anillo de conjuntos. Indique el valor de
es: verdad de las siguientes afirmaciones:
A) -2 B) -1 C) 1 D) 2 E) 3 I.At:"BER

13. Asuma que la funciónj, dada por. 11. A (\ B E R


III. 0 E R
1/']1/2
j(x) = [ x + 2a[ x + 2a[x + ...]1/2J -
A) VFF B) FVF C) VVV
está bien definida (los puntos suspensivos indican D)VVF E) VFV
un proceso infinito). Entonces también podemos
(1 + i tan8)?
escribir. 17. El número complejo Z = cos78 + i sen7fJ
A) j(x) = 2a + x es igual a:

= 2~a2 + x2
B) j(x)
A)cos1(8) B) cos(*) C) cos(78)

C) j(x)=a+~x2+a2
D)tan7(8) E)sec?(8)
D) j(x)=a-.Jx+a2
18. El valor de! determinante de
E) j(x) = a + ~a2 + x
a2 a 1
14. f),'dd .a sucesión de término general 2
F=lb b 1
S" =~-.fñ. c2 C 1
UNI 2004-11 MATEMÁTICA rw;:::¡~· .~§~ª""'N
G Z

es:
A) 8 - 4..f2 B) 12 - 8..f2 C) 12 - 4..f2
A) (a-b)(b-c)(c-a) 3 3 3
D) 8 + 4..f2 E) 12+ 4..f2
B) (a-b)(c-b)(a+c)
3 3
C) (b-a)(b+c)(a-c)
22. Halle (en m 2) el área de la región sornbreada
D) (a+b)(b-c)(a-c) en la figura:

E) (a -b)(b -c)(a -c)

19. Si:

xy =6' ~=8 . ~=6


5x+4y '3x+2z '3y+5z .

Determinar el valor de: E = ~


x -z
A) 36 B) 56 C) 62 D) 72 E) 87
A) 5 B) 15 C) 10 D) 25 I!) 25
2 2
23. En un triángulo ABC donde A C - AB = 13 m ,
la mediatriz de BC corta a la prolongación de la
20. Dados los siguientes polinomios: P(x) de gra- bisectriz del ángulo BAC en P. Calcule la longitud
do 2 y término independiente uno; y de la proyección de BP sobre AB en metros.

Q(x) = (x -l)P(x)+ 3x + 1. Si Q(2) =7 Y A) 4,5 B) 5,0 C) 6,5 D) 11,0 E) 13,0

P(l) = 2 . Halle la suma de raíces de Q(x) . 24. Se tiene un tetraedro de vértices V, A, B, C,


donde m <l: AVB = 90° , m <l: AVC = 30° ,
A) O B) 3 ª C) 10
3
D) 4 E) 5 m <l: BVC = 75°.
Si Y es el ángulo diedro que forman los planos
21. La figura, ABCD es un cuadrado de lado 4 u. AVC y BVC, entonces el valor de cos Y es:
M es punto medio del lado Be. Los puntos P y Q
pertenecen a MD Í\ AC y AC Í\ e
respectivamen- B) 3 - 2J3 C) 1-..f2
te, donde e
es la circunferencia que tiene por ra-
dio alIado AB. Calcule PQ.
E) 1- J3

25. En un triángulo ABC se traza la altura BH,


luego se trazan HP y HQ perpendiculares a los
ladosAB y BC respectivamente. Si m <l: BAQ = 51° .
Halle la medida del m <l: PCQ .

A) 36° B) 39° C) 49° D) 51° E) 56°

26. En la gráfica se muestra un cilindro circular


recto, donde AH = 2(HB) = 6cm, B punto medio
de la generatriz EC y AC diámetro de la base.
UN12004-11 MATEMÁTICA I ~~\mx
~%!:, ..

Calcule el volumen del cilindro en cm3 .


A) 4· 64 B) 5· 80 C) 6· 16
, 9 ,9 , 3
E
D) 7· 32 E) 8· 16
, 9 , 9

31. El área proyectada de un cuadro sobre un pla-


no que pasa por una de sus diagonales es de 18,6
cm2, si el ángulo formado por dichas superficies
es de 53°. Halle el área (en cm2) del cuadrado.
A) 30,0 B) 30,5 C) 30,75
D) 31 E) 31,5
A) 64/3n B) 69/3n C) 72/3n

D) 78/3n . E) 81/3n 32. Dado el conjunto

5 = {(x, y)/2x + 3y ~ 6, x ~ O, Y ~ O(x ,y)}


27. Una esfera cuyo radio mide 3 cm, está inscri-
ta en un cono circular recto. Se traza un piano tan- y el punto P = (2,3) . Cuál de las siguientes
é rec-
gente a la esfera y perpendicular a una generatriz tas separa a P de 5?
del cono. Si el plano dista 1 cm del vértice del cono,
el valor de la superficie total del cono en cm/ es: D) Y = .lx+~
16 16
A) 90 n B) 92n C) 94n
9
D) 961t E) 98n E) Y = x-
4
28. En un prisma triangular regular se inscribe un 1
C) y = -x + 1
cilindro. Si Ap es el área lateral del prisma, Ac es el 8
área lateral del cilindro y Ap = tAc entonces t es
33. En un triánguloABCse tiene m ~ B = 2m ~ C
igual a:
y 7(AH) = 4(BC) , donde AH es la altura relativa
A) 3/3 B) 4/3 C) 3..fi al lado BC(H E BC); calcule la cotangente del
rt n rt ángulo C.

D) 6/3 E) 2../6 A) .J97 -7 D) .J97 + 7


n rt 12 4

29. Tenemos un triángulo con ángulos interiores B) .J97 + 7 E) .J97 +7


a, 2a, 2a y lados b, a, a. Entonces el valor de % es: 12

A) -15 B) 2(-15 -1) C) 1+ -15


C) .J97-7
2
D) .J2 E) .J2 + 1 . ( e - -2
34. SI ese
5n) =
--15
2 .calcular el valor de:

30. En un cono de altura 16 cm y radio 9 cm se


inscribe un cilindro de radio 1: Determine el radio y ~tf=
(l3n e J
-seI1T- Icos(371t_e)1
'·1 ' 2
la altura del cilindro de mayor volumen si sabemos
que tiene radio entero .
-I5+cse ~1t
(
-e J \
/

G eZ
UN12004-11 MATEMÁTICA r.~~~~~'~"'~·=~=
<8>
A) -4JS B) -2>15 C) 2
2
D)
-2m
75 75 3 A) -3( 1 +mm )
1+m2
D) JS E) 3JS 2

35. Para qué valores de x E (O ; 21t) se cumple


1
B) -2( +mm )
E) -C+ m
m2)

senx + senx cosx < 1+ cosx + cos2 X .

A) (O; 21t) D) (1t; 21t) C) -e +mm


2
)

B) {0;21t}-{~} E) (3 1t; 21t)


2 39. De la figura mostrada calcule" tan a".

C) (O; 1t) - {~} Si tane = -32 y tane = ~


3b+ 2a

36. Sea la función! definida por:

!(x)=-cotx+ 2 2 )1/2 ,
( sec xcsc x-4
P-----~----~~c
x E (-1t ., -31t)
4 .
Halle el rango de f.

A) [-1 ; O} B) (-1; O] C) (-1; O)


D) [-1; O] E) (-1; 1)

37. Halle la medida en el sistema sexagesimal de


un ángulo mayor de una vuelta, si en la siguiente
ecuación R representa el número de radianes que A) 3 C) 5 D) 6 E)"7
mide dicho ángulo.
40. En la figura mostrada se tiene a = ~ . Se sabe
~+rf=5 que una rueda de 0,2 cm de radio de 45 vueltas

A) 3900 B) 4050 C) 555 0 para ir deA hacia B. Calcule el área (en cm2) del

D) 625 0 E) 810 0 sector circular AOB.


o
38. En la figura mostrada, halle tan e + cot e, sa-·
biendo que m> O .
y

A B

A) 3861t 13) 3881t C) 4681t


D) 4861t E) 4881t
UNI 2004-11 MATEMÁTICA

SOLUCIONARIO
1. Del enunciado: Valor actual del segundo pagaré:
Va2 = Vn2 - Descuento
1 000 < AABB < 9 999
5 72
~1 000 < .J AABB < 9 999 = V,'2 - 100 x 360 X V,'2
31,62 < XX < 99,99 - 99 V.
- 100 n2 ... (2)
Es decir, los valores qe puede asumir la raíz cua-
Sumando (1) y (2):
drada este número se encontrará en el intervalo
anterior; y además por ser un cuadrado perfecto, 9 9
Val + ~2 = 1 ;0 v,,1 + 1 ;0 V,'2
será un número entero:
XX = 33 ; 44 ; 55 ; 66 ; 77 ; 88 ; 99 De (*): SI. 8500 = :;O(V,'l + V,'2 )
Al elevar el cuadrado, este número debe tener la
=:} Vn1 + V,'2 = 8585,858
formaAABB
= 8585,9
Probando cada uno de ellos:
Clave: A
(33)2 = 1089 ; (44)2 = 1936 ; (55)2 = 3025
(66)2 = 4356 ; (77)2 = 5929 3. Del enunciado:
mameS) a <5 ... (1)
(88)2 = 7744 ... ¡Cumple la condición!
:=}

Luego: A =7 B = 4 Y X = 8 X3X(a) =:} a >3 ... (2)

Finalmente se pide calcular: De (1) y (2) se concluye:

A+B+X=7+4+8=19 a=4 ... (3)


Por condición del problema:
Clave: O
mam = X3X(a)

2. Valor nominal (V" ) : Es el valor monetario que De (3): m4ms = X3X(4)


asume un documento comercial para ser cancela- 25m + 20+ m = 16x + 12+ x
do en una fecha futura. 26 m + 20 = 17 x + 12
Valor actual (\{¡ ) : Es el valor que se paga por un 26m+ 8 = 17 x
documento al hacerlo efectivo antes de su fecha de
Por teoría: m < S Y x < 4 'ílm , x E N
vencimiento.
En el problema se tiene la suma de los dos valores Si m =1 ; x = 1:
actuales de los pagares:
26(1) + 8 = 17(1) ¡No cumple!
\{¡l + \{¡2 = SI. 8 500 ... (*)
Si m = 1 ; x = 2:
Valor actual del primer pagaré:
26(1) + 8 = 17(2)
\{¡1 = v"l - Descuento
34 = 34 ¡Cumple!
= v.n1 - --ª-- x ~x
100 360
V.
nl Luego:
X3X(4) = 232(4)
= 22. V.
100 nI ...
(1)
UN12004-11 MATEMÁTICA J~~~~~~~
En el sistema decimal: Entonces:
2x4 2
+3x4 1
+2 == 32+12+2 V(A)=a osa s 9} +
== 46 V(B) = b 0~b~9
En el sistema binario. O $ a + b ~ 18 ... (4)
46 2
Además se tiene:
00 23
O 01 11 2 x==U(A+B)
ls2
1 2 2 De (2) Y (3): ==U((tO+a)+(1°0+b))
O 1
~ o
=U(10+a+b)
~ 46 == 101110(2)
De (1): =U(a+b) ... (5)
En el sistema binario el número tiene 6 cifras. Por condición del problema (*): O ~ x $ 9
Clave: C Analizando las tres alternativas:

4. Según el enunciado:
1) x = U(A)+U(B)
=a+b
600 m 30 obr. 12 días 10 horas
900 m 36 obr: x días 6 horas Por condición: O $ a + b ~ 9 ; pero por la expre-
sión (4) se deduce que no siempre se cumple.
Aplicando el método de las proporciones:
(INCORRECTA) .
600 m 30 obr __ lO h-12 días
900 m X 36 obr __ 6 h __ x (días)
II) x = U(A + U(B))
o

Operando: =U((10+a)+b)

(600m)(36obr)(6h)(x) == (900m)(300obr)(lO h)(l 2 días) ==U(a+b)


Por la expresión (5) se deduce que esta es
(900m)(30 obr.)(10h)(12días)
~X==-'-----::-:'::''::-''''''''''-::-7-'-:---:-7-:-:-'-----'- CORRECTA
(600 m)(36 obr.)(6 h)
I1I) x = U(U(A) + U(B))
== 25 días
=U(a+b)
Clave: C
Por la expresión (5) se deduce que es CORRECIA.
5. Por condición del problema:
Clave: E
°
N == ... mnop == 10 +P
Donde: O~ P s 9 ... (*) 6. a , by r son enteros positivos menores que 10
Ira. Condición:
Del enunciado:
Si en la división:
o
U(N) == P == U(10 + p) ... (1)
a+blb
Además si tenemos:
rf3r"
~ 3r(b)+r==a+b
... (2)
~ a = 3rb + T - b ... (1)
... (3)
Por propiedad: r<b ... (2)
UNI2004-11 MATEMÁTICA l::::t~~===:=:':
~~'-l. z
2da. Condición:
y
b es número primo menor que 10:
b=2,3,S,7 ... (3)
3ra. Condición:
a> lSr
De (1): 3rb+r-b>lSr
r(3b-14»b ... (4)

Dado que: b > O 1\ r > O:


~ 3b -14 >O Analizando el conjunto:
B = {(x; y)/Ixl-Iyl ~ 4}
~ b > 14 = 4 6
3 '
Si: x>Ol\y>O ~ x-y~4
Luego podemos deducir que "b"puede asumir los
valores de 5 y 7 . (L2: y=x-4)
En la expresión (4): x>OI\Y<O ~ x+y~4

Si b=S ~ r(3(S)-14»S (L1 : y = -x + 4)


x<OI\Y>O ~ -x-y~4
r>S
(Por la propiedad (2) es absurdo) (L4: y = -x -4)

Si b=7 ~ r(3(7)-14»7
x<OI\Y<O ~ -x+y~4
(L3: y = x + 4)
7r > 7
r > 1

Si r asume valores: 1 < r < 7, cumple:


Clave:
E
x
7. Analizando el conjunto:
1.,

Analizamos la inecua :"11' Ixl + Iyl ~ 4 De los dos gráficos se puede deducir:

Si: x > O /\ Y > O ~ .- y :o:: 4 AnB={(-4;0); (4;0)}


11,,: y=-x+4) Clave: B

8. D, .. .nciado:
x > O /\ Y < 1" ~ X y:O:: 4 o
(L2: y =x - 4) R = n '--.r--'
00 ...0 = 11+ 6
11 veces
x c O /\ y c- O ~ -x+y:O::4 o
=nx10" =l1+h
(L3: Y' x+4) o
= n X (11 - 1)" = 11+ 6
x c O \ y eO =? -x-y:O::'
o o
(1.4: y=-x-4) =nx(11-1)" =IT+6
~~""1<f11Hij ••r '~135
UN12004-11 MATEMÁTICA,: ",?,,:;,,'._ ~
.•.
Para resolver la ecuación se presentan Jlb casos: Diferencia de peso de las bolitas:
• Cuando n es par: D = 20 g/bolita - 19.7 g/bolita
o o ...(1)
= 0,3 g/bolita
n(11-1)" =11+6
o o El peso de las 28 bolitas sin error det .ría. ser:
n(11+1) =11+6 ... (Propiedad)
o o P = (20 g/bolita) x (28 bolitas)
n(11)+n = 11+6
= 560 g ... (2)
o o
TI+n=11+6 Pero por dato se sabe que el peso de las 28 bolitas,
n=6 incluidas las que tienen error:
• Cuando n es impar: Pe = 558,2g ... (3)
o o
n(l1-l)" =11+6 La diferencia de estos dos pesos:
o o P - Pe = 560g - 558,2g
n(11-1) = 11-11 + 6
= 1,8g ... (4)
o o
n (11- 1) = 11- 5 Teniendo en cuenta la diferencia de peso de una
o o bolita con error (1); el total de bolitas con error
n(IT)-n = 11- 5 será:
n=5 P-Pe
Te=-D-
Se obtiene dos valores de n, por tanto existen dos
valores de R. De (4) Y (1): 1,8g
0,3 g/bolita
n=6 R=6x106=6000000
= 6 bolitas
=> R - 4 = 6000000 - 4 = 5999996
La caja con las bolitas más livianas será la No. 6.
¿ cifras = 5 + 9 + 9 + 9 + 9 + 9 + 6 = 56 ... (1) Clave: D

n=5 R=5x10s=500000
10. Del enunciado:
=> R-4 = 500000-4 = 499996
¿ cifras = 4 + 9 + 9 + 9 + 9 + 6 = 46 ... (2) Acciones compradas

En las alternativas de respuestas sólo se considera la


% de Cotización Incremento de
correspondiente a la (2). Tipo
acciones compra SI, cotización (%)
Clave: D
A 40 5,50 80

9. Según el enunciado: B 45 1200 25

Bolltasl (1) (2) (3) (4) (5) (6) (7)


e 15 16,00 12,5
extraídas]" o 00 000 0000 0°0'6 ggg 80°&

m• Ii• mi• Ii• a• IIHfl


1ra.
caja
2da.
caja
• •
3ra.
caja
4ta.
caja
5ta.
caja
6la.
caja
7ma.
caja
Cotización promedio
pradas inicialmente:
de todas las acciones com-

5
La suma de las bolitas que extraen de la cajas: Cp = 16°oX(5/.5,5)+ 16 0x(5/.12)+ 11ío(5/.16)

T = (1 + 2 + 3 + 4 + 5 + 6 + 7) bolitas = 5/.2,2+ 5/.5,4 + 5/.2,4


= 28 bolitas = 5/.10
UN12004-11 MATEMÁTICA

Promedio de incremento de cotizaciones, de (*): 12. Del enunciado:


(S/.2,2)(80) + (S/. 5,4 )(25) + (S/. 2,4 )(12,5) ~ x2 + 3x + 6 - 3x = x2 + 4
p¡ =. 100
= S/.3,41 ~ x2 + 3x + 6 = x2 + 3x + 4 + 2~ - 2
o
El incremento promedio con respecto al promedio
de compra inicial: ~ x2 + 3x + 6 = (x2 + 3x + 6) - 2 ... (1)

1 = S/.3,41 = O 341
Si hacemos: y = ~ x2 + 3x + 6 ... (2)
p S/.10 '
En la ecuación (1) tendremos:
En % : (%)Ip = 34,1%
y = y2_2
Clave: B
y2-y-2=0'

11. Para ordenar elementos a objetos en una fila, (y+ 1)(y-2) =O


se tiene varias formas, las cuales se pueden cuanti- ~ Y +1=O /\ y- 2 = O
ficar ejecutando una permutación.
y=-l y=2
Pn = n!
La ecuación (1), tendrá soluciones reales si y = 2.
Donde n es el número de elementos a ordenar o per- En (2):
mutar.
Del enunciado se tiene los cuadros: 2 = ~x2 + 3x+6
Picasso P1 4 = x2 +3x+6
Rembrandt: R1 x2 +3x+ 2 = O

Van Gogh : VI V2 (x + 2)(x + 1) = O

Para ordenar todo los cuadros de "Rembrandt" se ~ Xl = -2 x2 = -1


considera como si fuera uno sólo, pues estos siem- El producto de estos dos raíces reales de x:
pre deben estar juntos, es decir los cuadros a orde-
nar serán: Xl xX2 = (-2)(-1) = 2

Clave: O

Números de elementos a ordenar: n = 6 ; luego: 13. En la expresión:


P6 = 6 = 6 x 5 x 4 x 3 x 2 x 1
1
I . 112
,720
f(x) = ~x + 2a[x + 2a[x + ...]1/2J ... (1)

Pero los 4 cuadros de "Rernbrandt", si bien es cierto Si la función esta bien definida se cumple que:
que van juntos, también pueden ser ordenados en-
f(x)~O ... (2)
tre si. El total de maneras de ordenar será:
Elevando al cuadrado los dos miembros de la expre-
P, = 4! = 4 x 3x 2x 1
sión (1):
= 24
El total de maneras de arreglar todos los cuadros, (J(x)) 2 I
=x+2a~x+2a [ x+2a[x+ ...]112JI12
entonces estará dado por:
¡(x)
T = P6 X P4
~ f(x/=x+2af(x)
=-720x 24
= 17 280 f(x)2 - 2af(x)- X =O
Clave: O


UNI 2004-11 MATEMÁTICA I
Resolviendo: 16. Si para \tA;B e R se cumple:
-( -2a) ± ~(_2a)2 - 4(1)( -x) A'..JBe R ... (1)
f(x) = 2(1)
Luego cumple que:
2a±~4a2 +4x (A-B)e R ó ... (2)
2
(B-A)eR ... (3)
=a±~a2+x
También se cumple:
Por (2) se deduce que: ... (4)
0eR
f(x)=a+~a2+x Analizando las alternativas:
Clave: E 1) Por definición:
P t:J.Q = (P-Q)u(Q -P) ... (*1)
14. Dada la sucesión de término general: Considerando (2) Y (3), podemos reemplazados en
S" =./n+1-.fñ la expresión (1): A - B por A, y B - A por B, es
decir:
Para hallar su convergencia aplicamos límites a la
AuBeR
sucesión:
(A - B) u (B - A) e R
limS" =./oo+l-~ (Indefinida)
De (*1) : At:J.Be R (Verdadero)
Para salvar su indeterminación aplicamos el artifi-
11) Por definición:
cio al término general de la sucesión:
PnQ = P-(P-Q) .,. (*2)
S =(./n+1_.fñ)x.Jn+1+.fñ
" ./n+1+.fñ
En (2) reemplazamos B por (A - B) :
(n+1)-n
A-Be R
./n+ 1+.fñ
A-(A-B)e R
1
./n+1+.fñ De (*2): A (1 R e R (Verdadero)
Ahora aplicamos límites:
111)Por (4) se puede afirmar que:
limSn = 1
n-+~ ./oo+l+~ oe R (Verdadero)
1 Se concluye que 1, II, III son verdaderas.
Clave: C
=0
Clave: 17. Por definición:
A
Izie9 = Izl(costl + i sell8) ... (1)
1 5. Aplicando la definición a la expresión:
(lzle9)" = [z]" (cos n8 + isen n8) ... (2)
log .,[27=-1..
a 2 En la expresión:
1
a-Z=.,[27 (1 + i tan8l
z = -co-'-s:::-78;:-+--:-i
-se-'-n-=7-:::-8
~=.,[27
s:
~ a=~
27
Clave: C
3xy = 4yz

z =lx ... (1)


4
Reemplazando (1) en (Il):
(cos6 + isen6/
cos76 . x(%x) =8
cos76 + isen 76
cos76 + isen76 3X+2(%X)

De (1) y (2): cos76


%x2 = 8[3X+~X ]
cos76 + ise76
=_1_ 3x2 = 16(9x)
cos76
x2 = 48x
= sec7 e x(x-48) =O
Clave: E
~ xl = O ; x2 = 48 ... (2)
18. Reemplazando (2) en (1):
a2 a 1
z=1(48)=36
4 ...
(3)
i"b'" 1
c '><c>-:1
2
= a2b + b2c + e2a -e2b - a2e Reemplazando (2) en (I):
a 2>-::a>-: 1
- b2a + abe - abe
'-----v-----' (48)y =6
b2'" b"1 o 5(48)+4y
= ab(a -b) -ae(a - b) 48y = 6(240+4y)
+ e2 (a - b) - be(a - b) 24y = 1440
= (a -b)(ab -ae +e2 -be) Y = 60 ... (4)
Finalmente calculamos el valor de la exoresión:
= (a -b)[a(b -c)-e(b -e)]
E =---L-
= (a-b)[(b-e)(a-e)] x-z
= (a -b)(b-e)(a -e) _ 60
De (2), (3) Y (4):
- 48-36
Clave: E =5
Clave: A
19. En el sistema de ecuaciones:
20. Del enunciado:
~=6 ... (I)
5x+4y P(x)=Ax2+Bx+l ... (I)
~=8 ... (Ir) Q(x) = (x -l)P(x)+ 3x + 1 ... (Ir)
3x+2z
~=6 ... (llI) Datos: P(l) =2
3y+5z
Q(2) = 7
Igualando (I) y (llI)
Operando en (Ir) con x = 2 :
xy yz
Q(2) = (2 -1)P(2) + 3(2) + 1
5x+4y = 3y+5z
Por dato: (7) = (1)P(2) + 6 + 1
x(3y + 5z) = z(5x + 4y)
3xy + 5xz = 5xz + 4yz ~ P(2) = O ... (1)
UNI 2004-11 MATEMÁTICA I::::··~~§§gª
Operando en (1) con x = 2: 21. Del enunciado:

P(2) = A(2)2 + B(2) + 1


De (1): O= 4A + 2B + 1 ... (2)

Operando en (1) con x = 1


P(l) = A(1)2 + B(l) + 1
Por dato: (2)=A+B+1
~ A = -B+1 ... (3)

Reemplazando (3) en (2):

0=4(-B+l)+2B+l

~ B=~
2 Datos:
Reemplazando el valor de B en (3): BM = MC = 2u
AB =4u
A = -(~)+1 =-~ Por pitágoras:
Luego: AC = 4.J2 ... (1)
P(x) = _lx2 + ~x +.1 Por semejanza en los triángulos APD y MCP:
2 2·
Para calcular las raíces de Q(x) debemos igualarlo AP 4
PC ="2
a cero (O):
PC =.!..AP ... (2)
Q(x) = O 2
(x-l)P(x)+3x+l=0 Del gráfico se deduce:
AC=AP+PC
(x -1)[ _~x2 + ~x + 1] + 3x + 1 = O
= AP+.!..AP
De (2):
_lx3+~x2+x+lx2_~x-1+3x+1=0 2
2 2 2 2
=lAP
x(~x2 - 4x -%) = O 2

~ AP=~AC ... (3)


~ xl = O 1\ lX2 -4x-l =O 3
2 2 Del gráfico:
(3x+1)(x-3)=0

~ x2 = -t ; x3 =3 Cálculo de PQ:
AQ=AB =4u ... (4)

Finalmente la suma de los raíces de Q (x) : PQ =AQ-AP


S=xI+x2+x3 =R-~AC
3
=(O)+(-t)+('J De (1) y (4):
=4 -1(4.J2)
=-ª-3 12-8.J2
Clave: B 3
Clave: B
········~f~
GomeZ

22. En el gráfico: áreas de los triángulos FOC y OBC también se en-


B .contrarán divididos en las mismas proporciones:
AOBC = S+ 84 m2

S 84 m2
AOCF=W=2+-2-
En el triángulo ADC:

3x = r(30m2 + 35m2 +%+42m2)


A~------~-------------=~C ... (3)
=r(107m2+~)
Por propiedad se sabe que para triángulos que tienen
En el triángulo DBC:
la misma altura, su área se puede representar así:
b1 xh 4x=r(40m2+S+84m2)
A"'l= -2- = k1b1 ... (4)
=r(124m2+S)
b2xh
A"'2 = -2- = k2b2
Dividiendo (3)..;- ( 4) :

3x r(107m2+S/2)
En el triángulo ABF:
4x r(124m2+S)
2
AD = K(30m )

2
3(124m2 + S) = 4( 107 m2 +~)
DB = K(40m )
=> S=56m2
2
AD _ K(30m ) 3
=> Clave: B
... (1)
DB - K(40m2) 4

También: 23. Graficando según el enunciado y además agre-


gando algunas líneas adicionales.
FO=R(35m2)

OB = R(30m2 +40m2)= R(70m2)

2
FO _ R(35m ) 1
=> OB - R(70m2) 2 ... (2)

Considerando (lj y (2) podemos graficar:


B

Dato: AC = AB + 13m ó
b=a+13m

Si HP es mediatriz de lado BC del triángulo BPC,


entonces este triángulo es isósceles.
A~------~-------------=~C Como del punto P que se encuentra sobre la
bisectriz, y de él se trazan perpendiculares a pro-
. 1.1 segmento J<B,por las áreas FAO y OAB,s. encuen- yección del lado AB y sobre el lado AC, entonces:
tra dividido en 'y" y "2y" respectivamente. Lueg- s
UN12004-11 MATEMÁTICA IZilii~
•.~'~~~~~:

a+x=b-x En los triángulos rectángulo PVR y RVQ, respecti-


b-a vamente:
x=-2-
x2 = a2 + y2 (1)
Por dato: (a+13m)-a
2 z2 = b2 + y2 (2)

=6,Sm Luego, en el triángulo rectángulo PVQ (Dato):


Clave: C
c2 = x2 + z2

24. Graficando según el enunciado: De (1) y (2): =(a2 + y2)+(b2 + y2)

V = a2 + 2y2 + b2 ... (3)


Aplicando ley de cosenos en el triángulo PQR, y
reemplazando de (3):

c2 = a2 + b2 - 2abcosa
a2 + 2y2 + b2 = a2 + b2 - 2abcosa
2
B cosa =_L
ab
Del gráfico:
=-(~Xf)
= -(ctg300)(ctg7S0)
Para medir un ángulo diedro, éste debe hacerse
sobre un plano perpendicular a la arista de las dos = -(ctg300)(tg1S0)
caras. Específicamente, el ángulo diedro de dos
caras de un sólido se obtiene midiendo el ángulo De (1) y (II): = _( .f3)(J6- 12)
que forma las dos rectas trazadas en estas dos ca- J6+12
ras, y que convergen en un solo punto de la arista = 3 -2.f3
que se requiere medir el ángulo.
Clave: B
En el sólido ABCV,trazamos el triángulo PQR,conte-
nido en el en un plano perpendicular a la arista CV. 25. Del enunciado: '
B
V

A~------~--------------~~C
B
Propiedades por aplicar:

e
Por trigonometría:
ctg300 =.f3 ... (1)
2a=.4B
ctg 75° = tg 15° = J6 - 12 .... (II)
J6+12 Propiedad 1 Propiedad 2 Propiedad 3
·14~2======
Corno el cuadrilátero PBQH tiene 2 vértices de án-
gulos rectos y opuestos, podemos aplicar la própíe-
Por relaciones métricas en el triángulo
ABC:
rectángulo

dad 2, y luego también se aplica la propiedad l. AH AC


Los ángulos internos del cuadrilátero se obtiene a AC = AB
partir del triángulo HQC. 6m 2r ." (1)
B "'2'r= 9m
BC =HB
AB BC
h
"2 3cm ~ h=6.J3cm ." (2)
9cm=T
2:.
Cálculo del volumen del cilindro:
~~~----------------~C Vo = nr2h
El cuadrilátero APQC puede inscribirse en una cir-
De (1) y (2): =n(2Jcm2)(6.J3cm)
cunferencia por que cumple con la propiedad 3.
"
D = 81.J3ncm3
Clave; E

27. Del enunciado:

~~--------------~~~C

Aplicando la propiedad 1 se deduce que

x = Sl~
Clave: D

26. Del enunciado:


Dato: r = 3cm
Adicionando algunos trazos auxiliares:

rh

Datos AH = 6 cm ; HB = 3 cm

L
==~~~~~~
El triángulo VQO es triángulo notable (B = 3 7° ) En el triángulo equilátero ABC, por propiedad:
OB = 2r
Aplicando Pitágoras en el triángulo rectángulo HOB

HB = ~(OB)2 - (OH)2

=~(2r/-r2
= .f3r
Por propiedad de triángulo equilátero:
CH = HB =.J3r

CB = 2J3r
~ VO=5cm

11
AC = CB = BA = 2J3/
En el triángulo rectángulo AVH (e = 37° ) se deduce:
Área lateral del prisma:

VH:::i +O:'m <; 8cm


Ap = (AC+CB+
= 3(CB)h
=3(2.f3r)h
BA)h

AH = R = 6cm
= 6J3rh ." (2)
AV=g=10cm
A 6cm H
Área lateral del cilindro:
Cálculo del área total del cono:
Ac = 21trh '" (3)
AT = Abase + ALaterol
Finalmente calculamos el valor de t en(1):
= rrR2 +1tRg

= 1t(6cm)2 + 1t(6cm)(10cm) De (2) (3):

= 961tcm2 Clave: A
Clave: O
29. Del enunciado:
28. Del enunciado: B

~ b IC
Por propiedad:
C'
a + 2a + 2a = 180°
Dato: Ap '= tAc ~ t = Ac ... (1) ~ a'= 36° ... (*)
Ap
Del gráfico: Aplicando primera derivada al volumen del cilin-
dro con respecto a la variable r:
b
De r-» cos2a =1a V'e = 1;1t(9(2)rcm-3r2)
fL=__ I_
b 2cos2a Igualando a cero (O) Ve , para hallar máximos y
1 mínimos:

1;1t(18r cm - 3r2) =O
2( ./54-1) r(18cm -3r) =O
=> r1=0 /\ 18cm-3r=0
=_2_1
./5 -1 => r2 = 6cm
_./5+1 Reemplazando el valor de r = 6 cm en la expre-
--2-
sión ("):
Clave: C
16
h=-(9cm-6cm)
9
= 16 cm
3
Clave: C

31. Graficando según el enunciado yadicionan-


do algunas líneas auxiliares y datos.

! »-:" e
Datos: OB = R = 9 cm; OV = H = 16cm
Del gráfico, por semejanza de triángulos: ~ tia
D AO= oe = OB = -2-
O'B O'V
OD = OV Datos: Área de Sombra
9cm 16cm
r 16cm-h As = 2AMPe = 18,6cm2

~ h=196(9cm-r) ... (") En el triángulo rectángulo OBP:

Volumen del cilindro: V OP = 12 acos53"


2
Ve = 1tr2h
= J} a(~)
De e» =1tr2[1;(9cm-r)]
= 312a ... (1)
= 1961t(9r2 cm-r3)
10
a='§~·:::·"~:"'ri·~·:· =~:~~-; > 1__ u_N_I_2_00_4_-I_1 _M_A_:T_EM_ÁT_I_C_A_

En el triángulo APC:
A rea MPC -- ACxOP
2
Ahora debemos verificar cuando x = 2 ; ésta debe
2 J2ax 3.J2a pasar entre las ordenadas ~ y 3 de los puntos Q y P.
18,6 cm 10 3
2 2 Para la alternativa A:

=) a2 = 31 cm2 ... (2) Y=i(2)+*


Área del cuadrado ABCD: = 13
De (2): 4
= 3,25 (no cumple)
Clave: D
Para la alternativa B:
32. Del enunciado: Y=.!.(2)+2.
4 4
S={(x;y)!2X"+3y::;6 x zo y~O}
11
P=(2;3) 4
Graficando: = 2,75 ¡Cumple!
y Clave: B

33. Del enuciado:


L A

X
~
Ls:y=-1x+2 C m H n B
Del gráfico:
'La recta que separe P de S, será la que corte al eje Y
entre las ordenadas 2 y 3 ; Y también pase entre los • ctg2a=7f =) m=hctg2a ... (1)
puntos P(2; 3) Y Q(2 ; Yo) .
Calculo de la ordenada del punto Q, con la ecua- • ctga=* =) n e h ctg a ... (2)
ción de la recta Ls:
Por condición del problema tenemos:
y = -~x+2
4(CB) =7AH
2
Yo = -3(2)+2 4(m+n)=7h
2 De(1)y(2): 4(hctg2a+hctga)=7h
3 g2
4h [(ct 2ctga a-1) +ctga ] =7h
=) Q=(2;i)
g2
Por teoría se sabe que en la ecuación de una recta 4[ct a -1 + 2ctg2a] =7
Y = mx + b , el término independiente b es el valor 2ctga
de la ordenada cuando se intercepta con el eje Y.
2ctg2a-2+ 4ctg2a = 7 ctga
Analizando cada una de las alternativas las que cum-
plen con la condición de interceptar al eje entre 6ctg2a-7ctga-2=0
las ordenadas 2 y 3 son:
14,~6 5====~ UN12004-11 MATEMÁTICA 1-=~====':':'li!lIIII=~
'; G~iñez

,
cota=
7±~49-4(6)(-2)
2(6)
_sen(131t - e)
:=}
M = 2 Icos(371t - e)1
7±m rs +csc(2i1t -e) 2
12
Del gráfico se deduce que: ctg a > O sen(e-~) lcos(e 371t)1
.:=} ctga= 7+m rs-csc(e-2i1t) 2
12
Clave: B sen(e - 131t + 61t)
__ --'-:---=2::-:-_-.:' '-'....,.1
cos (e- 371t + 181t)1
rs -csc( e - 2i1t + 101t) 2
34. Del enunciado: csc( e - 5;) = -;S ... (*)

sen( e - ~) Icos(e 1t)1


Reduciendo:
rs -csc(e-~) 2

-2

- De (*): csc(e-~)= -;s


=
-;S) I+JSI
. rs - ( -7s
5

1t) =
sen ( e--
2
1
csc(e-~)
-2
=-
rs =~(~)
= -4rs
Graficando: 75
y Clave: A

35. Operando en la inecuación:

sen x + sen x cosx <1 + cos x + cos 2


X

2
senx(l + cosx) - (1 + cosx) - cos X < O
x senx(l + cosx) - (1 + cosx) - (1- sen2 x) < O

(1 + cosx)(senx -1) - (1 + senx)(l- senx) < O


(1 + cosx)(senx -1) + (1 + senx)(senx -1) < O
(senx-1)[(1+cosx)+(1+senx)] <O
(senx -1)[2 + cosx + senx] < O
~
rs (+)
5 :=} senx - 1 < O

Luego: cos(e _2!)2 = ±f!.: = ± rs5 senx < 1 ... (1)


Por definición se sabe que:

sen(e-~)= 1s Por condición del problema:


-T s sen e s ! .., (2)

Reemplazando los valores obtenidos en la expre-


s.ón: x E (O; 21t) .., (3)
.:.&.:

~
GomeZ
UN12004-11 MATEMÁTICA I
Considerando O), (2) Y (3) gráficamos:
ctg(-1t) = 00

y senx
~ 1 < etgx < 00 000 (2)
De O) y (2) se concluye que:
tg x < etg x
x
tg x - ctg x < O
-, Luego en la expresión ("):
XE (O; 21t)-{~} f(x) = -ctg x - (tg x - ctg x)
= -ctg x - tg x + ctg x
Clave: B
= -tg X
36. Identidades trigonométricas Considerando la expresión O) concluimos:
see2e = tg2e+ 1 0< tg x < 1
ese2e = ctge+ 1 -1 < -tg x < O
tg x, ctgx = 1 ~ Ram f(x) = {y/ -1 < Y < O}
En la función:
Y E (-1 ; O)
f(x) = -ctg x + ( see 2 X ese 2 x -4 )1/2
o

Clave: C

=-ctgx+~(t~2 x+1)(etg2 X+1)-4 37. En la expresión

= -etg x -+ j(tg2 x ctg2 x+tg2 x +ctg2 x+ 1)-4 p!+pj=s


= -ctg y + ~(1 +tg2 x+ctg2 x+ 1)-4 2~ 3a 5 + = O)

a
000

=-ctgx+~tg2 x-2+ctg2 x
= -etgx+~tg2 x-2tgx o ctgx +ctg2x
Si hacemos: ~ =n ~ =~ 000 (2)

Luego en la expresión (1):


= -etgx+~(tgx _ctgx)2

=-ctgx+ltgx-ctgxl 000(*)
2(n)+3(~)=S
2n2-Sn+3=0
Por dato del problema x E (-1t ; -!1t)
(2n - 3)(n - 1) = O
y
~ n - 3 n2 = 1
1 -2 ,o

Reemplazando estos valores en la expresión (2):

~ = i ~= R 9
4
1t 000 (3)
x
~=1 ~ R=1t 000 (4)

Por condición del problema R representa un ángu-


-1 lo mayor de una vuelta, es decir:

tg( -1t) = O 91t 9(180°)


R> 21t ~ R=-=---=40So
4 4
~ 0< tg x < 1 000 (1) Clave: B

(
~~=~;;===~[~U~N~122~00~4~-I[1
~~
~M~A~:r~EM~ÁT~I~C~A:'=====--'-W\s;
~ G~iñez

38. y En el triángulo AEC:

De (1): 2a 2
tg8=-=-
y 3
2(6) _ 2
De (2): --Y-"3
~ y =18 ... (3)

Del gráfico:
AB = x +y
Por definición, en la ecuación: y = mx 4b = x + 18
tg x =m ... (1) 4(S)=x+18
Las dos rectas son ortogonales entre si: ~ x=2 ... (4)
tg8. tgex =-1 Luego en el triángulo rectángulo BCE:
Oe(1): tg 8. (m) =-1
tgex = 2a
~ tg8=-1.. x
... (2)
m '_ 2(6)
De (2) Y (4): -T
Por trigonometría:

~ ctg 8 = ~ = _1_ = -m =6
... (3)
tg8 _.1 Clave: O
m
Reemplazando (2) y (3) en la expresión pedida: 40.

tg8+ctg8 = -~ + (-m) = -e +mm


2
)

Clave: C

39.
A B

I.,l,---=---~C Dato: ex = ~ ...(l)


3
Por condición, la longitud LAB es igual a la longi-
tud de 45 circunferencia, de radio r = 0,2 cm.
LAB = 45(21tr)
= 45[21t(0,2cm)]
= 181tcm ... (2)
Cálculo del área del sector circular AOB:
A
_ (LAB)2
Por condición del enunciado:
AAOB --za-
2 3a
tg 8 ="3 = 3b + 2a (181tcm/
De (2) Y (1):
a_6 ... (1)
~ ¡;-S 2(~)
Si a = 6 ~ b = 5 ... (2) 61t
Clave: O .
~~[J~~~.~
MATEMÁTICA

MATEMATICA 6. Un número d la forma ab representa la edad de


1. Similannente al caso de los números reales, se dice una persona q aún no alcanza la mayoría de edad.
que la matriz M es la raíz cuadrada de la matriz N si Si en una ba n (n < b) dicho número es capicua,
M2 =N. halle la su a de todos los números ab que cumplen
Entonces, el valor de x para el cual la matriz "1-.ll.LaHt:t:uO r.

[7~6] [1O] A) 15 B) 16 C) 31 D) 32 E)48


x -7 es la raíz cuadrada de O 1 ,es
7 • Determine la verdad o falsedad de los siguientes
enunciados:
A) O B) 3 C)-16
D) 16 E) no existe 1. Si a E Q, entonces a2 E Q

2. Se prestó un capital durante 6 meses, el interés 2. Si a E R/a2 E Q, entonces a E Q


resultó 20% del monto. Qué porcentaje del mon-
é

to se producirá en 2 años? 3.Si la +bl =Ial +Ibl; entonces a, b ~ O


A) 80% B) 60% C)50% A)VVV B)VVF C)VFV
D)40% E) 20% D)VFF E) FFF

3. Un padre deja una herencia a sus 3 hijos. La repar- 8. Un granjero vendió patos a 12 nuevos soles cada
te en partes inversamente proporcionales a los núme- uno y con el importe de la venta solo pudo com-
ros 6, 4 Y 3 empezando por el hijo mayor respectiva- prar pavos a 14 nuevos soles, quedándole 26 nue-
mente. Si el valor de la herencia asciende a 36000 vos soles. ¿Qué cantidad de pavos compró, si se
dólares. ¿Cuánto le corresponde al hijo menor? sabe que el número total de patos vendidos y pa-
A) $ 4 000 B) $ 8 000 C) $ 9000 vos comprados es 182?
D) $12000 E) $16000 A) 63 B) 79 C) 82 D) 83 E) 99

4. Si el número 8abe se divide entre 37, se obtiene 4


9. Si p, q, r.s son números primos, diferentes entre sí,
de residuo. Entonces el residuo que se obtiene al divi-
dir abe6 entre 37 es: tales 20 < P + q < 30,
que: 20 < r + 5< 30,
2 2
p2 +q2 = r +5 entonces, la suma p + q + r + 5
A) O B) 3 C) 13 D) 23 E) 33
es igual a:
5. Luís confecciona 5 docenas de pantalones cada
A) 50 B) 54 C) 58 D) 62 E) 66
semana y Eva 2 decenas de camisas por día. Cuando
Luís termina 3 docenas de pantalones, ¿Cuántas ca-
misas confecciona Eva? 10. La tabla siguiente presenta la distribución de los
trabajadores de una empresa según el tiempo de ser-
A) 70 B) 84 C) 90 D) 100 E) 101 vicio en años.
~~\a:,::::::::":
..
n
:;:í¡::::li[::=U~N~'2~0~0~5-I'~M~A~~]E~M~Á~T~IC~A~I~:OOI:.·::::::::

Tiempo de Número de
servicios (años) trabajadores , 4.$e, 1, matriz X " [ ~ ~ Entonces 1, m,

12-5) 12
triz Xll es:
15-8) 15

18-10) 18 10 O 1024 O 1024]


A) O 1 D) O 1 O
110-15) 12 [ [
10 O 1024 O 1024
115-20) 10
100 O 100] 59049 O 59049]
120-25) 8 B) O 1 O E) O 1 O
[ [ 59049
100 O 100 O 59049
El tiempo de servicios para e125% de los trabajadores es:
A) 5,55 años B) 6,35 años C) 7,10 años
1000 O 1000]
D) 14,82 años E) 15,30 años
C) O 1 O
[
. 11. Para elaborar un exámen de 06 preguntas se 1000 O 1000
dispone de un banco de 05 preguntas fáciles, 04
intermedias y 03 preguntas difíciles. De cuántas 1 5. Dada la ecuación al&ebraica
formas puede elaborarse dicho examen si el núme-
ro de preguntas fáciles debe ser estrictamente ma- x2 -4
--=-x 3
yor que las intermedias y el número de estas a su Ix+31 2
vez.mayor ó igual que las difíciles. Determine el número de raíces reales que posee dicha
A) 30 B) 60 C) 120 D) 180 E) 274 ecuación:
A) O B)l C) 2 D) 3 E)4
12. Sea la sucesión an (n » O) definida por:
all = log p si existe un primo p y un k entero no 16. Dado el diagrama
negativo tal que n = l y u
an =O en cualquier otro caso.
Entonces, la suma de los términos am ' donde m es un
divisor (positivo) de·72, es igual a:
A) log 8 B) log 24 C) log 36
D) log 72 E) log 144

13. Determine el valor de

Sn
()
=Lk=l
11 [2
--+--
4k2 -1 2n + 1
1] de las siguientes afirmaciones
1. A r. e contiene B - D .
2n 3n
A) 2nn+ 1 B) 3n + 1 C) 2n + 1 n. La intersección de B con el complemento de
e -D es 0.
D) 2n-1 n+1
E) 2n +1 III. tff(A) u tff(B) u tff(B r. D) =U
2n+ 1
son verdaderas.
UN12005-1 MATEMÁTICA

A) Todas B) Sólo Il C) Sólo 1 y II 21 . El radio de la base de un co ircular recto


D) Sólo 1y III E) Sólo 11y III mide R unidades y su altura mide H unidades.
La altura del cilindro de mayor área lateral inscrito
17. Sea J una función definida por en el cono es:

J(x) = x - ~ + 1, x < -4 !i !i
A) B) C) 2H D) !i E) 3H
4 3 5 2 4
halle j* (x) (inversa de!) indicando su dominio
22. En un triedro SABC las caras son:
A)j*(X)='¡(.Js-4X+l)2 (-00, -5)
Ese = 90° ; ASE = 60° ; ASe = 60"-
B) j*(x) = -'¡(.J4 - Sx + 1)2 (-00 , - 6) Se toma sobre SA una longitud SM = 2u Hallar el
ángulo formado por SM y el plano SBC.
C) j* (x) = -'¡(.Js - 4x + 1)2 (-00 , - 5) A) 25° B) 30° C) 35° D) 40° E) 45°

D) j*(x) = -'¡(.Js - 4x _1)2 (-00 , - 5) 23. En la figura mostrada, si BC = CD =AD, en-


cuentre x.
E) j*(x) = -'¡(.J4X-s -lf (-00, -6) B

18. El conjunto solución de la inecuación C

A) (-~ 1 D) R D
A) 12° B) 15° C) 18° D) 20° E) 30°
B) R-
~ 2
_l 3J E) R-(-~ 3)
24. En un triángulo ABC se cumple que
AB=BC=10cm y AC=12 cm.
C) R -{-~ 3} Encuentre la longitud, en cm, de la circunferencia
que pasa por los puntos A y C sabiendo que los la-
19. Dada la región dos AB y BC son tangentes a dicha circunferen-
cia.
A={zeC/lz-2-il:S:3 v Iz+2-il:S:3}
Halle z¡ y z2 en A tal que Iz¡ - z21sea el valor A) 5 1t B) 1O 1t C) 15 1t D) 2 O 1t E) 25 n
máximo. De como respuesta z¡.z 2
25. Dadas las siguientes proposiciones:
A)-29 B)-28 C)-26 D)-20 E)-18
1. Si dos planos son para.elos a una misma recta,
20. Determine el conjunto solución de la inecuación entonces los planos SO!1 paralelos entre si.
11. Si se tienen dos rectas que se cruzan, entonces
Ix-21-3Ix+211 < O.
existe siempre una recta perpendicular a ambas.
A) (-00 ; -32,s)u(-ls,2s ; +00) III. Una recta que interseca perpendicularmente a
una de dos rectas que se cruzan siempre interseca
B) (-00 ; -11,5) u (-4,5 ; + 00) a la otra.
C) (-115 ; -4,5) é Cuáles son verdaderas?
A) Sólo 1 B) Sólo Il C) Sólo III
D) (-32,5 ; -15,25)
D) 1 Y II E) 1 Y III
E) (-00 ; -32,5) (-4,5; +00)
UN12005-1 MATEMÁTICA I=:'=:i:=;;'~:;i~::;::!::!~
~=~ z
26. Con base en una de dos rectas paralelas se
A) 3..f3d3~1_4sen2e sen2e
constituye el triánguloABC de base AC = 16 m. Por 2
el punto Q E AB se traza otra paralela a las rectas
determinado en el lado BC el punto R tal que BR y B) :J...f3d2~1_ 4sen2e sen28
RC están en la proporción de 1 a 3. Si el área del 2
triángulo ABC es 192 m2, determine el área del C) ~..f3 d3~1 + 4sen28 sen28
triángulo QBR.
D) ~..f3 d3~1- 4sen28 sen8

E) ~..f3 d2~1 + 4sen2e sen2e

27. En el plano XY se tiene las rectas paralelas 30. A la región plana representada en (a) le falta el
y + 2x + 4 = O e y + 2x - 8 = O , halle la recta puntoA; la de (b) le faltan los puntos Cy D y a la de
equidistante a ellas contenida en el plano XY. (e) le falta su circunferencia frontera. cCuáles de las
siguientes proposiciones son correctas?
A) Y + 2x -1 =O D) Y + 2x - 3 =O
E) Y + 2x - S = O
, -,
B) Y + 2x - 2 = O
A C D , I \
\

,
I

~CJ
I

C) y+ 2x -~ = O I
\
, , I

28. Dos regiones rectangulares congruentesABCD y (a) (b) (e)


ABC'D' . Forman un ángulo diedro cuya medida es I. La intersección de los conjuntos en (a) y (b) es
60°. Si AD = 2AB , calcule la medida del ángulo que un conjunto no convexo.
forman las rectas Bv y AC' . Il. La intersección de los conjuntos en (b) y (e) es
un conjunto convexo.
A) are cos(~) D) are cos(~) III. La intersección de los conjuntos en (a), (b) y
(e) es un conjunto convexo.
B) are cosa) E) are eos(~) A) 1 Y III B) II Y III C) Sólo III
D) Sólo 1 E) Sólo II
C) are eos(~)
31. Para el círculo trigonométrico que se muestra en
29. En un prisma hexagonal regular ABCDEF -
la figura, calcule: y = sen 2cx .
A'B'C'D'E'F' la longitud de la diagonal mayor del
prisma es "d" y la medida del ángulo A' DF' es e . Ha-
llar el volumen del prisma.
B C

C) -~
S
~~ezt~~~~~=~§ ~:.[=:QU~N~12~0~0~5-~1
~M~A~T!E~M~Á~T~IC~A~]:,!IIII1:: :,'J===+':*:'' ':t~~~.
~~
32. Simplifique: 37. Calcule E = tal12 (* + 1) en términos de "a",
3 3 1
K =~_cas X +_ si secx = a + tanx.
sen 3x cos 3x 2
C) 1 D) 1.. E) 1.
A) 3 sen 2x casec 6x D) -~sen2x casec 6x a2 a a
2
B) -3 sen 2x casec 6x E) -sen2x cosec6x
38. Después de haber sido rotado el sistemaXYun
C) ~Sen2X casec6x ángulo a tal que tan a =.l, se obtuvo los puntos
4
A'( -4 ; 6) , B'(2; 4). Si P' es punto medio de
33. En el siguiente gráfico determine las abscisas de
A'B' en el sistema X' y', determine las coordenadas
los puntos A y B.
y de P' en el sistema XY.

A)
1 . 1)
(7'9 . (17 11)
C) -25; 15
x
-rt

39. Q' es la nueva ubicación del punto "Q" al girar la


B) _31t . 31t _ 41t . 41t rueda desde la posición (1) hasta la posición (2).
A) -~. ~ C)
8 ' 8 5 ' 5 5 ' 5 Determine la distancia (menor a 2rrr) que hay entre'
Q y la proyección de Q' sobre el plano horizontal.
D) _ 31t . 31t E) _31t . 31t
(1) (2)
4 '4 2 '2

e < o y sen 8 < tan 8. Al simplificar


34. Si cos

[sen B] ltanel [cor O] se obtiene:


K=--+--+-- ,
sen 8 tan 8 cat e

A) -3 B) -2 C) -1 D) 1 E) 2
A)l,lr B)2,lr C)3,lr D)4,lr sis.r.
35. Si lal ~~ determine el mayor valor que puede
40. SeanAOB, COD y EOF sectores circulares.
sen a tanl2al
tomar E ()a = --:---'-""':
Si la longitud del AB = a ; OE = a . Halle el área de
=l=: 1;1 la región AOB si las áreas de las regiones EOF, ECDF
y ABDC son iguales.
A) Jl8+J6' B) Jl8 -J6 C) m+J6
8../2 2 4 A) a
2
13
5 A
D)J18+J6 E).Jl8 +J6 2
3 2 B) a 13
4
36. Halle el número de cortes de la gráfica de 2
O
f(x) = ex sec x, con la gráfica de g(x) = cosx en
C) a 13
3
el intervalo [-201t , 201t].
2
(Nota: ex secx = secx -1)
D) a 13 B
2
A) 20 B) 30 C)40 D) 60 E) 80 E) a213
OLUOIONARIO
,. Del enunciado: %1(M) = 4x 100%
Sx

[~ ~] = [: -~n2 =50%
'Clave: C
3. Sea a, b y e las partes que se les corresponde él
[~ ~] = [: ~76][: -_In cada hijo:
~ a + b + e = $ 3600 ... (1)
1 O] = [49 -16x 7(-16) + (-16)(-7)]
[O 1 Donde: Parte del hijo mayor :a
7x-7x x(-16)+(-7(-7» Parte del hijo intermedio :b
0 Parte del hijo menor :e
[~ ~]=[49-016X 49_ 16X] Estas cantidades son repartidas inversamente pro-
porcionales a 6, 4 y 3, entonces:
Como las dos matrices son iguales, debe cumplirse:
6a = 4b = 3e
49-16x=1
Dividiendo entre el m.e. m (6 ; 4 ; 3) = 12
x=3
Clave: B 6a = 4b = 3e
12 12 12
2. Por definición se sabe: Se obtiene cantidades directamente proporciona-
M =C+1 ... (1) les:
a b e a+b+e
• Para 6 meses, si asumimos que el mon- "2=3=4=--9-
to MI = Sx , su interés del 20% que generó: a b e $36000
De (1): "2=3=4= 9
11 = I~OO(Sx)= x
Para obtener lo que le toca al hijo menor:
Reemplazando en (1):
f=$36000 ~ c=$16000
MI = C + 11 4 9
Clave: E
Sx = C + x
~ C=4x 4. Del enunciado se deduce que:
• Para 2 años, considerando los datos anteriores °
se construye el gráfico:
Sabe = 37 +4
S X 103 + abe = 3°7 + 4
4x [1 =x Sx [2 =3x Sx
(3°7 +S)+abe =3°7 +4
1 6 meses '\ (IS=3x6) meses '\ °
abc = 37 - 4
C MI M2 °
abexl0 = (~7 -4)10
El interés que se ha producido en 2 años será:
abe x 10 = 37 - 40
°
11 + 12 = X + 3x abcxl0=37 -3
= 4x ... (2) °
abcx10+6=37 -3+6
El interés que se produjo, en porcentaje con respec- °
abe6 = 37 + 3
to al monto, durante los dos años será: Clave: B
UN12005-1 MATEMÁTICA I:=::¡:::::::¡:::;:¡¡·¡m¡¡:· :ti~'-:;·~

5. Del enunciado tenemos: Número: 13: ninguno


Producción inicial de Luís: 14 : ninguno
15: 1111(2) ; 33(4)
P = 5 x (12) pantalones x 1 semana
IL semana 7 días
16: 121(3)
60 pantalones
=7 día ... (1) 17: 10001(2) ; 101(4)
Producción inicial de Eva: La suma de los números que forman números
capícuas en base n son:
PIE = 2 decenas de camisas/día
= 20 camisas/día ... (2) 5=15+16+17
= 48
Luego, si Luísproduce P2L = 36 pantalones, Eva pro-
Clave: E
duce P2E' es decir:
7. Analizando los enunciados:
PlL --? PIE

P2L --? P2E L- Si a E Q, entonces a2 E Q .... (Verdadero)

P2L xPIE
Por la propiedad de clausura de los números
~ P2E = -"-"':=--'-"- racionales en la multiplicación:
PlL
Si a E Q ~ ax aE Q
(36 pantalones) x (20 cam,isas)
2
De (1) y (2): dw 2,- Si a E iR/ a E Q ; entonces a E Q ... (Falso)
60 pantalones
7 día
= 84 camisas
Ejemplo; si ~E Q ~ 1 ¡; IQ

3 ..- Si. la + bl = lal + Ibl ; entonces a, b ~ O ... (Falso)


Clave: B
Ejemplo:

6. La mayoría de edad se considera cuando la perso- Si a = -5 Y b = -4


na tiene 18 años, es decir: I(-5) + (-4)1 = 1-51+ 1-41
ab < 18 ~ a=1 ... (1) 09 = 5 + 4
0<::;b<8 ... (2) (También cumple si a; b s O)

Si n representa la base de un sistema de numera- Clave: O


ción; entonces:
8.Asumiendo que:
n~2 ... (3)
Cantidad de patos: x
Además por condición del enunciado: Cantidad de pavos: y
b>n ... (4)
Dato: x+y=182
De (2) , (3) y (4) se deduce:
~ x=182-y ... (1)
Si cada pato se vende a 12 soles, el total de la venta
será:
~ b=3;4;S;6;7 ... (5)
T = 12 x ; (nuevos soles) ... (2)
Luego considerando (1) y (5), ab puede asumir los
valores: 13 ; l4 ; 15 ; 16 ; Y 17 los que en base Con el importe total "T' de la venta se pueden com-
n < b forman un número capícua: prar 'Y' pavos a 14 nuevos soles, sobrando inclusive
26 nuevos soles, es decir:
UN12005-1 MATEMÁTICA I:=;:r:~:::!:::~::;~t,. G meZ

T=14y+26 Luego los valores que pueden asumir:


De (2): 12x=14y+26 p=7; q=19; r=ll ; s=17
6x=7y+13
=> p+q+r+5=7+19+11+17
_ DeO): 6(182-y)=7y+13
= 54
=> Y = 83
Clave: D Clave: 8

9. Si: p ; q ; r y 5 son números primos de diferentes


10.
entre si. Tiempo de Número de
servicios (anos) trabajadores
Además: 20 < P + q < 30 ... O)
[2-5) 12
20 < r + 5 < 30 ... (2)
[5-8) 15
p2 + q2 = r2 + 52 ... (3)
[8-10) 18
Por teoría se sabe que:
[10-15) 12
Si a es par => a2 es par
[15-20) 10
Si b es impar => b2 es impar
[20-25) 8
=> a2 + b2 es impar ...(a)

Si a y b son impares => a + b es par ... W)


De la tabla, la suma del total de trabajadores:
De la expresión (1) y (2) Y considerando que estos
son números primos, p ; q ; r y 5 pueden asumir los S=12+15+18+12+10+8=75 (trabaj.)
valores: Cálculo del 25% de trabajadores:
2 ; 3 ; 5 ; 7 ; 11 ; l3 ; 19 ; 23 ... (4)
Analizando la expresión (3):
25%S = 12050X75 = 18,75 (trabaj.)
p2 + q2 = r2 + $2
.
'----'
1
~ 11 Considerando que este porcentaje de trabajadores co-
rresponde a los que tienen menor tiempo de servicio,
Si P o q asume el valor de 2, entonces 1 es impar
éste abarcará hasta parte del intervalo de "tiempo de
(a); luego II también sera impar. Los números que
pueden asumir r y 5 son impares (4), entonces por servicio" [5; 8) .
(P), II es par, lo cual es incoherente. Por lo ante- J+------ S -- ...
riormente mencionado, descartamos el 2 como po- 25%5= 18,75Trabaj.
sible valor que puedan asumir: p, q, r Ó 5. 5
Trabaja- __ -..¡. __
Considerando O) y (2) Y (4): dores: 12
Años de
20 < P + q < 30 servicio: 21-------4L-.,--4x---- l--
p2 + q' = ¡2 + .1'2 s 2c .)

20 < r + s < 30 d

3 19 370 Aplica: .do proporciones a "tiempo de servicio" y "tra-


3 23 538 bajadores".
5 17 314 5 d
5 19 386 25%S = 8 - 5
5 23 554 18,75 -12 x - 5
7 17 338 18,75 = 8- 5
7
19J 410J => x = 6,35 (años de servicio)
11 13 290
11 17 410 Clave: 8
-
UN12005-1 MATEMÁTICA [:~. ==~~~E::::
..
~
11. Definición: Si de n objetos formamos grupos
Calculando los am :
de k elementos, de modo que cada uno de los gru-
pos sea diferente del resto por lo menos en uno de
al = lag P 1 = pO P es un número primo
sus elemento, entonces habremos formado una com-
binación. El número de grupos que forman esta dado a2 = log2 2 = 21
por:
a4 = log2 4 == 22
n n!
Ck =(n-k)!k! a3 = log3 3 = 31
Del enunciado: a6 == O ; no existe un número primo que elevado
N° de preguntas fáciles :F = 5 a un K sea igual a 6.
N° de preguntas intermedias :1= 4 a8 == lag2
N° de preguntas difíciles :D = 3 a9 = log3
Por condición del problema se debe tomar 6 preguntas,
las cuales deben cumplir ciertas condiciones de des- alZ =O ; a18 == O ; a24 == O ; a36 == O a72 = O
, igualdades. Al seleccionadas se pueden presentar los
siguientes casos:
Cálculo de la suma de los a", :
F> l ~ D 5m == al + az + a3 + a4 + a6 + a8 + a9 + a12
Caso I 5 1 ° + al8 + a24 + a36 + a72
Caso 11
Caso III
4
4
Z
1
°
1 == logp + log2 + lag3 + lag2 + log2 + log3
Caso IV 3 Z 1 = lag(p x 2x 3x 2x 2x 3)
El número de formas que se puede seleccionar las == lag72p
preguntas (11 cada caso:
Por definición un número primo: p >2
Caso 1 : C~ x ct x C6 = 1 x 4 xl = '4
Si p==2 ==} 5m==log72p==log144
Caso II: C! x ci x C6 = 5 x 6 xl = 30
Obviamos calcular para valores p > 2 , pues éstos
Caso III: C! x ct x cl = 5 x 4 x 3 = 60 nos darán resultados mayores a los que se encuentran
C~ xci xcl = 10x 6x 3 = 180
en las alemativas de respuestas.
Caso IV:
Clave: E
El total de formas que se pueden seleccionar las pre-
guntas:

T = 4 + 30 + 60 + 180 13. 5(n) = f [-1-+-


k=14k -1
1 1]
+ 2 n
= 274
" 2 " 1
==I --+I
k=l 4k2 - 1 ,=1 2n + 1
Clave: E
n n

12. La sucesión al1 n > O, se define:


= I 2 + _1_ I
; k=l (2k + 1)(2k - 1) k=l 2n + 1
'------v--- ~ '----r-------'
1 Il
a" = lag P , si existe un número primo P y un K
k
entero no negativo, tal que 11 = p . En la expresión:
an = O , en cualquier otro caso:
2 ==~+_B_
Se pide calcular la suma de los términos am , donde (2k+1)(2k-1) 2k+1 2k-1
m es un divisor positivo de 72, es decir:
2k(A+ B)+B - A
m = 1 ; 2 ; 3 ; 4; 6 ; 8 ; 9 ; 12; 18; 24; 36 ; 72 (2k + 1)(2k + 1)
~ A+B=O
B-A=2
Resolviendo obtenemos: A = -1 Y B =1.
En la expresión (I):

SI = .t'í
n 2
"-{2-k-+-1.=!){'-2k:---1--:-) x" = x"-I.x" r2"-1'1
o
1
2"-1]
o ; 11=0;1;2;3 ...
L2"-¡ o 2"-1
n 1 1
=I---
k=l 2k - 1 2k +1 En el ejercicio hacemos n = 11

=(t-l)+(l-i)+ ..· -~lW) xll xll-1. [2~0 ~ 2~0]


= X =

210 210 210


+(~-2n1+1)
1024 O
=1__ 1_
2n+1 = O 1
[
1024 O
En la expresión (ll):
Clave: O
S - t _1_
u - k=l 2n + 1 15. Dada la ecuación:
x2 -4
--=-x
3
... (*)
= n(2n 1+1) Ix+31 2
n 1) Si x +3>O ~ x > -3 ... (a)
2n+ 1
En la ecuación (*):
Finalmente sumamos:
x2 -4
--=-x
3
. x+3 2
S(II) = SI + Su = (1- 2n 1+1) + 2n n+ 1
x2 + 9x + 8 = O
3n
2n+ 1 (x + l)(x + 8) = O
Clave: C

~l
14. Primero hallaremos una fórmula genérica, así: Pero por la condición (a): CS. = { -1}
0
O
11) Si x +3<O ~ x < -3 ... (P)
x'=xOx'=[i 1 O ='] [2O O1 O
En la ecuación (*):
O 1 20 O 20
x2 -4 3

'] [t ~]
---=-x
O O O -(x + 3) 2
x' = x' x = [i im O
1 1 O =
O 1
O
21 O
1
21
5x2 + 9x - 8 = O

h
~H~!]
2 - 4(5)(-8)
-9 ±
O o o x = 2(5)
x' = x' x = [~ 1
O mi 1
O 1 22 O
1
-9 +..f24i
10
______________________
UN12005-1 MATEMÁTICA ~I~-~@M~
r ....
:;;;;:
G.· L.jWGL

~ xI = -2,452 x2 = 0,652 III. Del Diagrama:

Pero por la condición (P): C.5.2 =0 u


'f5(BnD)
Finalmente podemos afirmar que la ecuación tiene
sola una (1) raíz real (x = -1)
Clave: B
Otra solución de II:

Si x +3 < O
~ x <-3
En la ecuación (*): BnD=0 ~ C(BnD)=U
+
r--"-, Luego:
x2 - 4
--=-x 3
C(A) u C(B) u C(B n D) = U (verdadero)
Ix+31 L ~
~ (-) U
Clave: A
Como se presenta una incoherencia, no tiene solu-
ciones.
17. f(x)=x-0+1 x<-4 ... e)

16_ ~ Domf(x)=(-00;-4)
u f( x) es una función continua, luego:
f(4) = -4 - J-( - 4) + 1 =-5
limf(x) = -oo-J-( -00) + 1=-00

~ Ranf(x) = (-00 ; - 5)
Por definición: Domf'(x) = Ranf(x)

~ Dom((x) = (-00 ; - 5)
I. Del diagrama:
Hallando la función inversa f' (x):
B-D=B
Luego:
Si r (x) = y ~ y = x - 0 +1
(B-D)cAnC (Verdadero)
11. Del Diagrama: Y = -(-x +0 +~)+ i+ 1

u -(0 + + ~
Y = tt
-(y - ~) = (0 + ~t
'f5(C-D)

1.~(5- 4y) =0 + 1.
2 2
!)
o =1)5-4y-1.
2 2

B Y C( C - D) son conjuntos disjuntos


o = ~(J5-4)' -1)

~ B n tiC C - D) = 0 (Verdadero) ~ x=-~(J5-4Y-l)2


Finalmente: Graficando:

¡*(X)=-i(.Js-4X-1)2 XE(-oo;-s) Jl
Iz-(-2+i)1 s3
Clave: O

18. En la inecuación:

log313 -4xl >2 ... (1)



Donde: 3 - 4x *O ~ x * }.
4
... (2)
IR.
Además por teoría:
2 = log39

Luego en la inecuación (1): I


Para·que el valor de z¡ - z21 sea máximo éste nú-
meros debe estar ubicado sobre la línea diametral de
log313 - 4xl > log39 los dos círculos y en extremos opuestos. Del gráfico
De donde: podemos deducir que:

13-4xl>9 z¡ = 5 + 1i = 5 + i
z2 = -5 + li = -5 + i
~ 3 -4x <-9 v 3 -4x >9
Multiplicando estos dos números:
x>3 v x <-~
2 Z¡.Z2 = (5 + i) = (-5 + i)

.
-00
1
3 3
I 1
3
,
00
= -25 + Si - Si + i2
= -25 - 1
-"2 4 = -26
Clave: C
c.s. = (~ ; -~) v (3 ; 00)

20. Propiedad: Si IM(x)1 < IN(x)1


= R -[-~ ; 3J
~ (M(x) + N(x))(M(x) - N(x)) < O
Clave: B
En la desigualdad del problema:
19. Por definición de números complejos Ix-21<3Ix+211
[x - 21 < 13x + 631

La expresión [z - zol es el modulo de la diferencia [(x - 2) + (3x + 63)][(x - 2) - (3x + 63)] < O
de dos números complejos, y representa un con- (4x + 61)(-2x - 65) < O
junto de puntos que forman un círculo cerrado de
(4x + 61)(2x + 65) > O
radio "r " y cuyo centro se ubica en la coordenada
de zo. Calculando los puntos críticos:

En la expresión: 4x + 61 = O 2x + 65 = O

A = {Z E C/Z - 2 - i! ::;3 v Iz + 2 - il ::;3} -61 -65


~ x¡=T ~ x2=T
~ Iz-(2+i)I::;3 v Iz+(2-i)I::;3 = -32,5
= -15,25
<8>.
;:;;;~~~~~i:
t-_U_N_I_20_0_5_-I_M_A_T_E_M_Á_TI_C_A......I

Graficando: 22. Según el enunciado:


A

-00
_-1--1--_. -32,5 -15,25 00

e.s = (-00 ; 32,5) u (-15,25 ; + 00) B


Clave: A s
21. Del enunciado:

e
Si desde el punto M trazamos la perpendicular sobre
el lado BSe, ésta bisecará con la bisectriz del ángulo
BSe; pues el triedro tiene por sección transversal a un
triángulo isósceles.
M A

A
_...J--+---~ B

Por semejanza de triángulos de O VB y O'VD :


R H e
r=H-h Si desde el punto Q trazamos una perpendicular sobre
r=B.(H-/¡)
... (*) se, entonces por el teorema de las tres perpendicula-
H res MP 1.. se .Además se tiene:
Area lateral del cilindro inscrito:
e mLSPM = 90°
AL =2rrrh eTriángulo rectángulo SPM notable de 30° y 60°
De (*): =2rr[~(H-h)J/¡ Si SM = 2 :=} SP = 1 ; PM = .J3 ...m
eTriángulo rectángulo SPQ notable de 45°
=2;: (H/¡_h2)
:=} PQ = 1
Aplicando la primera derivada con respecto a la va- Aplicando Pitágoras en el triángulo MPQ:
riable ñ:
MQ =~(PM)2 +(PQ)2
dAL =2rrr (H-2h)
h H
=~( !?f _(1)2
Igualando a cero (O) para hallar máximo y mínimo: .
=..[2
2rrR(H-2h)=0
H En el triángulo rectángulo MSQ:

:=} /¡=!i
2
Clave: O
Clave: E
~=====..·:·w.:'f¿:=:·==:JU~N~I~2~o~o~5-II =..:.:oi';:~:~i~::=:~~Z
~M~A~:r~E~M~A~-T~I~c~A~I·~=·:J:H:~:. ~
23. Graficando el cuadriláte- B Por propiedad:
ro de acuerdo a las proporcio-
OC J. BC
nes de los ángulos que se indi-
can: Por la longitud de sus lados el triángulo rectángulo
....-;;,..---=L..loo.c
HBC es notable de: e = 37° Y (1 = 53° .
A En el punto C :
De BD trazamos una línea par formar un triángulo CP=900-(1
equilátero pues BC = DC . También trazamos la lí- = 90° - 53°
nea DP perpendicular a AB, y luego colocamos los = 37°
ángulos: Luego en el triángulo HCO:
B sec 370= OC
HC
5 r
4=6cm

~
r=15 cm
2
Cálculo de la longitud de la circunferencia:

L = 21tr = 21t(1; cm) = 151tcm


Clave: C
El triángulo rectángulo HCD es congruente (igual) al
triángulo PAD. 25. Analizando las alternativas:
~ HD = PD = b 1) Si p¡ y P2 son dos planos paralelos a r,
En el triángulo rectángulo PBD, por pitágoras: entonces pueden presentarse dos casos:
a) Los planos p¡ y P2 son paralelos pero no se
PB=~(2b)2_b2
bisecan.
=J3b

De lo anterior se deduce que el triángulo rectángulo


PBD, es notable de 30° y 60°, luego:
7x-(90-x)= 30°
8x-900=300
x=15°
Clave: B

24. Del enunciado: b) Los planos p¡ y P2 son paralelos entre si; pero
B se bisecan.

QR //:t

Considerando los dos casos, ésta proposición no


siempre se cumple, por tanto es FALSA.
UN12005-1 MATEMÁTICA I=~~~~~~
11) Sean .ti y olí las dos rectas que se cruzan. Si Como: AC//QR, entonces los triángulos ABC y
QBR son proporcionales. Por definición se sabe que
trazamos una recta.t'{ ,paralela a .ti, que
las áreas de triángulos congruentes son proporcio-
interseque a olí entonces se formará un plano P: nales al cuadrado de una de sus lados, luego:

1
AóQBR=16xAMBC
(Por dato:) =116x(192m2)
P

Por la recta .ti trazamos un plano Pv' perpen- =12m2


dicular al plano P . Clave: C

27. Por definición se sabe que en la ecuación de


una recta y = mx + b ; m es el valor de su pen-
diente, y el término independiente b, es el valor de
la ordenada donde la recta interseca al eje Y.
Del enunciado:
P .ti: y=-2x+8
olí: y = -2x - 4
La interse. -íón del plano Pv y la recta olí for-
man un punto N. Luego podemos trazar la per- Y
pendicular NM.
Esta proposición es VERDADERA.

111) De .ti podemos trazar infinitas rectas perpendi-


culares; pero sólo una será intersectiva a olí,
como se demuestra en el caso (II).
Esta proposición es FALSA. x
Clave: B

26. Del enunciado:

~l----~----~r--------- La recta .t' paralela y equidistante a .t;


y olí tam-
bién debe intersecar al eje Yen un punto equidis-
tante a las ordenes 8 y -4, es decir:

8+(-4)
Yo=--r
=2
~2--~~A~--------~----- Luego, la recta equidistante a .ti y olí será;
Donde: .ti//olí//QR
y = -2x +2 ó y + 2x - 2 = O
Datos: AC=16m ; AMBC=192m2 Clave: B
16)4~:::::::::::::::[::~U~N~12~OO~5-II~M~A~~~E~M~Á!n~C~A~J:::::::::::::::~~1~~~
;. G eZ

28. Las dos regiones rectangulares y congruentes En el triángulo rectángulo MCD:


tienen un lado común, formando un ángulo diedro
de 60°. MD="2d ... (2)

C' Del gráfico se deduce que:

MO=C'P= JS d ... (3)


2
En el triángulo MOD aplicamos la ley de cosenos con
los valores de (2) y (3):

(MD)2 =(MD)2 +(OD)2 -2(MD)(OD) cose

("2d)2 =( ~5dr +( ~5dr -2(~)( "~ ) cose

2=~+~-~cos8
442
D
~ cos8=!.
Por el teorema de Pitágoras se puede deducir: s
AC'=BD=AC =J(2d)2 +d2 ~ e=arc cos(~)

=JSd ... (1) Clave: B


Como se pide el ángulo formado entre las rectas Bi5
y XC' ,trazamos el segmento OM paralelo a AC' y 29. Del enunciado:
sobre el punto medio de BD. Ahora 8 será el ángulo B C
entre las dos rectas mencionadas.
Las diagonales C~ yAC son iguales y forman el trián-
gulo isóscelesACC', donde P, O YM son puntos medios
de sus lados por propiedad.
C'

Por el teorema de las tres líneas perpendiculares


m4A'F'D=900
En el triángulo rectángulo A'F'D, cálculo del lado
del hexágono regular
A'F' =d sen e ... (1)

Por propiedad del hexágono regular:

A'D'=2(A'F')
De (1): ... (2)
=2d sen e
~~;::¡.;,"===;::"LI __U_N_12_0_05_-_1M_A_T_E_M_Á_TI_C_A_I
GlrmeZ ~ ,
:
Calculo de la altura "h" del hexágono. En el trián- Si trazamos una línea recta sobre la parte supe-
rior se genera U" sólo segmento de recta, por
gulo rectángulo A'D'D:
tanto este conjunto es convexo.
h = ~d2 - (A'D,)2 e) / ,
I \
De (1): = ~d2 -(2dsen6)2 I \ I \

I I I \
I
\ I I
\ I \ I
=d~1-4sen26 ... (3) , / \ I
/

Por definición, área de un hexágono regular (e)


(e)
3../3
Ae.xágol1o =Ta (a: lado del hexágono) ... (4) Si trazamos una recta :t' por cualquier parte del
área plana, siempre se obtendrá como intersección
Cálculo del volumen del prisma hexagonal regular: un sólo segmento de recta; por tanto este conjunto
es convexo.
Vp = Ahexágol1o X h
De las proposiciones:
De (4): = 3../3 a2 x h
2 1) La intersección
del conjunto (a) (no convexo)
con el conjunto (b) (convexo); por el teorema
2, da como resultado un conjunto que pueda ser
De (1) y (3): = 3f3(d sene)2 x(d~1-4sen2e) convexo o no convexo. La proposición es FALSA.
11) La intersección del conjunto (b) (convexo) con
= 3../3d3~1_4sen26 sen2e
2 el conjunto (e) (convexo), por el teorema 1, da
como resultado un conjunto convexo.
Clave: A La proposición es VERDADERA.
111) La intersección del conjunto (a) (no convexo),
30. "Si se traza una línea recta sobre una superfi- conjunto (b) (convexo) y el conjunto (e) (con-
cie plano convexa (conjuntos convexos), la inter- vexo); da como resultado un conjunto que
sección generará sólo un segmento de recta". puede ser convexo o no convexo.
Teoremas: La proposición es FALSA.
1) " La intersección de dos conjuntos convexos Clave: E
da como resultado otro conjunto convexo"
2) "La intersección de un conjunto convexo, con 31. En un círculo trigonométrico, su radio r = 111

otro no convexo, genera otro conjunto que


puede ser convexo o no convexo".
Analizando cada uno de los conjuntos del emmciado:

a) A

CJ
-1

(a) (a)
Si trazamos una línea recta :t' sobre la ~rte s~e-
rior, se generan dos segmentos de recta PA y AQ , ---- P
por tanto este conjunto no es convexo. En el triángulo rectángulo OPQ:
b) e

CJ (b)
D

=rr e

(b)
D
OP = ~(OQ/

= ~(1)2

=-Js
+ (QP)2
+ (_2)2
UN12005-1 MATEMÁTICA I==";:'~wm G meZ

Por trigonometría: 33. En la gráfica del enunciado:


QP -2
sena = OP = .fS 000 (1) y

cosa = OP =
OQ 1
15 000 (2)
Del enunciado:
y = sen2a
-1t

= 2sena o cosa

De(1) y (2): -2(-21...L)


0- .fS .fS
Identidads trigonométricas a usarse:
=_1
5
sen 38 = 3sen8 - 4sen38 000 (3)
Clave: A
cos28 = cos28 - sen28 000 (4)
32. Identidades trigonométricas:
Del gráfico se deduce que las curvas corresponden a
sen(a-p)=sena cosjí-cos c sen p 000 (1)
las funciones trigonométricas senos, cuyos periodos
sen 2a = 2sen a cos a 000 (2) (T) indicamos:
sen 3a = 3sena - 4 sen3 a 000 (3)
FI(x)=senx : TI=21tooo(1)
cos3a = 4cos3a - 3cosa 000 (4)
000(2)

En la expresión del enunciado:

K=sen3x_cos3x+.! Los arcos de las dos funciones trigonométricas son


sen 3x cos 3x 2 proporcionales a sus respectivas frecuencias (f):
3 3
=1[4sen ~
4 sen 3x
_ 4cos x+1]
cos 3x 2 De O): senx: t.=l=...L
TI 21t

(3),(4):
=1[3senx-sen3x
4
cos3x+3cosx +2J De (2): senkx :
f = .l.. = ...L= .l...
2
senx cos3x T2 21t 21t

=1[(3senx
4 sen3x
-1}(1+ 3cosx)+2]
cos3x Luego relacionando
3
los arcos con sus respectivas
frecuencias:
3 [ senx cosx ] x----t...L
=4 sen3x - cos3x 2rr

=~[senxocos3x-cosxosen3xJ kx----t.l...
21t
4 sen Sxcos Sx

De (1): 3[ sen(x-3x) ]
=2 2 sen 3x cos 3x => kx = x(irr)
.i,
=~[sen(-2X)] 21t
2 sen 2(3x) k=3
Luego:
De (2): =~[-sen2xJ
2 sen6x h(x) = sen3x

=-~ senzxcscóx
Igualando las dos funciones para hallar los puntos
2 comunes:
Clave: O
~ ~..~_--~.~::::U:U::·:¡:"=:I
GomeZ
= :;¡k¡;' ;;¡; ,:di;gW UN12005-1 MATEMÁTICA I:::t± tÚ: I

Luego se puede deducir que:


eEIlóIII
senx=sen3x
Analizando:
~ sen3x-senx=O
sene = ± : 8 E Il ~ sen8 > O
De (1): (3senx-4sen3x)-senx=O
aE III ~ sen8 < O ... (3)
2senx-4sen3 x=O
tga =± : a E Il ~ tg < O
2sen x(1-2sen2 x)= O a E III ~ tg;:, O ... (4)

2sen x (cos2 x - sen2 x) = O Si a E Il entoces tg 8 < O < sen 8, lo cual contra-


dice la proposición (2), luego podemos afirmar que
De (2): 2senx· cos2x = O
~~ a E III cuadrante.
1 Il
De (1):
sen a < O < tg (cumple la proposición 2)
Luego en la expresión:
senx =O ~ x = kt: k E Z
De (II):
K = jsen8j + jtg8j + jctgaj
sen8 tg8 ctg
sen2x=0 ~ 2x=(2k+1)~; kEZ
De (3) y (4): = -sen8 + tg8 + ctg8
sen8 tg8 ctg8
x=(2k+1)~
4 =-1+1+1
=1
El punto A se ubica en el intervalo (-rt ; - 23rt) Clave: D

Para k=-2 ~ xA =(2(-2)+1)¡ 35. Si jaj$i ~ -~$a$~ ... (1)

_3rt En la expresión:
4 E(a) = sena·tgj2aj

El punto B se ubica en el intervalo (23rt ; rt) cosl2a + ~I


sena·tgj2aj
xB =(2(1)+1)¡
Para k=l ~

3rt
=l= + 1~1
4 = sen u tgj2aj· sec(2a +
'-v-' ~
.E.)
12
... (*)
Clave: D F(<l) G(<l)~

34. Del enunciado: Graficando la función f (a ) :


cos 8 < O ... (1)
F(x)
sen 8 < tg 8 ... (2)
De (1) podemos graficar:

rr a
"2

F(~) = sen(~) = 0,5


~:;JI'=iO:.:,::':¡¡¡;¡¡¡¡:' :;:- ==:[=Ju~N~I~2~o~o~5-II
~
~M~A~:r~E~M~A~-
T~,~c~A~I~i~====' :M{=,~'B1:".:Ú:K'!ii!iL:,~,ma!'~,
.
~'L
«'G~it\eZ

36. Del enunciado:


Graficando la función G (ex) :
f(x) = e x secx = secx - 1
g(x) = cosx
Graficando las funciones:
y
1t ex f(x)
"4

G(~) = tg(~) = 1,73

Graficando la función H (ex) : x


h(x)
-1

1t
6"
1t
24
a
n'···n········n·
Del gráfico se puede deducir que en el intervalo
Cálculos para graficar:
(O; 2lt) existen 2 cortes entre las dos funciones.
Se sabe que: sec( O) =1
Para el intervalo (O; 20n) se puede inducir que
existen 20 cortes; y por simetría de funciones, en
Luego si : sec(2a+1~)=1 el intervalo [- 2 On ; 2 On] existirán 40 cortes.
~ 2a+...!L=O
12 ~ a=-~ 24
Clave: C

Además se tiene: 37. Identidades trigonométricas a usarse:

sec[ 2(-~) + l~J = 1.41 • sec2S=tg2S+1


sec2S-tg2 =1
sec[ 2(~) + l~J = 3,86
(secS-tgS)(secS+tgS) =1
Si observamos las tres gráficas de las funciones, pode-
mos deducir que para a = ~ , éstas asumen ~ secS+tgS= 1 ... (1)
secS-tgS
coencidentemente el mayor valor, luego en (*):
sen'ª
E(a) = sen(~). tgI2~1· secI2(~)+ llt21 • tag'ª = __ 2
2 cos'ª
= sen ~ . tgl ~ l· secl i;I 2
~
= !(J3(f6 ~ ./2) ~
2

../18 +.f6 2
2 =cscS-ctgS ... (11)
Clave: E
Del enunciado: Del enunciado:
sec x = a + tg x y
secx - tg x =a .... (1)
B'(2;4)
Considerando la identidad (1):

secx+tgx= 1 ,,
sec x-tg x
A'(-4;6)
1 , a
De (1): ... (2) ,,
a
,,
x
En la expresión trigonométrica: ,

=[tg(~+1)r
Dato: tga = 1.
4
sena=~

cos a=~
5
LJ3
4
= [tg~(~ + x)r En el sistema rotado, P'és un punto medio de A'B' ,

De (II): = l= (~+ x) - ctg(~ +x) r entonces:


P' = (xj, ; yj,)
= [cscx - (_tgx)]2
=(XA+XB. YA+YB)
2 ' 2
= [cscx+tgx]2
De (2): =(_4+2 . 6+4)
2 ' 2
=(-1;5) ... (3)
En el sistemaXY, el punto P:

Clave: C P = (x ; y)
De (l) y (2): = (x'cosa- y'sena; x'sena+ y'cosa)
38. En el gráfico:

y
=[-1(~)-5(~); -1(~)+5(~)J
De (3): = (_19 . 17)
5 • 5
Clave: D

39. Por condición del enunciado:


(2)

x
Se deduce:
x=x'cos8-y'sen8 ... (1)

y = x'sen 8+ y'cos 8 ... (2)


d
D
~.170=:=====:~===::JU~N~I~2~0205~-II
~.-:
:JJiiiiJ:.:-:.mtr.=.w:0r:,1'
~M~A~:r~E~M~Á~TEIC~A~):¡¡¡¡¡¡¡¡¡¡a¡¡¡¡=::··:·
%
f':J_:.",:rn;'~~i.J~IIIDJU«
Wilidb:dW= ~~z

La rueda al girar desde la posición (1) hasta posi- =>


AOAB = 3AoEF ... (1)
ción (2), la proyección de su centro, en el plano ho- Además por definición:
rizontal, se había trasladado desde el punto A hasta
el punto B. La rueda habrá girado el ángulo íiQ' . 1 (-)2
AOEF=2a OE
En la posición (2) del gráfico se puede observar
que:
=laa2 '" (2)
2

BQ' = 60 + 90 S6
0 0
=
1t
..• (1)
Reemplazando (2) en (1):

La distancia As que recorre la rueda será igual a la AOAB = 3(~aa2)


longitud del arco íiQ' . =1aa2 ... (3)
2
AB=L-
BQ'
Longitud del arco AB :
=BQxr
LAil =a(OA)
De (1): = (S6 1t
X) r ... (2)
(Dato:) a=a OA
La distancia, menor a 21tr, que existe entre Q y la
=> OA = g ... (4)
proyección de Q' sobre el plano será: a
d=D-x Por definición, el área del sector OAB:

= AB - rcos60o
AOAB = ~a(OA)2
De (1) y (2): = S6 1tr-r @
= 2,lr
=~ a(~t
Clave: B De (4): a2 ... (S)
= 2a
40. Del enunciado:
Igualando las áreas de (3) y (S):
A

o
Reemplazando el valor de a en (3):

Datos:
AbAB = ~( 1)a 2

= .J3 a2
OE = a 2
L- =a Clave: D
AB

Por condición del problema, las áreas:


~~=~¡:=~~,
'-""==...
. :.:
1•.. _u_N_'_2_o_o_5-_"_M_A_T_E_M_A_·T_,_c_A_I~~JW$i MP:: •. ~

MATEMÁTICA

1. Dadas las siguientes proposiciones. é Cuáles son écuál es el doble del número que se digitó en base lO?
verdaderas?
A) 31,5 B) 63,0 C) 117,5
1. Si:
D) 235 E) 470
a,bERla>OAlbl<l ~ (ab+a+1)
6. El primer término de la sucesión
es siempre mayor que l.
?; (3)8 ; (12)5 ; 16 ; (1022)3 en la base 2 es:
II. Si: a, bE R+ el máximo valor que toma
5ab es l. A) O B) 1 C) 1,01
a2+b2+3ab D) 1,10 E) 1,11
I1I.Si: 3+a2_a4 <M,VaER ~ elmenorva- 7. Se quiere preparar 50 litros de vino para venderlo
lar entero de M es 3. a 5/.95 cada litro, ganando 5/.5 por cada litro. Para
A) FFF B) VFF C)FVF ello, se hace una mezcla con vinos de 5/.60, 5/.70,
D)WF E)VVV 5/.100 y 5/.110 el litro. Si la mezcla debe tener 5
litros de vino de SI. 70, la mayor cantidad posible O"
2. El núme. o de alumnos en un aula es menor que 50 vino de 5/.110 Y por lo menos un litro de cada tipo de
entre hombres y mujeres. Si el número de hombres es vino. ¿Cuántos litros de vino de SI. 110 el litro se
mayor que el doble de mujeres y además ambos son necesita, sabiendo que los volúmenes de las 4 calida-
múltiplos de 10, determine el número de hombres. des son números enteros?
A) io B) 20 C) 30' D)40 E) 50 A) 17 litros B) 21 litros C) 25 litros
D) 29 litros E) 33 litros
3. ¿Cuántos ceros tiene el resultado de:
8. Con una muestra de tamaño m, se construyó la
1.2.3 ..... 100?
siguiente tabla de datos:
A) 20 B) 21 C) 22 D) 23 E) 24
(O; 2] (2; 4] (4; 6] (6; 8] (8; 10]
4. Si a y b son enteros mayores que 100 tales que
O n2-3a (n2 _a)2 n2 a
a + b = 300 écuáles de las siguientes alternativas es
la razón exacta de a/b ?
Entonces el valor de a es:
A) 9/1 B) 5/2 C) 5/3

D) 4/1 E) 3/2

5. Las computadoras almacenan información digital


en registros. Un registro es un grupo de celdas binarias. m-n2
C) 2
Si al digitar un número sobre el teclado se genera el
registro:
D) 2n2 -,11 + m
E) ~n2 +m
9. Al extraer la raíz cuadrada de un número se , 4. Sean los conjuntos
tomó por error al residuo cOIPo raíz y a ésta como V={A;E;I;O;U}
residuo, resultando un número que es inferior en B = {1 ;2;3;4;5;6}
372 unidades al original. Si la diferencia-de la raíz
menos el residuo es 3, calcule el número original. Se desea elaborar placas (para autos) de la forma
vlv2blb2b3b4 donde vk E V, bj E B demaneraque
A) 4149 B) 4150 CH157 no existan símbolos repetidos.
D) 4158 E) 4159
Entonces el número total de placas diferentes será:
10. ¿Durante cúanto tiempo estuvo depositado un A) 480 B) 1 321 C) 7200
capital al 12% anual, si los intereses producidos al-
D) 32250 E) 32400
canzan al 48% del capital?
A) 5 años y 1 mes D) 4 años 15. Dada la siguiente función:
B) 5 años E) 3 años y 11 meses J(x) = 4.fX - x ; X E [O ;1]. Halle JO(x), donde
C) 4 años y 8 meses ( es la inversa de f
11. Calcule el conjunto solución de la inecuación:
A) ((x) = (2 - ~ 4 _ x) 2
(x_T2)2 +4x+2<0
B) J'(X)=(3-~4-x/

A) ( 1; ;-%) D) (_1; ;-%) C) J'(X)=(2+~4-x)2

B) (_1; ;-~) E) (_~1;_ %) D) J'(x)=(3+~4-x)2

E) ((X)=(4-~4_x)2
C) (-*;-~)
, 6. Al resolver la ecuación
12. Sea Yun número real no nulo.
x + log1424 (1 + 2x) = xlog1424 712 + log142472
Calcule (E+L)-(T+U), siE,L,TyU
satisfacen el siguiente producto de matrices: entonces podemos decir, que el número de soluciones es:
A) O B) 1 C) 2 D) 3 EH

17. Indique la verdad o falsedad de los siguientes


C) 2 D) 3 E) 4 enunciados:
A)O B) 1
1. Sea

13.Sea p(x)=x2+x+l ylasucesión p( x) = ax3 + bx2 + ex + d , a ,t O , d ,t O si P

5/1 (x) =
/1

I,[p(x)]
k
. Entonces el menor valor de
tip,,~ "res raíces reales, entonces P = (i) tendrá

k=O las mismas raíces.


Sn (x) cuando n es arbitrariamente grande, es: 11. Todo polinomio complejo siempre tiene raíces
complejas y sus respectivas conjugadas.
A) O
III. Si la suma de las raíces de un polinomio es rucio-
B)4
nal, entonces cada una de ell. s también es racio-
C) 8 nal.
'1) arbitrariamente muv grande A) FFF B) FVV C) VFV
F.) no existe D)VVF E) VVV
18. Sean P y Q conjuntos tales que: ED se elige el punto F de manera que FB = BC . Des-
Si p E P , entonces p E Q . Luego se puede afirmar . de el punto F se traza la tangente Fe (e E e) tal
que: que Fe = 413 . Si DC = 4u entonces FB es igual a:

A) Si -3 E Q , entonces -3 E P A) 4,5 u B) 5,Ou C) 5,5u


B) Si 13 e P , entonces 13 e Q D) 6,Ou E) 6,5 [[
C) Si 10 e Q , entonces 10 e P
22. Sea el trapecio ABCD ( BC/ / AD Y BC < AD) .
D)Si 0,10E Q,entonces 0,10e P
Por el punto de intersección de la diagonales del trapecio
E) Si 1 e Q, entonces 1E P se traza una recta L' que interseca a AB y CO en P y
Q respectivamente, que se encuentran en el mismo
19. Resuelva: semiplano con respecto a la recta que contiene a la
mediana del trapecio. Si AA' , BB' , CC' y DO' son las
(h+rsf -+:(h-rsf $34 distancias de los vértices ala recta L'y AA' + DO' = a ,
BB' + CC' = b ; calcule la distancia del punto medio de
.A) -3 sx $ 3 D) -12 $ x s 12 la mediana del trapecio a la recta z:
B) rs $ x s 2rs E) -13 $ x s 13 A) a +b B) a - b a+b
C) -4 $ x $ 4 8 8 C) 4

20. Indique la gráfica de g( x) = J( x + Ixl) , si la D) a -b E) a +b


-6-
gráfica dejes: 4

23. El valor de

cos (27rr) + cos( ~rr) + cos( 6 rr) es:


7

A) - 2:1 B) O C) 1
2
D) -1 E) 1

24. La medida de un ángulo en el sistema sexgesimal


es XY" zw' y la medida del mismo ángulo en el siste-
ma centesirnal es 50~ 50'" .

Calcule: e = xz s-+ wy
A) 1 B) 2 C) 3 D)4 E) 5

25. Calcule el área, en u2 , que puede tener la re-


gión sombreada S cuando el área del círculo CI es
máximo. R =2u , e = rr/3 .

11_---....,~.
J 2 4

21. En un triángulo isósceles ABC recto en B, se


traza la circunferencia de diámetro CDe
A~--~----~~----~----~B
(D E BC) , en AC = {E} . En la prolongación de
~~::::::::::::::::1C:~U~N~I~2~O~05~-~IIJM~A~T~E~M~A~'T~IC~A!::J~:::::::::::::::;.,~dt~.~
~. cll"mez

A) 1,36 B) 1,45 e) 1,53


A) -!(sene+cose+tanS)
D) 1,67 E) 1,82
B) -!(senS-cose+tane)
26. En la figura mostrada, halla el valor de:
AB sen(x - y)
E = -_~---=-...!...
B
e) -i( sene + cose - tane)
cosy
D) -!( sene + cose - corü)

E) -!( sene + cose + cote )

30. Calcule el rango de la función

A~--L.::.--1..--J.J C f(x) = 2(cos2x - 3)(-2 - sen2x), 'dx E IR

A) BD B) se C) CD D) BC E) AD A) [7; 23] B) [8; 23] e) [8; 24]

D) [8; 25] E) [7; 25]


27. Sean las funcionesfy g, con reglas de correspon-
dencia f(x)=x",npar,y g(x)=4R2-x2 ,R 31. Determine para qué valores de x E [O ; 2TC] se
constante. Si P Y Q son los puntos de corte de las cumple:
gráficas defy g siendo a y p los ángulos en posición
normal determinados por P y Q respectivamente, en-
~12,f3- 3tanxl < [J3
tonces:
A) (~; ~ ) u ( ~ ; ~)
tan a + tan P + cota + cotP es igual a:

A) O C) h D) t E) 1[2
1 B) (~;
3
TC
~ ) u ( 2 TC ; 43 )

C) (~; ~ ) u ( 5 TC ; ~TC)
28. El valor de la expresión: 6

E = arctan(~)+arctan(~)+ardan(~)+
es:
arctan(~) D) (~; ~)ue:; ~TC)

A) ~ B) E. C) E. D)~ E) E. E) (~;~)
3 4 S 7 8

29. En la figura, halle el área de la región sombreada. 32.Si


y
sen1° + sen3° + sen5° + ... + sen59° = ~
Círculo
calcule el valor de k:
trigométrico
A) secl° B) cosec1° C) cosl °
D) sen1° E) tanl°

x 33. En un triángulo rectángulo ABC la hipotenusa


mide 2a unidades, entonces la suma de los cuadrados de
las longitudes de las medianas relativas a los catetos es:
A) 3a2 B) 5a2 C) 7a2
D) 9a2 E) 11a2
l'~"¡~::;:e=: ¡!5'~;=1 UN12005-11 MATEMÁTICA 1&1 ¡¡m*\i1=:~
y
34. En la figura siguiente, el triángulo AOC, es
equilátero de lado b; AC es diámetro de la
semicircunferencia y AoC es un sector circular. De-
termine el área de la región sombreada.

o x

A) x2 + y2 - 8x -10./3y + 48 ='0
B) x2+y2+6x+10./3y+48=0
o C
C) x2 + y2 +6x -10./3y+48 = O
2
b2 D) 1ta
A) 24 ( 6./3 - 1t ) D) x2 + y2 _ 6x + 10./3y + 48 = O
12
E) x2 + y2 -6x-10./3y+ 48 = O
2
b2 E) 1ta
B) 12 ( 6./3 - 1t )
8 38. En un tetraedro regular V-ABC, halle la medida
del ángulo diedro que forma el plano del L'lBMC con
b2 el plano del t.ABC , siendo M punto medio de VA
C) 12(./3 + 1t)

A) sen- 1
( 1) D) sen-1( 1)
35. ABC es un triángulo rectángulo, M e AB y
N e BC , T punto medio de MN , Q punto medio de
AC ,si AM =4u y NC = 6u , entonces TQ mide
B) sen- (
1
1) E) sen-t})
(en u).
C) sen-1(t)
A) 2 B) 3 c)m D)m E)4
39. Las longitudes de dos circunferencias coplanares
36. En una circunferencia se trazan los diámetros están en la relación de 5 a 2 y su suma es igual a 141t;
perpendiculares AB y CD , por C se traza una recta si la distancia entre sus centros es dos veces la dife-
L tangente a la circunferencia, en el arco DB se elige el rencia de sus radios, podemos afirmar que las circun-
punto E de manera que E, B Y G sean colineales ferencias son:
(GeL),la mEB=70D, AEnDC={F}.Deter- A) tangentes exteriores D) disjuntas
mine la m4.AFG. B) tangentes interiores E) concéntricas
A) 8~io B) 950
C) 100 0 C) secantes

D) 125 0
E) 155 0
40. En un
triángulo AOB recto en O,
AB = 2AO = 4u
. Si OM es perpendicular al plano
37. Encuentre la ecuación de la circunferencia mos- del triángulo y la medida del ángulo diedro
trada en el gráfico, si MN = 6./3 u , T es punto de O - AB - M es igual a 60 calcule ~M. 0
;

tangencia.
A) 1 B) 2 C) 3 D)4 E) 5
~===:,"*===!!:W:ii:,!tiW!1't=:{:JU~N~1 2~O~O~5~-I[1 ..=. :.'J¡;jll!!1t%'fW;m;r
JM~A~T~E~M~Á~T~IC~A=]:::¡:=¡¡¡:":":':;WWWWW:

SOLUCIONARIO
1.
l. "Si: a, b e Rla > 0/\ Ibl
< 1 ~ (ab + a + 1)
Por propiedad: (a 2 - ~f~ O

~ 13 _ M e R - {O} o también:
es siempre mayor que 1". VERDADERO. 4
Análisis:
De la condición: Me R-P:}
Clave: O
Ibl > 1 ~ -1 < b <1
0<b+1<2 2. Sea: h: Alumnos hombres.
~ b+1>0 m : Alumnas mujeres.

a(b+1»0(a) c c- O Por condición del problema:


ba +a > O h+m < SO ... (1)
~ ba +a + 1> 1 h > 2m ... (2)

Il. "Si: a, b e R+ el máximo valor que toma o


Por(l): m=10 ~ m=10,20,30
5ab o
2 b2 3 b es 1". VERDADERO.
a + + a Por (2): h=10 ~ h=10,20,30
Análisis:
De (1) y (2) se concluye que:
Por la condición: ab ~ O h = 30
Por propiedad: m = 10
o $ (a - b)2 Clave: C
3. Por teoría:
0$ a2 - 2ab + b2 ~ O
n! = Piu¡ p;2 p3U3 ... pkUk
5ab $ a2 + b2 + 3ab
5ab < 1
a2 + b2 + 3ab - Donde: (l. = (.!!.)+(...l!...)+...l!...+.
[ p¡ p/ p¡3
El máximo valor que puede tomar la expresión
es l. i=1,2,3, ... ,k

IlI. "Si: 3+ a2 - a4 < M , 'r:!a e R ~ elmenorva- Del enunciado:


lor entero de M es 3". FALSO.
1x2x3x ... x100 = 100! ... (1)
Análisis:
Para obtener los ceros que contiene 100! debemos
3+a2-a4<M hallar los factores Pi = 2a1 y P2 = 5'i2 , que son
a4 _a2 - 3>-M los que ganerarán los múltiplos de 10.

(( a 2)2 - 2a
2 + 4"1) - 4"13 > -M p =100+100+100+100+100+100
1 2 22 23 24 25 26

(a2-~f > l;_M =50+25+12+6+3+1


= 97
UN12005-11 MATEMÁTICA J :;::::~~
p = 100 + 100 En (11):
2 5 52
0,1(2) = 1 X TI
= 24
=lxL
El número de ceros que contiene 100! 2
24 24
= 0,5 ... (2)
Nceros =2 5 ; al = a2
Sumando (1) y (2):
= (2x 5)24
1110101,1 = 117 + 0,5 = 117,5
= 1024 ; (24 ceros)
El doble de éste número:
Clave: E
n = 2x117,5 = 235
4. Por condición del problema: Clave: D
• a,beZ/a,b>100 ... (1)
... (2)
6. En la sucesión:
• a +b = 300
Si hacemos a = xk 1\ b = yk ... (3) ?; (3)8; (12)5; 16 ;(1022)3
En la expresión (2) tenemos: Transformando a base 10:

(xk + yk) = 300· 3(8) = 3x8° =3


(x + y)k = 300 ... (4) J
12(5) =lx5 +2x5° =7
Donde x.y son números primos relativos.
16 = 16
• Para la alternativa A (9/1), en (4) :
1022(3) = 1x 33 + Ox 32 + 2x 31 + 2x 3° = 35
(a + 1)30 = 300
Para obtener cada uno de los siguientes términos de la
Reemplazando en (3), k=30: sucesión se aplica la relación:
a = 270 Y b = 30. No cumple la condición (1). nx2+i
donde: n, valor numérico de cada término .
• Para la alternativa E, en (4): Respectivamente:
(3 + 2)60 = 300 i = 0,1,2,3; (orden de cada término)
Reemplazando en (3), k =60 : Para obtener el segundo término (3), a parttir del
- primer término (?):
a=180 y b=120
Clave: E nx2+0=3

~ 11 = l
2
5.1110101,1 = 1110101(2) +0,1(2)
~-.......,...... Transformandolo el primer término a base 2:
(1) (n)
Transformando a números de base 10: n=1..=1+1
2 2
En(I):
= 1(2) + 0,1(2)
1110101 = l x 26 + l x 25 + Lx 24 + Ox z3 = 1,1(2)
+lx22+Ox21+1x2° Clave: D
= 117 ... (1)
~~:::::::::::::::::;c:~~~~~~~~~~::];::::::::::::::::::~~
178 UN12005-11 MATEMÁTICA QJIllQ '"
G~iñez

7. Se quiere preparar 50 litros de vino para ven- Cada una de las cantidades correspondientes a los
derlo a S/. 95 cada litro, ganando S/. 5.00 en cada intervalos que se muestran en la tabla son positi-
uno, es decir el costo de preparación será de 90 vos, y además, la suma de éstos es igual al tamaño
soles/litro. de la muestra, es decir.
Por condición del problema la preparación se eje-
0+(n2-3a)+(n2-at +n2+a=m
cuta así:
Costo por litro Litros de vino n4 -2n2(a-1}+a2 -2a=m
4 2
n -2n2(a-1}+a ·-2a+1=m+1
Vino A: CA = 5/. 60 a ~1

VinoB: CB = 5/. 70 b=5 n4 -2n2(a-1)+(a-1)2 =m+1

Vino C: e¿ = 5/.100 c ~ 1 [ n2 - (a - r
1) =m+1

VinoD: CD = 5/. 110 d =? n2 -(a-1) =±.Jm+ 1


Luego la preparación del vino se ejecutará así: n2-a+1=±.Jm+1
aCA+bCB+cCc+dCD=50x90 (Soles) De (1) y (11): n~2-3a+ 1 =±.Jm+1-2a
..•. "-.,---1""'-..--'
(+) (+) (+) (-)
a(60)+ 5(70)+c(100)+ d(110) = 4500
60a+ 350+ 100e + 110d = 4500 ~ n2 -a+1 =+.Jm+1
6a+ 10c+ lId = 415 ~ a=n2-.Jm+1+1
~a = 415-10e-lld Clave: A
... (1)
6
Los litros de vino de cada calidad que emplean: 9. Por teoría: rnL.L
. r I R
a +b+e +d = 50
(Dato): a + 5+e +d = 50 Donde: R2 + r = n (1)

a+ e +d = 45 Dato: R- r = 3 ~ R = r+3 (2)


De (1): 415 -10e -lld +e+d = 45 Reemplazando (2) en (1):
6
4e+5d=145 ... (2) (r + 3/ + r = n
Por condición del problema, se emplea la mayor can- ~ n = r2 + 7r + 9 ... (3)
tidad de vino de S/. 110 (calidadD), entonces d debe Cuando se toma los datos con error:
tener el mayor valor posible.
r2+R=n-372
Reemplazando en (2):
De (2): r2+(r+3)=n-372
4(5) + 5(25) = 145
~ n = r2 + r + 375 ... (4)
Clave: C
Nota: Sí se asume que d = 26 , la expresión (2) se Igualando (3) y (4):
transforma en incoherente. r2 + 7r + 9 = r2 + r + 375
~ r = 61
8.
Reemplazando r = 61 en (3):
(O; 2] (2; 4] (4; 6] (6; 8] (8;10]
(n2_a)2 11 = (61)2 + 7(61) + 9
O n2-3a

(1)
. n2
.
a

(II)
= 4157
Clave: C

w
cii~'-=,,:,,-':""'·
ZG~~~ w
'-,:::1
• -
UN12005-11 MATEMÁTICA ~·oo ;;gsg;;g :.~ ~~~

De (II):
10. Datos: 1 = 48% C
YL = O ~ L = O o •• (2)
r=12% ~
ano De (IlI):
1 : interés , C: Capital , r: tasa TL+ U2 = L
Por definición: De (2): T(O)+U2=O
I=rxTxC ~ U=O ... (3)
De datos: 48% C = (12%~)TC I De (IlI):
ano
~ T = 4años TE+UT =E
De (1) y (3): T(l) + (O)T = 1
Clave: O
~ T =1 ... (4)
2
11. 2
(x - 2- ) + 4x + 2 < O Cálculo de la expresión pedida, de (1) , (2), (3) Y (4):

(x-ir +4x+2<0
s = (E + L) - (T + U)
=(1+0)-(1+0)
x2 + 56 x + 33 < O
16 16 =0

16x2+56x+33<0 Clave: A
(4x + 1l)(4x + 3) < O
• 13. Del enunciado:
' 11
Los puntos cnucos o

seran: XI = -"4 /\ x2 = -"43 " k


S" (x) = I,[P(x)]
.. +1_ll - 1 + ~ k;O

4
_2-
4
=
"
I,[x2 +x
k;O
+lJ k

Por definición si la expresión es menor que cero (O) el k


11 1 2 3
conjunto solución estará dado por el intervalo nega-
= k~ [ (x+2) +4 ]
tivo que se muestra en el gráfico, es decir:

X E (_11.
4'
_l)4 El menor valor de Sn (x) se obtendrá cuando x = -~

Clave: B ~ (1)2
S" -
mi"
"[ (O) 2 +-3Jk
= I,
k;O 4
12.
(~ ~) (~ ~) = (~ n = at +(if +(ir +...+(ir
Ejecutando la operación de multiplicación de matrices: 1- - (3)"
4
YE = y ... (I)
l_l
YL = O •• 0 (II) 4
TE+UT = E ... (1lI) = 1- O
... (IV) l-l
TL + U2 = L 4
De (l): =4
YE=Y~E=l Y Y;éO ... (1) Clave: B
t8>~ ...
I__ U_N_I_2_0_0_5_-I_1
_M_A_T_E_M_Á_T_,c_A__ I==i:x::::;~l~::
14. En los conjuntos:
Si x=o =) r(0)=(2±~4-0)2
V = {A ; E ; I ; O ; U} = h ;v2 ; "s ; V 4 ; vs}
=2±2
tiene 5 elementos para seleccionar: =0
Para obtener el el valor de f * (O) , que correspon-

tiene 6 elementos para seleccionar de a su rango, se debe tomar el signo negativo (-)
del radical.
Considerando que los elementos que se selecciona no
pueden repetirse, y además para elaborar una placa
se tomará 2 elementos del conjunto Vy 4 elementos
del conjunto B; es decir tendrá la forma genérica: Clave: A

Placa: vI v2 bl b2 b3 b4
16. Resolviendo la ecuación:
Para hallar el mismo posible de combinaciones, con X

las características mencionadas aplicamos la multi- x +/0.1:1424(1+ 2 = X/O.l:1424712+/tJ.l:142472


)

plicación:
X+/O.l:1424(X)1+2 1424
=x/O.l:1424-2-+/og142472

x +[0.1:1424(1+ 2x) = x/O.l:14241424 -x[og14242 +/ogI42472

X+/tJ.l:1424(1+ 2x) = x -x/og14242+[tJg142472

/0.1:1424(1+ 2 X
) = /tJg142472-/tJg1424 2 X

Clave: D
/0.1:1424(1+ 2 = X
) /O.l:l424G;)
1+2x = 72
2x
15. f(x)=4IX-x; xE[O;l]
2

=) Domf(x)=[O;l]
=) (2 + 2x - 72 = O
X
)

Ram f(x) = [O ;4] (2x - 8)(2 + 9) = O X

Si f(x)=y =) 2x -8=0

=) y = 4IX-x 2x = 23
=) x=3
4 - Y = x - 4IX +4
4 - Y = (IX - 2)2
Del producto (2 x
+ 9) no se puede obtener valores
para x, pues esta expresión siempre es mayor que cero
±~4- y = IX-2 (O) y por tanto no satisface la ecuación.
. 2
=) x=(2±~4-Y) =) X E {3}
Clave: 8
=) t : (x) = (2 ±.J4 - X)2

Por definición: 17. Analizando los enunciados:

Dom f*(x)= Ranf(x) =[0;4] I. "Sea F(x) '- a.x3 + t,y2 + ex + d ; a ~ O, d '# O
Ram f * (x) = Domf(x) = [0;1] si P(x) tiene tres raíces reales, entonces p(~)
tendrá las mismas raíces". FALSO.
t:t4\¡~~I ...__U_N_I_2_00_5_-_II_M_AT_E_M_Á_T_IC_A_....,¡,i:;===¡;¡==':::::7::=='~" 3181

Análisis: Graficando:

P(X) = ax3 + bx2 + ex + d = O

a(x - Xl )(X - X2)(X -X3) =O


PcQ
Sus raíces serán: Xl; X2 ; X3 ... (1)

Si p(~)=a(~t +b(~)+C(~)+d=O Analizando las alternativas:

A) "Si -3 E Q, entonces -3 E P ". FALSO.


= a(~- Xl )(~ - X2 )(~ - X3) = O

Sus raíces serán: Jc.; Jc. ; -L ...(2)


Xl X2 X3

De (1) y (2) se deduce que las raíces de p( x) Y


p(¿) son diferentes. B) "Si 13 ~ P ,entonces 13 E Q ". FALSO.

11. "Todo polinomio complejo siempre tiene raíces u


complejos y sus respectivas conjugadas". FALSO
Análisis:
Sea P(x) = (x - 3i)(x + 2)
Sus raíces se obtienen:
e) "Si 1 Oe Q ,entonces 1O~ P". VERDADERO.
(x - 3i)(x + 2) = O

Xl = 3i ; x2 =-2
[No existe la conjugada de la raíz compleja de
Xl = 3i!.

111. "Si la suma de las raíces de un polinomio es ra-


cional, entonces cada una de ellas también es
racional". FALSO
D) "Si 0,10 E Q, entonces 0,10 e P ". FALSO.
Sea P( x) = x2 - 7
Sus raíces serán:
Xl =..fi ; x2 = -..fi
La suma: Xl + x2 = ..fi + (-..fi)
=0 E) "Si 1 e Q, entonces 1 E P ". FALSO.
Como al menos existe un polinomio cuya suma de sus
raíces irracionales es un número racional (O), enton- u
ces invalida el enunciado.
Clave: A
18. Considerando que "p" representa los elementos
de los conjuntos P y Q. Clave: C
Si P E P entonces p E Q
19. Por propiedad de transformación de radicales 20. Del enunciado:
dobles:
f(x)
b+.f8 =/2+1)
~ r: ... (1)
v3-,,8 =,,2-1
En la expresión del enunciado:
2 x

(b+.f8f +(b-.f8f ~34 ~ f() {l; O~ x ~ 1


x = -x + 2 ; 1 <x ~2
De (1): (/2+1f+(/2'-lf~34 ... (2)
Como O s x s 2, entonces:
(/2 + 1f + 1 ~ 34 x+lxl = x+x
(/2+1f =2x
Luego en la función compuesta:
Si hacemos (/2 + lf = a ... (3)
1 O~2x ~ 1
a+l~a 34 o~x~l
2
a+2.Jal+l~ 34+2 f(x + Ix 1) = f(2x)=
.Ja a -(2x)+ 2 1< 2x ~ 2

(.Ja + la J s 36
Graficando: f(x+lxl)=g(x)
l.<x<1
2 -

.Ja+la~6 ¡a+Ja~o
f(x+lxl)
a - 6.Ja + 1 ~ O

(a- (3- 2/2))(a - (3+ 2/2)) ~ O g

Hallando sus puntos críticos: .1..1 x


al = 3- 2/2 /\ a2 = 3+ 2/2 2

Clave: D
+ ¡ ¡---..;..+-
3-2f:2 3+2f:2
21. Del enunciado: F
~ 3- 2/2 ~ .Ja ~ 3+ 2/2
Dato: ~ABC es isósceles
De (3):
3- 2/2 s J(~ 2+ 1f ~ 3+ 2/2
x
2- 2/2 + 1s (.[2+ 1)2 ~ 2+ 2.[2+ 1
2 x 2
(/2-1) ~(.[2+1F~(/2+1)
(/2 + lr2 ~ (.[2+I)Í ~ (/2 + lt
~ -2<~<2
- 2 -
-4~x~4
Clave: C A
Q1\112005-11 MATEMÁTICA

Trazando algunas líneas auxiliares y an .udo pro- En el triángulo rectángulo AFE:


piedades.
tana = AE
EF
De (2): tans3° = AE
2 ED+FD
De (1): 1 AE
2 212 + 412
=> AE = 312 ... (4)
Del gráfico:

AC = AE+EC
De (4): = 3../2 + 2../2
= 5../2 ... (5)

En el triángulo rectánguloABC:
2 2 2
AB + BC = AC
2 2 2
Por el teorema de las tangentes: Dato: BC + BC = AC

EFxFD =FG
2
2BC2 = AC2

(FT' '-DE)XFD=FG
2
De (5): 2BC2 = (s..J2f
BC = 5
+ 2J2)FD = (4J3/
Finalmente, por dato:
FI> 1- 2J2FD - 48 = O
~ FD=4J2 ... (1)

El triángulo rectángulo FECes notable: Clave: B

22.Graficando segun el enunciado y adicionando


3~ los segmentos MP y NS perpendiculares a .e.
1

El punto B es circuncentro del triángulo AFC, luego


por propiedad.
m<1:AFC= ~

- !(m<1:ABC)

= 1(90°)
2
= 45° ... (2)
En el vértice F:
a+8=~ Datos:
a+ 37° = 45° AA' + DD' = a
2
53° ... (3) BB' +CC' = b
=> a=T
Por teorema:
sen!l
AA'+BB' = eos 411:.[2eos211: + 1JX-_7-x~
---=2--=m 7 7 sen!l 2
7
DD'-CC'
-----:2=--- =n 2eos 411:. sen 311:
De (IlI): 7 7
Sumando mienbro a mienbro: 2sen~
AA' + BB' DD' - CC'
2 + 2 =m+n sen 711:- sen!l
De (II): 7 7
AA' + DD' - (BB'+CC')
2 = m+n 2sen~

Dato: a-b
-2- = m+n ... C*)
En el cuadrilátero MRSN, en la base media 00' se Clave: A
cumple:
24. Sea S el número de grados sexagesimales y e el
00' = m+n número de grados centesimales, entonces:
2
a-b
~
De C*) 2
a-b ~ SQx400g = CK x 360Q
4 ~=.52..
Clave: D 9 10
Luego del enunciado se tiene:
23. Propiedades trigonométricas a aplicarse: • Para los grados:

cosa + cose = 2eoS(a; e). eos(a; e) (I) xy = SO


9 10
sena-sene=2eoS(a;e).sen(a;e) CII) ~ xy=45

sen3a = sena(2eos2a + 1) ~ x=4 1\ y=5


... CIII)
• Para los minutos:
En la expresión del enunciado:
zw = SO
F = eos 2 TC
+ eos ~TC+ tOS 6 TC 9 10
7 7 ~ zw = 45
= eos 6TC+ eos 2TC+ eos 4TC
777 ~ z=4 1\ w=5

. De (1) = 2,"f':2l{7 2~n} n


cos in En la expresión:

e= x+ y
z s- w

= 2eos 411: eos2TC + eos 411: = 4+5


7 7 7 4+5

= cos ~!l [2eos~TC + 1] =1


Clave: A
UNI 2005-11 MATEMÁTICA ~
~~
25. Para que el área del círculo sea máximo,éste
A o max = ITr2
debe estar ubicado en un lugar donde su diámetro sea
máximo.

= .!!. u2
4
Cálculo del área sombreada:

As = Asemic - (Asector AOP + AI10PB + Aa max )

A~------~L-~------~--~B
R a = 27t u2 - (237tu2 + J3 u2 + ¡u2 )
Datos: R = 2u 6 = E. = 1,67 u2
3 Clave: O
En el triángulo aTB :
26. Si en el gráfico trazamosBE, perpendicular a
h = Rsen6 = (2u)(i) = 1u la proyección AD, se trendrá:

Del gráfico ot: = R , entonces: B

R=h+2r
2u = 1u+ 2r
... (*)
~ r=lu
2
Para calcular el área de la región sombreada del grá-
fico, debemos restar las áreas que se muestran en
blanco, del semicírculo. Ak::......L::.---L----..u
1 2
Asemic = 27tR Del.gráfico se tiene:

=l7t(2u)2 BDcosy = ABsen(x - y)


2
... (*)
= 27t u2 ~ cosy = ~sen(x - y)

6 2
En la expresión:
Asector AOP = 2R
ABsen(x - y)
E = -----'---"---"
11 cosy
=1-(2u)2
ftÉ~~
De (*):
= 2IT u2 ftÉ sen{~
3 BD~-JJ
=BD
(2Rcos 30°) x h
A~OPB Clave: A
2

2(2 u) J3 (1 u) 27. Sean las funciones:


2
2
f (x) = ~" : n par

g(x)=~R2_x2 ; Rconstante
UN12005-11 MATEMÁTICA

Graficando las funciones: 29.Asignando sus longitudes respectivas:

y
1A

x
x

Como la función 1(x) es simétrica conrespecto al eje


y y g (x) es simétrica con respecto al eje X y al eje Y, H Itanel
las gráficas se cortan en puntos simétricos.
En la expresión del enunciado
E = tana + tan~ + cota + cot~
=l+1-+~+ -a Como eE II cuadrante, entonces:
a -a b b
lcosel.= -cose
= (~-!)+(~-~) ¡senel = sene
=0
Clave: A
Itanel = -tane

Cálculo del área AOC :


28. Aplicando la identidad:
AL>AOC = AL>ABC - AL>OBC
arctana + arctan~ = arctan( a + ~~)+ kt:
1-ax Icoselxl1+senel Icose Ix Isenel
Donde: <xx~<l => k=O 2 2
<x~~>1 a>O => k=l
-cosex(l+sene) -cosexsene
<xx~>l u e O => k=-l
2 .- 2
En la expresión:
... (1)
E = [arctan(t) + arctan(~)J+ [arctan(~) + arctan(~)]
Cálculo del área CDE :

=[arctan[ ~:~
-
1--x-
3 8
1] + [arctan[ ~:~
1--x-
5 7
1] A
L>CDE -
- I5ExCH
2
• 6 Itanel x (1-1cosel)
= arctan(;;) + a rcta n (1 7)
2
-tanex(l- (-cose))
=arctan[ M+f¿
l-llx~
l' 2
23 17 -tane x (1 + cose)
= arctan(l) 2
=IE = _l(tane + senü) ... (2)
4 2
Clave: B
Cálculo del área sombreada total: y

As = A"AOC + A"CDE

De (1) y (2): = (-~cosa) + [ -~(tana + sena)] tanx

= -.!(sena + cosa + tana)


2
Clave: A x
30. Operando en la expresión:

f(x) = (cos2x - 3)(-4 - 2sen2 x)


= (cos2x - 3)(-4.,...(1- cos2x))
= (cos2x - 3)(-5 + cos2x)
Del círculo trigonométrico se puede deducir que
= -5cos2x + cos2 2x + 15 - 3cos2x los valores que puede asumir x esta dado por::
= cos2 2x - 8cos2x + 15
=> xE (:!!.:!!) U (71t . 41t)
= (cos2x - 4)2 -1 ... (*) 6' 3 6 ' 3
Clave: D
Por propiedad de cosenos:
-1 $ cos2x $ 1 32. Porpropiedad:
-5 $ cos2x - 4 $ -3
sena+ sen(a + 2r)+ sen(a + 2r)+ ...
2
9$(cos2x-4) $25
sen(nr)
De (*): 8 $ (cos 2x - 4)2 - 1 $ 24 +sen[a+(n-l)r]=-_2_.sen(a+u) ... (1)
8 s f(x) $ 24 sen(~) 2
Clave:C Donde: Número de términos :n
Razón de la Progresión: r
31. En la expresión:
Primer término :a
~12./3 - 3tanx 1< m X E [O; 21t)
Último término
12./3- 3tanxl < ./3 En el enunciado:

Por popiedad: senl o+sen(l°+ 1x2°)+sen(1°+2x2°)+ ...

-./3 < 2./3 - 3tanx < ./3 +sen(1D+29x2o)=K ... (2)


4
- 3./3 < -3tanx < -./3 Donde:
./3 < tanx < ./3 a =J : r = 2° u = 59° ... (3)
3
De donde: En el último término:

Si: tanx =./3 => x=~


a + (n-1)r = 59°
3 6 1°+(n-1)2° = 59°
tanx =!3 => x=~ => n = 30° ... (4)
3
De la propiedad (1) en la expresión (2): 34. Complementando el gráfico:
A

De (3) Y (4)

01. b---l

1 Delgráfico:
=--Lxl h = bsen600
senl? 2
~ K = cosec1°
Clave: A
=b(~)
=J3b
33. Del enunciado: 2
B

Asemi e = ~( rr(%t )
=.1!b
8
2

a 2
AseclOr AOC ="2 R
·U-------~~------~C
n n rr
Aplicando Pitágoras en el triángulo rectánguloABC: = .Jb2
2
(2a/ = (2n)2 + (2m/ = !ib2
6
a2 =n2 +m2 ... (1)

En el triángulo AMC: bx(bsen600)


A""AOC = 2
MC2 = m2 + (2n)2

= m2 +4n2 ... (2)


_ bX(bJ])
- 2
En el triánguloABN:
-2 2 2 = J3b2
NB = (2m) +n 4
Cálculo del área sombreada:
= 4m2 +n2 ... (3)
Sumando (2) Y (3): As = Asemi e - (Asector AOC - A"" AOC)
2 2
MC + NB = (m2 +4n2)+ (4m2 + n2) = 1!b2 _ (!!b2 _ .J3b2 )
8 8 4
De (1): =S(m2+4n2)
= ~:(6.J3-rr)
=Sa2
Clave: B Clave: A
__ U_N_I_2_0_0_5-_II_M_A_T_E_M_A_'_T_IC_A __ I=::::*=:=i::~:~
35. Graficamos segun el enunciado y además tra- Aplicando propiedades se deduce que el cuadrilá-
zamos NA , para luego formar el triángulo TPQ, así: tero CFEG es inscriptible en una circunferencia, es
A decir:

W-----~~--~~--------~C
PQ // BC y PT // AM
Por definición:
Entonces TPQ es un triángulo rectángulo
Por semejanza en el triángulo AMN: EG =9
2
PT=2 ... (1)
EG = 800
Por semejanza en el triánguloANC: 2
PQ=3 ... (2) ~ EG=1600 ... (2)

En el triángulo rectángulo TPQ: Tambien:

2 m~EFG=EG
TQ = ~PT + pQ2 2
De (1) y (2): = h 2 + 32 De (2): = 160
2
0

=..[13
= 80
0
... (3)
Clave:C Luego por propiedad:

36. Del enunciado construimos la gráfica y además a. + m~EFG = 180


0

trazamos EC .
De (3): a. + 80 = 180
0 0

D
~ a. = 100°
Clave: C

37. La ecuación de una circunferencia se define como:


A~-----+-+~__4
C: (x-hl+(y-k)2 =r2 ... C')

y
C

Por propiedad:

m~ECG = 8 = 70° + 90°


2 .
~. 8 = 80° .i, (1)

x
En el gráfico del enunciado: 38. Graficando del enunciado y agregando algu-
y nos trazos auxiliares. V

~-+---~B

Dato: MN = 6../3 u
Corno el sólido es un tetraedro regular, entonces sus
En el triángulo rectángulo MTN:
cuatro caras son son triángulos equiláteros, y sus me-
dianas también son alturas de la misma longitud.
sen600 = MN
2r Del gráfico se deduce:
../3 = 6../3
AP = VP = ACcos30°
2 2r
~ r=6 ... (1)

En el triángulo rectángulo MTO:


... (*)
2r = OMsen60° El triángulo APV es un triángulo isósceles, por tanto
MP es altura. Luego en el triángulo AMP:
De (1): 2(6) = OM( ~)

~ OM=8../3 senS=AM
... (2) AP
Por proporcionalidad en el triángulo MTN: _ a
De (*)
- ../3a
MP=PN=~MN=~(6../3)=3../3 ... (3)
=../3
~ k=OM-MP 3
De (2) y (3): = 8../3 - 3../3
= 5../3
~ s=sen-
1
( 1)
Clave: A
En el triángulo rectángulo MO'P:
39. r ..los circunferencias tangentes exteriores:
sen30 =~
r
1 h
2=6" ~ h=3
Luego en (*) reemplazamos los valores de h, k Y r:

(x - 3)2 + (y _ 5../3)2 = 62
X2+y' -6x-10../3y+48=O
Clave: E
Si d < r1 + r2 entonces son circunferencias secan- 40. Graficando y adicionando trazos auxiliares:
tes. M
En el problema:
Sea LIla longitud de la circunferencia de radio R y
L2la longitud de la circunferencia de radio r, enton-
ces del enunciado se tiene:
Ira. condición:

... (1)

2da. condición:
B
LI+L2=141t Dato: a = 60°

De (1): (~L2)+ L2 = 141t En el triángulo rectángulo AOB:

~ ~ =41t ... (2) OB = J AB 2


- OA ~

Reemplazando el valor de (2) en (1): =~42_22


=2../3 ... (1)

Además por relaciones métricas:


... (3)
Para la circunferencia de radio R: OA OP
AB= OB
LI = 21tR
De (1): 2 h ... (2)
De (3): 101t = 21tR 4= 2../3
~ R=5 En el triángulo rectángulo MOP:
Para la circunferencia de radio r:
cosa = OP
L2 = 21tr MP
De (2): 41t = 21tr cos60° =!!.
d
~ r=2
De (2): 1 _../3 ~ d = 2../3 ... (3)
3ra. condición: 2-7
d=2(R-r) Tambien:

= 2(5-2) sen600 = OM
d
=6
De (3): ../3 = OM
Conclusión: 2 2../3
d=6<R+r=7 ~ OM=3
Ilas circunferencias son secantes! Clave: C
Clave:C
1 • Se ha mezclado tres sustancias de densidades a Se sabe que la moda de esta distribución es un valor
3 3
a g/em , b g/em y cg/em3ycuyasmasassonAg, impar y la diferencia de las dos mayores frecuencias
Bg y Cg, respectivamente, donde b < e < a y es 1. Calcule la esperanza matemática deX.
B < A < C .Entonces la densidad de la mezcla obte-
A) 4,0 B) 5,3 C) 6,3 D) 7,2 E) 8,6
nida en 9 /em3 , es:

(A - B + C)abe (A+B-C)abe 4. Si a, b, e son números positivos tales que:


B) beA + aeB + abC 2 6 6
A) beA + aeB + abC a +b 2
a =K
--"'---:--"---- = -b = _.
(A + B + C)abc (A + B +C)abe a + b6 + 3e c2 b6
C) beA + aeB + abC D) abA + beB + aeC entonces e - K es igual a:
(A + B + C)abe A) 1 B) 2 C) 3 D)4 E) 5
E) aeA + baB + ebC
5. Un grupo deA albañiles ha trabajado en una obra
2. El siguiente gráfico muestra las ventas y costos de D días a razón de Hhoras diarias; un segundo grupo de
una compañía entre los años 1996-1999 en miles de a (a < A) albañiles ha trabajado en la misma obra d
dólares. (d < D) días de h (h > H) horas diarias. En total re-
cibieron J nuevos soles. Entonces el primer y segundo
400 grupo recibieron respectivamente (en nuevos soles):
350
J .2J
300 A) 3'3
250 ------------------- ---------
adHJ ADhJ
200 B) adh + ADH ' adh + ADH
150 adHJ ADhJ
100
50

96
r-----
D Venta
~~-----

97
------

D Costo 1
98 99
r
• Costo 2
--
C) adH + ADh ' adH + ADh
D) adhJ . ADHJ
adh + ADH ' aelh + Ami
2adhJ . ADHJ
E) 3(adh + ADH)' 3(adh + ADH)
Entonces la utilidad (en dólares) en los cuatro años es:
6. Un capital estuvo impuesto al t % de interés anual.
A) 100000 B) 150000 C) 200000
Si se obtuvo un monto después de a añ.os deA nuevos
D) 300000 E) 350000
soles entonces el valor del capital es :
3. Se tiene la siguiente distribución de frecuencias de
Ata 100tA 100ra
una variable aleatoria discreta X, para un total de
A) -100 + ta B) 100 + ta C) 100 + ta
100 observaciones:
100A 100a
~1-'3=--_4-'--_.:..5_--'6=____'_7 _ _=.8_-'-9
_ _=.1_=_0
D) 100 + ta E) 100 +ta
110 12 18+p 18+q 4 8 15 10
7. Un número N de la forma N = abcabc ; ([ * O es A) 9 B) 10 C) 11 D) 12 E)l3
siempre divisible por:
A) 3 ,5 B) 7, 9, 11 C) 7, 11, 13 13. Como se indica en la figura adjunta se construye
progresivamente circunferencias tangentes de radio
D) 7, 17, 19 E) 9, 11, 19
cada vez menor, tangentes a dos semicircunferencias
8. Al descomponer en sus factores primos, los núme- de igual radio R. Use dicha construcción para deter-
ros A y B se expresan como: minar la suma de la serie infinita.

A = 3ub2;B = 31la _1_+_1_+_1_+ ... + 1 + ...


1x2 2x3 3x4 nx(n+1)
(con a y P consecutivos), sabiendo que su mínimo
común rnúltíplo y su máximo común divisor son 675 y
45 respectivamente, halle el valor más pequeño de
A +B.
A) 360 B) 368 C) 456 D) 720 E) 810

A) 2R B) R C) B. D) 1. E) 1
9. En base b se cumple que AAA x F = 1776. Enton- 2 2
ces , el valor mínimo de b para que se cumpla, la 14. Sea F(x1; x2) = ax1 + bx2, la función objeti-
condición anterior es: va del problema P.
A)7 B) 8 C) 9 P: minimizar F(xl ; x2)
D)lO E) No existe
sujeto a: (xl; X2) E S e lR 2
10. Calcule todos los restos posibles de la división de Si el lado CD de la región admisible S que se indica es
un cuadrado por 7: solución del problema P, determine a + b de modo
A) 1,2,4 B) O, 1,2, 4 C) O, 1,3,4 que el valor óptimo de F este entre 20 y 25.
D) O, 1, 3 E) 1,2,3,4, 5 x2

11 . Sea el intervalo cerrado [a; b] el complemento


del conjunto solución de la desigualdad: C=(4;3)

x2 _ (Vi + fi)x + 25/6 > O.

Sea también 6
\w -
\ s 3 Ya \z -
b6\ s 5. Entonces la
longitud del intervalo que recorre la variable real
A) 2 B)4 C) 6 D) 8 E) 10
w+ Z es:
A) 6 B) 8 C) 10 D) 13 E) 16
15. Al resolver la desigualdad
12. Una empresa contrató a un estudiante como
promotor de ventas de un producto y le dieron a elegir log¿ (!x2 - 3x + 3;) < O
dos modalidades de sueldo.
Determine la suma de todos los números x enteros que
Modalidad A: Una comisión de $3,20 por cada artí- la satisfacen.
culo vendido.
A) 2 B)4 C) 6 D) 8 E) 10
Modalidad B: Un sueldo fijo de $860 más comisión
de $1,80 por cada artículo vendido que exceda las 50 16. Determine la representación geométrica de to-
unidades. dos los puntos del plano complejo que satisfacen la
La suma de las cifras, de la cantidad mínima de artí- condición:
culos que debe vender para que la primera opción sea [z - 11 $ 6 -Iz + 11
más conveniente, es:
A) y 1. Sea f : IR ~ IR una función biyectiva y crecien-
(0;2f2) te, entonces f-1 : IR ~ IR es decreciente.

lI. Sean i .g : IR ~ IRfunciones decrecientes ta-


x les que fog existe, entonces fog es decreciente.
(-3;0) (3;0) III. Si f : IR
~ IR es una función creciente y defi-
namos una función g: IR ~:IR mediante
(0;-2f2) g(x) = f(lx!), 'i7' x IR. , entoncesg es creciente.
A) VW B) VFV C) FVV D) FVF E) FFF
B) y
(0;2f2) 18. Dados los 'números

= =
(-3;0) (3;0) x
O,ab
6
b 5 Y O,bá 5a ;6
1
Halle la tercera cifra decimal del valor que se obtiene
(0;-2f2) al sumarios.
A) 3 B) 4 C) 5 D) 6 E) 7
C) y
19. (xl,x2'···'x20) esun20-upladenúmeros
(0;2f2)
---- ,,
reales. Sea la ecuación:
,, ,,
2 2 2
f \ (xl-x2) +(x2-x3) +(x3-x4) + ... +
(-3;0) , X
, ,
" (3;0)
~
---
(Ó;-2f2)
(x19 -x20)2 + (x20 -xd2 =1

El número de 20-uplas de números enteros


y (xl' X2,···, X20) que son soluciones de la ecuación
D)
anterior es igual a:
A) O B) 1 C)l9 D) 20 E) 00

(-3;0) (3;0) X

entonces, la suma de los elementos de la diagonal de


E)
AIO es:

A) 40230 C) 60014
D) 60074
(2f2;0) X
21. Dos circunferencias el y e2 en un plano, son
tangentes exteriormente en P; Ll es una recta que
pasa por el centro A de el y es tangente a e2 en Q;
(0;-3) L2 es una recta tangente a e 1 y e 2 en B y E respec-
tivamente, tal que BE es paralelo a L1 . Si BP = 211,
17. Señale la alternativa que presenta la secuencia el radio de la circunferencia e2 es:
correcta, después de determinar si la proposición es
verdadera (V) o falsa (F).
C) ~ u D)~ II E)~ II
o I__ U_N_I 2_0_0_6_-I_M_A_T_E_M_Á_T_IC_A_ 1
•••••• * ._ID ",.~w;:~::r¡,,!':~
22. El área de la superficie total de un cono de revo- 12+../193
A) 12(12 + ../193) B)
lución es s u 2 y el Lriángulo rectángulo generador es 7
isósceles. Calcule el volumen. 12-../193
-12 +../193 D)

~U
C)

O) rr( ;, + ,) J u' E) -12 -../193


12 12

%( Hr
7
3 27. Se tienen dos circunferencias tangentes de radio
E) u R. Una tercera circunferencia de radio R rueda alrede-
dor de las otras dos. Determine la longitud del circuito
que recorre el centro de esta tercera circunferencia.

A) 41tR B) 81tR C) 41tR


3 3
23. Si a" + b" = nx , n E Z, determine el valor
D) 161tR E) 8rrR
simplificado de R. 3
cas(a¡ +k1)cos(a +k )cos(a
2 2 3 3 +k ) .. cos(o¡ +k;)
R=--~--~--~--~--~~~--~~~ 28. Sea h una función continua y par, definida por:
cas(k¡ - b¡ )cos(k2 - b2 )cos(k3 - b:¡) .. cosik, - b¡)
Donde: i = 1326 /¡(x) = (lsecx1 ; XE(_321t;a)U(b;321t)
A) -2 B) -1 C) ° D) 1 E) 2 Icosxl ; XE [a; b]

24. La ecuación de la recta que pasa por P(5; 6) Y Determine el valor de a - b :


por el baricentro del triángulo con vértices en los pun-
tos A(4;-3),B(-4;1l) y C(-6;1),es: A) -2n B) -n C) ° D) 1t E) 2n

A) 7x+3y-27=0 B) 3x+7y-27=0 29.Si


C) -3x+7y-27=0 D) -7x+3y+27=0
sen3 (S-*)+ sen3S+ sen3 (s+ *) = a sellS+ b scn38
E) 3x + 7Y + 27 = O
Determine: M = a - 3b
25. En el gráfico m(~§.!:rad~CD es un cuadrado de
A) 1,5 B) 2,0 C) 2,5 D) 3,0 E) 3,5
lado a unidades. Si AC y BD son arcos de circunfe-
rencia de radio a; ¿para qué valor de m, el perímetro
30. Si en la ecuación: cos3 x + sen3 x = 1 ,hacemos

A[]B
de la región sornbreada mide m(3 + x) unidades? 1t
y = X - 4" y luego z = cosy ,obtenemos una ecua-
ción polinomial en la variable z. Determine dicha
A) g B) g g
C) ecuación polinomial.
8 6 4

a
A) 12z3 + 3z + ..fi =O
D) E) g 2

D C
3 2
B)
12 3 -3,,2z-1
2z r:
= °
C) 12 z3 + 312 z + 1 = O
26. Si tan(a) = 3 - 2x y tan(S) = 10x - 2 ,
7 - 5x 4x + 1 D) ~ z3 - 312 z + 1 = O
donde ex y S son ángulos complementarios. Enton-
ces el valor de E) 12 z3 - 312 Z +1 =O
2
K = tan( %) tan( ex) es:
~~m:.-=~==j,-_U_N_I_2_0_0_6_-I_M_A_T;.;E;.M';';';';Á;.;.T.;.IC;.A..;..._I:=;:===~;:W 7@
G eZ

31 . En un triángulo ABC, el área es numéricamente del segmento EF que une los puntos medios de las
igual a seis.veces el circunradio, Determine: bases.
K = acosA + bcosB + ccosC

siendo a, b y e los lados del triángulo y


ángulos opuestos, respectivamente.
A) 18 B) 16 C) 14

3~. El valor de y = 2 are tan


D) 12
A ,B y

E) 10

(1) - are tan (_1)


C los

A) ADXBC/2
r \Y~ F

B) (BC - AD)/2
B

es Igual a: 3 7
C) (AB - DC)/2 D) (AB + DC)l2
A) ~ B) ~ C) ~ D) ~ E) ~
2 3 4 6 8 E) (AD + BC)/2
33. Un triángulo equiláteroABC está contenido en
un plano P, por el vértice A se traza un segmento AQ 38. La longitudes de los lados de un triángulo ABC
perpendicular al plano R Si I es el incentro del trián- son 3,4 y 6. Entonces el mayor radio de dos circunfe-
gulo ABC y AB = AQ = 6 u . Determine el área de la rencias iguales que pueden inscribirse en dicho trián-
región triangular QIB en u 2 . gulo, (dado en términos de r, radio de la circunferen-
cia inscrita al triángulo ABC) es:
A)4m B) m C) ~m
2 A)
6r
B) ~ C)
3r
r+3 , r+3 1'+3
D)3m E)2m
31' 21'
D) E)-
34. Las proyecciones de un segmento de recta AB 2r + 3 1'+2
sobre un plano y sobre una recta perpendicular al pla-
no son 40 u y 9 u respectivamente. Determine el área 39. Se inscribe un cono recto de revolución en una
(en u2) de la región encerrada por las rectas perpen- esfera, tal que la generatriz del cono sea igual al
diámetro de su base e igual a 2a. Calcule el área de al
diculares al plano trazadas por A y B, el segmento AB
esfera.
y la proyección de AB sobre el plano, si la menor
distancia que hay del segmento al plano es 21u. A) 5,33 rra2 B) 5,34rra2 C) 5,35 rra2
A) 1020 B) 1 080 C) 1140
D) 5,36rra2 E) 5,37 rra2
D) 1 200 E) 1 260

35. En el interior de un triángulo ABC (AB = BC), 40. El triánguloABC es equilátero de lado f y P es
se toma el punto "P" tal que PB = AC , un punto interior. Los segmentos PD , PE Y PF son
m L PBA = 10° Y rn L PBC = 30° . Calcule la medi- perpendiculares a sus lados.
da del ángulo PAB .
Entonces: PD + PE + PF es:
A) 15° B) 20° C) 25° D) 30° E) 35° BD+CE+ AF
B A) .f3
36. En un ángulo triedro O-ABC, los ángulos de 3
sus caras miden: nILAOC = nILBOC = 45° Y
m L AOB = 60° . Entonces, la medida del ángulo B) .f3
diedro A - OC - B es : 2

A) 30° B) 45° C) 60° D) 75° E) 90° C) .f3


D) 2
37. En el trapecio de la figura, los ángulos y y 8
AL-------~EL-----~C
son tales que y + 8 = 3rr/2. Determine la medida E) 2../2
t-
UN12006-1 MATEMÁTICA v::::="'",,, ,,;:te.WiJffl"m
I!;-::;· ;;¡V.a' .u ,~
~

SOLUOIONARIO
1 • Por dato se tiene que las densidades y masas, res- Año Ventas Costo 1 Costo 2 Utilidad
pectivamente, de las tres sustancias son:
1996 300 150 50 100
a g/m3
Sustancia 1
Ag) 1997 200 100 50 50
Sustancia II b g/m3 Bg ... (1) 1998 350 200 150 O
Sustancia III : c g/m3 Cg 1999 400 150 100 150

Por teoría, la densidad se define como: Siendo las unidades miles de dólares:
Utilidad total =300000 dólares
p _ masa _ m =} V = m ... (2)
- volumen - V P Clave: O

La densidad de la mezcla total: 3. Se tiene la tabla de distribución de frecuencias de


_ mT la variable aleatoria discreta X, para un total de 100
p Tocal - v:;: observaciones:

De (1)
A+B+C X34 5 678 9 10
"r + vII + v[f Ix 10 12 18+p 18+q 4 8 15 10
De (2)
A+B+C
~+-ª-+~ Por dato también se sabe que la moda es impar, es
a b c decir tendrá la mayor frecuencia, luego de la tabla:
(A + B + C)abc 18+p>18+q
bcA + acB + abC
~ p>q ... (1)
Clave: C
Por dato se sabe que el número de observaciones es
2. En el gráfico se muestra las ventas y costos de la 100; luego complementando con datos de la frecuen-
compañía: cia de la tabla:
10+ 12 + (18 + p) + (18 + q) + 4 + 8 + 15 + 10 = 100
400
=} p =5- q ... (2)
350
300 También, por condición del problema se tiene que la
diferencia de las dos frecuencias mayores es 1. Luego
250
considerando (1) se tiene:
200
150 (18+p)-(18+q)=1
100 p-q=l
50 De (2): (5-q)-q=1

'96 97 98 99 =} q =2
O Venta O Costo 1 • Costo 2 =} P =3

La utilidad de una operación comercial: Por teoría se sabe que la frecuencia relativa:

U = ventas - costos ... (*) ... (3)


Luego del cuadro y (*) construimos la tabla:
:.so:

Luego con los valores de p , q y (3) construimos la


~~------------~
J UN12006-1 MATEMÁTICA

Luego:
~,~
ComeZ

tabla: Pago total = J = ADHC + atlh C


X 3 4 5 6 7 8 9 10 LX ~ C= J
Ix 10 12 21 20 4 8 15 10 100 ADH + adli
Finalmente:
h 0,100,120,21 0,200,040,08 0,150,10 1
ADHJ
Pago 1= ADH + adli
Por definición de la esperanza:
Ex = I)lxX Pago II = adhJ
ADH +adh
= (0,1)3 + (0,12)4 + (0,21)5 + (0,2)6 +
Clave: O
(0,04)7 + (0,08)8 + (0,15)9+ (0,1)10
= 6,3 6. Según el enunciado:
Clave:C Capital: C
Interés: t % (anual)
4. Sean los números positivos a, b, c, tales que: Tiempo: a (años)
2 6 6 2 Monto : A (nuevos soles)
a +b = b = a =k
c2
a + b6 + 3c b6 Luego el monto ganado del capital, aplicando un
interes:
De esta expresión formamos las ecuaciones:
Monto = Capital + Interés
b6 2
I) -
2
= ~6 ~ b6 = ac ...(1) A = C + ( C x a x t% )
c b
~ C= __ A_
2 6 2
1 +at%
6
II) a +b - a = b = k 100A
a + b6 + 3c _ b6 c2 100 + ta
b6 b6 Clave: O
a+3c =-¿
7. Del enunciado se tiene el número:
~ c2 = a + 3c ... (2)
Para calcular c - k : N = abcabc a ~ °
(Propiedad) : = abc x 1000 + abc
c-k=c--- b6
a + 3c (Propiedad) : =abcx(1001) ... (",)
ac + 3c2 _ b6 Descomponiendo en sus factores primos a 1001, se
a + 3c tiene:
ac + 3( a + 3c ) - ac 1001=7xl1x13
De (1) y (2):
a + 3c
Entonces en (*) se tiene:
=3
N =abc (7xl1x13)
Clave: C
o <l fJ

=7=11=13
5. Asumiendo que C es lo que cobra cada albañil por
Luego se concluye que N es divisible por 7, 11 Y 13.
hora, se tiene:
Clave: C
Grupo Albañiles Días Horas Total Pago
horas 8. Al descomponer en sus factores primos los núme-
1 A D H ADH ADHC rosA y B se expresan como:
¡¡ a d h adh adhC A = 3ub2 ... (1)

... (2)
'~:::Zm- ~',~I
:2.!;¡~¡¡;:i14~,~
.. __ U_N_I_20_0_6_-_1 _M_A_T_E_M_A_' T_I_C_A __ I ~;;;,~
Por información del enunciado se sabe qw l ' máximo De la expresión (1), por propiedad se deduce:
cornún rnúltiplo de estos números:
b>7 :=} b=8;9;10 ... (3)
m.c.m.(A;B) = 675 = 3352 ... (3)
Por condición del problema se debe hallar bl1lill ,lue-
Por definición, el m.c.m. de dos o más números debe go de (3):
contener un número exacto de veces a estos números, bl1lill = 8
es decir puede ser dividido en un número exacto por
estos números, pero con la condición que sea mínimo. Reemplazando en (2), se verifica:
Para que lo anterior se cumpla, entonces necesaria-
A(b¡F(b) = 16(8)
mente tiene que cumplirse que:
A.F = 14
b=5 y a=5
lo cual se puede complementar confrontando (3) con F=7 ó A=7 F=2
las expresiones con (1) y (2). Entonces las expresio- Clave:B
nes (1) y (2), lo expresamos:

... (1') 10. La división de un número cuadrado por 7 es


equivalente a un número que sea múltiplo de 7 .
... (2')
Sea N el número múltiplo de 7, y para que la división
Observando las expresiones (1') y (2')se puede con- tenga residuo, este debe ser:
cluir necesariamente que: o., " () (1 <1 "

N = 7 ; 7± 1 ; 7+ 2 ; 7± ~ ; 7± 4 ; 7± 5 ; 7± 6
ex<~ ... (4)
Elevando al cuadrado, por propiedad:
Porque de lo contrario no se cumpliría la condición
del problema que A + B sea mínimo. Observe que N2=c7)2; c7+ 1)2; c7+2) 2 ; el +3)2; c7+4)2; c7 +5)2; el +6)2
3u esta multiplicando a 52 y 313 esta multiplicando =7;7+ 12 ;7+22;7+32;7+42 ;7+52;7+62
a 5, si ex > ~ entonces A sería rapidamente grande y o o o o o

habría contradicción con la condición del problema. =7; 7+ 1;7 +4;7 +2; 7 +2; 7+4;7 +1
o o o o

También, confrontando las expresiones (1') y (2')con =7;7+1;7+2;7+4


la propiedad del m.e.m. , la cual se resume en (3), se Luego, los residuos posibles que pueda tener el núme-
puede llegar a la conclusión que el exponente ma- roN son:
yor ~ del factor 3, por la condición (4) es: r=O;1;2;4
~=3 Clave: B
Además por condición del enunciado del problema,
11 . Dada la desiguldad :
ex y ~ son consecutivos, entonces:
cx=2 x2 T2 + J2)x + 2% > O
- (

Finalmente concluimos, el valor mínimo de : x2 - (T2 + J2)x + J2 T2 > O


( x - J2 )(x - T2) > O
Clave: A
+ 1 ! +
9. En la igualdad del enunciado:

AAA (b)' F(b) = 1776(b) '" (1) cs. = ( -00 ; T2) u (J2 ; 00 +)

111(b)·A(b).F(b) = 111(b) x 16(b) Luego, el complemento del conjunto solución (C.S.)


por condición del problema:
A(b)·F(b) = 16(b) ... (2)
ecs = [T2 ; J2 ] = [a ; b]
~i!:==::~===l"I
__ I,=~=::=:=~::
U_N_I_2_00_6_-_I_M_A_T_E_M_A_'_T_IC_A
__ lll~;:
De donde: La suma de las cifras:
". (1) 5+5+1=11
Clave: C
". (2)

Por el enunciado se cumple: 13. Como la serie infinita no contiene una relación
con el radio R, entonces esta es independiente de este
jw-a6j~3 valor, y por lo tanto, sólo se procede a operar:
De (1): Iw-41~3 , 1 1 1 1
S = 1.2+2:3 +3:4+' "11(11+ 1) +".
~ -3~W-4~3
1 s W ~7 ." (3)
=1
Por el enunciado tambien se cumple: Clave: E
jz - b6j ~ 5 14. En la función:

De (2): Iz-81~s
y el gráfico:
~ -s~z-:8~s
x2
3~z~13 ". (3)

Luego de (3) y (4) tenemos:


C=(4;3)
1+3~w+z~7+13
4 ~ w + z ~ 20
Entonces la longitud del intervalo que recorre la va-
--~----~--~-------~
riable real w + z es: Xl

20-4=16 Si P es la función que minimiza a F(x1 ; x2) y por


Clave: E condición del problema, el lado CD es la solución del
problema P , por lo tanto culaquier punto del seg-
12. Se debe cumplir:
mento minimiza.
Sueldo modalidad A > Sueldo modalidad B
Evaluando:
Si x es el número de artículos vendidos, se tienen los ". (1)
(2; 5): F",i/l = 2a + sb
siguientes casos:
1) » s so (4;3 ): F/11i/l = 4a + 3b
3,20x > 860 Como estas dos expresiones representan la cantidad
x » 268,75 mínima, entonces deben ser iguales:

iNo cumple la condición inicial! (x ~ 50) 2a + sb = 4a + 3b


2b = 2a
11) x> 50
b=a ". (2)
3,20x > 860 + 1,80(x - 50) Pero por condición del problema:
3,20x > 860 + 1,80x - 90
20 < F/11ill < 25
~ x » 550 De (1): 2 O < 2a + sb < 25
iSi cumple con la condición inicial! (x> 50) 20 < 7b < 25

Entonces, x /11;/1 = 551 ~ b = 3 (entero)


=1t.._.;.U_N_1 _20_0_6_-I_M_A_T_E_M_Á_T_IC_A __ I.· .~.t".'¡"W"¡;mm%~jb~
Luego, la suma de a y b: Pero por condición del problema x E Z , entonces:
a+b=3+3=6
Clave:C 3 2 5 7 4 9
2" 2" 2" 2"
15. En la desiguladad: x={2;4}

1
lag s ( x 2 - 3x + 3 5) < O
8
... (1) La suma de estos números:
2+4=6
Por propiedad se debe cumplir que:
Clave:C
lx2 _ 3x + 35 > O
2 . 8 16. De la expresión:
~ x2 _ 6x + 35> O ... (*)
4 Iz-ll$;6-lz+ll

( x - ~ )( x - ~ ) > O Por definición se sabe que: z = x + yi


Reemplazando en (*):
-----+--~~~+----- Ix+yi-ll $; 6-lx+yi+ll
5 7
2" 2" I(x-l)+yil $; 6-I(x+l)+yil
S}: xE (- 00 ; ~) u (~ ; 00 +) ... (2) )(X_l)2+y2 s 6_)(X+l)2+y2

Resolviendo la desiguladad (1):


Elevando al cuadrado ambos miembros y operando:
2
logsUx - 3x + 3 5) < O = logsl 3)(x_l)2+y2 $;9+x
8

~ 1x 2 _ 3x + 35 < 1 8x2 + 9y2 $; 72


2 8 2 2
L+_y--<l
lx2 _ 3x + 27 < O
2 8 32 (2J2)2-
x2 _ 6x + 27 < O La gráfica pertenece a la siguiente elipse:
4
y
( x - ~ )( x - ~ ) < O
2/2
+ 1 1 +
3 9
2" 2"
-3 3 x
S XE (1 ..2.)
2: 2' 2

Luego S1 n S2 : Clave:B

~----<ií=fj.----
~
2
5
2"
7
2"
9
2"
17. Analizando las proposiciones:
lo Falso (F). Sea f : R
creciente, entonces f
---7

-} : R
R una función biyectiva y
---7 R es decreciente,

Verificando:
c.s...(3.5)
2'2 u
(7.9)
2'2 Sea F biyectiva y creciente:
y y

x
x
G( x) no es creciente.
F-1 es creciente Clave: E

II. Falso (F). Sean 1, g : IR ~ IR funciones decre- 18. Dado los números:
cientes tales que log existe, entonces lag es
decreciente.
° , ab = b -6 5 ... (1)

Verificando:
Asumimos: F(x) = _x3 y G(x) =-x
°,ba = sa18+ 6 ... (2)

Donde F y G son decrecientes. Por teoría se sabe que:


Luego:
FoG(x) '= F(G(x)) °,ab = ab90- a ... (3)

= F( -x)
=_(_x)3
°'.ba ba90- b
= ... (4)

Igualando (3) y (1):


= x3
ab - a b - 5'
~ F oG( x) es creciente 90 -6-

~ 14b - 9a = 75 ... (5)


111. Falso (F). Si I :IR ~ IR es una función cre-
ciente y definamos una función g : IR ~ IR me- Igualando (4) y (2):
diante g (x) = f (Ix!), I¡f x IR, entonces g es cre- ba - b sa + 6
ciente. 90 18
Verificando: ... (6)
~ 3b - 8a = 30
Sea F creciente.
Resolviendo las ecuaciones (5) y (6):
a =1 ; b = 6 ... (7)
Sumando los primeros miembros de las ecuaciones
(1) y (2):

s = O,ab + O,ba
x De (7): = 0,16 + 0,61
=0,1666 ... +0,6111
= 0,7777
~
Luego: 3era cifra
decimal
G(x)=F(lxl)
La tercera cifra decimal es el número 7.
Clave: E
:¡;
G

19. Se muestra una 20-upla de números enteros, tal 21. Del enunciado:
que:

(Xj-X2)2+(X2-X3/+(X3-X4)2+ .. +(x20-x¡)2=1

Resolviendo:
A L¡
2(x~ + x~ + x~ + x¡ + ... + x~o

+X1X2 + x2x3 + x3x4 + ... + x20x1) = 1


2r
Se deduce que lo que se encuentra dentro del parénte-
sís también es un número entero, entonces:
2N = 1
B
Por semejanzas de triángulos:
~ 2 =I
ABP - QMP
Ésta igualdad es absurda. No existe soluciones por
tanto el nLlJ;1erode soluciones es nula (O). 2r r
2 PQ
Clave: A
~ PQ =1 ... (1)
20. Dada la matriz:
En el gráfico se observa que APB es un triángulo
isósceles, luego si trazamos la alturaAR y considera-
A=['~O O~ ~]3
2~VQ
mos (1), podemos graficar:
,

~ DIAG(A) = 11+21+31=6 ... (1)

Calculando el cuadrado de la matrizA:


5 5
A2 = AA =
[~ 2
O ¡m ¡] 2
O
B
Por propiedad se tiene:
15
22
39]
35
BA
2
= BRx BQ
o [: (2r)2 =l x 3
O 32
~ r = 13
~ DIAG(A2)=12+22+32 = 14 ... (2) 2
Calculando el cubo de la matrizA: Clave: D

22. Del enunciado, el triángulo generador del cono


1 35 223]
A
3
=A A=
2

[
O 2
3
133
de revolución es isósceles, es decir:

O O 33

~ DIAG(A3) = 13+23+33=36 ... (3)

Induciendo de (1) , (2) Y (3) se tiene:

DIAG(A10) = 11°+21°+31°= 60074


Clave: D
~~~~m'A_:L.I_.;;U:.;,N;.;,I.;;2;.O,;;.06,;;. I=:=;;';::'tlr:;
..;.I..;M;.;,;A..;T;.;E;;,;M.;...Á
..T;.;IC_A
__

El cono de revolución generado: Para un i = 1326; (condición del problema)


1326( 1326+1}
R=(-l) 2

= (-1 )879801 ; (exponente impar)


= -1
Clave: B

La superficie total del cono de revolución: 24. Se tiene los vertices del triánguloABC:
2
ST =S= rr(r u)2 + rrr(rJ2) u
A = (4;-3) ; B = (-4;11) ; C = (-6;1)
S = rrr2(1 + J2) u
2
Por definición, el baricentro del triánguloABC:

~ r = J rr( 1 !J2) u ... (*) (xc;yc) =(


XA+XB+XC
3
. YA+YB+YC)
' 3

El volumen del cono: = ( 4 - 4 - 6 . -3 + 11 + 1 )


3 ' 3.
Volumen = i( rr(r u)2 )(1" u) = (-2 ; 3)

= lrrr3 u3
3 B 11

De (*):

Clave: D

23. Para resolver la expresión:


cos( al + k1 )cos( az + k2) ... cos( C/¡ + k¡ )
R= ...(1)
cos( k1 - b1 )cos( kz - bz) ... cos( k¡ - b¡ )

Dato: a" = nrr - bll ... (2)

Considerando (2) podemos definir las expresiones:


-6 -4 4 5
cos(a" + k,,) = cos[(rl1r - bn)+ k,,]

= (-l)"cos(kn -bn) -3 A
Entonces:

cos(a,,+kn) (-l)"cos(k".-bn)=(_l)" ... (4)


Como: P y G E e (recta), donde: P = (5; 6)
cos( k" - b" ) cos( k" - bll ) Ecuación de la recta:
Aplicamos la iguladad (4), del término n-ésimo, para
Y-YP=YP-Ye.
resolver: 11 = 1 ; 2 ; 3 ; ... ; i: x-xp xp-xc

En (1): R = (_1)1 x( _1)2 x( _1)3 x ... x (-1)¡ y-6_ 6-3


x-S-S-(-2)
= (-l)x (l)x (-l)x ...(-1)¡
~ L1 : -3x + 7y - 27 = O
i(i+1}
= (-1}-2- Clave: C
__ U_N_I_2_0_0_6_-_1 _M_A_T_E_M_Á_T_I_C_A I:~ytffi==0
26. Se tiene las expresiones:
tga = 3 - 2x ". (1)
7 - Sx

tg8 = 10x - 2
". (2)
4x + 1
Por dato se conoce que a y 8 son ángulos comple-
mentario, es decir:
a+ 8 = 90°
~ tga = ctg8
De (1) y (2): 3-2x 4x+1
7 - Sx 10x - 2
e
~ x"= 13
11
Del gráfico se observa que AP y DP son radios e
iguales a "a"; por lo tanto el triángulo APD es Reemplazando el valor de x en (1):
equilátero.
193 7 3 2(13)
tga = 11 =L
Por propiedad, el arco pjj mide igual que el ángulo
central BAP, es decir:
~ 12 7- s(in 12

m PB = m <s:.BAP· Además por identidades trigonométricas:


1(

'6 ". (1) tg( % ) = csca - ctga ".(3)

Finalmente calculamos el valor de K en la expresión


Por definición, la longitud del arco PB :
K = tg( %) tga
Lp¡¡ = a( ~) ". (2)
De (3): = (csca-ctga)tga
= seca-1
Por simetría la longitud del arco PB y Fe son igua-
les, entonces el perímetro de la región sombreada: . = -/193 _ 1
Perímetro PBC = Lp¡¡ + a + Lp(; 12
_ -12 +.J193
De (2): =a(~)+a+a(~) - 12
Clave: C
=a(~+l) ".(3)
27. Sean el y ezlas circunferencias tangentes de
Pero por condición del problema, el perímetro de la radio R, y e 3 la circunferencia que va a rodar alrede-
región sombreada mide: dor de las dos, la cual también se debe encontrar en
Perímetro PBC = m( 3 + 1()
contacto para iniciar su movimiento.
""(4)
Igualando (3) y (4):

a(~+1)=11l(3+1()

Clave: D
K§>' ~·'~~;::~*~~.::"":,;::;'::N":.~:.
~..~ 1__ U_N_I_2_DD_6_-I_M_A_TE_M_A_' T_I_CA_.....Il=. '-::;::::;::;;:=:'2

Observe que si las circunferencias se encuentran en y


una posición de tangencia, sus centros formaran un
triángulo equilátero de lado 2R.
Cuando el centro de la circunferencia e3 ha recorri-
do desde el punto "A" hasta el punto "B" por la iz-
quierda, se ha desplazado la longitud LI . A lo largo
del recorrido, el centro de e3 se mantiene a una dis-
tancia de 2R del centro de la circunferencia el' por lo
tanto este será su radio de giro.

Como la función es continua y par, entonces esto no


debe tener interrupción y debe ser simétrica con res-
pecto al ejey, veamos:
y

I
I
\

-..•. _-_ ..• I

:-.- Eje de simetría


Del gráfico de I cosx I ,donde x
¡¡
"2

E [a;b J
¡¡ 3¡¡
2
,se puede de-
Por definición y del gráfico se puede deducir que: ducirque:
L1 = e(2R) a = -1t: Y b = 1t:

= ( ~1t )( 2R) => L1 = 8 1t:


R Luego para calcular:
3
a - b = (-1t:) - 1t:
Si la circunferencia e, rueda al rededor de e1 y e2,
= -21t:
su recorrido total es:
Clave: A
L = L¡ + L2
= 81t:R+ 81t:R 29. Identidades trigonométricas:
3 3
• sen3x = 3scnx - 4sen3 x
= 161t:R
3 => sen3 x = ~(3SC1~X - sen3x', ... (l)
Clave: O

28. Definida la función H, continua y par:

H (x)
_¡lsecx1; XE (- 3 1t:;
2
a)u (b; 3;)
-
[cosx] ; XE [a; b]

Graficando la funciones Is ec x I y Icos x] :


Operando en F¡ ,para llegar a algo sirnilar a Fz :


4l~~.~~~~~~~~ªªª
GomeZ
1·~::~~~::::::2;.~ U_N_I_2_0_06_-_I_M_A_T_E_M_Á_T_IC_A
~

También de (2):
F¡ =sen3( 0- ~)+scn38+scn3( 8+:}
:;2. = 1 (senx + cosx l
De (1) =i[ sen( e-* )-sen( 38- 3:) ]+i[3scn8 2
z2 =!( sen2 x + Zsen x cos x + cos2 x)
-sen 381+i[ 3sen( e+* )-sen( 38+ 3:)]
2z2 = 1+ 2senx cosx
=i{ 3[ sen( 0-* )+sen( e+* )+scnO]-
2
=} senx cosx = 2z - 1 ,., (4)
+[ sen( 38- 3: )+sen( 38+ 3: )+sen30 ]} 2

De (11): =i{ 3[ 2senO cos*+seIl8 ]-[ 2scn38 cos3: En la expresión (1), aplicando (1):

(cosx + senx)( cos2 x + sen2 x - se n x cosx) = 1


+sen381}
(cosx + senx)( 1 - se/u cosx) =1
=i{ 3(J2senO+senO )-( sen38-J2sen30)}
Reemplazando (3) Y (4):
=%( J2+1 )sen8+i( J2 -1)sen30 2
( J2z )( 1- 2z - 1 ) = 1
Comparando esta última expresión de F1 con F2 ' se 2
puede deducir que:
J2z3 - 3J2 z + 1 = O
2
Clave: E
Luego para calcular:
M = d> 3b 31. Identidad trígonométrica: Si a + /3 + 8 = 180 0
,

= ¡(J2 + 1) - 3[ i( J2 - 1)] entonces:

= 1,5
sen2a + sen/3 + sen28 = 4sena senj3 sen8 ,., (1}
Clave: A Propiedades de triángulos inscritos:

30. Identidad algebraica:


a3+b3=(a+b)(a2+b2_ab) ",(1)

Identidad trigonométrica:
cos( 8 - /3) = cos8cos/3 + sen8sen/3 '" (11)

En la ecuación dada:
cos3x + sen3x = 1 ,,' (1)
Además del enunciado se tiene:
y = x - ~ z = cosy
4

=} z=cos(x-%)

De (II): =cosx cos%+senx sen% Por propiedad:


mQR = 2m4:QPR
=cosx cos%+senx sen%
HR= rsena
= 1 (senx+cosx) ,., (2) =} QR = 2r sena '" (11)

=} cosx + senx = J2z '" (3)


~~=~::::::~~·~I
También:
__ U_N_'_20_0_6-_'_M_A_T_EM_A_'T_'C_A 1 -~~~~~~,=
En la expresión del enunciado:
.....
=.~.
,.. (III) K = acosA+bcosB+ccosC
OH = rcosa
El área del triángulo QOR; aplicando (II) y (III):
De (11): = (2RscnA )COSA+( 2RscnB )cosB+
(2RscnC )cosC
S _QRxOH (2rscna)(r cosa)
J - 2 2 = R( 2senAcosA+ 2scnBcosB+
=r2scna cosa 2scnCcosC)

=r2( ~sen2a) = R ( scn2A + sen2B+ sCIl2C)


1'2 De (1): = R( 4scnA senE scnC)
="Tscn2a
= 4R senA senB scnC
Por similitud se induce:
r2 De (2): =4R(~ )
S2 = "Tsen2p 2R2
Por dato se tiene que S = 6R , entonces:
1'2
S3 = "Tscn28
k = 4R( 6R )
El área total del triángulo inscrito: 2R2
= 12
5 = 51 + 52 + 53
2 2 Clave: O
r
= "Tscn r
z« + Z-scn r2
2[3+ Z-scn 26
.2 32. Propiedades:
=~(scn 2cx+scn213+scn 26)
.2
• Si xy c I =>arctgx+(Jrctgy=(~rctg( ;_+~) ...(1)
De (l): =T(4 senn senb senü )
• Si x2<1 =>2arctgx=arctg=(~) ... (ll)
= 2r2 scnascn13sen6 ...(IV) l-x2
Del enunciado del problema se tiene: • Si aretg( -x) = -are Lgx ... (111)
En la expresión del enunciado:

S: área del triángulo y = 2arctg( ~) - arctg( -~)

De(ll): = arc tg[ 2(~) 2]- arc tg( -~ )


1-h)
De(lll) : = arctg(%) + arctg( ~)
C 3 1 \
Dato:
S = 6R ... O)
De(J): =arctg
[1
4+7
_l(l)
I
Por (IV) se deduce que el área del triánguloABC es: 4 7 ,
= arcrgO)
5= 2R2 scn AscnBsenC
re
~ scnAsenBsenC = ~ ... (2) 4
2R Clave: C
-~~~~i:::::~
~1 U_N_12_0_0_6_-I_M
__A_T_EM__Á_T_IC_A _

33. Del enunciad»: Luego el área del triángulo QlB:


Q 5= QRx lB
2

De (3) Y (4):
(3J5)( 213)
2
= 3m
Clave: D

34. Del enunciado:


B

Datos: AQ = AB = 6 u ... (1)

Del enunciado se sabe que el triángulo ABC es


equilátero, entonces:
AB=AC=CB=6 u ... (2)

Además se tiene que l es el incentro del triangulo Aét",


entonces:
AC 1.BR R: punto medio de AC
Las proyecciones de AB sobre el plano y la recta son
También se tiene:
A"B" y A'B' respectivamente.
QA 1. AR => QR 1. RB
La menor distancia de AB al plano es:
(Teorema de las tres perpendiculares)
A'A"=21 u
Luego en el triángulo rectánguloAQR, por datos:
Cálculo del área sombreada:

6l
Por pitágoras: A'A"+B'B")-

r
AREA(BAA"B") = ( 2 B"A"
2
QR = JAQ2 + AR
. = .J62 + 32
=( 30 u;21 u op

A 3 R
= 3J5 ... (3) = 1020 u2
Clave: A
En el triángulo rectángulo RBC, por datos:
35. Del enunciado:
B
RB = .J62 _ 32
B~
= 3J3
R 3 C Datos:
AB=BC
Por propiedad del incentro, el segmento RB queda
dividido en : => ().AB C es isósceles
RI _ 1
lB - 2

=> RI = 13 ; lB = 2J3 ... (4) AL-~----~----~~C


~::::=::::=::J
__ U_N_I_2_0_0_6_-_1 _M_A_T_E_M_Á_T_I_C_A __ I;:~:;::;;~;:~~~;::~~~:~:;;:::¡;:::?~~~~
Por propiedad: 36. Graficando segun el enunciado; pero tomando
OB = BA arbitrariamente con el objetivo de facilitar
m<t:BCA = m<t:BAC = 70° los cálculos:
Además: BP = AC
Si trazamos el triángulo equilátero BQA:
B

Del gráfico se deduce que el triángulo OBA es


equilátero y el triángulo CBA es isósceles. es decir:

BC = CA ... (")

Del gráfico se observa que QB = B C y además: Por pítágoras:

m<t:QBP = m<t:BCA = 70° 1-2 -2 r:


VCB + CA = ,,2 a
entonces se concluye que: 1-2 -2 r:
VeB + Cñ = ,,2 a
/',QBP == /',BCA => QP = a
~ J2 eB = J2 a
Luego también se deduce que Q es circuncentro C A => eB = a
Considerando (*) se concluye:
Cñ = eA = a
Luego el valor del ángulo x corresponde al triángulo
notable de 45°.
=> x = 90°
Clave: E

37. En la figura dada trazamos las líneas paralelas a


los lados no paralelos del trapecio y completamos con
algunos datos.
Por condición del problema E y F son los puntos me-

7,b&
dios de los lados paralelos oe y AB del trapecio.
Del gráfico: m<t:BP = 40°

=> x = m<t:BP
2
40°
=2
= 20° Al lB
Clave: B •
M
a ----f. N
a---1
Del gráfico:

DC = 2b ... (1)

AB = 2a => a = AB (2)
2
Además del enunciado del problema se tiene:

8 + y = 3rc ... (3)


2 6 -------
En el vértice D por propiedad:
Como AC > AB > B C , las 2 circunferencias deben ser
a:+8=1t ... (4)"
tangentes aAC para que sean lo más grande posible.
En el vértice C por propiedad: Observe quel, My N son centros de las circunferen-
y+0=rc ... (S) cias; y "x" es el radio de mayor tamaño de las circun-
Sumando (4) Y (S): ferencias iguales.
Además por ser circunferencias tangentes al triángu-
(ex+8)+(y+f3)=1t+rc
lo, A, M Y l son colineales al igual que I, N Y C.
ex+ 0 + ( 8 + y) = 2rc
Luego podemos graficar:
De (3): ex+ 0 + 3rc = 21t ,
2 ,~l ..... ..,..-\',

=> a:+f3=~

Entonces se puede deducir que:

m<r.MEN =~
6 .IC
Luego en el triángulo rectángulo MEN: Del gráfico se deduce: AC / / MN
Por lo tanto, por semejanza de triángulos:
M/N - AlC

MN ~AC
Jp TQ
--2x 6
- => x = 2!:._
r - x T r +3
Por propiedad MN es diámetro de circunferencia, en-
Clave: C
tonces:
EF = a - b 39. Del enunciado:
p
De (1) y (2): AB
--- DC
2 2
AB-DC
2
Clave: C
UN12006-1 MATEMÁTICA

Proyectando sobre un plano perpendicular a la base F


del cono. r
p

c
~ :-H-]º~--
I
p
I
I
I
I
I
I
AL....l--...LL.+f---'-'E

Del gráfico:
c sen60° = r sen30° + t

cJ32=!:..+t
2 ... (1)

En el cuadrilátero PECD: se trazan las paralelas PD y DC


Del gráfico se deduce que el triángulo PMN es D
equilátero.
s
Luego en MOH:
p.
cos 30° = Q.
R
J3 a _ R = ~a (~)
2:=R - J3 ... <

El área de la superficie de la esfera:


5= 4rrR2 Del gráfico:

De (*): = 4rr( l3 a J a sen60°

aJ32 = l+
2
= t sen30°

s
+s

... (2)
= 5,33 rra2
En el cuadrilátero FBOP: se trazan las paralelas de FP y FB.
Clave: A
B
Del gráfico:
40. En el gráfico dado completamos algunos datos
b sen60° = s sen300+ r
que se generan y agregamos algunos datos a calcular:
B ... (3)

F D
r
P
Sumamos (1) (2) Y (3):

cJ3 +aJ3 +bJ3 =!:..+t+l+s+l.+r


2 2 2 2 2 2
Debemos hallar:
PD + PE + PF
BD + CE + AF
s+t +r
b +c+a
1 (c + a + b) = ~(r + t + s)

~ r+t+s_J3
En el cuadrilátero AFPE: se trazan las paralelas de PE a+b+c-T
yAE.
Clave: A
L- UN12006-11 MATEMÁTICA 1·;::=:
••••••.
:·.<:"':':"':'.:· ,:¡¡;;:,¡¡;:i&==: .. ~4
,~&; ni 1-Mb ..i· ~:¡¡.,.[".• ~

MATEMÁTICA

1. De la igualdad a2b(7) = aS 1( 11) calcule el va- 5. Cinco amigos recogieron en una isla un cierto
lor de a + b + 11 • número de cocos y acordaron repartirlos al día si-
guiente. Durante la noche uno de' ellos decidió se-
A)ll B )12 C) 13 D)14 E)15
parar su parte y para ello dividió el total en cinco
partes y dio el coco que sobraba a un mono y se fue
2. En una barra de madera de 30 c.m se realizan a dormir. Enseguida, otro de los amigos hizo lo mis-
n cortes tal que las partes obtenidas Al' A2, ... , mo, dividiendo lo que había quedado por S, dando
son proporcionales a los números 1,2, .... La me- el coco que sobraba a un mono, uno tras otro hicie-
dia aritmética de las inversas de la menor y mayor ron lo mismo, dando a un mono el coco que sobra-
de las partes es: ba. En la mañana se repartieron los cocos sobran-
tes quedando un coco. Cuál es el número mínimo
é

A) 120 B) 60 C) 120
11+2 de cocos que se recogieron?
(n + 2)2 (11 + 1/
A) l4 521 B) l4 581 C) l4 621
2 2
D)(11+1) E) (n + 2) D) 15581 E) 15 621
120 120
6. Determine la suma de todos aquellos números
3. Una persona dispone de un capital C nuevos naturales tales que su raíz cuadrada, con un
soles que lo ha dividido en tres partes para impo- . ., menor (1e S
aproxirnacion 3 ' es 48, .
nerlas al a % , al 2a% yal (2a + 2)%, respecti-
A) 230 B) 259 C)282
vamente. Sabiendo que todas las partes le produ-
cen igual interés. Entonces la parte impuesta al D) 289 E) 312
2a% es:
(2a + l)C 7. Sean aa, bC y (b + 1)( e - 2) tres números pri-
B)(2a+1)C
A) 4a + 1 mos, tales que el primero divide a la suma de los
4a + 3
otros dos. Si r, y r2 son los restos dividir el segundo
(a + 1 )C (a + l)C entre el primero y eltercero entre el primero res-
C) 4a + 1 D) 4a + 3
pectivamente entonces r1 - 1"2es igual a:
(a + 1 )C A) 8 B) 3 C) 1 D) --3 E) -8
E) 4a + S
8. Si se sabe que:
3. Sean a, b, e y d números naturales tales que ~-~~-~
MCD (aae, (a-l)(a-l)b)es 15
I) Q.=a+c=Q=k; keN/{1;2}
b d C
MCD (aac, da( a - 1)) es 66
I1) d-e=39
Determine la suma de todos los posibles valores de
Entonces el valor de d - b es: a+b +e + d.
A)l B) 3 C) S D) 7 E) 11
A) 23 B) 24 C) 25 D) 26 E) 27
9. A un trabajador le ofrecen 5/.400,00, una lava- A) B)
Y Y
dora y una refrigeradora por realizar un trabajo en
medio año, pero éste se compromete a realizar par- ,..• ---, ,
, ,,
I \
te del trabajo en 4 meses, pidiendo que le entre- I
(-5/4;0 ) \
X
guen SI. 50,00, la lavadora y la refrigeradora. Lue- \
\
J
I
(0;0) X
go de empezar el trabajo, el trabajador se retira al \
, r=3/4/
cabo de 2 meses y medio recibiendo SI. 10,00 Y la
.... _-_•.•. ,

lavadora. Halle el valor de la refrigeradora.


A) SI. 200,50 B) SI. 210,50 C) 5/.215,50 C) y
......• -- ......
D) SI. 220,50 E) SI. 222,50
I
I
. I

10. Indique la secuencia correcta después de de-


terminar si
la proposición es verdadera (V) o falsa
X X

(F):
I.Si A={0}, entonces AcP(A); peA)
potencia de A
E) Y
n. AóBE P(AvB)
","""--- ...
I ,
III. Si A \ B = 0 entonces A=B
" (-3/4; O) '. x
A)VVV B)VVF C) VFV
\ r=3/4 (9~0)
D) V F F E) F F F /

11.Katty, Lama y Liz hacen las siguientes afirma-


ciones respecto a un número irracional x.
14. El conjunto de los números reales que satisfa-
Liz: Si le sumo otro irracional el resultado i- ce la inecuación log¿ (x + 3 - 3rx+l) < 1 es:
gue siendo un número irracional.
Katty: Si a, b, e, d son números racionales tal A){a/-1::;a<0}
que, a + bx = e + dx ~ a = e y b = d· B) { a/O < a < 3}
Laura: La solución del sistema yl/11 - x = ° C){a/-1::; a< í s¡
es irracional, donde n E N
D) {a/3<a<lS}v{x/x>30}
Son correctas:
A) Liz B) Katty C) Liz y Laura E){ a/ - 1 s a < O} v {x / 3 < x < 1 5 }
D) Liz y K~tty E) Katty y Lama
15. Halle el valor de E = 4x + 3y , donde x e y
son los valores enteros que satisfacen el siguiente
2. Dada la función: ¡(x) = K + x ~ K ; V(x * K) sistema de inecuaciones.
Halle todos los valores que puede tomar K para que
sx-3Y>2

l
la gráfica de la función f y de su inversa sea la mis-
ma. 2x+ v« 11
y> 3
A) [1; 2) B) [O; 1] C) [-1; 1]
A)20 B) 24 C) 25 D) :12 E)36
D)[O;+oo) E) (-00; +00)

13. Si z = x + iy , grafique todos los puntos en


el plano cartesiano que representa el conjunto 16.Sean las matrices Q =[ ~
7 -1]
1 1 ,P = QI01
-1 4 -4
,-_U_N_I_20_0_6_-_II_M_A_T_E_M_Á_T_IC_A
__ I=t!=~~~~
20. Halle el argumento de un número complejo
que equidista de los complejos -2, - 2i Y
312 (cosrr/ 4 + sen rr/ 4) .
í

A)1t/8 B) rr/6 C) 1t/4


cierto número real. Entonces, el vector u y el D)1t/3 . E) 2rr/3
número a tales que Pü = aü son:
21 . Dadas las siguientes proposiciones:
r. El conjunto convexo más pequeño que contie-
Alm,O Blm,-1 Cl[~l,1 ne a tres puntos no colineales del plano es la
región triangular cuyos vértices son dichos
puntos.
Il. El conjunto S = {x lR/lx I > 1} es convexo.

EfHo
E

III. Si al borde de un círculo se le quita un sólo


punto, el conjunto resultante ya no es convexo.
Son correctas:
1 7. Dadas las siguientes proposiciones respecto a A) 1 Y III B) Solo II C) 1 Y II
la programación lineal:
D) II Y III E) Solo 1
1. Las restricciones de desigualdad son
polinomios de primer y segundo grado. 22. Las bases de un tronco de cilindro oblicuo de-
Ir. El punto óptimo se encuentra en la región ad- terminan un ángulo diedro de 90° y la distancia
misible. entre los centros de dichas bases es de' 16 m. L1
IlI. La región admisible contiene puntos, los cua- proyección ortogonal de las bases sobre un plano
les tienen alguna de sus coordenadas valor ne- perpendicular a la generatriz es un círculo de radio
gativo. 2 m. Si la generatriz hace un ángulo de 45° con
las caras del diedro. Calcule el volumen en 1113 de
Son correctas:
dicho tronco de cilindro.
A) Solo I . B) Solo III C) Solo II
D) Solo 1 y n E) Solo n y III A)40rr B)48rr C)56rr D)64rr E)72rr

18. Para la sucesión definida por 23. Uniendo todos los centros de las caras de un
tetraedro regular se obtiene otro tetraedro regu-
SK = t(---;-)
11=1
, 2 +11
K ~ 1 , se puede afirmar:
lar. Si llamamos AJ al área del primer tetraedro y
A2 al área del segundo tetraedro, entonces ~~
es igual a AJ
A) 1 s SK D) ~ s SK <1
1
1 . A) 24
1<S 1 2: s1
B) 4 - K < 2: E) < SK

1 1 24. Se construyen exteriormente los triángulos


C) 8 ~ SK s 2: equiláteros AEB y BFC sobre los lacios AB y BC
de un triángulo escaleno, tal que XF n CE = {p}.
19. Una ecuación cuadrática tiene como raíces Calcule la 111 <l: APC .
a!'>.+ 4 Y !'>.- 2 . Halle la suma de las cifras del
producto de estas raíces, siendo !'>.el discriminan- A) .!!: B) 2rr C) 3rr D):~ E) ~rr
te de la ecuación. 2 3 4 5 6

A) 10 B) 11 C) 12 D) 13 E) 14
~5:;:;¡¡¡¡;¡¡====:"=,,...,.........~=I__ U;;.;N..;.I..;2..;0..;.O..;.6-..;.II...;;.M.;..A;.;.T..;.E ..M_Á..;.T.;..IC;.;.A I~zw¡:k<

25. La suma de las medidas de cinco ángulos in- 29. Se tiene tres semicircunferencias como se
ternos de un polígono convexo es 760°. Calcule la muestra en la figura. El segmento EH es perpen-
suma de las medidas de los ángulos externos co- dicular al diámetro AB y mide li . Determine el
rrespondientes a los vértices restantes. área de la región sornbreada en términos de h .
A) 190° B) 200° C) 210° D) 220 0
E) 230 0
E

26. En la figura, EF es tangente a la circunferen-


cia inscrita en el triángulo ABe. Halle el perímetro
en metros del triángulo EBF; si AB= 10 m,
BC = 12 m y AC = 11 m .

B A) 8
L---------~~H--------~B
B) 9

C) 10
A) 1t(~ t
D) 11

E) 13 D) 1t(~ )
30. Sean las rectas LI y L2 que se cruzan en el
espacio formando un ángulo recto y AB un seg-
27. Indique la secuencia correcta después de deter-
mento perpendicular a ambas rectas con A E LI Y
minar si la proposición es verdadera (V) o falsa (F):
BE L2 . Sobre LI se ubica el punto C y sobre L2
l. Dos triángulos rectángulos con la misma
el punto D tal que
hipotenusa son congruentes.
Il. Dos triángulos rectángulos isósceles con un ca- AC2+BD2+AD2+BC2= 32
teto común son congruentes.
, IlI. Dos triángulos rectángulos con un ángulo agu-
Calcule la longitud del segmento C'l5.
do de igual medida son congruentes.
A)J2 C)2 E)4
A) F F F B) FVF C) VF F
D) VVF E) FVV 31. Un automovilista viaja en una carretera pla-
ha, en dirección a una montaña a 6 O k.m / h . En
28. La figura muestra una circunferencia de radio un instante observa la de la montaña con un ángu-
r inscrita en el triángulo rectángulo ABC, calcule el lo de elevación de 30° y 10 minutos más tarde vue-
valor de x (en función de e, by r). le observar la cima con un ángulo de elevación de
60°. Determine la distancia en k.m a la cima de la
montaña, cuando se encuentra en el segundo ins-
tante.

A).2..
B)6 C)S.J3 D)lO E)6J3
.J3

32.Se tiene una malla de longitud L con la que se


desea cercar un terreno que tiene forma de un tra-
pecio circular. Calcule el área máxima del terreno
A) b(b+c)_r b(b-c) b(b- c) que se puede cercar con dicha malla.
B) ---+r C)---r
c c c 2 2
2
B) .L C)~ D)~ L2
c(b- c) c(c-b) E)-
E)---+r 2 4 8 16
D)-b--r
c
:.e.
~'k:
GomcZ
QJ'" 2006-11 MATEMÁTICA
33. Las rcctas I. . 3x+8y = 48; L2: ~ y =]8;
A'_ B) 1 +.f3 C) 2.J3 D)4
L3: 3x + y = 3 y el conjunto S (figura sombreado)
se muestran a continuación. Halle los pun-
tos (x ,y) E S que dan el valor máximo y mínimo 37. Calcule el valor de F, si
para a = 2x + 3y, cuando esta recta se traslada F = arc tan ~ + arc tan ~ + arc tant + are tan ~
paralelamente a si misma.
A) II B) 1I C) II D) ~ E) ~~
6 4 3 3 6

38.Halle la suma de las soluciones positivas me-


nores de 211: de la siguiente ecuación:

2taIl2x+secx+1=0

A) 11: C) 1t D) 11: E) 211:


4 2
x
39.En la siguiente figura:
A) e72 ; 37°) ; (O; 3) D) (3 2; 37°) ; (0,1) e
7

B)(32.30). (1.0) E) (274; 37°) ; (1 ;·0)


7 '7 ' ,

C) (32.30). (3·0)
7 '7 ' ,

34. En un círculo de radio r = 3 se ubica el radio


vector en la posición (x, y) en el instante t = °.
Después de cinco unidades de tiempo de giro cons- Se conoce a, b, LA Y LBI. Si (1 > c2, enton-
tante, el radio vector está en una posición tal que ces se cumple que:
los valores del seno y del coseno son de signos
opuestos e intercambiados con respecto a la posi- A) cl +c2 = 2acos(B1) D)C1-C2 =2acos(BI)
°
ción inicial. Si al inicio y > y x = l. ; el ángulo
B) cl -c2 = 2bcos(J\) E)C1 +(2 =2bcos(BI)
de la posición final es: 2
C) cl + c2 = 2acos(A)
B) 1I C) ~1I D) 411: E) 711:
3 6 3 6
40. Determine la ecuación de la circunferencia
35. Dadas las ecuaciones x2 + y2 = 1 en un nuevo sistema trasladado XY,
sen(x - 45°)sen(x + 45°) = P cuyo origen está en el punto (-1; - 1) .

cos (x - 60° )cos (x + 60°) = q


Calcule el valor de: p + q
A) (x'+ ~ t + (y'+ ~ t = 1

B) Cx'+ 1)2 + (y'+ 1)2 = 2


A) - 1
4
B) ° C) ( x'+ ~)
2
+ (y'- 2)2 = 1
36. La gráfica de J(x) = 2 senx + 2.J3cos x ,
está desplazada en el eje X, una magnitud de ~ D) (x'_1)2 + (y'_1)2 = 1

hacia la izquierda con respecto a la gráfica de


g(x) = Asenx. La amplitud de la gráfica dejes:
E) ( x'- ~ t + (y'+ 1)2 = 1
SOLUCIONARIO
1. Enla igualdad: Por condición del problema, las partes obtenidas
a2b(7) = asl(,,) ... O) son proporcionales a 1, 2, 3, ... , es decir:

Como los digítos de las centenas son iguales, nos


queda por analizar los dígitos de las decenas, en
los cuales se observa que 2 < 5 , por lo tanto se Por propiedad de proporciones, pues solo nos
puede deducir que para que exixta igualdad: intereza la mayor y menor porción:
n<7 ... (2)
Al _ A,,+l _ Al +A2 +A3 + ... +A" +A"+I
Analizando el segundo miembro de la igualdad, por T-n+l - 1+2+3+ ... +11+(1l+1)
propiedad:
n > 5 ... (3)
Al _ A,,+l _ 30
De (1):
T - 11+ 1 - (11+ 1)[ (Il + 1) + 1]
Luego de (2) y (3):
2
5 < n < 7 => n = 6 ... (4)
Al _A,,+l _ 60
... (2)
La igualdad O) quedará definida así: ~ - ~ - (1l+1)(n+2)
III~
a2b(7) = asl(6) ... (1-1)

Por propiedades en la igualdad 0-1) : Igualando I Y III de la expresión (2):


Primer miembro : a < 7 ; b < 7
Al _ 60 => A = 60
T-(11+1)(11+2) 1 (11+1)(1l+2)
Segundo miembro: a < 6
=>a<6; n<7 ... (5) Igualando II y III de la expresión (2):

Ahora la igualdad (1-1) lo expresamos en base 10: AIl+l _ 60 => A = ---2-º-


Tl+l-(Tl+l)(11+2) ll+l11+2
a + 72 + 2 x 7 + b = a x 62 + 5 x 6 + 1
Las inversas de Al y A2 :
49a + 14 + b = 36a + 3 1
13a + b = 17 I = (11+ 1)(n + 2) . 1 n+ 2
13(1)+(4)=17 Al 60 'A"+I ()()

=>a=1 ; b=4 La media artmétíca de las inversas de Al y A"+2 :

Calculando el valor de la expresión: (n+l)(11+2) 11+2


-'-----~_...!..+---
a+b+n=I+4+6=11 M= 60 60_
2
Clave: A

2. Si en la barra de madera se hacen "n"cortes


Clave: E
Al A2 A3
I I I I 3. El capital C se ha dividido en el' C2, el para
1 2 3 n-l n
imponerlas al a%, 2a% y (2a + 2)% respectivamen-
Dato: te; entonces:
Al + A2 + A3 + ... + A" + AIl+l = 30 cm ... (1)
... (1)
y-:,--:==~', ,,,,;.a!g,;g::.,.~]LI__ U_N_I _2_00_6_-_II_M_A_T_E_M_A_' T_I_C_A_ ••••
..=:=.:=...>.
Datos: Operando en (l1):
Capital a imponer: C a+c=k
el
Porcentajes de interes: a%; 2a%; (2a + 2)%
ck2 +c
De (1): -d- =k
Por condición del problema, los intereses que ge-
neran las tres partes del capital C son iguales, es ~f¿=~
decir: d k2 +1
c _ k
; (Propiedad)
d-c - (k2+1)-k

De (i):
.s ; k
39 k2-k+l
La ecuación de 1 y II: c= 39k
... (3)
k2 -k + 1
C1 (l~O)t = C2( 12 a
0 o)t Reemplazando (3) en (2): .
~ C1=2C2 ... (2)
39k ( 1 )- k
La ecuación de II y IIl: k2-k+l eI-b -

~ d- b = 39
C2( l~O)t = C3( ~O+O
2
)t k2 - k + 1
13( 3)
... (4)
... (3) (k + 1)2 + k
Reemplazando (2) Y (3) en (1):
Como d Yb son números naturales, entonces" d- b "
(2C2) + C2 + -'L1C2 = C .es un número entero.
a+
Por lo tanto se tiene 2 casos:
~C =~C
2 4a + 3 Caso 1: (k - 1)2 + k = 3 ~ k =2
Clave: O
¡No se cumple porque k E N \ {1;2} !
4. Sean a, b, C y d números naturales.
Caso 2:
Dadas las siguientes proporciones:
i} Q.=a+c=12.=k kEN/{1,2} ¡Si cumple.x E N \ {1;2} !
....b ----..-...
d ....C
1 11 111 Reemplazando K=4 en (4):

ii)d-c=39
d-b= 13(3) =3
En (i), multiplicamos las expresiones (I) y (111), (4-1)2+4

Q..12. = k2 Clave: 8
b c
5. Inicialmente se tenía N cocos que recogieron
~ a = ck2 ... (1)
para repartirse en partes iguales entre los 5 ami-
En (i): operamos (l) y (Il). gos.
(a+c)-a =k El primero reparte los N cocos en S partes iguales
el-b de 111 unidades cada una, sobrandole r que se lo
c -k ... (2) dio a un mono; luego éste se lleva ma de las par-
~ d-b- tes, entonces:
N = Sil, + 1 =} N + 4 = S( 111+ 1) ... (1) Pero como n es Ull número natural, entonces éste
puede asumir los valores:
El segundo reparte los 411j cocos que sobraron tam-
bién en 5 partes iguales de 112 unidades, sobran- 11= 18; 19; 20; ... ; 29
dole 1que también se 10 dio a un mono; luego éste La suma de todos estos números:
se lleva una de las partes, entonces: ¿n = 18 + 19 + .. + 29 = 282
411, = 5112+ 1 =} 4h + 1) = 5(112+ 1) ... (2) Clave: C

El tercero procede en similar forma, entonces:


7. Sean aa, be y (b+1)(e-2) los tres números
4112 = 5113 + 1 =} 4(112 + 1) = 5(113 + 1) ... (3) primos.
El quinto también en forina similar: Por teoría se sabe que el único número primo COIll-
puesto por dos dígitos iguales es el 11, entonces:
4113=5114 +1 =} 4(113+1) =5(114 +1) ... (4)
El cuarto similarmente: aa = 11

4114 = 5115+1 =} 4(114+1) ~ 5(115+1) ... (5) De los números primos be y (b+1)(c-2), por
propiedad deben ser menor que la base 10:
Al día siguiente, después que recoge su parte el
quinto amigo, los cocos que quedan se reparten 1~b~9 /\ 1~b+1~9
entre los 5 , sobrandoles 1, entonces: 0~b~8
411s = 5116+ 1 =} 4(l1S + 1) = 5(116+ 1) ... (6) ... (1)

Con el fin de resumir las expresiones multiplica-


mos miembro a miembro cada una de las 6 igual- 0~e~9 /\ 0~e-2~9
dades que se han formado, así: 3~e~11

45(1l + 1)(111 + 1)(112+ 1)... (115 + 1)= 5" (n¡ +1)(112+ 1) ..(n6 + 1) ... (2)

Por condición del problema se sabe que el primer


=} 4S(N+4 )=56(116+1) número primo aa = 11 divide en forma exacta él
la suma de los otros dos números primos, es decir:
=} N=S6( 116~1 )-4 ... n
4" . bC+~c=2)
Para que el valor de N sea mínimo, en esta última 11= 11
expresión debe cumplirse que: _ e 1 O b + e ) + 1 Oe b + 1) + e e - 2)
- 11
N6 + 1 = 1
45 _2(10b+e+4)
- 11
Luego en la expresión(*) se tiene:
Para que esta última expresión sea .m número na-
N = 56(1) - 4 = 15621 tural debe cumplirse que" 1 Ob + e o,' 4 " sea multi-
Clave: E plo de 11:

6. Sea 11un número natural, cuya raíz cuadrada es 10b+e+4=11


. ., 1 3
4, 8 ; pero con una aproxirnacion menor ( e "5 ' es (11b-b)+c +4 = 1'1
decir:
c-b+4=11
• 48-1 <
'5
¡¡; < 48+1
'5 De (1) y (2): (9)-(2)+4=Ú
4,2 < ¡¡; < 5,4
17,64 < 11< 29,16 =}c=9 y b=2
:&o:
~I~ UN12006-11 MATEMÁTICA
GomeZ

Luego los tres números naturales son: Considerando los valores de a en (4) Yconfrontan-
dolo con aae de la expresión 11:
aa=l1; be=29; (b+1)(e-2)=37
aae = 330; 660; 990 ;
Los residuos al dividir el segundo y tercer número
íCumple que es 66 !
entre el primero:
Considerando los valores de a en (4) Y confrontan-
37~ doio con aa-é de la expresión JJ:
4 3
aae = 225; 555; 885 ;
¡No cumple que sea 66 !
El cálculo de:
Entonces asumimos que los valores que puede to-
r) - r2 = 7 - 4 = 3 mar a es la de la expresión (3), y "e" asume un
Clave: B único valor:
e = O ... (6)
o
8. Del enunciado se tiene el máximo común divi- Analizando la expresión: (a - 1 )( a - l)b = 15
sor de: o
Si: a=3 ~ 22b=lS ... (7)
M.CD( aae ; (a -1)( a -l)b ) = 15 ... (l) a=9 ~ 88b=lS ... (8)
De (6) Y (7) se concluye que:
M.CD(aae ; da(a -1») = 66 ... (I!)
b=S ... (9)
Por propiedad de numeración:
Expresión(I1) :
1~a~9 1\ 1~a-1~9
2 ~ a ~ 10 Como da( a - 1) = 66 , entonces:

~ 2~a~9 ... (1)


da(a-1)=21\ da(a-1)=3 /\ da(a-1)=Ú
'---v---'
También se tiene: ii
... (2)

... (3) Considerando (3) donde a = 3; 6; 9, e (i) de esta


última expresión:
Expresión (1):
a-1=2;S;8 ; CSnoespar)
Como el máximo común divisor de estos números
es 15, entonces estos son múltiplos de 15, luego: ~ a = 3; 9

aae = 15 Analizando (ii), donde ahora a = 3; 9


o o
da(a-1)=3.~ cl+2a-'¡=3
~ aae = 3 1\ aae = 5

102 (a) + lOa + e = 3 1\ (e = O v e = 5 ) Si a =3


o
a+a+e=3 1\ (e=Ove=S) ~ d+2(3)-1=3

2a + e = 3 1\ (e = O v e = 5) d+S=3

Considerando (1): <1+3+2=3


<1+2=3
Si: e=O ~ 2a=3 ~ a=3;6;9 ... (4)
=!> d=1;4;7 ... (lO)
Si: e=S~2a+S=3~a=2;S;8 ... (5)
o

Si: a = 9 ::) d + 2( 9) - 1 = 3 Por el trabajo de 2.5 meses recibe:

d+17=3 2,sT·
o 6 . 10+L ... (3)
d+1s+2=3
o Como el trabajo realizado y lo que cobra el trabaja-
d+2=3 dor son cantidades directamente proporcionales.
entonces podemos plantear:
::) d = 1; 4; 7 ... (11)
De (1) y (2):
Luego. considerando (10) y (11) se deduce que los
T _ 400 + L + R
valores que puede asumir des: d = 1; 4; 7.
lT -
50+ L + R
Analizando (uí): da( a -1) = Ú 3
3( 5 O + L + R) = 2( 40 O + L + R)
Si: d=l 1\ a=3 ::) 132=11 (cumple) ::) L=6s0-R ... (4)
a=9::) 198=1'1 (cumple) De (1) y (3):

Si: d = 4 1\ a = 3 ::) 432 * 11


" J-.- = 400 + L + R
2...T 10+L
"
a = 9 ::) 498 * 11 12
12(10+L)=s(400+L+R)
Si d = 7 1\ a = 3 ::) 732 * 11 7L-sR=1880
u

a=9 ::) 798*11 De (4): 7(650 - R) - SR = 1880

Luego se deduce que: ::)R=222.s0 (soles)


d=l ; a={3; 9} ... (12) Clave: E
Resumiendo de (6); (9) y (12) se cumc1uye final-
10. Analizando las proposiciones:
mente que:
I.SiA={0}, entoncesAcP(A);P(A) po-
a = {3; 9}; b = 5; e = O; d = 1 tencia de A. Verdadero (V)
La suma de estos valores: Verificación:
Suma 1: a + b + e + d = 3 + 5 + O + 1 = 9 Si: A = {0} y peA) = {0; {O}}
Suma 2: a + b + c + d = 9 + 5 + 0+ 1 = 15 ::) A c peA)
La suma de todos los posibles valores de a +b +c+d:
11. A t, B E P( A v B) . Verdadero (V)
S = 9+ 15 = 24 Verificación:
Clave: B
Por teoría: A t, B = (A - B) v (B - A) Y
9. Para realizar el trabajo completo T de 6 meses (A v B) c P( A v J3)

~aJ~
el.trabajador recibe:
S/ .400 + lavadora (L) + refrigeradora (R)
es decir:

(* )T: 400 + L+R ... (1) At,B:~~

AUBID
Pero por el trabajo de 4 meses recibe:
S/.SO+ lavadora + refrigeradora
es decir:
(t)T: sO+·L+R ... (2)
'A:.
4/Jill\i
GomeZ
UN12006-11 MATEMÁTICA

De las griÍfiGls se observa que: 1


y2 - X = O
At.Bc(AuB)
1 1
III. Si A \ B = 0 entonces A = B . Falso (F) y2 _ 32 = O

Verificación: 1 1
y2 = 32
Lo resolveremos con un contra ejemplo.
=> Y = 3 i es un número racional!
Conclusión: sólo lo que afirma Katy es correcto.
Clave: B

12_ Dada la función:

Del gráfico: J(x)=x+ x~k ; 'if(x;tk)


-A\B=A-B=0
Observe que x ;t k para que. la fracción x ~.k 110
- A ;t B
se vuelva indeterminada, pero k si puede asumir
Clave: B
cualquier valor de los números reales, es decir:
11 _Con respecto a un numero irracional. k E (-00 ;00) ... (1)
Liz: Si le suma otro irracional el resultado sigue
siendo un número irracional. Incorrecta La inversa de la fracción de la función J(x):

Verificación:
J(x)=x+ x~k
Se tiene el número irracional J3 y luego se le
suma el número irracional -J3 . Veamos: y=x+_1_
x-k
J3 + ( -J3) =O ; iEs un número racional! yk _k2 + 1
=>x= y-k
Katy: Si a, b, C, d son números racionales tal que: k(y -k)+ 1
x=--'=---;...-
Q + bx = e + dx => a = c y b = el . Correcta y-k
Verificación: x=k+_l-
y-k
Si a + bx = C + dx
, 1
=> ~ = (d -b)x => J(x)=k+ x-k
numero
racional Como la función inversa Jt~)tiene la misma ex-
presión que la función J(x), esta t imbién tendra
Si d;t b => (d - b)x es un número irracional.
la misma gráfica, y los valores que puede asumir k
Pero para que cumpla la igualdad de que ambos también serán los mismos.
miembros sean racionales: kE(-oo;oo)
d-b=O => d=b Clave: E
=>a-c=O => a=c 13. Definimos:
1
Laura: La solución de y" - X = O es irracional, z = x + iy ... (*)
donde n E N. Incorrecta
Verificación:
1
De la expresión: {z/I ~~ i1> 3}
Supongamos el número irracional x = 32 , donde
11 = 2, luego en la expresión: => 1z+l
z - 11> 3 ~ Iz -11
Iz+ll
>3
Resolviendo la in ecuación:
Iz-11 > 31z+ 11
=>
I(z + iy) -11 > 31(z + iy) + 11 a) Si: ~ -~>O
2 -
De (*): . J(x-1)+iyl>31(x+1)+iyl
=>~~~
=> J(X-1)2+y2 >3J(X+1)2+y2
x + 1>.2.
J(x2 -2x+1)+ y2 > 3J(x2 +2x+1)+ y2 -4
x>~
x2 _ 2x+ 1 +)'2> 9(x2 + 2x+ 1+ y2) - 4

8x2 +20x+8y2 +8 < O =>5¡ :xE[5/4;+00)

x2 + ~X + y2 + 1 < O Operando en (3):


l<~_~c;;
(x2 +~x+
2
25)_ 25 + y2 +1 < O
16 16 2 2 2

%<JX+1<~
(x2+*t +y2 «~t
2<~<4
Graficamente se tiene un círculo abierto de radio
4<x+1<16
~ y centro ubicado en (-*; O) . Veamos: 3 < x < 15
,"---, y => 52:XE (3;15)
I
/ r=3/4 " \
=> C.51 = 51 il 52
'. (-5/4; O) " O x
,
-,
I
= (5/4; +00) il (3; 15)
... _-_ •.•.
",/

Clave: A =(3;15) ... (4)

14. Dada la inecuación: b) Si: ~ -~<O


2
lag 6( x + 3 - 3~) <1 ... e)
=> 0::;Jx+1<~
Restricciones: 2
• x+1~O => x~-l ... (1) 0::;x+1c2
4
• x+3-3~ >O ... (2) -1< X<~
- 4
De la inecuación ("):
=> 53: XE [-1; 5/4)
log6(X + 3 - 3~) < 1 = log66
Operando en (3):
=> x + 3 - 3J x + 1 < 6
_l>~_~>_~
De(2): 0<x+3-3Jx+1 <6 222
O< x + 1- 3J x + 1 + 2 < 6 1>~>-1
1>~~0>-1
0«JX+1)2 -2(~)JX+1+%-i<6
l>x+l~O
0«JX+1-~t -i<6 O> x ~-1

=> 54: XE [-1; O)


1< (JX+1_~)2 < 25
424
C.52 = 53 il 54
1<IJx+1-~I<~
2 2 I 2 ... (13) =[-1; O) ... (5)
~
Qjl~
G~i'11eZ
I UN12006-11 MATEMÁTICA r": :w._ " '~-::i~

Finalmente de (4) Y (5): Para que x e yE Z pertescan a las tres superficies


C.S ~ C.S] U C.S2 o regiones S1' S2 y S3 ,estos deben estar ubicados
dentro del triángulo ABC, es decir:
=[-1; O)u(3;15)
={x/-1::>x<O}u{x/3<x<15} 11<x<4 => x=3
5
Clave: E
3<y<51 => y=4
11
15. Para hallar el valor de:
... Luego calculamos la expresión C'):
E = 4x + 3y (")

Donde x e y tienen valores enteros que pertenecen E=4(3)+3(4)=24


a las tres regiones, definidas por: Clave: B
s, ={(x; Y)E 1R2/5x-3y> 2}
16. Del enunciado se tiene:
S2 = {( x; y) E IR 2 /2x + y < 11}
S3 = { (x; y) E IR 2/ Y > 3 } 2 7 -1]
Las rectas límites de las regiones S1' S2 y S3 son
I)Q=
[ 1 1
-1 4 -4
1

respectivamente:
L1 : 5x - 3y = 2 => Y = ~x -1 11) P = Q101

L2 : 2x
L3 : y =3
+y = 11 => Y = - 2x + 11
III) Q = [ ~: ] = f:]; " ·
• LI !l L3 : 5x - 3(3) = 2 => x = ~1 ; Y = 3
IV) P ü = ati . ti vector y ex número.
• L2!l L3 : 2x +3 = 11=> x = 4 ; Y = 3 Definamos:
5 2 35
• L1 !l L2 ="3 -"3 x = -2x + 11 => x = TI

=> y=_2(35)+11
11
=> y=51
11 xf:] y o=m
=> A = (1.1 . 3) . B = (35 .~) . C = (4 . 3) Operando:
3" 11'11' ,
Graficando: 2 7 -1][ 8~
, x
,1
11+,
, 1
1

'L
1
Qx =
[ 1
-1
1
4 -4
1 .-3
-5 J
1

, 1
, 1
,
, 1
1 16 -21 +5]
,
, 1
1
= 8 - 3 -5
,
, 1
I
[
,
, 1
I -8 -12 +20
\8/
'~
I ,
1 ,

=m
I ,

\C A/
3+-----~-----~---------- I ,
L3
I ,

I
I
I
I ,
,
,
,
=0
01 / "
211 , x
=> Qx = 5 ... (1)
-~i \.L2
I
~~~!:~~I __U_N_I_2_00_6_-_II_M_A_T_E_M_A_'T_I_C_A __

Entonces: 11. El punto óptimo se encuentra en la región ad-


misible. Correcta
De (1): Q2x = Q(QX) = O El punto óptimo es uno de los puntos de la re-
Q3X = Q2(QX) =O gión admisible.
111. La región admisible contiene puntos, los cua-
les tienen alguna de sus coordenadas valor
Q101X = Q100(QX) = O negativo. Incorrecta .
De(II): Px=O ... (2) Las coordenadas de todos los puntos de la re-
gión admisible siempre tienen valor positivo. (ver
Luego, para hallar el vector Ü y el número x, en la gráfico de 1).
expresión (IV):
Clave: C
Pü= ax
18. En la sucesión:
Hacemos: ü= kx k

~ P(kX)=aü
Sk = I(-k-
11=1 2 +
1
-)
11
; k ~ 1

k(PX)=aü Los límites de la sumatoria es de 1 hasta 2k ,en-


De(2): 0= aü tonces:
1 s 11 s 2k
~ a=O
1 + 2k ~ 2k + 11 ~ 2k + 2k
Para que tenga solución:
2k < 1+ 2k s 2k + II ~ 2k+1

~ 2k < 2k + 11~ 2k+l


Tomando las inversas:

Clave: E

17. Analizando las proposiciones con respecto a Luego aplicando a sucesiones:


la programación lineal:
I. Las restricciones de desigualdad son políno- 2
k

¿ ( k+J
11=1 2
1) ~ 2
k

¿ ( ir:':
11=1 2 + 11
1 ) < ¿-;:
2( 1
11=1
k

2
) ...(1)
mios de primer y segundo grado. Incorrecta ~ ~
1 11
Las restricciones de desigualdad siempre son de
primer grado (lineal). Operando (1):
k 2k
2 (
1~1
1 )
2k+1 =
( 1
2k+1 }~1
i ( 1 }k 1 ... (2)
= 2k+l 2 ="2

Operando (11):
k k
2¿ ( 1) =,,-¿=-¡;-(2
-1: 1 2 1 k ),,] ... (3)
11=1 2 2 11=1 2
Reemplazando (2) y (3) en (1):
k
1<¿ 2 ( _1_ ) <1
Xl 2 - 11=1 2k + 11

Las rectas representan a ecuaciones de primer 1~Sk<1 ; \fk;:l


grado. Clave:D
, 1L.__ U_N_1 2_0_0_6_-1_1 _M_A_T_E_M_Á_T_IC_A __

19_ Una ecuación cuadrática tiene la r na: 20_ Dado los números complejos:
ax- + bx + c = ° z1 =-2
Luego, si definimos que xl y x2 son los das raices z2 = -2i
de esta ecuación y, a = 1 ; por teoría se sabe que:
z3 = 3J2( COS* + i: SCll*)
X + X = -Q = _JL = -b .,. (1)
l 2 a (1) El argumento de un número complejo que equidista
de estos tres números complejos, es el que se en-
x¡x2=-,,-=_c_=c (2)
a (1)
oO.
cuentra ubicado en el circuncentro (H) del trián-
gulo que formen estos números al representarlos
Por otro lado, del enunciado del problema se tiene: en un sistema de cordenadas cartesianas.
xl = 6. +4 ; x2 = 6. - 2 ... (3) Operando con estos números:
Donde ó es el discriminante de la ecuación que se
define como:
Z
3
= 3J2( J22 +i J22 )
... (4) =3(1+i)
Sumando y multiplicando las raices de las expre- =3+3i oO. (1)
siones de (3):
Además:
Xl + x2 = 26. + 2 (S)
zl=-2+0i
oO'

... (2)
x¡x2 = 6.2 + 26. - 8 ... (6)
z2 = 0-2 i ... (3)
Igualando las expresiones de (1) y (S): Graficando estos números en el plano cartesiano:
-b = 2ó + 2 ~ b = - 26. - 2 ... (7) 1m
Igualando las expresiones de (2) y (6): 3 _ z3 (3 ; 3)
c=ó2+2ó-8 ... (8)
Ahora reemplazamos (7) y (8) en (*):

(1 )x2 + (-2ó - 2)x + (6.2 + U - 8) = O


(-2; O)z¡
Aplicando la fórmula (4) en esta última ecuación ~~----~~~~----~--~
de segundo grado: 3 IRe

6. = (-26. - 2)2 - 4( 1)( ó 2 + 26. - 8)


~ 6. = 36 Z2(0; -2)

Reemplazando 6. = 36 en (6):
Por las coordenadas se deduce que el triángulo
x¡x2 = (36)2 + 2(36) - 8 = 1360 zl,z2,z3 es isósceles, de lo cual se deduce que la
La suma de los dígitos del producto de estas raices: altura y la mediana del lado z¡ z2 coinciden; en-
tonces P es punto medio ue z']z-Z:
1+3+6+0=10
Clave: A
P _((-2)+0.0+(-2))_(
- ---2--' ---2-- .
- -1,-1 )

El argumento de PZ3 :

3 - (-1)
talle = --- =1
3-(-1)
111. Si al borde de un círculo se le quita un sólo
=> arg(Pz3) = 45° ... (4) punto, el conjunto resultante ya no es convexo.
El argumento del circuncentro CH): • Incorrecta
Si en el bode de un círculo se le quita un sólo
HE PZ3 punto, el conjunto sigue siendo convexo. Todos
los puntos del conjunto siguen siendo continuos,
=> arg( H) = arg( PZ3 )

!t
De (4):
4
Clave: C

21. En las proposiciones:


1. El conjunto convexo más pequeño que contie- Clave: E
ne a tres puntos no colineales del plano es la
región triangular cuyos vértices son dichos 22. Del enunciado, graficamos frontalmente:
puntos. Correcta
Eje inclinado
Verificación:
Tres puntos no colineales de un plano pueden
estar ubicados en cualquier conjunto convexo, ya
sea un círculo, cuadrado o región polígonal cual-
quiera, pero si queremos que estos puntos esten
,,
/

ubicados en una área mínima, entonces esta debe


,,
ser una área triangular donde los tres puntos esten /

,,
/

ubicados precisamente en los vértices. ,


,,,
,

o
Conjunto convexo
que contiene
los tres puntos
Conjunto convexo,
el más pequeño, que
contiene los tres puntos
El plano es paralelo al círculo eje radio r del
cilindro
Datos:
Distancia entre los centros de las bases: O] 02 = 16 111

Radio del círculo perpendicular al eje: r = 2 111


11. El conjunto S = {x E lR/1 x] > 1} es convexo.
Incorrecta El volumen de un tronco de cilindro oblícuo se cal-
Verificación: cula multiplicando el área del círculo perpendicu-
lar a la generatriz por la longitud del eje de éste.
Si resolvemos la inecuación:

Ixl >1
=>x<-l v x>l '" !t(2 m)2(16 111)

Graficamente = 64rr 111


3

~------o-----o------~
-1 1 Clave: D
El conjunto S es descontinuo. por tanto no es
convexo.
23. Del enunciado: D El tetraedro regular interno tendrá a/3 de lado,
luego su área es:

A =4(
2 1(~t)=1 a2
... (3)

Luego de (2) y (3):

B Clave: C

24. Del enunciado:


E~
\'
__ ~ __ ~.- __ ~ -= F

Por propiedad:

... (1)

El área de una cara ADC del tetraedro regular ABCD

SADC
_ ACxHI5
- 2 C
El triángulo EBC es congruente al triángulo ABF
De O): _a(1a ) por tener la misma medida de uno de sus ángulos y
la misma medida de su lado adyacente. Luego po-
- 2
demos completar los ángulos así:
J3a2 E a B
--4-
F
Entonces el área de todo el tetraedro: 60
0
e
Al = 4 ~a2 = J3a2 ... (2)

En el triángulo isóseles HDT:


D

C
En el triángulo ABF:
a + e + 60° + P = 1800
a+e+p = 1200 ... (,,)

En el cuadrilátero EBFP:
<-------",--JT
a + 60 + e + 60 + p + x = 360
0 0 0

(a + e + p) + x = 240 0

PD
De (*): 120 + x = 240
0 0

HD 0
=>x=120
~HD - a
_3__ => x = 21t
De(1): => PR =:3 3
HD
Clave: B
~~::::::::i:$%:··:*¡:'::~
~
U

'z~:;~t:'::::A:~:.~:,:::::::;,gi~~
__N_12_0_0_6_-I_I_M_A_T_E_M_A_-T_'_c_A
Coiñez
25. Sea el polígono de n lados En la figura del enunciado
1
Datos:

AB = 10 m)
BC = 12 m ... (1)

AC = 11 m

Del enunciado se tiene: A~~~~--=-~1~2~--a----~C


i1 + i2 + i3 + i4 + is = 7600 .•• (1)
En el lado AC:
Por teoría se sabe que:
AC=(10-a)+(12-a)
e+i=180° =:> e=1800-i
De (1): 11 = 22 - 2a
La suma de los 5 ángulos externos:
s =:> a = 1 1 ; (m) ... (2)
Le¡ = (180° -i1)+ (180°- i2)+ ... + (1800-is) 2
¡~l
Cálculo del perímetro del triángulo EBF
= 9000-(i1 + i2 +i3 + i4 +is)
PEBF = EB + BF + EF
De(l): =900°-760°
Del gráfico: = (a - x) + (a - y) + (x + y)
=140° ... (2)
= 2a
Por teoria se sabe que la suma de todos los ángulos 1)
De (2): = 2(12
externos de un polígono de cualquier número de
lados, suman 360°, entonces: = 11 (m)
n
Le¡ = 360° Clave: O
¡~1
s 11

Le¡ + Le¡ = 360° 27. En las proposiciones:


¡~1 ¡~6
I. Dos triángulos rectángulos con la misma
11

De (2): 140° + Le¡ = 360° hipotenusa son congruentes. Falso (F)


¡~6
Dos triángulos rectángulos conqruentes deben
11
tener por lo menos dos elementos congruentes
=:> ¿ e¡ = 220° respectivamente, del cual, uno de ellos debe ser
¡~6
uno de los lados.

t~
Clave: O

26. Propiedad, los puntos de tangencia de una B~


circunferencia a los lados de un ángulo, generan
dos segmentos de igual medida. A e P R
Triángulo ABC no es congruente con el trián-
gulo PQR.
11. Dos triángulos rectángulos isóseles con un ca-
teto común son congruentes. Verdadero (V)
~_U __N_12_0_0_6_-I_1_M_A_T_E_M_Á_T_IC_A__ ~I~':MU~~~~~~"~;'~~~'~
Los dos triángulos rectángulos tienen por lo 29. En el gráfico trazamos el triángulo rectángulo
menos dos elementos iguales respectivamente, AEB ,por ser una sernícircunferencía.
ángulos de 45° y catetos,
E

D. ABe es congruente a D.eBD


A~~~----~-~--r----r-~B
111. Dos triángulos rectángulos con un ángulo agu-
do de igual medida son congruentes, Falso (F) Del gráfico:
Es el mismo caso de la proposición 1, los trián- El radio de la semicircunferencia mayor es R + r ,
gulos tienen un solo elemento respectivamente
Por relaciones métricas se tiene:
congruente, que es el ángulo agudo,
2
2R=ll. => Rr=(h)
/¡ 2r 2
El área de la región sombreada:

S=S AB- S sit: S HB

D. AB e no es congruente a D. PQR = !!(R + r)2 _ !!R2 _ !!r2


2 2 2
Clave:B
= !!(R2 + 2Rr + (2) _ !!R2 _ !!r2
2 2 2
28. En el gráfico: = rrRr
De(*):

Clave:.D
30. L1 Y L2 son las dos rectas qUE:se cruzan per-
pendicularmente, y AB el segmento que los une per-
a pendicularmente a ambas,

i-------b ----(
Por relaciones métricas se tiene:

E f =>
= y = c; '" (*)

Del gráfico:
PB = e - (x + y)

De (*): r = c - ( x + e; )
Dato:
c2
=>x=c-r-7J '" O)
c( b - e) Para visualizar mejor construiremos un plano que
=r+:': pase por los puntosABD; esto es, trazando una rec-
Clave: O ta L2 paralela a L2 y que pase PO! el punto A,
~:'::=~~~':z~1 UN12006-11 MATEMÁTICA

Por propiedad triángulo ADB es isósceles por tener


dos ángulos iguales, entonces:
Observe que: BC .1 BD Y AD.l AC x=d
De (*): = 10 k.m
La recta L] es perpendicular al plano formado por
las rectas paralelas L2 y L '2 ,BD YAD estan conte- Clave: O
nidos en este plano.
En el triángulo rectángulo BDC: 32. El terreno de forma de trapecio circular, el
cual se debe cercar con una malla de longitud "L".
BD2 + BC2 = a2
En el triángulo rectángulo ADC:
... (2)
'D d
.
AC2 + AD2 = a2 ... (3)

Sumando miembro a miembro (2) y (3):

BD2 + BC2 + AC2 + AD2 = 2a2


De (1): 32 = 2a2
El perímetro del terreno a sercar can la malla:
2d+a+b=L => a+b=L-2d
Clave: E El área del terreno:

5 = (a; b)d
31. Del enunciado:
D
De (1): = (L -/d)d

=> 2d2 - Ld + 25 = O

2
d=L±JL -165
4
Como d es una longitud de terreno, es decir deber
una cantidad positiva, entonces la cantidad
t=lOmin=lh subradical se debe asumir que sea positiva,así:
6
Cuando el vehículo ha recorrido de A hasta B: 2 L2
L - 165 ~ O=>5 ~ 16
d=vt=(60k.mlh)(t;h)=lOk.m ... (*) El máximo valor que puede tener el. terreno:
L2
Los triángulos ADC y BDC son triángulos notables, 5 = 16
tal como se puede observar en el siguiente gráfico: Clave: E
A
33. En la gráfico se muestran las rec:
UNI2006-11

lefinidas
MATEMÁT-ICA

34. Del enunciado:


y
por:
L} : 3x + 8y = 48 L2 : 3x + y = 18
L3 : 3x + y = 3

En el triángulo BNO:

sene = BN
x BO
_ 3/2
-3
La recta L definida por 2x + 3y = a, se conoce su = 1 ~ e = 30° =.rr
2 6
inclinación, pero no su ubicación porque la varia-
ble "a" no esta definida; al darle un valor a ésta Luego en ángulo en la posición final
recién quedará ubicada la recta. Observe que la
recta L podra desplazarse hacia arriba o hacia aba-
jo, y podrá b.: 'Ter el área S.
Cuando la recta L se traslade paralelamente a sí
misma, sobre el área S, asumira un valor rninimo
para 2x + 3y = a cuando ésta esté sobre el punto
A; y asumirá un valor máximo cuando se encuen-
tre sobre el punto B.
Clave: E
Cálculo del punto "A": L3 n X
3x + (O) =3 ~ x =1 35. Dadas las ecuaciones:
~A=(l;O) sen(x-45°)sen(x+45°)=p (1)

Cálculo del punto "B": L} n L2 eos(x - 600)cos(x + 60°) = q (11)

Por identidades:
L}: 48-8y=18-y ~ y=30
7 sen (x - 45°) = sen x eos45° - eos x sen45°
L2:3x+(3 0)=18 ~ x=3 2 =fisr.nx-flcosx ... (1)
7 7
2 :¿
~ B = (3 2; 37°)
7 sen(x+ 45°) = senx eos45°+ cosx sen45°
Los puntos que dan el mínimo y máximo valor para = fi senx-i- fieosx ... (2)
2x + y = a , son: 2 2

(1 ; O) y (3 2; 37°)
cos(x -60°) = eosxcos600+senx sen60°
7
= leos x+ .J3 senx ... (3)
Clave: 8 2 2
eos(x+600) = eosxeos60° -senx sen60° Gráficando:

= 1eosx- .J3 senx ... (4)


2 2
Reemplazando O) y (2) en (1):

x
fisenx- ficosxlfisenx+ ficosx)= p
( 222 2
lsen2 x-1eos2 x =p ... (5)
2 2
Reemplazando (3) y (4) en (II):

(~cosx+ fjsenx I~COSX- fjsenx )=q


x
1cos2 x-1sen2 x = q ... (6)
4 4
Sumando (5) y (6):
De los gráficos:

(~sen2 x- ~eos2 x) + (i-cos2 X - %sen2 x) = p + q J(x)=g(x+~)

-lsen2 x-1cos2 x =P +q De (1) y (2): 4sen( x + ~) = Asen( x + ~)


4 4
=> A = 4
.-i-( sen2 x+ cos2 x) = p +q
Clave: O
_1(1) = p + q
4
37. Por teoría, si xy < 1 se cumple la propiedad:
::} P + q =_1
4
are tanx + are tany = are tan(ox + y )
Clave: A l-xy
Para reducir la expresión del enunciado aplicare-
36. Del enunciado se tiene las funciones: mos esta propiedad:

J(x) = 2senx+ 2.J3cosx F = are tan~ + aretan~ + are tant + are tan~
g(x)=Asenx ... O)
= (are tan~+ are tant)+ (aretan~+are tan~)
Reduciendo J(x):

f(X)=4(~senx+ 1cosx)
"= are tan[ ~;
l--x-
t ]+
1 are tan[ i~
~1
l--x-
1
3 7 5 8

= 4( eos~senx + sen~eosx) = are tan ~ + are tan ~

= 4sen( x + ~) ... (2) = are tan[ ~ ~ ~1


l-ix 3)
I
Por condición del problema, J(x) esta desplazada
= areta~O)
~ respecto g(x).
1t
4
Clave: A
____ U_N_12_0_0_6_-I_I_M_A_T_E_M_A_·_T_IC_A __ ~I~~:::,:,1:!=:f:.m::::::::~~
38. Por teoría: Además del gráfico:
Identidad trigonornétrica: e] = AD + DB
tan2 = see2-1 ... (1) = beosA + aeosB] ... (2)
Propiedad:
e2 = DA'- DB
-00 < s ecü s -1 v 1::; see8 < 00 ••• (2)
= beosA'- aeosB]
Del enunciado del problema:
De (1): = beosA - aeosB] ... (3)
2tan2x + seex + 1 = O
Sumando e¡ y e2, de (2) y (3):
De (1): 2 ( see x2 - 1) + see x + 1 = O
e] +c2 ==(beosA+aeosB¡)+(bcosA-acosB¡)
2see2 x + see x - 1 = O
== 2bcosA ... (3)
(2seex -1)(seex + 1) = O
Restando e] - e2, de (2) y (3):
=> seex = 1. /\ seex =-1
2 e] -e2 == (bcosA+aeosB¡)-(bcosA-acosB1)
== 2acosB1 ... (4)
Por la propiedad (2), see x = ~ no existe, luego:
En las alternativas, solo existe respuesta para (4).
seex=-l => x=(2k-1)7t; kEN Clave: D
Por condición del problema, solo debemos consi-
derar valores menores que 21t, es decir k = 1 40. La ecuación de la circunferencia x2 + y2 == 1
en un sistema de origen (0;0).
=> x = [2(1) -1]1t = 1t
y
La única solución es : x = 7t

Clave: D
39. En la figura:
C -1 x

-1

Luego si este sistema XY, incluyendo la circunfe-


rencia, se traslada a un nuevo sistema X'Y', cuyo
origen está en el punto (-1;-1):
Y' YA,
A~--------~~~--------~A' ,,
,,
L-el e2-1 ,

---
, r
I
I
Datos: a ; b ; el ; e2 ; LA ; LB
x
el > e2

Del gráfico se observa que en el triángulo ACA', (0;0) X'


entonces:
La ecuación de la circunferencia en ('1sistema X' Y' :
AC =CA'
(x'_1)2 + (y'_1)2 == 1
=> mLA=mLA' ... (1)
Clave: O
1 • Obtenga la suma de los n primeros números natu- La respuesta aproximada es
rales que tengan todas sus cifras iguales a 7, más la A) descuenta 3,2% B) incrementa 3,2%
suma de los n primeros números naturales que tengan
todas sus cifras iguales a 1. C) descuenta 6,4% D) incrementa 6,4%
E) incrementa 5,2%
A) %(10 +1 11
- 9n -10)
5. Un automóvil usa gasolina de b octanos en la
11 1 cuidad A y de e octanos en la ciudad B. Al llegar a la
B) 88 (10 + - 9n - 9)
1 ciudad B, tras un largo viaje desde la ciudad A, el
conductor paró en un grifo para llenar su tanque de
11 1
C) 88 (10 + -1011 - 9) combustible, encontrando gasolina de a y d octa-
1
nos (a < b < c < d) . Entonces el número de galones
11 1
D) 88 (10 + - 9n-10) de octanaje a y octanaje d, respectivamente, necesa-
1
rios para completar su tanque, sabiendo que la capa-
11 1 cidad de éste es G galones y aún le quedang galones
E) 88 (10 + -9n)
1 en el tanque (g < G) ,es:

2. Para las fiestas de aniversario de un pueblo, la A) (d+c)G-(d+bk (c+a)G-(b+a)g


Municipalidad promueve un juego entre los poblado- d-a ' d-a
res, el cual consiste en que los pobladores hagan lle-
B) (d-c)G-(d-bk (c-a)G+(b-a)g
gar sus pronósticos de las posiciones finales de un
campeonato en el que participan 5 equipos. Se otor- d-a ' d-a
gará premios a los pobladores que acierten con los C) (d+c)G-(d+bk (c-a)G-(b-a)g
equipos en al menos 2 de las 3 posiciones ganadoras. d-a ' d-a
Determine la probabilidad de ganar premio.
A) 0,02 B) 0,05 C) 0,10 D) (d-c)G-(d-b)g. (c+a)G-(b+a)g
d-a ' d-a
D) 0,11 E) 0,16
E)(d-c)G-(d-bk (c-a)G- (b-a)g
3. Halle el valor de a+b+c+d si al extraer la raíz d-a ' d-a
cuadrada de 14abcd64 se obtiene adcd
A) 17 B) 18 C)19 D)20 E) 21 6. De un conjunto de 10 números, se calcularon el
valor de la media y la mediana.
4. Una tienda vende un producto haciendo descuen- Luego de analizar las siguientes proposiciones, indi-
tos primero uno de 15% y luego otro de 15%. Una que la secuencia correcta.
segunda tienda, que tiene el mismo producto y al
a) Si hubo un error en el cálculo de la media y se
mismo precio de lista, realiza un descuento del 30%, obtubo Xc entonces, qué ocurrió 10 con el va-
é

écuánto de descuento (en %) o de incremento (en %)


10 _ 2 10_2
debe efectuar la segunda tienda para que en ambas lorde ¿(Xi-Xc) respecto de ¿(Xi-X)
tiendas el producto tenga el mismo precio final? ;;1 i;1
~~~~~::~~=~
__ U_N_I_2_00_7_-I ~l::,::::::::::::~~
__M_A_TE_M_A_'T_I_CA
__

b) Si el dato menor se disminuye, sl'de con la


media y la mediana? D) (-1;1)
Considere: D = disminuye, P = permanece constante
y A = aumenta.
B)[-l;-~)U(~;ll E) [-1; 1J
A)A;PA B)A; DP C) A: AP
D)D;DP E) D:AA
C)[-l;-~JU[ ~ ;1)
7. Si se cumple que O,ab + O,bci = 1,4 obtenga el
valor de a + b.
13. Determine la suma de la raíces de la ecuación:
A) 2 BY 5 C) 7 D) 9 E) 13
16(z2 - 2iz - 1)2 = z4

8. Dos pagarés por igual valor nominal que se vencen A) 3 -4i B) 2 + 4i C) 48i
dentro de 30 y 60 días, respectivamente, son descon- 15 5 15
tados comercialmente hoy al a% anual. Entonces el
D) -2+4i E) 64i
valor nominal de cada uno de ellos, si se recibe un
5 15
total de S nuevos soles, es:

800a S 800S 14. Diga cuál de las siguientes gráficas representa


A) 400a S )
800 -a B 400 -a C) 400 + a aproximadamente a las funciones J,g: IR -{1} ~ IR ,

D) 400S E) 400S definidas por J(x) = TX+1 Y g(x) = 21/(x-l)


800-a 800+a

9. Determine el valor de "n" sabiendo que el mínimo


común múltiplo de A = 180".27 Y B = 40".60 tie-
ne 5400 divisores.
A) 6 B) 7 C) 8 D)9 E) 10

10. ¿Cuántos divisores primos tiene 130130? x

A) 3 B) 4 C) 5 D) 6 E) 7

11. Sea

A) 3-54
p(x)=ax2+bx+c

de x * de modo que p(x *) = O

8
tal quep(l)
p(2) = 3 Y p(5) = 34. Determine un

D) J2l7
8
+3
=-2,
valor
,!l
--- 1 --~
I
I

2
g

3 x
,

B) -3+J2l7 E) J2l7 +../3


8 8

C) -3+ J17
8

12. Halle el conjunto solución de la siguiente des-


igualdad:
~==!=:=I
__ U_N_I_2_0_0_7_-I_M_A_T_E_M_Á_T_I_C_A
__ I=::=i:;:::~==;!::;g~!~::
15. En la figura adjunta se muestra las gráficas de 1. No existe región admisible
las funciones J y g definidas por: II. El óptimo se da en el punto (60,0)
J(x) = ax2 + bx + e III. Una solución factible es el punto (0,75)
Son correctas:
g(x) = mx2 + nx + p
A) Solo 1 B) Solo II C) Solo III
y •
D) 1 Y II E) II Y III

1 8. Si TI = 8k Y k E z' ,calcule el valor de R.

x
°
De las siguientes relaciones:
19. Dados los conjuntos A, B Y e en U, simplifique la
expresión
I) 112 = 4mp II) ~ = !? I1I) abc = mnp [AÓ(B ÓC)] Ó [e ÓBeJ
m n
¿Cuáles son verdaderas?
A) Solo 1 B) Solo II C) Solo III
A) AC B) Be C) e: D)A E) B

D) 1 Y II E) II Y III
20. Supongamos queA varia directamente propor-
cional a X y Z, e inversa mente proporcional a W. Si
16. Sean las sucesiones S y P donde: A = 154 cuando X = 6, Z = 11 , W = 3 , determi-
1 1 ne A cuando X = 9, Z = 20, W =7 .
So = 1,51 = 0,52 = 0,53 = 2,· ..52k-1 = k,5 2k = O; k e2
A) 120 B) 140 C) 160 D) 180 E) 200
1 1
Po = 1,P1 = 7,P2 = 0'P3 = 2,,,,P2k-1 = pP2k = 1 ; k;:: 2
21 . La suma de los radios de las bases de un tronco
Entonces los limites a los que convergen las sucesiones de cono de revolución es 2, la altura z y la generatriz
S y P son respectivamente: forma un ángulo de 6()O con la base mayor. Calcule el

A) O; ° D) No existe; 1
área total del tronco.

B) O; 1 E) O; No existe A) 87r(1+.J3) D) ~¡¡;(.J3 -1)


C) No existe; No existe
B)~7r(.J3+1) E) JJ tt (.J3 - 1)
17. Considere el problema:
maximizar z = 30x1 + 20x2 C) JJ7r(.J3 + 1)
Sujeto a las restricciones
Xl::; 60 22. En un cono circular recto está irscrita una esfe-
ra. La relación entre los volúmenes del cono y de la
x2::; 75
esfera es igual a dos. Halle la relación entre el área de
10xI + 8X2 ::; 800 la superficie total del cono y el área de la superficie
xl;::: O esférica.
x2 ;:::O A) 2:1 B) 3:2 C) 5:2

D) 3:1 E) 5:3
Dadas las siguientes proposiciones referidas al pro-
blema.
"i~~.Z~"'::=:'=Ha;¡;;;;¡;¡@:i!il:'..:'~:· m".!:'l~:__ .,L,I __ U_N_I_20_0_7_-I_M_AT_E_M_A_·
T_I_C_A_...II~== :::=w::::~
23. La suma de las inversas de los números que re- A)VVV B)VVF C)VFV
presentan a dos ángulos suplementarios en grados D) VF F E) FVV
sexagesimales es 10 veces la diferencia de las inversas
de los números que representan a dichos ángulos en el
27. Sean A = (-2;1) Y B = (4;7) dos vértices de
sistema centesimal. Halle el mayor de ellos en el siste-
un triángulo ABC, se sabe que las alturas se cortan en
ma sexagesimal.
el punto P = (~ ; ~). Entonces la ecuación de la
A) 100° B)105° C)110° D) 115° E) 120°
recta que pasa por los puntos A Y C es:
24. En la figura mostrada el cuadrado de lado 2 c.m
A) 5x + 2y - 27 = O D) x -2y = O
rueda sin resbalar hasta que el punto A vuelve a tocar
el piso. Calcule la longitud (en c.m ) recorrida por el B) 5x+ y-27 = O E) x + 2y - 2 = O
punto A.
C) x + 2y = O

28. Consideremos la siguiente expresión

f(8) = Isen(8) - ~ 1- sen(*) donde 8 E \ 56rr;54rr]

entonces el rango de f se encuentra en el intervalo.

A) (1+J2)~ B) (1 + J2)rc C) (2+ 1} A) (-1;~) (-1;~J B)

D)(2+J2)rr E) (2+2J2)rr

25. Determine tan(u) en la figura mostrada si


D) [-1;~J [-J2;~J E)

AB=BCyMpuntomediodeAB,donde DM//BC.
A
29. Al calcular la expresión

sen (2are tan ~ - are tan 152), se obtiene:

~----~------~~D B) -/3 .
A) O D)l E)-/3
3

Bu.....----------~ 30. Si sen8a + cos8a es igual a la expresión


. A + Bcos4a + Ceos8a
-/3 C) 2-/3 para cualquier valor real de a, halle A + B + C
B) -/3 + 1 2-/3 +1
C) 1. D) --
1
E) 1
-/3 8 4
E) J2 + 1
31 . En un triángulo de lados 7, 8 Y 9 m se traza la
26. Sean las funciones tan, fy g, donde f(x) =~ , mediana relativa alIado de 8 m . Determine el cose-
g(x) = x - [x] . Indique la secuencia correcta des- no del ángulo comprendido entre ei lado 7 m y la
pués de determinar si la proposición es verdadera (V) mediana trazada.
o falsa (F):
A) 41 B) 43 D) 46 E) 47
1. f o tan es una función periódica. 49 49 49 49
II. tan o g es una función periódica.
IlI. tan o f es una función periódica.
32. Dadas las curvas cuyas ecuaciones son:
y = _2x2 -.3 e y = 4x2 - S
Determine el área de la región triangular cuyos vérti-
ces son el origen de coordenadas y las intersecciones 37. En el interior de un triángulo ABC(AB = BC) ,
de dichas curvas, se toma el punto "P" tal que mLPBA = 10° ,
A) 11-13 PB = AC Y mLPBC = 30° .
B) 11h C) 11F6
9 3 3 Halle mLPAB.

D) nJ2 E) 11-13 A) 10° B) 15° C) 20° D) 25° E) 30°


9 S
38. Las tres dimensiones de un paralelepípedo rec-
33. En un triángulo isóscelesABC(AB=BC= 13m),
tángulo suman 14 u . Si una de ellas es el doble de
AC= 10 m, se traza la altura BH y luego se cons-
otra y el área total del prisma es máxima, determine
truye el cuadrado BHEF perpendicular al plano del
la tercera dimensión de este sólido.
triángulo. Calcule el área del triángulo FHA.
A) 3u B)4u C) Su D) 6u E)7u
A) 20h B) zs-Ii C)30h
D)3Sh E)40h 39. Se tiene un vaso en forma de cilindro recto, que
tiene como altura el doble del diámetro de la base. Si
34. Se tiene un triángulo equilátero, donde la dis- el vaso inicialmente está lleno de agua, y comienza a
tancia del ortocentro a la recta que une los puntos inclinarse hasta derramar la mitad de su contenido,
medios de dos lados del triángulo es 2, calcule la
formando un ángulo a entre el eje del cilindro y la
longitud del lado del triángulo.
horizontal, entonces el valor de tan (a) es (aproxi-
A)2 B) 2-13 C)4 D) 4J3 E) 8-13 madamente) :
A) 0,44 B) 0,46 C)0,48
35. En la figura mostrada, calcule la medida del D) 0,50 E) 0,52
ángulo APc.

40. En una pirámide triangular regular, la arista de


la base mide a unidades y la distancia de un vértice de
la base a la cara lateral opuesta mide b unidades.
Calcule el volumen de la pirámide.

A~~----~-----------=~D
e
A) 100° B)lOSO C)1100 D)l1S0 E)1200

36. En la figura, A, B YC son puntos de tangencia.


Sea P un punto del segmento BC tal que PA es tangen-
te común a las circunferencias. Si AP = 10 m y
AB - AC = 4 m , calcule el área del triángulo APB.
B

¡¡
__ U_N_'_2_00_7_-_' _M_A_T_E_M_A_' T_'_C_A__ ..=:.:,¡:-:7:"'
ll~m~'"'~<.~ . ~:=:· =~:;'lk.~
SOLUGIO
I) B Y e correcto,A incorrecto:
1_Del enunciado:

5= 7 + 77 + 777 + ... + 777 ... 7 + 1 + 11 P(BCA) = PCB¡PCC)P(l1)


'--v-'
11
+111+ ... +111...1
=lxlx~
~ 543
11 2 ... (1)
= 8 + 88 + 888 + ... + 888 ... 8 60
'--v-'
n
11) A Y e correcto, B incorrecto:

= 8( 1 + 11 + 111 + ... + ~ ) P(ACB) = PCA¡PCt"¡P(S)

Multiplicamos por 9 y dividimos por 9. =lxlx~


S 4 3
2 ... (2)
60
S=%(9+99+999+ ... +~ ) 111) A Y B correcto, e incorrecto:
= %[ (10 - 1) + ( 102 - 1) + ( 103 - 1) + ...
P(ABC) = P(A¡PeB¡P(C)

=lxlx~
+ (10" - 1) ] S 4 3
2
= -ª:[
9 1O + 10
2 + 103 + ... + 10" - nJ 60
... (3)

IV) A, B, e correcto:
= %[ ( 1+ 10 + 102 + 1~3 + ... + 10" ) - n - 1]
PCABC) = PCA¡PCB¡PCC)
_~[101l+1_1 __ ]
- 9 10-1 n 1 =lxlxl
543
= %( 10"+1 -/n -10 ) 1
60 ... (4)
1 Luego, la probabilidad total de que por lo menos 2
= 88 (10"+ -9n-10)
1 pronósticos sean acertados, de O), (2), (3) y( 4):
Clave: D
P = p(ABC) + P(ABc)+P(ABC) + PCABC)
2. Son los pronósticos de los pobladores para los 2 2 2 1
= 60 + 60 + 60 + 60
equipos:
=2
60
~ = 0,116
Ira 2do 3ro
Son S equipos en total. Clave: D

Debemos calcular la probabilidad P de que al menos


2 prónosticos sean correctos:
Se tiene los siguientes casos:
~;. ::=i=~~=~~
__ ••••I:::ww'7Wcr~ijWW!j11'"
U_N_I 2_0_0_7_-I_M_AT_E_M_Á_T_IC_A_
L.::o!!='-"

3. Del enunciado: 5. Como el automóvil viaja desde la ciudad A, los g


-- 2 galones de gasolina que le quedan son de b octanos.
14 abed64 = ( abed)
Al llegar al grifo de la ciudad B llena el tanque así:
-- 2
14000064 + abedOO = (abed)
Galones Octanaje
14000064 = abed(aiJCd -100)
g b
14000064 = abed x a( b -1 )ed x a
Operando con la expresión: y d

abed xa( b -1 )ed = 26 x 3x 13 x71 x79 Como la capacidad del tanque del automóvil es G
galones, entonces:
= 24 X 22 X 3 x 13 x 71 x 79
g+x+y=G '" (1)
= (16x 3x79) x( 4x 13x 71)
=> x=G-g-y ... (2)
= 3792x3692
y =G - g -)(\ ... (3)
=> a=3 ; b=7 ; e=9 ; d=2
Pero el grifo está en la ciudad B y el automóvil en está
La suma de estos números:
ciudad usa gasolina de octanaje e, es decir:
a+b+e+d=3+7+9+2
gb+xa+ yd
= 21 "'----"-- =e
g+x+y
Clave: E
gb+xa+ yd =e(g+x+ y)
4. Se tiene el "producto" cuyo precio de listra es "P" DeO): gb+xa+yd=eG ... (4)
lera.Tienda: hace dos descuentos sucesivos de 15% De(3): gb+xa+(G-g-x)d=eG
y 15%, quedando al precio:
g(b-d )+x(a -d) = G(e -d)
P1 = P(100-15)%(100 -15)% (e-d)G-(b-d)g
8 5 8 5 => x= a-d
= p( 1 0 0)( 1 0 0)
Ahora, si en la expresión (4) reemplazamos (2):
=0,7225P ... (1)
gb + (G - g - y)a + yd = eG
2da. Tienda: hace un primer descuento del 30%
quedando al precio: (e - a )G - (b - a)g
=.»> d-a
P2 = P(100 - 30)%
Clave: E
= p( 17000) 6.
= 0,7P ... (2) a) Si x es la media, y Xc la media con error, luego
Comparando los precios P1 y P2 de las dos tiendas, cuando se calcula para los 10 números en la
se puede observar que P1 > P2, por lo tanto, debe- expresión:
mos incrementar el precio E2 para obtener el precio Con error:
P3 que sea igual a PI ,y de esta manera se cumpla
con la condición del problema:
Sea x % el porcentaje a incrementar, entonces
Sin error:
P1 = P2
10
07225
,
P = (O , 7P)(100 100+ x) S = ¿'(x¡ - x)2
i=1
=>x=3,2 (%) Luego si ejecutamos la resta:
Clave:B
;¿F~~.Z~~~~~:;~~,~~~J~
_ _IC_A U_N_12_0_0_7-_I_M_A_T_E_M_Á_T

lO 2 lO 2 8. Datos: La taza de intéres anual única: a %


Se - S = 2Jx¡ - xc) - ¿(xi - X) Cantidad total que se recibe: S
¡=1 ¡=1
Sea V" el valor nominal de ambos pagarés.
lO 2 2]
= ¿[(X¡-Xc) - (Xl -X)
¡=l Cálculo de los descuentos:
10 • Ven~e en 30 días (lmes):
= ¿[ X¡ - 2x¡xc+x/-X¡ + 2x1 X_X2 ]
¡=1
10
D¡ = VII (a%)
12 (1)
= ¿[2x¡x+x/-2x¡x -x2+(X C Xc-X Xc) ] ev,
¡=1
= 1200 ... (1)
10
= ¿[2x¡(x-xe) -Xe(X-Xe )-X(X-Xe)] • Vence en 60 días (2 meses):
¡=1
10
= ¿[(X-Xe )(2x¡ -Xe-X)] D2 = V" ( ~~ )(2)
¡=l
10
2aV" (2)
= 1200 ...
= (X-Xc )¿(2x¡ -Xc-X)
¡=l Si se recibe en total S nuevos soles, entonces:

= (X-Xc )[(2 )I,X¡ -10 Xe-10X] (V" - DI ) + (VII - D2) = S


1=1
2V" - (DI + D2 ) = S
= (X-Xc X(2)10x-10xc-lOx]
aVII 2aVII) S
= (X-Xc )[10x-10xe] De (1) y (2): 2V11 -
( 1200 + 1200 =

= 1 O( x-xe )( X-Xc ). 400 S


=> V" =, BOO _ a
= 10( x-xc)2

Observe que esta última expresión siempre será Clave: D


positiva, lo cual nos indica que la diferen-
cia Se - S , es positiva, entonces se ha aumenta- 9_ Para determinar el valor de "n" en las expresiones
do con respecto a S. de los números:
b) Si el dato menor disminuye, la media
A=IBO"x27
disminuye(D); pero la mediana permanece (P)
en su misma posición y conel mismo valor. B = 40" x 60
Clave: B Dato: La cantidad de divisores del mínimo común
multiplo deA y B:
7 _Del enunciado:
CD = 5400 ... (1)
O,ab + O,b~ = 1,4 Hallando la descomposición canónica de ambos:
Multiplicando por 10: A = 22" X 32,,+3 X 5"
a,b + b,éi = 14,4 B = 23"+2 X 3211+3 X 5"+1
a + 0,6 + b + O,éi = 14 + 0,4 El mínimo común múltiplo de estos números:
a+Q+b+g=14+1 m.c.m (A; B) = 2311+2 X 3 X 5"+1 ... (2)
9 9 9
10(a+b)=130 Por teoría, la cantidad de divisores (CD)de un núme-
9 9 ro, expresado en su forma canónica; en este caso del
=>a+b=13 m.c.m.(A;B), es:
Clave: E
0>. dhWWWW'7fWfPWWwt'Yml __ U_N_12_0_0_Y-_I_M_A_T_E_M_A_'T_I_C_A __

De (2): CDmc.m(A;B) =[(3n +2)+ 1][(2n+3)+ l][(n+ 1)+1] Para hallar un valor de x ' , de tal manera que:
De (1): 5400=(3n+3)(2n+4)(n+2) P(x*)=O
5400 = 3(n + 1)(2)(n +2)(11+ 2) En la expresión (S):
~ 900= (n +l)(n+ 2)2 P(x*)=1(x*)2 +x * _ll. = O
3 3
9x102 = (n+ l)(n+ 2)2
~n+1=9 -l± 1(1)2_4(1)(_13)
n=8 ~x*= V 3 3

Clave:C
2( 1)
_ -3+J217
10. En el siguiente número: - 8
N = 130130 Uno de los valores es:
= 130000 + 130
x* = -3 + J217
=130(1001) 8
Su descomposición canónica: Clave: B

N = 2xSx13x(7xllx13) 12. Dada la siguiente desigualdad:


=2xSx7xllx132 ,11- x + -/1+ x <':
~ ~
M-
'-.-'
(1) (11) (111)
Los números 2, S, 7, 11, 13 son primos, que en canti-
dad son S. Estos números constituyen los divisores De (1):
primos del número N. 1-x<':0 ~ x sI
Clave:C
~ cs.i :« E (-00;1] ... (1)

11 . Se tiene el polinomio: De (11):


l+x<':O~x<':-l
P(x) = ox2 + bx + e .,. (1)
... (2)
Datos:p(1)=-2; P(2)=3; P(S)=34
De(I1I) :
Reemplazando datos en (1):
x<Ovx<':O
0(1)2+b(1)+C=-2
~ o + b + e = -2 .,. (2) ~ CS3: XE (-00; 00) .._ (3)

Luego de (1), (2) Y (3):


0(2)2+b(2)+c=3
CS1,nCS2 nCS3 = [-1; 1]
~ 40 + 2b + e = 3 ... (3)
Clave: E
0(S)2 + b(S) +c = 34
~ 2So + Sb + e = 34 ... (4) 13. Teorema de Cardamo para un polinornio

Resolviendo el sistema de ecuaciones conformado por 0lx" + 02x"-1 + 03X"-2 + _.. + o '_lx + 0l!
(2), (3) Y (4), se obtiene:
o= 1 . b=1 . e= _ll. Suma de raices = _ a2
3 ' , 3 01

Luego el polinomio de (1) podrá expresarse: En la ecuación del enunciado:


P(x) = 1x2 + x_ll.
3 3
15.Se tiene las funciones:
2
16( Z2 - 2iz - 1) = z4 f(x)=ax2+bx+c
16( z4 - 4iz3 - 6z2 + 4iz + 1) = z4 g( x) = mx2 + nx + p

lSz4 - 64iz - 96z2 + 64iz + 16 = O Sus gráficas:


y
Aplicando el teorema de Cardamo:

de raí (-64i) 64·


Suma e ratees = --1-5- = El

Clave:E

14. Las funcionesfy g estan definidos en:


o x
lR-{l}=>xelR-{I}
Gráficando: Observe que A es un punto de tangencia con el ejex
para f (x) y g (x) , por lo tanto, ambas tendrán una
ef(x)=Tx+l; xelR-{I}
sóla raíz; Entonces para calcular sus raices:
y
Para f( x) : x = b + ~b2 - 4ac
"'-i< 2
2a
1 :

~ O 1 x O 1 x => b2 - 4ac = O
(I) (II)
=> b2 = 4ac ... (1)

Para g( x) , similarmente:

n2-4mp=O
O 1 x
(III) => n2 = 4mp ... (2)

Además, como las ecuaciones


e g(X) = 2l/(x-l)
ax2 + bx + c =O ; mx2nx + p =O
tienen las mismas raices estas son equivalentes, en-
tonces:
a b .f. ... (3)
O x O 1 x m n p
(IV) (V)
De las siguientes relaciones:
Uniendo las gráficas (IlI) y (V):
1. n2 = 4mp (verdadera); por (2)

II. -ª-m = Q (verdadera);


11
por (3)

III. abc = mnp; (falso). No es posible obtener esta


expresión de las ecuaciones anteriores.
Clave: O
x

Clave: B
16. La sucesión s. O;S: xl ;S:60 .... (1)
50 = 1 ; 51 = O ; 52 = O ; 153 = 2:1 ; ... O ;S: x2 ;S: 75 ... (II)
·5
, 2k-l
-l:sl.-O·
- k /2k - ,
k>2
- 5x + 4x2 :s; 400 ... (IlI)

Para calcular a donde converge en el límite esta suce- Gráficando las relaciones:
sión debemos tomar encuenta que ésta presenta dife-
(I) X2 (II)
rentes valores para los números pares e impares cons- X2

tituyendo dos sucesiones en una, veamos:


75
1
lim(52K_l)=
k-.~
lim (-k )
k-.~
= O 0$x¡$60
0$Xz$75
lim (52d = k-.~
lim (O) = O
k-.~

Como los límites de los dos subsucesiones convergen a 60


"O",entonces podemos afirmar que éste es el límite al
que converge la sucesión "5" si existe.
La sucesión P:

En el límite, considerando que debemos dividida en


forma similar a la sucesión anterior:
1 Intersectando las 3 gráficas:
lim (P - )
k-.~ 2k l
= k-.~
lim (-k ) = O
X2

lim(P d= lim(l)=l 100


k-.~2 k-.~
. Como los límites de los dos subsucesión convergen a
dos números diferentes, se concluye que el límite de P
no existe.
Clave: E

_-+- L--_-"- _
1 7. Para maximizar: 60 80 Xl
z = 30xI + 20x2 ... (1)
Luego se puede afirmar que los valores que pueden
Con la siguientes restricciones: asumir xl y x2 se encuentran en la región
xl ;S:60 sombreada. Los valores máximos de estas variables se
ubícaran en el extremo superior derecho, es decir, en
x2 :s; 75
el segmento PI P2 .
10xI + 8x2 ;S:800
Para que z sea máximo, los valores que deben asumir
xl ~ O las variables xl y X2 deben estar ubicado en uno de
X2 ~ O los vertices, ya sea Pl o P2 que pertenece a la recta:

Estas restricciones podemos resumidas en: 5xI + 4x2 = 400 ... (2)
r,s.:~.~
. ~';;WWY'W&
GomeZ
·~l Mi' '",dl'i;],% .
UNI 2007-1 MATEMATICA~i!íid%¡ 1=; ' *===r ;;:;~ +~" 241.~
.~

En el punto p¡ (x2 = 75):


="8j:1 [ ((1+i)2) 4k +((_1+i)2) 4k]
De (1): 5xI + 4(75) = 400 ~ xl = 20 22
= -k(l + 2i _1)4k + (1- 2i _1)4k
~ PI = (2 O ; 75) 2

En el punto P2 (xl = 60 ) = _1_[


4k
(2i )4k + (-2i )4k ]
2
De (2): 5( 60) + 4 x 2 = 40 ~ x2 = 25 = _1_[(2i)4k + (2i)4k]
24k
~P2=(60;25) = _1 [2(2i)4k]
Reemplacemos ahora estos valores en la expresión 24k
(1) para verificar con cual de ellos "z" es máximo: =_1 [2(2)4k]
24k
Con PI: =2
z = 30(20)+20(75) = 2100 Clave: C

Con P2: 19. Diferencia simétrica, por definición:

ItJDj
z=30(60)+20(25)=2300 ; (máximo)
En las proposiciones:
l. "No existe región admisible". INCORRECTA PaQ
- La región admisible si existe, y esta compuesta
Del gráfico:
por la región sombreada que se muestra en el
gráfico. P ~ Q : ( P n QC ) v (Q n pC ) ... (1)
]J. "El óptimo se dá en el punto (60; O) ".

INCORRECTA Deduciendo para P ~ QC


- La solución óptima (máxima) se dá en el punto
(60,25) Oe(l): P~Qc =(pn(Qc)" )V(Qc "pc)
III. "Una solución factible es el punto (O; 75) ".
= (pnQ)v(Qc npC)
CORRECTA
- Observando el gráfico se deduce que el punto =(p~Qt ... (2)
(O; 7 5) pertenece a la región sombreada, por
lo tanto éste es una solución facctible. Del enunciado, si A, B Y e E u , se pide simplificar:
Clave: C M=A~(B~C)~(C~BCI
18. Si n = 8k Y k E z' De (2): =[A~(B~C)J~(C~Bl
'-------r----' '---.r---'

R= (112+ 12L
l.)" +(1-12+ 12l.)" 1

= {[ A~(B\~
P

C)J~ (C~B)r
Q'

Jzit Jzif = {[ A~(B~ C)J~ (B~C)r ; c')

r
=(Jz+ +(-Jz+

= ( Jz t (1 + i )8k + ( Jz (-1 + i )8k


={A~[(B~C)~(B~C)J}C
={A~p'r
(*,,)

= [( Jz f (1 + i )8k + (-1 + i )8k ] = AC


Clave:A
~~~'=.!:~;I __U_N_I_20_0_7_-_1
_M_A_T_E_M_A_·T_I_C_A_....I

(*): Propiedad conmutativa Además del gráfico:


(*,,): Propiedad distributiva PQ + r + R = 2R

20. Del enunciado: De (3) y (1): ~J3+2=2R


A es directamente proporcional aXy Z
=}R=l+J3 ... (4)
A es inversamente proporcional a W 3
Entonces por propiedad se cumple: Reemplazando (4) en (1):

~'.~ =K=Collstantc ... (1)


(1+1)+r=2
Por dato del problema:
Si A = 154 =}r=l-J3 ... (5)
3
=}X=6; Z=l1; W=3 Del gráfico también se tiene:
Reemplazando estos valores en (1): sen60° = h.
g
(lS4)(3)=7=K ... (2) J3 2 (4)J3
(6)( 11) 2 g =} g=-3- ... (6)

Luego, si ahora se tiene los valores X = 9 , Z = 20, La superficie total, aplicando (1), (3), (4), (5) Y (6):
W = 7 , también se cumple la propiedad (1):
ASlIpcrjicic = AslIpc,jicic + ABases
A(7) tocal lateral
De (2):
(9)(20) = 7
= re(R + r)g + (rer2 + reR2)
=}A=180
Clave: D = re(2l(1J3) + re(1-1
= refiJ3 + refi
r + re( 1+ {3 r
21. Del enunciado: 3 3
,---'---""'4 = ~re(J3+ 1)
r:
I
I Clave: B
,,
I

I
I
:h
I
I
I
I

q9~.. ~-_-rL-~-__-~-~-~L~-~·~-~-~-~-_-~-~·~-~-~-:·:::'
Q
I----R r .1. R .1
Datos:
R+r=2 ... (1)

h=2 ... (2)

Del gráfico:

PQ = ctg S')" A R H R C

R : Radio de la base del cono
PQ _ 1 r : radio de la esfera .
De (2): 2 - J3 =} ... (3)
g : generatriz del cono
Dato: Por condición del problema:

... (1) 1+ 1=
x y
10(.l _.l)
C1 C2
Por teoría:
De (2) y (3): 1+ 1= 10(-.L _-.L)
x y 10x 10y
VCOIlO = ~1tR2h ; Vcsfcra = 11tr3 ... (2)

En el triángulo HBC, por semejanza:


x +y =1
xy 10 x _1)
o(~)(l y
OB = OT
CB HC ~
xy
= 9(Y-X)xy
-g-="R. r ~
h - r r
h="R.(g+R) ... (3) x + y = 9(y - x)
De (1): 180 = 9(y - x)
0

Reemplazando (2) en (1):


~y-x=20° ... (4)
11tR2h
_3__ =2 Resolviendo las ecuaciones (1) y (4) se obtiene:
i1tr3 y=100 x=80°
3
2 El mayores y = 100 0

Rh =2
4r3 Clave: A

Rt~(g+r)J --2 24. Del gráfico del enunciado:


De (3): B C
4r3
R(g+r)=2 ... (4)
4r2
D
La relación de áreas de las superficies de las dos figu-
ras geométricas: Dato: i! = 2 c.m ... C')
A _ 1tR
2 + 1tRg
collo Gráficando todas las posiciones del puntoA:
Acsfcra - 41tr2 I) B,--------( A B
R( R + g) : L¡

4r2 :, e
,
,,
De (4): =2
Clave: A A i! D C

23. SeanX e Y ángulos sexágesimales complemen- L = l(21te) = 1ti! ... (1)


1 4 2
tarios, entonces:
x + y = 1800 ... (1)
II)
Transformándolos a centesimales:
A A
Para x: ,,
i!:,
... (2)
,,
,
D
~ ~
Para y:
180 200
L2 = i[ 21t(f2 e) ] = f2 1ti!
2
... (2)
~C2=10y ... (3) .
9
"~-250 ~~=======:'[I=~~~1~2~O~~7-~~~~~~~!'~I@!=:I--
. - ···~Iittil\.
~~«.~~!~ _-;;;§:;::;iii;.;;;44;;.;t~;;;¡· _ .. '¡¡¡¡¡-¡¡¡¡¡'

GomeZ

III) D AC D
Del gráfico:
1- - - - -- - --

I
I
NC = -J3b + b ___(2)
I
I
I Reemplazando (2) en (1):
I
I

AN =NC
C B 1. A
AN = J3b +b ___(3)
L3 = l(2nl.) = nI. ... (3)
4 2 Cálculo de tga ,en el triánguloADP:
La longitud recorrida por el punto A
tga = DP
Ltolal = L1 + L2 + L3 AN+NP
= nP. + nP.-fi + nP. Reemplazando valores de (2), (3) Y gráfico:
De (1), (2) Y (3):
222
tga. = bJ3
= n p.(2 + -fi)
2
(bJ3 + b) + bJ3
= n(22cm)(2 +-fi)
J3
De (1):
2J3 + 1
= (2 + -fi)n cm Clave: A

Clave:D 26. Dadas las funciones:


25. Del enunciado:
J(x) = 1
x
... (1)
A

g(x)=x-[x] ._. (2)

Analizando las proposiciones:


I. "J o tan es función periódica". Verdadero (V)
Verificación:

J o tan(x) = J(tanx)
De (1): _1_
tanx
= ctgx
Datos: AB = BC ; M: punto medio deAB
Por propiedad: - Dom(J o tan) = lR

AN=NC ..- (1) - Período de ctg x = n

En el triánguloADC; asumimos DC = Zb : n. "tan o g es función periódica". Verdadero (V)


A Verificación:

-(tan o g)(x) = tan(g(x))


De (2): = tan(x - [x])

- Dom(tano g) = lR

- tan( x + t - [x + t]) = tan(x - [x])


El periódo (n solo depende de la función máximo
entero ([ x ]) cuyo período es 1:
Entonces el periódo de" tan o g" es 1.
III. tan 01 es periódica. Falso (F) Datos:
Verificación: (XA ;YA)=(-2; 1)

(tan o J)(x) = tan(J(x)) = tan(l/x) (xB ;YB)=(4; 7)

Identidad trigonometrica: (xp ;Yp)=(~;~)


sen a cosl3 - sen 13cosa = sen( a -13) ... (*)
Del gráfico, la pendiente de BM:
Por definición de función periódica. YB - Yp
mB", = mBP =
tan x !T = tan(¿) xB - xl'

7-~
__ 3
~ tan(x! T) - tan(~) = O 4-1
3
sen(xh) _ sen(~) = O = 2 ... (1)

cos(x !T ) cos(~) Como BM .1 AC ,entonces por propiedad:


sen(xh )cos(~) - sen(~ )Cos(xh) = O (111 BM )(111 AC ) = -1

cos(x !T )cos(~) 2mAC =-1

=> 111AC = -2:1 ... (2)


sen(-l- _1)
De C): x +T x =O La ecuación de la recta que pasa por A y C:
cos(x !T )cos(~) y - YA _
~ sen(_lx+T__ 1)
x
=O
----mAC
x -xA

~ _1 __ 1= kt: (k E Z) y -1 1
x+T x
De (2) : x-(-2) -2:
=> T = knx
2 x + 2y = O
knx + 1 Clave: C
Observe que Tes una variable porque depende de la
variable "x", por lo tanto la función no es periódica.
Clave: B 28. Dada la función: 1(8) = Isene - %1- sen* '

27. Del enunciado: donde 8 E (51t/6 ; 51t/4]

y Graficando las cotas de sene en el círculo


7 B(xB; YBJ trigonométrico:

(XA;YA)A¡
~~~~~----~~------
x

C
Partiendo de las cotas que estan definidas para e ,y 30. Del enunciado:
que se indican en el gráfico, vamos a llegar a formar
sen8a + eos8a = A + Beos4a + ecos8a (1)
la función f(e) ,así:
000

Por teoría:
_ J2 < senü < 1
2sen x eosx = sen2x (I)
2 - 2 000

_ J2 _ ~ < sene _ ~ < 1_~ cos2 x = 1 + cos2x 000 (II)


2 5- 5 2 5 2
_ J2 _ ~ < sene _ ~ <~ sen2 x = 1 - cos2x 000 OJO
2 5- 5 10 2
0<- Isene - 5 - ~I< J22 + ~5 Por identidades trigonométricas:
sen2a + eos2a
O - senJ!. < Isene - ~
4- 5
1- senJ!. < J2 + ~ - senJ!.
4- 2 5 4 2
=1

(sen2a+eos2a) =(1)2
_ J2 < Isene _ ~ 1_ senJ!.4-252
< J2 + ~ _ J2
2- 5 sen4a + eos4a + Zsen2aeos2a =1

- J2
2
< Isene - ~
- 5
1- senJ!. < ~
4-5 => sen4a+cos4a = 1- Zsen2aeos2a

= 1 - ~( 4sen2 aeos2 a)
~ - J2
2
< f(e) < ~
- - 5
De(I): = 1-1sen2Za
2
•~ Ran(J(e)) = [-1 ~] De(IlI) : = 1 _ ~(1 - e~s4a )

Clave: D = 1 + 1eos4a
4 4
29. Del enunciado: Elevamos al cuadrado:

(sen4u+cos4at
are tan 152)
= (cos4a)2
E = sen( 2are tanl- (1)

.
000

'-----v-----' sen8a+cos8a+2seI14u 4
cos a= (6 + /6cos24a+~cos4a
Por teoría, si x y < 1 , se cumple: Entonces se tiene:

are tanx + are tany = are tan(; _+; ) 000 (2)


sen8a+cos8a = ?6 + 1~cos24a+~cos4a-2seI14acos6a
=.2.. +.1...(1+cos8a)+ lcos4a-
16 16 2 8
Calculando la expresión (*) de (1):
~(16sen4a cos4a)
Zare tanl = are tan! + ~re tanl _9 1 1 3 1 4
-16 + 32 + 32cos8a+ gcos4a-gscll 2rx

De(2): =aretan ~
1+ 1 1 Oe(lll): 9 1 3 1 1(1-COS4a)2
= 16 + 32 +gcos4a+ 32cos8a-g --2--
[ 1- x
ss 9 1
= 1 6+ 3 2+~cos4a + 3 2
cos8a - ~(~ +
1
5
= are tan(1 Z) 000(3)

1cos24a -lCOS4a)
4 2
Reemplazando (3) en (1): Oe(II):
9 1 1 3 1 o

= 16 + 32 - 32 + gcos4a + 16cos4u+
5 .1...cos8u-.1...(1 + cos8a)
E = sen[ are tan(1 2) - are tan(t2)]
32 32 2
= sen(O) 7 1
= ?6 - 6~ + 1 6cos4u + 3 2cos8a - 6~ cos8a
=0 35 7 1 (2)
= 64 + 16cos4a + 64 cos8a 000

Clave: A
~=':'=:::i!I'llW!@iiiml'MWMl:=:';"~~.'1__ U_N_I_2_0_07_-_1 _M_A_T_E_M_Á_T_IC_A_ •••• I ::=¿;,
Reemplazando (2) en (1): y

~~+ :6 cos4a + 614cos8a = A + Bcos4a + C cos8a


Por comparación se deduce que:
_ 3S. _ 7. _ 1 x
A - 64 ' B - 16 ' C - 64

Calculando la suma:

Clave: E

31. Del enunciado:


B

La intersección de las dos curvas, de (1) y (2):

9
Luego en (2):

AL----7L---~--~~--~C
1-,-- ,1
Por relaciones metricas (teorema de la mediana):
Entonces:
-2
2BM2
+ AC
2
= BC
2
+ AB 2

A = (- ~
1
; - 13 ) B=(~
J3' .11)
2b +
2
2
ª 2
= 7
2
+9
2
El área del triánguloAOB:
-3

~b=7 ... (*)

Por la ley de cosenos:


s = ABxOM
2
2 2 2 _(xB-xA)x(YH-YO)
4 = 7 +b - 2(7)(b )cos6 - 2

[-33 - (--33 )] x [ J( -lf t -


De (*): 16 = 49 + 72 - 2(7)(7)cos6
~ cos6= 41 = O )
49
2
Clave: A

32. Dadas las curvas cuyas ecuaciones son:


C¡ : y = _2x2 - 3 ~ Y +3 = -2x2 (1) .

C2 : y = 4x2 - 5 ~ Y +5 = 4x2 (2) Clave: A


Graficando el área triangular formada por la in-
tersección de estas dos curvas y el origen de coorde-
nadas.
33. Del enunciado:
FHxHA
SplIA =-·-2-

(12J2)(5)
De (2) Y (4):
2
= 30J2 ; (m2)
Clave: C
C

34. Por teoría, en un triángulo equilatero:

Ortocentro (H) = Baricentro (G)


= Incentro (1) ... (1)

Del enunciado y de (1):

A
Datos:

AB = BC = 13 m} ... (1)
AC = 10 m

Como el triángulo ABC es isósceles, la alturaBHtam-


bién es mediana:

De (1): AH = HC = lAC = 5m ... (2)


2
Aplicando Pitagoras en el t:. ABH : A~~~------------~~~C
2 2
AB = +AH2 + BH Del gráfico:
De (1) y (2): 132 = 52 +BH2 sen30"= HP
~ BH=12 ... (3) HN
1= 2 ~ HN = 4 ... (2)
2 HN
Además, como BH es el lado del cuadrado BHEF, en-
tonces la diagonal FH es: También:
tg600 = CN
De (3): FH = J2BH = J2(12) ... (4)
HN
Por el teorema de las tres perpendiculares: De (2): J3 = b/2 ~ b= 8J3
4
FB 1. plano( ABC) y BH 1. AC Clave: E
~ FH 1. AC
35. Del enunciado y complementando datos:
El área del triángulo FHA :

F~: A A~~~~--~--------------~~D
UJi~~ .V,,"!!J'!.jJi;;,.;;¿lffffiflLKl
UN12007-1 MATEMÁTICA 1,,*¡¡¡¡¡¡¡¡¡"¡¡¡¡iU!:~1II:· iúi===~.255 ~
r G~ñleZ iiíi iGb •.• f~".~ "'""1 =~mti"MÚnÚm%f· ~l!:cW~~~

Se traza la ceviana BQ que pase por P. Clave: 8


Luego aplicamos los siguientes teoremas: 36_ Del enunciado:

I) Teorema de la bisectriz exterior en el triángulo


ABC, donde BD es la bisectriz exterior.
AB AD
... (1)
BC= CD
11) Teorema de Ceva, en el triánguloABC:

( AM )( BN )( CQ ) = (BM )( CN )( AQ )
(AM)(BN) (AQ)
~ (MB)(CN) = (CQ) ... (2) Datos:
IIl)Teorema de Menelao, en el triánguloABC y la AP = 10 m ... (1)
recta secante MD
AB - AC = 4 m ... (2)
(AM)(BN)(CD) = (MB)(NC)(AD) Por pripiedad de tangencia:
(AM)(BN) (AD) De (1):
~ (MB)(NC) (CD)
Entonces AP es mediana.
De (2) Y (1):
AQ AB
QC = BC B

De esta última expresión, por el teorema de la bisectriz C


interior, dado que BQ es bisectriz (P es in centro)
B
A
Donde:
2ex+2p=180°
ex+ p = 90°

Se concluye que el /j, BAC es rectangular enA.


A~------~--~C
Del gráfico:
En el triánguloABC:
2ex+ 28 + 30° = 180° m2 + n2 = (20)2
2(ex+8)=150° = 400 ... (3)

ex+ 8 = 75° ... (3) Por dato del problema:


En el triánguloAPC: AB - AC = 4
ex+8+x=180° m-n;'4
De (3): 75 + x = 180°
(m - n)2 = 16
~x=105°
2
m + n2 - 2mn = 16
También por propiedad:
De (3): 400 - 2mn = 16
x = 900 + mLABC
2 ~mn=192 ... (4)
= 900 + 30° Cálculo del área del triángulo APB, considerando
2
que PA es mediana:
= 105°
~i~;;~;;~=~::-:__U_N_I_2_00_7_-I_M_AT_E_M_A_·T_IC_A_....II;~:,...:_=~=;;;;=~.t....:=:.==...
Pero el thánguloABC tambiénes isósceles, entonces:
- AABC
=> A APB - 2 ~=600+a ... (2)

=}(m~ n) Por suma de ángulos internos, en el triángulo ABC:


(10° + 30°) + (60° + ex) + [3 = 180°
De (4): =}(1~2) ex + p = 80°

= 48 (m2) De (2): ex + (60° + ex) = 80°


ex = 10°
Clave: A
Luego tenemos el gráfico así:
B
37. Graficamos según el enunciado:
B

Datos:
AB =BC = a

BP=AC

10°
A~----~'-----~C
El trianguloABP es congruente al rriángulo Anr", por
De los vértices A y B trazamos dos segmentos de recta lo tanto podemos deducir que:
de longitud a, de tal manera que se forma un triángu- 6 = x = 20°
lo equiláteroABD.
B
Clave: C
38. Según el enunciado se tiene:

U
Por dato:

2a+a+b=14
3a + b = 14
2a =>b=14-3a ... (1)

El área total del paralelepípedo:


D AtoLa/ = 2( 2a2 + 2ab + ab)

'" 2( 2a2 + 3ab )


ex 2
De (1): = 2[ 2a + 3a(14 - 3a) ]
Como el triángulo BCD es isósceles, entonces:
= 14a(6 - a) ... (2)
<p = 6 + 60° ... (1)
En el triángulo BCD, por propiedad de suma de ángu- Analizando la expresión (1) :
los internos: b=14-3a => 3a < 14 => a < 14
3
20° + <p+ (6 + 60°) = 180°
Pero como a debe ser un número entero positivo:
De (1): (6 + 60°) + 6 = 100°
a=1;2;3;4
6 = 20°
_./tll~~";;=:·=:.;,t&:.:t&&l:·
.",,~u.:
~ G~nteZ
* !j¡¡t::: .:..=:}..:~:. =:[I~U~N~I
. *g*lk:h1W:'t4L..
--
~20~O~7~-I~M~A~T~E~M~Á~T~IC~A~=I·.'"
_

Luego en la expresión (2): Se traza la altura al plano que contiene al triángulo


equilatero ABC, donde Hes el baricentro.
Atotal = 14a[6 -a]

Para a=l: Atotal=14(1)[6-(1)J=70

Para a = 2: Atotal = 14(2)[6 - (2)J = 112


Para a = 3: Atotal = 14(4)[6 - (4)J = 126 ... (3)

Para a = 4: Atotal = 14(4)[6 - (4)J = 112


Luego el área total es máxima (3) cuando a = 3 .
Finalmente en (2):
b = l4 - 3( 3) = 5 A~--------~H~----~
Clave: C 1_.---./3 2
a ---- ...•.

En el triángulo rectángulo AQM:


39. Cuando el vaso de forma de cilindro recto se ha
inclinado a un ángulo a con respecto a la horizontal:
MQ = ~3a2 _ b2 = lJ3a2 _ 4b2 ... (2)
4 2
Por semejanza: t,. MHP - t,. MQA
.e. =AM
HM QM

.e. %J3
De (1) y (2):
af! = ~J3a2 -4b2

2
~ .e.=~~a~== ... (3)
Por condición del problema, la altura del "cilindro" es 2J3a2-4b2
el doble del diámetro de base:
El volúmen de la pirámide:
h = 2(2r) = 4r ... (*)
Vp-ABC =~(ApBd(b)
Luego del gráfico:
r = ~(~.e.a)b
tga = h/2
De(*):
= _r_ ~ tga = 0,5 De (3): = 11 a2 a } b)
4r/2 3 (2 2J3a2 _ 4b2
Clave: O a3b
40. Del enunciado: P

Clave: C

A
~L3~~~
MATEMÁTICA

1. Cuánros números de tres cifras tienen la raíz


é

cuadrada y la raíz cúbica con el mismo residuo no H¡


nulo?
1,00 - .. -.-- - .
A) 52 B) 53 C) 54 D) 55 E) 56
0,75 - - -.¡
2. La fracción -t ' como una expresión decimal en
0,50 :
base dos, tiene la expansión:
A) 0,00111111... D) 0,011011011... 0,25 --.. ,----~------~-----i
B) 0,00110011... E) 0,101101101... Notas
90 100
C) 0,10101010 ...
Determine los valores para las proposiciones 1, II Y III:
1. ¿Cuántos de los evaluados obtuvieron notas
3. Halle la cantidad de pares de números de modo
entre 70 y 80?
que su MCD sea 36 y estén comprendidos entre
750 y 950. Il. Qué porcentaje
é de evaluados tienen notas
menores a 65?
A) 9 B) 10 C) 11 D)12 E) 13
IlI. Si hay en total 400 evaluados, écuántos obtu-
4. De una baraja (52 cartas) se extrae un grupo de vieron notas entre 90 y 100?
cartas (menor a 52) tal que la tercera parte son A) O ; 50% ; 80 D) O ; 50% ; 100
corazones y la quinta parte son espadas. Obtenga
B) O ; 45% ; 80 E) O ; 50% ; 120
la cantidad de cartas de los posibles tréboles ex-
traídos; sabiendo que el número de diamantes co- C) 20 ; 45% ; 80
incide con el de corazones.
8. Halle la cantidad de oro puro contenido en un
A) 2 B)4 C) 6 D) 8 E) 10 aro de 18 quilates cuya masa es de 28 gramos.
A) 18 B) 20 C) 21 D) 22, E) 24
5. Cuantos números enteros positivos b tienen la
é

propiedad de que Logb531441 sea un número


9. Señale la alternativa que tiene la secuencia co-
entero?
rrecta, después de determinar la veracidad (V) o
A)2 B)4 C)6 D)8 E)12 falsedad (F) de las siguientes proposiciones.

6. En el número 16P61(n), P es 11; entonces la 1) aE (O; 1) <=} _a_ E (O '+ 00)


1-a '
raiz cuadrada en base n es:
II) 1 s x < 2 <=} 0< ~2 - x < 12
A) 113 B) 123 C) 130 D) 131 E) 132 - 2x 2

7. El siguiente gráfico representa las frecuencias


relativas acumuladas (H¡) de las notas en un exa- IlI) -2 < x < -1 <=} 4 < 2
I~ <8
men.
A) VW B)VVF C)VFF D)FFV E)FFF
~~~;:~~~M~-~'I
__U_r_J1_2_0_07_-_II_M_A_T_E_M_A_'T_IC_A_ •••••~~:..;. ;'~:m::d:lliI!Ii!===:1J.I'%1IIIi'iIl=~~

10. Siendo X = {x E lR'/ \X2 - 5x \ < 4} L! 15. Dado el sistema de ecuaciones:


4 S S
Y = {x E'llV\x2 -5x+6\::; 2} x +y - 1 2x - y + 3 -2:
Entonces, X í) Y es igual a
3 + 1 7
A) $ x+y-l 2x-y+3 S
el valor de x + y es igual a:
B) [1;4]

C) (-~;-1) u (4;00)
A) -1 B) ° C) 1 D) 2 E) 3

D)
ts\2 . J4i2' . ~+ J4i)
2 2
16. En relación al siguiente problema maximizar
Z = Xl + 1,5 X'2 sujeto a:
2x1 + 2x2 ::;160 Xl + 2x2 ::;120
E) (~-~ ; 1)U(4; ~+ ~) 4xI +2x2::; 280 Xl 2: 0,x2 2: O
Indique la secuencia correcta después de determi-
11 . El rango de la función J: ~ \ {O} ~ ~ defi- nar la veracidad (V) o falsedad (F) de las siguien-
nida por J(x) = x + 1es tes proposiciones.
x
I. No existe región admisible
A) ~ \ (-2,2) B) ~ \ [-2,2] C)~\(-l,l)
II. El óptimo es el punto (60 ; 20)
D)~\[-l,l] E) ~ \ {O} III. Una solución admisible es el punto (40 ; 40)
A) VW B) FFV C) VFV D) WF E) VFF
12. Determine el polinomio mónico de menor gra-
do de coeficientes enteros que tenga como raíces a
1 7. En un cuadrado de lado 4 se inscribe otro cua-
los números reales !2 - 3 Y .J3 - 2 . Dar como res-
drado uniendo los puntos medios de los lados de
puesta la suma de sus coeficientes, dicho cuadrado. Repetimos este proceso indefini-
A) 28 B)42 C) 56 D) 70 E) 84 damente. Entonces la suma de los perímetros de
todos los cuadrados así construidos será:
13. La suma de los cuadrados de dos números es
A) 64(2-!2) B) 48(2-!2) C) 32(1+!2)
29 y la suma de sus logaritmos (en base 10) es l.
Dichos números son: D) 16(2+!2) E) No se puede calcular
A) -2 Y S B) 4 Y S C) 2 Y -S
D) 2 Y S E) 3 Y 20 18. Las raíces de la ecuación X + ~x - 2 = 4 son:

A) Solo x = 6 B) Solo x = 3
14. Sea la matriz [: ~], donde a*- 0, b E n C) x = 3; x = 6 D) x = ~ ; x =3
E) No existen soluciones
Entonces los valores Xl' X2' x3' x4 tales que
19. Halle la intersección de los conjuntos
a
[b a
O] [Xl x2] = [10] son (en ese orden):
p ={XE ~/x2-2x+a'2: O}
x3 x4 01
y Q={XE~/x2-ax-2a2::;0},
A)-
1
a'
b
-- 2
a '
° 'a-1 1
D)-,0'--2'
a a
b 1
a donde 3/4::;a<1
1 b 1 1 b 1
B)-'2'0,- E)-, 0,2'- A) 0 B)[-a; l-~J
a a a a a a
1 b 1 C)(-oo; l-~J D)[l+~l-a; 00)
C)--, 2,0,--
a a a
E) [-a; l-~)u[l+~l-a ; 2a]
f.;"" . ;;;; ;;;;;

"1--------------------~
mi ':
260 . UN12007-11 MATEMÁTICA

20. Al simplificar: 25. En un triángulo ABC se trazan las cevianas


BP y BQ tal que AP = PQ = QC . Sobre los lados
Q = (am + an + bm - bn)2 + (am - an- bm - bn)2
AB y BC se ubican los puntos F y G respectiva-
(a4/3 _a2!3b2!3 +b4/3)R(m,ll)
mente tal que AF = 2FB Y BG = 2GC . Halle el
donde m; n E (O ; + ~) Y área de la región triangular determinada por
FG ,BP Y BQ si el área (tlABC) es 45 cm2 .
R(m,n) = (m-hmn + n)(hmn + m+ n)
A) 1 B) 2 C) 3 D) 4 E) 5
Entonces obtenemos:
A) 2(a + b) D) 2a2!3 _ 2b2/3
26. En un plano H, está contenido un ángulo BAC
B)2(a-b) E) a2/3 + b2/3 de 60°. Un punto Q que no pertenece al plano, dis-
C) 2a2/3 + 2b2/3 ta 25 cm del vértice A, 7 cm del lado AB y 20 cm
del lado AC . Determine la distancia, en' cm , del
21. En la figura: EF es la mediatriz de DC, punto Q al plano H.
AB / / DE Y AJ = 20 cm . Calcule BE (en cm )
G
A).[29 B) m C).J33 D)¡f35 E) m
A) 5
27. En un dodecaedro, en cada cara levantamos
B) 6 una pirámide; formándose un nuevo poliedro. Para
C) 7 este nuevo poliedro tenemos:
V' = número de vértices,
D) 8
A' = número de aristas,
A~--~~------~,-----~C E) la F = número de caras;
entonces V' - A' + F' es igual a:
22. En el triángulo ABC, recto en B, BD es A) 1 B) 2 C) 3 D) 4 E) 5
bisectriz interior. Si sabemos que BC = 6 Y
AB = 4 ,entonces la longitud BD es: 28. Halle el área lateral, en m2, de un tronco de
pirámide cuadrangular regular circunscrita a una es-
A) 2,[2 B) 152,[2 C) 3,[2 D) ~,[2 E) 4,[2 fera, siendo las áreas de las bases del tronco 9 y 36.
A) 78 B) 79 C) 80 D) 81 E) 82
23. En la figura mostrada: M, Ny P son puntos de
tangencia; O y O' centros de las circunferencias. 29. Al aumentar en 6 unidades el radio de un ci-
lindro circular recto, su volumen se aumenta en "x"
Si PM = 2PN , calcule ~
r unidades cúbicas. Si la altura del cilindro original
se aumenta en 6 unidades el volumen queda au-
A) 2
mentado igualmente en "x" unidades cúbicas, Si la
B) 3 altura original es 2 unidades entonces el radio ori-
C) 4 ginal es: (en unidades)

D) 5 A)4 B) 27t C) 6 D)61t E) 8

E) 6
30. En un tetraedro regular cuya arista mide
3,[6 u , está inscrito un cono de revolución (su base
24. Se tiene el triángulo ABC inscrito en una cir- está inscrita en una cara del tetraedro y su vértice
cunferencia, las proyecciones de los lados AB y es el vértice opuesto). Si un plano corta al cono
BC sobre el diámetro BF miden 6 m y 9 m res- paralelamente a su base tal que el volumen del cono
pectivamente. Calcule la altura en m relativa al pequeño que resulta es la octava parte del cono
lado AC. grande. Calcule el volumen del tronco de cono re-
A) ,[6 B) 2,[6 C) 3,[6 D) 4,[6 E) 5,[6 sultante (aproximadamente).
UN12007-11 MATEMÁTICA .Ift\\! 'Ij!
tG ,.z
35. Al resolver la ecuación
A)7,89lt(.(3 B) 7,87lt (.(3

D) 7,84lt (.(3 E) 7,82lt (.(3 cot( ~)+4wn(*) = 2cscx

determine cos (~)


31. Se obtiene un cono girando un triángulo
equilátero de lado .e. alrededor de una de sus altu-
ras. El volumen de la esfera circunscrita al cono es

36. Determinar el rango de la función:


J(x) = 3 arccosx + 6 arcsenx-lt

A)[~; 3;] B) [ 3 lt; 7 lt]


2 2 C) [-lt; 2lt]

32. En la figura, ABC es un triángulo, su


circunradio mide R = 6 m y su inradio r = 2 m . Cal-
D) [5; ; 7 lt]
2
E)[32lt; 5;]
cule x + y + Z en metros, si:
mAM = mMB; mBN = mNc y mAQ = mQC 37. Dada la funciónJ, definida por:
B
J(x-%)=coS2(X)+2COS(X)-2·
A) 12
calcule el rango de f:
B) 14
A) [-3;1] B) [-2;2] C) [ - 3;0]
C)l6
D) [-2;1] E) [-3;2]
D) 18
38. Simplifique:
E) 20
E = Isen( 9~lt + e)1 + Itan(7~lt + e)1 + Isec(3~rr + e)1
Q Si 8 = 330 0

33. Sea la hipérbola .xy = 2. Halle el área del A) 12 +.J3 B) 12 + 2.J3 C) 15 + 2.J3
triángulo que se forma con una recta tangente a 6 6 6
esta hipérbola, y los ejes coordenados.
D)15+3J3 E) 15 + 6J3
A) 2/2 B) 2,[3 C)4 D) 3,[2 E) 3,[3 6 6

34. De la siguiente figura: 39. Simplifique:

B R = cos( 7;lt + o: )+ sen(nlt + 0:) , n E N

A
LL
Determine el valor de J
8 D

= mcos2(0:)
3 C
A) -1 B) (_1)" C)O

40. La medida de un ángulo en los sistemas


sexagesimal y centesimal están representadas por
dos números impares consecutivos. Halle la medi-
da de dicho ángulo en radiartes.
D)l E)2

D)10
1 B) to C) ~
6
D) 2rr
3
E) 5rr
3
UN12007-11MATEMÁTICA

SOLUCIONARIO
1. El número, segun el enunciado, puede expresar- El 28 es el número que continúa a 27 y está como
se como: una de las posibles soluciones de P segun (1), en-
tonces:
N = p2 + r (I)
272<N<282
N = Q3 +r (II)
729<N<784
De (1) y (II), por propiedad, asumimos residuo por
defecto. La cantidad de números N es:

N = S6 + r
(784-729)-1=54

Cómo el número es de tres cifras, segun el enun- Clave: C


ciado, entonces:
2. Del enunciado se tiene la fracción:
100~N~999
La raíz-cuadrada de este número: N=.l=lx3=...:L
5 5x3 15
10 ~?:IN~ 31,6 Llevandolo a base 2 el dividendo:
~ p = 10;11;12; ... ;28;29;30;31 ... (1)
N = 1.~~2)
La raíz cúbica:
112
4,6 s TN ~ 9,9
(2 4
- 1)
~ Q = 5;6;7;8;9 ... (2)
0011(2)
La raíz sexta:
24 -1
2,15 ~ lfij ~3,16 Propiedad, esta última expresión:
~ S = 3 ; (única solución de S) 0011(2)
N= 4 =0,00110011..'(2)
De esta última solución en (1) y (2) restringimos: 2 -1
P=27 y Q=9 Clave: B
Se debe hallar la cantidad de números de 3 cifras
3. Del enunciado, el máximo común divisor de los
que tienen raíz cuadrada y cúbica con el mismo
números A y B:
residuo no nulo.
Analicemos: MCD(A,B) = 36
Para la raíz cúbica, donde Q = 9 : ~ A = 36a ... (*)

93; ;103 B ~ 36b


~
93+r Donde a y b son Pesi.
Pero 10 no se encuentra en las posibles soluciones Por condición del enunciado del problema:
de Q segun (2). 750 < A < 950
Para la raíz cuadrada, donde P = 27 : De (*): 750 < 36a < 960
272. . 282 20,8 < a < 26,38
''-.r---''
N
~ me;:===?::=::~:::i=l,-_U_N_I_2_0_0_7._I_1 I==r~=i=::;~:::~
_M_A_T_E_M_A_'___T_IC_A
=> a = 21; 22; 23; 24'; 25; 26 15
De(1): (le = (lo =:3 =S
= 3 x 7 ; 11 x 2 ; 23 ; 3 x 8.; 5 x 5 ; 2 x 13
Lo mismo se cumple para b por tanto se obtienen
los mismos valores,
De(2): bE = 1; = 3
Como a y b son Pesi: Para N = 30 en (3):

a b No. pares • cT = 12 5(30) = 4 ... (S)

21 22 ; 23 ; 25 ; 26 4
De(l): ae=ao=:f=10
30
22 23; 25 2
23 24; 25; 26 3
De (2): bE = 35° = 6
24 25 1
25 26 1 Para N = 45 en (3):
11

El número de pares de números es 11. De (1): ae = ~S = 1S > 13 corazones, labsurdol

Clave: C Finalmente, las posibles cantidades de tréboles ex-


traídos, de (4) Y (5):
4. Del enunciado, se tiene un juego de barajas de
52 cartas. cT = 2+4 = 6
Clave: C
13e (corazones)
5. Del enunciado:

lB
13E (espadas)
logbS31441 = N NEíZ
130 (diamantes)
13T (tréboles) =>bN =531441
--
52 cartas = 312

Se extrae N cartas, que por condición del enuncia- Los posibles valores de bN son:
do, la tercera parte son corazones y la quinta parte
son espadas. El número de diamantes extraídos es bN = 312; 96; 813; 274; 7292; 531441]
igual que el de corazones; es decir:
En el conjunto de posibles valores se observa que
N •
"3 = ae = ao ... (1) los exponentes son divisores de 12, entonces tam-
bién se puede calcular la cantidad de b enteros.
N
S =v« ... (2) 12=22X31
La cantidad de divisores de 12, por propiedad:
Entonces, las cartas extraídas:
ae+aO+bE+cT =N CD(12) =(2+1)(1+1)=6
De(l)y(2): !'!.+!'!.+!'!.+c =N Clave: C
3 3 5 T

=> cT = 125N ... (3) 6. Del enunciado:


16P61n , donde P = 11
Analicemos, de (1) y (2) se deduce que N debe ser
múltiplo de 3 y S: Si esta expresión representa el cuadrado de un nú-
mero.
ParaN = 15 en (3):
N2=16(1l)61
2
cT = 15(15) = 2 ... (4)
= 71
4
+ 6n3 + 11n2 + 671 + 1 ... (1)
Observe que el primer término del polinomio de 11. é.Que porcentaje de evaluación tiene notas me-
esta última expresión tiene exponente 4; luego el nores a 65?
exponente del primer término de la raíz cuadrada
Solución:
de este mismo número debe ser 2. Veámos este
número en su forma genérica: La nota 65 se encuentra en la fila 3, donde se
tiene registrado que nadie tiene esta nota, por
N = abc ... (2) lo tanto, sólo se toma en cuenta a los que se
encuentran en los rangos anteriores de las fi-
=) N = an2 +bn+c las 1 y 2:
2
N2=(an2+bn+c) fe = 0,25 + 0,25 = 0,50
=a2n4 +2abn3 +(b2 +2ac)n2 +2bcn+c2 ... (3) =) Pe = 50% ... (2)

Comparando (1) y (2), se deduce: III. Si hay en total 400 evaluados, é Cuántos
obtuvierón notas entre 90 y 100?
i) a2 =1 =) a =1 ... (4)
Solución:
ii) 2ab =6 El rango de 90 y 100 se encuentran en la sexta
fila, que tiene una frecuencia relativa de 0,25
iii) c2 = 1 =) c =1 ... (5)
(fracción de evaluados). Luego el número de
Reemplazando (4) en (ii): evaluados de los 400 en este intervalo es:
2(1)b = 6 =) b=3 ... (6) n90-10 = h6h
Reemplazando (4), (5) Y (6) en (1): =(0,25)(400) ... (3)
N = 131" = 100
Clave: D De (1),(2), y(3) se obtiene la respuesta:
0;50% ; 100
7. Del gráfico del enunciado se deduce la tabla:
Clave: D
Frecuencia
Intervalos Frecuencia
Fila relativa 8. La masa de la aleación (mezcla) de oro de 28
(Notas) acumulada (H,) relativa (h¡)
gramos.
1 [40 - 50) 0,25 0,25
m( aleació,,) = 28 9 ... c')
2 [50 - 60) 0,50 0,25
3 [60 -70) 0,50 0,00
Por teoría se sabe que al oro puro se le asigna un
valor de 24 kilates.
4 [70 - 80) 0,50 0,00
5 [80 - 90) 0,75 0,25
Según el enunciado, el oro de la aleación es de 18
kilates. La masa de la aleación esta compuesta oro
6 [90 - 100) 1,00 0,25
puro y otros metales, es decir, el oro puro en la
La frecuencia relativa nos da la fracción de los "' Ios con f orman 24
a Ieacion 18 ava.
alumnos evaluados en el examen.
18
Analizando las proposiciones: m(oro) = 24 maleacio"
I. ¿Cuántos de los evaluados tuvierón notas en-
De(*): = li(28 g)
tre 70 y 80? 24
Solución: = 21 9
Este rango de notas se encuentra en la fila 4, Clave: C
donde la frecuencia relativa es O, es decir, nin-
gún alumno ha tenido este rango de notas.
n(70-BO) = O ... (1)
UN12007-11 MATEMÁTICA ~
mI ili .' ~

9. Analizando las proposiciones: Para resolver esta inecuación con valor abso-
luto se presentan dos casos :
I.Si:aE(O;l)~-la =-11 E(O;oo+)
-a --1 Caso 1: x - 1 > O ~ x >1
a
Verdadero (V) En la inecuación (*):

Verificación: 2<x-1<3
Formando la desigualdad 3<x<4 ~xE(3;4)
Caso 2: x - 1 < O ~ x <1
Ocu cI ~ 1>1
a En la inecuación (*):
1-1> O
a -2> x -1>-3
_1_>0 -1>x>-2 ~ XE (-1;-2)
1-1
a Considerando los dos casos:
XE (-1;-2)v(3;4)
~_l_=_a_ E (0'+00)
1-1 1-a ' Clave: C
a
10. Dadas las funciones:
11. 1 s x < 2 ~ O S ~ 2 - x < .J2 . Falso (F)
2x 2 X = x E lR/lx2 - Sxl < 4 ... (1)
Verificación:
En la expresión radical: Y = X E lR/lx
2
- Sx + 61 ~ 2 ... (2)

Para hallar, primero verificaremos si esta intersec-


p;/ =~~-~ ción es nula, pues en las alternativas existe esta
solución. Esta verificación lo haremos usando un
Formando la desigualdad:
artificio:
1~x<2
En la función X de (1)
1> 1 > 1
-x 2 -4 < x2 - Sx < 4 ... (3)
1 1 1
2~x-2>0 La expresión de X que depende de los valores de x
se encuentra en: x E (-4; 4)
h > ~1_1
2 - x 2
>O
Luego obtendremos la expresión de la función Y,
partiendo de (3):
~ ~1_1
x 2
= ~2 -x
2x
E (O' '2
h] - 4 < x2 - Sx < 4
2
1 -4 + 6 < x2 - Sx + 6 < 4 + 6
X -
x+l1
111. - 2 < x < -1 ~ 4 < 2 I <8. Falso (F) 2 < x2 - Sx + 6 < 10
~
• Verificación: [

Calculando el dominio: La expresión de Y que depende los valores de x se


encuentran en el intervalo:
21
X - 1 IE(2;10)
4 < 2I x+l < 8
Pero por otro lado se tiene de (2) que:
IX-llx+11
22 < 2 Tx+lI < 23 -2 ~ ~ s2 ~ Ir E [-2; 2]
[1
22 < 2lx-ll < 23
Si hallamos la intersección de los intervalos de las
~2<lx-11<3 ... (*) expresiones I y II.
~~=====:i~:I
__ U_N_I_20_0_7_-I_1 _M_AT_E_M_A_·T_I_CA_ ••••• ;.:;;~~:==~~
[-2;2]n[2;10]=0 12. Si las raices de un polinomio son:
Lo cual no indica que no existen valores de x comu- xl = J2 - 3
nes a las funciones X e Y. x2 = .J3 - 2
Clave: A
Por el teorema de paridad del polinomio mónico se
cumple que también deben ser raices:
11. Datos:
f: lR/{O} ~ lR ... (1)
x3 = -./2-3
x4 = -,[3 - 2
f(x):x+1 ... (2)
x Luego el polinomio mónico de menor grado es:
Por teoría, la media aritmética MA es mayor o igual
a la media geométrica MG .
P(x) = [x-(./2 -3)][ x -(-./2 - 3)][x-
MA~MG
(,[3 - 2)J[x - (,[3 - 2)J
a+b>M ayb~O
= [x2 + 6x + 7][ x2 + 4x + 1 J
2 -
La suma de coeficientes:
=> a + b ~ 2M ... (3)
L,coeficientes = P(l)
Aplicando la relación (3) en la expresión de la
función f (x) de (2): =[1+6+7] [1+4+1]
= (14)x(6) (propiedad)
X+~~2~(X)(~); x>O(por(l)) =84
Clave: E
x+l~2 ... (4)
x
13. De las ecuaciones del enunciado:
=>f(x)~2
X2+y2=29 ... (1)
Verificamos si f(x) es par:
logx+logy=l ... (2)

f(x)=x+1 x c- O Operando en (1):


x
logx + logy = 1 x,y > O
=> f(-x) = (-x)+ (_Ix)
=> log(xy) = 1

= -(x+~) => xy = 10

De la expresión (2): => xy =2xS Ó xy = 1 x 10


Considerando (1) asumimos que:
f( -x) = -( x + ~ ) = - f( x)
x=2/\ y=S
Luego de la expresión (4): Clave: O
Sif(x)~2 V'x>O
14. Por teoría de matríz inversa.
=>f(x)~-2; V'x<O
AA-I =1
De donde podemos concluir que:
Donde:'
Ranf= (-00; -2]v[2; 00)
= lR\(-2; 2) SiA=(mP qn) =>A-I_1 (n- fAT -m
-q)P
Clave: A
:.e.:
•.·gl~ ; ;;
eg"mez
UN12007-11 MATEMÁTICA le::::::. =~ 267~

Del enunciado del problema: Multiplicamos por 5 a la ecuación (4), luego lo su-
mamos con (3) para obtener:

19 = _19
a 2
Además en la expresión siguiente debemos hallar
~ a =-2
los valores de: xl' X2' X3' X4
Reemplazando a = -2 en (1):

x+y-1=(-2)
~ x + y =-1
~ AX = 1 ... (3)
Comparando la expresión (3) con la (1) se deduce: Clave: A

X = A-1 16. Dada la función:


De (2): =-.L(
IAI
a
-b
O)
a
... (4) Z=x1 +1,sx2 ... (*)

y las inecuaciones:
El determinante de la matriz A:
2x1 + 2x2 ~ 160 ... (1)

IAI= I~ ~1=a(a)-b(0)=a2 ... (5) xl + 2x2


4x1 + 2x2 ~ 280
~ 120 ... (2)
... q)
Reemplazando (5) en (4):
xl ~ O , x2 ~ O ... (4)

X = a\( ~b ~) De la expresión (1):


xl+x2~80 ~ L1 :x2=-x1+80 ... (5)

~(:: ::)=[! -:] a2 2


De la expresión (2):

xl + 2x2 ~ 120 ~ L2 : x2 = -!x1 + 120 ... (6)

Luego, por comparación en esta última expresión: De la expresión (3):


Ox-b. 2x1 + x2 = 140 ~ x2 + 140
x 1 = '2'
l·x 2 = ; 3 - -2 ' x4
a
= a
1 L3 : = -2x1 ... (7)

Gráficando las inecuaciones:


Clave: O

15. Dado el sistema de ecuaciones:


455
x+y-12x-y+3 2
3 + 1 =_Z
x+y-12x-y+3 5
Si hacemos:
x + y - 1= a (1)

2x - y + 3 = b (2)
Luego el sistema de ecuaciones quedará así:
1_~ =_3 ... (3)
a b 2
l + 1=_Z ... (4)
a b 5
~268
~..:::
"':'lIt::,,:, ====":%1f""':=·I __ ~
. .~-
I '
NI 2007- I MATEMATICA
1=ywnww IW!&lW!&dWY4WW&' #'W?a!'fz
NWYwWí&"'w ""

El área sombreada representa la intersección de las El punto A = (40; 40) es justamente el punto
regiones que representa a las S inecuaciones, a la óptimo y pertenece a la región admisible.
cual se le conoce como la "región admisible". Clave: B
El punto óptimo es aquel donde los valores de las
coordenadas hacen que el valor de la función Z sea 17. Del enunciado:
máximo. Este punto en uno de las vertices del polí-
gono, el que se encuentra a la derecha y/o en la
parte superior.
Vamos a intersectar las rectas para hallar los pun-
tosA y B :

Intersección de L1 Í\ L2 ' de (1) y (2):


1
-xl + 80 = -2:xI + 120
~ xl = 40

En (1): x2 = -( 40) + 80
= 40

~A=(xA;x8)=(40;40)
Se debe hallar la suma de los prímetros de los cua-
Intersección de LI Í\ L3 de (1) y (3):
drados:
-xl + 8D = -2xI + 140
5 = 4(4)+4(z./2)+4(2)+4(,[2)+ ...
~ xl = 60
= 4(4+ 2,[2 + 2 +,[2 + )
En (1): x2=-(60)+80
= 20 = 4(4+ l2 + ~ + 2Í2 + )
~ B=(x8;Y8)=(60;20)
Reemplazando las coordenadas deA y B en (") para
= 16(1+ h + ~ + 21 + )
verificar cual es el punto que máximiza a Z: 5 1 1 1
~ 16 = 1 + ,[2 + 2: + 2,[2 + ... ... (*)
Para A = (40; 40)
5 1 1 1
Z = (40) + 1,S( 40) = 100 16 -1 = ,[2 + 2: + 2,[2 + ...

Para B = (60; 20) Multiplicando por,[2 a ambos mienbros de esta úl-


tima expresión:
Z = (60) + 1,S( 20) = 90
Luego en las proposiciones del enunciado: In( 5)
••2 16 -1
1
= 1 + J2 + 2: + 2J2
1 1
+ ...
l. "No existe región admisible". Falso (F)
La región admisible esta dada por la región De (*): J2( 156- 1) = 156
sombreada del gráfico.
~ 5=16(2+J2)
11. "El óptimo es el punto (60; 20) ". Falso (F)
El punto es A = ( 40; 40) porque es el que Clave: O
máximiza a Z.
III."Una solución admisible es el punto A = (40;40) ".
Verdadero' (V)
__ U_N_I 2_a_a_7_-II_M_A_T_E_M_Á_T_IC_A __ I=;~~~
18. Dada la ecuación: Entonces podemos afirmar que:
x+~x-2=4
P= (-oo;l-Jl-a]u[l+Jl-a ;+00)
Completando cuadrados en el primer mienbro:
x-2+~+l=2+l
4 4 ----'--+--+1 ---1 -,ce
O I I
(~)2 +;Ix - 2 + t t =
l-.Jl=O-
1
1+.J1=Ci
( ~+2
1 )2 =-;¡-9 Operando en el conjunto Q:
x2-ax-2a2$a
I~+tl=~ ... (*)
(x - 2a)(x -a) $ O

- Si:.Jx -.2 + 1> Por teoría de puntos críticos se deduce que:


2- a
Q = [-a; 2a]
De (*):
Analizando nuevamente la restricción (1):

%$ a < 1
~ x= 3
~ -1 < -a ~-~
- 4
-Si: ~+1<Q"
- _1
2 -1 + 1 < 1- a <
4+ 1
De ("): .Jx - 2 + 1=-~
2 2 O<l-a<l
-4
.Jx - 2 = -2
O<Jl-a<l
-2
~x=6
Clave: C l<l+Jl-a<~ ... (2)
-2

19. Se tiene los conjuntos definidos como: De la condición (1) también se tiene:

P ={XE 2
IR/x -2x+a ~ a} l<a
4
~ ~<2a
2
... (3)

y Q={xElR/x2-ax-2a2$a}
~ -a <_1 .... (4)
4
Donde: 3/4 $ a < 1 ... (1)
Considerando la expresión (3) en (2):
Operando en el conjunto P:
1+Jl-a<~<2a ... (5)
x2 a- 2x + a ~ -2
(x - 1)2 - (1 - a) ~ a Considerando (4) Y (5) podemos graficar:

((x -1) - J:l=Cl)((x -1) + J:l=Cl) ~ a

[x - (1+ J:l=Cl)][ x - (1- J:l=Cl)] ~ a


-2J~I------~I~----+I
-a 3
-4"
a ll1.
••

2 4
~I-+I-4I-+.~I
__ ~
a 1 11. 2
2a

Analizando la condición (1) para verificar la posi- 1- ..¡r:a. 1+{Hi


ción de estos "puntos críticos".
Finalmente PnQ :
l<a<l ~ O<l-a
4-
..J>--f-I --+-1 L-~
O<l-a<l -a 3
-4"
O
---+1
III
2f 4
a 1 11. 2
2a
O<Jl-a<l
O<l-Jl-a <l+Jl-a 1-~ 1+{Hi
~
~,
..2lli.i:.,.;:".,.fu'.~;·,:.:,;.;;.~'.:.,;=., ..;;~;.,.~.;,;:,:!.,.·~.,:m.::;,.t.· ..:.,.~.·.;,~
&L"" »,~"y,,' 'm~~··V,~_,·., "~~"~'. ,::;~;S;'~~~;$;;~~m;~;;~~;;~~::~~~;~~~~~:~;;~~~~~~~~;:~~;?
_M_A_T_E_M_A_·_T,;.,IC;.;A..;..._I"''''''''''''''''''''''''''''';x"",
..'.; ..,I,-_U_N_I_2_0_0_7-_11
..;....•,"·.:,.'~.':'.;:: ..:.;~.;.'.'.'".;:,'
,;
..,'w·.·,=« ••·,." ..·..,,·ww= .• .~:~
,-",-G=ie""Z-"
Reemplazando las expresiones reducidas en:
=} P!\Q=[-a;l-h-a]U[l+h-a ;2a]
Q=-.Jv!+N.
Clave: E (P)R(m,n)

20. De-la expresión del enunciado: = • 2(m 2 + 112)( a 2 + b 2 )

M N 2
(a2~~:~~/3 fm +1!2)
Q = (am + al! +'bm - bn)2 + (am -al1 ~ bl11- bl1/
= 2(a2/3 +b2/3)
(a4/3 _ a2/3b2/3 + b4/3) R(m; 11)
, P , Clave: C
Donde:
21 . Del enunciado se tiene el gráfico:
R(m,n)= (m-)2mn+I1)(hml1 +m+l1) ;
G

m,I1E (O; 00)'


Resolviendo por partes:

M = [am + un + bl11- bl'!]2

= [111(a+b)+I1(a-b)]2
= 1J12(a + b)2 + 112(a - b)2 + 2ml!(a +b)(a -b)
. 2
N = [alJ1 - al1 - b111- bn]
2 Donde: AB // DE Y se asume que GB = a
= [111(a-b)-I1(a+b)]
El triángulo GBl es ísósceles:
= 1112(a _b)2 +n2(a +b)2 -211111(a -b)(a +b)
~ lB = GB = a
Sumando las últimas expresiones de M y N: Ademá~en el triángulo rectángulo GDC, como:
M +N = m2[(a+b)2 +(a_b)2]+n2[(a _b)2 +(a+b)2] EF / /GD Y F punto medio de De, EF es base
media.
(a
= 21112 2 + b 2) + 2/12 (a + b 2) =} EC = GE = x +a ... (1)
= 2(1112+ /12)(a2 + b2) Luego el triángulo ABC es isósceles:
Aplicando cocientes notables para reducir P: AB = BC = BE + EC

P = a4/3 _ a2/3b2/3 + b4/3 De (1) y del gráfico:


20+a=x+(x+a)
_ (a2/3)3 + (b2/3 )3
=}x=10 ; (cm)
- a2/3 + b2/3 Clave: E
a2 + b2
22. Del enunciado:
B
Reduciendo R( m; 11)

R(m;l1) = [(m + Il) - h11111 ][(111+ 11)+ h11111]


6
= [(m + 11)2 - 211111J
=1112+112
AL-------~------------~C
~¿¡,I_II9.._ ,'~.'
"~n;tz-~'-"--"- =.=
.• ,.~.,,- .. _¡¡¡
w~ ?'~.,.:
_..,....•.~.•.•••¡¡.•<·.. ·.,%WI%
...• •..•··I
,..,:~~.<,,~
.::,<-'- UN12007-11 MATEMÁTICA rilfi'illmF' . ~~
-...II-1\~dtUIL.2W""~~

Por Pitágoras en el triángulo ABC:


En el trapecio MNOO' :
2 2
AC =4 +62

AC=2m ,.. (1)

Por el teorema de la bisectriz interior:


AB = BC
AD DC
~ 6 Por pitagoras en el triángulo rectángulo GHO
AD AC-AD
4 _ 6 (r'+ r)2 = (r'- r )2 + MN2
De (1):
AD - 2m-AD => MN = 2M .., (1)
=>AD=~m .., (2) En el gráfico 1:
Del gráfico se observa que: 2MQ = 2QP =MN

AC =AD +DC De (1): => MQ = QP = MN = JTr .. , (2)


=> DC = AC - AD 2

De (1) y (2): = 2m _1.[3


s
Por semejanza de triángulos:
llMQP - llNOP
=2m .. , (3)
s MQ _ NO
Luego, por relaciones métricas (bisectriz interior) MP - NP

De (1) y (2):
P¡: r
x2 = (AB)( BC) - (AD)( DC) 2Q a
De (2) y (3): = ( 4 )( 6 ) - ( ~J3)(~m) => ~
r
=4
288 Clave: C
2S
24. Del enunciado:
=> x = 12J2
s
Clave: B
Datos:
23. Del gráfico del enunciado: BM =6 m

M BN = 9 tn

BF: Diámetro

Del gráfico: BF = 2R .., (1)


Por relaciones métricas en triángulos rectángulos:
En el triángulo ABF:
En el triángulo MO'P:
2a+2~=1800 AB2 = BMxBF
=> a + r3 = 90 0 De (1): AB2 = 6x(2R) ,.. (2)
:.e.:
~~~~~~~ UN12007-11 MATEMÁTICA qll.~
GomcZ

En el triángulo CBF:

BC2 = BNxBF
De (1): BC 2 = 9 x (2R) ... (3)

Por el teorema del producto de los lados:

(/¡)(2R) = ABxBC

4R2/¡2 = AB ~BC 2

De (2) y (3): 4R2]¡2 = (6)(2R)x(9)(2R)


1¡2 = 54

~ h = 3J6 AL---=---~---II¡--~~~~--~C
Clave: C
En el triángulo ABQ:
25. Del enunciado: Como: N es punto medio de BQ
B P es punto medio de AQ
~ PN// AB//PR

De (1): BR = RG
BG = = 2b
2 2
~ BR = RG = b
En el triángulo BQG; el segmento NR es base me-
dia, entonces:
NR= QG
2
AL-------~---,ll--~~~~--~C
2a ~ NR= a ... (2)
2
Dato: En el triángulo PRC, el segmento QG es base me-
2 dia, entonces:
StJ.BCtJ. = 45 em
PR = 2QG
Del gráfico:
=2(2a) ~ PR=4a ... (3)
• AB=6a=l) Por semejanza de triángulos:
AF 4a 2
flFBM '" ó.NPM
~g = ~~~= ~
~
FB _ PN
FM - MN
... (l) .FB PR- NR
FM MN
• De (2) y(3): 2a =4a-a
FM MN
~ MN =1
FM 2
... (IJ) ~ MN _3_ (propiedad)
FM +MN 2+3
De las conclusiones (I) y (I!) se deduce que BGQF 3
es un paralelogramo. 5 ... (4)
Luego: En el gráfico:
5l1FBG _ (2a)(2b) Q
5.!.BCA - (6a )(3b)
=>
2
5l1FBG = 9511BCA
B7
M /

i(
De dato:

Del gráfico se observa que:


=

= 10 em2
45 em2)
... (5)
_
.. ......•. /
/
FN=NG ... (6) Se proyecta RH hasta AB , formándose el triángu-
En el triángulo FBG: lo retángulo AMR de 60 y 30°. 0

5M'BG _ 5 En el triángulo rectángulo AQR:


FG - MN
5l1FBG 5
De(6): 2FN = MN
En el triángulo rectángulo AQ5:
=> 5 - l(MN\::
-"2 FN [I1FI3G A5=~(2S)2_72=24 ... (2)

De(4)y(S): =1(~)10eI1l2 En el triángulo rectángulo AMR:

= 3 em2 sen300 = AR
AM
Clave: C
De (1): 1= 15 => AM=30 ... (3)
2 AB

26. Según el enunciado: Del gráfico:


Q
AM =AS+5M
De(2)y(3): 30=24+5M

Datos:
p
7 De(4): .L =
J3
=> 5M = 6
En el triángulo rectángulo HM5:

rg300 = H5
5M
Hfi
6
=> HS = 2J3
... (4)

... (5)

AQ = 25 el1l) Luego en el triángulo rectángulo HQS:


QS = 7 em ... (1)

QR = 20 eln x = ~(7/ - (H5)2

Los segmentos Q5 y QR deben ser perpendicula- De (5): = )49 _ (2J3)2


res a AB y AC, respectivamente, por ser distan-
cias. =m ; (em)
Para hallar la distancia del punto Q al plano, traza- Clave: E
mos QH , de.tal manera que se cumple el teorema
de las tres perpendiculares.
__ U_N_' 2_0_0_7 -_"_M_A_T_E_M_Á_T_'C_A __ I=- 0_","",,"
27. Teorema de Euler: En todo poliedro: Proyectando la figura sobre el plano 2Y:
C+V=A+2
C: número de caras del poliedro Del gráfico:
V: Número de vertices del poliedro 2a+ 28 = 180 0

A: número de aristas del poliedro. a+8 = 90°


En el problema, se forman un poliedro a partir del
dodecaedro, donde para este nuevo poliedro se
cumple:
V' = número de vértices;
A' = número de aristas;
F = número de caras.
Aplicando el teorema: Por semejanza de triángulos: óOTH '" ÓN'SO

F+ V' = A'+2 OT SN'


=> V' + F - A' = 2 TE' OS

Clave: B
lL 1. => R = '1.fi ... (3)
3 R 2
2:
28. Del enunciado:
Por Pitagoras en el triángulo MA'[-{:
B
2
/¡2 = (2R)2 + (~)
2 2
De (3): /¡2=(2x~fi ) +(~)
=> h=2 ... (4)
2
Finalmente calculamos el área lateral.

Q
SLarcral = 4( AB ~ MN Y
Datos:
De O), (2) Y (4):
= 4( 3 ~~)( ~)
2
SABCD = 9 m = 81 (m2 )
2
SMNPQ = 36 m
Clave: O
Como ABCD es un cuadrado, entonces:

SABCD =
-)2
(AB
29. Por teoría:

VCOIlO = ~TCr2/¡
De dato: 9 n¡2 = (AB /
=> AB=3m ... (1)
Donde: r : radio de la base.
h : altura del cono.
El cuadrilátero MNPQ también es un cuadrado
-2 Dato: h = 2
SMNPQ =MN
Por condición del problema, si aumentamos el ra-
De dato: 36 m2 = MN2 dio en 6 unidades:
=> MN = 6 m ... (2)
1 2
... (1)
"3TC(r + 6) h = Vcono + X
Ti ·=r UN12007-11 MATEMÁTICA
llt ~. ~~1ii!lIm
G lteZ

Por segunda condición, si aumentamos la altura en Del Gráfico:


6 unidades:
OM = tg300
MC
~11:r2(h + 6) = Vcono + X ... (2)
~=...l.. ~ R=l..fi .,. (1)
Igualando (1)y (2):
1.,[6,f3 2
2
~11:(" + 6 )2 h = ~11:r2(h + 6) Además se tiene:

(,. + 6)2 h = r2 (h + 6) OC = c5c300


OM
De dato: (r+6)2(2)=r2(2+6) De(1): OC =2 ~ OC = 3.fi
l..fi
~ (r+6)2 =,.2(4) 2
~ r+6=2r Pero el triángulo DOC es isósceles, entonces:
~ r=6 DO = OC = 3J2 ... (2)
Clave: C En el tetraedro que se muestra en el primer gráfico:
2 2 2
AD = D0 + A0
30. Del enunciado:
De(2) : (3,[6)2 = (3.fi)2 + A02

~ AO=6 ... (3)

El volúrnen del tronco de cono resultante:


," V M'N'-MN = V A-MN - V A-M'N'
"'N"~"" ~
,~r,.•.
..•...•."
De dato: = VA-MN -iVA-MN

... »>" i O' ,\' -,


A
----~--/--
\ l'
= ~VA-MN
~-'M' -
, ,' = ~[~rrR2(AO)J

De (l) y (3): = ~[~11:(~.fit(6)]


B

1 = 7,8711: (u3)
Datos: VA-M'N' = SV A-MN
Clave: B
D - ABC :tetraedro 31. Del enunciado:
En el triángulo BCD: Cono generado
D

Cono inscrito
en la esfera
I.;;2...00...7_-I•..1 •..M•..A
~~;~~~~~:-IL._U_N ... ...T.;;E...M ...Á•..TI;.;;C...A~I
Graficando la proyección frontal: Teorema de Steiner:
P ... e)

Analizando para el lado izquierdo (13A)


B

Aplicando ley de cosenos:

PB = JR2 + R2 - 2(R)(R)eos(120)

e =J2R2 - 2R2( -~) Del gráfico: MI = MA ... (1)


Por teoría:
jI = ~3R2
EA es bisectriz de P AB y
e = RJ3 lA es bisectriz de BAC .
~ R = J3
e ... (*) ~ <.EAI = 90°

El volumen de la esfera circunscrita: Por lo tanto de (1):

- 4 3 MA = MI = EM ... (2)
V
esfera - "3rrR Tomando en cuenta (2) se deduce:
De ("):
= 1rr(

4rr
!i3'
3
r Analogamente:
x = Re + r
2
... (3)

=--f
9J3 . y=_II-
R +r
... (4)
Clave: B 2

Rf3 +r
32. Del enunciado: z=--- ... (S)
2
B Calculamos la suma de (3), (4) Y (S):

Re+ r RA + r RIl +r
x+ y+z = --2-+--2-+-2

_ (Re + RA + RJ3 ) + 3r
- 2
(4R+r)+3r
De (',) :
2
= 2( R + r)
Datos: = 2( 6 + 2)
Q = 16 In

Datos: R = 6 m r = 2 m Clave: C

"
~~z~,~~=:.~~:.=%:&JIZ:I=¡~.
;:]i7:.;I __ U_N_1 2_0_0_7_-1_1 _M_A_T_E_M_A_' T_I_C_A ':;;:":,mwwP;:I,:;=.0
33_ Gráficarnos la hipérbola xy = 2 El área del triángulo sombreado:
S - ab
l!. - 2

De (5): =-ª2
=4 (u2)

Clave: C

34. En la figura del enunciaclo:


B

Del gráfico: P E (xy = 2) Y P E L ~


A 8 D 3 C
En la ecuación de la hipérbola: Por relaciones metricas:
xy = 2 ~ Y = l ... (1)
(AB2)(DC)+
2
(BC )(AD) = (BD
2
)(AC)+ (AD)(DC)(AC)
x
La ecuación de la recta:
(1112 )(8) + (1112 )(3) = (S2 )(11) + (8)(3)(11)
L: y = I11X+ /¡ ... (2)
11m2=S39
El punto (O; a) pertenece a la recta L, entonces en (2):
=> ,,/ = 49
a = 111(0)+ It ~ a = /¡ .,. (3) => m=?
Por ley de cosenos en el triángulo ABD.
Similarmente el punto (b, O) , en (2):
AD2= AB2+ BD2_2(AB)(BD)cosa
0= 111(b)+(h)
82 = 1112 + 52 - 2(m)(S)cosa
De (3): O=m(b)+(a)~I11=-% ... (4)
De (l): 64 = 72 + 25 - 2(7)(S)cosa
Reemplazando (3) y (4) en (2):

L .. Y = --ª-xb + a ~ cosa = ~
... (2)

Finalmente cá1culamos la expresión:


Para hallar el punto P igualamos (1) y (2):

±=
x
--ª-xb + a J = 111cos2(a)

~ ax2 - abx + 2b = O
De (1) y (2): = (7)(~ t
_ 1
Como sólo debe haber una solución, entonces tJ. = O -7
(ab/-4(a)(2b)=0 Clave: B
a2b2-8ab=0
(ab )( ab - 8) = O

~ ab = O Incorrecto

ab = 8 Correcto ... (5)


35. En la siguiente ecuación: Entonces graficamos:
cot(x/2) + 4tan(x /4) = 2c.scx ... (*)

Para calcular cos( x / 2) 1~


N
Identidades trigonométricas: ti
• sen2a = 2sena cosa ... (1) Finalmente, apoyándonos en el gráfico. calculamos:
• cos2a cos2a - sen2a ... (2)
=
cos( ~) = cos
2
( *) - sen2( ~)
• tan2a =

• sen2a + cos2a
2tana
1 - tal12a

En la ecuación (*):
=1
... (3)

... (4) De(2):


cos ( ~) =( ~

=3
r-(13 r
Clave: B
cot(~)+4tal1(*) = se;x
36. Por teoría:
cos(L) arccosx+arcsenx = ~ , XE [-1; 11
De (1): __ 2_ + 4tan(L) = 2
sen(~). 4 2sen(~) cos(~) En la función del enunciado:
f(x) = 3arccosx + 6arcsenx-11
= 3(arc cosx + are senx) + 3arc senx - 11

= 3(~)+ 3are senX-11


= 3arcsenx+~
Por teoría:

- 1I < arescnx < 1I


2- -2
_ 311 < 3arescnx < 3rr
2 - - 2
_ 311 + 1I < 3arcscnx + 1I < 111+ 1I
2 2- 2-2 2
~ -11::; f(x)::; 2rr

~ Ran(f) = [-1t ; 211 1


Clave: C

37. Dada la función:

f( x - ~ ) = cos2 (x) + 2cos( x) - 2

= cos2 (x) + 2cos( x) + 1 - 3


=[cos(x)+1]2-3 ... (1)
Haciendo:

y=x-lI ~ x=y+lI ... (2)


2 2
UN12007-11 MATEMÁTICA

Reemplazando

J( y)
(2) en (1):

= [ cos ( y + ~ ) + 1

=[-sen(y)+lf-3
r- 3
Para 11= 2:

R = cos( 7~lt + a )+ sCIl(2lt + « )

Hallando el rango de la función:


= cos( 4~lt + a) + scn(ex)
- 1 ~ SCIl(y) ~ 1 = cos( i + a) + sena
=> 1~-scn(y) ~-1
= -sen( a) + sell( ex)
=> 2 ~ 1- scn(y) ~ O
=0
0~[1-sen(Y)f~4
Para n = 3:
0-3~[1-sen(Y)f-3~4-3
=> -3~f(x)~1 R = cos( 7~ lt + a )+ sCIl(3rr + a)

=> Ran(J) = [-3; 1J = cos(3~3rr + ex)- sell( ex)


Clave: A
= cos(3 rr+
2
a) - sena
38. En la expresión:
= scn(a)+scn(rx)
E = ISCI1(9~rr
+ 9)1+ r(l1f~rr + 9~ + ~ec(3~rr + B)I =0

= ~en(45rr + 9)1 + ran(23rr +} + 9)1 +~ec(l 5rr+ ~ + B~


Para n = Il :
. =scn(rr+B)+tan(rr+}+e)+sec(rr+~+e) R=O
Pero por dato del problema e = 330 0
, entonces:
Clave: e
E = ¡scn(180° + 3300~ + ¡tun(180° + 600 + 3300~ + 40. Si C y S representan a un mismo ángulo en el
sistema centesirnal y sexagesimal respectivamen-
¡sec(180° + 900 + 3300~
te, además estos están representados por dos nú-
= ¡sen(l 500~ + ¡tall(21 Oo~ + ¡scc(2400~ meros pares consecutivos, es decir:

=1~H~Hfl C-S=2 ... (1)


Por teoría de "Conversión entre sistemas angulares".
=1+/3+2 s C B. ... (2)
2 3
180 200 rr
15 + 2/3
--6- Operando en (2):
Clave: C S C => C =lQs ... (3)
180 200 9
39. De la expresión del enunciado: Reemplazando (3) en (2):

R = COs( 7; lt + a )+ sen(nlt + a) ; Il = 1 ; 2; 3 ;... 10S_S=2=> S = 18 ... (4)


9
Operando en (2):
Para n = 1 :
S R
R = cosC2lt + a) + sell(n + a) 180 lt
1lL R => R = rr/10
= scn(a) sCIl(a) De(4): no.d.
180 rr
=0 Clave: B
UNI2008 - I MATEMÁTICA

~~~~(;J~~
MATEMÁTICA

1. Dados tres conjuntos A, By C, tales que 5. Si se cumple:

(AuB)c(AuC)y(AnB)c(AnC) y ~= az =2= k
b1 bz b3 '
AcxC
donde k es un entero positivo, y que
entonces:
al a~ - 2~
A) B e C B) B = C C) C e B -"-'----=6
b¡ bi-b~
D)(AuC)cB E)(AuB)cC
entonces el valor de k es:
A) 1 B) ') C) 3 D) 4 E) 5
2. La función polinomial
F(x,y,z) = [Cx - y)(y - z + 3)f + 6. Para cubrir el puesto de mecánico-electricista
se recibieron solicitudes de 200 postulantes. En el
+ [( z - y )( y - x + 3) J4 + (x + Y + z - 3 )z cuadro siguiente se presenta la distribución de los
postulantes según experiencia laboral en el área.
tiene N raíces (x, y, z) . Entonces N es igual a: Experiencia Porcentaje
A) O B) 1 C) 2 D) 3 E)4 laboral (años) acumulado
[5 - 7) 8%
[7 - 9) 18%
3. Calcule Q(A), si Q(x) = (1 + x)(l- x) siendo [9 - 11) 34%
[11 - 13) 65%

A=(~ ~) [13 - 1S) 100%

Entonces la experiencia laboral mínima para el

A) (~ ~) B) e ~) 90% de los postulantes es:


A) 7,4 años
D) 12,4 años
B) 8,4 años
E) 14,4 años
C) 10,4 años

D) -{~ ~) E) -14( ~ ~)
7. Dada la promulgación de una ley que fija un
impuesto para las ganancias por los ahorros ban-
4. La suma de tres términos consecutivos de una carios, se aplicó una encuesta de opinión a 600 ciu-
progresión geométrica es 13. Sabiendo que si los dadanos, obteniéndose los siguientes resultados.
dos primeros términos se incrementan en dos uni-
dades y se disminuye en la misma cantidad al ter- Opinión respecto a la ley
Partido Total
cero, los números forman una progresión aritméti- A favor En contra Neutra
ca. Determine la razón de la progresión geométrica A 120 60 20 200
decreciente. B 48 42 30 120
A) 1/3 B) 1/2 C) 2/3 D) 2 E) 3 Otro 126 112 42 280
Total 294 214 92 600
~
. ....;..¡¡;
~-
~-.;.,
UNI200B -1 MATEMÁTICA

Calcule la probabilidad de que un ciudadano sea 14. Sea la ecuación 4x2 - 2x + 3 = 0, cuyas raí-
del partido B o no opine a favor. ces son a y b. Halle otra ecuación cuadrática que
A) 0,507 B) 0,510 C) 0,590 tenga por raíces (2a - 1) Y (2b - 1) .
D) 0,600 E) 0,710
A) y2 - Y +1= ° D) y2 _ 1y
2
- 2 = °
8. Si se cumple que: y'Y(II) + 101,01(2) = B,A(16) Y B) y2 _ Y -.2 = O E) y2 _1 y + 3
4
= °
xx,x = 21,5(16) C) y2 + Y + 3 = O

halle x + y+n 2
15. Dada la función f( x) = 5x - 7x - 6 defi-
A) 13 B) l4 C) 15 D) 16 E) 17 x + 3/5 '
nida sobre ( -~ ; ~ ] . Halle el rango de Ifl:
9. En una reunión de profesionales hay 131 perso-
nas, la mayor parte son varones. Si la octava parte
de los varones son ingenieros y la séptima parte de
A)( -V; -;] B) [ -V; -~)
las mujeres son economistas, écuáncos varones no D) [7; 13) E) (7; 13]
son ingenieros?
A) 12 B) 21 C) 30 D) 84 E) 96
16. Halle el valor numérico de:
N2 N3
10. Si tiene 63 divisores y tiene 130 di-
p =( 11-3 + m-3 )-l
visores écuéntos divisores tiene N4? Calcule la m-311-3
suma de las cifras de esta cantidad.
A)4 B) 5 C) 6 D) 7 E) 8 Si m + 11= m ; mn 2m =

11 . Halle el número de elementos de la clase de A) -24 B)-12 C)-~


24
equivalencia de 7/11 , de modo que el numerador
tenga 3 cifras y el denominador 4. 1
D) 24
A) 50 B 51 C) 52 D) 53 E) 54

17. Sean A y B matrices de orden 2 x 2. Señale la


12. Se da un número positivo que no tiene raíz
secuencia correcta, después de determinar si la pro-
cúbica exacta. Si a este número se le disminuye en
posición es verdadera (V) o falsa (F):
721, entonces su raíz cúbica disminuye en una uni-
dad, pero el residuo no se altera. Determine la suma 1. Si A
2
= ° => A = °
de las cifras de la diferencia entre el número y el
residuo.
II. Si AB = °
=> A = ° ó B = °
III: (A+B)(A-B)=A2_B2
A) 16 B)17 C)l8 D)19 E)20
A)VVV B)VVF C)FFV
D) F F F E) FVV
13. Si {x1,x2} es el conjunto solución de
18. Un grupo de estudiantes decide aportar en
31x+11 -13x -11 = 3x + 2
cantidades iguales para contratar un profesor de
entonces la suma de xl y x2 es: Física. Si hubieran 10 estudiantes más, cada uno
pagaría S/.10 menos. Sin embargo, si el número
A) -4 B) -2
C) ° D) 2 E)4 de estudiantes fuera 2 menos, cada uno pagaría
S/.5 más. Cuántos estudiantes forman el grupo
é

y cuánto se le paga al profesor?


t8>=~:===,-__ U_N_I_2_0_0_8_-_I_M_A_:r_E_M_Á_:r_IC_A_....I~::W~~:===~~~;:

A) 20 ; SI. 120 D) S ; SI. 200 21. En un triángulo ABC se traza la media-


B) 10 ; SI. 200 E) 20 ; S/.200 naBR; tal que AB=AR , mLRBC = 14°.
C) S ; SI. 160 Halle LBAC
A) 104 0
B) 1050 C) 106 0
D) 107 0
E) 10So
19. Al maximizar: x + y ; x ,y E lR sujeto a las si-
guientes condiciones:
22. Sean a,b,e las longitudes de los lados de un
2x + 3y ~ 6 X~O triángulo. Supongamos que: .
2x+ y:S;6 y~O 1) a < b < e ,y
y:S;4 2) a, b, e forman una progresión aritmética. Deno-
tando por r el radio de la circunferencia inscrita
Indique la alternativa correcta después de deter-
y por R el radio de la circunferencia circunscrita
minar si la proposición es verdadera (V) o falsa (F):
al triángulo, entonces el valor de rR es:
1. Los puntos (2;2) y (4; 1) pertenecen a la re-
A) -ª...."t.f B) a + b C) Q..~
gión admisible.
6 6 6
lI. La región admisible es un polígono de cuatro
D) a e be
lados. 6 E) "6"
IlI. El valor óptimo es 5.
23'. Dados dos polígonos regulares convexos, cu-
A) V V F B) V V V C) V F V
yos números de diagonales se diferencian en 4 y
D) FVV E) FVF cuya medida de sus ángulos centrales están en la
relación 5: 6. Determine la difci encia entre la me-
20. Sean a y b números reales. Si se cump dida del ángulo interior del polígono regular con-
quexlI+1 = a XII +b, n=0;1;2; ... vexo que tiene menor número de lados y la medida
entonces: del ángul« exterior del polígono de mayor número
de lados.
A) XII = n( Xo + b), si a =1 Y

XII =allxo+(\-_a~1 l' si (/ ",,1


2·4-. En un triángulo rectángulo AB C recto en B
se traza la bisectriz interior BD . Por D se levanta
B) XII = Xo + nb, si a =1 Y una perpendicular al segmento AC que intersecta

XII = a"xo + (\-_a~1}, a si ""_1


a BC enM.
Si AD = 30 cm y DC = 40 cm • entonces la medida
del perímetro del triángulo BMD es:
C) XII = nxo + b", si a = 1 Y
A) 30 + 24.J2 D) 35 + 24.J2
XII = (1 - n )xo + a"b, si a *1 B) 32 + 24.J2 E) 36 + 24.J2
D) XII = x~ + nb, si a = 1 Y C) 34 + 24.J2

XII = axo +(\++ a~1}, si a * 1 25. En un tronco de pirámide cuadrangular regu-


lar, las aristas básicas son 2 cm y 6 cm • el apotema
E) XII = (1 - n )xo - nb, si a = 1 Y del tronco mide 4 cm . Calcule el volumen del tron-
co (en cm3).
XII = (l-a)xo +( \~a~1}, a "" si 1
A)S2J3 B) 7SJ3 C) 104.Jl
3 3 3
D) 130J3 E) lS6J3
3 3
UNI2008·'¡ MATEMÁTICA I~~
26. En la figura: eB = fi, O centro de la circun- 29. La figura representa un prisma exagonal re-
ferencia, la razón de r y BA es de 2 a 3, gular de arista a y altura ,f8a,Entonces el ángulo
Si AT es segmento áureo de AB , Determine AT , 8 de la figura mide:

A
A)arecos(l~a) D) are cosn~)
A) ~(/5 -1) B) ~(rs-1)
1
B) are COS(2 ia) E) are eos (181)
D) *(/5 -1) E) ~(/5 -1)
e) are cos( ~ )
27. El volumen que genera un cubo de arista a,
cuando gira 360 alrededor de una de sus aristas es
0

30. Determine la medida del ángulo obtuso que


forman las asíntotas de la hipérbola

x2 - 3y2 - 8x -18y = 14
28. En la figura, AD es el diámetro de la circun- 1t
A)~ e) t8 D) 23 E) ~
6 5
ferencia de centro O, por A se traza la recta tangen-
te L que contiene a los puntos B, e y F,
31 . Dada la función f , definida por:
Si AB = 11r Be = ~r AE = OB Y oc/ I EF,
5' 5 '
J(x) = ~a,.esce(x)-a,.eese(x) +areSCIl(-x_)
Determine AF x2 +1
lAD
Determine el dominio de la función
donde lAiJ: longitud del arco AD A) IR
eónsidere: 1t = 3,14 B) (-00;-1] v [1;+ 00)
L
e) (-00; - 1] v [ Ji, + 00)
~F D) (h.;+ 00)
E) (-00;-1] v [2;+ 00)

A)1 B) ª-2 e)2 D)~


2
E)3
~X:~®:=;;=:::IL,,
__ I==~=:;:V::=:;3rme;:
U_N_I_2_0_0_8_-_I_M_A_j_E_M_Á_T_IC_A
__

32. Los números que representan la medida de 37. En la figura mostrada, calcule RA en cm si,
un ángulo en los sistemas sexagesimal y centesimal BI=acm, IR=bcm (b<a).
son x100 y x100 + 1 respectivamente. Halle el va- B
lor del complemento del ángulo, expresado en
radianes.
.•• .......•.. ,
A) 7rr B) 8rr C) 9rr D)10rr E) 11rr ,,
20 20 20 22 23
,/" ,
\
\

33. De la figura AOB, COD y EOF son sectores ,


\
\

circulares. Si lAB
= 36 u y el área de la región o
EOF esS, deCOD es 3S ydeAOBes 6S ,calcu-
le lCD. 2b2 C)L
B) a _ b a-b
lEF
A 2
E)_a_
a-b

38. Un poliedro convexo tiene como caras 12


o 'triángulos, 16 cuadriláteros, 24 pentágonos y 13
exágonos. Halle su número de vértices.

A) 84 B) 85 C) 86 D) 87 E) 88
B

B) 2h C)../3 D) 2../3 E) 16 39. En un prisma triangular regular, la arista de la


. ~.
base mide x unidades y la altura mide. ~ x
34. Calcule el ángulo e que hacen las rectas v2 - "3
unidades. Si e es el ángulo formado por las
diagonales de dos caras laterales que parten del
mismo vértice, entonces el valor de e es:

A) are tan( ~) D) are tan( ~) A) 15° B) 30° C) 45° D) 60° E) 75°

B) are tan( *) E) are tan(*) 40. Se inscribe una esfera en un cono de revolu-
ción. Sabiendo que en el cono, dos generatriccs
C) are tan(l) opuestas determinan un ángulo de 60° y el diáme-
tro de su base es 18 unidades. Calcule el volumen
35. ¿En cuántos puntos del intervalo [-rr, rr 1 ,las de la esfera (en unidades cúbicas).
funciones eosx y eos3x toman el mismo valor?
A) 108rr../3 B) 324rr C) 324rr../3
A) 2 B) 3 C)4 D) 5 E) 6
D) 972rr E) 972rr../3
36. Sean a,~, y los ángulos internos de un trián-
gulo, tal que (tana)(tan~)(tany) = 2006. En-
tonces podemos afirmar que el valor de
1 + tana + tan~ + tany es:
A) 2006 B) 2007 C) 2008
D) 2009 E) 2010
UNI 2008 -1 MATEMÁTICA

SOLUOIONARIO
1 _ Dadas las siguientes afirmaciones:
a(b=O)+(c=O)d+(e=O)=O (IIl)
(AuB)c(AuC) ... (1)

(AnB)c(AnC) ... (2) a( b = O) + c( d = O) + (e = O) = O (IV)

A<zC ... (3) Para el caso en:


De la expresión (1), como: x-y=O /\ z-y=O /\ x+y+z-3=O
~=y /\ z=.>; /\ x+y+z=3
x;y;z
(AuB)c(AuC)
~x=l ; y=l ; z=l
~BcC

Clave: A La raíz: (x; y; z)l = (1; 1; 1)

2. En la función polinomal, si: Para el caso (11):

F(x;y;z)=[(x- y)(Y-z+3)J2. + x-y=O /\ y-x+3=O /\ x+y+z-3=O


p x = y /\ Y - x = -3 /\ x + y + z = 3
[(z- y)(y-x+3t +[x+ y+z_3]2 =0 .. C*) (x)-x=-3
~
Q R absurdu

Se deduce que:
Para el caso (III):
• P?O; Q?O Q?O
y-z+3=O /\ z-y=O /\ x+y+z-3=O
• P+Q+R=O y - z = -3 /\ z =y /\ X +Y +z = 3
Entonces se concluye que: y-(y) =-3
'-----.r----'
P=O ; Q=O ; R=O absllrdo
Luego en (*) se tiene:
Para el caso (IV):
P = [(x- y)(y- Z+ 3)J2 = O ... (1)
~~ y-z+3=0 1\ y-x+3=0 1\ x+y+z-3=0
a b y - z = -3 1\ Y - x = -~ 1\ X +Y +z = 3

Q = [(x - y)(y - x - 3)J2 = O ... (2) Z;x 2x+y=3 1\ y-x =-3


~~ x;2; y=-l; z=2
e el

R = (x + y + z - 3)2 =O ... (3) La raíz: (x; y; z)2 = (2; -1; 2)


~
e
Conclusión final: el polinomio F( x; y; z) tiene 2
Entonces para que la función polinomial
raices.
F(x;y;z)=O •
Clave: C
y considerando (1), (2) Y (3), debe cumplirse los
siguientes casos:

(a=O)b+(c=O)d+(e=O)=O (1)

(a=O)b+c(d=O)+(e=O)=O (II)
z
3. Dada la matriz De (4)y(6):
Si: r = 3 = 1
A=(~ ~) Si: r = 1
:=}

:=}
II

11= 9
3
Si Q(x)=(l+x)(l-x)
Como se debe hallar razón de la progresión
:=} Q(A) = (I + A)(I - A) geométrica decreciente, entonces, r < 1 , entonces:

= [(~ ~)+(~n][(~ ~)-U ~)] r=l3

= (~ ~r
= [2(0) + 2(-2)
~2 -0 )
2

2(-2) + 2(0)J
5. Del enunciado se tiene:

al = a2 = a3 = k
Clave: A

... (1)
al a2 a3
2(0) + 2(-2) 2(-2) + 2(0)
al a~ - a~
= (-4 -4)
-4 -4
-+---=6
al . b~ - b~
... (2)

De la expresión (1):
= -4(~ ~) al = blk ; a2 = b2k ; a3 = b3k
Luego reemplazando estos valores en (2):
Clave: D
2 2
b] k + (b2k) (b3k) = 6
4. Se tiene la progresión geométrica: b¡ b~ - b~
n;nr;nr2 ... (1)
Del enunciado, si a los dos primeros términos se le k + k2( b~ - b~ J=6
b~ - b~
incrementa 2 unidades, y se le disminuye 2 al
tecero, entonces se obtiene la siguiente progresión k + k2(1) = 6
aritmética:
:=} k2 +k - 6 =O
n + 2 ; nr + 2; nr2 - 2 ... (2)
(k+3)(k-2)=0
además por condición del problema, de (1):
:=} k = -3 /\ k = 2
n+nr+nr2=13 ... (4)
Por condición del problema, k es un entero positi-
Además por propiedad de progresión aritmética, vo, entonces se asume: k = 2
de (2):
Clave: B
(nr + 2) - (n + 2) = (n r2 - 2) - (nr + 2)
6. En el cuadro:
nr - n = nr2 - nr - 4
Frecuencia
Experiencia Porcentaje
2nr + 4 = nr2 + 11 ... (5) relativa
:=}
laboral (años) acumulado (H¡x 100%)
Reemplazando(S) en(4):
[5 - 7) 8% 8%
n + nr2 + nr = 13 [7 - 9) 18% 10%
(2nr + 4) + nr = 13 [9 - 11) 34% 16%
:=} 11r=3 ... (6) [11 - 13) 65% 31%
[13 - 15) 100% 35%
Construimos el gráfico: La probabilidad de que un ciudadano sea del par-
tido u y 110 opine a favor (en contra o neutral) .

... (3)
99:'/0 25% 10%
35 -------- /~--------------------------'--------------------'r==P'
31--------------------------------------
I
I
Luego, la probabilidad de que un ciudadano "sea
I
I
del partido B o no opine a favor".
I
I
I
I
P = P(B) + P(C;N) - P(B;C;N)
16---------------------------~ I
I 1 51 3
I De (1), (2), Y (3): =5+100-25
10--------~
I
I
I
8 -------- I
I
= 0,590
I

5 7 9 11 13 115 Clave: C
~2-a¡a~
I 8. Dadas las expresiones:
x=lS-a
y"yell) + 101,ól(2) = 8,,1(6) ... (1)
90% = 8%+ 10%+ 31%+ 25%
Ejecutamos interpolación para hallar el valor de x: xx,x = 21,5(16) ... (2)

25% 10% Operando en (1):


2 -a a y 01 A
~ a = 0,57 Y + 11- 1 + 10 1(2)+ 22 _ 1 = 8 + 16 _ 1
Entonces: Y 1 A
Y+I1_1+S+3=8+15
x = 15 - a
=15-0,57 Y(l+ _1_)
11-1
= 3 _1 + A..
3 15
= 14,4 ; (uños)
Y (_il_)
11-1
= 40 + A
15
... (3)
Clave: E
De la expresión (1), por propiedad se deduce que:
y < 11 ; A < 16
7. Del cuadro de encuestas
Operando en (3), donde al número A se le debe ir
Opinión respecto a la ley
Partido Total dando valores desde 1 al 15; pero por ahorro de
A favor En contra Neutra tiempo sólo se ha tratado de presentar los núme-
A 120 60 20 200 ros que nos generan una respuesta coherente para
48 42 30 120 la solución.
B
Otro 126 112 42 280 A=15: (_11_)= 40+(15)=11
Total 294 214 92 600 Y 11-1 15 3

La probabilidad de que un ciudadano sea del parti- A = 14 : (_11_) = 40 + (14) = 3 x 6


Y 11-1 15 5
doB:
~1l-1=5~11=6
120 1 ... (1)
P(B) =_600 = 5 ~ y=3
La probabilidad de que un ciudadano no opine a
A = 10: (_11_) = 40+(10) = 10 = 3x10
favor, es decir, que este "en contra" o "neutral". y 11-1 15 3 9
214 + 92 51 ... (2) ~ 11-1=9 ~ 11=10
P(C;N) = 600 = 100
~ y=3
~=:::=:I__ I:==:~~:::::;:!:!~=:::~\1;:
U_N_I_2_0_0_8_-_I_M_A_T_E_M_A_'_T_IC_A
__ lile::;
10. Se tiene el número N, que lo expresamos en
n )_40+(S)_3_2X3
A=S: y ( n -1 - -1-5- -1 - -2- su descomposición canónica:
N = aXP ... (1)
=} 11-1=2 =} 11=3
Por teoría se sabe que la cantidadde divisores (CD)
=} Y = 2
de un número esta dado por:
Operando en (2): CD=(x+l)(y+l) ... (2)
x x ,x = 21,5(16) En el problema, N2 tiene 63 divisores, entonces:
5
xx+~ = 21(16) + 16 _1 N2 = (axbx )2 = a2xb2y
x(ll)+~ = 2(16)+1+ 155 =} CD=(2x+l)(2y+1)=63
100x=33+1 (2x + 1)( 2y + 1) = 9x7
9 3
=} x =3 =} 2x + 1 = 9 /\. 2y + 1 = 7
x=4/\y=3
Para hallar la suma de x + y + 11:
Para A = l4: x + y + n = 3 + 3 + 6 = 12 (4) N3 tiene 130 divisores, entonces:
3
ParaA=10: x+y+n=3+3+10=16 (S) N3 = (aXbY) = a3xb3y

ParaA=S: x+y+n=3+2+3=8 ... (6)


=} CD=(3x+l)(3y+l)=130
Verificandolas alternativas sólo existe respuesta ( 3x + 1 )( 3 Y + 1) = 13 x 10
para (S).
Clave: D 3x + 1 = ~3 /\ 3y + 1 = O
x=4 /\ y=3
9. Si definimos:
Como cumple para ambos casos, entonces se acep-
Número de varones: V ta estos valores:
Número de mujeres: M
Para N4:
Como en total hay 131 profesionales, y además N4" = a4xa4y
como se trata de personas donde la octava parte de
varones son ingenieros implica que la cantidad de = a4(4)b4(3)
varones es multiplo de 8, y que la séptima parte de
=} CD = (16 + 1 )(12 + 1) = 221
mujeres sean economistas implica que el número
de mujeres sea múltiplo de 7, entonces: La suma de las cifras:
2+2+1=5
V+M=131
Clave: B
8v+7m = l31
8(12)+7(S)=l31 11 . Un número de la clase de equivalencia 7/11
esta dado por:
=} V=8(12)=96 7k
fequivalente = llk
M=7(S)=3S
El problema condiciona que el numerador tenga 3
Del enunciado, la octava parte de varones son in-
cifras, entonces:
genieros, entonces los varones que no son ingenie-
ros: 100~7k~999
14,28 ~ k ~ 142,71

Clave: D CSl = {k E N/k = IS;16;17; ... ;142}


"el lJNI 2008 -1 MATEMÁTICA

El denominador tiene 4 cifras: 13. En la ecuación:


1000 $llk $ 9999
90,9 $ k $ 909
31x+11 _[3x - 1[ = 3x +2 ... O)

Para la solución se presentan 2 casos:


eS2 = {k E N/k = 91; 92; 93; ... ;909}
Caso 1:
Luego, es¡ n eS2 :

es = {k E N/k = 91; 92; 93; ... ;142}


El número de elementos: ~ xzO
N = 142 - 90 = 52 Luego en O):
Clave: C 3x+1 _ (3X _ 1) = 3x +2
3x+1 _ 3x + 1 = 3x + 2
12. Del enunciado, se tiene el número N, que par~
extraerle la raiz cúbica tiene el residuo R. 3x (3 - 1 - 1) + 1 = 2

N = p3 + R ... O) 3x = 1
Del enunciado también se tiene: ~x=O ... (2)

N - 721 = (P - 1)3 + R ... (2) Caso 11:

Restando O) menos (2):

721 = p3 _ (p3 _ 3P2 + 3P - 1)


~uego en (1):
721 = 3P2 - 3P + 1
3Ix+11_[ _(3x -l)J = 3x + 2
~ O = 3P2 - 3P - 720
31x+11+ 3x -1 = 3x + 2
~ O = p2 - P - 240
31x+11 = 31
~{P=16 =} P=16 ... (3)
P =-15 x+l=l~x=O' (no)
~lx+ll =1 ~ ,
{ -(x + 1) = 1 ~ x = -2 ; (si)
Operando en (1):

N = p3 + R El conjunto solución de la ecuación O) es :


eS={0;-2}
N - R = p3
La suma de estas raices:
De(3): N-R=(16)3
Xl + x2 = 0+ (-2) =-2
N - R = 4096
Clave: B
La suma de las cifras:
14_ Se da la ecuación:
4+0+9+6=19
Clave: O 4x2 - 2x + 3 = O ... O)

Donde a, b son las raíces


El teorema de Cardamo: si Ax2 + Bx + e = O ,
donde A -# O ,y Xl' X2 sus raices, entonces:

... (2)
e 16. Si se tiene:
Xl X2 = A ... (3)
m+n=m ; mn=2m ... (*)
Aplicando (2) para la ecuación (1):
Para hallar el valor númerico de:
a+b=-(-2)=1 ... (4)
4 2
Aplicando (3) para la ecuación (1):

ab = ~ ... (5)

La ecuación cuadrática cuyas raices son (2a - 1)


Y (2b-1):
(y - (2a - 1) )(y - (2b - 1)) = O
(y - 2a + l)(y - 2b + 1) = O
y2 _ 2by + Y - 2ay + 4ab - 2a + y - 2b + 1 = O
y2 _ 2y[(b + a)-lJ + 4(ab)- 2(a + b)+ 1 = O 1
- ~
Oe(2),(3): y2 -2Y[(1)-1]+4(%)-2(1)+1 =O - m3 + n3
n3m3
y2 +3y+3 =O
1
Clave: C m3 + 113
Aplicando identidades algebraicas:
15. Dada la función: 1
(2m +112 -mil )
XE(-~5' '~J ... p = (m+n)
2
f(X)=5x -7x-6 . (1)
x + -ª- ' 5 1
5 2
(m + 11)( m + n2 + 2mn - 3mll)
5x2 + 3x - 10x - 6
x+-ª- 1
5 (m + n )( (m + n)2 - 3m n )
5X( x + ~ ) - 1 o( x + fa) De (*): 1
x+-ª-
5
m[(m)2 -3(2m)]

= 5x - 10 ; x * -~ ; si cumple 1
12 - 36
Del intervalo de la expresión (1): _-.l
24
-~<x<~
5 - 5 Clave: C
- 3 < 5x :s: 3
-13 < 5x - 10 s -7 1 7. Si se tienen matrices A y B de orden 2 x 2 .
-13<f(x):S:-7 Analizando las proposiciones:
=:) 7:s:lf(x)I<13
I. Si A 2 = O =:) A = O . Falsa (F)
=:) Ran[lf(x ~J = [7; 13) Si A 2 = O , A no es necesariamente
matriz nula.
igual a la

Clave: O
11. Si AB = O =:> A = O ó B = O . Fnls.: (F) 19. Para maxirnízar:
Si AB = O . A YB no necesariamente son igua- x+y ; X,YEIR
les a la matriz nula. Sujeto a las siguientes condiciones
IlI. (A + B)( A - B) = A 2 - B2 . Falsa (F):
2x + 3y ~ 6 =:> LJ : y = -~x + 2
(A + B )( A - B ) = A 2 - AB + BA - B 2
2x + y 56=:> L2: y = -2x + 6
Clave: D.
y54
x~O
18. Sea n el número de estudiantes, y x lo que
paga cada alumno, y P la cantidad que se paga al y~O
profesor; entonces: Graficando estas 5 inecuaciones en el plano carte-
nx = P ... (1) siano para hallar la región admisible.
I. Si ubieran 10 estudiantes más, entonces cada
uno pagaría s/.lO menos:

(n+lO)(x-lO)=P ... (2)

Il, Si el número de estudiantes fueran 2 menos,


cada uno pagaría si. 5 más.
y=4
( 11 - 2)( x + 5) == P ... (3)

Ahora resolvemos el sistema de ecuaciones de (1),


(2) Y (3):
Igualando (1) y (2):

l1X = (11+10) (x-lO) x


n x = nx -IOn + 10x -100
=:>n=x-lO ... (4) En las proposiciones:
Igualando (1) y (3): I. "Los puntos (2;2)y (4;1) pertenecen a 1<1 re-
gión admisible". Falso (F)
nx=(n-2)(x+S)
Del gráfico se puede observar que:
nx = nx + 511- 2x - 10
(2;2) si pertenece
0= Sn-2x-10
(4; 1) no pertenece
De(4): 0==S(x-10)-2x-10
11. "La región admisible es un polígono de cuatro
=:> x = 20 ... (5)
lados". Verdadero (V)
Reemplazando el valor de x = 20 en (4): El polígono se observa en el gráfico.
11=20-10=10 ... (6) 111: "El valor óptimo es 5". Verdadero (V)
Reemplazando (5) y (6) en (1):
Verificación:
10(20)==P =:> P=200 Interceptamos las rectas L: y == 4 con L2
El número de estudiantes es 10 y se le paga al pro- 4 == -2x + 6
fesor S/.200. =:>x=l
Clave: B
=:>P=(1;4)

Luego: (x + Y)1I10X = 1+ 4 = 5
Clave: D
~::~I:":'===fM1:W:iIIi1tI!!!I:.~:;[=~U~N~I~2~00~8[-JI=M~A~T~EM~Á:!T~IC~A~=I

20. Dada la expresión: Finalmente se concluye:


X"+l =ax" +b ; 11 =0;1;2;3... ... (l) Si a = 1 x" = Xo + I1b

Donde a y b son números reales


Si a#- 1 x = a"x + b( 1 - a" )
En O), si a = 1 : " o 1- a
xn+l = x" + b 11 = 0;1;2;3 ... Clave: B

Evaluando:
21. Del enunciado:
... (2) B

11 = 1: xI+1 = x2 = Xl + b
Oe(2): x2 =(xo+b)+b
x2=xo+2b ... (3)
A
~ a R a C
11 = 2:
Complementamos con los siguientes trazos:
Oe(3): x3=(xo+2b)+b B
x3 = Xo +3b ... (4)

Observando (2), (3) Y (4) se puede inducir a:


x" = Xo + 11b
En O), si a#-1 :

xn+l = ax" +b 11 = 0;1;2;3 ...


Evaluando:
11 = O: oo. (5) Los triángulos ABD, ADR Y RCE son congruentes:
Por propiedad:
11 = 1:

Oe(5): =a(axo+b)+b ~1 • oo. (1)


4
=a2xo+b(a+1) ... (6) Del gráfico:

11=2: x2+1 =x3 =ax2+b tg14° = JL


3m
Oe (6): = a2x a[ o + b(a + l}J + b De (1):
1_ b
'4 - 3i1l => b =lm
4
oo. (2)

=a3xo+b(a2+a+1) ... (7) En el triángulo ADR:

Analizando (5), (6) Y (7) se .llega a la siguiente tga = !!!


b
regla de correspondencia: .
111
De (2):
x" =a"xo+b(a"+a"-I+ ... +a3+a2+a+1) 1m
4
Por cocientes notables se reduce a:
= 1 => a = 53°

x
11
= a"x
o + b( a"
a-1
- 1)
Luego:
m..iBAC = 2a = 2(53°) = 106°
= anxo + b( 1- a" )
1-a
Clave: C
22. Del enunciado se tiene: Por condición de problema también se sabe que los
e ángulos centrales de los dos polígonos regulares,
estan en la relación de 5 a 6, es decir:
360 0

111 5
360 0 6 ; 111> n
n

a<b<c ~1I=tm ... (4)

Reemplazando(4) en (3):
m2 _ 3m - 112 + 311= 8
2
Por condición del problema, las longitudes de los m2 - 3111....,(t111) + 3( tm ) = 8
lados a, b, y c del triángulo ABe forman una pro-
gresión aritmética, entonces se cumple: 11m2 -18m - 288 = O
b-a=c-b
18±~(18)2 -4(11)(-288)
~ a+c=2b ... (1) ~111= .
. 2(11)
Por teoría, el área del triángulo:

S abc
~ 111}= 6 ; 1112= i~ (descartado)
• ABC = 4R
... (2)

Reemplazando 111= 6 en (4):


• SABC =(a+~+c)r ... (3)
5
n = 6(6) = 5
Igualando (2) y (3):
El ángulo interior del polígono regular de 5 lados:
( a + 2b + c) r = abc
4R
. 180 (n
15= ---'-/1----'-
0
- 2)

De O): (2b) + b) = abc 180°(5 - 2)


( 2 r 4R
5
~ rR = ac = 108°
6
El ángulo exterior del polígono regular de 6 lados:
Clave: O
e6 = 360° = 60°
6
23. Si un polígono es de N lados, entonces el total
La diferencia de estos dos ángulos:
de sus diagonales.
is-e6 =108°-60°=48°
... O) Clave: A

Del enunciado del problema, el número de sus


24. Del enunciado:
diagonal es de dos polígonos se diferencia en 4:
B
DIII -DIl = 4

ffi
m(m-3) n(n-3)
De (1) : 2 - 2 = 4

~2 _ 3m _ n2 + 3n = 8 ... (2)

A D e
UNI2008 -1 MATEMÁTICA 1;!=:i;=r;;::=::::i:;~!:4Jtru;:i\t1 GomcZ

Datos:
==} BM = lAB
7
AD = 30 cm} ... (1)
DC = 40 cm De (6): =t(42)

==} AC = 70 cm ... (2)


= 6 ; (cm) ... (9)

Por propiedad de bisectriz: Aplicando la Ley de cosenos en el triángulo ABD:

AB BC BD = JAB2+AD2_2(AB)(AD)coss3°
AD = CD
De(1) : AB BC ... (3) = ~( 42)2 + (30 r- 2( 42)( 30)( ~)
30 40
= 24Fz
Luego en (3), por proporciones:
El perímetro del triángulo BMD:
AB = BC = k
3 4 P = BM + DM + BD
==} AB = 3k ; BC= 4k ... (4) De (7), (8), (9): = 6 + 30 + 24Fz
De (4) se deduce que se trata de un triángulo rec- = 36 + 24Fz
tángulo notable, entonces: Clave: E
xc = sk 25_ Del enunciado del problema:
De (2) 70 = Sk: ==} k = 14 ... (5)
2
Reemplazando en'( 4): 2, ../-._ ...•. P

AB=3(14)=42 (cm) ... (6)

BC=4(14)=s6 (cm) ... (7)

En el gráfico, el cuadrilátero ABM es inscriptible,


donde AM es díametro (Propiedad).
//~~~-- __ B
En el triángulo rectángulo PQR:
,
,,
I
I h = ~42 _ 22
I
I
I = 213 ... (1)
I
I
\
Del gráfico podemos deducir que:
\

\ 3r Área de base inferior: A¡ = 62 (2)


~.~'~,~,,~------~~--------~~~C
Área de base superior: A2 = 22 (3)
Entonces en el triángulo AMD El vólumen del tronco de pirámide regular esta
AD = DM = 30 ,(cm) ... (8) definido por:

El triángulo ABM es notable, entonces por propie- V = ~(Al + A2 + ~A¡A2 )h


dad:
De (1), (2), (3): = M 62 + 22 + J62 (22) )213
~a a = lb = 10413
7
b 3
Clave: C
26. Según el enunciado: 27. Cuando el cubo de arista "a" gira alrededor de
una de sus aristas.

C~-----------.~ Datos:
eB =.J7 ... (1)
r 2 ... (2)
AB 3

AT es segmento áureo de AB R=J2a


por definición en un segmento PQ, el segmento
El volumen generado:
áureo x es la porción de éste, tal que:
V = rrR2a
x =( J5 2- 1 )PQ ... (3)
= rr( J2a )2 a
En el gráfico del problema, como AC es diámetro,
entonces AC = 2r ; luego en (2): = 2rra3

(2)r (2)2
Clave: C
AB =-3-
28. En el gráfico del enunciado complementamos
fA =.1 => CA = .1AB ... (4)
algunos datos auxiliares:

~:'*
AB· 3 3
Aplicando Pitágoras al triángulo CBA
AB2 = (AC)2 _ (BC )2
A~L
De (4): AB2=[1(AB)rjBC)2 r e
~AB2= (BC)2 o
r
De (1): ~AB2=(.J7)2
~ D
=>AB = 3 ... (S) E

Como -AT es segmento áureo de AB, entonces: ... (1)

AT= (..JS -1 fE Por Pítagoras en el triángulo AOB:


De (3):
2

De (S): = (..JS
2
-1 1 3)
OB = ~ OA2+ AB2

= J,.2 + (Vr t
= ~(..JS -1)
= .J146 r ... (2)
Clave: 8 S
Por semejanza de triángulos:
Ll ACO - Ll AFE
Como el prisma exagonal es regular, el triángulo
AC == AF ABC es isósceles.
AO AE
... (2)
Ur Ur+CF
5 _ -,,5,--;==~_ En la base superior del prisma:
-r-- -J146
r
5 ,,
,,
~ CF == 13.Ji46,- _ 13,- Gráfico 11
,,
25 5 ,,
Luego: ,,
,,
AF == AC + CF ,
A -------------

== 13 r +( 13.Ji46 r _ 13 r ) Del gráfico II se deduce que:


555
== 13.Ji46 r ... (3) AB == 2( 1)==-J3a a

25
Como AD es una semicircunferencia, la longitud Del gráfico 1 también se deduce:
de esta es: AB == CD ==-J3a ... (3)

- 1 El triángulo ACD del gráfico 1:


lAD == 2(2rrr) == 11:r ... (4)

Dividiendo, mienbro a mienbro, (3) entre (4): AC == JAD2+ CD2

13.Ji46 r De (1) y (3) == ~( Jaa)2 + (-J3a)2


AF _ 25
-=-
lAD 11:r ==am ... (4)
Por la ley de cosenos en el triángulo ABC:
13.Ji46
25 AB 2 ==AC2 +BC2 - 2(AC)(BC)cose
(3,14 )
De (2) AB 2 ==AC2 +AC 2- 2(AC)( AC )cose
==2
Clave: C AB2 == 2AC2-2AC2cose
De(1),(3j,(4): (.fja)2 ==2(.Jila/ _2(ma)2 COSe
29. Del enunciado se tiene el prisma exagonal re-
gular de arista a y altura a rs ~ COse== 19
22
B
~ e ==arccos(;;)
Gráfico 1 Clave: O

h 30. En la ecuación de la hipérbola:


x2-3y2-8x-18y==14
(x2-8X+~ )-3(y2+6y+ y)== 14
2 2
(x-4) -3(y+3) ==3
Datos: (x_4)2 (y+ 3/
------==1
... (1) (13)2 12
UN12008-1 MATEMÁTICA =:",===:~
Graficando: Uniendo las dos gráficas anteriores:
y y

Ll

y
are csc(x) ---.j-----'
~l
x

El área sombreada es la solución de (1).


En el punto P:

tga = ,k .~ a = 300 are see( x) = are ese( x) ... (2)

Por teoría:
Las asintotas de la hipérbola son las rectas
L1 y L2' las cuales forman un ángulo obtuso de : are see(x) + are ese (x) =~

e = 180 0
- 2a = 180 0
- 2(30°) = 120° De (2): aresee(x)+aresee(x)=~
~ e = 271:
3
Clave: O
are see(x)
~x=.f2
= *
31. Dada la función: Del gráfico:

f(x) = ~are see(x )-areese(x) + arcsen (_x_) x E (-00 ; - 1] v [ ..fi ; 00 + )


x+1
Restricción: Clave: C

aresee(x)-are esc(x) ~ O 32. Dados los números:


are see(x) ~ are ese(x)
Xl00: representan la medida de un ángulo en
Gráficandoare see(x): sexágesimales (S). .
y
x100 + 1: representa la medida de un ángulo en
~ .... -~n centesirnales (e).
Por teoría:
---------------------¡--- "2 ------r----·········------
-1 x S ~ R
200
'-v----'
180 7t

Gráficando are ese( x) : II

y Reemplazando en (1):

~ ..----~ x100 x1oo+1


180 200
x ~ x100 = 9 ... (1)
Reemplazando O) en (Il) para hallar el ángulo ex- Operando en (4):
presado en radianes:
AAOB = 65
5 R
180 ¡¡ !(OA)2 = 65
x100 R
180 1t De (S): !(3e6f := 65 ~ e= 1~8 ... (6)
1t
De O): 1~0 ~ ~ R = 2 O ... (2) Operando en (3):
Ahora hallamos el complemento de R: AeoD = 35

If(R)=~-R !(OC)2 = 35

De (2): 11: 11: ~ If ( R) = 911:


De (6): 1 0 8(OC)2 = 35 ~
-
OC=~
5
... (7)
2 20 20 2 5 3,,2
Clave: C
Operando en (2):
AEOF = 5
33. En la figura AOB, COD y EOF son sectores cir-
culares: !(EO)2 =5
A
8
1 °5(EO)2 = S ~
- S
De (6): EO = -r:: ... (8)
2 3,,6

o 36 La longitud del arco CD :


.tCD = e(oc)
B
De (6) y (7): = 1~8C~)
Datos:
_ 36
.tAB = 36 ... O) -fi
Las áreas de los sectores:
AEOF=5 ... (2) La longitud del arco U:
AeoD = 35 ... (3)
.tU = e(EO)

AAOB = 65 ... (4) De (6) y (8): = 1~8( 3~ )

Por definición de longitud de arco y área de sector 36


circular: J6
.tPQ = aR Finalmente:

="2aR
2
ApOQ

Luego en O):
.tAB = e(OA) Clave: C
36 = e(OA)

~ OA = 36 ... (S)
e
;o...;,:";:..:.;;;:'==..::::::::.:;;.=_:=~:::'::::.!IIli ... =. =. :;;::;;;;;;;;; __ U_N_I_2_0_08_-_1 _M_A_T_E_M_Á_T_IC_A
__ r·~;:¡:~~i.:"=II'i:m:in:":1 =;:;:¡¡¡¡w¡¡:.:_~~
34_ Dada las ecuaciones de las rectas:
Si: SC1l2v=0~x=bt ;xe[-7t;7t]
Ll : y = 2x + 1 ... O)
x = -7t; O; 7t
L 2· .y_X-2-2:1 ... (2) Si: cosx=0~x=~(2k+l) ;xe[-7t;7tj
En el sistema cartesiano: x = _2I . 2I
2' 2

Las soluciones son s: -lt ; - ~ ; O ; ~ ; 1t

Clave: O

36. Del enunciado:


x

A a+y+p"=1800 ... O)
~
La ecuación de una recta está dado por:
Por teoría:
y = mx + b
senx eosy+eosx seny = sen(x+ y) ... (2)
De O) y (2) se deduce que las pendientes de las
rectas son: cosx eosy - senx seny = eos(x + y) ... (3)

... (3)
Dato: (tan a)(tan p)(tan y) = 2006 ... (4)

En la expresión del enunciado:


Por propiedad, si mI es la pendiente de la recta Ll
y m2 la pendiente de la recta L2 ' entonces: F = 1 +tamx+tan!3+tany
= 1 + sena + sen~ +tany
tgñ = mI - m2 cosa cos~
1 + mI' 1112
= 1 + senacos~+sen~cosa +tan
cosx cosü y
De(3):
2-a) =1+ sen(a+~) + seny
[de (2)]
1+ 2(!) cosacos~ cosy
= 1 + sen(1800-y) + seny
_ 3 [de (1)]
-4 cosncosü cosy
= 1+ seny + seny
~ 8 = are tg( ~) cosncosb cosy

= 1 +seny a;c.s:...:e,,-nL~_+--,sc:e.:..:n.::.a..:.se.:..:'=l~
(_co_s...!.y_+_e_o_sa_e,-,o-,-s!:.~_--,-s..:.el-;l )
Clave: E cosncosücosv

= 1 + sen (COsy+cos(a+~)+senasen~). [de (3)]


35. Identidad trigónometrica: y costxcosücosv ,

eos 3a = 4eos3 a - 3eosa 1 (cosy+ cos(lt - y)+ senaSen!3)


[de (1)]
= +seny ccsrrcosñcosv
Por condición del problema:
= 1+ senysenasen~
cosx = eos3x cosacosf3cosy
cos = 4cos3 X - 3cosx = 1 + (tana)(tan~)(tany)
~ 4cosx - 4cos3 x = O =1+2006 [de (4)]
=2007
cosxÜ-cos2x) =O
2 Clave: B
eosx(sen x) = O
37. En el gráfico del enunciado complementamos El número de aristas del poliedro:
algunos trazos y datos auxiliares. N
NA=-f
B
De (2): 298
C 2
= 149 ... (3)
,,
,, Luego, por el teorema de Euler, hallamos el núme-
,
\
\
ro de vertices (N v ):
\
\
I C + Nv = NA + 2
a:
~~~----~O~------~,Q De (l)y(3): 65+Nv =149+2
I

I
I
~ Nv = 86
b+x I
I

I
I
Clave: C
,I

---_ ...
39. Del enunciado, gráficamos:
El triángulo RBC es semejante al triángulo PRA,
entonces:

a+b ;¿
z x
=} zy = x( a + b) ... (1)
Aplicando el teorema de las cuerdas: ;/3 -1
---x
h-/3
yz = b( 2x + b) ... (2)

Igualando (1) y (2):


x( a + b) = b( 2x + b)
x( a + b) = 2bx + b2 En el triángulo rectángulo CBD:
x(a-b)=b2 ( BC f = (CD)2 + (BD )2
=}x=L
a-b =x2+(~x)2
Clave: C J2-F3
= x2 + F3 -1 x2
38. El políedro tiene como caras: 2-F3
12 triángulos
=X2[1+ (F3-1)(2+F3)]
16 cuadriláteros
( 2 - F3 )( 2 + F3 )
24 pentágonos
13 exágonos = x2[ 1 + ( F3 + 1)]
Entonces el número de caras (C):
=} BC = J2 + F3x ... (1)
C=12+16+24+13=65 ... (1)
Como la pirámide es regular, el triángulo ABC es
El número de lados de los polígonos de las caras:
isóseles, entonces:
NL = 12(3) + 16( 4) + 24(5) + 13(6)
... (2)
= 298 ... (2)

i
~1u~~
Gom~z
~========[JIU~N~I II
2~O~O~8:- !M~A~:r~E!M~Á~TEIC~A~J=========~301
...
Aplicamos el teorema de cosenos en el triángulo En el triángulo rectángulo COB: •
rectángulo ABC:
tg300 = OB
(AB)2 = (AC/ + (BC)2 - 2(AC)(BC)cos8 OC
De (1): J3 9
De (2): (AB)2 = (BC)2 +(BC)2 -2(BC)(BC)cose 3 2r+h

(AB)2 = 2(BC)2(1- cos8)


~ 2r+ h = 9J3 ... (2)

En el triángulo rectángulo CHD:


2
De (1): x2=2(h+43X) (1-cos8)
sen300 = HD
x2 = 2(2+ 43)x2(1-cos8) HC
1 _r_
1 = 2(2+ 43) - 2(2 + 43)cose "2 h+r
~ h =r ... (3)
-3 - 243 = -2(2 + 43)cos8
Reemplazando(3) en (2):
~ cose =243 + 3 2r+(r)= 9./3
243 + 4
(243 + 3)(243 - 4) r = 3./3 ... (4)
El volumen de la esfera inscrita:
(243 + 4 )(243 - 4)
43 v = .1rrr3
=2 3
De (4): = 1rr(3J3f
Clave: B = 108rrJ3
Clave: A
40. Presentamos al cono de revolución en una vis-
ta frontal.
C

Dato:

... (1)
;
1
APTITUD ACADEMICA
y CULTURA GENERAL
EXÁMENES DE ADMISiÓN

UNIVERSIDAD NACIONAL,
DE
INGENIERIA

LIMA - PERU
CONTENIDO 1

Examen de Admisión 2001 - 1 •••••••••••••••••••••••••.•••••........••................•.•..•.•••••••.••••••• 1


Solucionario o •••••••••••••••••••••• 9

Examen de Admisión 2001 - 11 ....•..•••....•....•••......................................•..•.•..••••..• 18


. Solucionario 28

Examen de Admisión 2002 - 1 •.••.•.•.•.•.•.•.•.•.•.•.•...•...................................•.••...••.•• 36


Solucionario o ••••••••••••••••••••••••••••••••••••••••••••••••••••••••••••••••••••••• 44

Examen de Admisión 2002 - 11 ......................•••.. ..•• 54


Solucionario 63

Examen de Admisión 2003 - 1 ..•.........................•.....•....•.•.•.•....•.•.•..••.••.••.•.•.•.•.•. 72


Solucionario '.' 82

Examen de Admisión 2003 - 11 92


Solucionario o.. 102

Examen de Admisión 2004 - 1 ••••......••.•.•..•.•.•.•.•...•..•.•.•.••.•.••.••.•...•.•••.••.•.•...•.•... 113


Solucionario '.... 122

Examen de Admisión 2004 - 11 •........................................................................ 132


Solucionario 142

Examen de Admisión 2005 - 1 ......•.........•...•...•.•.•.•.•.•...•...•...•.•.•...•.•.....•........... 155


Solucionario 166

Examen de Admisión 2005 - 11 •....... ..........•...•.•.•.........•.•...•...•.•.•.•.•.•.•.•............ 179


Solucionario 189

Examen de Admisión 2006- 1 •.•.•.••.••.•.•.•.•.•.••.•.•........•.•..•..•............................... 200


Solucionario : 212

Examen de Admisión 2006 - 11 : 226


Solucionario 238

Examen de Admisión 2007 - 1 •.•.••..................•.•.•.•...••.•.•.•...•.•.••.•.•.•.•.••.•.•.•.•.•.• 251


Solucionario 263

Exameñ de Admisión 2007 - 11 .. 277


Solucionario o o................ 289

Examen de Admisión 2008 - 1 ..••. •...•....•..•..... .............•.....•......•. ..•...•.....•.......... 305


Solucionario 317

APTITUD ACADEMICA y CULTURA GENERAL


~l UNI 2001-1 APT. ACADÉMICA Y CULTURA GENERAL 1~0

APTITUD ACADÉMICA Y CULTURA GENERAL

APTITUD ACADÉMICA 7. Indique la alternativa que contiene los sinónimos


respectivos de las palabras o frases subrayadas en el
ANALOGíAS siguiente enunciado:

1. CREATIVO: TRIUNFADOR: "El plebiscito se realizó con toda la prohidad. eviuindosc


A) Propiciador Reconocido cualquier tipo de infundio".
B) Inteligente Empeñoso A) La elección - propiedad - decomiso
C) Imaginativo Solidario B) La votación - honradez - embuste
D) Inventivo Ganador C) La selección - prudencia - latifundio
E) Empresario Emprendedor D) El comicio - probabilidad - patraña
E) El escrutinio - cordialidad - querella
2. GENEROSO: EGoíSTA:
A) Productivo Altruista ANTÓNIMOS
B) Dadivoso Avaro
8. FRUSLERÍA
C) Liberal Anarquista
A) Casualidad B) Futilidad C) Importancia
D) Comprensivo Oprobio
E) Bondadoso Entuerto D) Nimiedad E) Intrascendente

3. TEORíA: PRÁCTICA: 9.COLOFÓN


A) Abstracto Concreto A) Iniciación B) Prefacio ci Incoloro
B) Esfuerzo Éxito D) Presentar El Síntesis
C) Estudio Conocimiento
D) Libro Sabiduría 10. Tomando en cuenta el siguiente enunciado. seña-
E) Todo Parte le la alternativa que contiene el antónimo de la palabra
PROBO.
4. ARQUITECTURA: PLANOS:
A) Altar Sacerdocio "Nadie puso en duda lo que dijo pues sabían que él era
B) Dibujos Arte un hombre probo".
C) Medicina Remedios A) Vengativo B) Amable C) Justo
D) Microscopio Biología D) Deshonesto E) Loco
E) Música Partitura
11. CICLÓPEO
SINÓNIMOS Al Ridículo B) Coloso C) Pigmeo
5. Señale el sinónimo de VADEMÉCUM D) Simple El Cursi
A) Fórmula B) Premio C) Prontuario
D) Remedio E) Valoración ORACIONES INCOMPLETAS
6. Señale el sinónimo de la palabra subrayada 12. Complete el texto con los términos de la opción
"Antenor elucida la Biblia" más adecuada.
A) Alaba B) Da C) Interpreta La no consiste en carecer de querer llevar la
D) Lee E) Ora razon que uno pueda tener hasta las últimas consecuen-
cias. (Julio R. Ribeyro).
.:.6>:
UJI~
Glrili¡;z

A) cuestión dinero - como Del texto anterior se puede concluir que:


B) locura razón sino en A) El ingeniero también debe ser economista.
C) angustia - tranquilidad para B) La ingeniería se ocupa de la asignación eficiente
D) situación - argumentos - sino en de los recursos.
E) idea público - a quien
C) La ingeniería es la única profesión que uuns- for-
13. Complete el texto con los términos de la opción ma la naturaleza para beneficio de la humanidad.
más adecuada. D) La ingeniería se basa en el conocimiento teórico y
aplicado de las matemáticas y ciencias naturales.
La realidad se impone la fantasía, la realidad
E) Todo economista puede ser ingeniero.
el camino discurren nuestras vidas.
A)en buscando cuando 17. "Melgar murió muy joven. Y aunque resulta siem-
B) por ordenando para el que pre un poco aventurada toda hipótesis sobre la proba-
C) desde construyendo al que ble trayectoria de un artista. sorprendido prematura-
D) con - asumiendo - con que mente por la muerte, no es cxesivo suponer que Melgar.
E)a - siendo - donde maduro, habría producido un arte más purgado de re-
tórica y amanerarnientos clásicos y, por consiguiente.
14. Complete el texto con los términos de la opción
más nativo, más puro".
más adecuada.
Para el autor del texto:
El mérito del ..... consiste en buscar el máximo de
A) La muerte truncó la evolución artística de Mclgur.
en el mínimo de (A. Vaselovski).
B) Melgar fue un artista inmaduro.
A) ingenio - artificios - prototipos C) Melgar fue un artista nativo y puro.
B) artista - sus obras - espacio
D) Melgar rechazó la retórica en el arte.
C) estilo pensamiento - palabras
E) Se puede decir poco sobre el arte de Melgar.
D) examen incógnitas - problemas
E) hombre - posibilidades - aventuras 18. Más allá del argumento humanitario. los médicos
deben mostrar preocupación. además de ofrecer una
COMPRENSiÓN DE LECTURA cura. Hay otras razones para considerar la realidad pxi-
cológica y social de los pacientes como algo que porte-
15. "En la escuela nos enseñan a escribir y se nos da
nece al reino médico. En la actualidad existe un mar-
a entender. mas o menos veladamente, que lo más im-
gen de eficacia médica en prevención y en tratumicn-
portante y quizá lo único a tener en cuenta- es la gra-
to, que se lograría tratando el estado emocional de las
mática. La mayoría aprendimos a redactar pese a las
personas con el estado físico. Existe un aumento sufi-
reglas de ortografía y de sintaxis. Tanta obsesión por
ciente de las prestaciones médicas como para inferir
la epidermis gramatical ha hecho olvidar a veces lo que
que una intervención emocional debería ser habnual
tiene que haber dentro: claridad de ideas, estructura,
en la atención.
tono, registro, etc".
El tema principal es:
Del texto se puede concluir que:
A) Los médicos deben mostrar preocupación antes
A) La escuela sólo enseña gramática. de la cura.
B) La mayoria sabe redactar bien. B) Pertenece al reino médico la realidad psicológi-
C) Lo más importante en la redacción es la gramática. ca y social.
D) No importa saber ortografía ni sintaxis. C) La eficacia médica tanto en prevención C!J1ll1l en
E) Para redactar bien no sólo hay que saber gramática. tratamiento.
D) La necesidad de la intervención emocional en 1,1
16. "La ingeniería es la profesión en la que el conoci- atención médica.
miento de las matemáticas y ciencias naturales, obte- E) La mayoría de médicos se preocupa slÍlo por ~I
nido mediante estudio, experiencia y práctica, se apli- estado físico.
ca con juicio para desarrollar formas de utilizar econó-
micamente los materiales y fuerzas de la naturaleza para 19. Al estudiante no le basta con entender las causas
beneficio de la humanidad". y los efectos de los problemas ambientales en términos
cuantitativos. También debe expresar la solucrón po-
; 1 UNI 2001-1 APT. ACADÉMICA Y CULTURA GENERAL l---=,-~--_.
tencial de manera cuantitativa. Muchas cuestiones am- 23. Indicar la locución correcta:
bientales son muy complejas. El problema se divide en A) ¡Vistes a los asaltuntcs?
varios componentes que, a su vez. se analizan con ba- 13) Eso te lo dije a tí.
lances de materia o energía para cada componente. que C) Primeramente oremos a Dios.
conduce a la solución del sistema total. O) Donde dean esa película, voy.
El tema del texto es: E) ¿Nadies de ustedes lo hizo?
A) La complejidad de las cuestiones ambientales.
24. Indicar la frase coloquial correcta:
B) Cuantificación de los problemas ambientales.
A) Habló en nombre del pueblo y gobierno peruano.
C) Análisis de los problemas ambientales. B) Estoy leendo una novela de Yargas LLosa.
O) Soluciones a los problemas ambientales. C) La abogado se entrevistó con el detenido.
El El sistema total a solucionarse, O) Me ha irrogado un gran perjuicio.
E) [Sube arriba rápido que te necesitan!
20. ¿Cuántas generaciones quedan referidas en el si-
guiente fragmento literario? 25. Señale la opción que contiene una definición des-
criptiva.
"Mi lengua y cada molécula de mi sangre nacieron aquí.
Me engendraron padres que nacieron aquí, de padres A) El hombre es un ser racional.
que engendraron otros padres que nacieron aquí, de B) El hombre es un ser capaz de sentir, pensar, y
padres, hijos de esta tierra y de estos vientos también". resolver situaciones problemáticas.
A)2 B)3 C)4 0)5 E)6 C) El hombre es un proceso de conocimiento de sí
mismo.
21. La justicia consiste en dar a cada hombre lo que
O) El hombre es la medida de todas las cosas.
legitimamente le corresponde: démonos, pues, a noso-
E) El hombre es el centro del universo.
tros mismos la parte que nos toca en los bienes de la
tierra. Nacer nos impone la obligación de vivir, y esta 26. Un postulante de la UNI escribe a su padre.
obligación de tomar, no sólo lo necesario, sino lo có-
Indicar el mensaje correctamente escrito:
modo y lo agradable. La resignación y el sacrificio nos
volverán injustos con nosotros mismos. Por el sacrifi- A) Querido viejo: Pasado mañana doy la prueba fi-
cio y la abnegación de almas heroicas, la humanidad nal, hasta el momento estoy muy regular.
va entrando en el camino de la justicia. Nocuentes a nadie, quiero dar una sorpresa a to-
dos ustedes o que nadie se entere, que postulé.
La justicia, según el texto, consiste en:
Yo se que me deseas lo mejor..
Al Vivir cómodamente.
B) Papá: Me falta solo el último examen. ojalá ten-
B) Tomar lo cómodo y lo agradable.
ga suerte. Estoy estudiando como me enseñaste.
C) Tomar lo necesario.
Quisiera tenerte a mi lado. usted me daría fuer-
O) El sacrificio de almas heroicas.
zas ...
E) Tomar los bienes que nos corresponden.
C) Viejo: Estoy contento. me está yendo bien. La
3ra prueba es mi fuerte, venis que salgo airoso.
Le aseguro que esta vez no te defraudo. Cuide mi
REDACCiÓN
perrito ...
22. Ordene las oraciones que se presentan; para lo O) Querido papá: Me falta rendir la tercera prueba y
cual debe tener en cuenta que forman parte de un texto confió tener éxito. Estoy preparado para ésta.
titulado "La UNI: ayer y hoy" Espero, pronto darte una grata noticia. Qué feliz
1. Asumió su actual nombre en 1955. se pondría mama, disculpa, no quiero po-
nerte triste ...
11. Hoy tiene II Facultades y 26 Especialidades.
111. La UNI fue fundada en 1876 por Eduardo de Habich. E) Papiro: El viernes terminan los ex.imcnes, estoy
algo nervioso, le aseguro que estoy estudiando.
IV. Fue conocida como Escuela de Ingenieros.
Tú sabes cuanto deseo ingresar a la UNI. Deséerne
A) 1, IV. 111. 11 B) 1, 111. IV, 11 C) III,II,I,IY suerte. Reza por mí ...
0)111, IY, 1. 11 E) IY, 111, 11, I
~
1 UNI 2001-1 APT. ACADÉMICA Y CULTURA GENERAL I ~1l!Q
G~nleZ

27 _Para expresar que una persona es de fácil o buen 33. Hallar el número que debe ocupar el casillero UNI
trato.debe decirse:
A) El profesor Díaz es muy asequible, a todos presta
atención.
A)O B) I C)2 D)3 EH
B) Por su carácter asequible es diferente a los demás.
C) Es un "alma de Dios", un hombre asequible a todos.
D) El doctor es accesible a cualquier consulta médica.
SERIE DE FIGURAS
E) Es una persona asequible por su accesible bondad. 34. Indique la figura que debe ocupar el casillero UNI.

28_ Señalar la oración scmánticamente

n
apropiada:
A) La vergüenza
B) Del accidente
lo dejó incólume.
salió incólume.
t ~
~
UNI

C) Quedó incólume ante el agravio.


D) Incólume se mantuvo durante el juicio.
E) Su carácter incólume le da fortaleza.

29_ Dé usted el orden lógico a la secuencia de los


textos para lograr una lectura adecuada del conjunto.
l. Una especie de dinosaurios anacrónicos, y no creo
35. Indique la figura que debe ocupar el casillero UN 1.
que ello contribuya a la confianza pública.
2. Sé que existe una gran estima
que tenemos, dijo Woolf.
por las tradiciones

3. El nuevo presidente de la Cámara de los Comunes


W LiJ
Y
decidió romper con la tradición: no usará peluca
sino en ocasiones especiales.
4. Pero también sé que el llevar peluca hace que pa-

rn [%J
rezcamos desconectados de la realidad.
A) 3-2-4- I B) 1-3-4-2 C) 3-1-4-2
UNI
D) 1-3-2-4 E) 2-3-4- I

SERIES NUMÉRICAS
30_ Señale la alternativa que continúa coherentemente
la siguiente secuencia numérica:
5, 10, 25. 60, 125, ...
A B C D E
Al 220 B) 230 C) 130
D)210 E) 200 36. Indique la alternativa que continúa adecuadamente
la siguiente serie grMica.
31. Indique el número que sigue en la serie:
18 - 7 - 72 - 9 - 90 - ...
A) 16 B) 12 C) II D)78 E)5

32. Hallar el número que debe ocupar el casillero UN!.

A) 55 B) 58 C) 65 D) 74 E) 85

,-

37. En los cuadro' I y II las Iiguros cumplen una mis- 40. Indique la alternativa que continúa adecuada-
ma relación, idcntil ique e indique la alternati va que debe mente la siguiente serie de figuras.

~(8 ••••
A_B*Ce
OC~'i~

• • • UNI
··0··.· ..

!:~,1~1~1!~!,0~1!"
0* E~.
una misma relación, identifique dicha relación
que la alternativa que debe ocupar el casillero
e indi-
UN!.

11
RAZONAMIENTO MATEMÁTICO
Y LÓGICO

~~ @~ 41. En un colegio hay 35 niños. Cada llllO de ellos


tiene una bandera que puede ser monócromu. bicolor o
tricolor, habiéndose usado únicamente tres colores: rojo.

I~I~I~I~I~I
amarillo y azul.
El número de banderas bicolor es el doble del número
de banderas monócromas. mientras que el número de
banderas que tienen el color rojo es igual al número de
A B C D E banderas que tienen el color azul e igual al número de
banderas que tinen el color amarillo.
39. Indique la alternativa que continúa adecuadamente Si sólo ocho niños tienen banderas tricolor y dos alum-
la siguiente secuencia gráfica: nos banderas de color amari 110. i. Cuántas banderas
bicolor rojo-azul hay?
A)2 B)3 C)5 D)7 E) 10

42. Una sala de espectáculos tiene capacidad para mil


personas. El costo normal del derecho de ingreso es
SI. 10.00; Cuando una persona lleva un acompañante.
éste paga la mitad del costo normal del derecho de in-
greso.
Cierto día la sala estuvo completamente llena y se re-
caudó SI. 8250.00. Los asistentes fueron solos y en pa-
reja. ¿CuiÍntos espectadores más fueron en pareja que
solos?
A) 300 B) 120 C) 240
D) 350 E) 400
43. Un reloj se adelanta 75 segundos por hora. Si el 48. Tres amigos ejercen oficios distintos y por casua-
reloj es puesto a la hora exacta a las 6:00 a.m. ¿Qué lidad sus apellidos coinciden con los nombres de estos
hora marcará cuando realmente sea las 8.00 p.rn.? oficios, aunque no cada uno con el suyo. Al ser
A) 20:17'30" B) 20:42'30" C) 20:00'00" preguntados por sus respectivos oficios. respondieron
D) 21:22'30" E) 20:22'30" .así: "De las siguientes proposiciones. tres "Son falsas y
una es verdadera".
44. En un libro de 700 páginas hay historias de tic- 1. El señor Carpintero no es pintor
ción e historias reales. 11. El señor Albañil no es carpintero
En cada diez páginas de historias de ficción hay 12 III. El señor Carpintero es carpintero
ilustraciones del tema, mientras que en cada diez pági- [Y. El señor-Albañil no es pintor
nas de historias reales hay 11 ilustraciones del tema. ¿Cuál es la proposición verdadera'!
Si en total hay 810 ilustraciones en el libro, ¿Cuántas A)I B)II C)1I1
ilustraciones más hay de un tema que de otro? D) IY E) Las premisas son contradictorias.
A) 120 B) 150 C) 180 D) 240 E) 30
49. Tres animalitos. el gusano. el gato y el murciélu-
45. La masa de un péndulo recorre 27 cm en la osci- go, amigos de Alicia en el País de las Maravillas. fue-
lación inicial. En cada una de las oscilaciones siguien- ron acusados de haberse robado la sal y de habérscla
tes la masa recorre 2/3 de la oscilación anterior. ¿Cuál comido. Al ser interrogados. declararon:
será la distancia que habrá recorrido dicha masa hasta Gusano: El gato se comió la sal
el momento de detenerse?
Gato: Eso no es cierto
A) 35 cm B) 54 cm C) 72 cm Murciélago: nunca comí la sal
D) 81 cm E) 108 cm Si se sabe que al menos una de las declaraciones es ver-
dadera y al menos una es falsa. ¿quién se comió la sal?
46. Una pelota de hule cae desde una altura determi-
nada y cada vez que rebota alcanza una altura equiva- A) Gusano B) Gato Cl Murciélago
lente a 4/5 de la altura alcanzada en el rebote inmedia- D) Faltan datos E) Hay más de una solución
to anterior. ¿Cuál ha sido la altura desde la cual se dejó
caer la pelota de hule. si cuando se detuvo habia reco- 50. De las premisas:
rrido 180cm.? Todos los cerdos vuelan
A) 20 cm B) 30 cm C) 50 cm Ningún cerdo tiene cola
D) 60 cm E) 80 cm ¿Cuáles de las siguientes conclusiones son verdade-
ras?
47. La siguiente tabla muestra el resultado de los par- 1. No todos los cerdos tienen cola.
tidos de un torneo de ajedrez. Si los partidos ganados 11. Ningún animal que vuela tiene cola.
abonan 2 puntos, los empatados I punto y los perdidos O
111.Existen animales sin coja que vuelan.
puntos y sólo falta el encuentro entre José y Martín. ¿a
quién le ganó José? A) Sólo I B) Sólo 11 Cl Sólo 111
D)lIylll E)lylll
Nombres P.J. P.G. P.E. P.P. Puntos
Juan 6 6 O O 12
CULTURA GENERAL
Carlos 6 5 O 1 10
Javier 6 3 1 2 7 51. Virrey que defendió a los indígenas contra el tra-
bajo esclavizante en las minas, afirmando: "No he ve-
Eduardo 6 2 O 4 4
nido al Perú para arriesgar la salvación de mi alma".
José 5 1 2 2 4
A) Agustín de Jáurcgui
Marlin 5 1 O 4 2 B) Pedro Fernández de Castro. Conde de Lcmos
Pedro 6 O 1 5 1 e) Francisco de Tolcdo
Al Carlos B) Martín C) Pedro D) José Fernando de Abascal
D) Eduardo E) Javier E) Manuel Amar y Juniet
~,-
:.<>:

Góiñ'ez
I UNI 2001-1 APT: ACADÉMICA y CULTURA GENERAL ~

52_ "Durante qué gobierno se obtuvo el retorno de 58. Señale la oración donde se usa correctamente la
Tacna al seno de la Patria" preposicrón (a. de. en. para. por).
A) Manuel Pardo D) José Paruo Al Andrés se empeña por ingresar aia universidad.
B) Oscar R. Bcnavides E) Luis M. Sánchcz Cerro 13) Carmen mantiene su negativa para enseñar.
C) Augusto B. Leguía C) Diana no tiene prisa en salir de la academia.
D) Luis insiste en estudiar geometría.
53. Uno de los principales promotores y Protector de E) Pablo tiene preferencia de los cursos de letras.
la Confederación Perú-Boliviana fue:
59. Su obra cumbre presenta la historia de siete ge-
A) Agustín Gamarra
neraciones de los Buendia. ¡,Quién es'!
13) Andrés de Santa Cruz
A) Altredo Bryce Echeniquc
C) Felipe Santiago Salaverry
13) Juan Rulfo
D) José de la Riva Agüero y Osma
C) Gabriel García Miirqucz
E) Ramón Castilla
D) Mario Vargas L10sa
54. Señale la opción que sigue las reglas de una co- E) José María Argucdus
rrecta definición.
60. ¿Cuál de las siguientes personas no ha recibido
A) La democracia es contraria a la autocracia.
el Premio Nóbel de Literatura'!
13) La lepra es una enfermedad infecciosa crónica A) Miguel Angel Asturius 13) Gabriela Mistral
producida por el bacilo de Hansen.
C) Jorge Luis Borgcs D) Camilo José Cela
C) La esperanza es cuando uno tiene confianza en E) Octavio Paz
lograr lo que desea.
D) El escudero. es el criado del caballero andante. 61. Si el precio del dólar aumenta de 3.5 soles a 4.5
E) Lo genérico es lo perteneciente al género. soles. se puede decir que ha habido
A) Dcflación
55. Señale la oración donde se usa incorrectamente B) Un aumento de la tasa de inflación
el pronombre relativo ( que, cual. quien. cuyo). C) Una devaluación del sol
D) Una disminución del tipo de cambio
A) El doctor Sánchez es el médico que atiende a los
E) Una rcvaluación de la moneda nacional
alumnos.
13) El profesor Contreras es quien enseña álgebra. 62. En el país. el valor tic las importaciones de hicncs
es mayor a la de las exportaciones correspondientes,
C) Los postulantcs cuyos nombres se mencionan es-
por lo que hay un déficit en:
tán aprobados.
D) Mi padre es la persona por la cual tengo que in- A) Cuenta corriente D) La balanza de pago,
gresar. 13) Cuenta financiera E) La balanza de servicios
E) Te devuelvo el libro que me prestaste. C) La balanza comercial

63. Suponga que hay muchas empresas vendedoras


56. Señale la oración que usa correctamente el
de hojas ópticas, pero la Universidad Nacional de In-
gerundio.
geniería es la única demandante de ese bien. En ese
A) Te envío un regalo conteniendo un juguete. caso el mercado de hojas ópticas sería un:
13) El salón se llenó de niños gritando. A) Monopolio 13) Monopsunio
C) Estando viendo yo el teatro, se me acercó un des- C) Oligopolio D) Oligopsonio
conocido. E) Mercado libre
D) El hampón huyó, habiendo sido apresado al día
64. Juana de Arco, la "doncella de Orleans", comba-
siguiente.
tió en:
E) Estoy preparando un almuerzo delicioso.
A) La Batalla de Valencia (1094)
13) La Guerra de las Dos Rosas (1453-14H5)
57. No es un verbo irregular,
C) La Guerra de los Cien Años (1337-1453)
A) Amar 13) Contar C) Ir
D) La Primera Cruzada (1 (}96-1 099)
D) Ser E) Tener E) La Octava Cruzada (124H)
65. Señale la alternativa que no establece una rela- Al El Rey León r» PctcrPan
ción correcta entre un personaje del siglo XVII y su B) El Jinete del Espacio El Pokcmon
correspondiente contribución científica. el Hany Poner
Al Descartes: geometría
74. Señale el país que no tiene monurquu constitu-
B) Grimaldi: curvatura de la luz
cional.
el Kepler: ley de los gases
Al España B) Holanda el Inglaterra
D) Népcr: lognriunos
D) Japón El Portugal
El Newton: tcoría de la gravitación universal

66. Complete con la mejor alternativa.


75. Mar en el que se hundió el submarino nuclear
ruso Kursk:
Un es diferente de un "incentivo" porque éste pro-
cede de fuera del individuo. Al Mar Negro D) Mar Tirrcno
B) Mar Cuspio El Mar del Norte
Al Reflejo B) Hábito . el Acto imaginativo
el Mar de Barcnts
D) Instinto El Motivo
76. El "Plan Colombia", irnplcmcntado por el gobierno
67. A la organización integral y dinámica de las ca-
de Andrés Pastruna con el apoyo de Estados Unidos. for-
racterísticas cognoscitivas, afectivas, volitivas y físi-
malmente tiene como su principal objetivo inmediato:
cas del individuo denorrunasc:
Al derrotar a las guerrillas.
Al Personalidad B) Afectividad
B) eliminar el narcotr.ifico.
el Temperamento D) Identificación
el evitar un golpe militar.
El Socialización
D) impulsar el terrorismo.
68. Simbolizar lógicamente la expresión "Juan Pérez El promover la presencia militar norteamericana.
saldrá elegido y será congresista, si y sólo si obtiene
77. Poeta de nacionalidad peruana que ha obtenido
apoyo en su provincia"
recientemente el premio Gabriela Mistral:
AlpHq,r BlP,q--7r el(p/\q)Hr A) Marco Martos D) Mario Flori.in

Dl(pvq)--7r ElP--7(q,r,.I) 131Mirko Luucr El Eduardo Eiclson


e) Antonio Cisneros
69. "Nunca vamos a saber quién ganó realmente las
elecciones", es una afirmación que respecto a la posi- 78. El peruano Cetso Garrido Lccca recibió cl pre-
bilidad del conocimiento, pertenece al: mio Tornas Luis de Victoria al mejor:
Al cantante lírico internacional
Al criticismo B) ernpirismo el escepticismo
B) compositor iberoamericano
D) estoicismo El objctivismo
el ingeniero civil sudamericano
70. Después de Loreto el departamento más extenso D) novelista hispanoamericano
del país es: El periodista de investigación americano
Al Arcquipa I3l Cuzco el Madre de Dios
79. Señale la alternativa que establece la relación co-
D} Puno El Ucayali
rrecta entre el organismo electoral y su respectivo titular.
71. ¿En'qué departamento se encuentra ubicada la Al ONPE- José Portillo Carnpbcll
mayor central hidroeléctrica del Perú? 13) JNE- Celedouio Méndcz Jurado
Al Ancash 13l Ayacucho el Cerro de Paseo el RENIEe- Manuel Sánchcz Palacios
D) Huancavclica El Junín r» ONPE- Fernando Tuesta Soldcvilla
72. ¿eu,íl de los siguientes reservorios hidráulicos El RENIEe- José Ugaz Srinchcz Moreno
está localizado en el departamento de Arequipa? 80. i.eu<Í1 de los siguientes congresistas. que han apa-
Al Pocchos 13l San Lorenzo el El Fraile recido con Vladimiro Moruesinos en los videos divul-
D) Tinajones El Gallito Ciego gados recientemente, ha sido desaforado del Congreso
de la República?
73. Señale el personaje de una serie exitosa de nove- A) Ernesto Gamarra D) Guido Pcnnano
las para ruños, llevado recientemente al cinc y creado B) Alberto Kouri El Luz Salgado
por una ama de casa inglesa. el Carmen Lozada
SOLUCIONARIO
6. En la oración "Antenor elucida la Biblia". la pala-
APTITUD ACADÉMICA
bra ELUCIDA deriva de la palabra "dilucidar" que sig-
ANALOGíAS nifica poner en claro o interpretar,
Clave: C
1. CREATIVO: TRIUNFADOR
CREATIVO y TRIUNFADOR son cualidades referido 7.
a personas que sobresalen en un medio social competi- PLEBISCITO: Decisión de un pueblo tomado por
tivo. de la misma manera que los vocablos INVENTIVO votación general, Su sinónimo votación.
yGANADOK PROBIDAD: Honradez, rectitud e integridad,
Clave: D
INFUNDIO: Patraña, mentira. noticia falsa () em-
2. GENEROSO: EGOÍSTA huste.
Entre ambas palabras existe una relación de antonimia, Clave: B
pues GENEROSO, es aquel que se muestra propenso a
ayudar a los dernas, mientras EGOÍSTA es aquel que
atiende desmedida mente su propio interés, sin cuidar- ANTÓNIMOS
se de los demás, El par análogo lo constituye DADI-
8. FRUSLERÍA,- Cosa de poco valor o entidad que
VOSO: AV ARO que también son antónirnos.
carece de importancia; en consecuencia su antónimo
Clave: B es relevancia o IMPORTANCIA,
Clave: C
3. TEORÍA: PRÁCTICA
Entre ambas palabras existe una relación de antonimia, 9. COLOFÓN,- Nota que se pone al final de un libro.
dado que TEORÍA es el conocimiento especulativo pu- para indicar el nombre del impresor y la fecha en el
ramente racional, independientemente de toda aplica- que se concluyó, su sinónimo es epílogo y sus
ción y PRÁCTICA es la aplicación de un arte o cien- antónirnos son introducción. prólogo. PREFACIO,
cia, De forma similar, ABSTRACTO: CONCRETO son
Clave: B
antónimos,
Clave: A
10. PROBO,- Rectitud. integridad. honradez. hom-
bre de bien, Por lo tanto, en la expresión el antónimo
4. ARQUITECTURA: PLANOS de un hombre probo, honesto, es un hombre venal, DES-
La ARQUITECTURA es el arte de proyectar y cons- HONESTO,
truir edificios sobre la base de PLANOS, de igual ma- Clave: D
nera la MÚSICA es el arte de combinar los sonidos
que pueden ser plasmados en PARTITURAS, 11. CICLÓPEO,- Relativo a los cíclopcs. gigantes de
un solo ojo, en la mitología griega, Su sinónimo es gi-
Clave: E
gantesco, colosal y su antónimo es pequeño, de baja
estatura o PIGMEO,
SINÓNIMOS Clave: C

5. VADEMÉCUM, libro de poco volumen que puede


llevar consigo, y que contiene las nociones más nece-
ORACIONES INCOMPLETAS
sarias de una ciencia o un arte, lo mismo que 12. Entre las frases "no consiste en carecer de"
PRONTUARIO que significa compendio de las reglas y "querer llevar la razón que uno pueda tener hasta las
de una ciencia o arte, últimas consecuencias", existe una oposición: por lo
Clave: C que entre estas frases debe ubicarse una conjunción
adversativa "sino". También de la última frase deduci- 19. Del texto se desprende que no hasta conocer las
mos un estado de locura. Por lo tanto. el texto más ade- causas y los efectos de los problemas umhicntalcs. sino
cuado será: "la locura no consiste en carecer de razón, qUI:!hay que proponer soluciones. Por lo que el tcuut del
sino en querer llevar la razón que lll10 puede tener has- texto es: "Soluciones a los problemas ambientales".
ta las últimas consecuencias". Clave: O
Clave: B
20. En el texto. el autor se expresa cn primera perso-
13. Las expresiones "la realidad se impone" y "la fan- na. DI:! las frases: "me engendraron padres que nucic-
tasía", debe estar unida mediante una preposición que ron aquí". segunda generación. "De padres que cugcn-
indica contrariedad. De las claves. la más adecuada es draron otros padres que nacieron aquí". tercera y cuar-
"a". también al referimos al "camino" debe estar acom- ta generación. "De padres hijos de esta tierra". quinta
pañado por un adverbio de lugar. el más adecuado "don- generación.
de". por lo que el texto más adecuado será: "la realidad Clave: O
se impone a la fantasía, siendo la realidad el camino
donde discurren nuestras vidas". 21. Scgún el texto, la justicia consiste en dar a cada
Clave: E
hombre lo que legítimamente le corresponde. de tomnr
no sólo lo necesario, sino lo cómodo y lo agradable.
14. Frase literaria de A. Vaselovski: "El mérito del
Por lo tanto. la justicia según el texto consiste en: "To-
estilo consiste en uhicar el máximo de pensamientos
mar los bienes que nos corresponde".
en el mínimo de palabras".
Clave: C Clave: E

22. El orden de las oraciones debe estar dado


cronológicarncnte por lo que el texto de he empezar
COMPRENSiÓN DE LECTURA con la fundación de la UNI. (111) luego como se le
15. En el texto. el autor señala que en la escuela nos conocía en su fundación (IV). después cuando curn-
enseñan a redactar dando prioridad excesivamante a la bió a su actual nombre (1) y finalmente corno está cons-
gramática. dejando de lado otros aspectos importantes para tituida hoy (11).
poder redactar bien. De donde podemos concluir que: Clave: O
"Para redactar bien no s610 hay que saber grunuitica".
Clave: E' 23. LOCUCIÓN.- Es una construcción xinuicticu Iu«:
por lo tanto invariable para expresar una misma Idea.
16. Según el texto. el hombre mediante la ingeniería De las alternativas es locución: "Primeramente orcmus
desarrolla formas de utilizar econornicamentc los ma- a Dios".
teriales y fuerzas de la naturaleza para beneficio de la Clave: C
humanidad. Por lo tanto podemos concluir que: "la in-
geniería se ocupa de la organización eficiente de los 24. COLÓQUIO: Plática. conversación. donde xc pre-
recursos" . senta una serie de incorrecciones rolcruhlcs.
Clave.: B
De la expresión: "La abogado se entrevisto con el de-
17. Para el autor del texto, la muerte de Melgar sien- tenido", si bien hay una incorrección en ahogado Iaho-
do aún muy joven. impidió su evolución artística. Por gada). es tolerable en el lenguaje coloquial.
lo tanto. la muerte truncó la evolución urustica. de Clave: C
Melgar.
Clave: A 25. DEFINICiÓN DESCRIPTIVA.- Es aqucllu que
se realiza a través de la enumeración de aspectos () ca-
18. El texto tiene como tema principal la necesidad
racterísticas que tratan de definir al~l): en ellas no hay
de que los médicos tornen en cuenta cl estado emocio-
evolución en el tiempo.
nal de los pacientes y no sólo de proporcionar medica-
De la definición anterior la opción que contiene una
mentos. Por lo cual. es necesario la intervención crno-
definición descriptiva es: "EI hombre es un ser capa/
cional en la atención médica.
de sentir. pensar y resolver situaciones problemáticas".
Clave: O
Clave: B
=:::===:~~I UNI 2001-1 APT. ACADÉMICA Y CULTURA GENERAL I
26. Una buena redacción debe tener un mensaje cla- 31. Busquemos la regla de formación de la serie para
ro. preciso. sin errores de ortografía y de sintaxis. hallar el número que sigue:
Analizando las alternativas: +2 +2
A. " ... Yo g que me deseas lo mejor. .." ~~
Con'cción: sé 18 7 72 9 90 11

B ..•... Me falta solo el último examen ..."' ¡ ¡ ¡


Corrección: s610 1+8
'-y-"
7+2
'-y-"
9+0
'-y-"
9 9 9
C. Mensaje correctamente escrito.
D ..•... Me falta rendir la tercera prueba y confió te- Clave: C
ner éxito".
32.
Correción: confío
E. " .. .Tu sabes cuanto deseo ingresar a la UNI. .."
Correción: cuánto.
Clave: C

27. La frase "alma de Dios", atribuida a una persona,


denota generosidad, candidez y libertad. por lo tanto Clave: A
para expresar que una persona es de fácil o buen trato.
debe decirse: Es un "alma de Dios", un hombre asequi- 33.
ble a todos.
Clave: C
~ ~~'--=-../'--=-../
28. SEMÁNTICA.- Ciencia que trata de la significa-
2 2 4 12 48
ción de las palabras. ~~~~
En todas las claves, aparece la palabra incólume que xl x2 x3 x4
significa sano, sin lesión, ni menoscabo. Por lo tanto la
Clave: O
oración más apropiada será: "Del accidente salió incó-
lume".
Clave: B

29. El conjunto de textos. trata de una tradición. del SERIE DE FIGURAS


uso de las pelucas en la cámara de los comunes y de
34.
como su presidente rompe ésta tradición (3). recono-
ciendo el respeto por las tradiciones (2), y contrastan-
do, el respeto a la tradición con el perjuicio que ésta
trae (4). finalmente concluye con el perjuicio que trae.
aferrarse a la tradición ( I ).
u n e; ~ U 3-
Por lo tanto el orden será: 3, 2, 4, l. La figura U gira 90" en sentido antihorario:
Clave: A

SERIES NUMÉRICAS u ~nc


La figura' gira 45" en sentido antihorario:
30. Analicemos la sucesión para encontrar el número
que sigue:
5 . 10 . 25 . 60 : 125 . 230 /
~~~~~
5 15 35 65 105 Finalmente:
~~~~
10~0~0~40 El casillero U I es ocupado por: 3-
10 10 10 Clave: E
Clave: B
35.

-, [JJ5J Anñlogamcntc:

II

/ )/

Analizando
-, m CtJ
J

los casilleros:
La figura que ocupa el casillero

borrado la mitad.
UNI es: ~
~
Clave: B

38. En el cuadro I la figura se ha girado I XO° y luego

~+[JJ 5J -
-

+ / )/
r -
-

~+rn CtJ -
-
Análogamente:

La figura que ocupa el casillero UNI es: 1'" I


Clave: C

36. Como se puede observar. en la serie gráfica, cada


cuadrado sombreado desciende sobre la columna, se- La figura que ocupa el casillero Ur:J1 es: ~
gún el lugar que ocupan, el primero desciende un casi-
llero. el segundo dos. el tercero tres. el cuarto cuatro y Clave: B
el quinto cinco casilleros, teniendo en cuenta que cuan-
do llegan a la última fila vuelven a empezar por la pri- 39. Los 4 casilleros sombreados en el exterior del
mera fila. cuadrado. convergen hacia el centro a lo largo de la
horizontal y vertical: luego retornan por el mismo <.:;1-
mino. Los cuatro casilleros sombrcados de las esqui-
nas del cuadrado convergen hacia el centro sobre las
diagonales del cuadrado y luego retornan pm cl mixrno
Clave: D camino.

37. En el cuadro I las figuras son las mismas, s610


están en posiciones diferentes.

Clave: A
~~~
trtili trt=§
::i~~~=l UNI 2001-1 APT. ACADÉMICA Y CULTURA GENERAL ~

40. Todas las partes sombrcadas de la figura van gi- Sumando: (6) + (7):
rando 90" en sentido horario, excepto la del centro que
a+2=x+e
aparece y desaparece.
h+2=x+d
a +/}+4 = Tx+c+d
o+b+x+4=
'-----.r----' 2x+x+e+d
_
IR 7
A
=> x=5
Clave: A
Hay 5 banderas bicolor rojo-azul.
Clave: C
RAZONAMIENTO LÓGICO
42. Datos:

¡
Y MATEMÁTICO
• Asistieron: 1000 personas ... (1)
Monócromas: y
41. Datos: • Costo de las entradas:
Bicolor: 2y
1.. 35 banderas Trieolor: 8 Solos: S/IO.OO ~ x personas ... (2)

/ La pareja: SI 15.00 ~ Y parejas ... (3)


Amanllo: 11

2.- Número de banderas


que tienen el color
j Rojo:
Azul:
11

11
• Recaudación: S/82S0.00

De (2). (3) y (1 ) se tiene:


... (4)

x+2.l' = 1000 ...(5)

3.· Banderas rnonócrornas de color amarillo: 2 10x+ 15)' = 82S0 ...(6)

Representación gráfica: De (5) en (6):

10(1000- 2y) + ISy = 8250


=> Y = 350
Reemplazando" y" en (4):

x+2(3S0) = 1000
=> x = 300

Amarillo En pareja fueron 700 personas y solos fueron ]00.


La diferencia de personas que fueron en pareja y solos
De la condición 1 se tiene:
será:
)"+2)'+8= 35
700 - 300 = 400
=> y= 9 Clave: E

=> d+e+2=9 43. Datos:


• El reloj se adelanta 75 segundos por hora
d+e= 7 ... (4)
• Tiempo transcurrido (6:00 am a 8:00 prn): 14 h.
=> (/+b+x=18 ... (5) El reloj se adelanta:

De la condición 2 se tiene: 14x7Ss= 10SOs

... (6) 1050" = 17'30"


a+2=x+e
Finalmente, cuando realmente son las 8:00 pm el reloj
b+2=x+d ... (7)
marcará 20: 17' 30" .
d+a=e+b ... (8) Clav-e: A
0:::=:=:=l UNI 2001-1 APT. ACADÉMICA Y CULTURA GENERAL I"::=;:=;;;~111.:1
44. Datos: Reemplazando tenemos:

f: 1
Páginas Ilustraciones
Historias de ficción: 10x 12x
Historias reales: IOy 11)' If¡O= !-I+2[
Total: 700 810 1--
S
Resolviendo: 10x+ 10y = 700 ... ( 1) ~ !-I=20clII

...(2)
Clave: A
12x+ Ily =810
De(l)y(2): x=40; y=30 47.
Luego.la diferencia de ilustraciones: Nombres P.J. P.G. P.E. P.P. Puntos
12( 40) -11(30) = 150 Juan 6 6 O O 12
Hay 150 ilustraciones más de un tema que de otro. Carlos 6 5 '0 I lO
Clave: B Javier 6 3 1 2 7
45. La distancia total será: Eduardo 6 2 O 4 4
José 5 1 2 2 4
Martín 5 I O 4 2

+ +r 27+(+)\~+ ...]
Pedro 6 O 1 5 1

=27+; [~7+( )27+( De la labia observamos que:


L (serie "infinita) l. José empató dos partidos y fue ante Javier y Pc-
dro, puesto que son los únicos ~ue han empata-
do.
=81
2. José perdió ante Juan y Carlos puesto que .1uan
Finalmente. la distancia total recorrida será 81 cm. ganó a todos y Carlos sólo perdió con Juan.

Clave: D 3. José tiene un partido ganado que necesariamente


es con Eduardo puesto que aún le íultajugnr con
Martín.
46.
H: altura inicial Clave: D

48. Del enunciado se tiene que los apellidos no coinci-


den con sus oficios, por lo tanto hay dos posibilidades:
l. Si el Sr. Pintor es albañil. entonces el Sr. Cal'
!-I
piruero es pintor y el Sr. Albañil es carpintero.
1/1
5 2. Si el Sr. Pintor es carpintero, entonces el Sr. ClI'-
pintero es albañil y el Sr. Albañil es pintor.
Analizando las proposiciones en:
(1) (2)

El recorrido total será: IF I V


II F II V
111F III F
IV V IV F
Serie infinita decreciente de razón Finalmente, sólo cumple con la condición del prohic-
ma (1) siendo IV la proposición verdadera.
4/5 cuyo equivalente es: {/.¡I )
( Clave: D
1-"5
~.~~ ""'~~l
GomeZ
UNI 2001-1 APT. ACADÉMICA Y CULTURA GENERAL I==::"=~ ~

49_ Del enunciado tcncncmos: y 1930. se firmó el Tratado de Lima por Rada Gamio y
Figucrou Larrui n. en el cual se establece el retorno de
Animal Declaración Posibilidades Tacna al Perú y la pérdida definitiva de Arica en favor
de Chile.
Gusano El gato se comió la sal V F F
Clave: C
Gato Eso no es cierto F V V
Murciélago Nunca comí la sal V F V 53. Andrés de Santa Cruz fue nombrado en el Con-
greso de Tacna, como Protector de la Confederación
De la tabla observamos que hay tres posibilidades que formada por tres Estados Nor-Peruano. Sur-Peruano y
cumplen con la condición del problema. Boliviano.
Clave: E Clave: B

50. De las premisas: 54. Definición.- Proposición que expone con clari-
• Todos los cerdos vuelan dad y exactitud su género próximo y su diferencia es-
• Ningún cerdo tiene cola pecífica de un término. Por lo tanto es correcta la dcfi-
nición de: La lepra es una enfermedad infecciosa cró-
Tenemos:
nica (género próximo) producida por el bacilo de
Hansen. (diferencia específica).
Clave: B

55. Analizando las oraciones. la única que usa inco-


rrectamente el pronombre relativo es: Mi padre es la
persona por el cual tengo que ingresar.
Se presenta una discordancia de género entre "la per-
Analicemos las conclusiones: sona" y "el cual", debe decir: "la persona" y "la cual".
1. No todos los cerdos tienen cola. Clave: D
Falso, esta conclusión contradice la premisa de que
ningún cerdo tiene cola. 56. Gerundio.- Forma verbal invariable del inl'initivo.
11. ingún animal que vuela tiene cola. que denota la acción del verbo en abstracto. y por lo
Falso, no podemos concluir ésto, puesto que no hay común como ejecutándose en presente: aunque puede
información al respecto. referirse a cualquier tiempo. En tal sentido la oración
111. Existen animales sin cola que vuelan. que utiliza correctamente el gerundio es: Estoy pre-
Verdadera. podemos concluir esto, ya que hay parando un almuerzo delicioso.
cerdos sin cola. que vuelan. Clave: E
Clave: C
57. Un verbo es irregular cuando su ruiz sufre cam-
bios fonéticos en la conjugación. Analicemos las cla-
CULTURA GENERAL ves:
Amar contar ir ser tener
51. El Virrey Pedro Fernández de Castro, 'Conde
Am-o cuent-o voy soy teng-o
de Lemos, en el siglo XVII. cerró varias minas, debido
regular irregular irregular irregular irregular
al trabajo esclavizante de los indígenas, que eran
rcclutados de los pueblos llamados Reducciones. Clave: A
Clave: B
58. Analicemos las alternativas:

52. En el Tratado de Ancón se establecía el desarrollo A) Andrés se empeña por ingresar a la universidad.
de un plebiscito que definiría la situación de los depar- Se debe utilizar en y no a.
tamentos de Arica y Tacna; contraviniendo este trata- B) Carmen mantiene su negativa para enseñar.
do durante el gobierno de Augusto B. Leguia entrc 1919 Se debe utilizar a y no para.
0'= UNI 2001-1 APT. ACADÉMICA Y CULTURA GENERAL I~::::;:~:::!::;: eZ
--=="'-"
C) Diana no tiene prisa en salir de la academia. 64. Juana de Arco, conocida corno la "doncella de
Se debe utilizar por y no en. Orletins", campesina francesa, combatió en la Guerru
D) Luis insiste en estudiar geometría. de los Cien Años (1337-1453), entre los franceses e
Se usa correctamente la preposición en. ingleses por la posesión de los feudos de Guyena y
Flandes, en 1453 mediante la capitulación de Burdeos
E) Pablo tiene preferencia de los cursos de letras.
Francia, recuperó su territorio ocupado por Inglaterra
Se debe utilizar por y no de.
terminando así la Guerra.
Clave: D
Clave: C

59. En la obra "Cien Años de Soledad" se relata la


historia de los Buendía a traves de sus siete generacio- 65. Johannes Kepler, científico que en el siglo XVII
nes, cuyo autor es el colombiano Gabriel García sobresalió por sus estudios de astronomía establecien-
Márquez. do las leyes de la órbita y no la ley de los gases.
Clave: C Clave: C

60 .• Miguel Angel Asturias, Premio Nóbel 1945. 66.


• Gabriela Mistral, Premio Nóbel 1967. • Incentivo: Lo que incita o mueve el interés en un
individuo.
• Jorge Luis Borges, éste autor a sido propuesto
en varias ocasiones, pero no ha recibido el Pre- • Motivo: Dícese de la causa impulsiva. de la que
mio Nóbel, determina la vo luntad. Por lo tanto la ex-
presión quedaría así: Un motivo es diferente de
• Camilo José Cela, Premio Nóbel 1989.
un "incentivo" porque éste procede de fuera del
• Octavio Paz, Premio Nóbel 1990.
individuo.
Clave: C
Clave: E
61. Si el precio del dólar aumenta de 3.5 soles a 4.5
67. La personalidad es la diferencia individual que
soles, podemos decir que el sol se ha devaluado o ha
distingue una persona de otra y en su estructura irucgru
habido una pérdida del valor de la moneda nacional.
los procesos: cognocitivos (formas de pensar y pcrci-
Clave: C bir la realidad), atectivas (formas de reaccionar CIlHl-
cional y sentimentalmente), volitivas (formas de oricn-
tal' y dirigir la actividad) y físicos de manera din.imica
62. Si en el país, el valor de las importaciones de bie-
nes es mayor al de las exportaciones correspondientes, Clave: A
se dice que hay un déficit en la balanza comercial.
Clave: C 68. Simbolizando:
• P : Juan Pérez saldni elegido
63. Analizando las alternativas. • q : Juan Pérez será congresista
• Monopolio, un vendedor muchos compradores. • r : Juan Pérez obtiene apoyo en su provincia
• Monopsonio, muchos vendedores un comprador.
En la siguiente expresión lógica:
• Oligopolio, pocos vendedores y muchos compra-
dores. "Juan Pérez saldrá elegido y será congresista,
"---v-----' ~ '--------v-----
Oligopsonio, muchos vendedores pocos compra- P /\ q
dores. si y sólo si obtiene apoyo en su provincia"
'---v---' . .
• Mercado libre, muchos vendedores y muchos com- H r
pradores (se regula por la oferta y la demanda).
Del enunciado, muchas empresas vendedoras de hojas Sirnbolicarncnte: (p/\q) H r
ópticas, pero la UNI la única compradora, por lo tanto Clave: C
el mercado de hojas óptica sería un Monopsonlo,
Clave: 8
~~~~~"~. """'2;:J. UN12001-1 APT. ACADÉMICA Y CULTURA GENERAL I'':=~~;;
69_ Analizando I;\'; alternativas: 74. La República de Portugal con su capital Lis-
A) Criticismo.- Sostiene que sólo conocemos la for- boa, es el país que no tiene monarquía constitucional.
ma en que se nos aparece la realidad. pero no su Presidente de la República es Jorge Sampaio.
podemos conocer la realidad en sí misma (Kant). Clave: E
B) Empirismo.- Sostiene que la fuente de todo co-
nocimiento es la experiencia."Nada hay en el en- 75. El submarino nuclear ruso Kursk, se hundió en
tendimiento que no haya estado antes en los sen- las aguas del Mar de Barents, al norte de Rusia.
tidos" (Locke).
Clave: C
C) Escepticismo.- Sostiene la imposibilidad del co-
nocimiento en general, restringiéndole o cuestio-
76. El "Plan Colombia" fue creado por la secretaria
nándole (Pirrón).
de Estado M. Albright e irnplcmcntado por el gobierno
D) Estoicismo.- Sostiene una moral fatalista y de de Andrés Pastruna con la finalidad de eliminar el
resignación (Séneca).
narcotráfico en la región.
E) Objetivismo.- Doctrina filosófica sostenida por Clave: B
Platón en la cual el ámbito axiológico gnoseoló-
gico otorga plena primacía al objeto en relación 77. El premio "Gabriela Mistral'' en el año 2000 fue
al sujeto. -
concedido al escritor peruano Antonio Cisncros por
De las definiciones tenemos que la afirmación del enun- sus trabajos literarios como"Contra un oso hormigue-
ciado respecto a la posibilidad del conocimiento perte- ro", "Canto Ceremonial", cte.
nece al escepticismo. Clave: C
Clave: C
78. Celso Garrido Lecca recibió el premio Tomás
70. Después de Loreto el departamento más extenso Luis de Victoria al mejor compositor iheroamerjcn-
del país es Ucayali, con un área territoreal de 97 86~ no debido a su destacada labor de músico y cornpoxitor
Km2 el cual fue creado en Julio de 1980. de música clásica.
Clave: E Clave: B

71. La mayor central hidroeléctrica del Perú es la cen- 79. Fernando Tuesta Soldevilla es el actual jefe de
tral del Maruaro, que está ubicado en el departamento la ONPE, sustituyó a José Potillo Carnpbcll. que fue
de Huancavelica. destituido por sus vínculos con el ex-asesor Vludimiro
Clave: D Montesinos.
Clave: D
72. Los reservorios de:
80. El congresista Alberto Kouri fue desaforado del
• Poechos y San Lorenzo se enuentra en Piura.
Congreso de la República. por aparecer en un video
• El Fraile se ubica en el curso del río Chili en con Vladimiro Montesinos Torres, cn un acto de co-
Arequipa. rrupción.
Tinajones en Chancay. Clave: B
• Gallito Ciego en el río Jequetepeque.
Clave: C

73. Harry Potter fue creada por:


La ama de casa inglesa J.K. Rowling cuyos títulos son:
• Harry Potter y la piedra filosoful.
• Harry Potter y la cámara secreta.
• Harry Pouer y el prisionero de Azcabán.
Clave: C
0::=:::!:=~1 UNI 2001-11 APT. ACADÉMICA Y CULTURA GENERAL 1:~:=::;:::;:::~1~::;

APTITUD ACADÉMICA ANALOGíAS


SINÓNIMOS
6. Marque la alternativa análoga.
1. Marque el sinónimo de la palabra subrayada, to- CONSTITUCiÓN: LEY::
mando en cuenta el significado que tiene en la oración.
A) País capital
"No deben arrugarse frente a los problemas".
B) Sujeto: cualidad
A) amilanarse B) arriesgarse C) arrobarse
C) Ley decreto supremo
D) arrojarse E) arroparse
D) Causa: efecto
2. Marque el sinónimo de la palabra subrayada, to- El Tela traje
mando en cuenta el significado que tiene en la oración.
7. Señale la palabra que no tiene el mismo significado
"Aquel juez fue sancionado por desacato".
de las otras.
A) arbitrariedad D) modoso
A) culto D) unción
B) desleal E) pusilánime
B) fervor El veneración
C) insubordinación
C) sentimiento

ANTÓNIMOS 8. Marque la alternativa que corresponda.


CONJURO CONJURA ::
3. Marque el antónimo de la palabra subrayada, to-
mando en cuenta el significado que tiene en la oración. A) masculino femenino

"No lo condenaron, porque su falta fue consecuencia B) ruego juramento


de un reacción maquinal". C) agregado intriga

A) consciente Bl enredada C) individual D) exorcismo complot


D)_libre E) manual E) maniobra hechizo

4. Marque la alternativa del término que no es antóni-


mo de: PETULANTE COMPRENSiÓN DE LECTURA
A) afable B) sumiso C) respetuoso
9. "Analizar el impacto de las reformas en la inver-
D) cortés E) reverente sión en América Latina es una tarea compleja, porque
muchos procesos tuvieron lugar ximuluincnrneutc. Ade-
5. Señale el antónimo que corresponde a la palabra
más, las reformas tuvieron efectos diferentes a lo largo
subrayada en el texto.
del tiempo. Los estudios sobre las experiencias de al-
"La carroza y la sección servidumbre, ejercieron siem- gunos países sugieren la necesidad de distinguir un 1)(;-
pre una extraña fascinación sobre Julius". . riodo de transición de uno consolidado, con el fin de
Al desasimiento B) liberación C) aversión entender el eventual impacto de las reformas, casi to-
D) repelús El retención dos los países están aún en periodo de transición".
""1 UNI 2001-11 APT. ACADÉMICA Y CULTURA GENERAL r~::::,'~'0
Del texto se deduce 'quc: tan como lo hicieron las dos Alcmanias y como están
A) Las reformas han tenido un efecto positivo en comenzando a haccrlo las dos Corcas y las diversas
América Latina. Chinas. Las sociedades unidas por las ideologías o las
circunstancias históricas. pero divididas por la civili-
B) Los efectos diferentes de las reformas permiten
zación, o 'se des-hacen (como la Unión Soviética. Yu-
derivar conclusiones generales sobre el impacto
goslavia y Bosnia) () están sometidas a una gran ten-
de las mismas.
sión. como es el caso de Ucrania. Nigeria. Sud.in. In-
C) Todavía no es posible llegar a conclusiones defi-
dia. Sri Lanku y muchas otras".
nitivas sobre el impacto de las reformas.
De acuerdo al texto. el fin de la guerra fría
D) El periodo de transición de las reformas no son
afectadas por los otros procesos sociales. A) consolida la paz mundial.

E) Los efectos similares de las reformas asegu- B) disminuye la posibilidad de guerras civiles.
ran el éxito de las mismas. C) iguala el significado de los conceptos civiliza-
ción y cultura.
10_ "Por definición, Utopía es lo que no tiene lugar ni D) provocará la división de la India.
en el espacio ni en el tiempo. Pero en los Andes, la
E) reduce la importancia geopolítica de las ideolo-
imaginación colectiva terminó ubicando a la sociedad
gías.
ideal-en la etapa histórica anterior a la llegada de los
europeos. Una historia de rnilenios que identificada con
la de un imperio, y un mundo en el que existieron des-
igualdades e imposición, se convirtió en una sociedad
ORACIONES INCOMPLETAS
homogénea y justa".
De acuerdo al texto, el Imperio lncaico 13_ Marque la alternativa con el término que comple-
ta adecuadamente la oración.
A) duró milc-, de años.
"Debido a la corrupción, en los tribunales no se .
B) ha sido idealizado en los Andes.
justicia".
C) impuso una mayor desigualdad que los europeos.
A) compartía D) repartía
D) terminó siendo una sociedad justa.
B) impartía E) suministraba
E) tuvo características utópicas.
C) proporcionaba
11 _ "En el principio creó Dios los ciclos y la tierra.
La tierra era confusa y el espíritu de Dios se cernía 14. Complete la siguiente oración:
sobre la haz del abismo. Dijo Dios" Haya luz ", y hubo La UN I al comenzar el siglo XXI reafirma su
luz. Vio Dios que la luz era buena y apartó la luz de la de formar personas capaces de sentir. valorar. inter-
oscuridad. Y llamó Dios a la luz "día ", y a la oscuri- pretar y el mundo de la existencia individual
dad la llamó "noche". Y atardeció y amaneció; día pri- así como el social y colectivo.
mero. Génesis 1-5.
A) responsabilidad realizar
¿Cuál es la idea principal del texto?
B) compromiso transformar
A) En el primer momento de la creación. la tierra C) postulado desear
era caos.
D) acuerdo reducir
B) Hay un solo Dios.
E) declaración mantener
C) El mundo no fue creado en un solo acto.
D) Dios hizo la luz- 15. Marque la alternativa con el término que comple-
E) Dios creó el mundo. ta adecuadamente la oración.
"Lo condenaron a cadena perpetua, sobre la base de
12_ "En el mundo de la postguerra fría. la cultura es a pruebas contundentes y de acuerdo a ley. Los jueces
la vez una fuerza divisiva y unificadora. Gentes sepa- han actuado con ".
radas por la ideología pero unidas por la cultura se jun-
0;wrV:::~=1 UNI 2001-11 APT. ACADÉMICA Y CULTURA GENERAL li;:=~=;::l~:;:~!~;i
A) rectitud D) indignación 19. Marque la alternativa que tiene los términos que
B) indulgencia E) severidad completan adecuadamente la oración.

C) parcialidad "Un mundo le es dado al hombre; su gloria


es soportar despreciar este mundo.
enriquecerlo construyendo universos"
16. Marque la alternativa que contiene los términos
que completan adecuadamente el texto. A) entonces - o sin
B) no - o sino
"Montesquieu comparte con Locke de haber
acabado con muchas de ideas que hasta enton- C) no - y aunque
ces reconstruir la sociedad humana". D) por tanto - y pero
A) la infamia - las buenas - habían permitido E) siempre pero no
B) la etapa - nuestras - impedían
20. Marque la alternativa que tiene los términos que
C) el esfuerzo - las recientes - facilitaban completan adecuadamente la oración.
D) la fama las falsas - permitían
"Faltaban 3 días para las elecciones Tomás no
E) la gloria las falsas - habían impedido sabía votar por un candidato el otro.

A) y - si o
B) pero tampoco con
CONECTORES C) por eso ni o

17. Complete el texto con la alternativa que corres- D) aunque - entonces u


ponde. E) ya que si u
El ex-presidente Clinton dio este lunes en Jaipur
(India) el paseo a lomo elefante que había sido CONCORDANCIA
negado •........... razones de seguridad, durante una vi-
sita oficial hace tan sólo un año. 21. Marque la alternativa que tiene los términos que
completan adecuadamente la oración.
A) durante de - vista las
"Se puso camisa, ..corbata y saco
B) recién en - con
que le regalamos".
C) al fin de sobre- desde
A) la - el - el D) su - la - el
D) por fin de por
B) la - la - el E) su - su - su
E) a partir de de un - debido a C) su - el - el

18. Complete el texto con la alternativa que corres-


ponde. 22. Complete el texto con la alternativa que corres-
El mimo vuelve su plaza. Un día se fue del Perú ponde.
sin avisar. su público no supo más de él hasta A los 19 Gran Maestro Internacional. Parecía
hace unas semanas, apareció en la plaza des- estar encaminado a ser uno de los grandes en el depor-
pués de veinte años de ausencia. te de los trebejos. Su alarmó a muchos. El .
A) a Y cuando dice: Es mi opción de vida y deben respetarla.

B) por Cuando entonces A) pudo ser trayectoria repl ica y


C) cerca a Pero y B) se convirtió en retiro sonríe y
D) pronto a Si bien Sólo C) dejó de ser derrota a caso
E) también a Y como pronto D) pensó ser victoria siempre
E) debió ser ausencia dudando
I UNI 2001-11 APT. ACADÉMICA Y CULTURA GENERAL I
COHERENCIA

23. Dé usted el orden lógico para lograr una lectura SERIES DE FIGURAS
adecuada al conjunto.
26. Indique la alternnriva que corresponde al des-
l. La lluvia probablemente tuvo algo que ver. pero
pliegue mostrado.
no puede haber sido la única razón del derrum-
bamiento.
2. Habitualmente es punto de atracción para los tu-
ristas. La policía ucordonó la zona.
3. La muralla fue construida entre los años 271 y
275 para defender Roma. Se desplomó en las últi-
mas horas del domingo.
4. La policia dijo el lunes que nadie resultó lasti-
mado.
Al4 -2 -3 - I Bl3 - 2 - 4 - I Cl3 - I -2 -4
D) I - 3 - 2 - 4 E) I - 4 - 3 - 2
A B C o E
24. Ordene los textos de una manera lógica teniendo
27. Las figuras

de los
cuadros I y II cumplen una
en cuenta el título general que se propone.
misma relación; identifique dicha relación e indique la
"Los escritores menores"
'alternativa que debe ocupar el casillero UN!.
!. De ellos podemos aprender algo.
I!. Estas trampas
II!. Trampas
no son visibles
literarias,
IV. Hay autores que no son grandes
V. Se les suele llamar "menores".
por ejemplo.
en los grandes.

escritores.
I~~I~I~I
~~I~~
Al 1- 111- 11- V - IV DlIV-V-I-III-11
B) 11-111 - V -IV - I El V - IV - I -111 - 11
C) 111- 11- IV - I - V
A B C o
25. Usted disertuni sobre "La corrupción en el Perú y
cómo supcrarla". Ordene los subtítulos de su confe-
28.EI dibujo adjunto. es una vista desde arriba. ¡,A
rencia.
cual de las siguientes figuras gcométricas no represcn-
l. El camino de la corrupción en las últimas dé- ta'!
cadas:
2. Condiciones que favorecen
hace muchas generaciones.
3. El marco mundial
del problema.
4. Medidas urgentes
la corrupción

- Antecedentes

para poner en practica


desde

y definiciones

las pro-
00
Al A un cilindro dividido
tes,
diagonal mente en dos par-

puestas generales. B) A un cilindro dividido transversal mente en dos


5. Proyectos y perspectivas para resolver el proble- partes.
ma en lo moral. social. jurídico y político.
C) A dos esferas.
Al 3 - 2 - I - 5 - 4 B) 3 - I - 2 - 4 - 5
D) A una esfera partida en dos partes iguales.
C) 4 - I - 3 - 2 - 5 D) I -2-3-5 -4
El A dos cilindros.
El I-4 - 3 -2 -5
29. Indique la alternativa que corresponda al desplie-
.9 2N
gue de una pirámide de base cuadrangular.
~ 15 3 10 3

11' 1if :tf id ~ ~


A)21 B)22
~ 12 6

C) 23 D) 24
~ M 3

E) 25

30. Si en los tres lados no visibles del cubo adjunto 35. Determine el valor de P + Q en la serie propuesta.
hay: un cuadrado al costado del circulo blanco. un trián-
2,6,4. <), 7,12,11. 15. P, Q
gulo negro al costado del circulo negro. además el nú-
mero 3. ¿Cuál es la alternativa correcta? A) 24 B) 30 C) 34 D)44 E) 46

A)I,D~G B)L.~~
C)I,~B 36. Identifique
mero Ialtante.
la secuencia numérica y defina el nú-

1, 5, 'l. 57, 121. 221

A) 16 B) 36 C) 21 D) 24 El4lJ

37. Indique la alternativa que continúa la serie.


31. Identifique entre las alternati vas dadas el lado -2. 2, 18, 52, 110.
"x" de la figura adjunta en la que se cumple una serie
gnifica. A) 152 B) [<)8 C) 218 Dl252 El2lJX

~A¡g¡B~C¡§ 38. Indique la alternativa


2, 10.
que continúa
24. 44.
la serie.

A)64 B) 70 C) 90 r» <)4 El 104


~D~E~
39. ¿Cu.íles son los números faltantes en la serie nu-
mérica siguiente?
32. Indique la alternativa que contenga a todas las
piezas mostradas. 4 8 12
? 42 63
~ • +IPiezas
~ V mostradas ? 72 108

+++ ~ ••••
A B C D E
All6y36
D) 21 y 36
B)21y30
El 24 y 36
C) 24 Y 4X

40'. Las dos supeficies no visibles de la figura adjun-


SERIES NUMÉRICAS ta. siguen una misma serie numérica ¡,Cuáles son los
números de la fila inferior de la superficie "z" '!
33. Indique la alternativa que debe ocupar el casille-
ro UN!.

34. Los números considerados en los tres cuadros,


cumplen una misma relación: identifique dicha rela-
ción y determine el valor de M + N.
I UNI2001-11 APT. ACADÉMICA Y CULTURA GENERALl~~-=:0
A) 18, 17,22 13) 22. 23. 26 C , . 23, 28 47. Si una persona camina Z ccntirnctros por segun-
D) 21, 26, 25 E) 21, 23, 28 do, ¡.cuántos metros caminará en Q horas?

ZQ ZQ
RAZONAMIENTO LÓGICO
A) 3600 ZQ B) 60 C) 360000
D) 36ZQ E) 360 ZQ
41. Claudia y Pnuicia no sabian nadar. Por descuido,
dijeron, cayeron al mar. ¿Cuál es la frase más lógica?
48. Un grupo de amigos decidió realizar una camina-
A) Ambas se ahogaron. ta de cinco días de duración, con la intención de reco-
B) Una logró salvar a la otra, falleciendo después. rrer siempre la misma distancia cada día.
C) Ambas se salvaron, pues el mar no era profundo. El nrirner día recorrieron el 80% de la distancia fijada.
D) Recibieron auxilio y se salvaron. el segundo día recorrieron el 70% de la misma distan-
E) No se puede precisar cómo, pero se salvaron. cia, el tercer día el 60% de ella y el cuarto día el 40%
de la misma. Si al final de la caminata sólo cubrieron el
42. Manuel, Juan, Enrique, César y Víctor son her- 60% de la distancia total, ¿qtlé porcentaje de la distan-
manos. Si Manuel es mayor que Juan, Manuel es me- cia fijada recorrieron el último día?
nor que Enrique, Víctor es meyor que Enrique y Víctor A) 50% B) 52% C) 58% D) 64% El 66%
es menor que César. ¿Cuál es el mayor de todos los
hermanos?
SUFICIENCIA DE DATOS
A) Manuel B) Enrique C) Víctor
D) César E) Juan 49. Determinar la edad de un niño, si se tiene la si-
guiente información.
43. La hija de la hija del tío de mi padre, es mi: 1. Tiene la sexta parte de la edad del padre.
A) Sobrina B) Hermana C) Tía 11. El padre tenia 42 años hace 6 años
D) Abuela E) Prima Para responder el problema:
A) La información I es suficiente.
RAZONAMIENTO MATEMÁTICO B) La información II es suficiente.
44. A una asamblea de padres de familia asisten 240 C) Es necesario emplear ambas informaciones.
personas, de las cuales, las madres representan el 70% D) Cada una de las informaciones, por separada, es
de los asistentes. Si deseamos que el número de varo- suficiente.
nes represente el 40% del total de asistentes. ¿Cuántas E) La información brindada es insuficiente.
parejas deben llegar a esta asamblea? .
A)IIO B)120 C) 130 D) 136 E) 140 50. Si M, N y Z son números enteros, determine el
número mayor, si se tiene la siguiente información:
45. Un cuadrado cuya área es a2, ha incrementado
I.M>ZyN>Z
su lado en un 'quinto de su medida inicial. ¿En cuánto
11. (M + 1) > N
se ha incrernentado su área?
2 Para responder el problema:
92
A)-(-I 112
B)_o- 122
C)_o_ 92
D)~ E) 110
25 25 25 5 5 A) La información 1 es suficiente.
B) La información 11 es suficiente.
46. Una empresa siempre ofrecía descuentos sucesi- C) Es necesario emplear ambas informaciones.
vos del 20% y del 15% en la venta de sus productos;
D) Cada una de las informaciones, por separada. es
pero decidió efectuar un único descuento equivalente
suficiente.
a los que ofrecía. ¿Cuál es el valor de este nuevo des-
E) La información brindada es insuficiente.
cuento?
A) 17.5% B) 22,5% C) 28%
D) 32% E) 35%
0:~w:=~ UNI 2001-11 APT, ACADÉMICA Y CULTURA GENERAL I;;,i:::;:;::::~;~::;::!~l;~
'"
56. Una de las siguientes citas no corresponde a la
obra mencionada.
CULTURA GENERAL
A) "Canta, [oh Diosa!, la cólera del pélida Aquilcs".
51. Señale la alternativa donde las 3 palabras tienen lliada (Hornero)
hiato. B) "En medio del camino de la vida. errante me en-
A) desear teología - cloaca contré por selva oscura".
B) beodo biología - baile Divina Comedia (Dante)

C) caer audacia - jueves C) "Ser o no ser: he ahí el dilema".


Macbeth (Shakespeare)
D) ahora - fuerza horario
D) "Al despertar Gregario Samsa una mañana, tras
E) mueca - cuentas muelle
un sueño intranquilo.

52. ¿Cuántos errores ortográficos hay en el texto que La metamorfosis (Kafka)


sigue? • E) "Raskolnikov permaneció largo tiempo acosta-
do. A veces parecía despertar
" El impacto de las Reformas económicas enprendidas
Crimen y Castigo ( Dostoievsk i)
por los países de América Latina y el caribe en las últi-
mas dos décadas, se encuentra en el centro del débate
57. ¿Cuál es el comentario correcto sobre las siguien-
sobre las politicas económicas de la región". tes obras literarias: "La casa verde", "No me esperen
Al4 B)S C)7 D)6 E) 8 en A'bril", "Conversación en la Catedral", "Un mundo
para Julius?
53. En cada alternativa se escribe una palabra de ma- A) Son las mejores creaciones de Julio Ramón
nera correcta e incorrecta. Marque la alternativa donde Ribeyro.
la incorrecta está entre paréntesis. B) Son los mejores cuentos de Alfredo Bryce y Mario
Al aereolínea taerolínea) Vargas Llosa.
B) beneficiencia (beneficencia) C) Son todas obras de Allredo Bryce
C) cónyugue (cónyuge) D) Son novelas de Mario Vargas L10sa y Allredo
Bryce
D) costipado (constipado)
E) Son novelas de Julio Ramón Ribeyro, Vargas
E) dentífrico (dcntrffico) L10sa y Alfredo Bryee.

54. Escoger las oraciones cuyos significados son equi- 58. ¿Con qué países el Perú ha resuelto la delimita-
valentes. ción de sus fronteras?
l. Poco a poco, el poder adquisitivo se deterioró en A) Ecuador y Bolivia
el país. B) Chile y Colombia
2. El poder adquisitivo, por poco, deterioró el país. C) Colombia y Bolivia
3. El país se deterioró poco por ~I poder adquisitivo. D) con todos sus países fronterizos
4. El poder adquisitivo en el país, se deterioró poco E) Bolivia y Brasil
a poco.
59. Una de las fases siguientes no corresponde a una
S. El país poco a poco, deterioró el poder adquisi fase de la Luna. ¿ Cuál es ')
tivo.
A) Luna llena D) Cuarto menguante
A) Ira y Sta B) 2da y 4ta C) 2da y 3ra
B) Media luna E) Cuarto creciente
D) 3ra y Sta E) Ira y 4ta C) Luna nueva

55. ¿ Cuál es la palabra con error ortográfico? 60. El departamento que actualmente es el principal
A) discímil B) dieciocho C) diezmado productor de oro fi no es:
D) diócesis E) discernir Al Cajamarca B) Ancash Cl Junín
D) Madre de Dios El Puno


Al\:
~~
GomeZ
.•.
~ª~'$iiíWl
UNI 2001-11 APT. ACADÉMICA Y CULTURA GENERAL I
61. Relacione adecuadamente los siguientes desier- 65. El tcrrritorio de Jaén se integró al Perú luego de
tos subtropiculcs con los continentes en que están ubi- la independencia. a través de
cados. A) el cambio por Guayaquil
1. Atacama a) África S) el principio de posesión
11. Victoria b) América del Norte C) la libre determinación de sus habitantes
111. Kalahari e) Asia D) un conflicto militar
IV. Nuevo México d) América del Sur E) una cesión del Ecuador
V. Thar e) Oceanía
A) Id. ne, lila, IVb, Ve D) le, lid, lile, IVb, Va
66. Los personajes que se mencionan. asumieron la
Presidencia de la República durante más de un perio
S) le. lid, IlIb, IVa, Vc E) la, lIe, lile, IVb, Vd
do. El que gobernó una menor cantidad de años fue
Cl Id, lib. lila, IVe, Ve
A) Manuel Odría D) Fernando Bclaúnde

62. Relacione adecuadamente cada río con el país al S) Alberto Fujirnori E) Manuel Prado
que pertenece. C) Augusto S. Leguía
1. Orinoco a) Ecuador
67. Inca que en su juventud llevó el nombre de
11. Muule b) Venezuela
Yupanqui, A pesar que no fue designado inicialmente
111. Napo e) Argentina como sucesor, logró acceder al poder. porque defendió
IV. San Juan d) Chile con éxito al Cuzco contra los chancas.
V. La Plata e) Colombia A) Huayna Ciipac D) Mayta Cápac
A) lb, Ile, lila, IVd. Ve D) le, lid, IIlb, IVe, Va B) Lloque Yupanqui E) Túpac Yupanqui
S) lb, lid, lila, IVc, Ve E) le, IIb, Illc, IVa, Vd C) Pachacútcc
C) lb, lid. lila, IVe, Ve
68. A través de qué indicador se mide el nivel de la
63. ¿Qué trascendencia tuvo el fin de la "Guerra Fría"? actividad económica.

A) Puso en crisis las ideologías, desapareció la Al Empleo D) Producto Bruto Interno


bipolaridud y aceleró la globahzación. S) lnvcrsión E) Reservas Internacionales etas
S) Se puso fin a la ocupación de la Antártida. C) Liquidez
C) Dio paso a. la "Guerra Caliente", caracterizada
69. El país que ha reemplazado el uso de su moneda
por el abierto uso de las armas nucleares.
nacional por el dólar norteamericano es
D) La recuperación sustancial de las relaciones co-
A) Argentina S) Ecuador C) Nicaragua
merciales entre Estados Unidos y la Unión
Eropea, D) Paraguay E) Uruguay

E) Permitió la caída del muro de Berlín, la creación


70. Como "Riesgo - País" se entiende:
de la Unión Europea y el apoyo de Estados Uni-
dos a lrak. A) Una calificación utilizada en los medios finan-
cieros internacionales para identificar lus esce-
64. A continuación se mencionan cinco procesos re-
narios de mayor rentabilidad en la inversión.
volucionarios del siglo XX. Señale la alternati va en que
B) Una parte importante de la política gubernamen-
figuran el más antiguo y el más reciente.
tal de un país. para financiar su déficit pre-
A) Revolución Rusa y Revolución Cubana. supucstal,
S) Revolución Mexicana y Revolución Sandinista. C) Una calificación que forma parte de la agenda
C) Revolución China y Revolución Sandinista. XXI, para medir las amenazas de desastres natu-
rales en cada país.
D) Revolución Rusa y Revolución Sandinista.
D) Un indicador que mide la estabilidad de un país.
E) Revolución Mexicana y Revolución Cubana
a 'consccucnciu de políticas integrales apropia-
~:::::=:::I
~
UNI 2001-11 APT. ACADÉMICA Y CULTURA GENERAL ~~::::!~:~!;::,::~y GmeZ
'vn

das. Sirve de orientación a los inversionistas in- B) Ecologistas chilenos han respaldado la defensa
ternacionales. de los pantanos.
El El grado de violencia social de un país, que es C) Se ha propuesto que los ciudadanos protesten.
utilizado para orientar el desempeño de las em- negándose a consumir los fideos producidos en
presas turísticas internacionales. la zona de los pantanos.
D) Los Pantanos de Villa constituyen la más notable
71. Señale la alternativa que corresponde: zona de protección ccológica dentro de la ciudad
"Abarca a todo lo que el hombre realiza de forma libre y de Lima.
voluntaria. Ofrece criterios y referencias para diferen- E) La empresa de fideos que instaló su fábrica en el
ciar lo bueno y lo malo, lo aceptable y lo rechazable" área de los pantanos, ha asegurado que la trasla-
dará a otra parte de la ciudad.
A) La Lógica D) La Metafísica
B) La Ética E) El Derecho 76. Nombre del empresario norteamericano que este
C) La Psicología año dió inicio al turismo espacial. al pagar unos 20 millo-
nes de dólares para participar en un vuelo espacial ruso.
,.72. Una de las frases no corresponde al autor indica- A) Al Gore D) George Soros
do.
B) ain Gutes El Harrison Ford
Al Sólo sé que nada sé Sócrates
Cl Dermis Tito
B) Pienso, luego existo Descartes
C) La filosofía es la ciencia del ser. Aristóteles. 77. Estando excluido de los préstamos del Banco
D) Dios ha muerto. Nietzsche Mundial desde 1962, su presidente Jarncs Wolícnson
reconoció recientemente que ha hecho un trabajo ex-
E) El ser lrs y el no ser no es. Heráclito
celente en salud y educación. ¡.De qué país latinoame-
ricano se trata?
73. Un trabajador, sin quererlo, saluda a su jefe que
cumple años, dándole el pésame. ¿Cómo se denomina A) Brasil B) Costa Rica C) Chile
este tipo de manifestación? D) Cuba E) Uruguay
A) Sublimación B) Antipatía C) ilusión
78. Sector social que últimamente ha tenido un papel
D) Falsa percepción E) Acto fallido
protagónico en las movilizaciones populares en Méxi-
co, Ecuador, Bolivia y otros países latinoamericanos.
74. Una de las alternativas no expresa ninguna forma
de afectividad. ¿Cuál es? A) comunidades indígenas
A) Un empleado amargado por problemas con su B) empleados bancarios
esposa se desahoga con sus compañeros de tra- C) estudiantes universitarios
bajo. D) mineros
B) Un chimpancé es adiestrado para canjear objetos E) soldados
por comida.
79. País cuyas relaciones con Estados Unidos sc han
C) Está cocinando y nerviosa. Por descuido, se que-
deteriorado en estos años, por incidentes graves como
ma el arroz.
la colisión de aviones militares, y el bombardeo de su
D) Todos están tranquilos y los jugadores del equi- embajada en Yugoslavia por fuerzas de la OTAN.
po nacional entran a la cancha.
A) Carea del Norte B) Cuba C) Rusia
El Juana, enamorada de su novio, le perdona a me-
D) Irán El China
nudo sus torpezas.
80. ¿Qué país sufrió por primera vez grandes estra-
75. Una de las afirmaciones referidas al tema de los
gos por el llamado "mal de las vacas locas"?
Pantanos de Villa no es cierta.
A) En las últimas décadas gran parte del pantano ha A) Estados Unidos B) Francia C) Inglaterra
sido destruido para urbanizarlo. D) Japón E) Rusia

"
sr Ole:mWJUVWIlJiilltMFi'WH:nWUNI 2001-11 APT ACADÉMICA
Y CULTURA
GENERAL~:=:;!:~:;:~:0

SOLUCIONARIO
APTITUD ACADÉMICA
7. SENTIMIENTO, acción y efecto de sentir, esta pa-
SINÓNIMOS labra no tiene el mismo significado que las otras. que
comparten el sentido de expresión religiosa.
1. "No deben arrugarse frente a los problemas".
Clave: C
En la expresión, el término arrugarse significa enco-
gerse, asustarse, cuyo sinónimo es amilanarse.
8. CONJURO, sinónimo de EXORCISMO.
Clave: A
CONJURA, sinónimo de COMPLOT.

2. "Aquel juez fue sancionado por desacato". Se trata de una analogía en paralelo.

En la expresión, el término desacato se refiere a que el Clave: D


juez a actuado fuera de la ley, de alli que su sinónimo
es insubordinación.
COMPRENSiÓN DE LECTURA
Clave: C
9. Del texto se deduce que: todavía no es posihle lle-
ANTÓNIMOS gar a conclusiones definitivas sobre el impacto de
3. "No lo condenaron, porque su falta fue consecuen- las reformas, no se puede asegurar el éxito o el fraca-
cia de una reacción maquinal". so de las mismas.
Clave: C
De la oración, se desprende que no recibió ninguna con-
dena, porque su acto no fue voluntario, fue inducido. 10. De acuerdo al texto, el Imperio lncaico ha sido
Por lo tanto su antónimo, es una reacción voluntaria, idealizado en los Andes, la colectividad andina consi-
es decir, consciente. deraba que en el Imperio existía una sociedad homo-
Clave: A génea y justa lo cual es una utopía.
CLave: B
4. PETULANTE presunción ridícula de una perso-
na,' que está convencida de su superioridad sobre los
11. ¿Cuál es la idea principal del texto'!
demas; sus antónimos, cortés, respetuoso, afable y re-
El mundo no fue creado en un solo acto. Del texto se
verente pero no sumiso que significa obediente, dócil.
desprende que el mundo fue creado en varios actos.
Clave: B
CLave: C
5~ "La carroza y la sección servidumbre, ejercieron
siempre una extraña fascinación sobre Julius". 12. De acuerdo al texto, el fin de la Guerra Fría redu-
ce la importancia geopolítica de las ideologías.
Del texto, el término fascinación se entiende como,
Las sociedades unidas por las ideologías o las circuns-
encantar, deslumbrar, atraer. Su antónimo es aversión,
tancias históricas, pero divididas por la civilización; o
es decir repugnancia, oposición.
Clave: C se des-hacen o estan sometidas a una gran tensión.
Clave: E
ANALOGíAS

6. CONSTITUCiÓN y LEY, entre ellos existe una ORACIONES INCOMPLETAS


relación de jerarquía, la constitución tiene mayor ran-
go que la ley; en forma análoga, la LEY tiene mayor 13. En el contexto jurídico: "Debido a la corrupción,
rango que el DECRETO SUPREMO. en los tribunales no se impartía justicia".

CLave: C Clave: B
"'··28 ~-=!b!!!iU#*nIUNI2001_11 APT. ACADÉMICAY CULTURAGENERALI=;::;~=''K
meZ
14. La oración se completa así: 20. En el contenido del texto, se expresa la duda de
"La UNI al comenzar el siglo XXI reafirma su com- Tomás, 3 días antes de la elección, de votar por uno u
promiso de formar personas capaces de sentir, va- otro candidato.
lorar, interpretar y transformar el mundo de. la exis- "Faltaban 3 días para las elecciones y Tom.is no
tencia individual así como el social y colectivo". sabía si votar por un candidato o el otro".
Clave: B Clave: A

15. EL texto indica que: los jueces, han condenado a


cadena perpetua, sobre la base de pruebas coutunden-
tes y de acuerdo a la ley. Se entiende que han actuado
CONCORDANCIA
con rectitud. 21 • Analizando la oración
Clave: A
concordancia concordancia concordancia
r> r:»
16. La oración
"Montesquieu
completa:
comparte con Locke la fama de ha-
"Se puso la camisa,
L-J
género femenino
la corbata
L-J
género femenino
'"
y el saco
Lj
género masculino
ber acabado con muchas de las falsas ideas que hasta número singular número singular número singular
entonces habian impedido reconstruir la sociedad
humana". que le regalamos".
Clave: E Clave: B

22. En el contenido del texto se expresa el retiro de


CONECTORES un gran prospecto en el deporte del ajedrez, por dcsición
propia.
17. EL texto alude un evento esperado, que por cir-
cunstancias especiales no se pudo realizar hace un año. "A los 19 se convirtió en Gran Maestro lnternucio-
nal. Parecía estar encaminado a ser uno de los gran-
"El ex-presidente Clinton dió por fin este lunes en
des en el deporte de los trebejos, su retiro alarmó a
Jaipur (India) el paseo a lomo de elefante que la
muchos. El sonríe y dice: Es mi opción de vida y
había sido negado, por razones de seguridad, du-
deben respetada".
rante una visita oficial hace tan sólo un año".
Clave: B
Clave: D

18. El contenido del texto trata de un pesonaje públi-


co que regresa sorpresi vamente después de varios años. COHERENCIA
"El mimo vuelve a su plaza. Un día se fue del Perú
23. El orden lógico para lograr una lectura adecuada
sin avisar. Y su público no supo más de él hasta
es de la siguiente manera:
hace unas semanas, cuando apareció en la plaza
después de veinte años de ausencia". 3.- La muralla fué construida entre los años 271 y
275 para defender Roma. Se desplomó en las úl-
Clave: A
timas horas del domingo.(Según el orden
19. El contenido del texto, expresa que el hombre cronológico empezamos evocando el origen de
debe asumir una actitud positiva y creadora, frente al la zona turística).
mundo en que vive y no soportar lo que se le impone. 1.- La lluvia probablemente tuvo algo que ver, pero
"Un mundo le es dado al hombre; su gloria no es no puede haber sido la única razón del derrum-
soportar o despreciar este mundo, sino enriquecerlo bamiento. (Segundo expresamos lo que sucedió
construyendo otros universos". y su probable causa).

Clave: B 2.- Habitualmente es punto de atracción para los tu-


ristas. La policía acordonó la zona. (Tercero, se
:: me: 1Wt%'1tJlU:mmi.W'4WiWiillt.i@ UN12001-11 APT. ACADÉMICA Y CULTURA GENERAL ::.::!:,::=0
expresa las medidas de seguridad que se tomó
SERIE DE FIGURAS
por ser una zona concurrida por turistas).
4.- La policía dijo el lunes que nadie resultó lastima- 26. Al plegar la figura tenemos:
do. (Por último se informa si causó o no daños
personales).
Clave: C

24. El órden lógico de los textos teniendo en cuenta


el título general:
IV. Hay autores que no son grandes escritores. D

V. Se les suelen llamar "menores". Formándose un sólido tridimencional.


1. De ellos podemos aprender algo. Clave: O
lll. Trampas literarias, por ejemplo.
27. IY IY
11. Estas trampas no son visibles en los grandes.
Se empieza mencionando la existencia de escritores que
no son tan famosos (IV), luego se les da un nombre
~ :~
I
I
I
I
I

I
I

(V), y lo que podemos aprender de ellos (1), como las


I ~ I ~
trampas literarias (111), que no son muy evidentes en 1
los grandes (11).
Clave: D
"
Las figuras del cuadro (1) son simétricas al eje Y, la
misma relación deben tener las figuras del cuadro (11).
25. Ordenemos los subtitulos de: Clave: E
"La corrupción en el Perú y como superarla". 28. Si el corte es diagonal y corta al cilindro, al me-
nos a una de sus bases.
3.- El marco mundial - Antecedentes y definiciones
del problema.
2.- Condiciones que favorecen la corrupción desde
hace muchas generaciones.
1.- El camino de la corrupción en las últimas déca-
das.
5.- Proyectos y perspectivas para resolver el proble-
ma en la moral. Las partes del cilindro dividido no se puede represen-
4.- Medidas urgentes para poner en práctica las pro- tar por dos esferas.
puestas generales. Clave: A
Se empieza ubicando el problema en el contexto inter-
nacional (3), luego se analizan factores que favorecen
29. Pirámide de base Despliegue de
la corrupción (2), el desarrolllo de ésta en las últimas cuadrangular. la pirámide.
décadas (1); seguidamente, se señala los proyectos y. V
perspectivas para resolver el problema (5) y finalmen-
D C
te las medidas inmediatas a tomar para resolver el pro-
blema (4).
Clave: A

D C
Clave: E
~-:=;l UNI 2001-11 APT. ACADÉMICA Y CULTURA GENERAL I ·_'"
..... ~c,·_·""
~
UJI~
G meZ

30. Despliegue del cubo. 34. Determinemos M y N :

Un cuadrado Un triangulo negro


7+2=9
aliado de un aliado de un
círculo blanco 6. círculo negro :ri·7.·:·2.::?'·,

i~2
L...9 5...Ji
o •
7 7
3+N=IO
3

[~r
Además el número 3

Plegando la figura y rotando 1800 para observar los => N=7


M=15
tres lados que estaban no visibles inicialmente,
se tiene.

Por lo tanto: M +N = 15+ 7

Clave: O =22
Clave: B
31. Despliegue del sólido:

5 35.
18
+3 +3 +3 /-3 I
~,r-~~~t
2 6 4 9 7 12 11 15 P Q
I O 1 2 3 4 ~~~~t
+2 +3 +4 .,.5
L Líneas horizontales Lado "X"J 16

Clave: E Por lo tanto: P + Q = 34


32. Piezas mostradas: Clave: C

V 36.
3

Figura formada con las 5 piezas ..

Clave: C

Clave: E
37.
-2 2 18 52 110 198
'-.tV~'-Jj./~ '-J:i/
SERIES NUMÉRICAS 4 16 34 58 88
33. '-.tV~ '-Jj./ ~
12 18 24 30
\..t:V '-Jj./ ~
666

Clave: B
Clave: O
=;;::::::=::1
L.::*",,··=.o.::~...•. UN12001-11 APT.ACADÉMICA
Y CULTURA
GENERALI=~:~i:::=0

38. 43.
2 10 24 44 70
'-tl/'JY~'-.tl/
8 14 20 26 Hcnnanos
'-tl/'JY~ ~ ~
o,
6 6 6
Clave: B !Hijo
'0<-
\.....
0
<$-.'
'0,,-<-
~ !
lIija
zr
e:
"'
o-
!1>

39. ¡;;-
<;}

~~
(=) H .,
e-
e:

º!
o-
4 8 12 Primos !!.
H M~
(=) o'

p'r
o-
~~ "
21 42 63 :3
¡IfijO Hija .."
(=) (-)
'"
~
~~ !1>

36 72 108
YOI Prima

40. Despliegue

3, 8
!/ 9,
de la figura:

14 )3 15 20
\ 1,,-
19 21
Clave:

, ;S
26
D
Por lo tanto la hija de la hija del tío de mi padre es mi
prima.

RAZONAMIENTO
Clave: E

MATEMÁTICO
º
"s
6 10 12 '11 16 18 "17 22 24 "23 28 44. Total (padres y madres) = 240
,
Superficie
. "Z"
. Madres: 70% (240) = IGR

Luego, los números de la fila inferior es: 24; 23; 28.


Padres: 30% (240) = 72
Si llegan x parejas ahora el 40% del total son hombres.
Clave: C
luego tenemos:

(72+x)=40% (240+2x) =::} x= 120


Por lo tanto llegan a la asamblea 120 parejas
RAZONAMIENTO LÓGICO Clave: B
41. De la expresión, sabemos que "Claudia y Patricia 45. Inicialmente: a
no saben nadar", además ellas contaron que "cayeron
al mar",
pero no
de donde deducimos que ambas se salvaron,
podemos precisar como.
Clave: E
I
a~} a
42. Del enunciado tenemos: Si su lado es a+ S({ :
l. Manuel es mayor que Juan.
2. Manuel es menor que Enrique.
3. Víctor es mayor que Enrique.
4. Víctor es menor que César.
De (1); (2); (3) y (4):
Incremento del área:
César> Víctor > Enrique> Manuel> Juan
Por lo tanto el mayor es César.
Clave: D Clave: B
0-:.=!:::::::.
.. :;:::::~~ U~N~I
~20~O~1-!1I
~A~P~T.~A~C!A~DE~'
M~I~CA~Y~C~UL~T~U~R!A~G~E~NE~R~A~L 1:W~:::~m.'m:
46. Precio inicial :M 50. De la información (1), determinamos que 2 es el
Primer descuento : 20% M número menor, de la información (11),obtenemos dos

Primer importe 80% M posibilidades, M > N ó M = N , por lo tanto la in-


formación brindada es insuficiente para determinar
Segundo descuento: 15% (80% M) el mayor.
Segundo importe 85% (80% M) = 68% M Clave: E
Si se desea realizar un único descuento, éste sería de
32% para tener un precio de venta equivalente de CULTURA GENERAL
68% del M. 51. HIATO, pronunciación de dos vocales sucesivas
Clave: O en sílabas diferentes, esto ocurre cuando: las dos vo-
cales son abiertas y cuando una vocal es abierta y la
47. Velocidad de la persona: v = Z cm = L..!!!.
s 100.1" otra vocal cerrada acentuada .

Tiempo que camina: t = Q h = 3600Q s Vocales cerradas: i, u.


Vocales abiertas: a, e, o.
Distancia que camina: d = vt
Analizando las alternativas.

Reemplazando: d = L..!!!. x 3600Q.f A) de-se-ar ; te-o-lo-gí-a ; clo-a-ca


100 s • Las tres palabras presentan hiato.
=36ZQm B) be-o-do ; bio-lo-gí-a ; bai-le
Clave: O • Dos palabras presentan hiato.
48. C) ca-er ; au-da-cia ; jue-ves
• Una palabra presenta hiato.
: ler. día: 2do. día: 3er. día : 4to. día: Sto. día:
D) a-ho-ra ; fuer-za ; ho-ra-rio
,d Id Id I di d:
''----./'~'~'~' '----./' • Ninguna palabra presenta hiato.
~ 80%d ~ 70%d ~ 60%d ~ 40%d ~ x %d ~ E) mue-ca ; cuen-tas ; mue-lle
• Ninguna palabra presenta hiato.
Al final del Sto. día sólo recorrieron el 60% de la dis-
Clave: A
tancia total.

; 3d , 52. Errores ortográficos del texto:

'-----'
: 60%(Sd) :
"El impacto de las Reformas (reformas) económi-
cas enprendidas (emprendidas) por los paises (paí-
Encontremos qué porcentaje de la distancia (el) fijada ses) de América Latina y el caribe (Caribe) en las
recorrieron el último día. últimas dos decadas (décadas), se encuentra en el .
centro del débate (debate) sobre las politicas (po-
60%(5el) = 80%d + 70%d + 60%d +40%d + x%d líticas ) económicas de la región".
~ x= 50 Total de errores: 8 (incluye la coma colocada innece-
Clave: A sariamente).
Clave: E

53. Incorrecta Correcta


SUFICIENCIA DE DATOS
aereolínea aerolínea
49. De la información (11), obtenemos que el padre beneficiencia beneficencia
tiene 48 años y de la informacin (l), determinamos la cónyugue cónyuge
edad deÍ niño que es 8 años; por lo tanto es necesario
costipado constipado
emplear ambas informaciones.
dentrífico dentífrico
Clave: C
:=::::;;::::::~:::::·::""~lUN12001-11 APT. ACADÉMICA Y CULTURA GENERALI

• Dentífrico. proviene de las palabras latinas: "Ser o no ser; he ahí el dilema".


dentis ~ diente Célebre cita de Hamlet, gran obra de William
Shakespeare, cuyo tema es la duda. Esta obra no
Iricare ~ frotar
corresponde a Macbeth, también escrita por
Clave: E Shakespeare, cuyo tema es la ambición.
"Al despertar Gregario Samsa una mañana. tras
54. Oraciones cuyos significados son equivalentes:
un sueño intranquilo ..."
1.- Poco a poco. el poder adquisitivo se deterioró en
Esta cita pertenece a la "Metamorfosis" de Frank
el país.
Kafku, cuyo protagonista es Gregorio Samsa.
Esta oración señala, que el que sufre el deterioro
"Raskolnikov permaneció largo tiempo acosta-
es el "poder adquisitivo". do. A veces parecía despertar. .."
2.- El poder adquisitivo, por poco, deterioró al país.
Esta cita pertenece a la primera novela psicoló-
Esta oración señala, el deterioro del país a causa gica "Crimen y Castigo" de Fedor Dostoievski,
del poder adquisitivo. cuyo protagonista es el estudiante Raskolnikov.
3.- El país se deterioró poco por el poder adquisitivo. f":¡ve: C
Esta oración señala que el país no ha sufrido un 57. Obras:
gran deterioro a causa del poder adquisitivo.
"La casa verde"
4.- El poder adquisitivo en el país, se deterioró poco Autor: Mario Vargas Llosa
a poco.
"No me esperen en Abril"
Esta oración es equivalente a la primera, seña- Autor: Alfredo Bryce Echenique
lando que el poder adquisitivo se deterioró.
"Conversación en la Catedral"
5.- El país poco a poco, deterioró el poder adqui- Autor: Mario Vargas Llosa
sitivo.
"Un mundo para Julius"
Esta oración señala que la causa del deterioro del Autor: Alfredo Bryce Echenique.
poder adquisitivo es el país.
Son novelas de Mario Vargas Llosa y Alfredo Brycc.
Clave: E
Clave: O
55. CORRECTO:
B) dieciocho C) diezmado 58. En la actualidad el Perú ha resuelto la delimitación
con todos sus países fronterizos (Brasil. Colombia, Chi-
D) diócesis E) discernir
le, Bolivia y Ecuador), el último fue con Ecuador (199H)
INCORRECTO: con el "Acuerdo Global de Paz Ecuador - Perú".
A) discirnil, lo correcto disímil que proviene del Clave: O
latín dissimilis, que significa distinto.
Clave: A '59. Las fases de la Luna son:
• Luna nueva • Luna llena
56. Analizando la cita que no corresponde a la obra
• Cuarto creciente • Cuarto menguante
mencionada:
La media luna no corresponde a ninguna de las tases
• "Canta, [Oh Diosal, la cólera del pélida Aquiles".
de la Luna.
En efecto esta cita pertenece a la obra la "Iliada", Clave: B
donde uno de los protagonistas es Aquiles, el au-
tor de la misma fue Homero. 60. En la actualidad el Perú se ha convertido en el
primer productor de oro en Latinoarnérica y esui entre
• "En medio del camino de la vida, errante me en-
los primeros a nivel mundial, debido principalmente al
contré por sel va oscura".
yacimiento de Yanacocha, ubicado en el departamento
Esta cita pertenece a la "Divina Comedia" de de Cajamarca.
Dante Aligueri.
C ave: A
61. Ubicación de los desiertos subtropicales en el • Libre determinación de los puehlos.- Era una
mundo. consulta a los habitantes de una rcgión sobre la
nacionalidad a la cuál querían pertenecer.
Desierto, ." Continente
El territorio de Jaén se integró al Perú después de
I Atacama (d) América del Sur la independencia, a través de la lihre determina-
/[ Victoria (e) Oeeanía ción de sus habitantes.
lTl Kalahari (a) África Clave: C
IV Nuevo México (b) América del Norte
66. Personajes que asumieron la Presidencia de la Re-
V Thar (e) Asia
pública durante más de un periodo:
Clave: A • Manuel Odría, gobierna 8 años, (1948 - 1')56)
• Alberto Fujímorí, gobierna 10 años, (1990 -
62. Ubicación de los rios en el país al que pertenecen. 2000).

Río País Augusto B. Leguía. gobierna 15 años cn total,


(1908 - 1912) Y (1919 - 1930).
[ Orinoco (b) Venezuela
• Fernando Belaunde Terry, gobierna 10 años en
[[ Maule (d) Chile
total, (1963 - 1968) y (1980 - 1985)
lTl Napo (a) Ecuador
• Manuel Prado, gobierna en total 12 años, ( 1931)-
IV San Juan (e) Colombia
1945) Y (1956-1962).
V La Plata (e) Argentina Clave: A

Clave: C 67. En los inicios del siglo XV. se inicia una guerra
entre los Incas y los Chancas, el rey lnca era Wiracocha
63. La Guerra Fría empezó al término de la Segun- y su sucesor el auqui lnca Urce, los Chancas logran
da Guerra Mundial, entre EEUU y la URSS debido a sitiar al Cuzco, fue entonces que asume la defensa del
las profundas diferencias ideológicas, políticas, socia- mismo Cusi Yupanqui quien logró vencer a los
les y económicas. Con la desintegración de la Unión Chancas en la batalla de Yahuarpampa, fue así que asu-
Soviética en 1991, concluyó la Guerra Fría con el pre- me la dirección del Imperio con el nombre de
dominio de los Estados Unidos, que trajo como conse- Pachacútec.
cuencia la crisis en las ideolgías, desapareciendo la Clave: C
bipolaridad y acelerando la globalización.
68. El indicador que mide el nivel de actividad eco-
Clave: A
nómica de un país, es el Producto Bruto Interno (PBl).
que muestra en forma aproximada el nivel de empleo.
64. De las revoluciones mencionadas, la más antigua
ahorro, consumo, inversión, etc, registrando el valor
fue la Revolución Mexicana en 1910, de carácter
de la producción de bienes y servicios finales genera-
agrarista cuyo lider fue Emiliano Zapata.
dos dentro de una misma economía durante un periodo
De las mencionadas, la más reciente fue la Revolución de tiempo.
Sandinista en Nicaragua, que se agudizo en los años Clave: D
1977 y 1979. El nombre Sandinista se debe al líder
popular asesinado Augusto César Sandino. 69. EL gobierno ecuatoriano presidido por Jamil
Mahuad, en el año de 1999, ha reemplazado el uso de
Clave: B
su moneda nacional por el dólar norteamericano. dcbi-
do a la constante devaluación e inestabilidad de su mo-
65. Dos principios jurídicos fueron la base para de-
neda.
terminar el patrimonio territorial de las nuevas repú-
Clave: B
blicas independientes en Latinoarnérica.
• Uti possidetis.- Las repúblicas conservan su te- 70. Se entiende como "Riesgo - País" al indicador
rritorio colonial respaldado por documentación que mide la estabilidad de un país, a consecuencia de
real, anterior a 1810. políticas integrales apropiadas. Sirve de orientación a
los inversionistas internacionales que desean tener cada tccción ccológica dentro de la ciudad de Lima, dada su
vez mayor seguridad para sus inversiones. importancia C0l110 hábitat natural de especies de nora
Clave: D y fauna.
Clave: E
71. La Ética.- Es la disciplina filosófica que estudia
las normas morales de un grupo humano, las cuales la 76. EL empresario norteamericano que este año dio
realizan en forma libre y voluntaria, tornándolas como inicio al turismo espacial es Dennís Tito. quien traba-
referencia para diferenciar lo bueno de lo malo, lo acep- jó en los años sesenta en la NASA, siendo descalifica-
tab1e de lo rechazable. do en aquellos años a participar en viajes de explora-
Clave: 8 ción por deficiencias técnicas.
Clave: 8
72. Parménides, filósofo afirmaba que "el ser es y el
77. Cuba, es el país latinoamericano que más avances
nu ser no es", haciendo referencia a que la realidad no
ha logrado tanto en salud como en educación, obtenien-
podía estar en movimiento, porque ello implicaría pasar
do los mas elevados indices de escolaridad, bajas tasas
del ser al no ser, lo cuál es absurdo. Por el contrario
de deserción escolar, bajos niveles de mortalidad y des-
Heráclitu decía "todo fluye", haciendo referencia que,
nutrición infantil. Desde 1962 sufre un bloqueo cconó-
para él, la realidad estaba en movimiento constante.
mico por el cual está excluido de los prestamos del Ban-
Clave: E co Mundial para financiar programas sociales.
Clave: D
73. Actos fallidos, son manifestaciones de la vida'
cotidiana de las personas, que cometen errores u olvi-
78. EL sector social que en los últimos años ha tcni-
dos que esconden motivos o deseos inconscientes. Del
do un rol proragónico en las movilizaciones populares
enunciado el trabajador comete el error de darle el pé-
son las comunidades indígenas. En Mex ico en 1994
same, en lugar de felicitarlo por su cumpleaños, exte-
se inició en el estado de Chiapas las acciones armadas
riorizando sentimientos negativos hacia su jefe.
del Ejército Zapatista de Liberación Nacional, forma-
Clave: E do por las masas campesinas indígenas. En Ecuador,
en 1999, tuvo lugar un conjunto de protestas princi-
74. Los procesos atectívos se manifiestan en: emo- palmente por campesinos indígenas que determinó la
ciones, pasiones, sentimientos y estados de ánimo. caída del gobierno de Jamil Mahuad. En Bolivia tam-
Analizando las alternativas: bién se producen manifestaciones de los carnpesi nos y
• En (A), se manifiesta que el empleado está carga- mineros ayrnarus.
do de emociones negativas. Clave: A
• En (B), no se expresa ninguna forma de afectivi-
79. En. estos años, se han producido incidentes gra-
dad, sino de aprendizaje del chimpancé.
ves entre EEUU y La República Popular China. como
• En (C), se manifiesta un estado de ánimo de an- la colisión de aviones militares y el bombardeo de la
siedad. embajada de China en Yugoslavia por fuerzas de la
• En (D), se manifiesta un estado de ánimo de tran- OTAN de la cuál forma parte EEUU.
quilidad. Clave: E
• En (E), se manifiesta un sentimiento amoroso.
80. Inglaterra, es el país que sufrió por primera vez
Clave: 8
grandes estragos por el llamado "mal de las vacas lo-
cas", que afecto a sus grandes empresas ganaderas;
75. Hasta el momento, la empresa chilena que instaló
agudizando este problema la prohibición de importa-
su fábrica de fideos en los Pantanos de Villa no ha ase-
ción de carne de ese país por la Unión Europea. Esta
gurado que se transladarñ a otra parte de la ciudad, por
enfermedad ataca al tejido enceíñlíco de vacunos.
el contrario persiste en seguir operando en el ugar, lo
ovinos y seres humanos.
que ha generado la protesta de los ecologistas de dis-
tintas partes del mundo, incluyendo Chile. Los Panta- Clave: C
nos de Villa constituyen la más notable zona de pro-
0~·
=:==:~-:»'=I UNI 2002-1 APT. ACADÉMICA Y CULTURA GENERAL I~:

~~[J~~~
APTITUD ACADÉMICA Y CULTURA GENERAL

6. Elija la alternativa que relaciona adecuadamente los


APTITUD ACADÉMICA elementos del enunciado.
TÉRMINO EXCLUIDO __ mantener la economía como eje la super-
estructura como condicionante, el marxismo dcviuo
1. Elija el término que no guarda relación de significa- mecanicista.
do común con los otros y con el término base.
A) Por consiguiente - y - incluso
MENUDO
B) A causa de - y - de este modo
Al minúsculo B) diminuto C) breve C) No obstante - y - entonces
D) exiguo E) mínimo D) Por tanto - y - por consiguiente
El Dado que - y - incluso
2. Elija el término que no guarda relación de significa-
do común con los otros y con el término base. 7. Elija la alternativa que relaciona adecuadamente los
PERFECCIONAR elementos del enunciado.
A) progresar Bl mejorar C) afinar ___ el invierno fue intenso para muchos des-
D) pulir E) depurar agradable: algunas veces pudimos disfrutar de
las lluvias matinales.
3. Elija el término que no guarda relación de significa- A) Si bien - o - ya que
do común con los otros y con el término base.
B) Dado quc - y - pues
GLORIA C) Puesto que - mas - y
Al fama B) prestigio C) popularidad D) Aunque - y - sin embargo
D) goce E) celebridad El Ni - ni - es decir

4. Elija el término que no guarda relación de significa- 8. Elija la alternativa que relaciona adecuadamente los
do común con los otros y con el término base. elementos del enunciado.
TUMEFACCiÓN Esa empresa perdió la licitación no tuvo un
A) edema B) herida C) hinchazón buen representante el expediente técnico en re-
gia: , a nuestro juicio, era la mejor alternativa.
D) tumor E) chichón
A) pues - o - mas
B) ya que - y - si
CONECTORES
C) dado que - o - es decir
5. Elija la alternativa que establece la relación adecua- D) y - ni - pues
da entre los segmentos de la oración. E) porque - ni - no obstante
____ muchos medios de comunicación se vendie-
ron y mintieron, no se puede ir contra 9. Elija la alternativa que relaciona adecuadamente los
la libertad de prensa." elementos del enunciado.
A) Ya que - más aún - o sea Los bomberos llegaron muy pronto con todos
B) En efecto - en verdad - a continuación sus implementos requeridos; no pudieron hacer
nada el incendio había destruido toda la habita-
C) Por ejemplo - incluso - aún cuando
ción.
D) Primero - luego - finalmente
A) y - Y - mas
E) Aunque - además - no obstante
B) pues - sin embargo - ya que
C) y - mas - porque C) Las plantas terrestres eran demasiado duras y no
O) pero - y - y podían digerirlas. Esas criaturas tuvieron que vol-
E) o - no obstante - dado que ver al agua.
O) Pero la poderosa dentadura de un reptil del tamaño
ORACIONES ELIMINADAS de una ardilla podía cortar las duras hojas como una
tijera de podar.
10. Elija la oración que es redundante o no pertinente
respecto al sentido de las otras. E) El SUMINIA es el vertebrado m~s antiguo que se
conoce capaz de masticar y digerir plantas
A) Por medio de la máscara, se intenta asustar y con-
eficientemente.
trarrestar a los enemigos.
B) A partir de la Primera Guerra Mundial, ha apareci- 14. Elija la oración que es redundante o no pertinente
do un tipo especial de máscara. respecto a las ideas de las otras oraciones.
C) Este es un tipo de máscara llamado máscara antigas. A) En setiembre de 1939, Alemania invadió Polonia.
O) La máscara antigas consiste en una careta para pro- B) Este hecho dio inicio a la 2da. Guerra Mundial.
tegerse.
C) Previamente, Hitler había firmado un pacto de no
E) Esta careta se ajusta herméticamente al rostro hu- agresión con la Unión Soviética.
mano.
O) La Unión Soviética jugó un papel decisivo en la 2da.
11. Elija la oración que es redundante o no pertinente Guerra Mundial.
respecto al sentido de las otras. E) Al final, la Unión Soviética se unió a las fuerzas
A) El violín es uno de los instrumentos musicales más aliadas que enfrentaron a Alemania, lo que pcrrni-
importantes. tió la derrota de Hitler.

B) Es, además, el principal instrumento de la familia


de las violas. COMPRENSiÓN DE LECTURA
C) Técnicamente hablando, el violín es un instrumento
muy sensible.
15. "El descubrimiento de la estructura del ONA fue
muy importante. Permitió que los científicos entendie-
O) Los orificios del violín sirven para la salida del so-
ran cómo la información necesaria para construir un or-
nido.
ganismo vivo está codificada en genes y cómo la infor-
E) El violín posee una riqueza tonal y una expresividad mación se trasmite de una generación a la siguiente.
tan delicada. Tambien abrió la puerta a la ingenieria gen ética, por la
que los investigadores deliberadamente cambian los
12. Elija la oración que es redundante o no pertinente
genes y crean nuevas formas de vida."
respecto al sentido de las otras.
Del texto se puede plantear que:
A) El insomnio es la dificultad para conciliar el sueño.
A) Antes, no se conocían los genes.
B) El insomnio es también la dificultad para mantener
un sueño prolongado. B) El descubrimiento de ONA constituyó una revolu-
C) Mantener y conciliar el sueño es importante para la ción científica.
salud. C) La ingeniería genética es peligrosa.
O) La existencia de alguna enfermedad puede ser una O) Los investigadores, ahora, pueden crear nuevas for-
causa del insomnio. mas de vida.
E) Los problemas personales o el estrés pueden ser otras E) Los genes sólo se pueden cambiar deliberadamente.
de las causas del insomnio.
16. "La excavación de pozos ha sido siempre parte de
13. Elija la oración que es redundante o no pertinente
la solución frente al problema de la escasez de agua dul-
respecto al sentido de las otras.
ce. Mientras la extracción sea inferior a la alimentación
A) Cien millones de años antes, cuando los primeros natural del manto acuífero, el abastecimiento se mantine
vertebrados salieron de las aguas e incursionaron constante. Los problemas empiezan cuando la extracción
en tierra, hallaron muchos alimentos que no podian supera la realirnentación; y cuando esto ocurre, todo el
comer. acuífero queda expuesto a la contaminación y ocasionan
B) Los paleontólogos vienen estudiando los fósiles de daños que inutilizan los mantos acuíferos."
pequeño tamaño hallados en Rusia. El texto trata sobre todo acerca de:
0=:e~=1 UNI 2002-1 APT. ACADÉMICA Y CULTURA GENERAL I~:=:;=;;:;Xome: V

A) los problemas del agua en las ciudad~s modernas.


B) los beneficios y limitaciones de los mantos
COHERENCIA DE REDACIÓN
acuíferos. 21. Lea los enunciados y elija la alternativa que estable-
C) la forma de abastecimiento del agua a una ciudad. ce la secuencia correcta de las oraciones según el título.
D) la realimentación y la contaminación del agua potable. MANIERISMO
E) las operaciones en la extracción del agua del acuífero. l. Constituye una reacción frente a los ideales de per-
fección y equilibrio del clasicismo.
17. "Cualquier palabra de una frase, cualquier sentencia
11. Investiga en los campos de la expresividad. compla-
del párrafo, cualquier párrafo de un capitulo, cuyo signifi-
ciéndose en lo desconcertante y artificioso.
cado se capta por el contexto, constituye parte del contex-
111. En el arte de la imagen. las figuras se alargan, sugie-
to general y este último, la concepción del texto que
ren ingravidez, se atenúa la espacialidad.
se convierte en la unidad básica de la comunicación."
IV. Estilo artístico que se originó en Italia en la segunda
El texto trata principalmente sobre:
década del siglo XVI.
A) La relación entre los elementos del lenguaje.
V. En la arquitectura. estas características son menos
B) La inserción de la palabra dentro del párrafo. marcadas.
C) La estructura de los párrafos y los capítulos A) IV - 1 - 11- V -I1I D) IV - 1- Il - 111- V
D) El valor de los contextos durante la comunicación.
B) IV - II - 1 - IlI- V E) IV - 1 - 111- II -IV
E) El texto como unidad esencial de la comunicación.
C) IV - [][ - I - J] - V
ORACIONES INCOMPLETAS
22. Lea los enunciados y elija la alternativa que establece
18. Complete el enunciado con la alternativa adecuada.
la secuencia correcta de las oraciones según el título.
"Algunos dicen que los corruptos a la muche-
dumbre para al gobierno, e incluso lograr su LA MANO DEL HOMBRE
l. La sustitución ocurre, por ejemplo, en el caso de los
A) azuzan - desestabilizar - caída sordomudos.

B) pagan - sacar - censura 11, El movimiento de las manos constituye un impor-


tante recurso expresivo.
C) provocan - ilusionar - complacencia
1Il. Las manos muchas veces sustituyen el lenguaje ver-
D) usan - complacer - pasividad
bal.
E) utilizaban - atarantar - aprobación
IV. Los sordomudos se comunican exclusivamente gra-
19. Complete el enunciado con la alternativa adecuada. cias al abecedario manual.
V, Este amplía y matiza el lenguaje verbal.
Para mantener la __ el príncipe lucía el atuendo típico
de sus A) II - V - [ - IV - III D) 111- 1 - V - 11- IV

A) tradición - ancestros B) IV - II - V - I - J] I E) J] - V - 111- 1 - IV


B) cossnnbre - hijos C) IV - II - 1 - III - V.
C) historia - adversarios
23. Lea los enunciados y elija la alternativa que estable-
D) identidad - vecinos
ce la secuencia correcta de las oraciones según el título.
E) duda - abuelos
BOMBA ATÓMICA
20. Complete el enunciado con la alternativa adecuada. 1. La bomba atómica genera una enorme cantidad de
A pesar de haberse convertido en el de la causa calor.
democrática, sus adversarios lo como persona 11, En su teoría de la relatividad, Albert Einstein anun-
violenta. ciaba ya la posibilidad de convertir la materia en
A) autor - reconocieron energia.
B) contrario - imputaron 111.La radioactividad o emisión de partículas radioacti-
vas destruye o altera las células vivas.
C) paladín - tildaron
IV, En Hiroshima perecieron 80000 personas y los cdi-
D) héroe - lisonjearon
ficios comprendidos en un radio de dos ki lómetros
E) enemigo - calificaron desaparecieron.


V. El desarrollo é' estas armas de destrucción a gran 27. Determine el valor de A + B ,en la siguiente serie:
escala comenzó a principios de este siglo. 5 ; 12 ; 8 ; 10 ; 12 ; 7 A; B
A) V - II - 1 - fll - IV D) 1 - [[ - III - IV - V A)13 B)18 920 D)24 E)28
B) V - 1 - II - 1II - IV E) 1 - III - Il - IV - '! 28. Determine el número que completa la serie:
C) 1 - V - II - 1lI - IV
7
3 ;"2 9 13 21, '.i
"2' T; T
24. Lea los enunciados y elija la alternativa 'que esta-
blece la secuencia correcta de las oraciones según el tema
A) 2; B) 29 C)ll , E) il.
del título. 2 2 2
ESTADOS UNIDOS BOMBARDEA HOSPITAL 29. Identifique la alternativa que completa la serie:
l. El soldado Obaidullah, un trabajador del hospital, dijo 1 2 18
que el número de muertos podia ascender a 15. S ' -S ;-5; -21 ; ?
11. Esa mañana del 31, por lo menos se realizaron otros
A)-140 B)-130 C)-IIO D)-80 E)-56
I I bombardeos contra Afganistán, en lo que fue uno
de los ataques más fuertes de Estados Unidos desde 30. Las letras colocadas en los casilleros de la siguien-
el 7 de octubre. te figura representan a los ocho primeros números ente-
111. También informaron que 2 casas vecinas fueron des- ros positivos y están ubicados de tal manera que, no exis-
truidas. ten dos números consecutivos en casilleros que tengan
IV. El 31 de octubre, bombas norteamericanas destruye- algún elemento en Común (lado o vértice).
ron un hospital de Kandahar. Calcular: (a + b) (e + ti) - (e + h) (/+ g).
V. Según voceros del Talibán, el hospital era de la Cruz
Roja Internacional.
A) TI - V - I - III - IV D) IV - V - 1 - III - Il ~
B) II - IV - V - 1 - III E) V - IV - II - [[[ - 1 ~
C) IV - V - III - I - [[ A) -1 B) I C) O D) 2 E) -2

25. Lea los enunciados y elija la alternativa que establece 31. Determine el valor de M + N en la siguiente serie:
la secuencia correcta de las oraciones según el título. 6 ; 3 20 ; 8 ; 42 ; 15 ; M; N
LA EROSiÓN A) 106 B) 98 C) 96 D) 86 E) X4
1. La erosión se origina por distintas causas.
11. Una de las causas es la acción humana.
ANÁLISIS DE FIGURAS
III.La superficie terrestre varía conforme a una serie de
acciones externas. 32. Las figuras muestran un sólido y su desarrollo
IV. Este fenómeno de disgregación se conoce con el nom- (despliegue). De acuerdo a la información brindada, iden-
bre de erosión. tifique la "cara incógnita".
V. Las acciones externas desgastan la materia de deter- M p
minadas zonas.
A) III - V - I - IV - Il D) V - [[[ - IV - Il - I Q
B) 1 - Il - III - IV - V E) III - V - IV - I - II
C) I - II - IV - V - 1II
~
R
SERIES NUMÉRICAS ~inferior
incógnita
26. Indique el número que completa la serie:
A)S B)P C)Z D)Q E) M
L . _.l . -3 ., -15 ., ?
3' 3' 33. Elija el sólido que encaje en la figura adjunta para
formar un cubo.
A)-79 B)-91 C) -120 D) -57 E)-39
~~~==..I
~~AI.. UNI 2002-1 APT. ACADÉMICA Y CULTURA GENERAL 1:;::=~=f:W=WCJ\$i\til
,'};'::'> ~',~.f' ..,.~:%::;. N:···G'i)iñ'ez·m

RAZONAMIENTO MATEMÁTICO
38. El cuádruple de la edad de César es igual a la suma
de la mitad del triple y el doble de la edad de Luis; si
ambos son adolescentes, ¿quién de ellos es mayor y por
cuántos años?
A) Luis por 2 años
B) Carlos por I año
C) Luis por I año
D) Carlos por 2 años
34. ¿Cuál es la figura que se forma mediante la combi- E) Ambos tienen la misma edad
nación de las siguientes formas?
39. ¿Cuántas permutaciones pueden realizarse con las
letras de la palabra: INGENIERIA?
A) 162420 B) 151 200 C) 170 540
D) 18642 E) 252 600

40. A una fiesta asistieron 156 personas. En un mo-


mento determinado, bailaban algunas parejas (hombre y
mujer) y se observó que 31 mujeres y II hombres no
35. Identifique la figura que no sigue la misma ley de bailaban. ¿Cuántos hombres asistieron a la fiesta?
formación de las demás.
A) 68 B)74 C) 76 D) 78 E) 8~
B)
41. Se desea imprimir cierta cantidad de facturas, las
_ c:::JO cuales deben de tener una numeración compuesta por tres
c:::JO- vocales seguida de tres dígitos.¿Cuál es el máximo nú-
mero de facturas que se pueden imprimir?
A) 91 125 B) 110625 C) 145650
E) .•.

DD •••
D) 135415 E) 125000

42. Cuando son exactamente las 6:00 a.m.,un reloj mar-


ca las 5:40 a.rn.; se sabe que el reloj siempre se retrasa 4
36. ¿Cuántos triángulos y cuadrados hay en las siguien- minutos cada 2 horas. ¿A qué hora marcó correctamente
tes figuras? la hora por última vez?
A) 4:00 a.m. B) 8:00 a.m C) 4:00 p.m.
D) 6:30 p.m. E) 8:00 p.m.

43. En un grupo de personas, 10% son adultos; 70%


son jóvenes y 20% son niños. Si el peso medio de los
adultos es 80 kg, el peso medio de los jóvenes es 60 kg Y
A) II Y 8 B) 12 Y 8 C) 13 Y 9 el peso medio de los niños es 40 kg; entonces el peso
D) 13y 13 E) II Y 12 medio del grupo es:
A) 56 kg B) 57 kg C) 58 kg
37. Indique la alternativa que no tiene relación con las
demás. D) 59 kg E) 60 kg

.6 © g] 6 . O O RAZONAMIENTO LÓGICO
8.
A
O
B
o
C
O 8
D
+ EE
E
44. Ubaldo, Nicolás e Ignacio postulan a la UNI a ca-
nales diferentes (Il, I1l ó V); ellos culminaron su secun-
daria en los colegios: Francisco Bolognesi, Guadalupc o
Humboldt. Además se sabe que:
APT. ACADÉMICA Y CULTURA GENERAL 1~",0
- Ignacio no postula al canal III Para responder la pregunta:
- Ubaldo no postula al canal 11 A) Es necesario emplear ambas informaciones.
- El egresado del colegio Francisco Bolognesi no pos- B) La información 11 es suficiente.
tula al canal 11 C) La información I es suficiente.
- El egresado del colegio Guadalupe postula al canallll D) Cada una de las informaciones, por separado es su-
ficiente.
- Ignacio no es egresado del Humboldt
E) La información brindada es insuficiente.
Nicolás ¿de qué colegio egresó y a qué canal postula?
A) Guadalupe, canal III
OPERADORES MATEMÁTICOS
B) Humboldt, canal 11
C) Humboldt, canal V 48. Si: mtsn = m -n y a *b=7;+2, determine el
D) Francisco Bolognesi, canal V valor de "t" en la siguiente igualdad:
E) No se puede determinar
(466) */ = {-
45. De las siguientes premisas:
A) ~ B) t C) 14
1
D) 11 E) 26
1
"Todos los ingenieros son personas cultas"
"Algunos ingenieros no son científicos" 49. Si se cumple que: ~=W2+1 y 0=Z2+I,
Se concluye que: determinar el mayor valor de s, t ", en la siguiente igual-
A) Algunas personas cultas no son científicos. dad:
B) Todos los científicos
C) Los que no son científicos
D) Todas las personas
son cultos.
no son cultos.
cultas son ingenieros.
@) [QJ 14

E) Todos los científicos son ingenieros. A) -} B) I D)2 E) t


50. Si se cumple que: 111+ 31= 4u +I ,determine el
SUFICIENCIA DE DATOS
valor de Wen la siguiente igualdad:
46. Un vehículo viaja de una ciudad A a otra B con una
velocidad promedio de 100 kmlhora. Si el rendimiento
de este vehículo es 50 km por galón de gasolina, calcular
I W + 81 + I W - 71 = 54
el costo total de la gasolina consumida en el viaje, si se A)7 B)9 C)II D)13 E)15
dispone de las siguientes informaciones:
1. Cada galón de gasolina cuesta S/.7
Il. El tiempo total del viaje es 3 horas. CULTURA GENERAL
Para resolver el problema: 51 . Marque la alternativa en la que se usa correctamen-
A) La información 1 es suficiente. te las letras b o v.
B) La información 11 es suficiente. A) Va por la rivera del río.
C) Cada una de las informaciones por separada es Sir- B) Vive por el óvalo de la Brasil.
ficiente. C) Tienes que cabar un hoyo.
D) Es necesario usar ambas informaciones. D) Es un asunto tribial.
E) Las informaciones dadas son insuficientes. E) El puesto está bacante.

52. En relación a la siguiente frase:


47. Una bolsa contiene canicas rojas, azules y blancas;'
las canicas blancas son el doble de las canicas
rojas.¿Cuántas canicas de cada color hay en la bolsa? Seleccione la puntuación incoherente.
Información brindada:
A)
1. Hay 60 canicas azules.
11. Las canicas azules son el triple de las canicas rojas. B) : j
0===1 UNI 2002-1 APT. ACADÉMICA Y CULTURA GENERAL 1'::==:;:=:::::;7~me: Gi

57. La mutación de Gregorio Samsa a un miserable


C)
insecto en la obra La Metamorfosis de Kafka refleja
D) ,i !: A) el autoritarismo de los padres.
B) el absurdo de la existencia humana.
?¿ ? C) la vida holgada de los familiares.
D) la libertad de elegir cualquier trabajo
53. "Este examen es fácil, pero dicen que el del miér- E) la indiferencia del hijo hacia los padres.
coles será dificil y el del viernes será dificilísimo. Para
ingresar tengo que ser más inteligente que la mayoría de 58. Señale la alternativa que no tiene la relación co-
postulantes". rrecta entre departamento y yacimiento de cobre.
¿Cuántos adjetivos calificativos, y de qué grados, hay en A) Ancash : Antamina
el texto? B) Arequipa : Cerro Verde
A) Dos positivos, un comparativo, un superlativo. C) Cajamarca : Michiquillay
B) Dos positivos, un superlativo. D) Junín : Tintaya
C) Un positivo, un superlativo. E) Moquegua : Cuajone
D) Un positivo, dos comparativos.
E) Tres positivos, un comparativo, un superlativo. 59. Señale la alternativa que no tiene la relación co-
rrecta .entre departamento y resto arqueológico.
54. Los descubrimientos de finales del siglo XV amplia- A) Amazonas : Fortaleza de Kuélap
ron extraordinariamente el mundo conocido por los euro- B) Ancash : Centro ceremonial de Vilcashuamán.
peos. C) Cajarnarca : Santuario de Cumbcmayo
Seleccione el párrafo equivalente: D) La Libertad : Huacas del Sol y la Luna
A) Los europeos ampliaron los descubrimientos a fi- E) Larnbayeque : Pirámides de Túcume
nes del extraordinario siglo XV.
B) Los extraordinarios descubrimientos del siglo XV, 60. Señale la alternativa falsa, respecto a las Islas
ampliaron el mundo de los europeos. Galápagos.
C) El mundo de los europeos, a fines del siglo XV, fue A) Son de gran valor e interés, cientítico y turístico.
ampliado extraordinariamente con los descubri- B) Entre las especies que caracterizan su fauna están:
mientos. galápagos, iguanas y albatros.
D) A fines del siglo XV el mundo de los europeos fue C) Los cactus, el palo santo y el mangle caracterizan su
descubierto extraordinariamente. flora.
E) Extraordinariamente, los descubrimientos a fines del D) La población es reducida.
siglo XV, ampliaron el mundo conocido por los eu- E) Se localizan frente al litoral peruano.
ropeos.
61. Los mayores yacimientos petrolíferos del país se
55. Marque la opción donde hay uso correcto de las
localizan en:
mayúsculas.
A) la cuenca de los ríos Corrientes y Pastaza (Lorcto)
A) El Quechua fue la lengua en el imperio incaico.
B) la selva sudoriental (Madre de Dios, Puno),
B) El diario la república no se publicará el domingo.
C) las cuencas de los ríos Huallaga y Ucayali (Maquia
C) Los Ministros se presentaron ante el congreso.
y Aguas Calientes)
D) La población indígena vivía en América del Sur.
D) la zona noroeste del Perú (Talara)
E) En la edad media, los Reyes eran muy poderosos.
E) la selva del sur del Alto Ucayali y el Bajo Urubamba
(Carnisea) .
56. El "niño Goyito" es el personaje central de un cuento
de: 62. Elija la proposición errónea respecto a la Antártida.
A) Abraham Valdelornar A) Es frío, helado, deshabitado y se sitúa en el hemis-
B) Alfredo Bryce ferio sur.
C) Felipe Pardo y Aliaga. B) Posee gran riqueza ictiológica.
D) Manuel Ascensio Segura C) Posee abundante vegetación.
E) Ricardo Palma D) Está cubierta por gruesas capas de hielo.
~~~~=:I UN12002-1 APT. ACADÉMICA Y CULTURA GENERAL ,:;:~0
l~'~:·:
..
'-"'=~
E) Se utiliza exclusivamente para investigaciones 69. La década pasada el país se acogió a! Plan Brady
científicas. para:
A) enfrentar al narcotráfico
63. Dirigente de la India, asesinado en 1948 por un
fanático hindú, que rechazaba sus intentos por lograr la B) importar tecnología de punta.
paz entre los hindús y musulmanes. C) incrementar las exportaciones.
A)Anwar Sadat B)Indira Gandhi D) obtener donaciones de alimentos.
C)Mahatma Gandhi D)Rajiv Gandhi E) renegociar la deuda externa.
E)Jawaharial Nehru. 70. Si hace un año un dólar se compraba a 3,55 soles, y
hoy se compra a 3,45 soles, se puede decir que:
64. A fines de la década de 1960, Israel, con el apoyo A) el dólar se ha revaluado.
de Estados Unidos y el Reino Unido, invadió la Penínsu-
B) hay una mayor demanda de dólares.
la de Sinaí e inició su Segunda Guerra contra los países
árabes, como reacción ante la C) hay una menor demanda de soles.
A) acción militar de la Organización para la Libera- D) la inflación ha bajado.
ción Palestina. E) la moneda nacional se ha apreciado.
B) cuadruplicación del preció del petróleo.
71. Elija la proposición incorrecta:
C) invasión de Irak a Kuwait.
A) J-Ieráclito, Dernócrito y Parrnénides son filósofos
D) nacionalización del canal de Suez por Egipto. griegos.
E) nacionalización del petróleo en Libia. B) Sócrates fue maestro de Platón.
65. Constituyeron la base del pequeño ejército con el C) "La Política" es obra de Platón.
que el general Andrés Avelino Cáceres desarrolló la "Cam- D) La Lógica es la teoría de la deducción y la mferencia.
paña de la Breña", de lucha contra la invasión chilena. E) Ser persona moral, es estar consciente de si.
A) Campesinos de las haciendas de Junín.
B) Docentes y estudiantes de la Universidad de San
72. Elija la propuesta incorrecta:
Marcos. A) El conocimiento vulgar es adquirido y usado empí-
C) Dueños de las haciendas serranas. ricamente.

D) Mineros de la sierra. B) La Axiología estudia los valores.

El Notables Iirneños y de otras ciudades C) La Filosofia es eminentemente problemática y crítica.


D) La ciencia y la tecnología se sustentan en la verdad y en
66. La Primera Revolución Industrial, que se desarro- la productividad respectivamente.
lló en Inglaterra sobre la base de la energía a vapor, im- E) Descartes, filósofo griego, dijo: "Pienso, luego existo."
pulsó las exportaciones peruanas de
73. Debido al cansancio provocado por el estudio, Juan
A)carbón B)cobre C)plata D)petróleo E)guano
confundió el 3 por el 8 al leer su código.Sufrió una
67 _Se ha establecido la relación entre personajes de la
A) alteración nerviosa. B) alucinación mental.
historia peruana, con su principal misión o función pú-
C) alucinación visual. D) ilusión objetiva
blica. Identifique la propuesta incorrecta.
E) ilusión subjetiva.
A) Femando de Abascal -Virrey
B) Túpac Arnaru 11- Precursor de la Independencia. 74. "La pobreza es más que un estado de depresión eco-
C) Guamán Poma de Ayala -Pintor. nómica y una forma de injusticia social. Influye sobre la
D) Garcilazo de la Vega -Literato conducta de los individuos estableciendo un patrón moral
E) José Carlos Mariátegui -Ensayista de vida entre la gente que se encuentra sometida a ella. Ge-
nera sistemas de valores, actitudes, estilos de pensar, sentir
68_ Inflación es: y reaccionar más o menos uniformes, que ponen a los gru-
A) subida del dólar. pos pobres en franca diferencia con los individuos de los
B) aumento del desempleo. estados sociocconómicos medios y elevados."
C) caída del nivel general de precios. Señale el titulo más adecuado al contenido de este párrafo.
D) subida del nivel general de precios. A) Economía, desigualdad y desarrollo.
E) emisión inorgánica. B) Ambiente y comportamiento humano.
C) Deterrninismo económico y pobreza extrema. 78. ¿Cuál de los siguientes hechos generó, reciente-
D) Aspectos psicosociales de la cultura de la pobreza. mente, discrepancias entre las autoridades de Cusco y
E) El ambiente y el yo de los estratos pobres. Puno'!
A) La explotación del gas de Camisea,
75. Diga el nombre del Secretario de Estado (Cansiller) de B) El trazo de la carretera Transoceánica.
Estados Unidos. Estuvo en el Perú, en una reunión de la OEA, C) El contrabando entre sus jurisdicciones.
el II de setiembre del 200 1, día del atentado en Nueva York.
D) La escasa afluencia turística.
A) Colin Powell D) Dick Cheney
E) La irrigación del valle de La Convención.
B) Rudolph Giuliani E) Donald Rumsfeld
C) George Patakis 79. ¿Cuál de las siguientes instituciones perunas se en-
carga, entre otras funciones, de salvaguardar los dere-
76. Marque la altemativa que no define correctamente
chos de autoria de personas y empresas'!
al término relacionado al Islam o a Afganistán.
A) Al Qaeda: Organización político-militar de Osama A) CONCYTEC B) INDECOPI
Bin Laden. C) SENCICO D) FONCODES
B) Ramadán: Lugar musulmán de oración. E)Instituto Nacional de Cultura (INC)
C) Pashtu: Grupo étnico mayoritario en Afganistán.
80. Señale la cantidad de alternativas correctas.
D) Yihad: Guerra Santa islámica.
Los cargos públicos que se mencionan son ocupados en la
E) Talibán: Grupo de estudiantes islárnicos que tuvo
actualidad por los personajes que se señalan para cada caso.
el poder en Afganistán.
1. Zar Anticorrupción-Martin Belaúnde Moreyra.
77. Presidente sudamericano que a un mes de los ata- 2. Ministro de Agricultura-Alvaro Quijandría Salmón
ques de Estados Unidos a Afganistán, los criticó dicien- 3. Zar Antidrogas-Ricardo Vega Llana
do: "No se pude responder al terror con más terror".
4. Ministro de la Presidencia-Roberto Dañino Zapata
A) Hugo Chávez (Venezuela)
5. Contralor General de la República-Genaro Matute
B) Fernando de la Rúa (Argentina)
Mejía.
C) Gustavo Noboa (Ecuador)
A) Todas son correctas B) Una correcta
D) Fernando Henrique Cardoso (Brasil)
C) Dos correctas D) Tres correctas
E) Ricardo Lagos (Chile)
E) Cuatro correctas

SOLUCIONARIO
APTITUD ACADÉMICA hecho de alcanzar el mayor grado de excelencia.

TÉRMINO EXCLUIDO Clave: A

1. MENUDO, está referido a algo de pequeño tamaño, 3. GLORIA, alude a la obtención de fama, prestigio,
diminuto de pequeña estatura. Desde otra concepción sig- popularidad por acciones meritorias, es decir convertirse
nifica de poca importancia, exiguo o mínimo. en una celebridad.
De las alternativas BREVE es la palabra que no guarda De las alternativas la palabra GOSE, es la que no guarda
relación, pues está referida a lo que dura poco y es de relación con las demás, pues significa placer o satisfacción.
corta extensión. Clave: D
Clave: C
4. TUMEFACCIÓN, término usado en la medicina y
significa hinchazón de una parte del cuerpo, las palabras
2. PERFECCIONAR es darle a una cosa mayor grado
que guardan relación son: tumor, edema o chichón.
de excelencia, es decir mejorar, afinar, pulir, depurar.
La palabra que no guarda relación con las demás es HE-
De las alternativas PROGRESAR significa mejorar o
RIDA.
avanzar positivamente no se refiere al proceso si no al Clave: B
~':;:~=~=~;:I
UN12002-1 APT, ACADÉMICA Y CULTURA GENERAL 1-=~~-'0
CONECTORES 12. Las oraciones especificamente se refieren al in-
somnio, la alternativa (e), no es pertinente a las otras
5. En el párrafo el autor sostiene que a pesar de que
porque se refiere a la salud en general.
algunos medios de comunicación se vendieron y mintie- Clave: A
ron, no es razón suficiente para ir en contra de la libertad
de prensa. El primer conector es de carácter concesivo 1'3. Las oraciones se refieren a los primeros vertebrados
(aunque) el segundo aditivo (además) y el tercero ad- que salieron del agua, y su capacidad para alimentarse
versativo (no obstante). de vegetales. La alternativa (B) no es pertinente a las otras
Clave: E porque trata el estudio de un fósil.
Clave: B
6. En el párrafo el autor señala porque el marxismo
14. Las oraciones desarrollan el tema de la Segunda
devino de este modo mecanicista, señalando las causas
Guerra Mundial. Siguiendo una relación de antecedente
que lo llevaron a ésto.
y consecuente. La alternativa (D) corta la secuencia, ade-
El primer conector es de caracter causal (A causa de), el más es redundante con la alternativa (E).
segundo conector es de cararter copulativo (y) y el Clave: O
el tercero de caracter explicativo (de este modo).
Clave: 8 COMPRENSiÓN DE LECTURA
15. La alternativa que mejor resume la lectura es la (8),
7. En el párrafo el autor señala que a pesar de la situa- pues trata los acontecimientos y hechos que suceden cons-
ción negativa del clima y el tiempo, no impide algunas tituyendo una revolución cintífica.
veces disfrutar de las lluvias matinales. Clave: B
"El primer conector debe ser de caracter concesivo (Aun-
que), el segundo copulativo (y) y el tercero de caracter 16. El texto trata acerca de los beneficios y limitacio-
adversativo (sin embargo). nes de los mantos acuíferos .
Clave: O Clave: B
8. En el párrafo se señala los motivos por los cuales una 17. El texto trata principalmente sobre el valor de los
empresa perdió la licitación y que a pesar de ello era la contextos durante la comunicación, señalando además
mejor alternativa. los componentes de una comunicación escrita; palabra,
El primer conector es de caracter causal (porque), en el frase, párrafo, capítulo, cuyo significado se capta por el
segundo debe ir una negación (ni) y en el tercero se debe contexto.
Clave: O
ubicar un adversativo (no obstante).
Clave: E
ORACIONES INCOMPLETAS
9. En el texto las dos primeras proposiciones deben estar
conectados por el copulativo (y); asi mismo están unidas a 18. La oración señala la relación de los corruptos con la
una tercera proposición por el conector adversativo (mas), muchedumbre frente al gobiemo con una finalidad extre-
y finalmente se admite un conector casual (porque). ma, la caída del gobiemo, la oración completamos asi:

Clave: C "Algunos dicen que los corruptos azuzan, a la muche-


dumbre para desestabilizar al gobierno, e incluso lograr
ORACIONES ELIMINADAS su caída".

10. Las oraciones desarrollan el tema sobre la máscara Clave: A


antigas, aparecida a partir de la Primera Guerra Mundial. La
19. La oración quedará formada en forma mas cohe-
primera oración (A) no concuerda respecto al sentido de las
rente y lógica así:
otras, pues el uso que se les dá no son propios de la máscara
"Para mantener la tradición el príncipe lucía el atuendo
antigas.
típico de sus ancestros".
Clave: A
Clave: A
11. Las oraciones describen al violín por sus caracte-
20. La oración más lógica queda formada así:
rísticas musicales (sonido).
"A pesar de haberse convertido en el paladín de la causa demo-
La oracion (D) no es pertinente con las demás pues des-
crática, sus adversarios lo tildaron como persona violenta".
cribe una parte fisica del violín.
Clave: O Clave: C
l. Según el trabajador del hospital Obaidullah, el nú-
COHERENCIA DE REDACCiÓN mero de muertos podría ascender a 15.
21. Para que la oraciones que tratan sobre el MA· H. Esa mañana de131, por lo menos se realizaron otros
NIERISMO, tengan una secuencia coherente, deben es- I I bombardeos contra Afganistán.
tar ubicadas así: Clave: C
IV. Estilo artístico que se originó en Italia en la segun-
da década del siglo XVI. 25. LA EROSiÓN
1. Constituye una reacción frente a los ideales de per- El orden lógico es el siguiente:
fección y equilibrio del clasicismo.
III. La superficie terrestre, varía conforme a una serie
n. Investiga en los campos de la expresividad, com-
de acciones externas.
placiéndose en lo desconcertante y artificioso.
V. Desgastan la materia de determinadas zonas.
111. En el arte de la imagen, las figuras se alargan, su-
gieren ingravidez, se atenúa la espacialidad. IV. Este fenómeno de disgregación se conoce con el
nombre de erosión.
V. En la arquitectura, estas características son menos
marcadas. 1. Se origina por distintas causas.
Clave: O 11. Una de ellas es la acción humana.
Clave: E
22. Los enunciados sobre LA MANO DEL HOMBRE,
tendrán una secuencia coherente, en el siguiente orden:
11. La mano del hombre constituye un importante re- SERIES NUMÉRICAS
curso expresivo.
V. Ampliando y matizando el lenguaje verbal. 26. 1 1
3" 3 -3 -15 1-791
IIl.En muchas ocasiones sustituye el lenguaje verbal.
~~~~
1. Esto ocurre, por ejemplo, en los sordomudos. '!"x2-1 -.!..x3-2 -3x4-3 -15x5-4
3 3
IV.Ellos se comunican exclusivamente gracias al abe-
cedario manual. :. -15x 5-4 = -79
Clave: E Clave: A

27. 5 . 12 8 . 10 12 . 7 A;B
23. los enunciados sobre la BOMBA
ben tener el siguiente orden lógico:
ATÓMICA de-
o o o
17 18 19
\..±/
20
V. El desarrollo de estas armas de destrucción a gran
escala comenzó a principios de este siglo. :. A+ B = 20
11. En su teoría de la relatividad, Einstein anunciaba Clave: C
ya la posibilidad de convertir la materia en energia.
28. Redefiniendo terminos:
l. La bomba atómica genera una gran cantidad de calor.
III.La radioactividad destruye o altera las células vivas.
.2.. 7 2. 13 21 37
2 2 2 2 2 2
IV. En Hiroshima perecieron 80 000 personas y los edi-
~~~~~
ficios comprendidos en un radio de dos kilómetros 20 21 22 23 24
desaparecieron. 22222
Clave: A 21 24 37
.. 2+2=2
24. Secuencia correcta sobre ESTADOS UNIDOS
BOMBARDEA HOSPITAL
Clave: O

IV. Fecha y lugar del bombardeo; el31 de octubre, bom- 29.


bas norteamericanas destruyen un hospital en
Kandahar.
V. Segun los voceros del talibán, el hospital era de la
Cruz Roja Internacional.
-21x6-4 = -130
111. También dos casas vecinas fueron destruidas
Clave: B


=1 UNI 2002-1 ,. PT ACADÉMICA Y CULTURA GENERAL I
30. Los S primero' números enteros po' • LIS: 34. Formando las figuras con las formas dadas:

¡r'2T45 6· f
7 J

h
1e-
g - -Necesariamente pues! y g
~oseen 6 casilleros contiguos.

-Unica posibilidad después de


elegir{ y s- Clave: C

el 4 )6 -Luego colocamos los 35. Si giramos 1800 en forma vertical, de abajo hacia
números que faltan de
le 7 f I gs Ir 2 I manera que no haya
arriba la primera figura, se obtiene:

números consecutivos
L
3 "5 en casilleros contiguos
La figura 2
(a+b) (c+d) - (e+h) if+g) =(4+6) (3+5) - (7+2) (1+8)
= 80 - 81 =-1
Clave: A

31. 6 ; 3 20 ; 8 42; 15 M;N La figura 2


\U \.V' \Y \U
----2.- -.2L ...2!-.-
~
9xl 14x2 19x) (19+S)x4 No se obtiene
1 ~5 +1 ~5 +1 +5 + la segunda figura

M +N = (19 + 5) x 4
= 96 _~o La figura 2

Clave: C

ANÁLISIS DE FIGURAS La figura 2


~ ...,
32. Desplegando, el sólido de acuerdo a la informa-
ción brindada. ~~ La figura de la clave "e" no sigue la misma ley de
/- T 1- formación.
M p / Clave: C
36. Para el triángulo:
Q
/'
s M

R ~
(oro inferior incógnito

.. La cara incógnita es "M". De un número: I ,2,3 ,4,5 ,6, 7 , S ,9.


Clave: E De cuatro números: (1; 2; 3; 4), (2; 5; ó; 7), (4;7;R;9)
El triángulo mas grande: (1,2,3,4,5, Ó, 7, S, 9)
33.
:. Número total de triángulos: 9 + 3 + I = 13
Para el rectángulo:

1 2
10
I) 4 5
6
11
-8 9
Clave: C 71
):;:;::;;::::;;=1 UNI 2002-1 APT. ACADÉMICA Y CULTURA GENERAL I '"
~~
Gome:Z

De un número: 1, 2 , 3 , 4 , 5 , 6 , 7 , 8 40. En cierto momento, las personas que asistieron a


De dos números: (10; 3) ; (11; 7) la fiesta:
De tres números: (6; 8; 9) • Parejas bailando: x
De cuatro números: (1; 2; 4; 5) • Mujeres sin bailar: 31
El cuadrado mas grande. (1; 2; 3; 4; 5; 6; 7; 8; 9;10;11) • Hombres sin bailar: II
:. Número total de cuadrados: 8 + 2 + I + I + I = 13 Total de personas que asistieron a la fiesta: 156

Clave: D ~ 156=2x+31+11
x = 57 (parejas)
37. Enumeramos los cuadros en sentido antihorario, ob-
Luego, los hombres que asistieron ala fiesta:
servando que el lugar vacio avanza un lugar excepto en "A".
(x + 11) = 57 + 11
vacío 3 vacío 1 vacío 2 vacío 3 vacío 4
= 68

@j.4.6.Bj4~E:;J4.6.EB.4O ~o4
21\
es;
3 2
O
3
@

o
2 3 2
0 <:)
3 2+3
41.
Clave: A

A) B) C) D) E)

Clave: A
I: 17'1'; I ~
a a a O O O
e e e 1 1 1
RAZONAMIENTO MATEMÁTICO i i i 2 2 2
o o o
38. Edad de César: x
Edad de Luis: y --
u
S
u
5
u
5
1- 1- 1-
10 10 10
Del enunciado: El máximo número de facturas que se pueden imprimir
será:
4x= 3i +2y
5x 5x 5x lOxlOx 10 = 125000
8x= 7y
Clave: E
Valores que pueden tomar x e y:
x=7 y=8 42. HORA Reloj que se retrasa 4

-------
-
x = 14 Y = 16 EXACTA ¿ninutos cada 2 horas,

¡600,m
-
x= 21 y = 24 5:40 a.m.

-
4:00 a.m. 3:44 a.m.
De la condición, si ambos son adolecentes tenemos:
x=14 y = 16 2:00 a.m.

-
1:48 a.m.
¡
-
0:00 horas 11:52 p.m.
De donde se deduce que Luis es el mayor por 2 años.
10:00 p.m. 9:56 p.m.
Clave: A 8:00 p.m. ~ 8:00 p.m.¡

39. El número de letras de la palabra INGIENERIA El reloj marcó por última vez la hora exacta. el dia ante-
es 10, y además: rior a las 8:00 p.m.
El número de letras 1 es 3. Clave: E
El número de letras N es 2.
El número de letras E es 2. 43. Si consideramos un grupo de 100n personas, de
los datos tenemos:
Entonces el número de permutaciones esta dada por:
PESO
lO! 4x5x6x7x8x9xlO PERSONAS CANTIDAD MEOIO
P,v = 3!x 2!x 2! 2x1x2x1 Adultos 10 n 80 kg
=151200 Jóvenes 70 n 60 kg
Clave: B Niños 20 n 40 kg
~~~.-~~~~
clíiñez
I UNI 2002-1 APT. ACADÉMICA Y CULTURA GENERAL I
De donde el peso medio del grupo está dado por el 45. Analizando gráficamente las prernisas:
promedio ponderado.
f' = 10nx 80/cg+ 70nx 60/cg+ 20nx 40/cg
m lOn+70n+20n "Todos los ingenieros
= 58Jcg son personas cultas."
Clave: C

RAZONAMIENTO LÓGICO
44. Del enunciado:
l. Ignacio no postula a canal Ill.
2. Ubaldo no postula a canal 11 "Algunos ingenieros no son científicos."
3. El egresado del colegio Francisco Bolognesi no Superponiendo gráficos se concluye que:
postula a canal 11.
4. El egresado del colegio Guadalupe postula a canal
III
5. Ignacio no es egresado de Humboldt.
COLEGIO
I CANAL
Bolognesi Guadalupe Humboldt
G
N 3
A
X X 4 5
X II
"Algunas personas cultas no son científicos."
e 1x 1 1X
I
X III
Clave: A
6 4 s
o V X X v
SUFICIENCIA DE DATOS
6. Ignacio es egresado del colegio Bolognesi y pos-
tula a canal V. 46. Datos generales:
COLEGIO = 100 klll
CANAL 'í' h
U Bolognesi Guadalupe Humboldt
Rendimiento:
B
A
2
X 32
X
2
X Il R = 50 /c~
gaLon
"
>'
L :.'\. 4
'>( '
O V III

O ./ '-/
/
-; V

7. Ubaldo es egresado del colegio Guadalupe y pos- A~I ~d ~BI


tula a canallll.
Se pide el costo total de la gasolina consumida en el via-
COLEGIO je.
N CANAL
Bolognesi Guadalupe Humboldt Información adicional:
I
e
3
X
4
X V Il I = 3 horas ; Precio por galón = 7 soles
O .,
L
A
:..~

X
4
X
7
>: III => d = v,, X I = 100 k;:, x J h = 300 km
s
&
)'"
4
X >" V
Los galones de gasolina consumida

1/ = 300klll
k =
será:

b gaiones
50~
Conclusión, Nicolas egresó del colegio Humboldt y postu- galon
la al canal 11. Costo total: C-
Clave: B soLes
e = 6 gaLones x 7 --L-'-
gaon
= 42 soles
Finalmente, es necesario usar ambas informaciones para 49. Si: !lE] = W2 +1
hallar el costo total de la gasolina consumida durante el
viaje. 0=Z2_1
Clave: D Aplicando los operadores a la expresión:

47. En la bolsa debemos hallar las canicas:


= 2R
R, A Y B:
@) ITm =14

®-
Datos: B ... (a)
Información:
1¡2-11.= 14
A = 60
A =3 R
(1)
(11)
(52 + Ir -1-[(12 -1)+ 1]= 14

(f-1t=9
~f-l=+3
~ 't=±2
Finalmente, tomamos el mayor: I= 2
Para hallar el número de canicas rojas (R) necesitamos
usar la información de (1) en (II): Clave: D
A =3 R
60= 3 R 50. Primero transformamos el operador.
~R=20 Gill = 4(u + 3)-11
Luego reemplazamos el valor de R en (a):
Si u+ 3 = x tenemos el nuevo operador.
B = 2(R)

= 2(20)
o = 4x- 11
Aplicamos este nuevo operador al ejercicio.
=40
~+~ =54
Finalmente, para hallar R, A Y B es necesario emplear
ambas informaciones. 4(W + 8) -11 + 4(W - 7) - 11 = 54
Clave: A
~W=9
Clave: B
OPERADORES MATEMÁTICOS
48. Si rnan = m-n ...(a) CULTURA GENERAL
a*b= %+2 ... (~) 51 . Analizando las oraciones:

En la igualdad: A) Va por la rivera del río.

(4t.6)*t = t Lo correcto es --7 (ribera)


B) Vive por el óvalo de la Brasil.
De (a): (4-6)*1 =± Usa correctamente
C) Tienes que cabar un hoyo.
vy B

-2* t =1. Lo correcto es --7 (cavar)


4
-2 1 D) Es un asunto tribial.
De (~): T+2=/f Lo correcto es --7 (trivial)

~t=~ E) El puesto está bacante.


7 Lo correcto es --7 (vacante)
Clave: B
Clave: B

!i
I UNI 2002-1 APT. ACADÉMICA Y CULTURA GENERAL lb ~ -c>
~

52. Analizando las claves: 55. Analizando las claves.


A) Ayer, todo cambió: la muerte dejar in su es- A) El Quechua fue la lengua en el imperio incaico.
tela. en el uso de mayúsculas, los nombres de las len-
El complemento circunstancial (aycr), puede ir o guas se escriben con minuscula.
no seguido de coma si va al inicio de la oración. B) El diario la república no se publicará el domingo.
Los dos puntos indican que lo que sigue es una
Los nombres própios de los periódicos y revistas,
explicación de lo ya mencionado.
etc. se escriben con mayúscula.
B) Ayer, todo cambió: ¡la muerte dejaría su es-
C) Los Ministros se presentaron ante el congreso.
tela!
Según la intención del hablante, los segmentos del La palabra ministro es una expresión genérica por
enunciado pueden llevar signos de afirmación, sin tanto se escribe con minúscula. Congreso es el nombre
que se produzca una incoherencia en el sentido. propio de una institución por lo tanto se escribe con
C) Ayer todo. cambió: la muerte dejaría su este- mayúscula.
la ... D) La población indígena vivía en América del Sur.
Si el hablante interrumpe su discurso, puede colo- Se usa correctamente las mayúsculas.
carse los puntos suspensivos. E) En la edad media, los Reyes eran muy poderosos.
D) Ayer, ¡todo cambió!: la muerte dejaría su
Los periodos o épocas se escriben con mayúsculas,
estela.
las jerarquías o cargos si está presente el nombre o
Similar a (B) , se puede escribir así la frase.
retleja un valor genérico con minúscula.
E) ¿Ayer todo cambió? ¿la muerte dejaría su es-
Clave: D
tela?
Los signos de interrogación se usan para hacer
56. El "niño Goyito" es el personaje central del cuento
preguntas por lo que es incoherente su presencia
"Un viaje" cuyo autor es Felipe Pardo y Aliaga quien
en la frase.
Clave: E destacó en poesías satíricas y articulas de costumbres.
Clave: C
53. Adjetivo Calificativo.- Son palabras que expre-
san, caracteristicas o cualidades del sustantivo. 57. La obra la Metamorfosis de Franz Kafka, tiene
Grados del adjetivo, según el interés subjetivo del ha- como personaje central a Gregorio Samsa, quien se
blante puede ser: convierte en un miserable insecto. esto representa la
Positivo (bueno). deshumanización del hombre, quién se despoja a sí mis-
Comparativo (más bueno que). mo de los valores inherentes al ser humano,
Superlativo (buenísimo). convírtiendose en una máquina de producción manipula-
do por el sistema capitalista, reflejándose asi lo absurdo
En el texto tenemos 4 adjetivos calificativos:
de la existencia humana.
2 positivos (fácil y difícil). Clave: B
I comparativo (más inteligente).
58. El cuadro muestra los principales yacimientos de
I superlativo (dificilísimo).
cobre en el pais:
Clave: A
Departamento Yacimiento
54. Analizando el párrafo:
• Los descubrimientos de finales del siglo XV (sujeto). Ancash Antamina
• ampliaron (N.P.). Arequipa Cerro Verde
• extraordinariamente (complemento circunstancial). Cajamarca Michiquillay
• El mundo conocido por los europeos (objeto directo) Cusco Tíntaya
El párrafo equivalente. Moquegua Cuajone
• Extraordinariamente (complemento circunstancial)
Del cuadro se observa que el yacimiento de Tintaya se
• Los descubrimientos a fines del siglo XV (sujeto) encuentra en el Cusco y no en Junín.
• ampliaron (N.P.)
Clave: D
• el mundo conocido por los europeos (objeto directo).
Clave: E
0::~¿:===1 UNI 2002-1 APT. ACADÉMICA Y CULTURA GENERAL 1::=!:li:[¡~!::';;:::::~g;;ieZ
59. El siguiente cuadro muestra la relación conecta, quiso lograr la paz entre hindúes y musulmanes pero en
entre el departamento y su resto arqueológico. su intento fue asesinado por un fanático hindú en 194R.
Clave: C
Departamentos Resto Arqueológico
Amazonas Fortaleza de Kuélap 64. El Estado de Israel fue creado en abril de I 94¡;, los
Ayacucho Centro ceremonial de Vilcashuamán árabes se opusieron, combatiendo en sucesivas guerras.
Cajamarca Santuario de Cumbemayo En 1956 el presidente egipcio Abdul Nasser nacionalizó
La Libertad Huacas del Sol y la Luna el canal de Suez e impidió el tránsito de armamentos
Lambayeque Pirámides de Túcume hacia Israel, provocando el inmediato ataque de este Es-
tado con el apoyo de Estados Unidos y el Reino Unido.
Clave: B Clave: D
60. Las islas Galápagos, estan ubicadas en el Oceáno 65. El general Andrés Avelino Cáceres desarrollo la
Pacífico, frente al litoral del Ecuador. Campaña de la Breña contra la invación chilena, con la
Clave: E finalidad de modificar las exigencias territoriales en el
tratado de paz. Formó un pequeño ejercito con campesi-
61. El petróleo es la fuente de energía más usada en el nos de las haciendas de Junin con el cual obtuvo victo-
mundo, en nuestro país hay tres sectores de explotación: rias en Pucará, Concepción y Marcavallc.
Sector Nor-oriental: Se ubica en el departamento de Clave: A
Loreto tiene una reserva aproximada del 64%, compren-
de la cuenca de los rios Corrientes y Pastaza. 66. La Primera Revolución Industrial se desarrolló
en Inglaterra, sobre la base de energía a vapor, que dio
Costa Norte: Se ubica en el departamento de Piura y
origen a un proceso de producción acelerado mediante
Tumbes, posee una reserva aproximada de 22%.
máquinas. Este proceso necesitaba desarrollar la
Zócalo Norte: Ubicada frente a la costa norte, represen- agrícultura para obtener las materias primas y así poten-
taeI14%. ciar las actividades agro-industriales por ello demanda-
Clave: A ban de abonos, y fue así que empezaron a importar gua-
no de África y luego de América, impulsando así las ex-
portaciones de el guano de las islas peruanas, convir-
62. La Antártida es un continente situado en el hemis-
tiéndose en la mayor fuente de divisas para el Estado
ferio sur, cubierta casi en su totalidad de hielo, formando
peruano.
asi el glaciar más grande del mundo que representa la
mayor reserva de agua dulce del mundo el 70% aproxi- Clave: E
madamente. Su clima es frío y helado llegando la tempe-
ratura en algunos lugares hasta -89°C, debido a estas 67. Guamán Poma de Ayala fue un cronista indio,
condiciones climáticas la .vegetación es escasa, consti- que en su obra literaria "Nueva Crónica y buen gobier-
tuida generalmente, por musgos y líquenes sin embargo no" denunció ante el rey Felipe 1I, los abusos contra los
su riqueza ictiológica es abundante constituidas por lo- indios con la finalidad de mejorar el gobierno en el
bos marinos, ballenas, krill, etc. Este continente es utili- virreynato, también es cierto que en su obra incluye, al-
zado exclusivamente para investigaciones científicas. gunos dibujos sobre todo sátiras contra las autoridades
españolas.
Clave: C Clave: C

68. La inf1ación.- Es la elevación del nivel general


63. La India, fue colonia de Inglaterra desde 1877. En de precios de un país, tiene como consecuencia la pérdi-
1947 logró su independencia gracias a su dirigente da del valor de la moneda nacional, aumento de la espe-
Mahatrna Gandhi, que impulsó la lucha mediante la no culación, etc. A la caída del nivel general de precios se
violencia y la desobediencia civil, inmediatamente des-
le llama deflación o inflación negativa.
pués de su independencia política, surgieron los nacio- Clave: D
nalismos entre los hindúes y musulmanes que dio origen
a la división territorial de la India, creándose al país de 69. Durante la década pasada el gobierno del Perú se
Pakistán y con ello una rivalidad entre estos paises, que acogió al Plan Brady para renegociar la deuda exter-
asta la actualidad prosigue, por la posesión de territo- na, que tiene con la Banca Privada Externa.
rios, siendo hoy potencias nucleares. Mahatma Gandhi Clave: E
I UN12002-1 APT. ACADÉMICA Y CULTURA GENERAL I ~0
70. Si hace un año un dólar se compraba a 3.55 soles, Estados Unidos estuvo representado por su Secretario
y hoy se compra a 3.45 soles, se puede decir que la de Estado Colin Powell.
moneda nacional se ha apreciado, pues está au- Clave: A
mentando su valor respecto a la moneda extranjera,
pues ahora se paga 0.10 soles menos por un dólar, lo 76.
contrario, la elevación del tipo de cambio se le denomi- - Al Queda, es la organización política-militar de Osama
na depresiación. Bin Laden, millonario saudi que fue preparado por la
Clave: E CIA para combatir a la invasión de la Unión Soviéti-
ca.
71. "La Política" no es obra de Platón es de su maes- - El Rumadán, es una fiesta religiosa musulmana don-
tro Aristótelcs donde este desarrolla sus tesis morales, de se realizan ritos, ayunos, etc. (Incorrecto¡
sociales, políticas y define al hombre como "animal po-
lítico", anteponiendo el bienestar individual al bienestar - Pashtu, es el grupo étnico mayoritario en Afganistán
colectivo. Platón escribio "La República". que profesan la religión musulmana (Islam).
Clave: C - Yihud o Guerra Santa Islárnica contra los infieles.

72. Rcné Descartes, nació en Francia (Turena) en - Talibán, grupo de estudiantes islárnicos que gober-
1596, filósofo nacionalista, en su obra Meditaciones Me- naron Afganistán desde 1994 hasta el 200 I liderados
tafísicas podemos encontrar su célebre frase "pienso, lue- por el Mulá Mohamed Ornar.
go existo".
Clave: E Clave: B
77. Hugo Chávez asumió la presidencia de Venezuela
73. Dentro de las alteraciones de la percepción se en- en febrero de 1999, quien ha buscado el acercamiento
cuentran: con personalidades enfrentadas con EE.UU., cómo Fidel
La alucinación o falsa percepción es aquella Castro, Sadan Fusein y Muamar Khadaffi. Chávez se pro-
donde no hay estimulo externo. nunció contra el ataque de los EE.UU., contra Afganistán,
La Ilusión, es una percepción equivocada donde diciendo: "No se puede responder al/error COI! más te-
rror".
la percepción no corresponde al objeto habiendo
de dos tipos: Clave: A
- Ilusión objetiva, donde la distorsión se debe a
7 8. El trazo de la carretera Transoccánica que uniria
las características del objeto.
la costa del Pacífico con el Brasil, con la finalidad de
- Ilusión subjetiva, donde la distorsión se debe facilitar la extracción de materias primas y la exporta-
a estados internos del sujeto (estado emocio- ción brasileña al continente Asiático, ocasionó discre-
nal, estado de salud, cansancio). pancias entre las autoridades del departamento del Cusco
y Puno, disputandose el quedar incluídos en el trazo
Por lo tanto si Juan debido al cansancio confunde el 3
proyectado para la carretera transoceánica.
por el 8 al leer su código, sufrió una ilusión subjetiva.
Clave: B
Clave: E
. 79. INDECOPI o INSTITUTO NACIONAL DE DE-
74. La pobreza no sólo significa escases de recursos FENSA DE LA COMPETENCIA Y DE LA PROTEC-
económicos e injusticia social, sino influye sobre la CIÓN DE LA PROPIEDAD INTELECTUAL, es la ins-
conducta del individuo, estableciendo una cultura de titución creada por el Estado para actuar contra la "pira-
la pobreza, donde se ven aspectos psicosociales de tería", comercialización de productos adulterados, publi-
la gente sometida a ella como sus valores, sus actitudes, cidad engañosa y para salvaguardar los derechos de
estilos de pensar, etc. que los diferencia de los indivi- autor debidamente patentados.
duos de los estados socio-económicos medios y altos.
Clave: B
Clave: D
80. Roberto Dañino es el Primer Ministro (Premier) y
75. La VIII Asamblea General de la O.E.A. se no ministro de la Presidencia quien ocupa ese puesto es
realizó en Lima, y fue convocada para aprobar la Carta Carlos Bruce, las demás alternar ivas son correctas, por
Democrática Interamericana que se firmó el Il de se- lo tanto cuatro son correctas.
tiembre del 200 1, día del atentado en Nueva York, los Clave: E
0''=9== UNí2002-11 APT. ACADÉMICA Y CULTURA GENERAL 1:::=:~:~:¡::;~:~L.~;",{",n!!!le",Z,-,

~::~~[J ~ ~ ~~
AeT1TUD ACADÉMICA y CULTURAGEfJERAL

APTITUD ACADÉMICA D) Los recursos técnicos.


E) La sensación de algo real que transmite al público,
COMPRENSiÓN DE LECTURA 3. "En el caso de la arquitectura latinoamericana, la con-
1."En ninguna civilización la vida urbana se ha desa- dición sincrética en lo cultural, transforma todas las im-
rrollado independiente del comercio y de la industria. Esta posiciones estilistico-tipológicas que acarrea el proceso
leyes insensible al clima, pueblos o religiones, como a colonizador: ello queda bien evidenciado en la arquitec-
la diversidad de épocas. Su universalidad se explica con tura de los templos que, configurados en las pautas ba-
la necesidad. En efecto, un conjunto urbano no puede rrocas que preconiza la iglesia, se constituye en lo que
subsistir si no se importa alimentos desde fuera. Pero a algunos denominaban barroco americano".
esta importación corresponde una exportación de pro- De acuerdo con el texto, se deduce que:
ductos manufacturados que constituyen el contra valor" .
A) La iglesia católica desarrollaba pautas estilístico
Tomando en cuenta el texto, indique el enunciado co- tipológicas apropiadas a Latinoarnérica.
rrecto:
B) La Contrarreforma propugna la implantación del
A) La universalidad de la vida urbana se explica en . estilo baITOCOamericano.
base a la necesidad del hombre por alimentarse y C) La cultura arquitectónica estuvo guiada por la igle-
comerciar.
sia y determinó la construcción de templos.
B) Es un hecho universal que, para alimentar a la vida D) El sincretismo cultural está referido a la asimiliación
urbana, se requiere importar un contravalor.
de los modelos y su transformación de acuerdo a su
C) Si una ciudad no importa productos, no puede ex- realidad.
portar alimentos ni constituir un contravalor. E) La arquitectura en Latinoamérica desarrolló prefe-
D) Es universal que exista una ley por la cual la vida rentemente tipologías en estilo barroco americano.
urbana es insensible al clima.
E) La vida urbana se desarrolla estrechamente rela-
ORACIONES INCOMPLETAS
cionada con el comercio y la industria.
4. Complete el enunciado con la alternativa correcta.
2. "El cine ha superado al teatro en recursos técnicos y
Decidieron darles un plazo para _
ha sustraído de las salas teatrales masas espectadoras con
al nuevo régimen de tributación fraccionada.
el señuelo de una compensación ideal. Sin embargo, hay
algo que el cinematógrafo no ha podido dar: Transmitir A) prudencial - asociarse
al público una sensación real de que en el mismo instan- B) adicional afiliarse
te en que se alza el telón algo comienza a crearse, una
C) prorrogable - retirarse
ilusión envolvente brota, algo nace".
D) indefinido - distanciarse
Según el autor, el cine no ha superado al teatro en lo
referente a:
E) perentorio - acogerse

A) La escenografia real y vistosa del teatro. S.Complete el enunciado con la alternativa adecua~la.
B) La interacción que permite el teatro entre el direc-
La agresión también fOI111aparte de . De-
tor y los actores.
bemos hallar un equilibrio entre nuestros y
C) La sensación de algo ideal que transmite el teatro. la manera como los _
IUNI 2002-11 APT ACADÉMICA Y CULTURA GENERAL I
A) la naturaleza 'iumana - instintos - ex: o<lmos. V. La sangre se recoge posteriormente en una red capi-
B) la realidad natural - deseos - disfrutamos. lar y regresa al corazón.
A) 1 - II - III - V - IV
C) las ideas - negocios - precisamos.
B) II - 1 - IV - V - II!
D) la ciencia - semejantes - seleccionamos.
C) 11 - IV - I - III - V
E) la política - descendiente - conocemos.
D) Il - 1 - IV - III - V
6. Complete el enunciado con (a alternativa correcta.
E) I - III - V - II - IV
Era un hombre , pero ahora su cuerpo se
hallaba totalmente _
9. Lea los enunciados y elija la alternativa que estable-
A) tranquilo - inerte ce la secuencia correcta de las oraciones, según el título.
B) leal - quebrado LOSSAURIOS
C) frágil - delicado 1. Algunos saurios también cazan animales pequeños.
D) gentil - recio II. Los saurios son vertebrados pertenecientes a la fa-
E) fornido - raquítico milia de los reptiles.
111. Los saurios poseen cinco dedos provistos de garras.
IV. La mayoría de los saurios son insectívoros.
COHERENCIA DE REDACCiÓN
V. Los saurios se caracterizan por tener un cuerpo alar-
7. Lea los enunciados y elija la alternativa que establece gado y estrecho.
la secuencia correcta de las oraciones, según el título.
A) II - V - 1 - IV - III
DARWlN B) V - II - IV - 1 - lfl
l. La selección natural significaba que sólo sobrevi- C) IV - I - III - V - 1I
vían los individuos mejor dotados de cada especie.
D) V - 1 - III - IV - II
H. Darwin denominó a este fenómeno: selección natu-
ral. E) 1I - V - III - IV - I

]11. Según Darwin, los individuos más débiles sucum- 10. Lea los enunciados y elija la alternativa que estable-
bían. ce la secuencia correcta de las oraciones, según el título.
IV. Darwin sostuvo igualmente que la naturaleza se en-
EL SISTEMA PARLAMENTARIO
cargaba de paliar las altas tasas de reproducción.
V. Darwin afirmó la existencia de modificaciones de 1. El sistema parlamentario puede ser de dos tipos:
Carácter hereditario en cada especie. dualista y monista.

A) V - IV - 11 - III - I II. Como sistema de organización, éste surgió en Ingla-


terra en el siglo XVII.
B) III - I -V - II - IV
]11. En el sistema dualista, el poder ejecutivo se divide
C) V - IV - 11 - I - 1II entre el Jefe del Estado y el Parlamento.
D) III - V - IV - I - II IV. El Parlamentarismo es un sistema de organización
E) V - IV - 1lI - I . - II política.
V. En el sistema monista, el Jefe del Estado se limita a
8. Lea los enunciados y elija la alternativa que estable- actuar como cabeza visible.
ce la secuencia correcta de las oraciones, según el título.
A) IV - 1I - 1 - V - III
EL HíGADO B) 1 - III - V - 1I - IV
I. Unido al aparato digestivo, es una víscera muy im- C) IV - II - 1 - III - V
portante en los procesos metabólicos.
D) 1 - IV - II - III - V
11. Es el órgano más grande del cuerpo.
E) 1 - IV - V - III - Il
111. Las células hepáticas reciben sangre oxígenada del
corazón. 11. Lea los enunciados y elija la alternativa que estable-
IV. También actúa como centro de desintoxicación. ce la secuencia correcta de las oraciones, 'según el título.
ªªªª~Mh~.I UNI 2002-11 APT. ACADÉMICA Y CULTURA GENERAL I :A
~Illld
ctúleZ

LOS VIENTOS 16. Hallar el valor de la incógnita:


1. Su formación se debe a que los rayos solares calien- 4,4,1,2,0,2,'1
tan más en unas regiones de la Tierra que en otras.
A) O B) 1 e) 2 Dl4 El x
n. Es el aire en movimiento.
Hl, Se distinguen dos tipos de viento según su altura: 17. En el círculo adjunto hay una serie numérica. ldcn-
los de superficie y los de altura. tifique la secuencia del cuadrante UN 1.
IV. Se produce en las capas bajas de la atmósfera.
V. La dirección de los vientos es modificada por dife-
rentes fuerzas.
A) I - II - IV - V - III
B) II - [ - [V - [[[ - V
e) I - II - IV - [11 - V
D) II - [V - I - V - III
E) Il - III - I - IV - V A)~ B)~ C)~
5 7 5 4 . 6 13
6 6 7
SERIES NUMÉRICAS
1 2. Indique la alternativa que completa el cuadro siguiente: D)~ E)~
6 6 6 11
1/4 1/2 1/8 1/24 3 7 7
2/3 1/2 1/3 2/3 1/2
6 x 48 Y 6 ANÁLISIS DE FIGURAS
x y x y x y 18. Indique la alternativa que debe ocupar el casi llcro UN 1.
A) 6 8 B)6 6 C) 5 8
D)7 9 E) 8 8

13. Indicar la alternativa que continúa adecuadamente


la serie siguiente:
~~ 1 00 EJ U ~U ~~ 1

00 00• 00
3,2,9,4,15,6,21
N 1 N N
A) 6 B) 8 e) 12 D) 18 E) 28
~ • 1 •
~N U U

14. Indique la alternativa que continúa correctamente A) B) e) D) E)


en la siguiente secuencia:
7776,625,64,9 19. Dado el dibujo. identifique a cuál de las figuras
corresponde, sabiendo que es la representación de una
A)O B) 1 C) 2 D)3 E) 4
vista desde arriba.

15. Los números consignados en los tres cuadros, cum-


plen una misma relación. Identifique dicha relación y de- 11111
termine el valor de A + B.

8 20
C1feeJ ~ d1EJ- 5JC) ateeJ
A) B) e) Dl El
~ 18
~ 17 11 20. Dadas las siguientes figuras, identifique aquella to-
-
mada de un ángulo distinto al resto.

~
tct:=±::d @
A)9 B) 10 C) 11 D) 12 E) 14 A) B) e) D) E)
,,$.1 UNI 2002-11 APT. ACADÉMICA Y CULTURA GENERAL ~",,'¡¡an2~==

21 _Indique la alternati va que no tiene relación con las A) WE B) L Cl WU D) W El ZU


demás. U W E Z E

28. Un ómnibus salió del paradero A con destino al


paradero B, en el trayecto se detuvo en 1/ paraderos. Un
pasajero que viajó de A hasta B observó durante el tra-
A) Bl C) D) El yecto lo siguiente:
22_ Seleccione la figura que no tiene la misma caracte- • En el paradero que subía gente no bajaba ninguno.
rística de las demás. • En 9 paraderos subió o bajó gente.
• En 6 paraderos no bajó ninguno.
• En 9 paraderos no subió ninguno.
Según esto 11 es:

Al Bl C) D) El A) 12 B) 13 C) 14 D) 15 E) 17

23. Identifique la columna "x" dentro de [as alternati- 29. Enrique compró un automóvil en "1/1" soles. Pasa-
vas propuestas. do algún tiempo decidió venderlo, para lo cual incrcmentó
su valor en "1/" por ciento del precio original. Si José le
pidió un descuento del "1/" por ciento, que fue aceptado

11 ~~~~~
por Enrique, ¿cuál fue el precio de venta final'!

A) l1I+n B) 10000+111 C) 11(10000-11I2)


1000 11 10000

111(10000 - n2) n(10000-1/2)


A B C D E
D) 10000 E) 10000

RAZONAMIENTO MATEMÁTICO
24. En una clase de 12 alumnos, el promedio de las RAZONAMIENTO LÓGICO
notas de los 6 más aplicados es 18 y el de [os restantes es 30. Respecto de [as calidades de los productos: Q, T, Z
14. Hallar el promedio del tercio inferior, si los prome- y P se conoce lo siguiente:
dios de los dos tercios restantes son 18,5 y 15,5.
• La calidad de Q no es mejor que la de T.
Al 14 B)13,5 C)14,5 D) 15 E) 13
• Z es de calidad inferior a Q, pero mejor que P.
25. En un club deportivo hay 70 jugadores. De éstos, Luego de evaluar dicha información, podemos afirmar
50 juegan fútbol, 32 juegan Ping Pong y 27 juegan que:
básquet. Si sólo 8 practican los 3 deportes, ¿cuántos prac-
A) El producto Q no es mejor que Z.
tican exactamente un deporte'!
B) La calidad de Z es superior a Q.
A)36 B)37 C)38 D)39 E) 40
C) El producto P no es de la más baja calidad.
26. Entre las personas menores de una familia, cada
D) El producto Z tiene mejor calidad que T.
niño tiene tantos hermanos como hermanas, pero cada
niña tiene dos veces más hermanos que hermanas. El E) El producto Q no es el de más baja calidad.
número de personas menores de la familia es:
31. Una familia está formada por los padres y cuatro
A)7 B)5 C)6 D)4 E) 9 hijos. Dos son hijos de padre y madre, uno es sólo hijo
de padre y el otro sólo de madre. Dos de los abuelos/las,
27. El precio de una calculadora y el precio de un libro
están en la relación t. Si con Z soles, se compran
han fallecido. ¿Cuál es el mínimo número de abuelos vi-
vos?

exactamente W de dichos libros, ¿cuántas calculadoras A)2 8)3 C)4 D)5 ElR
se pueden comprar con Z soles?
=:25=:=;"-*-~JUNI 2002-11 APT. ACADÉMICA Y CULTURA GENERAL I
INSUFICIENCIA DE DATOS TÉRMINOS EXCLUIDOS
32. Dos móviles A y B se desplazan en el mismo
37. Elegir la palabra que debe excluirse por no guardar
sentido sobre una línea recta. Estando B 200 m. adelan-
relación con la palabra base:
tado respecto a A. Determinar a partir de ese instante el
tiempo que tarda A en alcanzar a B, considerando las DEGRADAR
siguientes informaciones: A) Envilecer B) Deshonrar C) Enaltecer
1) La velocidad de A es el triple de la velocidad de B. D) Humillar E) Deteriorar
Il) La velocidad de B es de 10 mis
Para resolver el problema: 38. Elegir la palabra que debe excluirse por no guardar
relación con la palabra base.
A) La información I es suficiente.
INTERPELAR
B) La información Il es suficiente.
C) Es necesario utilizar ambas informaciones. A) Cuestionar B) Implorar C) Inquirir
D) Cada una de las informaciones por separado, es su- D) Preguntar E) Requerir
ficiente. 39. Elegir la palabra que debe excluirse por no guardar
E) Las informaciones dadas son insuficientes. relación con la palabra base.

33. Calcular el área de la región sombreada de la figu- CONFABULACIÓN


ra, si se dispone de las siguientes informaciones: A) Contradicción B) Maquinación C) Conjura
D) Intriga E) Conspiración
1) AB = AC = 5m
40. Elegir la palabra que debe excluirse por no guardar
I1) El triángulo ABC es
relación con la palabra base.
rectángulo en A.
INGRESO
Para resolver el problema.
A) Admisión B) Asignación C) Entrada
A) La información 1 es suficiente.
D) Renta E) Pensión
B) La información Il es suficiente.
C) Es necesario utilizar ambas .informaciones.
D) Cada una de las informaciones por separado, es su- CONECTORES
ficiente. 41. Elija la alternativa que, al insertarse en los espacios
E) Las informaciones dadas son insuficientes. en blanco, dé sentido pleno al texto.
El Mundial de Fútbol hecho comercial, fue
OPERACIONES MATEMÁTICAS un éxito, en lo deportivo, hay serias dudas.

A) en tanto - debido a que


34. Si a * h = ~: b ,calcular 1*8 B) más que - no obstante

I I C) como - mientras que


A) 16 E) 243 D) en tanto - si bien
E) como y
35. Si se define: ~ = 2a +3 ,determinar el
42. Elija la alternativa que, al insertarse en los espacios
valor de Wen: 14W 1=[2!J + 14 en blanco, dé sentido pleno al texto.

A)3 B)5 C)7 D)9 E) II La televisión por cable variada y especiali-


zada, es menos comercial, menos alienante.
36. Si
2
M 1 =!L±l
2ac
calcular: A) en tanto - por lo tanto
B) a pesar de - también
S = /41 + /42 + 4 M3 + ... + (n+I)M" C) puesto que - por tanto

A)n B) 1/; I C) -2-


1/-1
E)}
D) además de
E) además de
- por lo tanto
- aunque
.~ mi;;! UNI 2002-11 APT. ACADÉMICA Y CULTURA GENERAL 1~~::;.0
43. Elija la alternativa que, al insertarse en los espacios O) La Hagiografia tenía como objetivo fijar qué san-
en blanco, dé sentido pleno al texto. lOS podían ser objeto del culto.

"Estados Unidos es el mayor productor de anhidrido car- El Gracias a la Hagiografía paulatinamente fue
bónico, produce casi la cuarta parte del to- incrcrnentándose el culto a los santos.
tal mundial, su emisión per cápita es seis
47. Elija la oración que es redundante o no pertinente
veces mayor que el promedio del resto del mundo.
respecto al sentido de las otras.
_____ es el principal responsable del ataque al
ecosistema" . A) Los mensajes publicitarios pasan por varios proce-
sos antes de su emisión.
A) además - entonces - Finalmente
B) Los mensajes publicitarios eran comunes ya en la
B) pues - pero - Por ejemplo
antiguedad.
C) si - pese a que - Luego C) Esto se demuestra con los hallazgos de Pornpeya.
O) y - o - Entonces O) Pero en sentido moderno, la publicidad se afianzó
E) ya que - por lo que - Por lo tanto con las grandes transformaciones económicas.
E) Para este desarrollo, tuvo mucho que ver la inven-
44. Elija la alternativa que, al insertarse en los espacios
ción de las técnicas de impresión rápidas.
en blanco, dé sentido pleno al texto.
Su sensibilidad fina la inclinaba a la ternura, _ 48. Elija la oración que es redundante o no pertinente
no llegaba a ningún arrebato su imagina- respecto al sentido de las otras.
ción estaba siempre dentro de la realidad le A) La meteorología es una ciencia.
hacía ver las cosas sin deformaciones.
B) Esta disciplina estudia los fenómenos atmosféricos.
A) mas - pues - o C) La base de esta ciencia la forman los llamados ele-
B) si - ni - y mentos meteorológicos.
C) pero - porque - y O) En el estudio de estos elementos, la meteorología
O) aunque - sin embargo - o hace uso de distintos instrumentos.
E) si bien - y - pero E) Los instrumentos que utiliza el científico van cam-
biando con el tiempo.
45. Elija la alternativa que, al insertarse en los espacios
en blanco, dé sentido pleno al texto. 49. Elija la oración que es redundante o no pertinente
respecto a las ideas de las otras oraciones.
Carmen no era tan calculadora como su padre _
A) El examen de admisión consta de 3 pruebas.
tan impulsiva como su madre; entre la premedi-
B) La primera es de Aptitud Académica y Cultura Gc-
tación implacable de aquél ciega arremeti-
neral,
da de ésta, ella era una muchacha tan singular.
C) La segunda es de Física - Química y la tercera es de
Ala - y - Y O) aunque - mas - o Matemática.
B) ni - pues - 111 E) pero - si - o O) Todas las pruebas son difíciles.
C) ni - pero - y E) Los resultados se publican el mismo día de la prueba.

50. Elija la oración que es redundante o no pertinente


respecto al sentido de las otras.
ORACIONES ELIMINADAS
A) Las gramíneas han influido de manera decisiva en
46. Elija la oración que es redundante o no pertinente la historia humana.
respecto al sentido de las .otras. B) La recolección de sus semillas era una manera de
A) El término Hagiografía hace referencia al estudio buscar sustento.
de la vida de 'los santos. C) Esta recolección culminó cuando el hombre domes-
B) La Hagiografía se convirtió en una rama de la his- ticó ciertas especies gramíneas.
toria de la Iglesia. O) El hombre, más tarde, extendió su dominio hacia la
ganadería.
C) La Iglesia Católica siempre ha combatido el culto a
los santos. E) Las especies domesticadas se llamarían posterior-
mente cereales; es decir, fruto de algunos gramíncas.
E) Acciones bélicas dirigidas por Cáccrcs contra los
CULTURA GENERAL chilenos, en la sierra central del país.

51. Señale la alternativa que tiene una posición incorrcc- 55. A fines del siglo XIX, muchos inmigrantcs y 'sus
ta sobre el Islam, la religión que propugna las sumisión a hijos ya participaban activamente en la vida económica
Alá: del país. Uno de ellos fue Carlos Fcrmin Fitzcarrald, quién
A) Abraham es uno de sus patriarcas. promovió la explotación del:
B) Es monoteísta. A) algodón B) azúcar el caucho
C) La Meca es su principal ciudad santa. O) cobre E) guano
O) Su libro sagrado es el Corán.
56. Señale la alternativa donde por lo menos una de las
E) Mahorna, contemporáneo de Jesús, es su principal
palabras subrayadas es tú mal escrita.
profeta.
A) El ejército no pudo debclar la rebelión.
52. Señale la alternativa incorrecta con relación a las B) Los de esa tribu no pierden el tiempo en asuntos
razones por las quc la Iglesia Católica tuvo papel pre- tribialcs.
ponderante en la Edad Media y logró considerable in- C) Ganó el Premio Nóbcl a pesar de ser novel en J;¡
fluencia y autoridad en aquella época. materia.
A) La idea irnpcrantc del predominio del poder espi- O) No lo van a absolver pese a que está alabando al
ritual sobre el poder temporal. Gobierno.
B) La férrea unidad, disciplina y organización del Cle- E) Ticnes que hervir el agua antes de remojar J;¡ hícd1<1.
ro en todos los niveles.
C) El resurgimiento de las ciudades y la formación 57. Sólo en uno de los siguientes enunciados hay uso
de la burguesía. correcto de las preposiciones. i.Cuúl es'!
O) El Clero era la única clase ilustrada y dirigía las A) Pedro salió de despedirse de Alía.
escuelas, colegios y universidades. B) Tu madre es tú harta con tus quejas.
E) Sus grandes riquezas y el derrumbe del Imperio C) Aníbal sc mudó y ahora vive por la zona norte.
Romano de Occidente.
O) Me caí y en el colmo me rodé las escaleras.
53. i.En qué período de la historia se encontraba Euro- E) El fiscal examinó una a una las momias.
pa cuando en los Andes se desarrollaba la cultura Huari?
58. Señale en qué caso no está correctamente definido
A) Edad Antigua
el prefijo correspondiente:
B) Edad Media
A) a : falta, negación
C) Humanismo y Renacimiento
B) eis __ : del mismo lado
O) Edad Moderna
E) Edad Contemporánea C) sin : desunión
D) pre __ : anterioridad
54. Sobre la "Campaña de la Breña", elija la proposi- E) peri __ : alrededor de
ción correcta.
A) Trágico episodio de la guerra civil entre Cáceres e 59. Señale en qué caso no está correctamente definido
Iglesias, culminó en el distrito limcña que lleva su el sufijo correspondiente:
nombre. Al _ gamo : que come
B) Épico episodio contra los chilenos, bajo la sagaz B) _ pode : que tiene pies
dirección de Andrés A. Cáceres, en la costa norte C) _ ptcro : que tiene alas
del país. O) _ patía : enfermedad
C) Conjunto de acciones bélicas dirigidas por Iglesias, E) fero: que produce
para combatir a los chilenos en el centro del país.
O) Exirosas acciones de Cáeeres contra los chilenos, 60. "EUFEMISMO:
que culminaron en el distrito limeño que lleva su Manifestación suave o decorosa de ideas cuya recta y
nombre. franca expresión sería dura o malsonante".
~w~r~;~~:::::=~::~=1
GOllleZ
UNI 2002-11 APT. ACADÉMICA Y CULTURA GENERAL ,::~;:;:;::::~
~

A) Dirigiéndose a las damas, el policía p.rra no ser A) Son ríos de corta longitud. El más largo es el río
chocante, recurrió a un eufemismo. Majc». con sólo 450 km. de longitud, seguido por el
B) Entré en duda, no sabía si contar la verdad o recu- rio Santa.
rrir a algún eufemismo. B) Son ríos mayormente torrentosos, es decir, durante
C) El ruido era infernal, y para ser escuchado, recurrí algunos meses del año sus aguas descienden cstrcpi-
a un eufemismo. tosamente debido a la fuerte pendiente de su cauce.
D) Un tono eufcmístico caracterizó a los discursos, C) Son ríos de régimen regular, es decir, mantienen un
luego de la fatal decisión. caudal constante durante todo el año, con muy pc-
E) El médico, prudentemente, utilizó un eufemismo queñas variaciones.
frente al paciente.
D) Son ríos que tienen su origen en la Cadena Occi-
61. Indique en cuál de las siguientes obras el autor del dental de los Andes.
texto es el protagonista de la obra: E) Los ríos Marañón, Huallaga y Ucayali no pericnc-
cen a la Región Hidrográfica del Pacífico.
A) "La lIiada" de Hornero.
B) "La Divina Comedia" de Dante Alighieri. 66. Uno de los siguientes picos no corresponde a la
C) "Aventuras del Ingenioso Hidalgo Don Quijote de Cadena Occidental de los Andes. ¿Cuúl es'!
la Mancha" de Miguel de Cervantes Saavedra.
A) Monte Meiggs B) Salkantay C) Amparo
D) "Fausto" de Johann Wolfgang Goethe.
E) "El Señor Presidente" de Miguel Angel Asturias. D) Coropuna E) La Viuda

62. La novela "Conversación en la Catedral" de Mario 67. De las ciudades que se mencionan, la que está a
Vargas L10sa trata de: mayor altitud sobre el nivel del mar:

A) Una crítica a la religión en el Perú. A) Cerro de Paseo en el Perú


B) La remodelnción del edificio de la Catedral de Lima. B) Potosí en Bolivia

C) Reflexiones sobre la situación social y política del C) Toluca en México


país. D) Tulcán en Ecuador
D) El impacto social de las guerrillas en el Perú en la E) Tunja en Colombia
década del 60.
E) La guerra del fin del mundo.
68. Las ciudades palestinas de Yenin, Rarnala y Naplusa,
se hicieron conocidas en abril de este año, porque
63. Señale la alternativa incorrecta: A) Estados Unidos quiso instalar bases militares en
Las unidades para la protección y conservación de los ellas.
ecosisternas, flora, fauna, belleza paisajística o lugares B) Fueron objeto de intensos ataques militares israelitas.
históricos, se clasifican en: C) Recibieron la visíta del presidente norteamericano
A) Reservas Nacionales George Bush.
B) Parques Nacionales D) Se descubrió petróleo en las zonas aledañas.
C) Santuarios Históricos m Sufrieron el incendio de templos islárnicos.
D) Reservas Ecológicas
E) Santuarios Nacionales.
69. En la base militar norteamericana de Guantánamo,
ubicada en territorio cubano, están encarcelados los
64. La distancia que existe entre un punto cualquiera A) Acusados de crímenes de guerra en los Balcancs.
de la Tierra y el ecuador se denomina:
B) Cabecillas de la "mafia rusa" detenidos en Estados
A) Latitud B) Equinoccio C) Altitud Unidos.
D) Longitud E) Solsticio C) Guerrilleros de las FARC capturados en Colombia.
D) Miembros de Al Queda capturados en Afganistún.
65. Respecto a los ríos de la Región Hidrográfica del
E) Narcotraficantes colombianos cxrradiiados a Esta-
Pacífico o Vertiente del Pacífico, indique la alternativa dos Unidos.
que contiene un enunciado incorrecto.
::62~~:W~1 UNI 2002-11 APT. ACADÉMICA Y CULTURA GENERAL I::::::::;!~;:::~;i:
me: lJ

70. ATPA son las siglas en inglés de una ley renovada


y ampliada recientemente por el Congreso notteameri- 76. En el Perú la mayor de la Población Económica-
cano, que favorece a países como el Perú, porque facili- mente Activa (PEA) se encuentra
ta el ingreso a Estados Unidos de A) Adecuadamente empleada
A) Familiares de inmigrantes latinoamericanos. B) Totalmente desernpleada
B) Productos de los países andinos afectados por el C) Desempleada
narcotráfico. D) Empleada
C) Trabajadores de los países andinos afectados por el E) Subempleada
narcotráfico.
D) Transporte pesado proveniente de países latinoame- 77. El enunciado "nada es verdad o mentira, todo es
ricanos. según el cristal con que se mira", lo sostiene un
E) Vuelos de aerolíneas latinoamericanas. A) Empirista B) Idealista C) Realista

71. Las vías transoceánicas son D) Relativista E) Racionalista

A) La alternativa para el desarrollo descentralizado en 78. Indique si son proposiciones atómicas (A) o
el Perú.
moleculares (M)
B) Ejes estratégicos de infraestructura de transporte
internacional. l) Juan es ingeniero
C) Sistemas de comunicación de la Cuenca del Pacífico. ll) La lógica y la Matemática son ciencias formales.
D) Sistemas de comunicación de los países mediterrá- lll) No hay desaprobados en el examen.
neos. IV) La universidad es grande
E) Sistemas de comunicación para los países costeros. V) Si te has sacado 15, entonces puedes ingresar.
A) AMMAM B) AAMAM C) MAAMM
72. Actualmente, uno de los estudios arqueológicos más
importantes ha permitido demostrar que -,- D) MMAMA El AMAMM
es la ciudad más antigua de América (~3000 años A.e.)
79. Respecto a las relaciones entre personalidad, tem-
Elegir la alternativa conecta: peramento y carácter, señale que alternativa no corres-
A) Puruchuco B) Sipán C) Kuélap ponde:

D) Caral E) Pacatnamú A) La personalidad es una organización dinámica inter-


na de lo que somos, y se manifiesta en el carácter.
73. Nombre del cuarto tenor peruano, triunfador en la B) La impulsividad y la emotividad del individuo de-
Scala de Milán, que debutó en 1996.
penden de la personal idad.
A) Ernesto Palacios . D) Juan Diego Flores C) En la base del carácter, se encuentra el tempera-
B) Alejandro Granda E) Francisco Petrozzi mento.
C) Luis Alva D) En la personalidad, se concentran factores heredi-
tarios, ambientales y personales,
74. De acuerdo con la Constitución Política vigente, el
E) El carácter es el conjunto de rasgos singulares del
Estado no tiene la siguiente función en la economía:
ser humano en su comportamiento ante sí y los
A) Garantizar los derechos de propiedad privada. demás.
B) Vigilar y promover la libre competencia.
80. Una de las siguientes referencias no corresponde al
C) Velar por el cumplimiento de los contratos sin in-
Psicoanálisis.
terferir en ellos.
A) Fue fundado por Sigrnund Freud. (1 R56-1939)
D) Actuar como árbitro privado.
B) En el comportamiento influye la libido.
E) Garantizar el suministro de servicios públicos.
C) Es una teoría y una terapia.
75. No es un tributo administrado por la SUNAT: D) Propugna que se estudie sólo la conducta observa-
A)IGV B) ~SC C) Impuesto a la Renta ble.
E) Descubrió que tenemos fobias inconscientes.
D) Aranceles E) Peajes
SOLUCIONARIO
11. Le da un nombre selección natural.
APTITUD ACADÉMICA
L Aclara la tesis.
COMPRENSiÓN DE LECTURA lIL Concluye complementando la aclaración.
1. En el texto, se señala que la vida urbana en toda civili- Clave: C
zación, por necesidad se desarrolla estrechamente con el 8. Los enunciados hablan del hígado.
comercio y la industria, independientemente del clima,
La secuencia correcta es la siguiente:
pueblos o religiones, como a la diversidad de épocas.
Il. Define al hígado.
Clave: E
l. Compara y señala su importancia.
2. Según el autor, el cine no ha superado al teatro en lo IV. Señala su función secundaria de éste órgano.
referente a transmitir al público esa sensación real, que 1Il. Señala como funciona.
comienza a crearse en el instante que se alza el telón.
V. Señala como funciona posteriormente.
Clave: E Clave: D

3. Del texto, se deduce que la condición sincrética en lo 9. Los enunciados hablan de los Saurios
cultural, se refiere a la asimilación de los modelos y su
La secuencia correcta es la siguiente:
transformación de acuerdo a su realidad, ello queda bien
evidenciado en la arquitectura de los templos. 1/. Concepto de vcrtcbrados y clasificando a los saurios
en la familia de los réptiles.
Clave: O
V. Característica física principal.
ORACIONES INCOMPLETAS 111.Otra característica física.
IV. Clasificación por costumbres alimenticias.
4. En la oración se plantea darles un plazo definitivo (pe-
rentorio) con una finalidad de acogerse al nuevo régi- l. Otra costumbre alimenticia.
men de tributación al nuevo régimen de tributación Clave: E
fraccionaria. 10. El Sisiema Parlamentario:
Clave: E
La secuencia correcta es la siguiente:
5. De la oración se deduce que la agresión es parte de la IV. Definición del sistema parlamentario.
naturaleza humana y la necesidad de un equilibrio en- 11. Donde surgió el sistema parlamentario.
tre nuestros instintos y la manera de expresarlo.
l. Señala los dos tipos de sistemas parlamentarios:
Clave: A dualista y monotcista.
III. Desarrolla el Sistema Dualista.
6. En la oración, las dos palabras que debemos comple-
tar tienen una relación de oposición, deducido del V. Desarrolla el Sistema Monoteísta.
conectar pero, en las altemativas la única que tiene ésta Clave: C
relación es fornido y raquítico. 11. Los Vientos:
Clave: E La secuencia correcta es la siguiente:
1/. Define los vientos.
COHERENCIA DE REDACCiÓN
IV. Señala los vientos donde se produce.
7. Los enunciados hablan de DARWIN. 1. Indica como se forman.
La secuencia correcta es la siguiente: V. Señala porque se modifica la dirección de los vientos.
V. Señala la tesis de Darwin en relación con las especies. 1lI. Clasifica a los vientos según su altura.
IV. Señala la característica de éste. Clave: D
~
0~.~~~§~~-1
2002-11 UNI APT. ACADÉMICA Y CULTURA GENERAL I ~I~
GomeZ

16. Analizando la secuencia:


SERIES NUMÉRICAS
44120 20]
12_ En el cuadro, si multiplicamos cada valor de la pri-
mera columna por cada valor de la segunda columna,
obtenemos la tercera columna, de igual forma si multi-
UUUUUU
O -3 +1 -2 +2 -1
plicamos cada valor de la cuarta columna por la quinta
columna, tambien obtenemos la tercera columna, aSÍ:
UUUUU -3 +4 -3 14 -3

Clave: B

1/4 1/2 118 1/24 3 17. Analizando cada cuadrante:

2/3 1/2 1/3 2/3 1/2


6 x 48 Y 6
De la fila 3:
6xx = 48 /\ yx 6 = 48
~ x =8 y=8

Clave: E Del gráfico se observa que:


(Cuadrante 1) 5 + I = 6 1+2+3 (,
13. Analizando la serie:
(Cuadrante Il) 7 + 5 = 12 3+4+512
~.--.,,/'-.---,'-..../,--
+2 +2
..
+2
... (Cuadran te 11I)7 + II = 18 5+ 6 + 7 18
3 . 2 . 9
, ','
'-......~
+6
-,. 4 . 15 . 6 . 21
•.. --""
+6
'/ ' '/ '-_/ +6
(Cuadrante IV) 7 + 17 = 24 7 + 8 + 9 = 24
Clave: E
Clave: B
18. Analizando el punto y las letras:
14. Dándole forma a cada término de la secuencia dada:
"." avanza así: "un avanza así:

4~1
Clave: C

15. Identificando

')
la relación en cada cuadro:
3~2

00
18 IIN" avanza así: HIt! avanza así:

22 2 + 22 + 7 = 18 + 13
13 7 3Pffi2

8
18
20 4bitjl
8 + 18 + 11 = 20 + 17 Finalmente el casillero UNI queda así:
17 11

~
4
B
12
A+B+9=12+8
00 Clave: B

~ 8 9 19. Vista desde a~iba se observa que se trata de 5 figu-


ras independientes.
A+B=II

Clave: C 11111
I UNI 2002-11 APT. ACADÉMICA Y CULTURA GENERAL I
Vista tridimensional según las claves:
RAZONAMIENTO MATEMÁTICO
Vista de arriba
t 24. Datos:

ateeJ Clave: A
Número
Promedio
total de alumnos:
de los 6 más aplicados:
12
IS
Promedio de los 6 restantes: 14
20. La representación de las siguientes figuras es una
También:
vista desde arriba.
Promedio del tercio superior (4 alumnos): I X.6
Promedio del tercio medio (4 alumnos): 15,5
Promedio del tercio inferior (4 alumnos): x
... El promedio del total de alumnos en ambos casos deben
Excepto el cubo que se observa de un ángulo distinto:
ser igual es por lo tanto:
Vista Tridimensional. 6xl8 + 6xl4 4xlR6 + 4x15.5 + 4x
12- 12
~ Efectuando: x = 14
VISTA
Clave: E
Clave: A
21. De las alternativas observamos que:
25. Datos:
- Por debajo de la línea quedara el número de bolitas de
un color que excede al del otro color, excepto Total de jugadores : 70 ... (1)

'o l' Io o que 00 cumple " relación

Clave: A
Jugadores
Jugadores
Jugadores
de fútbol
de Ping Pong : 32
de básquet
: 50

: 27
... (2)
... (3)
... (4)
Jugadores que practican los 3 deportes: R
Por conjuntos:
22. En las alternativas observa-
mos que todas las figuras están di-
ClD
vididas en triángulos, excepto en: \JD Ping pong

Clave: B

23. Del cuadro se observa:


Comienza en blanco Comienza en negro Del gráfico:
~-::::;,:::--~.
~"-.
'-
------- .
x +y + + + b + e + R = 70

~"f~)f-~I'~~~'~
-: Z ti

x +y + Z + ti + b + e = 62 ... (1)

x + II + b + 8 = 50
x + (/ + h=42 ... (2)

y + b + e + 8 = 32
C~na Y + b + e = 24 ... (3)
incógnita
x + a +;; + 8 = 27
que los cuadros en blanco consecutivos aumentan de uno x+a+z=19 ... (4)
en uno por cada columna.
Sumando (2) (3) Y (4):
Clave: E
x +y + Z + 2 (a + b + e) = 85 (a)
~~~~~»=~I. UNI 2002-11 APT. ACADÉMICA Y CULTURA GENERAL I '.&:

4»lliQ
GomeZ

Restando (1) de (a): Ec x X


=>1
UI_ W
x+ y+ z + 2(a+h +c)-(x+ y+ z +a +h+c)= 85 - 62
=> E; W
=> a+b+c =23 U¿ X
Reemplazando el valor de a+b+c en (1): De (*) : E_IV
U-X
x + y + z + (23) = 62
=> X = UW
=> x + y + Z = 39 E
Clave: C
Clave: D
26. Si: Número de niños: a 28. Analizando gráficamente.

Número de niñas: b 1) En un paradero que subía gente no bajaba ninguno.

Del enunciado: Baja No baja


De (1)
"Cada niño tiene tantos hermanos como hermanas" Sube , O x
a - I =b
=> a=b+1 ... (a)
"Cada niña tiene dos veces más hermanos que hermanas" y
No sube z
a = 3 (b - 1) ... ({3)
De (a)y (f3): 2) En 9 paraderos subió o bajó gente.
b + I = 3 (b - 1) x + z = 9

=> b =2 3) En 6 paraderos no bajó ninguno


x ~ y = (,
Reemplazando el valor de b en (a):
4) En 9 paraderos no subió ninguno
a = (2) + 1=3
z + y = 9
Finalmente: a +b =5
Sumando: (2), (3) Y (4):
• Clave: B
2 (x + y + z) = 9 +6 +9
27. Datos: Precio de 1 calculadora: E, x+y+z=12

Precio de 1 libro: UL Clave: A

Relación de Precios: 29. Del enunciado: Precio de compra: p,. =/11

Precio de venta inicial, "aumenta 11 %. del p,. "


~ E ... (*)
U¿ U
Pvi = (1 + I~O) P.:
Con Z soles se podran comprar W libros, es decir:
= (1 +_'100
_1 )/1/ ... (1)
... (a)
Precio de venta final, "disminuye n% del [>,., ".
Con Z soles se podran comprar X calculadoras, es decir:
p. =
'1
(I-~)P
100 ."
Z = EcX ... ((3)
De: (1) (1- I~O)(I + I~O)III
Dividiendo ({3) entre (a):
m(IOOOO _112)
Z E,X
10000
Z U¿W Clave: D
_~ ,:=,~""L, I UNI 2002-11 APT. ACADÉMICA Y CULTURA GENERAL l·
RAZONAMIENTO LÓGICO
De (11), velocidad del móvil B: I'H =10 mis
30. Del enunciado:
(1) La calidad de Q no es mayor que la de T
=> d= (IOm/s)/ ... (*)
QSJ
,
(2) Z es de calidad inferior a Q, pero mejor que P. De (1): velocidad del móvil A: ~, = 3"11
P<Z<Q 200 m +d = lA 1
De (1) Y (2): P < Z < Q S T De (*): 200," + fCI U m/s)1 l= [3(1 O mls )1/
Del cual se determina que el producto Q no es el de más => t = l Ov
baja calidad.
Por lo tanto es necesario utilizar ambas informaciones
Clave: E para hallar el tiempo.

31. Según el enunciado, para que el número de abuelos Clave: C


sea mínimo:
33. En el gráfico:

4 abuelos De(I):

'"5'~~

A r=5m B
Área de la región sornbrcada

4 padres s = Área del sector circular- Área del triángulo

_a ('i)2 (511/) (511/) SC!1IU.


S -2"".111 - 2

Necesitamos introducir (l =1f (11) para hallar "S".

S=¡(2511/
2
)-ei 1I/2)sc!III

4 hijos Por lo tanto es necesario utilizar ambas informaciones


para hallar el úrea sombreada.

Del gráfico: N° de abuelos :4 Clave: e


° abuelos/as fallecidos :2
OPERACIONES MATEMÁTICAS
N° de abuelos/as vivos :2
Clave: A h*a
34. Si: a'" h = . (1)
a+h
INSUFICIENCIA DE DATOS
.'.
=:> b » ({ = ---
Jh ':' a ... (2)
Punto de encuentro
a+h
32.
Reemplazando (2) en (1) Y suuplificundo:

,
I

AI. ~,,---_BI
200111 d
a '" " = -'----,--
0~1 UNI 2002-11 APT. ACADÉMICA Y CULTURA GENERAL 1:::~:::i=i1=~f~ z
<-=-==->
I cargue un hecho cualquiera, la palabra que no guarda rela-
=>a*b=-( \2 ción es implorar, qi.¡e significa suplicar, pedir mediante
a + b,
ruegos.
_ 1
Luego: Clave: B
I * 8 - (1 + 8)2

I 39. CONFABULACIÓN, significa ponerse de acuer-


TI do entre dos o más personas para hacer algo, general-
Clave: C mente con el fin de hacer daño, de ahí que son sinóni-
mos, conspiración, maquinación, intriga, conjura no así
la palabra contradicción que significa oponerse. .,
35.Si: ~ =2a+3
Clave: A
= 2a+3
40. INGRESO, significa admisión, entrada desde otra
Si hacemos: x a+ 1
concepción también significa renta, pensión. La palabra
=> 0=2x+1 que no guarda relación es asignación que significa dar,
entregar, asignar.
En la ecuación: 14WI =~+ 14 Clave: B·

2 (4W) + 1 = 2 (3W) + 1 + 14
=> W = 7 CONECTORES
Clave: C 41. Del texto, observamos que en el primer espacio en
blanco debe ir un conector comparativo (como), para en-
lazar el Mundial de Fútbol como un hecho comercial. En
el segundo espacio debe ir un coneetor adversativo (mien-
tras que), pues el texto señala como un éxito el Mundial
s = 2M¡ + 3 M2 + 4 3 M + + (/l+¡f1/l de Fútbol en lo comercial no así en lo deportivo.
+1 11 +1 +111 11+ 1 11
Clave: C
= 2 (2)(1)+ 2 (3)(2)+ 2 (4)(3)+ ... + 2 (11+1)01)
42. El texto, habla de la televisión por cable, el primer
+ 1 {I
n 1 1 1} conector debe ser aditivo (además de) para enlazar con
=-2- l2m + (3)(2)+ (4)(3)+ ... 01+1)01) sus características que se señala en el texto, luego el se-

=~{(I_i\(l_l)+(l_.l)+
2 ¡¡- '2) t 3 2 t:J 4 ...
+ (1__+1 )}
Il 1 1
gundo debe ser un conector consecutivo (por lo tanto),
pues como consecuencia de lo expresado en el texto es
menos alienante.
=~{I-_I } Clave: D
2 n+1
n
"2 43. Del texto, se atribuye a los EE.UU. ser el mayor pro-
Clave: E ductor de anhidrido carbónico, el primer conector debe ser
causal (ya que) para enlazar con la parte que señala que
produce la cuarta parte del total mundial. Luego el segundo
TÉRMINOS EXCLUIDOS conector debe ser explicativo (por lo que) para enlazar con
la parte del texto donde se señala que su emisión per cápita
37. DEGRADAR, significa bajar de grado, en cuanto a
es seis veces mayor que el promedio del resto del mundo,
la dignidad de una persona significa rebajar la honra, sig-
finalmente el tercer conector debe ser consecutivo (por lo
nificado similar tienen: envilecer, deshonrar, humillar y
tanto) pues el texto señala que como consecuencia de lo
deteriorar no así enaltecer que tiene un significado con-
anterior es principal responsable del ataque al ecosisterna.
trario.
Clave: C Clave: E
44. El texto describe un individuo que posee una sensi-
38. INTERPELAR, significa cuestionar, preguntar, in- bilidad fina, pero sin llegar al extremismo, debido a que
quirir, requerir a alguien para que de explicaciones o des- su imaginación estaba siempre dentro de la realidad y
W2~.,:óI\:~._gffl7;W~mt0WJi, . Il©tttmmllAAtWlíEWW10"'" 69"1
.....
1 UJI~'::~~H-:;;;_"'=" ..";;,~;;;;UN12002-11
f,po,_'$:.:<:;,;%;x<:**;.::i:::::i::::r.«:>:>;;;::i::;:~,;>w;
APT. ACADEMICA y CULTURA GENERAL."0WJi_
.;.
WiWNWlill
{-.XM:&.;~.= m., , ..»:.
GomeZ

que le hacía ver las cosas sin deformaciones. Luego el


primer conectar debe ser adversativo (pero), el segundo
causal (por que) y el tercero copulativa (y).
CULTURA GENERAL
Clave: C 51. Islam, es la religión mahometana, formulado en el
siglo VII después de Cristo por su principal poeta
45. El texto trata del carácter de Carmen, primero com- Mahorna, también se considera como sus patriarcas a
parándole con el de su padre, luego con la de su madre y Moisés y Abraharn y a Jesucristo como uno de sus pro-
concluyendo que no había heredado ninguno de ellos, fetas; tienen como único Dios a Ala y su templo princi-
sino que tenía un caracter singular. El primer conectar pal el Kaaba. Mahorna no fué contemporánea" de
es una conjunción copulativa (ni), el segundo adversati- Jesús.
vo (pero) y el tercero copulativo (y). Clave: E
Clave: C
52. La Iglesia Católica tuvo un papel preponderante en
la Edad Media, que se inicia con la caída del Imperio
Romano en el año 467 d.n.e. hasta el año 1453 d.n.e.
ORACIONES ELIMINADAS
debido a la idea imperante del predominio del poder es-
46. El tema del texto es la Hagiografía, señalando su piritual sobre el poder temporal, su disciplina y organi-
concepto, sus objetivos y su relación con la Iglesia Cató- zación pero sobretodo por ser el clero la única clase ilus-
lica, la oración que no es pertinente es la que señala que trada que dirigía las escuelas, colegios y universidades.
la Iglesia Católica siempre ha combatido el culto a los El surgimiento de las ciudades y la formación de la bur-
santos pues no tiene relación con el tema que es la Ha- guesía estuvo contra el poder de la Iglesia.
giografía, además esta afirmación es contradictorio con
Clave: C
la postura de la Iglesia Catól ica.
Clave: C 53. La Cultura Wari se desarrolló en los Andes entre
los años 600 d..n.e. y 1200 d.n.e. En Europa se encontra-
47. El texto trata de los mensajes publicitarios dentro ba en la Edad Media que comienza en 476 d.n.e. con la
del contexto de su evolución y desarrollo. La oración que caída del Irnperior Romano de Occidente y termina en
señala que los mensajes publicitarios pasan por varios 1453 d.n.e.
procesos, antes de su emisión pues no está dentro del Clave: 8
contexto de evolución y desarollo.
Clave: A 54. La "Campaña de la Breña" fue dirigida por Andrés
Avelino Cáceres contra la invasión chilena, su ejército
48. El tema principal de las oraciones es la meteorolo- estaba compuesto por campesinos de la sierra central y
gía, la oración; los instrumentos que utiliza un científi- obtuvieron victorias en Pucará, Concepción y Marcavalle,
co van cambiando con el tiempo, no tiene relación ex- entre los años 1881 y 1883.
clusiva con el estudio de la ciencia meteorológica. Clave: E

Clave: E 55. En el siglo XIX muchos inmigrantes ingresaron al


Perú quienes emprendieron diversos negocios, como
49. Las oraciones tienen como tema principal el examen Ferrnín Fitzcarrald quien se dedicó a la explotación del
de admisión, los tipos de pruebas en que se divide y la pu-
caucho en la región de Ucayali, al igual que Julio César
blicación de los resultados de las mismas. La oración donde
Arana en el río-Amazonas, ésta actividad produjo un gran
se señala que las pruebas son difíciles, no es pertinente pues
genocidio en la población nativa, desapareciendo mu-
menciona una característica de una prueba cualquiera.
chos grupos amazónicos como los Piros,
Clave: O
Clave: C
50. El tema central de las oraciones son las grarnineas
56. Enla oración: los de esa tribu no pierden el tiempo
a través de la historia humana, la oración que señala; el
en asuntos tribiales. No se usa correctamente la palabra
hombre más tarde extendió su dominio hacia la gana-
"tribiales" plural de trivial la forma correcta es trivialcs,
dería no es pertinente pues escapa del tema central.
según la Academia de la Lengua Española.
Clave: D
Clave: 8
:r..
~I~
GomeZ

57. Los enunciados se escriben correctamente así:


65. Los ríos de la Región Hidrogrática del Pacifico tie-
- Pedro salió a despedirse de Ana. ne las siguientes características:
- Tú madre está harta de tus quejas.
• Son de corta longitud.
- Aníbal se mudó y ahora vive por la zona norte.
• Son ríos de caudal irregular.
- Me caí y para colmo me rodé las escaleras.
• Tienen su origen en la cadena occidental de los Andes.
- El Fiscal examinó una por una las momias.
• No son caudalosos.
Clave: C
• No son navegables. ~
58. El prefijo sin significa unión en la alternativa C Clave: C
está definida como desunión que es su antónimo.
Clave: C 66. La Cadena Occidental.es una de las tres cadenas
que posee el Perú, en esta podemos encontrar los princi-
59. El sufijo gamo significa unión de Sexos, el sufijo pales picos:
voro significa que come.
• Huascarán • Amparo
Clave: A
• Yerupaja • Monte Meiggs
60. En la frase: El ruído era infernal y para ser escu- • Coropuna • La Viuda.
chado, recurrí a un eufemismo. Es incorrecta la utiliza-
ción de la palabra eufemismo, pues la frase requiere de El pico Salkantay, no pertenece a ésta cadena, está loca-
un término que indique elevación del volumen de la voz. lizada en el Cusco y pertenece a la Cadena Central.

Clave: C Clave: B

61. El autor de "La Divina Comedia" es Dante Alighieri. 67. De las ciudades mencionadas, Cerro de Paseo es la
más alta con 4338 msnm.
Clave: B
CIUDAD PAIS rn.s.n.m.
62. En la novela "Conversación en la Catedral", de
Cerro de Paseo Perú 4338
Mario Vargas Llosa trata sobre la situación social y polí-
tica del país durante el gobierno de Odría. Potosí Bolivia 4000

Clave: C Tulcan Ecuador 3001


Toluca México 2800
63. El Estado, ha creado varias unidades para la pro- Tunja 2070
Colombia
tección y conservación de los ecosistemas, flora, fauna,
belleza paisajistica o lugares históricos tales como: Clave: A
- Parques Nacionales
68. Las ciudades palestinas de Ycnin, Romala y
- Reservas Nacionales Naplusa, fueron objeto de intensos ataques militares is-
- Santuarios Nacionales raelitas con el pretexto de ser campos de concentración
- Santuarios Históricos del Hamas y Yihad islámico. Este hecho motivó la críti- .
ea de la comunidad internacional al gobierno de Aricl
Por lo cual, reservas ecológicas no corresponde a las uni-
Sharon Primer Ministro de Israel.
dades clasificadas por el estado.
Clave: O Clave: C

69. En la base militar norteamericana de Guantúnarno,


64. Las Coordenadas Geográficas ubica cualquier punto
territorio cubano ocupado militarmente, están encarce-
de la Tierra y son:
lados los miembros de Al Qaeda captur.ados en
• Latitud.- es la distancia hacia la Línea Ecuatorial Afganistán en la operación militar denominada "liber-
• Longitud.- es la distancia hacia el Meridiano de tad duradera" llevado a cabo por el gobierno de los
Greenwich EE.UU.

Clave: A Clave: O

"
~LG~~==:m:,,:,,:¡=!!~;1
leZ
UNI 2002-11 APT. ACADÉMICA Y CULTURA GENERAL I~=~::e:!:==;::~.~ ~

70. La Ley dc Preferencias Arancelarias Andinas 76. La Población Económicamente Activa (PEA) se
(APTA), fue promulgada por los EE,UU, en 1991, meca- divicle en tres categorías:
nismo por el cual retira los aranceles a los productos
1) Empleada
andinos de países como el Perú, Bolivia, Ecuador y Co-
lombia. El APTA tiene como fin, enfrentar la producción 2) Sub-Empleada
y tráfico de drogas, 3) Desempleada

Clave: B Según datos estadísticos en la actualidad la mayor parte


de la PEA se encuentra en la categoría de subcmpleada.
71 . Las vías transoceánicas son ejes estratégicos de in- Clave: E
fraestructura de transporte internacional, principalmente
77. El Relativismo, plantea que la verdad está en fun-
en el transporte de carga, siendo las vías más utilizadas
ción de cada sujeto es decir "nada es verdad o mentira,
el Oceáno Pacífico y el Oceáno Atlántico,
todo es según el cristal con que se mira", un representante
Clave: B de esta corriente fue el sofista Protágoras de Abdera cuya
sentencia es: "El hombre es la medida de todas las cosas
72. Actualmente, los estudios arqueológicos han permi- de las que son en cuanto son y de los que no son en tanto
tido demostrar que Caral es la ciudad más antigua de que no son",
América ubicada en el valle de Supe 200 km al norte de Clave: D
Lima cuyas excavaciones están dirigidas por la arqueóloga
78. Las proposiciones pueden ser:
peruana Ruth Shady Solís, que ha encontrado pirámides,
viviendas, anfiteatros, el templo mayor, cte. • Atómicas.- cuando no llevan enlaces lógicos ni ad-
verbio de negación expresando un sólo mensaje,
Clave: D
• Moleculares.- son proposiciones atómicas unidas me-
73. El tenor peruano Juan Diego Flores quién estudió diante enlaces lógicos y adverbios de negación,
en el Conservatorio de Lima y luego en el Coro acio- 1. Atómica
nal, triunfador en diversos escenarios de EE,UU, y Eu- 11,Molecular
ropa como en la Scala de Milán.
111.Molecualar
Clave: D IV, Atómica
V, Molecular
74. Según la Constitución Política vigente, el Estado:
Clave: A
A) Garantiza y protege la propiedad privada (Art. 70),
B) Facilita y vigila la libre competencia (Art. 61), 79. Personalidad.- es una organización dinámica cons-
C) Vigilar y garantizar la libre contratación (Art. 62), tituido por temperamento, ambiente y carácter.
D) Del Art. 61 se concluye que el Estado no puede ac- Tempcramcnto.- es la base orgánica de la personalidad,
tuar como árbitro privado, esto sería atentar contra está vinculado a los espectos afectivo-emotivo como la
la libre competencia, impulsividad.
E) Promueve el empleo, salud, educación, seguridad, Carácter.- es el conjunto de rasgos singulares social-
servicios públicos e infraestructura (Art. 58), mente condicionado que expresa la personalidad ante sí
Clave: D y los demás,
De las definiciones la clave B define el temperamento
75. La Empresa Municipal Administradora de Peajes
no la personalidad,
(EMAPE), es la encargada de administrar los recursos pro-
Clave: B
venientes del peaje en la provincia de Lima y en el resto
del país es el Ministerio de Transportes y Comunicacio-
80. El Psicoanálisis es una corriente psicológica fun-
nes, La SUNA T es la encargada de administrar los tribu-
dado por Sigrnud Freud que propone el estudio del
tos provenientes dellGV, ISC, impuesto a la renta y aran-
inconciente. Quien propugna el estudio de las acciones
celes,
observables de la conducta es el conductismo.
Clave: D
Clave: D.
~ 72~¡:===:%:· :l&~1i ~UEN~12~O~O~3-iI2A~P~T:.
A~C~A~D~ÉM~IC~A~Y~C~U~LT~U~RA~~G~E!N~E~R~A~LJ; @=:=~:*: meZ
~=""-"

~L3~~~~
APTITUD ACADÉMICA Y CULTURA GENERAL

C) o - sin embargo - debido a


APTITUD ACADÉMICA
D) y - antes bien - y
E) no obstante - puesto que - pero
TÉRMINOS EXCLUIDOS
6. Elija la alternativaque, al insertarse en los espacios
1. Elegir la palabra que debe excluirse por no guardar
en blanco, dé sentido lógico al texto.
relación de significado con los demás.
numerosos antropólogos han rea-
A) Vejar B) Esquivar C) Satirizar
lizado estudios en variadas comunidades campesinas del
D) Mortificar E) Censurar Perú en un esfuerzo por investigar el funcionamiento
actual de la reciprocidad, otros han comparado culturas
2. Elegir la palabra que debe excluirse por no guardar
antiguas cuya característica común fue el desconoci-
relación de signi ficado con las demás.
miento del dinero".
A) Trigo B) Arroz C) Café
A) Porque B) Mientras C) Asimismo
D) Perejil E) Cebada
D) Sin embargo E) Además
3. Elegir la palabra que debe excluirse por no guardar
7. Elija la alternativa que, al insertarse en los espacios
relación de significado con las demás.
en blanco, dé sentido lógico al texto.
A) Delicado B) Nimio C) Pequeño
"Esta estrategia es, , contundente,
D) Exiguo E) Minúsculo _____ , muy eficaz"

4. Elegir la palabra que debe excluirse por no guardar A) por supuesto - no obstante
relación de significado con las demás. B) sin duda - a pesar de ello
A) Sanear B) Recuperar C) Recomponer C) también - además

D) Reconstruir E) Erigir D) indudablemente - y por lo tanto


E) realmente - pero

CONECTORES 8. Elija la alternativa que, al insertarse en los espacios


en blanco, de sentido lógico al texto.
5. Elija la alternativa que, al insertarse en los espacios
en blanco, dé sentido lógico al texto. "Era un tipo elocuente, expresaba
claramente sus ideas. , su elocuencia
"El incremento de informaciones la
no correspondía a sus actos. , fre-
proliferación de emociones vicarias a través de la tele-
cuentemente sus palabras y sus hechos entraban en fla-
visión no conducen al enriquecimiento de la persona;
grante contradicción"
, la potencia de este medio
audiovisual su utilización al servi- A) esto es - Por eso - No obstante
cio del libre mercado con lleva a perfilar seres B) en otras palabras - Además - Por ejemplo
alienados". C) vale decir - Sin embargo - Por el contrario
A) ni - pues - o D) en realidad - No obstante - Porque
B) mas - porque - es decir E) además - También - Realmente

I
___ .i'tIillmt_%:'1
••••••• UN12003-1 APT. ACADÉMICA Y CULTURA GENERAL 1.;;;;:;;~;t50
9. Elija la alternativa que, al insertarse en los espacios su volumen se determina fácilmente mediante fór-
en blanco, dé sentido lógico al texto. mulas.
,_-:- , la realidad y los deseos se con- V) No ocurre lo mismo con los cuerpos irregulares,
funden. , es necesario distinguir entre lo como los objetos de la vida real, para cuya medi-
real y lo deseable, no frustramos en el ción se requieren métodos más complejos.
propósito de alcanzar el objetivo" A) I B) II C) 11I D) IV E) V
A) A menudo - Porque - además de
12. Elija la oración no pertinente respecto a la cohe-
B) Aunque - Además - para
rencia del texto.
C) Por tanto - También - con la finalidad de
1) En la superficie terrestre, existen dos áreas cuyas
D) Con frecuencia - Asimismo - a causa de temperaturas, en muchos casos, no superan los
E) Frecuentemente - por ello - a fin de
o"e y donde los mares contienen grandes masas
de hielo flotante.
Il) Los mares polares contienen diversas especies de
ORACIONES ELIMINADAS
animales y plantas que sirven de alimento a las
10. Elija la oración no pertinente respecto a la cohe- ballenas.
rencia del texto. In) Estas áreas corresponden a los círculos polares
1) Las primeras manifestaciones teatrales aparecie- ártico y antártico.
ron en Grecia como rituales religiosos; luego vie- IV) El ártico está situado a 66"32 al norte del Ecua-
nen la tragedia y la comedia. dor.
Il) El teatro continúa en la Edad Media como una V) El Circulo Polar Antártico que está situado a
manifestación litúrgica o religiosa, pero, poco a
66") 2 al sur del Ecuador, rodea al continente casi
poco, se va pagan izando, saliendo a calles y pla-
en su totalidad.
zas y a las ferias.
A) I B) II C) [11 D) IV E) V
111) Las plazas, pequeñas o grandes, devinieron en el
espacio ideal para las manifestaciones teatrales.
13. Elija la oración no pertinente respecto a la cohc-
IV) Durante el romanticismo, el drama romántico y la rencia del texto.
comedia son retlejo de la vida cotidiana para lue-
1) Los cetáceo s usan el sistema de ccolocación si-
go transformarse en un teatro con ideas sociales,
milar al de los murciélagos.
morales y políticas.
Il) La palabra cetáceos viene del latín "cetus" que
V) El teatro contemporáneo toma diferentes formas,
significa animal grande y del griego "ketos", o
pero su mensaje sigue siendo el mismo; cada au-
monstruo marino.
tor y cada escuela le da un sello particular
intluenciado por el movimiento social y político 111) Emiten un sonido en su garganta que se propaga
que vive. en el mar y rebota a ellos.

A) I B) II C) 111 D) IV E)V IV) Según como perciben este eco, reconocen las ca-
racterísticas de la superficie, la forma de los ob-
11. Elija la oración no pertinente respecto a la cohe- jetos y la distancia a que se encuentran.
rencia del texto. V) Esto les permite cazar en la oscuridad como tam-
1) El cálculo del volumen de los cuerpos general- bién reconocer su territorio.
mente se realiza a través de fórmulas y de manera A) [ B) I[ C) II[ D)[V E) V
indirecta.
14. Elija la oración no pertinente respecto a la cohe-
11) Por ejemplo, el volumen de un cono se determina
rencia del texto.
multiplicando la tercera parte del área de su base
por su altura. 1) Muchos trabajadores están expuestos a determi-
nados agentes nocivos en su ambiente laboral.
111) La palabra calcular como sinónimo de considerar
es reflexionar sobre algo con atención y cuidado. ll) La exposición a dichos agentes puede originar en-
fermedades, muchas veces, crónicas y terminales.
IV) En el caso de los cuerpos regulares, el cálculo de
0~~~~~3 UN12003-1 APT. ACADÉMICA Y CULTURA GENERAL I,;:;;~~;:
Ill) Las causas de este tipo de patologías pueden ser C) El éxito de las almas mediocres es sustancial.
físicas, químicas o por radiaciones. D) Solo los mediocres ascienden a la cclebridad.
IV) La protección de la salud mediante el uso de equi- E) El hombre mediocre ambiciona el triunfo efímero.
pos protectores es importante.
V) Así, la exposición a los Rayos X pueden generar
17. Albert Einstein comentaba: "La formulación de
diversas afecciones, cuyos riesgos dependen del
un problema es, a veces, más importante que su solu-
tiempo de exposición a los rayos.
ción, que puede convertirse en algo rutinario". En efec-
A) 1 B) 11 C)III D) IV E) V to, un problema bien definido es un problema medio
resuelto. Definir un problema es establecer claramente
qué se desea resolver, los alcances de la solución y las
COMPRENSiÓN DE LECTURA condiciones, limites y restricciones que deben tenerse
en cuenta. No es una tarea fácil; una mala definición
15. Antes de Gutenberg, había sólo 30,000 libros en puede llevar a resolver el problema equivocado y a mal-
toda Europa. La mayoría estaba constituida por biblias gastar en ello esfuerzos, tiempo y dinero.
o comentarios bíbl icos. Hacia el año 1500 gracias a
El propósito predominante del autor del texto anterior es:
Gutenberg, había nueve millones de libros de diferentes
temas. Los folletos y otros materiales impresos afec- A) Demostrar que, para resolver un problema, basta
taron la política, la religión, la ciencia y la literatura. con formulario adecuadamente,
Por primera vez, las personas que no pertenecían a la B) Convencer acerca de que la solución de un pro-
élite canónica o religiosa tenían acceso a la informa- blema es algo rutinario.
ción escrita. Internet, la red interactiva global, transfor- C) Definir, exactamente, qué es un problema para
mará nuestra cultura en forma tan radical como la impren- Albert Einstein.
ta dc Gutenberg tranSf0J111Ólos tiempos medievales.
D) Resaltar la importancia que tiene una adecuada
¿Cuál de las siguientes oraciones sintetiza mejor la idea formulación del problema.
principal del texto anterior? E) Destacar la figura de Albert Einstein como un gran
A) La imprenta de Gutenberg produjo la mayor trans- solucionador de problemas.
formación cultural de la humanidad.
B) Internet transformará nuestra cultura en forma ORACIONES INCOMPLETAS
radical y negativa.
C) Los grandes afectados por la imprenta de 18. Elija la alternativa que dé sentido a la oración.
Gutenberg fueron la política, la religión, la cien- Cuando uno conoce sus y tiene una
cia y la literatura. buena estrategia se siente de lo que es.

D) Al igual que la imprenta de Gutcnberg, internet A) fortalezas y debilidades - seguro


transformará radicalmente la cultura en el mundo. B) oportunidad y ventajas - entusiasta
E) Gutenberg e internet han transformado radical- C) limitaciones y opciones - triste
mente la cultura de la humanidad. D) cualidades y deficiencias - preocupado
E) amigos y adversarios - inseguro.
16. "EI hombre mediocre que se aventura en la losa
social tiene apetitos apremiantes e inmediatos; el éxito.
19. Elija la alternativa que dé sentido a la oración.
No sospecha que existe otra cosa; la gloria que es ambi-
cionada solamente por los caracteres superiores. Aquél "El fortalecimiento de las instituciones per-
es un triunfo efímero, al contado; ésta es definitiva, in- mitió al Estado imponer una indiscutible".
marcesible en los siglos. El úno se mendiga; la otra se A) tutelares - anarquía
conquista" . B) privadas - legislación
De acuerdo con el texto. C) militares - autoridad
A) La vanidad es propia de los hombres probos. D) financieras - estatización
B) Los genios sólo persiguen el éxito. E) civiles - censura

I
kI>:

~,I]!~
GomeZ
---.....
;1 UN12003-1 t\PT.ACADÉMICAYCULTURAGENERAL 1::=:;:0
20. Elija la alternativas que dé sentido a la oración. V. Por ejemplo, para los extranjeros, el estereotipo del
"Los no suelen inspirar epitafios francés se compone de un conjunto de atributos.

A) II - 1 -V -IV - 111
A) malvados - confusos
B) 11 - I - 111- V - IV
B) ladrones - amigables
C) 11 - 1II - I - V - IV
C) traidores - piadosos
D) I - 11- rrr - V - IV
D) traidores - ofensivos
E) 1 - 11 - IV - V - 111
E) maestros - burlones
23. A partir del título, determine la secuencia lógica
correspondiente.
COHERENCIA DE REDACCiÓN
Origen de la Matemática
21 . A partir del título, determine la secuencia lógica 1. Dicha ciencia se basó en relaciones numéricas ra-
correspondiente. zonadas y exactas.
Albert Einstein 11. Estos problemas tuvieron que resolverse, en un
1. En 1880, la familia Einstein se desplazó a Múnich, comienzo, de manera empírica.
donde logró establecer un pequeño y no demasia- !ll. Posteriormente, los egipcios también dieron so-
do próspero negocio de electricidad. lución a los cálculos de volumen y superficies.
Il. Posteriormente, Albert completó su educación IV. Con el tiempo, en Egipto y en ciudades de
en Zúrich, donde se graduó en la Escuela Mesopotarnia, este tipo de operaciones tomó el
Politécnica Federal, en 1900. carácter de ciencia.
[11. A1bert Einstein, el creador de la Teoría de la
V. Las primeras civilizaciones se enfrentaron con
Relatividad, nació en Ulm, Alemania en 1879. problemas de cantidades, números y relaciones.
IV. En 1894, el negocio de electricidad quebró y la
A) 1 - V - 11 - II! - IV
familia Einstein se traslado a Milán.
B) I - rrr - V - IV - 11
V. En Múnich, Albert no fue un niño prodigio, pero
C) V - 11 - I - IV - 1I1
las afirmaciones de que sacaba muy malas notas
escolares parecen ser una exageración. D) V - Il - IV - I - rrr
A) 111- I - 11 - V - IV E) [- V - 111- 11 - IV

B) II! - I - V - IV - 11
24. A partir del título, determine la secuencia lógica
C) III - I - IV - V - 11 correspondiente.
D) 111- V - IV - 11 - 1 Ingeniería Civil
E) 111- IV - 11- V - I
!. Para dedicarse a esta disciplina se requiere gusto
22. A partir del título, determine la secuencia lógica y disposición por el dibujo.
correspondiente. I!. Sus profesionales pueden dedicarse a varias fun-
ciones técnicas.
Los estereotipos
l. Como representaciones imaginarias, exigen de no- 1lI. También se requiere interés por la mecánica estática.
sotros reacciones y modos de conducta, y se for- IV. Por ejemplo, pueden ser calculistas, interventores
man a partir de nuestras experiencias. y constructores.
11. Los estereotipos son representaciones imaginarias V. La ingeniería civil es las más antigua de todas las
esquemáticas de un grupo con respecto a los miem- ramas de la ingeniería.
bros de otros grupos,
A) V - II - I 1I - IV - I
111. Por otro lado, pueden transmitirse de generación
en generación. B) V - 111- I - 11- IV

IV. Se suelen asegurar que el francés es "comedor de C) V - I - 111- 11 - IV


pan y filetes con patatas, bebedor de vinos, seduc- D) V - IV - I - 11 - 111
tor de mujeres". E) V - I - Il- 111- IV
:;~~~3 UN12003-1 APT. ACADÉMICA Y CULTURA GENERAL ~

25. A partir del título, determine la secuencia lógica A) 2880 B) 1920 el 1280
correspondiente. D) 2560 E) 5120
Armas in teligen tes
30. Señale la alternativa que continúe a la sucesión
1. En su lugar, ahora intervienen, más bien, los saté- siguiente:
lites, rayos láser, robots y computadoras.
3; 16;45;96; 175; ...
Il. Dicho sistema armamentistico depende cada vez
menos de controles humanos. A) 244 B) 288 el296 D) 350 El 356
[I!. El desarrollo de este sistema armamentístico es
31 . Indique la alternativa que continúa la siguiente serie:
consecuencia directa de los avances tecnológicos.
6 ; 20 ; 42 ; 72 ; 110 ; ...
IV. Las armas inteligentes son un conjunto de siste-
mas armamentísticos de última generación. A) 156 Bll65 el 256 Dl281 El 324
V. La utilización de estos dispositivos seguirá modifican-
do, aun más, la planificación de la producción bélica. ANÁLISIS DE FIGURAS
A) III - IV - Il - V - I
32. La figura adjunta se encuentra ubicada frente a un
B) 11- II! - IV - V - I
espejo. Indique Ud. la alternativa que representa la
C) IV - III - II - I - V imagen en el espejo.
D) 111- 1I - IV - V - I
E) 11- I - Il l- IV -V

SERIES NUMÉRICAS
26. Indique la alternativa que continúa correctamente
en la siguiente sucesión:
I ; 19 ; 55 ; 109 ; 181; 271; ...
A) 424 B) 280 e) 325 D) 379 E) 282

27. Los números consignados en los tres cuadros cum- A) B) C) D) E)


plen una misma relación. Determine el valor de: X + Y + Z
33. La figuras de los cuadros I y 11cumplen una mis-
ma relación; identifique dicha relación e indique la al-
ternativa que debe ocupar el casillero UN!.

A) 16 B) 18 C) 20 D) 24 E) 25

28. Indicar la alternativa que continúa adecuadamen- 11


te en la siguiente sucesión:

1~1~ltltl~1
4; 4 ; 12 ; 8, 20 ; 12 ; 28 , ...
A) 30 B) 36 C) 12 D) 24 E) 16

29. Indicar la alternativa que continúa adecuadamente


la siguiente sucesión: A) B) C) D) E)
2;6; 16; 40; 96;224;512; 1152, ...
;:~!~;:====:l UN12003-1 APT.ACADÉMICA Y CULTURA GENERAL 1=~~=0
34. Observe los siguiente sólidos e identifique la alter- 39. De dos ciudades A y B parten todos los días y a la
nativa que no tiene la misma relación. misma hora dos vehículos que van al encuentro, con
velocidades de 50 km/li y 75 km/li respectivamente.
Siempre se encuentran en un determinado punto a las
12 m. Un día, el que sale de A encontró a las 3 p.ni. al
vehículo que sale de B, que se había malogrado de repen-
A) B) C) te. ¿A qué hora se malogró el vehículo que partió de B?
A) lO a.m. B) 9:30 a.m. C) 10:30 a.m.
D) II a.m. E) 9 a.m.

D) E) 40. Cuatro número son tales que los tres primeros for-
man una progresión aritmética de razón 6, los tres últimos
35. Indique la alternativa que debe ocupar el casillero UN!. una progresión geométrica y el primer número es igual al
cuarto. La suma algebraica de los cuatro números es:

m~I~IB~ A)-18 B)-14 C)-IO D)-6

41. Si el trabajo realizado por x - I hombres en x + I


E)-2

mmm~m
36. Seleccione
B)

la figura
C) D)

que no tiene la misma


E)

ley
días es al trabajo hecho por x + 2 hombres en x - I días,
como 9 es a 10; entonces el valor de x es:
A)7 B) 8 C)9 D) 10

42. En una progresión geométrica, el quinto término


E) 11

vale 5 y el octavo término vale 135. El valor del sépti-


mo término de la progresión es:
fu~~w. .
A) 40 B) 45 C) 50 D) 55 E) 60
•• • • • • • • •
• •
• • • 43. Al inicio de una clase hay 64 alumnos presentes;
• • • posteriormente ingresaron 16 que llegaron tarde. Si an-
•• • • • • • tes del término de la clase se retiraron el 30% de los
A) B) S;) D) E) presentes. ¿Cuántos alumnos quedaron en el aula'?
A)40 B) 41 C) 44 D)56 E) 63
37. Indique la alternativa que no guarda relación con
los demás.
RAZONAMIENTO LÓGICO
44. Si se asumen las siguientes prernisas:
1) Si me pagan, trabajo.
B) C) D) E) 2) Si no me pagan, renuncio.
3) Si me dan un incentivo, no renuncio.
4) Me dan un incentivo o denuncio a la empresa.
RAZONAMIENTO MATEMÁTICO
5) No trabajo.
38. Una digitadora se comprometió a tipear un infor-
¿Cuáles de las siguientes proposiciones son conclusio-
me en cinco días. El primer día tipeó 80 páginas; el nes lógicas de estas premisas?
segundo día los 4/7 de lo que faltaba; el tercer día los 6/
1) No renuncio.
II de lo que quedaba por tipear; el cuarto día los 3/5 del
resto; y el último día 24 páginas. ¿Cuántas páginas tie- 11) No me dan un incentivo
ne el informe? Ill) Denuncio a la empresa.

A) 256 B) 278 C) 307 D) 354 E) 388 A)lyll. B)lyllL C)llylll.


D) 1, [[ Y 111. E) Solo 11.

I
45. En los locales comerciales L, My N, las compras D) Son necesarias ambas informaciones.
se pagan en 12 meses, con interés simple. Si en L los E) Las dos informaciones son insuficientes.
productos cuestan 10% más que en M, y en N cuestan
5% menos que en M, y los intereses en L, M Y N son 15,
10 Y 20%, respectivamente. OPERADORES MATEMÁTICOS
¿En qué orden de preferencia usted recomendaría comprar?
A) L, M, N. B) N, L, M. ciu. N, L. 48. Si 1'(•• 1)=,,)-1 , G(•. 1)=,,2+ 1 ,entonces F(.1)· (i(.l)

D)N,M,L. E) M, L, N. es igual a:
A) 35 B) 119 q 130 D) 260 E) 390

si 11es impar
INSUFICIENCIA DE DATOS 49. Si a(II)={ ~
si 11 es par
46. La figura ABCD es un cuadrado, ¿qué datos son
necesarios para determinar el área de la región si n es par
y ~(II)={O
sombreada. -1 , si n es impar
Datos alternativos:
entonces el valor del cociente:
1. Al área del triángulo EBC es de 24 cm2
a(u +1)+ 1 donde 1I E z: +
11. DE = EC = 8 cm. ~(u-I)+2

1) Siempre es menor que l.


11) Puede ser un número negativo.

:CSJ:
111) El mayor valor siempre es el cuádruple del menor valor,
IV) El menor valor es 112.

A) I Y 11 son verdaderas.
B) 11I Y IV son verdaderas.
C) Sólo I es verdadera.
A) Alternativa 1.
D) Sólo IV es verdadera.
B) Alternativa 11.
E) Sólo 11\ es verdadera.
C) Ambas alternativas simultáneamente.
D) Cualquiera de las alternativas. 50. Si el operador' a(lI) = 2/1 + I Y ~(II) = 112entonces
E) Falta información.
el valor de cociente (a(~(3»)+ 1)+a(2) es igual a:

47. Una orquesta está formada por 20 músicos que A) 2,25 B) 4,00 C) 4,25
ejecutan instrumentos de cuerda, de viento y de percu-
D) 5,00 E) 5,25
sión. Hay algunos que ejecutan de cuerda y viento a la
vez, pero lo que ejecutan de percusión no ejecutan otro
instrumento. Sabiendo que 15 no ejecutan de percusión
hallar el número de los que ejecutan de cuerda y viento CULTURA GENERAL
a la vez si se conocen las siguientes informaciones:
51. ¿Cuál de las siguientes oraciones están acentua-
1) 10 ejecutan de cuerda. das correctamente?
11) 8 ejecutan de viento. 1) Pero, ¿qué haré, Ña Catita?
Para resolver el problema: 11) Ayer condecoraron al vicerrector.
A) La información I es suficiente. 11I) El publico vibró de entusiasmo.
B) La información" es suficiente. A) 1, 11I B) 1, 11, 111 q 1, 11
C) Cada información por separada, es suficiente. D) 11, 11\ E) Sólo 11

J
UN12003-1 APT. ACADÉMICA Y CULTURA GENERAL 1.~~=~~·0
52. ¿Cuál de las siguientes enunciados prcxcnta buen 57. "Yo no me río de la muerte
uso de la ortografla? sucede, simplemente,
A) Sus propiedades incluían una basta, inmensa ha- que no tengo miedo de morir
cienda. entre pájaros y árboles"
Es uno de los poemas más conocidos de:
B) Al sumo sacerdote le encanta el sumo de limón.
C) Ese conjunto de medidas es suscinto y atinado. A) Emilio Adolfo Westphalen
O) Las ollas hidrográficas del país son materia de ese B) César Moro
estudio. C) César Vallejo
E) Era importante reivindicar los derechos humanos O) Javier Heraud
en el mundo. E) Juan Gonzalo Rose

53. Señale la oración que no tiene un verbo copulativo. 58. Identifique la proposición errónea.
A) Carmen canta y baila. A) El efecto de invernadero, se debe a la acumulación
B) Juan está sentado. de gases en la atmósfera.
C) Manuel permanece quieto. B) El efecto de invernadero produce excesivo calen-
O) Pedro parece contento. tamiento de la tierra.
E) Teresa fue inteligente. C) La capa de ozono protege a los seres vivos, de la
acción de los rayos ultravioletas.
54. Señale la oración que tiene el recurso gramatical
O) Los compuestos químicos que contienen cloro y
conocido como epíteto.
emanan de la Tierra, destruyen la capa de ozono.
A) El primer examen siempre es fácil.
E) Los fuertes vientos destruyen la capa de ozono.
B) Extraña los verdes campos de su pueblo.
C) La policía probó que es un ladrón. . 59. Uno de los siguientes no constituyen factor geo-
gráfico en la determinación del clima ¿Cuál es?
O) Tiene la conciencia sucia.
E) Todos dicen que es un tonto. A) La continentalidad (ubicación interior)
B) La energía solar (según la posición de la tierra
55. Indique cuál de los siguientes enunciados presenta respecto al sol)
redacción correcta. ' C) La oceanidad (proximidad a mares o lagos)
A) El auto que su motor reparamos el mes pasado O) La latitud (la temperatura es influida)
sufrió un choque.
E) La vegetación (regulador del calor)
B) No creo que haiga sido una buena idea establecer
estas fórmulas, 60. Los factores que determinan la carencia de llu-
C) Hubieron muchos incidentes que comentar duran- vias en la Costa Central y el Sur del país, son:
te la semana pasada. A) El anticiclón del Pacífico Sur y la humedad relativa.
O) Esta propuesta satisfará a la mayoría del país. B) El clima subtropical y la Corriente Peruana.
E) Se prevee una nueva estrategia nacional. C) La Corriente Peruana y la Cordillera de los Andes.
O) Las llanuras aluviales y el anticiclón del Pacífico Sur.
56. ¿Qué afirmación sobre el indigenismo es incorrecta?
E) La Cordillera de los Andes y la Placa de Nazca.
A) Se desarrollo desde el último tercio del siglo XIX,
hasta la actualidad. 61. Señale la alternativa que 110 establece la relación
B) "Ollantay" es la primera obra indigenista. correcta entre la obra y el departamento en que se en-
C) Ciro Alegria y José María Arguedas hicieron lite- cuentra.
ratura indigenista. A) Proyecto Chinecas - Ancash
O) "Aves sin nido" es una de las primeras obras B) Proyecto Pasto Grande - Arequipa
indigenista. C) Irrigación la Esperanza - Lima
E) "Todas las sangres" es un ejemplar novela O) Reservorio Gallito Ciego - Lambayeque
indígenista. E) Reservorio de Poechos - Piura
0-=;;=:;~:¡1 UN12003-1 APT. ACADÉMICA Y CULTURA GENERAL I';;!=::,::~::;:::~~~:H
62. Las aguas de los dos primeros ríos forman al terce- 67. ¿Cuál fue la actividad económica impulsada a
ro. Señala la alternativa incorrecta. mediados del siglo XIX, en el mundo por el norteame-
A) Pichis, Palcazu, Pachitca ricano Edwin Drake, y en el país por el inmigrante
S) Mantaro, Apurímac, Ene Faustino Piaggio?

C) Lampa, Cabanillas, Coata A) Azucarera S) Bancaria C) Cuprífera

D) Ene, Perené, Ucayali D) Petrolera E) Textil


E) Marañón, Ucayali, Amazonas
68. ¿Cuál es el rubro cuyo pago no constituye un gasto
63. Las esculturas Venus de Milo y Victoria de corriente del Gobierno Central?
Samotracia, así como el antiguo templo denominado A) Transferencias Bl Servicios
Partenón, son obras de la antigua:
C) Remuneraciones D) Bienes
A) Mesopotamia S) Grecia C) Roma
E) Amortizaciones
D) Creta E) Italia
69. La forma de capital externo más volátil cn la
64. Indique cuál de los siguientes hechos sucedió en actual situación de globalización financiera es
la Edad Media.
A) El capital a corto plazo.
A) Fundación de Roma por Rómulo y Remo.
S) El préstamo privado.
S) Construcción del partenón en Atenas.
C) La ayuda oficial al desarrollo.
C) Coronación de Caria Magno
D) La inversión extranjera directa.
D) Carlos V llega a Roma.
E) La inversión externa en privatizacioncs.
E) Toma de la Bastilla.
70. Suponga que la balanza de pagos de un país tiene
65. En el imperio de los incas existían diversas for- en cuenta corriente un déficit de 1000 millones de dó-
mas de trabajo comunitario, como. lares, en cuenta financiera un superávit de 1000 millo-
1. El ayni nes de dólares, en financiamiento excepcional un saldo
Ir. La minca positivo de 30 millones de dólares, y en errores y omi-
siones netos un saldo positivo de 400 millones de dóla-
IIr. La mita
res. Entonces, la reservas internacionales netas han
IV. El ayllu
Al Aumentado en 370 millones de dólares.
V. Los yanacunas
S) Aumentado en 430 millones de dólares,
La alternativa correcta es:
C) Disminuido en 30 millones de dólares.
A) 1, n, IV S) 1, n, V C) 1, Il, III
D) Disminuido en 1400 millones de dólares.
D) !l, Ill, IV E)I,IV, V
E) Permanecido constantes.

66. Fueron autoridades en el virreynato: 71. ¿A qué clase de valor atribuiríamos la actividad
A) El Virrey, la Real Audiencia, el Corregidor y el sexual reproductiva?
Gobernador. A) Ético S) Cultural C) Vital
S) El Virrey, el Cabildo, el intendente y el Superintendente. D) Religioso E) lógico
C) El Virrey, la Real Audiencia, el Cabildo y la Santa
Inquisición. 72. Señala la alternativa que corresponde.

D) El Virrey, la Real Audiencia, el Cabildo, el corregi- "Todos los hombres son mortales; luego:
dor y el Intendente. Algunos hombres son mortales".
E) El Virrey, la Real Audiencia, el Cabildo, la Inquisi- El ejemplo anterior es:
ción y el Intendente. A) Una inferencia inmediata
S) Un silogismo
~~=~~~'·I UN12003-1 APT.ACADÉMICA Y CULTURA GENERAL 17llll!~0
~==>.
C) Una falacia 78. El programa "Techo Propio" puesto en marcha
D) Una tautología por el gobierno, ha sido concebido para
E) Una experiencia deductiva. A) Los fonavistas con más de diez años de aporte.
B) Facilitar créditos para la inversión inmobiliaria.
73. El rasgo se utiliza en psicología para:
C) Dar crédito para vivienda a grupos organizados de
A) Clasificar la personalidad del individuo bajos ingresos.
B) Clasificar los tipos de aprendizaje.
D) Reemplazar al Banco de Materiales.
C) Calificar una forma de la percepción
E) Dinamizar el programa "A Trabajar Urbano"
D) Determinar la motivación
1;) Definir los procesos efectivos. 79. Dadas las siguientes proposiciones:
1. La deserción escolar tiene un alto grado de inci-
74. En la fiesta, no todos se conocían. Sin embargo dencia en el país.
demostraron familiaridad total con la canción de moda,
ll. El país tiene un perfil epidemiológico muy hete-
pues la bailaron siguiendo la misma coreografía
rogéneo, desigual, polarizado, estratificado y de
novedosa.
carácter regresivo, por lo que se ha logrado des-
Esta frase refleja: aparecer las enfermedades transmisibles.
A) Una costumbre. IIl. Durante el año 200 1, los tres principales produc-
B) Un efecto de la sugestión. tos de importancia, fueron: el oro, el cobre y la
C) Una manifestación del lenguaje corporal. harina de pescado.
D) La influencia de las creencias. ¿Cuáles son correctas respecto de la situación
E) La imitación. socioeconómica del Perú actual?
A)I,lIylll B) 1 Y 11 C) I Y 111
75. lrak es oficialmente, de acuerdo a sus actuales D)llylll E) Sólo 1
gobernantes.
A) Un emirato árabe 80. La división política adoptada en el actual
B) Una monarquía constitucional. regionalización, se ha basado en:
C) Una república federal. A) La utilizada en 1985.
D) Una república islámica B) La geografía regional.
E) Una república socialista árabe.
C) La integración intrarregional.
D) Los departamentos y la Provincia Constitucional.
76. En el Consejo de Seguridad de las Naciones Uni-
E) La propuesta del Instituto acional de Planificación.
das, hay países que son miembros permanentes y con
derecho a voto, entre los que se encuentra:
A) Alemania B) Canadá C) Francia
D) Italia E) Japón

77. "Tarija", "Puerto", "gasoducto", "Perú", constituyen


las partes de un proyecto internacional. Identifiquelo ..
A) Exportación del Gas de Camisea.
B) La vía transoceánica en el sur peruano.
C) La explotación y exportación de los fosfatos del
norte peruano.
D) Exportación del gas boliviano.
E) uevo puerto de primer orden ubicado en el sur
peruano.
0:~=;'::;1 UN12003-1 APT. ACADÉMICA Y CULTURA GENERAL I:=~~=:::;~~¡~:~

SOLUOIONARIO
APTITUD ACADÉMICA 6. El contenido del texto trata sobre el estudio de la
conducta social del hombre.
TÉRMINOS EXCLUIDOS
"Mientras numerosos antrópologos han realizado cstu-
1. ESQUIVAR es la palabra que no guarda relación con dios en variadas comunidades campesinas del Perú en un
las demás, ya que está referida a la acción de evitar con esfuerzo por investigar el funcionamiento actual de la
habilidad lo que no es conveniente, por otro lado vejar, reciprocidad, otros han comparado culturas antiguas cuya
satirizar, mortificar y censurar son palabras sinónimos características común fue el desconocimiento del dinero".
que significan hacer padecer a alguien mediante la pala- El conectar mientras indica una relación de paralelismo.
bra o critica.
Clave: B
Clave: B
7. "Esta estrategia es, indudablemente, contunden-
2. PEREJIL es la que no guarda relación con las de-
te, y por lo tanto, muy eficaz".
más porque ésta es una planta umbelífera, cuya hoja es
usado como condimento, sin embargo trigo, arroz, ce- El conectar deben cumplir la relación de causa consecuencia.
bada, café son plantas que pertenecen a la familia gra- Clave: B
mínea que producen sus frutos en forma de granos.
Clave: D 8." Era un tipo elocuente vale decir expresaba clara-
mente sus ideas. Sin embargo, su elocuencia no co-
3. DELICADO es la palabra que debe excluirse ya que rrespondía a sus actos. Por el contrario, frecuente-
se refiere a la fineza o cuidado con que se trata un tema mente sus palabras, y sus hechos entraban en tlagrante
o asunto. En cambio las palabras nimio, pequeiio, exi- contradicción".
guo, minúsculo se refiere a que la importancia del tema
El primer conectar frecuentemente es un advervio
es de escaza magnitud.
de modo, el segundo Por ello es de causa consecuencia
Clave: A
y el tercero a fin de es de condición de finalidad.
4. ERIGIR es la palabra a excluirse ya que significa Clave: C
construir, instituir, por otro lado las palabras sanear, re-
cuperar, recomponer, reconstruir expresar la idea co- 9. "Frecuentemente, la realidad y los deseos se con-
mún de hacer una reparación. funden. Por ello, es necesario distinguir entre lo real y
Clave: E lo deseable, a fin de no frúsn anos en el propósito de
alcanzar el objetivo".

CONECTORES El primer conectar, frecuentemente es un advervio


de modo, el segundo por ello es un conectar consecuti-
5. El contenido del texto trata sobre las consecuencia vo que cumple la relación de causa consecuencia, y el
de las informaciones y la televisión en la sociedad. tercero a fin de es de condición o finalidad.
"EI incremento de informaciones y La proliferación de Clave: E
emociones vicarias a través de la televisión no condu-
cen al enriquecimiento de las personas; antes bien, la
potencia de este medio audiovisual y su utilización al
ORACIONES ELIMINADAS
servicio del libre mercado conlleva a perfilar seres ali-
neados" . 1 O. La oración 111 es la pertinente porque trata de la
El conectar y es de adición, antes bien es una conjun- importancia que son las plazas para el teatro, mien-
ción adversativa y el último conectar y es de adición. tras el contenido de las demás oraciones están relacio-
nadas porque expresan el origen y desarrollo del teatro
Clave: D
en cada una de las etapas de la historia (antigua Grecia, 17. Según el contenido del texto el propósito predo-
Edad Media, Romanticismo, Edad Contemporánea) minante es resaltar la importancia que tiene una ade-
Clave:C cuada formulación del problema.
Clave: D
11 . La oración no pertinente es la J[[ porque destaca
el significado de la palabra calcular, por otro lado las
demás oraciones se relacionan porque describen los me-
ORACIONES INCOMPLETAS
dios y formas para el cálculo de volumen de los cuerpos 1 8. El contenido del texto hace mención a dos carac-
regulares e irregulares. terísticas de la personalidad.
Clave: C Fortalezas y debilidad para luego relacionarse con la
estrategia y como consecuencia obtener seguridad. Por
1 2. La oración que no está relacionada con las demás lo tanto en el segundo espacio corresponde la palabra
es la 11 porque destaca la flora y la fauna en los mares seguro.
polares, en cambio las otras oraciones expresan la exis-
Clave: A
tencia y la ubicación de dos áreas geográficas de tem-
peratura muy baja.
19. El contenido del texto destaca la importancia del
Clave: B fortalecimiento de las instituciones militares del Es-
tado para imponer una autoridad indiscutible.
13. La oración 11 es la impertinente porque hace alu-
Clave: C
sión a la etimología de la palabra cetáceo en cambio las
otras oraciones describen el sistema de ecolocación
20. El contenido del texto se refiere a una conexión
como un proceso de vida conductual de los cetáccos.
de causa-efecto, teniendo en cuenta además que la pala-
Clave: B bra epitafio significa inscripción sobre una sepultura,
por lo que la alternativa que da sentido a la oración es:
14. La oración que no está relacionada con el texto es
la IV porque trata de la prevención de la salud, lo * Los Traidores no suelen inspirar epitafios piadosos.
cual no ocurre con las oraciones restantes que destacan Clave: C
los efectos que sufren los trabajadores debido a los agen-
tes nocivos en sus ambientes laborales.
COHERENCIA DE REDACCiÓN
Clave: D
21. Ordenando cronologicamente seguimos la siguiente
COMPRESiÓN LECTURA secuencia lógica:
ALBERT EINSTEIN:
15. En términos generales el texto trata sobre los
grandes cambios que se dan en la humanidad. Uno debido 111. Nace en ULM, Alemania en Ig79
a la imprenta que se dio en la época Medieval y otro que l. Se desplaza con su familia a Munich y fundan
se está dando en esta época Contemporánea debido al un negocio en 18~W
Internet. Por lo tanto la oración que mejor sintetiza es: V. De niño no sobresalía en los estudios
"Al igual que la imprenta de Gutenberg, Internet trans- IV. Se desplaza con su familia a Milán en I R94
formará radicalmente la cultura en el mundo"
V. En el año 1900, completo su educación en Zurich.
Clave: D Clave: B

16. En líneas generales el texto hace la comparación 22. LOS ESTEREOTIPOS: imagen o idea aceptada
entre dos logros que puede alcanzar el hombre la gloria por un grupo, opinión o concepción muy simplificada
y el éxito, destacando a la gloria como el objetivo am- de algo o de alguien.
bicionada solamente por los hombres con caracteres su-
Teniendo en cuenta lo anterior la secuencia adecuada será:
periores y el triunfo efímero como el objetivo que am-
biciona el hombre mediocre. Il. Definición de estereotipos
Clave: E l. El estereotipo y la conducta a partir de nuestras
experiencias.
~...'~i11mí~.i'~Y«~jl UN12003-1 APT. ACADÉMICA Y CULTURA GENERAL iiiUlmmilltliliW¡¡;¡L0b7fdí~T~ll
~~W&~il¡M&jl...:::':':':'':';::';;'~~~;;';'';;~;';';';';;''';' _
G~I'\teZ§

Ill. Por otro lado pueden transmitirse de generación


a generación.
V Un ejemplo el conjunto de atributos del francés. SERIES NUMÉRICAS
IV Descripción del conjunto de atributos del Francés. 26. Analicemos la sucesión para encontrar el núrue-
Clave: B ro que sigue:

23. Ordenando cronologicamente las afirmaciones acerca I . 19 . SS . 109 . 181 . 271 . 379
del ORIGEN DE LA MATEMATICA se tiene.
~~~~~~
+18 +36 +54 +72 +90 +108
V Las primeras civilizaciones y sus problemas sobre "----"" "----"" "----"" <c.::« "----""
+18 +18 +18 +18 +18
cantidades.
11. Los problemas sobre cantidades en un comienzo Clave: O
tuvieron que resolverse en forma empírica.
IV En Egipto este tipo de operaciones tomó el ca-
rácter de ciencia. 27. Analicemos los cuadros para determinar; X; y> z.
1. Dicha ciencia, se baso en relaciones numéricas
razonadas y exactas.
III. Posteriormente los Egipcios emprendieron y so-
lucionaron problemas de g~ometría.
Clave: O

24. Con respecto a la INGENIERIA CIVIL el orden


adecuado de las afirmaciones será:
Cuadro I 3 + 7 = 10 7 + 1=8
V Definición
Cuadro 11 11+6=17 13+2=15
1. Condiciones para ejercerla, gusto y disposición
por el dibujo. Cuadro 111 21 + Y= 24 Z+X= 22
IIl. Otra condición, el interés por la mecánica estática. Y=3
11. Sus profesionales pueden desempeñarse en dife- => X + Y + Z = 22 + 3 = 25
rentes áreas técnicas.
Clave: E
IV Pueden ser calculistas, interventores y construc-
tores.
28. Busquemos la regla de formación de la sucesión
Clave: C para encontrar el número que sigue.
25. La secuencia lógica que mejor se adecúa es el si-
+8
,...-----...... +8 +8
,...-----......,...-----......
guiente orden:
ARMAS INTELIGENTES 4 ; 4 ; 12 ; 8 ; 20 ; 12 ; 28 ; 16
~~~
IV Concepto general +4 +4 +4

Ill. Descripción de como se desarrolló este sistema Clave: E


armamentista.
11. Este sistema va dependiendo cada vez menos del 29. Analicemos:
control humano.
l. Ahora intervienen mecanismos de control auto- x2+22 x2+24 x2+26 x2+28
mático. ~ ~ ~ ~
2 . 6 16' 40 . 96 . 224 . 512 . 1152· 2560
V La utilización de estas armas, permitirá modificar ~ ~'~'~'
x2 +2' x2 +23 x2 +25 x2+27
la producción bél ica.
Clave:C Clave: O
30. Analicemos: 34. Al observar en forma general los pares de sólidos
nos damos cuenta que en cada par uno de ellos está
3 , 16 ; 45 , 96 ; 175 ;288
<c:> '------.../
<c.::« <c::« <:.:» parado sobre su base y el otro está recostado sobre un
+13 +29 +51 +79 +113 cara lateral, siendo la E el único porque no cumple con
'------.../
<..:» <c.::« '------.../
dicha relación. Porque uno de ellos está parado sobre su
+16 +22 +28 +34
'------.../
<c.::« ~ base menor.
+6 +6 +6 Clave: E
Clave: B

31. Analicemos:
35. Analizando los gráficos:

6 ; 20 , 42 ; 72 ; 110 ;156:
<c.::« <:.:» '------.../
<c.::« <c.::»:
+_14 +22 +30 +38 +46
<c.::« '------.../
<c:> <c::«
+8 +8 +8 +8

Clave: A (1) (3) (4=?) (5)

Observamos que los cuadros, de izquierda a derecha, van


ANALlSIS DE FIGURAS haciendo un giro antihorario ascendente, el primero
giro 90°, el segundo gira 180°, el tercero 270° y el
32. Cuando una figura se ubica frente a un espejo la cuarto 360°, siendo el cuarto gráfico igual que el quinto
imagen se observa invertida horizontalmente. La parte porque llega a la misma posición.
que está a la derecha de la figura aparecerá en la parte
Clave: A
izquierda del espejo y la parte que esta a la izquierda de la
figura aparecerá a la derecha del espejo. Por lo tanto la
alternativa correcta es la C. 36. En cada gráfico se forman dos líneas rectas con 3
puntos cada una. Siendo la alternativa A la que no tiene
una misma Ley formativa con respecto a las otras pues
sólo se puede formar un-línea recta con 3 puntos.

Clave: C
¡: X >\ X ~
A) B) C) D) E)

Clave: A
33.
El triángulo hace un giro de
37. De las figuras:
180° y sus líneas interiores
giran 90°.

CUADRO I

A) B) C) D) E)
Debemos obtener la nisma
relación, que la del cuadro 1 En las alternativas A, ·B, C y E intervienen dos figuras
geornétricas cerradas, en la alternativa D intervienen
cuatro.
Clave: O
CUADRO 11

Clave: B
UN12003-1 APT. ACADÉMICA Y CULTURA GENERAL

El vehículo B dejo de recorrer 2 horas, es decir se malo-


RAZONAMIENTO MATEMÁTICO
gro dos horas antes de las 12 11l.
38. Sea x el número de páginas 12m-2h = 10a/1/
Se tipea Falta tipear Clave: A
Primer día : 80 x-80
40. De los cuatro números, los tres primeros forman
Segundo día : ~Cx-80) tCx-80) una progresión arítmetica y los tres últimos forman una
progresión geométrica. Además el primero y el último
Tercer día : f¡ (f(x-80») 2.. (1(x-80»)
JJ 7 son iguales:

Cuarto día ~ (f¡-(f(X-80»)) t[f¡-(t(x-SO»)] _Progresió').,aritmétic,:


x ; (x+r) ; (x+2r); x
Quinto día 24 O
Progresión geométrica
El quinto día tipeará lo que le falta tipear el cuarto día
Dato: r=6
=> t[f¡-(tCx-80»)] = 24 Por propiedad, en una progresión geornétrica se cum-
ple que el producto de los medios es igual al producto de
x = 388 (páginas)
los extremos:
Clave: E
(x+2r)2 =(x+r)x
39. Datos:
[x+2(6)f =(x+6)x
Velocidad de vehículo A: v A = 50 km/ li
x2 +24x+ 144 = x2 +óx
Velocidad de vehículo B: v 8 = 75 km/ h
18x = -144
Inicialmente: x=8
VA 12 m VB Los cuatro números serán:
x= -8
~ 1"'--_"'--"'--"'--""~"7.""'5:B
x+6 =-2
Encuentro nor~alJ x+12=4

Cuando el vehículo B se malogra, el vehículo A emplea x=-8


1= 3 horas adicionales para encontralo. La suma:

vA
3 horas
,-------A----.. v
B
s = (-8)+(-2)+(4)+(-8) = -14
A- I L -B Clave: B
~

~ncue~;ro ~or;:'J JC':;j C~~~~~~~Z'


12,m

.
6%tE%'~ .... ~

41. Considerando que todos los hombres trabajan por


igual, entonces tendrán una producción "P" por día.
Sea A el grupo de (x-I) hombres que trabajan
dA = \~ I
= (50km/h)(3h) (x + 1) días, entonces:

=150 km. Producción total(A) = .[(X-I).,,--,I': ]x(x+ 1) días


(la
La distancia que recorría el vehículo A es la distancia
=(x-l)(x+I)P ... (1)
que dejó de recorrer el vehículo B, en el tiempo 18'
Sea B el grupo de (x+2) hombres que trabajan
d
lB =.....!l...
(x-I) días, entonces:
"s

= 75 km/h
150 km Producción total(B) = [(x+ 2) (;a ]X(X-I) días

=217 =(x+2)(x-l) P .. (2)


Por condición del problema,
UN12003-1 APT. ACADÉMICA

de (1) Y (2):
Y CULTURA GENERAL

Quedando en el aula:
=~ ;lil:Ú,0

T - R = (80 - 24) alumnos


Producción total(A) _ 9
Producción total(8) - 10 = 56 alumnos

(x-I)(x+ 1) P 9 Clave: D
(x+2)(x-l) P 10
x+1 =-.2.. RAZONAMIENTO LÓGICO
x+2 10
x=8 44. Analicemos las prernisas del enunciado
1: Si me pagas trabajo
Clave: 8
2: Si no me pagas renuncio
42. Datos: as = 5 ag = 135 ... (1) 3: Si me dan un incentivo, no renuncio
Por definición de progresión geométrica 4: Me dan un incentivo o denuncio a la cm presa
5: No trabajo.
(1" =ar,,-I ... (*) Las conclusiones lógicas que se derivan son:
a: No me pagan de I y 5
Para 11 =5 :
b: Renuncio de a y 2
De (1): ... (2) c: No me dan incentivo de h y 3 ... 11
d : Denuncio a la empresa de C y 4 ... 111

Para 11 =8 : Entonces las conclusiones 11 y 111son las derivables


Clave: C
De (1): 135=ar7 ... (3)
45. Sean los locales comerciales: L, M Y N.
Dividiendo (3) entre (2):
Asumiendo el costo de un producto en cllocal M igual a
ill ar7 SI. 100 se deduce que:
5 a/"4
Local costo interés costo total
~ /"=3 ... (4) L -7 S/.IIO +15%110 = S/, 126.5

Reemplazando (4) en (2): M -7 S/.IIO +10%100 = S/.I l O


N -7 S/.95 +20%95 = S/.I 14
5 = a(3)4
El orden recomendable sería M, N, L.
5
a =81 ... (5) Clave: C
Finalmente, reemplazando (4) Y (5) en (*):
INSUFICIENCIA DE DATOS
6
a7=(il)3 46. Analizando según el gráfico y los datos de las alterna-
tivas:
=45
Con el dato I no se puede determinar la ubicación del
Clave: 8 punto E por lo que no se podría calcular el área
sombreada.
43. Del enunciado
Con el dato 11 se sabe que E cs punto medio de DC
Inicialmente ingresaron: 64 alumnos Además.
Posteriormente ingresaron: 16 alumnos
DE = EC = 8 cm =J.DC
Total T= 80 alumnos 2
~ DC=16clII
Se retiran el 30% de total:
Con el valor de DC que es el lado del lado del cu~drado
R = 130~x (80 alumnos'¡ = 24 alumnos se puede calcular el área sombrcada.
Clave: 8
0~·
~
~-~l
88 - --

47. Por criterio de conjuntos:


~ __ ' ,
UN12003-1 APTo ACADÉMICAYCULTURA GENERAL I~::!-::!~·:=·$
~W~-:':o:_>._.X:
(\.JonH~L \

a(a+ 1)+ I
Nos piden calcular: ~((/ -1) + 2 '00 (1)

Analicemos a la variable a:

• Si a es par, entonces a+ I Y a -1 es impar:

En (1): a(a+I)=1
En (11): 13(0-1)=-1
Reemplazando estos valores en (111):
21
a(a+I)+1 =~=2
Los músicos de percusión no tocan otro instrumeto. ~(a-I)+2 -1+2 '" (2)
=> Pr.C=Pr.V=Pr.Cr.V=0
• Si a es impar, entonces C/ +I Y a -1 es par:
Músicos que tocan cuerda y viento a la vez: x
Del enunciado del problema se tiene: De (1): a(a+I)=O
x+y+z=15 '.0 (1)
De (Il): ~(a-I)=O
w=5
Reemplazando estos valores en (111):
De las afirmaciones (1) y (11) se tiene:
a(a+I)+1 0+1 I
x+y=10 ... (2) ~(a-I)+2 = 0+2 =2 " (3)
x+z=8 ···0) Luego de (2) y (3) se deduce que los únicos valores que
Para hallar la información requerida se deben resolver
puede tomar la expresión (3) son: el menor 1/2, Y el
el sistema de 3 ccuaciones con 3 incógnitas, para el cual
se ha utilizado la información (1) y (11).
mayor 2.
Clave: D Las afirmaciones III y IV son verdaderas,
Clave: B

OPERADORES MATEMÁTICOS 50. Datos:


48. Datos: a(n) = 2n+1 ." (1)
f(1/+1) = /13 -1 ... (1)
~(/1)=n2 '" (2)

G(I/_I) =n2+1 ... (11) Calculando los valores numéricos con las reglas
operativas anteriores:
Operando en (1) y (2):
3 De (2): ~(3) = 32 = 9
De 1: f(3) = f(2+1) = 2 -1 = 7
De(I): a(9)=2(9)+1=19
De 11: G(3) = G(4_1)=42 + 1= 17
De(I): a(2)=2(2)+1 =5
::} F(J)xG(3)=7xI7=119
Reemplazando éstos valores en la expresión:
Clave: B
a(~(3)) + I a(9) + I
49. Datos: a(2) 5
19+1
I; si /1 es impar
a ()n ={ '" (1) 5
O ; si n es par =4
si n es par Clave: B
~(n)={ O ... (11)
• -1 si 1I es impar
;;~JNi::=::::~=1 UN12003-1 APT, ACADÉMICA Y CULTURA GENERAL 1;:i~:;~:=0
C) INCORRECTA: El verbo haber (Hubieron) es im-
personal por lo tanto no se puede conjugarse en
CULTURA GENERAL plural.
D) CORRECTA: Este enunciado presenta una correcta
51. En las oraciones: redacción.
l. Hay una expresión interrogativa, la palabra "qué" E) INCORRECTA: El verbo prever (prevec) en pre-
lleva tilde. Por otro lado la palabra "haré" es sente se escribe prevé
ay~da y termina en vocal y por t~nto lleva tilde. Clave: D
I!. En esta oración ninguna palabra llevan tilde se-
gún las reglas de acentuación. 56. Indigenismo es un género literario que elige tipos
Ill. En la oración III la palabra público es una pala- y asuntos indígenas, también es un movimiento ideológi-'
bra esdrújula por lo tanto lleva tilde en la u. ea social americano en favor del elemento indígena.
Por lo que las oraciones bien acentuadas son la I y Il De'las alternativas que se dan, la (B) no se relaciona con
el tema del indigenismo. La obra OIlantay es un drama
Clave:C
que está relacionádo con la literatura española.
52. En el enunciado Clave: 8
A) La palabra basta se escribe Vasta ya que se refiere a
57. Este poema:
extensión.
B) La palabra "sumo" de limón, se escribe Zumo por- "Yo no me río de la muerte
que se refiere a jugo, extracto. sucede, simplemente,
(
C) La palabra "suscinto' es la errada pues debe ser que no tengo miedo de morir
sucinto ya que significa conciso, breve, etc. entre pájaros y árboles"
D) La palabra "ollas" en la cual hace referencia se es-
cribe hoyas. forma parte de la obra literaria "El Viaje" escrita por
E) Todas las palabras presentan buen uso de la 0110grá- Javier Heraud, quien fue asesinado por las fuerzas
fica. militares en Madre de Dios a los 21 años,
Clave: E Clave: D

53. La oración de la alternativa (A) presenta el verbo


58. Capa de ozono es un estrato que contiene variedades
cantar y bailar éstos son verbos predicativos.
alotrópicas de oxígeno y está ubicada en la zona
En las oraciones de las alternativas B, C, D Y E los estratosférica de la atmósfera. Esta protege a los seres
verbos estar, permanecer, parecer y ser son vivos de la acción de los rayos ultravioletas y se destruyen
copulativos porque no expresan una idea completa (sin al reaccionar con compuestos químicos que contienen cloro
sentido). que son emanados de la tierra, más no por vientos fuertes.
Clave: A Clave: E
54. Epíteto es el adjetivo o equivalente que no determi-
59. Los factores geográficos del clima están referidos
na ni califica al sustantivo sino que acentúa su carácter.
a la superficie terrestre. Ejrn. continentalidad, oceanidad,
Ejemplo: La negra noche latitud, vegetación. Más no la energía solar pues este es
De lo anterior la oración que presenta epíteto es: un factor cósmico del clima.
Clave: 8
Extraña los verdes campos de su pueblo
Clave: 8 60. Analizacernos los factores.
La corriente Peruana es una corriente de agua fría,
55. De los enunciados su redacción es:
esto hace que las aguas del Pacífico se enfríen y por lo
A) INCORRECTA: Las palabras "que su", no son las tanto las masas de aire húmedo se estabilicen,
adecuadas deben cambiarse por cuyo. ocacionándose así la escasez de las precipitaciones en la
B) INCORRECTA: La palabra "haiga" debe cambiarse costa central y sur del país.
por haya.
I
~
~:liQ
G~meZ

La cordillera de los Andes, debido a su altitud no 66. En la época del Virreynato se nombraron las si-
permite el paso de la humedad ejercida por los vien- guientes autoridades:
tos alisios del lado oriental, por lo que se hace escaza
Virrey : Representante directo del rey en la co-
la precipitación de las lluvias en las zona occidental
lonia.
de los Andes.
Audiencia : Fue la segunda autoridad en importan-
Clave: C
cia en la colonia.
61. De las alternativas, la incorrecta es el proyecto Cabildo : Administraban los centros urbanos.
Pasto Grande, pues está ubicada en el departamento
Corregidor: Se dedicaba al comercio privado den-
de Moquegua y no en Arequipa. Esta es una obra
tro de sus provincias sustituido poste-
agroenergética que tiene como finalidad la de generar
riormente por los intendentes.
energía hidroeléctrica y el riego el; los valles.
Clave: D
Clave: B
67. En la época imperialista de EE.UU. en el siglo XIX,
62. Los ríos Ene y Perené se unen en el departamento
el norteamericano Edwín Drake impulsa la actividad
de Junín formando el río Tambo, más adelante este río
petrolera generando así la segunda revolución industrial en
con el Urubamba forman el río Ucayali.
el mundo. En nuestro país fue el inmigrante italiano
Clave: D Faustino Piaggo quien se dedicó a la actividad petrolera.

63. Las esculturas Venus de Milo, Victoria de Samotracia Clave: D


y el Templo Partenón son obras de la antigua Grecia,
68. En el rubro de gastos corrientes incluyen los
pues en esta sociedad se desarrollo una arquitectura donde
gastos operativos del gobierno, estos son: Transferen-
cultivaron tres estilos; El Jónico, el Dórico y el Corintio,
caracterizándose cada uno por el tipo de columnas usa- cias a poblaciones de bajos recursos, compra de bienes y
das en mas edificios. servicio, pago de remuneraciones de los empleados.

Clave: B Por otro lado los pagos por amortizaciones están cn


el rubro de servicio de la deuda.
64. Al decaer el sistema esclavista surge otro sistema Clave: E
(Feudalismo) donde el dueño de la tierra: Señor feu-
dal, es el que tiene todo el poder, y explota al siervo de 69. En la globalización existen 2 tipos de inversiones:
la pleba, mayoríade la población. El principal intento
Inversión extranjera: Consiste en construcción de
de organizar el sistema feudal en base a la centraliza-
inmueblcs, compra de empresas públicas, etc.
ción del poder político lo emprendió el rey Franco Carlo
Magno en alianza con la Iglesia Católica quien respal- Inversión a corto plazo: Consiste en depósito en el sistc-
dó este proyecto. Esto se expresó en la coronación de ma financiero, compra de acciones en el mercado bursátil.
Caria Magno por el Papa León 11I en Roma, durante la Siendo la inversión a corto plazo muy inestable o volátil
edad media (476 a 1453d.c) porque ingresan y se retiran fácilmente de una economía.
Clave: C Clave: A

65. En el imperio de los lncas el trabajo era en con-


70. Según el enunciado del problema la balanza de
junto, obligatorio y además organizado y lo desarrolla-
pagos de un país se distribuye así:
ban de la siguiente formas:
Ayni : Trabajo dentro del Ayllu como apoyo entre Cuenta Corriente déficit (egreso) $1000 millones
familias. Cuenta Corriente Superávit (ingreso) 100 millones
Minka: Trabajo comunal de tierras del inca y del sol. Financiamiento excepcional (ingreso) 3 O mi llenes
Mita : Trabajo obligatorio desde 18-50 años en la Errores u omisiones netos (ingreso) 400 millones
construcción de caminos, andenes, camellones. Saldo final (ingreso) $430 millones
Clave: C Clave: B

L
~ÚJ:ÜIlB
G~meZ
illt ~I UN12003-1 APT. ACADÉMICA Y CULTURA GENERAL

71. Según la axiología, ciencia que estudia los valores 77. Tarija es un yacimiento de gas natural ubicado en
morales, la actividad sexual reproductiva está -atribuida Bolivia. Actualmente se está definiendo cuál será la ruta
a la clase vital. que recorrerá para ser exportado al mercado Norteamc-
Clave:C ricano, puede ser por un puerto chi leno o peruano.

72. Las inferencias son deductivas e inductivas. Clave: D


Las deductivas pueden ser mediatas (dos o más premisas)
e inmediatas (una premisa): 78. El programa Techo Propio ha sido implerncntado
En el enunciado: por el gobierno con la finalidad de dar crédito para
vivienda a los pobladores de bajos recursos económi-
Premisa : Todos los hombres son mortales cos, que deben contar con un ahorro [0% más un bono
familiar hipotecario y el crédito respectivo.
Consecuencia: Algunos hombres son mortales, esto
es una inferencia inmediata. Clave: C
Clave: A 79. Según las preposiciones:
73. En psicología el rasgo se define como una ten- 1. Verdadero
dencia a sentir, pensar o actuar de una manera determi- La deserción escolar tiene un alto grado de inci-
nada y frecuente, permitiendo con ello determinar la dencia en el país, debido al estado socio económi-
personalidad de un sujeto. co de la mayor parte de la población que no tiene
Clave: A los recursos económicos necesarios.

[1. Verdadero
74. En' una fiesta existen muchas formas de manifes-
taciones sociales, tales como: las costumbres, sugestio- Debido a la presencia de niveles socio económi-
nes, el lenguaje, las creencias, la imitación, etc. Está cos muy diversos y desiguales en nuestro país se
última hace referencia aun mecanismo de aprendizaje tiene un perfil epidemiológico muy heterogéneo,
social en el cual se reproducen las conductas de un mo- así podemos encontrar sectores con alto índice de
delo. En la pregunta, la expresión" bailaron si- enfermedades infectocontagiosas, pero también
guiendo la misma coreografía novedosa", alude a la de- encontramos sectores en donde se han erradicaqos
finición anterior. estas enfermedades. ,
Clave: E
Ill. Verdadero
75. Irak durante el gobierno de Saddam Hussein desde Durante el año 200 1, según aduanas, se exportó
1979 se autoproclamó REPUBLICA SOCIALISTA 6955,97 millones de dólares siendo los tres prin-
ÁRABE respaldado por su Partido Político BAAS (re- cipales productos de exportación tradicional, en
surgimiento). En la actualidad, soportan una invasión el orden de importancia:
por parte de los ejércitos de Inglaterra y EE.UU, quie- El Oro:
nes eligirán un nuevo gobernante. I 166,15 millones de dólares
Clave: E El Cobre:
985,85 millones de dólares
76. El consejo de seguridad de las Naciones Unidas (O.N. U.)
es el órgano cuya función principal es mantener la paz y La harina de pescado:
seguridad mundial. 835,07 millones de dólares.
Está formado por 5 miembros permanentes y con dere- Clave: A
cho a voto, éstos son:
80. La actual regionalización se basa en los 24 dcpar-
EE.UU., Francia, Rusia, Inglaterra y China y 10 miem-
tarnentos y la provincia constitucional del Callao es así
bros no permanentes designado por la asamblea gene-
como en las últimas elecciones se eligieron a 25 presi-
ral de las Naciones Unidas para un período de dos años.
dentes regionales.
Clave: C
Clave: D
~S:====@UN~ll2~00~3~-"1A~P~T.~A~C!A~D~ÉM~I~C~A!Y~C~U~lT~U~RA~G~E~NE~RA~l]:~:¡h:B:.~=~~~:~

~~~~~~~

APTITUD ACADÉMICA Y CULTURA GENERAL

RAZONAMIENTO MATEMATICO 7.

SUCESIONES NUMÉRICAS
En cada una de las sucesiones numéricas propuestas,
?•
indique la alternativa que continúa. ~ ~ ~ ~
1. 7, 13 ,37, 145, ?
A) 652 B) 66g C) 694 D) 712 E) 721

2. 25,49,121,361,?
A) 625 B) 729 C) 900 D) 1225 E) 961
~ ~ ~ ~ ~
3.3,12,28,51.? A) B) C) D) E)

A) 83 B) 82 C) 81 D) 80 E)79 8.

4. 2, 3 , 5 , ? , I7
A)9 B) 10 C)ll D) 12 E) 14 • mBIl ?•
5. Determinar la suma de los números del grupo que
ocupa el casillero UN!.

I 3-5-4

A) 338
I 8-11-9

B) 339
127-31-281

C) 340 D) 342
UNI

E) 356
I IIgm ••
A) B) C) O) E)

9_

ANÁLISIS DE FIGURAS
Indique la alternativa que continúa, en las siguientes
series gráficas propuestas:
6.
~{O~
Ir---~Ol
? ISoI:r I~I
~&~
A) B) C) D) E) {O~ ?•
," ·1 UN12003-11 APT. ACADÉMICA Y CULTURA GENERAL r
A) 20 000 Bl 22 000 C) 25 000

r» 2R 000 E) 32 000

14. Un granjero cria patos, gallinas y conejos. La can-


tidad de gallinas duplica al número de patos, así como los
A) B) C) conejos son tantos como los patos y gallinas juntos.
Si el granjero vende cinco patos y diez gallinas, el nú-
mero de conejos es el doble del número palos y gallinas
que quedan. ¿ Cuántos conejos existen'?
A) 15 B) 30 C) 40 D) 45 El 60

D) E) 15. Cuando la vía esta libre, un automóvil se desplaza


con el doble de la rapidez de un bus, pero cuando en la
10. vía hay otros vehículos sólo se desplaza 20'Y., más

~~~~
?. rápido que el bus.
Si el auto tarda 4 horas, para recorrer cierta distancia,
cuando el 25% del tiempo la vía esta libre ¿Cuánto

EOOEW~~EW3
A) B) C) D) E)
tardará el bus en recorrer la misma distancia bajo las
mismas condiciones
A) 4,2 horas
que el automóvil'?
B) 4,5 horas C) 5,2 horas
D) 5,4 .horas E) 5,6 horas
11.
16. José sólo gasta en pagar pasajes cuando va a la
Academia porque su padre lo recoge.
Cuando toma el bus en la esquina de su casa, gasta
SI. I ,20, pero si camina cinco cuadras gasta sólo SI. O,gO.
Si después de 30 días, gastó en pasajes SI. 28. ¿Cuántas
cuadras caminó para ahorrar en sus pasajes?
A) 20 B) 40 C) 60 r» so El 100

RAZONAMIENTO LÓGICO
RAZONAMIENTO MATEMÁTICO 17. ¿Qué es respecto a mi el abuelo materno del
12. Un cajero automático debe entregar 740 soles, mellizo de Mauro, si la madre de Mauro es la hermana
empleando billetes con las siguientes denominaciones: de mi hermano gemelo?
100, 50, 20 Y 10 soles. Si debe emplear todas las deno- A) Abuelo B) 11 ijo C) Tío
minaciones y el menor número de billete ¿Cuántos bi- D) Padre E) Yerno
lletes entregará el cajero?
A) II B) 12 C) 13 D) 14 E) 15 18. Una evaluación escrita, consta de cinco pregun-
tas, las cuales tienen sólo dos alternativas: falso (F) y
13. A un estadio, sólo asisten hinchas de los siguientes verdadero (V). Si se sabe que:
equipos de fútbol: 50% son del Alianza Lima, el 50% del 1) Hay más preguntas eon clave falso, que con clave
resto son de Universitario de Deportes. Los hinchas del verdadero.
Sporting Cristal son el doble del Wanka y los del Sport
11) No existe tres preguntas consecutivas con la mis-
Boys son la misma cantidad que los del Sporting Cristal.
ma clave.
Si los hinchas del Wanka son I 000 ¿Cuántos hinchas III) La primera y la última pregunta tienen respuestas
asistieron, en total, al estadio? contrarias
0=1 UN12003-11 APT. ACADÉMICA Y CULTURA GENERAL I:=-::;::=;~::me: U

¿Cuántas preguntas tienen como respuesta, la clave


verdadero?
A)O B)2 C)I D)3 OPERADORES MATEMÁTICOS
E) Faltan datos. 22. Se define los operadores:

19. Alfredo, Braulio y Carlos son: electricista, solda-


dor y carpintero (no en el mismo orden necesariamen-
0= N-l

te) y llevan uniformes: blanco, amarillo y rojo. Carlos


Determine el valor de R, si:
y el soldador juegan a menudo bingo con el que usa el
uniforme rojo quien no es amigo de Alfredo.
Sabiendo que el electricista usa el uniforme blanco, ¿qué
oficio tiene Braulio y de qué colores el uniforme que usa?
A) Soldador - Rojo D) Soldador - Amarillo
Para N= 5
B) Electricista - Blanco E) Carpintero - Rojo
A)2 B)3 C)4 D)5 E)7
C) Carpintero - Blanco

23. Se define el operador Ó , tal quc:


SUFICIENCIA DE DATOS Ó(N)=IO, Si N> I
20. ¿Qué edad tiene el menor de tres hermanos,si el Ó(N) = O, Si N::; I
mayor tiene 10 años más que él y 3 años más que el
Determine el número de valores que puede tomar la
segundo?
función R. si Z E [1.2 99]
Información brindada:
1) El segundo tiene I I años
(1) La suma de las edades de los tres hermanos es 29
R= ó[ Ó(30~ - Z)]
aiios.
Para resolver la pregunta: A) I B) 2 C)3 D) 149 D) 29g .

A) La información 1 es suficiente.
2
B) La información 11 es suficiente. 24. El operador P( /1 - 1) = /1 - 1
C) Es necesario emplear ambas informaciones.
Hallar el valor de P( a ) + P( 3)
D) Cada una de las informaciones, por separado, es suficiente.
E) La información brindada es insuficiente. 2 2
A) a + 2a + 15 D) a - 2a + 3

B) a2 + 2a + 16 E) a2 - 2a + I (i
21. Una empresa necesita cubrir 17 nuevos puestos de
trabajo, para lo cual realiza un concurso de selección. Si 2
C) a + 2a + 3
en el concurso se presentan 36 postulantes en total y
en 13 puestos se presenta más de una persona por pues-
to, ¿qué dato es suficiente para saber en cuantos pues-
tos se presentarán 3 postulantes?
25. Si [ x" ] = bxb-I y

1) El número máximo de postulantes


(1) El total de puestos donde hubo dos postulantes
por puesto es de 3.
fue 7.
f (x + 1) = [x 2] + 3[ x 3] + f (x)
111) En todos los puestos, por lo menos hubo un postulante. Calcular f (2) sabiendo que I (4 ) = 2
A) 1, 11, III B) 11Y III C) 11
A)-S5 8)-105 C) -120
D) I Y 11 E) Falta información
D)-40 E) 125
t ~i~\\~:=';:=~~::_:"1
G~iñe~
UN12003-11 APT ACADÉMICA Y CULTURA GENERAL l' ';~ ~
~

A) Si - o - por eso,
RAZONAMIENTO VERBAL
B) Porque - pues - pero
C) Si bien - y - empero,
TÉRMINO EXCLUIDO
D) A pesar de que - como - no obstante,
26. Identifique el término que no guarda relación de
E) Dato que - ni - también
significado común con los demás,
A) prototipo B) bosquejo C) modelo
32. Marque la alternativa que, al insertarse en los espa-
D) arquetipo E) muestra cios en blanco del enunciado, dé un significado correcto:

27. Identifique el término que no guarda relación de Este formulario permite ahorrar tiempo evita
significado común con los demás, hacer gastos adicionales; no debemos escribir
innecesariamente, anotar sólo lo indispensable,
A) sojuzgar B) someter C) Avasallar
D) oprimir E) invadir A) pero - por eso - y
B) ya que - entonces -y
28. Identifique el término que no guarda relación de C) y - pero - sino
, significado común con los demás,
D) o - y - aunque
A) superávit B) renta C) ganancia E) porque - por ello - más
D) déficit E) beneficio
33. Marque la alternativa que, al insertarse en los
29. Identifique el término que no guarda relación de
espacios en blanco del enunciado, dé un significado
significado común con los demás,
correcto:
A) homicidio B) fratricidio C) suicidio
"Los viejos deben temblar __ los niños, __ la
D) magnicidio E) parricidio
generación que se levanta es siempre acusadora
juez de la generación que desciende",
CONECTORES
A) a - pero -O
30. Marque la alternativa que, al insertarse en los B) ante - porque - y
espacios en blanco del enunciado, dé un significado
C) con - si bien - y
correcto,
D) por - pero - aunque
has omitido letras algunas palabras,
E) sin - dado que - ni
__ no has respetado los signos de puntuación, repí-
tela logres una lectura fluida,
34. Marque la alternativa que, al insertarse en los espa-
A) Entonces ~ y - pero - para que
cios en blanco del enunciado, dé un significado correcto:
B) Pese a que -o - y - aunque no
C) Aunque y - y - sin que "Cuando el uso de la fuerza se hace necesario e inevita-
ble, la civilización exige que, , ante el triun-
D) Pero - y - y - sin que
fo, se muestre ponderación y decoro; y, ' fren-
E) Si - o - o - hasta que
te a la derrota, resignación y esperanza",

31. Marque la alternativa que, al insertarse en los A) por un lado - por otro lado
espacios cn blanco del enunciado, dé un significado co- B) primero - después
rrecto: C) mientras - luego
D) por ejemplo - por eso
en el mundo científico la casualidad rara ves es
reconocida __ un factor que contribuye a los descu- E) si bien - también
brimientos importantes; ' en la realidad exis-
ten notables excepciones,
UN12003-11 APT. ACADÉMICA Y CULTURA GENERAL I'~~~~ ~.
UJllU
G~il,eZ

ORACIONES ELIMINADAS
38. Elija la oración que no tiene relación directa con
35. Elija la oración que no tiene relación directa con el tema central del texto.
el tema central del texto:
l) A la emisión de agua y vapor a gran altura, se
1) Para sus fines la psicología observa, describe y denomina géiser.
explica los fenómenos psicológicos. [1) El géiser es un fenómeno beneficioso e imprevisi-
(1) Los tipos caracteriales se definen según el modo ble.
peculiar de comportamiento. 1Il) El géiser es una manifestación devolcanismo ate-
lll) Los métodos de la psicología son la introspección, nuado.
la extrospeeción y la experimentación entre otros. IV) La periodicidad en la emisión de géiseres varía.
IV) Los fenómenos psiquicos son subjetivos, tempo- Y) Los géiseres han sido considerados como aguas
rales y no perceptibles por los sentidos. medicinales.
Y) La psicología es la ciencia que estudia los fenó-
menos de nuestra experiencia mental. A) I B) II C) III D)IV E)V

A)I B) II C) 1I[ D) [V E)V 39. Elija la oración que no tiene realción directa con
el tema central del texto.
36. Elija la oración que no tiene realción directa con
el tema central del texto. I) El 31 de octubre se encontró una gran cantidad de
lingotcs de oro y plata en Nueva York.
1) La conquista significó la evangelización forzada y
no forzada de las poblaciones indígenas. ll) Este hallazgo fue entre los escombros de las To-
rres Gemelas.
(1) Ya en el año siguiente del Descubrimiento, Alejan-
dro VI exhorta al rey de Castilla: "Quiera y deba [11) La elaboración de los lingotes de oro necesita pa-
inducir a las poblaciones de estas islas y tierras a sar por un proceso de rcfinación muy laborioso.
acoger la religión cristiana". IV) Un barco canadiense había informado que en sus
1(1) Para Wachtel, la expedición de Colón coincide oficinas en las Torres Gemelas, tenia lingotes por
con una nueva era para Europa. más de 200 millones de dólares.
Y) Los lingotes fueron trasladados a un lugar seguro.
[V) Así fue como se iniciará una expansión misionera
que acompañará la expansión territorial. A) [ B) 11 C) 111 D) IV E)V
Y) Mientras tanto, en Europa se desarrolla el debate
respecto a la "naturaleza humana" de los indios.
A) I B) II C) III D)IV E)V COMPRESiÓN DE LECTURA
40. "La hipótesis o supuesto, es una afirmación ten-
37. Elija la oración que no tiene relación directa con tativa a partir de la cual se descubren un conjunto de
el tema central del texto. proposiciones. Si se comprueba cmpiricamcnte, ella
l) La restauración se ocupa de reparar el deterioro adquiere la condición de una proposición científica".
de un objeto histórico. De acuerdo con el texto, una hipótesis se transforma
1I) La restauración, también, es una actividad que está en proposición científica cuando se:
orientada a interrumpir el proceso de deterioro A) analiza una teoría.
de dicho objeto.
B) aplica la norma rigurosamente.
lll) En la restauración, se utilizan técnicas que varían
C) propone una explicación tentativa.
en función del objeto que se va reparar.
D) verifica su validez empírica.
IV) Además de reparar su deterioro, el propósito es
E) siguen métodos comparativos.
devolver el aspecto original de dicho objeto histó-
rico. 41. "Un líder no es el que hace o piensa por los demás,
Y) El objeto histórico es patrimonio de cada grupo si no el que capacita y promueve que la gente piense y
humano. haga por sí misma".
A)[ B) [[ C) ut D)IV E) V Del contenido del texto-:-se concluye que:
:: m.:::::::S~=:~::1 UN12003-11 APT. ACADÉMICA Y CULTURA GENERAL 1::,:aw::=:!0
A) es más práctico para el líder hacer él mismo [as A) ignorante - CÍnico
cosas que esperar que hagan los demás. B) ingenuo - cándido
B) es líder el que contribuye a que los demás resuelvan C) indeciso - irrésoluto
ellos mismos sus asuntos.
D) traidor - cobarde
C) por [o general, [a gente quiere que [e enseñen a
E) eximio - utópico
hacer las cosas.
D) [os líderes se hacen en [a práctica adiestrando a sus 45. Indique la alternativa que completa adecuada-
seguidores. mente el significado de la siguiente oración.
E) hay incompatibilidad entre lo que hace [a gente y "¿Cómo podré a los demás si no tengo el
piensa su líder. ______ de mi mismo'?"

42. "Galileo es famoso, con todo derecho, por defen- A) educar - alumno y paciencia
der la idea de que [a Tierra gira alrededor del sol, en B) complacer - espíritu burlón
contra de [as autoridades religiosas que no consideraban C) convencer - don y la palabra
conveniente esta idea. Fue el primer mártir de [a cien- D) elegir - elector y los votos
cia, a pesar de que abandonó públicamente sus ideas,
E) gobernar - dominio y control
sometido por [a Inquisición. Pero Ga[i[eo no inventó [a
teoría heliocéntrica, sino Nicolás Copérnico, que pos-
teriormente fue perfeccionada por Kepler, No obs-
COHERENCIA DE REDACCiÓN
tante, [a verdadera grandeza de Galileo radica en que él
implantó el método experimental, del que [a ciencia 46. Lea los enunciados y elija la alternativa que esta-
moderna depende". blece la secuencia correcta de [as oraciones según el
título.
De acuerdo con el texto:
LASVÁRICES
A) Gal ileo fue el primero en sostener [a teoría
heliocéntrica. 1) Las várices con dilataciones patológicas de las venas.
B) La religión siempre ha rechazado a [a ciencia. Il) Por los vasos sanguíneos circulan la sangre al
C) Galileo no debió renunciar a su teoría. corazón.
II 1) Las várices, también pueden presentarse en otras
D) Copérnico fue superior que Galileo.
partes de [ cuerpo.
E) Galileo desarrolló el método experimental.
IV) Las venas afectadas son generalmente las de las
piernas.
ORACIONES INCOMPLETAS V) Esta dilatación se produce en los vasos sanguí-
neos.
43. Indique la alternativa que completa adecuada-
mente el significado de la siguiente oración. A) I-V-Il-IlI-IV D) I-V-IV-IlI-II

Quedó completamente con la imagen que B) IV-I-V-Il-IlI E) IV-V-I-IlI-1I


se presentaba ante sus ojos. Desde lo alto de aquella C) I-V-IlI-IV-lI
colina, podía la llanura en toda su _
A) sorprendido - instaurar - proporción.
47. Lea los enunciados y elija la alternativa que establece ,
la secuencia correcta de las oraciones según el título.
B) comprometido - integrar - condición.
C) impresionado - coludir - amplitud. MOTORES Y CENTRALES HIDROELÉCTRICAS
D) maravillado - comtemplar - vastedad. 1) Las plantas de energía hidroeléctrica requieren dife-
E) ilusionado - descifrar - conjunción rentes turbinas con funciones y potencias especificas.
Il) La turbina Pelton, por último, sólo puede usarse
44. Indique la alternativa que completa adecuada-
en las usinas de alta presión en las que el agua
mente el significado de la siguiente oración.
desciende más de 200 metros.
"Es posible tolerar y aún perdonar al ; pero
I1I) La turbina Kaplan, por ejemplo, se usa cuando el
al , jamás".
0=:-~:=:1 UN12003-11 APT. ACADÉMICA Y CULTURA GENERAL I:~=::=¡:;:
L.-"'.>='-" eZ

agua desciende a menos de 150 metros. 50. Lea los enunciados y elija la alternativa que
IV) La energía de la caída del agua se usa en las plantas establece la secuencia correcta dc las oraciones, según
hidroeléctricas para generar electricidad. el título.
V) La turbina Francis, en cambio, se usa en las usinas EVOLUCIÓN DEL DIBUJO
donde el agua desciende entre 150 a 200 metros.
1) El uso posterior del pincel estaba condicionada a
un tipo de superficie.
A) IV-I-IlI-V-1I D) IV-I-II-V-1I[
B) 1II-I-IV-V-lI E) 1-IIl-V-IV-[[ I1) Los dibujantes que realizaban su labor con tinta
C) I-IV-IIl-Il-V china aún se servían, en algunos casos, del pincel.
IlI) Esto se debió, gracias, al descubrimiento del papi-
ro y el pergamino.
48. Lea los enunciados y elija la alternativa que estable-
ce la secuencia correcta de las oraciones según el título. IV) Así, durante la Edad Media, lograron elaborar su-
perficies para el uso del pincel.
EL CUBISMO
V) Problamente, las técnicas de los dibujos hechos en
1) Supuso el rechazo del estilo narrativo y figurativo que cabón fueron los más antiguos.
había imperado durante tanto tiempo en la pintura.
11) Buscaba representar todos las características de A) V-I-I1I-IV-1I D) I-V-III-IV-ll
los objetos. B) I1-IV-IlI-I- V E) I1-V-llI-IV-1
lll) Género, además, una serie de estilos nuevos. C) V-I-IV-IlI-I1
IV) Estos trabajos chocaron al público por la tosque-
dad de su forma.
V) El cubismo es una de las vanguardias artísticas más CULTURA GENERAL
importantes del siglo XX.
51. El descubrimiento de las tumbas reales del Sciior
A) V-J-IV-lI-IlI D) V-I-lI-IV-1II de Sipán, ha permitido:
B) 1-V -I1-I1I-IV E) I-¡¡-IV-V-IIl A) Desarrollar estudios sobre el universo ideológico
C) V-III-I1-I-IV Moche.
B) Reunir el mas j'alioso conjunto de tejidos
49. Lea los enunciados y elija la alternativa que estable- prehispáuicos.
ce la secuencia correcta de las oraciones, según el título. C) Investigar los enterramientos en altura, tradicio-
nales en la Cultura Andina.
EL TRANSPLANTE
D) Incrementar el conocimiento sobre las Necrópolis
1) El trasplante más frecuente es la transfusión de Paracas.
sangre. E) Consolidar la visión sobre la Arquitectura de los
I1) El transplante es la sustitución quirúrgica de cier- incas.
tos tejidos u órganos por otros similares.
1Il) Tiene dos problemas: Las características propias
52.¿ En qué gobierno se impulsó un vasto proceso de
reformas estructurales y nacionalización de empresas
de toda intervención y la posibilidad de rechazo.
extranjeras?
IV) Para evitar!o, se debe seleccionar muy bien el teji-
do del donante. A) Fernando Belaúnde Terry.

V) Pueden ser procedentes del mismo individuo, de B) José Luís Bustamantc y Rivero.
otro, e incluso de un animal. C) Manuel Prado Ugarteche.
D) Juan Velasco Alvarado.
A) Il-Ill-IV-V-I D) Il-V-I-IV-IlI
E) Augusto B. Leguía.
B) Il-V-IlI-IV-I E) I1-I-lII- V -IV
C) I1-I1I-IV-I-V 53. Una diferencia entre los gobiernos dictatoriales
de Augusto B. Leguía y de Alberto Fujimori, es que el
primero:
UN12003-11 APT. ACADÉMICA Y CULTURA GENERAL ~¡_~==~
A) Dio amplias facilidades al capital cxt .11,).
57. Señale la alternativa que contiene una afirmación
8) Fue reemplazado por una junta Militar. incorrecta.
C) Firmó tratados de límites con países vecinos, que
A) El pronombre designa a los seres sin nombrarlos.
fueron criticados por sectores de la población.
B) Son agudas las palabras cuya penúltima sílaba es
D) Promovió la corrupción en el Estado.
tónica.
E) Promulgó una nueva constitución.
C) El verbo en su modo imperativo expresa una or-
54. La Universidad Nacional del Ingeniería se fundó den, un ruego.
en 1876 como Escuela Especial de Construcciones Ci- D) Los pronombres relativos se acentúan con tilde
viles y de Minas del Perú, en momentos en que: cuando se usan para exclamar.

A) Los ingresos por la exportación de guano dismi- E) La sumilla es la síntesis o resumen del pedido cn
nuían inexorablemente. una solicitud.

8) Se necesita ingenieros para reconstruir el país 58. ¿Cuántos objetivos contiene el siguiente párrafo')
luego de la Guerra con chile.
"Esa bella muchacha esperaba resignada el final de aquella
C) El Perú había decidido construir el Ferrocarril del
tarde gris. Un negro crespón adornaba su hombro iz-
Centro.
quierdo y dos tímidas lágrimas rodaban por sus mejillas
D) En el país se desarrollaba el periodo político cono- rosadas".
cido como "segundo militarismo".
A) 6 B) 7 C)5 D) 8 E) 4
E) La Cerro de Paseo Mining Company iniciaba sus
actividades en la minería.
59. Señale la alternativa que cqrresponda:
55. Seiiale la alternativa que no corresponda: Fue un gran poeta latino (70 a 19 a.e), contribuyó a
revivir el espíritu nacional y a exaltar el trabajo del
Los defectos de dicción que atentan contra una buena
campo. Entre sus obras representativas figura:
lectura son:
Geórgicas, Bucólicas y la Eneida.
A) Omisión de acentos (maíz, nivea).
A) Plauto B) Horacio C) Virgilio
B) Supresión de letras (usté, mercao).
D) Terencio E) Ovidio
C) Cambio de letras (diabetis, tuavía)
D) Agregado de letras (dijistes, cónyugue)
60. ¿Quién es el autor de" Alturas de Machu Picchu" y
E) Omisión de mayúsculas (josé, el callao) del poema "Puedo escribir los versos .. ",!

56. Elija la correspondencia entre las especies líricas A) Alejandro Romualdo D) Martín Adarn
y sus respectivas definiciones. 8) César Vallejo E) Pablo Ncruda
l. La oda a. Breve poema que expone pensamientos C) Rubén Dario
satíricos.
2. La elegía b. Personajes pastoriles expresan el 61 .Señale la alternativa en que figuran dos poetas la-
sentimiento del poeta. tinoamericanos y dos europeos.
3. La égloga c. Expresa sentimientos amorosos en A) Dylan Thornan - V. Maiakovski - Antonio Ma-
forma breve y espontánea. chado - Grabiela Mistral.
4. El madrigal d. Canta la pérdida de un ser querido o 8) Saint John Pcarse - César Vallejo - Ernesto Carde-
algún dolor profundo. nal- Juana de Ibarbouru.
5. El epigrama e. Se ocupa de temas diversos como el C) Rafael Alberti - T.S. Elliot - Octavio Paz - J.L.
amor, la admiración por hechos notables o el Borges.
éxtasis religioso. D) Paul Eluard - André Brcton - BIas de Otero -
A) I e-2d-3b-4c-5a D) I a-2d-3e-4b-5c Pablo Neruda,
B) Id-2a-3e-4b-5c E) Ic-2e-3b-4d-5a E) Ezra Pound - Rainer M. Rilke - Nicolás Guillén -
Alejandro Romualdo.
C) I e-2b-3c-4a-5d
UN12003-11 APT. ACADÉMICA Y CULTURA GENERAL I;~~=;
62. La ciencia económica se preucupa fundamental- 2. Sinathropus 11. Chuku-Tien (Pekín-China)
mente de cómo: Pekinensis.
A) Hacer dinero. 3. Hombre de 111. Isla de Java
B) Tener éxito en los negocios. Neanderthal.
C) Evaluar a la empresa privada. 4. Hombre de IV. Dusseldorf (A lernan ia)
D) Gastar el dinero. Cro-Magnón.
E) Utilizar de la mejor manera los recursos de la
A) 1 [ ; 2 11 ; 3 IV ; 4 [11
sociedad.
B) 1 11 ; 2 I ; 3 111; 4 IV
63. ¿Qué sucede si el gobierno aumenta los salarios en C) I IV ; 2 1II ; 3 I ; 4 11
10% y simultáneamente el índice General de Precios
D) I III ; 2 1/ ; 3 IV ; 4 [
aumenta en 20%?
E) 1 [ ; 2 IV; 3 1/ ; 4 111
A) El salario nominal disminuye
B) El salario nominal sigue igual. 67. Los egipcios desarrollaban el tipo de escritura que
C) El salario real aumenta. se conoce como • mientras que en la
antigua Mesopotamia se tuvo un tipo de escritura que
D) El salario real disminuye Ilamamos_· _
E) El salario real sigue igual.
A) Jeroglífica - arábiga,
64. Es una causa directa de la disminución del poder de B) Arábiga - latina.
compra de la moneda: C) Jeroglífica - cuneiforme.
A) El Contrabando. D) Cuneiforme - simbólica
B) La deflacióu. E) Simbólica - elitista.
C) La Inflación.
68. Marque la proposición incorrecta cn relación a
D) La Recesión.
los aportes de la cultura árabe en la Edad Media:
E) La Corrupción.
A) Crearon el Álgebra.
B) Impulsaron el desarrollo de la química.
65. Marque la afirmación incorrecta:
C) Crearon la Farmacopea.
A) El Perú exporta principalmente materias primas
D) Emplearon la anestesia.
tradicionales.
E) Destacaron en la representación pictórica de figu-
B) El principal mercado de exportación del Perú es Ja-
ras humanas.
pón.
C) Estados Unidos es un importante mercado de ex- 69. A la capacidad de poder percibir características u
portación del Perú. propiedades comunes en objetos diferentes se denomina:
D) El Perú importa principalmente insumos para pro- A) Observación
ducir bienes finales .. B) Generalización
E) El porcentaje de las importaciones totales que re- C) Descripción
presenta la importación de bienes de consumo es D) Asociación de ideas
menor que el de la importación de insumos y bie-
E) Inducción
nes de capital.
70. ¿Cuál de las siguientes proposiciones es incorrecta?
66. Indique la alternativa que señale la relación co- A) La expresión "la luz solar es una fuente de energía"
rrecta entre los restos humanos y el lugar donde fueron corresponde a un juicio.
hallados tal como se indica a continuación:
B) Concepto es la representación mental de un objeto.
l. Pitecanthropus 1. Dordoña (Francia) C) Una oración está compuesta de sonidos organiza-
Erectus dos en palabras de acuerdo a ciertas reglas.
z.mf~~··§;;~~~~.1
Glriñez
UN12003-11 APT. ACADÉMICA Y CULTURA GENERAL I·~""
.

D) Las tareas del pensar son cl juzgar )' ~I Iazonar, C) El Collao y Bombón.
E) Para entender un lenguaje escrito es preciso mane- D) La de Cundinamarca.
jar un vocabulario. E) El Altiplano Peruano - Boliviano.

71 . Para que nuestras acciones sean morales es indis- 76. El Perú es uno de los países megadivcrsos del
pensable: mundo. Señale las afirmaciones correctas:
A) Dejar hacer, dejar pasar I. El Perú dispone de mas del 80% de las zonas
B) No confundir "lo que se puede" con "lo que se debe" biogeográficas existentes.
C) Hacer lo que se nos permite hacer Il. El Perú presenta su mayor diversidad étnica en la
D) Obtener siempre buenos resultados. sierra.
E) Obligar a que todos cumplan su deber. Hl. El Perú es el país con mayor número de especies
vegetales nativas domesticadas.
72. Completa la oración con la alternativa que corresponda: A) Sólo I B) Sólo 1I C) Sólo III
Un modelo de es un conjunto de característi- D) Sólo I y 1Il E) I , Il y III
cas o cualidades que debe reunir cierto tipo de perso-
nas, cosas, obras, o conductas para ser llamadas buenas. 77. El estudio de las características, composición y
A) Sociedad B) Personalidad C) Valoración distribución de los grupos humanos, se denomina:

D) Convivencia E) Justicia A) Etnografía B) Sicología C) Geografía


D) Sociografía E) Demografía
73. Señale la alternativa incorrecta:
Todo conocimiento científico, para ser considerado 78. Diga usted cuál es la denominación del premio que
como tal, tiene necesariamente que: la universidad de Alcalá de llenares ha entregado en su
versión del año 2002 al escritor colombiano Álvaro
A) Ser verificable.
Mutis, y que, anteriormente, entre otros grandes litera-
B) Expresar algo con independencia de la opinión de tos latinoamericanos, lo ha recibido Jorge Luis Borges.
quien lo formula.
A) Alfaguara D) Miguel de Cervantes
C) Ser expresado en un lenguaje.
B) Planeta E) Rórnulo Gallegos
D) Ser adquirido por contacto directo de los sentidos
con el mundo real. C) Príncipe de Asturias
E) Ser expresado como una proposición.
79. ¿Cuál de las siguientes afirmaciones sobre el fenó-
meno de "El Niño" es errónea?
74. Señale las afirmaciones correctas:
A) Es un fenómeno global.
1. En el Hemisferio Sur florecieron dos altas cultu-
B) Tiene Antecedentes remotos
ras: la Egipcia y la Andina.
C) Comprende los procesos vinculados a las sequías.
11. En el Hemisferio Sur sólo floreció una alta cultu-
ra: La Andina. D) Es constantemente monitoreado.

111. En el Hemisferio Sur florecieron dos altas cultu- E) Ha aparecido con el proceso de calentamiento del
ras: La Andina y la Azteca. globo terráqueo.

A) Sólo 11 B) Sólo I C) Sólo III 80. Seleccione el proyecto internacional quc se deba-
D) I Y 11 E) Ir Y III te en la actualidad para unir las cuencas del Atlántico y
el Pacífico.
75. En América del Sur hay extensas mesetas
intermontañas a más de 4.000 m. de altitud, y de A) La Modernización del Canal de Panamá.
piedemonte. Una de las alternativas no corresponde. B) El Canal de Beagle (Chile - Argentina)
¿Cuál es? C) La Vía Transarnazónica
A) Las punas de Atacama y Patagonia D) La carretera marginal (Brasil - Perú - Venezuela).
B) La Pampa Argentina. E) El "Interfe", Vía Férrea Internacional.
~'~==;i.¡¡j¡¡@,¡¡¡¡¡¡¡;¡¡¡¡¡¡ ¡¡@'tE,;IUNI2003-IIAPT.ACADÉMICAYCULTURAGENERAL

SOLUCIONARIO
ANÁLISIS DE FIGURAS
RAZONAMIENTO MATEMATICO 6. Las figuras completas van girando 90° en sentido
horario de un casillero a otro.
SUCESIONES NUMÉRICAS
1 • En la sucesión:

7 ; 13 ; 37 ; 145 ; 721
~.
17x2-l
l
(1) (2=?) (3)
'113x3-2{137x4-3! i145x5-4!
Clave: C
Clave: E
2. En la sucesión:
7. Para interpretar la sucesión
25 49 121 361 1225 gráfica a cada figura de la serie lo 1;'3"
¡ ¡ ¡ ¡ ¡ dividiremos en 3 rectángulos.
52 72 112 192 352
'-----"'"
<c::« '-----"'"'-----"'"
+2 +4 +8 +16
<c:> ~ <c:>
x2 x2 x2 • La línea recta diagonal va avanzando hacia abajo, de
Clave: D rectángulo en rectángulo, al llegar al rectángulo "3"
sigue en el "1 "; pero invirtiendo su inclinación.
3. En la sucesión:
• El cuadradito pequeño va avanzando hacia arriba, de
3 ; 12 ; 28 51; 81 rectángulo en rectángulo. Al llegar al rectángulo" 1"
<c:> <c.:;« <c.:»: '-----"'"
+9 +16 +23 +30 sigue en el "3".
<:.:» <c.::« <c:»:
+7 +7 +7 • El grupo de los circulitos negros van avanzado hacia
Clave: C arriba; pero al trasladarse de un rectángulo a otro
disminuye uno de ellos. Al llegar el rectángulo" 1"
4. En la sucesión:
sigue en el "3".
2 ; 3 ; 5 ; 9 17

12x2-J~\,5x2-1r~
Clave: A
5.
(1) (2) (3)
Clave: B

8. Para entender la secuencia gráfica numeramos a


cada cuadradito de color negro.
La suma de los números del casillero UN!:

S=112+117+113
=342

Clave: D (1) (4) (5 =?)


~~
GomeZ
. ",:1 UNI2003-11 APT. ACADÉMICA Y CULTURA GENERAL I~. ==::;:;~~
Cada cuadradito de color negro de un gráfico a otro, va
haciendo los siguientes movimientos.
• El cuadradito "1" se mantine siempre en su posición
• El cuadradito "2" se desplaza "un" casillero hacia la
izquierda. Si llega al extremo entonces continúa en
el extremo derecho.
• El cuadradito "3" se desplaza "dos" casilleros hacia
la izquierda. Si llega al extremo entonces continúa
en el extremo derecho.
• El cuadradito "4" se desplaza "dos" casilleros hacia
la derecha. Si llega al extremo entonces continúa
en el extremo izquierdo.
• El cuadradito "5" se desplaza "un" casillero hacia la Clave:C
derecha. Si llega al extremo entonces continúa en
el extremo izquierdo. 10. En la secuencia gráfica existen tres pequeñas figu-
ras: X; • ; ~.
9. La sucesión gráfica en general está compuesta de
una elipse dividida en 4 partes. X : Avanza un casillero hacia la izquierda de un grá-
fico a otro. Cuando llega al extremo continúa
por el casillero del extremo derecho.

~ : Avanza un casillero hacia abajo de un lugar a


otro, pero girando 90° en sentido horario. Cuan-
do llega al extremo entonces continúa por el
• En la parte superior izquierda siempre existe una de casillero del extremo superior.
las siguientes pequeñas figuras geométricas: triángu-
.: Avanza un casillero hacia la derecha y luego dos
lo, rombo y circulo. Estas pequeñas figuras tienen un
secuencia ordenada de pintado de una fila a otra. Al hacia abajo. Cuando llega a uno de sus extremos
pasar de una fila de arriba hacia abajo cada figurita se continúa cogiendo los casilleros del extremo
desplaza hacia la izquierda, y van cambiando su pin- opuesto.
tado en la forma siguiente:

••••
•• (3) (4) (5 =?)
Clave: C
• El achurado (rayado), está relacionado con el pintado
de las pequeñas figuras geométricas en la siguiente for- 11. La sucesión gráfica tiene 3 elementos que cambia
ma: de un gráfico a otro:

Cuando la pequeiia figura está pintada a la derecha el


1) La figura:
rayado "horizontal" se ubica también en la derecha;
pero en la parte superior.
Cuando la pequeiia figura está pintada a la izquierda el
rayado "horizontal" se ubica también en la parte iz-
quierda pero en la parte inferior.
Si la pequeña figura está pintada por completo en- Se le ha asignado un número a cada una de sus tres
tonces el rayado "vertical" se ubica en la parte opues- áreas simples. Esta figura gira 90° en sentido
ta a la figura (inferior derecha). antihorario de un gráfico a otro.
Il) El circulito blanco (O), va de un gráfico a otro, HUD = 50%(50% H)
alternado su posición entre el área "2" y"3". = (0,5)(0.5 H)
111) El pintado de color negro va trasladándose de un =0,25 H
gráfico a otro, siguiendo la secuencia de áreas ... (2)
1~2-73~l... Por condición del problema:
Cantidad de hinchas del Sporting Cristal : 2,

···El jJ (5) (6 = ?)
Clave: D
Cantidad
Cantidad
de hinchas de Wanka
de hinchas de Sport Boys

H,,'r,,;,
= 2x+x+2x
= 5x
:x
: 2x

Por dato la cantidad de hinchas del Wanka x = I 000, luego:

Hulru;, = 5(1 000)


12. Para que el cajero automático entregue 740 soles
con el mínimo de billetes con nominaciones de S/.I 00 ; =5000 ... (3)
S/.50 ; S/.20; y SI. I O , debe emplear la mayor cantidad Por condición del problema se deduce que:
de billetes de mayor nominación cuidando de que el
producto de este no exceda la suma total, o quede una HUD = Holru.\'
diferencia para los billetes de menor nominación in- De (2) Y (3): 0,25H = 5000
mediato inferior. Así H = 20000
6xS/.100 = S/.600 Clave: A
2xS/.50 =S/.IOO
IxS/.20· = SI. 20 14. Del enunciado:
2xS/.10 =S/. 20 Cantidad de patos: P= x
SI. 740 Cantidad de gallinas: G = 2x
Total de billetes = 6+2+1+2=11 Cantidad de conejos: C = x + 2x = 3x
Clave: A Por condición, si vende 5 patos y 10 gallinas el número
de Conejos es el doble del número de sus patos y gallinas
13. Dato: que le quedan:
Total hinchas del Wanka: Hw = I 000 C= 2[(P-5)+(G-10)]
Según el enunciado: 3x= 2[(x-5)+(2x-10)]
(50%) (50%) x= 10
Hr-------r------, La cantidad de conejos que existen:
Universitario
de Deportes C = 3x = 3(10) = 30
Alianza (50"10)
Lima Clave: 8
Otros
(50"10) 15. Del enunciado
Total de hinchas asistentes: H Vía libre:

Cantidad de hinchas de Alianza

HAL =50% H
=0,5 H
Lima

... (1)
---2v

Cantidad de hinchas de Universitario de Deportes: d


._:;;;~ ... '... '.:~... '..;.'.,! UN12003-11 APT. ACADE' MICA y CULTURA GENERAL 1''Wt''''~?K¡¡¡'Zi1tM'¡j¡::~'
'MflX:'*'UZ' .,,=., 105

Con trafico:
••.. w

~--~------------------------~ La cantidad de cuadras que caminó durante el mes:


1,2~ N = 5x = 5(20) = 100
~
Clave: E
;;:;§;. .z.;
17. Del enunciado:
d
Madre
Por condición:
3 horas 1 horas

-- --
A

1,2 v 2v
~

G§r: ~ ~
.1. b
I a
I Madre de Mauro
""-....

H.,m"~1
Con tráfico Vía libre
Por definición:
Distancia = Velocidad x Tiernpo
Para el automóvil:
Tramo a:
, Yo
! Hijo
a = 1,2vx3h <i
... ( 1)
= 3,6v(h) .•Mellizos •.
Tramo b:
~
b = 2vxlh Mauro
= 2v(h) ... (2)
Clave: O
Para el bus:
Tiempo = distancia 18_ Del enunciado:
velocidad
a+h 1nf (III) Inl' (11) Y (1)
v P¡: (V:F) P¡: (V) P¡: (V)
De (1) Y (2): 3,6v(h) + 2v(h) ~: (F)
~: (V;F) ~: (V;F)
v ~ p,,: (F)
P:J: (V;F) ~ P:J: (V;F)
= 5,6(h)
p,¡: (V;F) p,¡: (V;F) p,¡: (V)
Clave: E Ps: (F)
Ps: (V;F) Ps: (F)
16. Datos: Clave: B
Total de veces que viaja :30 19. Del enunciado:
Cantidad de veces que camina 5 cuadras :x "Carlos y el soldador juegan a menudo bingo con el que
Cantidad dc veces que no camina 5 cuadras :30 - x usa el uniforme rojo quien no es amigo dc Alfrcdo. El
Costo del pasaje cuando no camina : SI. 1,20 electricista usa el uniforme blanco". Luego:
Costo del pasaje cuando camina 5 cuadras : SI. 0,80 Tres personas que se encuentran jugando:

En total al mes ha pagado 28 soles en pasajes. <Carlos -Soldador -Rojo


El de uniforme"rojo" no es amigo de Alfredo:
(30-x)(1,2)+x(0,8) = 28
x= 20 -Cartos -Soldador -Rojo
Alfredo Braulio
~~I
~WWWWt©? UN12003-11 APT, ACADEMICA ' y CULTURA GENERAL

El electricista usa uniforme "blanco": Nota: Verificando total de postulantes y puestos por
cubrir:
• Carlos .Soldador • Rojo
7 puestos con 2 postulantcs 7x2 = 14
Electricista Alfredo Braulio
Blanco Amarillo Carpintero ,6 puestos con 3 postulantes óx3 = IR

Clave: E De (IlI): 4 puestos con 1 postulante 4xl= 4


Total = 36
SUFICIENCIA DE DATOS
20. Del enunciado: OPERADORES MATEMÁTICOS
Edad del hermano mayor : x + 10
22. De los operadores:
Edad del hermano intermedio: x+7
Edad del hermano menor : x ~= N-l
Por la información (1): En la fórmula:
x+ 7 = 11

iSe definen las tres edades!


Por la información (II):
x=4
R = A-~
=:::::::::::::::::=-==-

IN-II
I
(x+ 10)+ (x+ 7)+ x = 29
x=4
~- (N-l)
iSe definen las tres edades!
(N-I)-I
Entonces, "Cada una de las informaciones por separado
es suficiente"
( N - 1)2 - ( N - 1)
Clave: O
N-2
N2-3N+2
21. Datos: N=5
N-2
Total de puestos a cubrir : P = 17 52-3(5)+ 2
Total de postulantes (trabajadores) : T= 36
5-2
N° de postulantes por puesto : n =4
Se afirma que se presentan a 13 puestos mas de un Clave: C
postulante:
n>1 => n=2;3;4; ... ... (1) 23. El operador Ii definido así:

Entonces para los 4 puestos restantes de las 17: Ii(N) = 10 N>I ... (1)

n~1 => n=O;I; ... ... (2) Ii(N) =O N~I ... (2)
Por la información (1) para los 13 puestos, en (I)
En la expresión:
11 = 2;3 ; ... ... (3)
R = Ii[ 1i(30~-Z)]
(3)
Además podemos afirmar la cantidad de puestos "x"
con 2 postulantes y la cantidad "y" de puestos con 3
postulantes. ZE [1; 299] . (4)
x+y=13 ... (4)
• Si: (300-z) >l
Por la información (11), tenemos x = 7 , entonces en (4):
=> 300-1> z
(z)+y=13 Z < 299 ... (5)
y=6
Clave: O => 1i(300-z)=10
En (3):
••••¡~~I UN12003-11 APT. ACADÉMICA Y CULTURA GENERAL

24. En el operador:
=~=-~
R ~[~(30~-Z)]
=
P(n-I)=112_1

=~[I~] ... (6) Calculo de la 1(,,) :

pea) = p((a+I)-I] = (a+ 1)2_1


Teniendo en cuenta (5): z < 299
=a2 +2a+I-1
Si: 10S;z<299
= a2 +2a ... (1)
~ !Q.S; 1
z Calculo de 1(3)
En (6):
P(3) = P(4-1) = 42_1
= 15 ... (2)
(1 valor) Finalmente; de (i) y (2):

P(a)+ P(3) = a2 +2(/+ 15


Si: 1 S; z S; 9 ~ !Q> 1 Clave: A
z
En (6):
2'5. En los Operadores:
R2 = ~[I~J [xbJ =h xb-I ... (1)
=10 (1 valor)
... (2)
.Si: (300-z) S;1
Operando en (1):
~ 300-IS;z
z ~ 299 f(x+I)=[x2J+3[x3J+ f(x)
Por (4): z = 299 ... (7)
2
De (1): = 2x+ 3(3x )+ f(x)
~(300 - z) = ~(300 - 299)
= 2x+9x2 + f(x) ... (3)
= ~(I)
::::) =O (Repetido) Por Dato: f(4) = 2

f(x+I)=f(3+1) ~ x=3
En (3):
En (3):
R ~[~(30~-Z)]
=
f(3 + 1)= 2(3)+ 9(32)+ f(3)
2 = 87+ f(3)
=~[2~9J ::::) f(3) = -85 ... (4)
=O .(Repetido)
Aplicando (3):
Finalmente podemos afirmar que R puede asumir 2 valores. f(2 + 1)= 2(2)+ 9(21)+ f(2)

Clave:B De (4): -85=4+36+f(2)

~ f(2)=-125
Clave: E
$>:;;i:m:r~:1 UNI2003-11 APT. ACADÉMICA Y CULTURA GENERAL ';;:::':::;:1::7--===-- z
'~~~

RAZONAMIENTO VERBAL 32. Los conecto res a insertarse son:


"Este formulario permite ahorrar tiempo y evita hacer
TERMINO EXCLUIDO
gastos adicionales; pero no debemos escribir innecesa-
26. Las palabras que guardan relación de significado riamente, sino anotar sólo lo indispensable.
común son: Prototipo, modelo, arquetipo y muestra,
Clave: C
su significado es algo definido que sirve de ejemplo. En
cambio la palabra bosquejo significa algo previo no 33. Los conectores que deben insertarse en el enuncia-
bien definido. do.
Clave: B "Los viejos deben temblar ante los niños; porque la
27. Los términos o palabras que guardan una realción de generación que se levanta es siempre acusadora y juez
significado común son: sojuzgar, someter, avasallar y opri- de la generación que desciende".
mir, su significado de estas alude a privar de la libertad, Clave: B
La palabra invadir significa tomar algo sin el consen- 34. Los conectores a insertarse son:
timiento de su propietario. . "Cuando el uso de la fuerza se hace necesario e inevita-
Clave: E ble, la civilización exige, por un lado, ante el triunfo,
se muestre la ponderación y decoro; y, por otro lado,
28. Guardan una relación de significado común: supe-
frente a la derrota, resignación y esperanza.
rávit, renta, ganancia y beneficio, estos no dan la idea
"de un resultado positivos de una actividad comercial. Clave: A
La palabra déficit significa un resultado negativo o ORACIONES ELIMINADAS
pérdida de un operación comercial.
Clave: D 35. El tema central del texto es los objetivos y méto-
dos de estudio de la Psicología.
29. Los términos:
La oración que no tiene relación con este tema es la 11,
homicidio : matar a una persona
fratricidio : matar a un hermano que trata los tipos de caracteres que definen el modo del
magnicidio: matar a un magnate comportamiento de una persona.
parricidio : matar a un padre Clave: B
Tienen un significado común, que es el matar a otra 36. El tema central es: La evangelización cristiana
persona. En cambio la palabra suicidio significa ma-
tarse a si mismo. por parte de España en las tierras conquistadas. La ora-
Clave: A ción 111trata sobre la importancia de la expedición de
Colón para Europa.
CONECTORES Clave: C
30. Los conecto res que deben insertarse en el enunciado. 37. El tema: La restauración, sus objetivos técnicos.
"Si has omitido letras o algunas palabras, o no has La oración de la alternativa V se refiere a la propiedad
respetado los signos de puntuación, repítela hasta que de los objetos históricos.
logres una lectura fluida". Clave:
"Si" es una conjunción condicional, la primera y segunda
"o" son disyuntivas y "hasta que" es una preposición. 38. El tema: como se presenta el fenómeno geiser.
Clave: E La oración V trata sobre las propiedades curativas del
31. Los conecto res que deben insertarse: fenómeno geiser.
Clave: E
"A pesar de que en el mundo científico la casualidad
rara vez es reconocida como un factor que contribuye 39. El tema: la historia de lingotes de oro y plata halla-
a los descubrimientos importantes; no obstante, en la dos en los escombros de las torres gemelas de Nueva York.
realidad existen notables excepciones.
La oración III se ocupa de explicar como se elaboran
"A pesar de que" es un conector adversativo restricti- los lingotes de oro.
vo, "como" es un couector comparativo y " no obstan- Clave:C
te" es un adversativo.
Clave: D
~·~~~·~.;::::::::::[~U~N~12~O~O~3-~II~A~P~T~.A!C~A~D~ÉM~IC~A~Y~C~U~L~T~U~RA~G~E~N~E~RA~L~~;::::::2·~
Gom~ ~

IH) También puede presentarse en otras partes.


COMPRENSiÓN DE LECTURA l l) Por los vasos sanguíneos circula la sangre.
40. Del contenido del texto se concluye que: Clave: O

Para que una hipótesis se convierta en una proposición 47. La secuencia correcta de las oraciones del tema:
científica, primero debe comprobarse su validez empíri-
ca, es decir comprobar su veracidad con la realidad. MOTORES Y CENTRALES HIDROELECTRICAS:

Clave: O IV) Fuente de energía para las plantas hidroeléctricas.


1) La planta hidroeléctrica y las turbinas que necesita.
41. Del contenido del texto se concluye:
IH) La turbina Kaplan por ejemplo se usa cuando el
El líder es el que capacita y promueve para que la gente tiene poca altura.
piense y haga por sí misma, es decir contr ibuye a que
V) La turbina Francis, en cambio se usa en las usinas,
los demás resuelva ellos mismos sus problemas.
11) La turbina Pelton, por último sólo puede usarse en
Clave: B las usinas de alta presión.
42. De acuerdo al texto: Galileo hace grandes descu- Clave: A
brimientos con respecto a los movimientos de la tierra
con respecto al sol, renuncia a su teoría por presión de 48. La secuencia correcta del cubismo:
la inquisición; pero sin embargo desarrolló el méto-
V) Definición del cubismo, como una corriente artís-
do experimental del cual depende la ciencia moderna.
tica importante en el siglo xx.
Clave: E 1) Trascedencia del cubismo sobre otros esti los.
11) Forma de representar los objetos
ORACIONES INCOMPLETAS
IV) La forma de representar los objetos (tosquedad)
43. La oración indica que una persona queda impresio-
chocó al público.
nado al observar desde lo alto de una colina.
111) Consecuencias, generó otros estilos.
"Quedó completamente maravillado con la imagen que
Clave: O
se presentaba ante sus ojos, desde lo alto de aquella coli-
na, podía contemplar la llanura en toda su vastedad". 49. La secuencia correcta sobre el trasplante:
Clave: O H) Definición de trasplante.
44. "Se puede tolerar, inclusive perdonar a una perso- V) Procedencia de lo que se va a transplantar
na que no sabe lo que hace". 11I) Problemas que se presentan, la intervención en si
y el rechazo.
"Es posible tolerar y aún perdonar al ignorante; pero
al cínico, jamás". IV) Como evitar el rechazo.
Clave: 1) Un ejemplo de transplante frecuente.
Clave: B
45. Una persona no puede dirigir bien a otras perso-
nas, si no puede dirigir bien su propia vida. 50. La secuencia correcta sobre Evolución del dibujo
"¿Cómo podré gobernar a los demás si no tengo el V) Las técnicas de dibujo a carbón son las mas antiguas.
dominio y control de mi mismo?" 1) El uso posterior del pincel necesita de una su-
Clave: E perficie especial.
III) El uso del pincel se debió al descubrimiento del
papiro y el pergamino.
COHERENCIA DE REDACCiÓN
IV) Por ejemplo en la Edad Media se lograron elabo-
46. LAS VÁRICES. rar superficies para el uso del pincel.
1}Definición de várices, dilatación de venas 11) Mas adelante aun los artistas que realizan sus dibu-
V) La dilatación se produce en los vasos sanguíneos. jos con tinta china usaban el pincel.
IV) Generalmente se presenta en las piernas. Clave: A
eZ

4. El Madrigal: e: Expresa sentimientos amorosos


en forma breve y espontánea.
CULTURA GENERAL
5. El Epigrama: a: Son poemas breves y satíricas
51. El descubrimiento de la tumbas reales del Señor de
Clave: A
Sipan, ha permitido desarrollar estudios sobre el
universo ideológico de la cultura preinca Moche. El
57. La afirmación incorrecta es la alternativa B, pues
hallazgo hecho en 1987 por un grupo de arqueólogos
son agudas las palabras cuya última sílaba es tónica. En
dirigidos por Walter Alva y Luís Chero en la I-Iuaca
. la alternativa se afirma que está sílaba tónica (con tilde)
Rajada de Lambayeque, puso en evidencia la forma de
es la penúltima.
vida que llevaban y como concebían la existencia des-
Clave: B
pués de la muerte.
Clave: A 58. Los Adjetivos (calificativos) que contiene el pá-
rrafo: "Esa bella muchacha esperaba resignada el fi-
52. En el Perú en el gobierno dictatorial de Juan nal de aquella tarde gris. Un negro crespón adornaba
Velasco Alvarado se impulsó un vasto proceso de re-
su hombro izquierdo y dos tímidas lágrimas rodaban
formas estructurales y nacionalización de empresas ex-
por sus mejillas rosadas".
tranjeras. Este hecho sucedió en I 968 a I 975, implan-
tándose la reforma agraria, industrial y educativa. En total son 7 adjetivos calificativos.
Clave: B
Clave: D
59. Virgilio Maron, poeta latino (70 -19 a.e) autor de:
53. La diferencia entre gobiernos dictatoriales de
"Las Bucólicas", serie de diez eglogas de composición
Augusto B. Leguía y de Alberto Fujimori, es que el
pastoríl.
primero fue reemplazado por una Junta Militar y
el segundo se dio a la fuga a Japón por presión social. "Las Geórgicas", admirable poema a la gloria del campo.

Clave: B "La Eneida", gran epopeya nacional y religiosa que


dejó sin terminar.
54. La Universidad Nacional de Ingeniería se fundó en Clave: C
I 876 como Escuela especial de Construcciones Civiles
y de Minas del Perú, en momentos en que los ingresos 60. Pablo Neruda, poeta chileno, entre sus obras:
por la exportación de guano disminuían inexorable-
"Canto General", que entre sus poemas que lo compo-
mente. Fue creada durante el gobierno de Manuel Pardo
nen está "Alturas de Machupicchu".
y Lavalle con el objetivo de desarrollar otras activida-
des que sustituyan la explotación del guano: "Veinte poemas de amor y una canción desesperada
donde expresa: "Puedo escribir los versos ...
Clave: B
Clave: E
55. La alternativa B es la que no afecta a la buena 61. Analizando las alternativas donde figuran dos poe-
lectura, pues está corresponde al campo de la ortología, tas latinoamericanos y dos europeos:
que se encarga de la buena pronunciación.
A) Dylan Thomas : Gales
Clave: B Vladimir Maiakovski : Rusia
Antonio Machado : España
56. La correspondencia entre las especies líricas y sus
Gabriela Mistral : Chile
respectivas definiciones son:
B) Saint John Pearse : Francia
1. La Oda: e: Se ocupa de temas diversos como el
César Vallejo : Perú
amor, la admiración por hechos notables o el
Ernesto Cardenal : Nicaragua
éxtasis religioso.
Juana Ibarbouru : Argentina
2. La Elegia: d: Canta la pérdida de un ser amado o
algún dolor. C) Rafael Alberti : España
Thomas S. Elliot 1nglaterra
3. La Égloga: b: Para su composición usa personajes
Octavio Paz Méxíco
pastoriles y expresan los sentimientos del poeta.
Jorge L. Borges Argentina
~:I UN12003-11 APT. ACADÉMICA Y CULTURA GENERAL I~ .~
D) Paui Elvard : Francia B) FI salario nominal sigue igual. FALSO
André Breton : Francia C) El salario real aumenta. FALSO
Bias de Otero : España
De (4): RlI <R
Pablo Neruda : Chile
E) Ezra Pound : Estados Unidos D) El salario Real disminuye. CORRECTO
Rainer M. Rilke : Alemania
RlI=0.917 R
Nicolás Guillén : Cuba
Alejandro Romualdo : Perú E) El salario Real sigue igual. FALSO
Clave: C Cláve: D

62. La ciencia económica se preocupa fundamental- 64. La Inflación es una causa directa de la disminu-
mente de como utilizar de la mejor manera los ción del poder de compra de la moneda. Proviene del
recursos de la sociedad. Aumento del papel moneda.
Clave: E
Clave: C
63. Por definición el Salario Real mide la cantidad que
65. Según fuentes oficiales la Balanza Comercial del
se pueden comprar con el Salario Nominal, dándose un
Perú en el a110 2002.
nivel de precios o lndice General de Precios si definimos:
Total Exportaciones 7687 (:'\1ill(\II';~ \1.: dul;U\.'s)
Salario Real :R
Salario Nominal :N Tradicionales (materias primas) 5291
Indice Geral de Precios: [ No Tradicionales 2272
Estos tres elementos guardan relación:
Otros 24
R=N ... (1)
[ Total importaciones 7626 (\lilluu.:s lit: Ih.l.,r.:¡.,)

Por condición del enunciado: El gobierno aumenta los Bienes de consumo 1741
salarios (sueldo nominal) en un 10%, es decir: lnsumos 3809
Na = N + 10% de N = 1.I N ... (2) Bienes de Capital 1931

y también aumenta simultaneamente el In dice General Otros s


de Precios, es decir:
También se sabe que los principales países que exporta-
la = 1+20% de l = 1.2 [ ... (3) mos el Perú son:
Luego, el nuevo Salario Real sera: EE.UU de Norteamérica : 25 %
Na Reino Unido : 11.5%
De (1): Ra=¡ China : 7.8%
a
Suiza : 7,4%
!,IN
.De (2) Y (3): Japón : 4,9%
1.21

=0.917(~)
De la información dada, se deduce que la alternativa B
De (1): =0.917 R es incorrecta, pues Japón no es el principal mercado de
exportación, lo es EE.UU de Norteamérica."
Entonces se puede afirmar que:
Clave: 8
n, <R ... (4)
66. La relación correcta entre los restos humanos y el
Con la información obtenida podemos afirmar que: lugar donde fueron hallados:
A) el salario nominal disminuye. FALSO 1. PiteCanthropus Ercctus . 111: Isla de Java
De (3): 1" > l 2. Sinathropus Pekincnsis : 11:Chucu-Ticn (Pekín - China)
~~:":~;I UN12003-11 APT. ACADÉMICA Y CULTURA GENERAL

3, Hombre de Neanderthal : IV : Dusseldorf (Alemania) yen el Hemisferio Sur solo se desarrolla la Cultura
4, Hombre de Cro-Magnón: l: Dordoiia (Francia)' Andina.

Clave: O De lo anterior podemos deducir que: I es incorrcc'c, 1I


es correcto y 111es incorrecto.
67 _Los Egipcios desarrollaron el tipo de escritura que Clave: A
se conoce como jeroglífica, mientras en la antigua
Mesopotamia se tuvo un tipo de escntura que llamamos 75. En América del Sur hay extensas mesetas a más
cuneiforme, que creada por el pueblo sumerino usando de 4 000 m de altitud, que se encuentran entre monta-
tablillas de arcilla, Los Egipcios primero hicieron sus ñas al pie de ellas. Entre estas tenemos: Las punas de
escrituras en piedra, posteriormente en papiros. Atacama, Patagonia, el Callao, el Bombón, la de
Cundinamarca, el Altiplano Peruano-Boliviano,
Clave: C
La Pampa de Argentina (8) no es una mcseta
68. La cultura en la Edad Media aportaron con cono- intermontañosa y además se encuentra a un promedio
cimientos de Matemática (Algebra). Química (Alqui- de I 000 m.s.n.m. Se considera una llanura,
mia), medicina (farrnacopia), y anestesia y arquitectura Clave: B
entre otro. Por su religión, el Islam, se prohibía la
representación pictórica de figuras humanas. 76. El Perú es un país megadiverso porque:
Clave: E T) Correcta: Dispone de mas del 80'Vo de las zonas
biogeográficas existentes,
69. A la capacidad de poder percibir características o [1) Incorrecta: Presenta la mayor diversidad étnica
propiedades comunes en objetos diferentes se denomi- cn la Sierra, Pues no es cierto porque el Perú pre-
na generalización,
Clave: B
senta la mayor diversidad étnica en la Selva con 'f'
algo 60 diferentes grupos linguísticos.
70. La Proposición incorrecta corresponde a la alter- Ill) Correcta: Es el país con mayor número dc cspc-
nativa B. Pues un concepto es una representación ge- cies vegetales nativas domesticadas, '
nérica y precisa de un objeto o tema, Clave: D
Clave: B
77. La Demografía estudia las características compo-
71 . Para que nuestras acciones sean morales es indis- sición y distribución de los grupos humanos, General-
pensable "hacer lo que se nos permite hacer", Las mente usa los censos para obtener los datos estadísticos,
acciones morales están sujetas a la buenas costumbres y Clave: E
reglas de la sociedad,
Clave: C 78. El premio que la Universidad de Alcalá de Henarcs
ha entregado en su versión del año 2002 al escritor
72. Un modelo de valoración es un conjunto de ca- Colombiano Alvaro Mutis, tiene denominación "Mi-
racterísticas o cualidades que debe reunir cierto tipo de guel de Cervantes". Anteriormente también se le
personas, cosas, obras o conductas para ser llamadas entregó a Jorge Luís Borges, entre otros,
buenas, Clave: D
Clave: C
79. El fenómeno de "El Niño" no ha aparecido con el
73. Todo conocimiento científico, para ser conside- proceso de calentamiento del globo terráqueo, pues
rado como tal, no tiene necesariamente que ser ad- este siempre ha existido desde tiempos anteriores al
quirido por contacto directo de los sentidos con fenómeno de "Efectos invernadero",
el mundo real. El conocimiento muchas veces se ad- Clave: E
quiere a base de conocimientos abstractos, por ejemplo
la matemática, la lógica. 80. El proyecto internacional que se debate en la ac-
tualidad para unir las cuencas del Atlántico y el Pacífico
Clave: O
es la vía transamazóniea (Río Branco-Iñapari-Ilo)
74. Las culturas que se desarrollan en el Hemisferio Clave: C
Norte son: La Azteca, Maya, Egipcia, Griega y etc.;
3
APTITUD ACADÉMICA de 2.5 111 por hora. Si a las 6 am el tanque está lleno
hasta la mitad ¿A. qué hora el tanque no tendrá agua
RAZONAMIENTO MATEMÁTICO almacenada?
1. Una unidad de un producto, requiere para su fabrica- A) 7 atn. B) 8 ani. C) 9 ani.
ción el empleo de dos factores o insumos en forma simul-
tánea y complementaria. Del primer factor necesita 2 uni- D)10 amo E) 11 a ni.
dades y del segundo factor 3 unidades. ¿Qué cantidad de
6. Si un motociclista viaja a velocidad promedio de 40
unidades del producto se podrán fabricar con 6 unidades
kni/l: ¿Cuántas horas empleará para recorrer" L " kui,
del primer factor y 12 unidades del segundo factor?
si hace" a " paradas de " b " minutos cada una?
A) 1 B)2 C)3 D)4 E) 5
A) L+40ab B) 3L + 2a/¡ C) L +ab
2. Un ómnibus que se dirigía de Lima a Arequipa co- 40 120 4()
bró por pasaje único S/.60 y se observó que cada vez
3L + ab E) 30L + 2ah
que bajaba una persona subían tres. Si dicho ómnibus D) -6-0-
120
llegó a Arequipa con 50 pasajeros y una recaudación de
S/.4 392 incluido el seguro de S/.l ,00 por cada pasajero
¿Cuántas personas partieron del paradero inicial? RAZONAMIENTO LÓGICO
A)6 B) 12 C)14 D) 15 E)18 7. Si todos los no creyentes son apostadores y ningún
alpinista es creyente, entonces:
3. Cuando Pepito pasea junto con su mamá, se observa
A) Todos los no creyentes son alpinistas.
que cuando ella da un paso Pepito da tres. Si la mamá de
B) Ningún alpinista es apostador.
Pepito se demora 2 minutos en recorrer una cuadra y
C) Algunos alpinistas no son apostadores.
considerando que la rapidez es la misma para ambos
¿Cuántos minutos demora Pepito en recorrer dos cuadras? D) Todos los alpinistas son apostadores.
E) Todos los no creyentes no son apostadores.
A)2 B)4 C)6 D)8 E) 10
8. Un granjero tenía cuatro perros: Rocky, Laica, Pin-
4. Un empresario ha observado que al procesar la me- to y Manchas. Rocky es el mayor de todos, hace más
nestra (precocer) el peso se duplica. El precio de com- de un año la perrita Laica, que tenía más dc cinco años
pra de la menestra sin procesar es de 1 500 soles por falleció al parir a Pinto y Manchas.
tonelada métrica. El precio de venta por kilogramo de
Al preguntarle por la edad de sus perros, dijo que no
la menestra procesada es de 3 soles
recordaba bien, pero su hijo intervino diciendo "actual-
El empresario prevee un gasto de producción y venta de mente el producto de las edades (en años) de los tres
50% del costo de la menestra sin procesar ¿Cuál será la perros es 36" ¿Cuál es la edad de Rocky, en años?
posible utilidad por tonelada métrica que vende?
A)4 B) 6 C)9 D) 12 E) 18
A) 1875 B) 2 250 C) 3 750
D) 5 000 E) 6 000 9. Un cuadrado, muestra dentro de sí regiones cuadra-
das sombreadas, de acuerdo a la posición que ocupa en
5. Un tanque de agua tiene la capacidad de almacenar la sucesión.
4 /113 ,el agua ingresa al tanque por una tubería a razón Determine la cantidad de cuadrados sornbrcados en la
de 2 m3 por hora, y el consumo es aproximadamente posición 25.
~i:==~;1 UN12004-1 APT. ACADÉMICA Y CULTURA GENERAL I;i~;:r:~;l::::;;::;:;:;:
~=~

A) B) C) D) E)

D
Posición 1 Posición 2 Posición 3
1 5. Indique la alternativa que completa la serie mostrada.

A) 500 B) 560 C) 580 D) 600

10. Seis amigos se ubican alrededor de 'una mesa cir-


E) 610
J7~LfM~?
cular. Malchi no está sentado aliado de Pina ni de Lito.
Zory no está aliado de Yalú ni de Lito. Pina no está al
lado de Yalú ni de Zory. Pepe está junto y a su derecha
de Pina. ¿Quién está sentado a la izquierda de Zory?
'pJ!JbJsfr!r
A) B) C) D) E)
A) Yalú B) Lito C) Pepe D) Pina E) Malchi
SUCESIONES NUMÉRICAS
RAZONAMIENTO ABSTRACTO
16. Seiialar la alternativa que continúa correctamen-
11. Indique la alternativa que completa la serie mostrada. te la siguiente secuencia.
4, 9, 20, 43, 90, 185, 376,?
A) 864 B) 487 C) 542 D) 759 E) I 005

17. Determine el número que completa la serie.


4, 9, 26, 106, 528, 3 171, '!
A) 22 194 B) 23 236 C) 25 6R6
D) 27 942 E) 28 642
12. Indique la alternativa que continúa en la serie:
18. Señale el número que falta en el casi lIero para
completar la serie.

A) 64 B) 144 C) 128 D) 256 El32

19. Indique la alternativa que continúa adecuadamente


A) B) C) D) E) la siguiente serie numérica.
3, 6, 4, 8, 6, 12, 10, 20, 18, 36, 34, ,)
13. Indique la alternativa que continúa en la serie.
A) 32 B) 48 C) 56 D) 6R E) I 12

SUFICIENCIA DE DATOS
20. ¿Cuánto vale Z?
Información brindada:
5
1. La relación entre W y Z es 3

14. Indique la figura que debe ocupar el casillero UN!. 11. El exceso de W sobre Z es 12.
Para responder la pregunta:
UNT A) La información I es suficiente.
B) La información II es suficiente.
~ me:WW/ill7itill%W;r©;nCWMU UNI 2004-1 APT. ACADÉMICA Y CULTURA GENERAL ::=:,~=:~
C) Es necesario emplear ambas informaciones. Determine el número de valores que puede tomar R,
D) Cada una de las informaciones, por separado, es
para :: E {40, ... ,60}; z es un número entero.
suficiente.
E) Falta información. R=~(SO-z)
z
21 . Para pintar las aulas del pabellón A, se escogió el co- A)2 B) 8 C) 10 D) II E) 9
lor, calidad y rendimiento de la pintura ( 2 O m 2 por galón).
25. Se define el operador W(N) , tal que:
Además se tiene acceso a la siguiente información:
Si: N e [0;9] 4 W(N) = O
1. Todas las aulas son idénticas y tienen una super-
Ne ]9;99] 4 W(N)=I
ficie para pintado de 140 1112 .
Ne ]99;999] 4 W(N)=2
11. Cada galón cuesta S/.48
Hallar el valor de R.
Si se requiere conocer el monto total de la inversión
que implica esta actividad, es necesario: WC34S) + W(SO) - W(S)
R
W(70) + W(22)
A) Sólo la información 1
B) Sólo la información 11. A) O,S B)!,O C)!,S D) 2,0 E) 3,0
C) Ambas informaciones a la vez.
D) Cada una de las informaciones por separado.
RAZONAMIENTO VERBAL
E) Se requiere mayor información.
TÉRMINO EXCLUIDO
22. En la figura el cuadrado está inscrito en la cir-
cunferencia. Identifique el término que no guarda relación de signi-
ficado común con los otros y con el término base,
26. FUERZA
INFORMACIÓN:
A) Energía B) Potencia' C) Fortaleza
l. El área del círculo.
D) Eficacia E) Firmeza
11. El área del cuadrado.
lll. La diagonal del cuadrado. 27. ATAVío
Para determinar el área de la región sornbreada, se re- A) Atuendo B) Traje C) Ropa
quiere: D) Prenda E) Equipaje
A) Sólo la información I y 11.
B) Sólo la información 1.
28. Marque la opción que no corresponde al conjunto.

C) Sólo la información Il. A) Sol 8) Relámpago C) Linterna


D) Sólo la información lII. D) Rayo E) Luna
E) Cualquiera de ellas.
29. Elija la palabra que debe ser excluida por no guar-
dar relación de significado con las demás.
OPERADORES MATEMÁTICOS
A) Trivialidad B) Volubilidad Cjf'ruslcria
23. Sean los operadores F(Il) = 1,8/1+ 32 ; D) Nadería E) Chuchería

G(11 - 1) = /13 - 2 ; hallar la relación: ~g~ ANALOGíAS


A) 0,603 B) 1,38S C) 1,496 30. INTANGIBLE PERCEPT1I3LE
D) S,933 E) 6,233
A) Irreal Propio

24. Se define el operador t., tal que: B) Impropio Tangible


C) Inmaterial Concreto
Si: N>I 4 ~(N)=IO
D) Intrascendente Irreal
Si: N$.! 4 ~(Nl=O E) Impoluto Consecuente
31. REBUZNO ASNO 36. realizó una excelente exposición,
A) Mugido Toro ____ el análisis de los datos no satisfizo a los
asistentes; el jurado le recomendó ser más
B) Crujido Rana
exhaustivo en futuras presentaciones.
C) Bramido Oca
A) Si bien - pero - y
D) Quejido Persona
B) No bien - y - pues
E) Maullido Felino
C) Pese a que - sin embargo' - por eso
32. ESCRITORIO ESCRITOR D) Si - no obstante - aunque
A) Colegio Profesor E) Porque - pero - y
B) Instrumento Músico
37. " había nacido en Génova y, ,
C) Pincel Decorador era italiano, , engrandeció a España".
D) Taller Mecánico
A) A pesar de que - por eso - además
E) Caballete Pintor B) En realidad - efectivamente - aunque
C) Porque - por ello - no obstante
ORACIONES INCOMPLETAS D) Aunque - por tanto - sin embargo
Indique la alternativa que completa adecuadamente el E) En primer lugar - entonces - luego
significado de la siguiente oración.
38. La Revolución Industrial rompía con
33. "Varios alumnos, pequeños como él, se le lo anterior, necesitaba una base social y po-"
____ y Paco, cada vez más , se pegó a lítica. Gran Bretaña la lidcraria .
la pared, y se puso. _ A) dado que - sin embargo - En efecto
A) Abalanzaron - alegre - morado B) ya que - además - También
B) Acercaron - tímido - colorado C) en tanto - incluso - Naturalmente
C) Alejaron - contento - a gritar D) dado que - también - Por esta razón
D) Encaramaron - valiente - a llorar E) no obstante - también - Dado que
E) Amontonaron - nervioso - sereno
39. "Albert Einstein aseguraba que la imaginación es
34. "Es un catedrático moderno que propone méto- más poderosa que la razón. , debemos es-
dos de enseñanza". tar prestos a desarrollar y optimizar nuestra creatividad".
A) Didácticos B) Eficaces C) lnnovadores A) Sin embargo B) No obstante C) Por tanto
D) Prácticos E) Seguros D) Además El Por supuesto

35. "Era un hombre ' intolerante y terco que


ELIMINACiÓN DE ORACIONES
no hacía concesiones; por ello, con el pasar de los a110s,
fue cayendo en un cierto que 'terminó por Elija la oración que es redundante o no tiene relación
llevarlo a vivir en _ con el tema del texto.
A) inocuo - desamparo - retiro 40.
B) conspicuo - silencio - aislamiento 1) En una pared de nuestra sala principal hay un pla-
C) iracundo - desfalco - periferia ca con una inscripción en latín que, traducida, dice
más o menos: "Los demás tampoco saben".
D) afable - retiro - alejamiento
E) intransigente - aislamiento - soledad ll) Puede parecer extraño que este tipo de ideología
aparezca en un recinto universitario, pero es apro-
piada. El sentimiento de no saber puede provocar
CONECTORES LÓGICO - TEXTUALES mucha ansiedad.
Marque la alternativa que, al insertarse en los espacios lll) Nuestro lema nos recuerda que hay vulnerabili-
en blanco, de un significado coherente y correcto al dad y aprendizaje. A veces para aprender hay que
enunciado. reconocer cuánto no se sabe.

"
UNI 2004-1 APT. ACADÉMICA Y CULTURA GENERAL =:::::::.~
IV) Cuando los estudiantes vienen a ver me yo tengo COHERENCIA Y COHESiÓN TEXTUAL
la esperanza y creo saber algo que me permita
Lea los enunciados y elija la alternativa que establece la
ayudarles, pero supongo también que tengo mu-
secuencia correcta de las oraciones según el título.
cho que aprender.
V) He tomado la decisión de seguir aquí porque apre- 44. TEORÍA DE CONJUNTOS SEGÚ CANTOR
cio tanto nuestra ideología como la comunidad de l. Un conjunto queda definido por comprensión
colegas que la practican. cuando reúne los elementos que tiene una propie-
A) I B) l! C)lll D) IV E) V dad en común.
Il. De acuerdo con Cantor, un conjunto queda defi-
41. Los "Tigres" de Asia nido por la totalidad de los elementos que tienen
1) No había nada de "milagroso" en el milagro eco- una propiedad común.
nómico que todos celebraban. lll. Esta definición puede realizarse por extensión o
11) La disciplina y el espiritualismo orientales cons- por comprensión.
tituyeron la base de todo el proceso. [V. G. Cantor, matemático alemán, fue quien primero
lll) Los capitalistas nacionales y extranjeros, instala- estableció la teoría de conjuntos.
dos en los "tigres asiáticos", no habían cesado de V. Un conjunto está definido por extensión cuando
aumentar sus cuotas de mercado. se nombran uno a uno todos sus integrantes.
IV) El éxito se debería a una mano de obra abundan- A) IV - [[ - 111 - I - V D) III - [V - 11 - I - V
te, barata y sobreexplotada.
B) III - V - 11 - IV -1 E) IV -11 -111- V-I
V) También, ello obedecía a la intervención sistemá-
C) 11 - IV - III - I - V
tica del estado proteccionista.
A) 11 B) 111 C) IV D) I E) V 45. LA MÁQU[NA
l. Una herramienta, sin embargo, no hace más q u e
42. aumentar la fuerza muscular misma sin tomar su
lugar.
I) El láser es en esencia un rayo de luz.
ll. Las máquinas a vapor, por eso, iniciaron la revo-
11) La palabra viene de las siglas en inglés de "Light
lución industrial.
Amplification by Simulated Emisión ofRadiation",
lll. Las herramientas del hombre son contemporáneas
lll) El rayo láser tiene un color único y es fácil de ser
de la humanidad misma.
dirigido.
IV. La máquina, en cambio libera al hombre de reali-
IV) La bombilla eléctrica produce una luz multicolor
zar el mismo trabajo fisico.
y dispersa.
V. Lo hace en una escala que supera todo cuanto el
V) Hay rayos láser de cristal liquido y gas.
hombre pueda realizar por sí mismo.
A) I B) 11 C)lll D)[V E) V
A) 111 - IV - I - II - V D) IV - I - 1Il - V - 11
43. B) III - [ - IV - V - [1 E) [[[ - I -[1 - V - IV
I) Las dimensiones del cosmos son enormes. C) II - [ - [[[ - V - [V

11) Es una distancia enorme. 46. ACERCA DEL PERFIL DEL [NGENIERO
111) Por eso, medimos la distancia con la velocidad de l. La primera pregunta que se debe hacer para espe-
la luz. cificar este perfil es "¿Qué hará este ingeniero en
IV) En un segundo, un rayo de luz recorre 300 mil kiló- su ejercicio profesional?"
metro, es decir da 10 veces la vuelta a la Tierra. Il. Estas características se refieren a los conocimien-
V) La distancia que la luz recorre en un año se llama tos y a todo aquello que contribuye al ejercicio de
año luz. una profesión, en este caso, a la ingeniería.
A) I B) [[ C) [11 D)IV E)V lll. Si se hablara del perfil de los ingenieros que se
graduarán en el 20 I O, será necesario proyectar el
ambiente de trabajo de la sociedad del 2010.
IV. Se entiende por perfil profesional, las característi- A) Los mares ayudaron a hacer las playas y
cas que deben reunirse para ejercer adecuadamente erosionaron la tierra.
la profesión. 8) La tierra estaba antes que el aire y el agua.
V. Para ejercer este perfil, además, debe conocerse C) Las rocas estaban en épocas tempranas muy ca-
cuál será el entorno en el que se trabajará. lientes o ígneas.
A) IV - 11 - 1 - V - III D) IV - 1I - ¡-1II - V D) El agua, el aire, los mares y los océanos ayudaron
B) IV - II - V -1Il - ¡ . E) r- IV - II - V - III a la formación de la tierra.
C) 1 - IV - II - III - V E) Existen abundantes estudios sobre las rocas y los
sedimientos.
47. DI ERO SUCIO
50. "La pobreza es un fenómeno que tiene muchas di-
r. Los bancos usan estos depósitos para financiar mensiones, por lo que no existe una única manera de de-
sus actividades. finirIa. Para efectos de un estudio práctico, la mayor parte
Ir. El Estado, incluso, puede estar sujeto a esta in- de las veces, la pobreza se ha definido como la incapaci-
fluencia. dad de una familia de cubrir su gasto familiar. Este cnfo-
III. Las organizaciones criminales influyen, de esta que metodológico clasi fica a las personas como pobres o
manera, en la vida económica. no pobres. En el caso de que cl gasto familiar no logre
IV. El producto de la actividad criminal es deposita- cubrir los requerimientos de una canasta alimcntaria, se
do, generalmente, en la red bancaria. identifica a la familia como de pobreza extrema".
V. El dinero sucio es canalizado, igualmente, a otros Señale la alternativa que no corresponde al texto
mercados financieros, incluido el de los bancos A) La definición de la pobreza es compleja.
estatales. B) Las familias pobres son aquellas que no llegan a
A) I - V - II - III - IV D) I V - V - 1- 1I - 11I cubrir su gasto familiar.
8) 11I - IV - I - V - II E) V - IV - I - II - III C) La pobreza extrema es uno de los problemas que
aflije a una parte importante de la población.
C) IV - I - V - III - II
D) El costo de la canasta familiar cs mayor que el de
COMPRENSiÓN DE LECTURA la canasta alimenraria.
E) Las familias se clasifican en pobres y no pobres de
48. "La historia nos informa que los logros científi-
acuerdo con la metodología enunciada.
cos, así como los adelantos que producen el desarrollo
tecnológico, se han incrcrnentado en un contexto que
admite controversias; y, en cambio se restringen cuan- CULTURA GENERAL
do hay limitaciones para el debate libre".
Del párrafo, se infiere que la ciencia influye de manera COMUNICACiÓN Y LENGUAJE
importante en: 51. Señale las afirmaciones correcta.
A) El ambiente dc libertad. l. La comunicación humana no verbal incluye ges-
8) Los adelantos técnicos. tos, mímica, ideogramas, señales de colores, cte.
C) Las restricciones para el debate constructivo. II. El universo referencial es todo aquello designado
D) El diálogo libre. por los signos y sus combinaciones.
E) La investigación y la discusión. III. En su función apelativa el lenguaje pretende ex-
presar sentimientos o emociones.
49. "EI estudio de las rocas demuestra que el agua y
A) Sólo I 8) I Y 11 e) 1, 11 Y III
el aire aparecieron en épocas muy tempranas de la his-
toria terrestre. Los mares y los océanos, en especial, D)Iylll E) 11Y 111
ayudaron a enfriar la superficie, permitiéndole engro- 52. Señale las afirmaciones correctas.
sar y consolidarse. Las rocas formadas por el enfria-
miento de este materia en fusión se denominan ígneas". r. La proposición es una estructura incluida en una
oración. Tiene sentido pleno e indcpendencia
El texto ilustra que: sintáctica.

L
11. Las conjunciones y preposiciones sirven para unir 57. La novela "Cien Aiios de Soledad" de Gabriel Garcia
palabras o proposiciones. Márqucz relata la historia de la siguiente familia.
111. La oración compuesta coordinada es lo que inclu- A) Los Rivera B) Los García C) Los Bucndia
ye una o más proposiciones dependientes de otra D) Los Palacios E) Los Morales
principal.

A) Sólo I B) I Y [[ C) 1, [[ Y [[[ 58. Señale quién es el autor cuyas obras más destaca-
das son: "La Metamorfosis" y "el Proceso".
D)lylll E) Sólo [[
A) Fedor Dostoicvski, D) Franz Kafka.
53. Señale cuáles son las palabras que carecen de B) Honorato Balzac. E) William Faulkner.
significado por sí solas:
C)Marcel Proust.
A) Adverbio, verbo.
B) Pronombre, adjetivo.
HISTORIA DEL PERÚ (Y ACTUALIDAD)
C) Preposición, conjunción.
D) Verbo, pronombre. 59. Señale las afirmaciones correctas a partir del in-
forme de la Comisión de la Verdad y Reconciliación.
E) Adverbio, adjetivo.
1. La mayor parte de las víctimas estuvo compuesta
por indígenas que hablaban Quechua u otra len-
54. Señale en qué oraciones figuran vocales en hiato.
gua originaria.
l. E[ moho no permite abrir [a ventana del aula.
11. Sendero Luminoso esclavizó a un gran número de
Il. E[ león no siempre es el rey de la selva. Ashaninkas.
lll. Las piedras del camino eran demasiadas para liT. Ya no hay exclusión étnica en el Perú.
proseguir.
A) Sólo I B) [ Y 11 C)[,lIyl[[
A) [y I[ B) Sólo [ C) r, 11 y IIJ
D) I Y 111 E)Sólo[11
D)lIylll E) [ Y [[[
60. Señale las afirmaciones correctas sobre la Gue-
55. ¿Cuántos murieron hoy? Es un caso de: rra del Pacífico.
A) Acentuación según las reglas generales. 1. Comenzó con la ocupación por Chile del tcrrito-
B) Acentuación de ruptura. rio boliviano de Atacama.
C) Acentuación diacrítica. 11. Pese a que muchos pueblos, sobre todo andinos,
D) Acentuación enfática. resistían eficazmente, Iglesias proclamó la derrota.
E) Acentuación diagráfica. 111. Luego de la Guerra el Perú conservó por lo menos
la administración de su riqueza guanera.
LITERATURA A) [ B) Sólo II C) 1, I[ Y [[1
56. La literatura indigenista ha tenido plena vigencia D) I Y [[1 E) I Y II
durante gran parte del siglo XX en el Perú y en ciertos
países latinoamericanos. Señale el comentario correcto. 61. La "Federación de los Andes", concebida por Bolí-
var, consideraba como integrantes a los siguientes países.
A) "Cuentos Andinos'' es una obra destacada del
indigenismo. A) Panamá, Argentina Colombia, Venezuela y Ecuador.
B) La obra de José María Arguedas no tiene influen- B) Colombia, Ecuador y Venezuela
cia del indigenismo. C) Perú, Chile y Bolivia
C) Ciro Alegría, Enrique López Albújar y José Ma- D) Colombia, Venezuela, Ecuador, Perú y Bolivia
ría Eguren fueron exponentes del indigenismo. E) Perú, Bolivia, Ecuador y Chile.
D) La orientación indigenista está ausente en la
62. Complete la siguiente frase: "A inicios del Perú
obra de Ciro Alegría.
Republicano, Jaén que pertenecía a la , por li-
E) Vargas Llosa ha revalorado el indigenismo en la
bre determinación decidió formar parte de ''.
literatura.
A) Audiencia de Quito ..... El Perú. A) Piura - Santa- Rimac- Cañete - Majes
B) Gobernación de Guayaquil Colombia. B) Reque - Santa - Cañete - Lurín - Chillón
C) Intendencia de Guayaquil Colombia. C) Reque - Piura - Chillón - Santa - Cañete
O) Audiencia de Lima .... El Perú. O) Majes - Chillón - Lurín - Rírnac - Cañete
E) Comandancia General de Maynas ... El Perú. E) Piura - Santa - Reque - Cañete - Lurín
63. Señale las afirmaciones correctas sobre la confe- 68. Señale las referencias correctas sobre las relacio-
deración Perú - Boliviana. nes entre Perú y Brasil:
l. Chile declaró la guerra a la Confederación porque I. Las regiones peruanas que limitan con Brasil son
su política comercial perjudicaba a Valparaíso. Loreto, Ucayali y Madre de Dios.
Il. El Gran Mariscal Andrés de Santa Cruz fue elegi- Il. Perú ha firmado un convenio de integración eco-
do Protector de la Confederación. nómica con Brasil.
II!. En la batalla de Yungay (1839) las fuerzas de la !l1. Una de las vías de comunicación transamazónica
Confederación derrotaron al ejército restaurador comienza en el puerto de 110.
organizado por Chile.
A) Sólo [ B) 1 Y 11 C) 1, 11Y 111
A) Sólo I B) Sólo [1 C) Sólo III 0)1 y111 E) Il Y 111
O) I Y II E) Il Y III
69. Complete la información.
64. Señale cuál era el mecanismo de control que la
En Sudamérica dos grandes bloques: el MERCOSUR,
corona aplicaba al Virrey al término de su gobierno.
donde son socios plenos Brasil, Argentina, Uruguay y
A) Real Acuerdo. O) Visita de Inspección. ____ e integrantes asociados , Bolivia
B) Auto de fe. E) Examen de Intendencia. y Perú, y la CAN (Comunidad Andina de Naciones)
C) Juicio de residencia. cuyos socios plenos son , Colombia, Ecua-
dor, Perú y Bolivia.
GEOGRAFíA A) Chile - Paraguay - Venezuela
65. S;iiale qué referencias sobre el Mar peruano son B) Paraguay - Chile - Surinam.
correctas: C) Venezuela - Surinarn - Venezuela
1. Debería ser de aguas cálidas por encontrarse en O) Paraguay - Chile - Venezuela
zona tropical. E) Venezuela - Chile- Surinam
[1. El zooplancton es el primer eslabón de la cadena
HISTORIA UNIVERSAL
alimenticia.
Complete la información.
111. El color de sus aguas es normalmente verdoso.
70. La primera computadora no tenía sino
A) I Y II B)lylll C) 1, Y ILy III _____ , y se fabricó en en la década
O) Il y III E) Solo I del _

66. Señale que ubicaciones de las regiones naturales A) transistores - tubos - Japón - 50.
son correctas. B) tubos - transistores - Alemania - 50.
1. Yunga : entre 500 y 2500 m.s.n.rn. C) tubos - transistores - Francia - 40.
11. Suni : entre 3500 y 4100 I11.S.n.l11. O) transistores - tubos - Estados Un idos - 40.
Ill. Quechua: entre 4100 y 4800 m.s.n.m. E) transistores - tubos - Unión Soviética - 50.

A) Sólo I B) Sólo 111 C)lyllf 71. Señale la alternativa que contiene una de las cau-
O) Il y III E) I Y lf sas de la reforma de la iglesia Catól ica en el siglo XV 1.
A) La repartición de la riqueza acumulada por el alto
67. Señale la alternativa que contiene los nombres de clero para beneficiar a los pobrcs.
cinco ríos de la Cuenca del Pacífico ordenados según B) El deseo de los monarcas de donar sus bienes a la
su ubicación de norte a sur. Iglesia.
:::=::::::~;;==~W'~UNI"l 2004-1 APT. ACADÉMICA Y CULTURA GENERAL I
C) La aceptación de las doctrinas o práct: .. ' conteni- A) El Banco de la Nación.
das en la biblia. B) El flanco Central de Reserva.
D) La simplificación del culto. C) La Banca Comercial.
E) El Humanismo que impulsó el sentido del análisis D) La Superintendencia de Banco y Seguros.
e interpretación de los dogmas y de la Biblia.
E) La Conasev.
72. Señale las afirmaciones correctas acerca del ac-
PSICOLOGíA
tual orden jurídico internacional.
1. Ha sido un avance fundamental la creación del 77. Señale la alternativa que corresponde, en relación
Tribunal Penal Internacional (TPI). al proceso de socialización.

11. Estados Unidos reconoce y se somete al TPI. A) El proceso de adaptación del individuo a su medio
socio cultural termina en la niñez.
111. Estados Unidos se niega a someter sus tropas al
B) La socialización es permanente debido a la situa-
TPI.
ción cambiante del medio.
A) 11 Y 111 B) I Y III C) 1, II Y III
C) La asimilación de conocimientos termina en la
D) I Y 11 E) Sólo II juventud.
D) La socialización no afecta la personalidad del in-
ECONOMíA POLíTICA
dividuo. '
73. El valor monetario de todos los bienes y servicio E) Las creencias y prácticas religiosas no se asimilan
finales producidos en un país en determinado período, a través de la socialización.
es la definición de :
A) Ingreso disponible. 78. Un jefe de personal considera que las mujeres no
B) Ingreso nacional. son eficientes, por lo tanto prefiere no contratarlas. Su
C) Producto bruto interno. negativa puede considerarse como una
D) Producto nacional bruto. A) actitud. B) idea. C) reacción.
E) Producto nacional neto.
D) creencia. E) decisión.
74. Suponga que el gobierno recibe préstamos del
extranjero por US $800 millones, y paga al extranjero FILOSOFíA Y LÓGICA
amortizaciones por US $400 millones e intereses por
US $300 millones. Entonces, el servicio de la deuda 79. Indique cuáles son ciencias formales:
externa es de: 1. Economía. IV. Psicología.
A) US $100 millones D) US $ 700 milllones 11. Lógica. V. Sociología.
B) US s 300 millloncs E) US s 800 milllones III. Matemática.
s
C) US 400 milllones A)II B) 111 C)I1,I1lyIV
D) IV E) 11Y ni
75. Señale las afirmaciones.correctas sobre inversión
extranjera directa (IED): 80. Señale la alternativa que contiene la conclusión
1. Representa una fuente de financiamiento genera- lógica del siguiente razonamiento.
dora de deuda. Ningún animal es mineral.
11. Puede facilitar la transferencia de tecnología. Todas los paquidermos son animales.
111. La IED no debe impedir que se promueva la in- Por lo tanto:
versión nacional. A) Ningún mineral es animal.
A) Sólo I B) I Y 11 C) 1, 11 Y III -B) Todos los animales son paquidermos.
C) Ningún paquidermo es mineral.
D) Sólo 111 E) 11 Y 111
D) Algunos paquiderrnos no son minerales.
76. El organismo encargado de la emisión de mone- E) Algunos animales son paquidermos.
das y billetes en el Perú, es:
SOLUCIONARIO
APTITUD ACADÉMICA P = PL + P,.

RAZONAMIENTO MATEMÁTICO =x+3y ",(1)


El importe total T que se recauda al llegar a Arequipa
1 • El producto P está formado por dos factores A y B
en la proporción: T = Px (P,;'+ Su)

A B 43~2=Px(60+1)
," (1)
2 3 P = 4392
Cada unidad del producto está formado por: 61
= 72 personas '" (2)
UB
Factor A: 2!!L Factor B' , 3-
up La cantidad de pasajeros que llegan a la ciudad de
up
Arequipa:
Para tener N unidades del producto se necesita o uni-
PA = PL - Pb + P"
dades del factor A y b unidades del factor B, es decir:
SO = X - Y + 3y
N=_o_=_b_
'" (2) x+2y=50
uA
2- "s
3-
up up 50 -x
y=-2- ,,(3)

, Por dato: o = 6 uA Reemplazando (2) y (3) en (1):

b = 9 uB (Solo se toma 9 u de las l S u) 72=x+3(50;x)

Reemplazando en (2): x=6


Clave: A
N= 6uA = 9uB
2 uA 3 uB 3. Como "Pepito" (P) y su "mamá" (M) camina con la
up IIp misma rapidez, es decir en un mismo tiempo ambos
= 3up recorren la misma distancia,

Si M recorre 1 cuadra, emplea 2 minutos, entonces:


Clave: C
Si P = M recorre 2 cuadras emplea 2 x 2 = 4 minutos
2. Datos: I
Clave: B
Costo de pasaje por persona : P" = 60 soles/per.
Costo de seguro por persona : Su =1 sol/per. 4. Datos:
Recaudación total del viaje : T= 4 392 soles Precio por l 000 kg sin procesar : sr'l 500
Pasajeros que llegan a Arequipa: F'.4 = SO per. Precio unitario sin procesar Pusp = S/,I ,50/kg
Además: Precio unitario procesado : PUl' = S/,3,00Ikg
Cantidad de personas que parten de Lima: PL = x
Por condición del problema, por una cantidad en peso
Cantidad de personas que bajan P¡, = y de menestra procesada que venda, tendrá que com-
Cantidad de personas que sube~ P,. = 3y prar la mitad en peso de menestra sin procesar. El im-
porte de la venta y de la compra por una tonelada mé-
Cuando llega a la ciudad de Arequipa el total de pasa-
trica (1 000 kg) que venda será:
jeros P que subieron al ómnibus:
~~~~~::~:::::::[~U~N~12~0~0~4-~I~A~PT~.~A~C~A~D~É~M~IC~A~Y~C~U~L~T~U~R~A~G~E~N~E~R~A~L:J:::::::;~~123~
Gomr~

T venta -- I000k-u x 300 soles El tiempo empieza a correr a partir de la 6 am, en que
b'----¡¡;;-
b se encuentra el tanque eon 2 111 3 de agua. La hora en
= 3000 soles ... (1)
que se encuentra vacío será:

TCompra = 500 kg x 1,5 s~~s 1=6am+4h


=IOam
= 750soles ... (2)
Clave: O
El empresario tiene un gasto correspondiente a produc-
ción y venta equivalente al 50% del costo o soporte de 6. Datos:
la menestra sin procesar.
Velocidad promedio del motociclista: v = 40 km
G = 15000x 750 soles h
Distancia que recorre el motociclista: d = L km .
=375 soles ... (3)

La utilidad del empresario: de (1), (2) Y (3) :

u = Tventa - Tcompra - G
=3000soles-750 soles-375 soles
= 1875 soles
Si el motociclista recorre la distancia el a la velocidad v
Clave: A empleará un tiempo I .
1= E..
5. Datos: v
3 Lkm
Flujo de agua que ingresa F, =2~
h 40 km.

m3
Flujo de agua que egresa FE =2,5
T =.l... h
40 ... (1)
Agua existente en el tanque V =2 m3
El tiempo que demora el motociclista en hacer "a" pa-
Capacidad del tanque Vr = 4m3
radas de "b" minutos será:

t" =ax(bmin)
h
=ax(6 Oh)

= axb h ... (2)


V 60
La cantidad de horas que emplean el motociclista en
recorrer la distancia de L km, haciendo las a paradas
de b minutos, será:

Para que no quede agua almacenada en el tanque debe r=».


(4LO /¡ ) + (~~ h)
transcurrir un tiempo T, es decir:
De (1) Y (2): =
(TxF,+V)-TxJ;E;=O
= 3L+2ah /¡
3 120
[T ( 2 ~13 ) +2m 3
] - T( 2,5 1/;1 ) = O
Clave: 8
T = 411
12.~4~~~~;;~J UN12004-1 APT. ACADÉMICA Y CULTURA GENERAL I~~=~;..
/'
~=~
9. Analizando la cantidad de cuadrados sornbreados cn
RAZONAMIENTO LÓGICO cada posición.

7. Si todos los no "creyentes" son "apostadores" y

5E bffi···
ningún "alpinistas" es creyente.
La palabra todos nos indicas un universo:

Apostadores
D
O=lxO 2= 2x 1 6=3x2

8 La cantidad
sión:

donde
IxO

11
de cuadros

; 2xl ; 3x2
sombreados

es el número de cada posición,


; ...
obedece

/l( /l
a la succ-

-1)

De las 5 alternativas la que- se ajusta a nuestro diagra- Para la posición n = 25 tenemos:


ma es "Todos los alpinistas son apostadores",
11(11-1)=25(25-1)
Clave: O
= 600
8. Los años que tienen cada perro son: Clave: D

Rocky :R 10. Del enunciado:


Laica :L Malchi
Pinto :P
Manchas: M
Hace un año murió Laica, cuando ya tenía más de 5
años.
L> 1 +5
L>6>P ... ( 1)
Pina
"Manchas" y "Pinto" nacieron juntos cuando murió El que está sentado a la izquierda de Zory es Pepe
Laica. Clave: C
P=M ... (2)
RAZONAMIENTO ABSTRACTO
"Rocky" es un perro que tiene mas años
R>L>6>P
R >6>

Por condición del problema,


P .., (3)

la edad actual de los pe-


11.~ o

(1)
x o..-x')
~ [I](1-1)
~ ~
(2)

rn
rros que viven:
RxPxM=36 o..-x')

De (2): RxP2 =36 ~ ~ ~


~
(2) (2-2) (3)
De (3): 9x22=36
~ R=9
Clave: C
(4) (4-4) (5 = ?) .
"t==~;1=:~l
.c..=='-'
UNI 2004-1 APT. ACADÉMICA Y CULTURA GENERAL li:::=:::~:~~~
Primer paso: el "circulito" (o) y el "aspa" ( l, mante- 15. En la serie gráfica tenemos tres secuencias.
niendo el mismo orden, giren en sentido antihorario, La 'imera secuencia, la flecha (-.) en dos proposi-
manteniendo la figura (cuadrado) en la misma posi- cienes continuas se mantiene en el mismo lugar, los
ción. siguientes dos gira 90° en sentido horario yasí succ-
Segundo paso: realizado el primer paso, la figura total sivamente.
debe girarse 1800para obtener la siguiente figura de la La segunda secuencia, el pequeiio triángulo de color
secuencia gráfica
Clave: C
negro e...•) ubicado en una vértice del cuadrado va
avanzando de vértice en vértice en sentido horario.

12. Si numeramos a cada tipo de sombreado, se obser- La tercera secuencia, es con respecto al círculo de co-
va que estos van desplazándose de lugar en forma conti- lor negro (- ), que recorre la diagonal del cuadrado.
nua y manteniendo el mismo orden de los números.

m~l*J ~rn
(2) (3) (4
1

= 7)
Clave: D
.u~rl"'J!JJ:1
(1)

16. En la sucesión:
(2)

SUCESIONES
~

NUMÉRICAS
(5) (6 =

Clave: D
7)

13. En la serie gráfica se observa dos secuencias:


4 ; 9 ; 20 ; 43 ; 90 ; 185 ; 376 ; 759
En la primera secuencia la circunferencia va }LFando '----'" '----'" <c.::«<c::« '----'" <c::« <c:«
90° en sentido horario, juntamente eon su pequeño cír- x2+1 x2+2 x2+3 x2+4 x2+5 x2+6 x2+?

culo que va alternado de color: Clave: D


17. En la sucesión:

EBffi ~EB
(1) (2) (4) (5=7)
4

X2+1
.
~~~~~~
9

x3-1
. 26'

x4+2
106'

x5-2
528'

x6+3
3171

x?-3
. 22194

Clave: A
En la segunda secuencia el achurado (rayas) van avan-
zando de lugar en cada área numerada.
18. En la sucesión:

~~ (I~
(1) (2) (4) (5=7)
1
~~~~~~
xg
. 8

+2
. 4

xg
. 32 . 16 . 128·

+2 x8 +2
64

Clave: C
Clave: C 19. En la sucesión:

3 ; 6 ; 4 ; 8 ; 6 ; 12 ; 10 ;20 ; 18 ; 36 ; 34 ; 68
14. Los cuadrados, incluyendo sus áreas sombreadas, '-.J
de un gráfico a otro van girando 900en sentido horario. x2 -2 x2 -2 x2 -2 x2 -2 x2 -2 x2

En los vértices el circulito de color negro va alternando Clave: D


entre los dos vértices opuestos (1 Y 2).
SUFICIENCIA DE DATOS
G0 .AA 21. Dato:Rendimiento R == 20 m2/ galón

tBiJOO ~.
de pintura:

•~~(1)
Además se tiene la información:
(2)
• (4) (5) (UNI) 1) Cada aula tiene una área:
Clave: D A==140m2
I UNI 2004-1 APT. ACADÉMICA Y CULTURA GENERAL I :JlI:
U;iliL:;
G~'¡¡eZ

11) El costo de cada galón de pintura: 24. En los operadores:

C" = 48 soles! galán. I'1(N) = 10 N> l ... ( 1)

Para calcular el monto total de la inversión para pintar I'1(N)=O N$I ... (2)

las "n" aulas del Pabellón A: En la expresión:

T=Il(~XC,,) R=I'1(50-z)
... (*)

= n [ 140 m
20~
2
x48 soles
galon
1 • Si
Z E [40 ; 60]
50 - z > I ~ z < 49
Z E Z ... (3)

galon
De(3): z=~O; 41; 42; .... ; 4~
= 11(336 soles) 9 valores

Observe que para obtener el monto total nos falta el En la fórmula (*):
número n de aulas del Pabellón A: R=I'1(50-z)
Clave: E 2

22. Para hallar la área sombreada: De (1): =!Q


z
_ 10 . 10. 10
- 40 . 41 .... 48
l 2 R2 '-v-------'
A,;ombreada = ¡nR - 2 9 vulo r es

2 • Si 50 - 2 l ~ 22:49
R
=-(n-2)
$
4
De (3): 2=49;50; ... ; 60

En la fórmula (*):
Para hallar el área sombreada sólo se necesita el valor
de radio R de la circunferencia. R = 1'1(50-z)
2
Analizando la información:
[) El área del CÍrculo: es función del radio. De (2):
2
Il) El área del cuadrado: es función del radio
=O (1 valor)
11I) La diagonal del cuadrado: es función del radio
En total R podrá asumir 10 valores
Luego se puede afirmar: "cualquiera de las informa-
ciones es suficiente". Clave: C
Clave: E 25. En los operadores matemáticos:

W(N)=O NE[0;9] ... (1)


23. En los operadores matemáticos:
W(N)=I N E ]9; 99] (2)
F(I1)=1,811+32
W(N)=2 N E ]99; 999] ... (3)
G( 11 - 1) = /13 - 2
Luego:
Luego:
De (3): W(345)=2
F(3)=1,8(3)+32=37,4
De (2): W(50)=1
G( 3) = G( 2 - 1) = 23 - 2 = 6
Finalmente: De (1): W(5)=0
F(3)=37.4=6233 De (2): W(70)=1
G( 3) 6 '
Clave: E De (2): W(22)=1
::~~-=-:=:I UN12004-1 APT. ACADÉMICA Y CULTURA GENERAL r::===;~
En la expresión: 32. ESCRITORIO: Mueble que usa un escritor.
W(345)+ W(50)- W(5) ESCRITOR: Persona que escribe, generalmen-
R = W(70)+ W(22) te lo hace sobre un escritorio.
(2)+ (1)+ (O) Las palabras análogas son caballete y pintor, general-
(1) + (1) mente un pintor pinta sobre un caballete.
= 1.5 Clave: E
Clave: C
ORACIONES INCOMPLETAS
TÉRMINOS EXCLUIDOS
33. Las palabras que guardan un relación lógica de
26. La palabra FUERZA guarda relación de signifi- secuencia son: acercaron - tímido - colorado.
cado con energía, potencia, fortaleza y firmeza. No
"Varios alumnos, pequeños como él, se le acercaron
guarda relación con eficacia que significa capacidad
y Paco, cada vez más tímido se pegó a la pared, y se
de obtener siempre un buen resultado.
puso nervioso.
Clave: O Clave: B
27. La palabra ATAVÍO significa adornar para ropa
o vestimenta, las palabras que se relacionan son atuen- 34. Un Catedrático moderno por lógica propondría
do, traje, ropa, prenda. La palabra que no guarda rela- métodos innovadores de enseñanza.
ción es equipaje, este suele ser una maleta para cargar Clave: C
ropa u objetos personales.
35. La oración trata sobre una persona que tiene un
Clave: E carácter negativo, por lo cual se va aislando poco a
28. Con las palabras: Sol, relámpago, rayo y luna, poco del resto de personas. '
podemos el conjunto de elementos que puedan origi-
"Era un hombre intransigente, intolerante y terco que
nar luz natural. Con la linterna se puede originara luz
no hacía concesiones por ello, con el pasar de los años,
artificial.
fue cayendo en cierto aislamiento que terminó por lIe-
Clave: C vario a vivir la soledad".

29. Las palabras: Trivialidad, fruslería, nadería y chu- Clave: E


chería guarda una relación entre sí porque todos tienen
CONECTORES LÓGICOS TEXTUALES
un significado similar, se refiere a algo de poca impor-
tancia. 36. El texto se refiere a una expresión que para ser
La palabra volubilidad no guarda relación con el res- buena no satisfació a los asistentes, por ello se le hace
to, significa algo que puede cambiar con facilidad. recomendaciones.

Clave: B "Pese a que realizó una excelente exposición, aunque


el análisis de los datos no satisfizo a los asistentes; por
ANALOGíAS eso el jurado le recomendó ser más exhaustivo en
futuras presentaciones.
30. INTANGIBLE: Es algo que no se puede tocar. Clave: C
PERCEPTIBLE: Es algo que se puede percibir,
notar. 37. El enunciado trata de una persona que nació en
Génova por eso es Italiano; pero engrandece a España.
Entre sí guardan una relación de antonomía. De las op-
El primer conector debe ser concesivo, el segundo de
ciones, la palabra inmaterial es análoga a intangible y
consecuencia y el tercero adversativo.
concreto a perceptible.
Clave: C "Aunque había nacido en Génova y, por tanto, era
Italiano, sin embargo, engrandeció a España.
31. REBUZNO: Voz propia del asno
ASNO: Animal mamífero vertebrado. - Clave: D
Las palabras análogas son mugido y toro. El mugido 38. El texto trata sobre la Revolución Industrial, que
es propia del toro o vaca que es un marni fero vertebrado. rompe lo tradicional para introducir nuevos esquemas
Clave: A de vida, por ello necesita uno que lo liderc. El primer
~~:::=:I UNI 2004-1 APT. ACADÉMICA Y CULTURA GENERAL ~::=;~;r:~'~:::;,:::; me''';'

conectar debe ser casual, el segundo de adición y el 45. LA MÁQUINA


tercero de consecuencia. La secuencia correcta de las oraciones:
"La Revolución Industrial dado que rompía con la IlI) La aparición de la herramienta.
anterior, también necesitaba una base social y políti-
1) La limitación de la herramienta
ca. Por esta razón Gran Bretaña la Iideraría."
IV) La máquina supera a las limitaciones de las herra-
Clave: D mientas.

39. En la primera oración nos da un antecedente y la V) La máquina lo supera en gran escala a las herra-
segunda oración es consecuencia de la primera, el mientas.
conector que falta debe ser de consecuencia. Il) La máquina a vapor es un ejemplo de máquina.
"Albert Einsten aseguraba que la imaginación es más Clave: B
poderosa que la razón. Por tanto, debemos estar pres-
46. A CERCA DEL PERFIL DEL INGENIERO
tos a desarrollar y optimizar nuestra creatividad."
Clave: C La secuencia correcta es:
IV) Definición del perfil profesional, basado en las ca-
ELIMINACiÓN DE ORACIONES racterísticas .
11) Características del perfil profesional.
40. Las oraciones 1, ll, III Y IV trata sobre la preocupa-
ción por la falta de conocimientos. La oración V no tie- 1) Actividades que debe desarrollarse el ingeniero
ne relación con el resto, pues trata sobre una ideología. para definir su perfil profesional.

Clave: E V) Entorno donde se desarrolló el perfi I profesional.


11I) Por ejemplo, el entorno de la sociedad en el 2 O I O I

41. Las oraciones 1, 11, IV Y V están relacionadas con necesita un perfil profesional adecuado con la época.
el éxito económico de los "tigres" asiáticos, en cam- Clave: A
bio, la oración 111trata sobre los beneficios de los ca-
pitalistas nacionales y extranjeros. 47. DINERO SUCIO
Clave: B La secuencia correcta:
42. Las oraciones 1,11, III Y V, trata sobre el tema de IV) El producto de la actividad cl:iminal (Dinero su-
los "rayos laser". La oración IV no tiene relación con cio) va a la red bancaria.
las otras porque trata sobre un bombilla de luz que no V) El dinero sucio también va a las bancos estatales.
es rayo laser.
1) Los bancos usan este dinero sucio.
Clave: D
11) El estado también es influenciado.
43_ Las oraciones 1, 1Il, IV Y V, tratan sobre el cos- Ill) En consecuencia los organizaciones criminales in-
mos, sus grandes dimensiones y sus mediciones. La fluyen el la vida económica.
oración II redunda en inforamción sobre las grandes Clave: O
dimensiones.
Clave: B
COMPRENSiÓN DE LECTURA
COHERENCIAS Y COHESiÓN TEXTUAL 48. El adelanto científico y el desarrollo tecnológico
44_ TEORÍA DE CONJUNTOS SEGÚN CANTOR se realizan dentro de marco de discusiones, para lograr
La secuencia correcta de las oraciones: los objetivos muchas veces atenta contra la libertad
de derechos de las personas.
IV) G. Cantor estableció la teoría de conjuntos.
11) Cantor da su definición. Clave: A
111) Esta definición se da por: - Extensión
49. La lectura trata sobre la historia de la tierra: "El
- Comprensión.
agua, el aire, los mares y los océanos ayudaron a la
V) El conjunto se define por extensión. formación de la tierra".
1) El conjunto se define por comprensión.
Clave: O
Clave: E
50. La alternativa e no corresponde al texto, puesto
que esta hace mención a la problemática de la pobreza LITERATURA
extrema de la población. En el texto trata sobre la defi- 56. Con la publicación de "Cuentos Andinos'' para
nición de la pobreza, utilizando algunos criterios. Enrique López Albújar se inició la corriente indigenista
Clave: C en el Perú. También aportaron con esta corriente: Ciro
Alegría, José María Arguedas.
Vargas Llosa con su obra "La Utopía Arcaica" desesti-
CULTURA GENERAL
ma el indigenismo.

COMUNICACiÓN Y LENGUAJE Clave: A

51. 57. Gabriel García Márquez en su obra cumbre "Cien


1) Correcta: La comunicación humana no verbal, años de Soledad" que trata sobre la realidad latinoame-
se puede realizar por mímicas, signos, gestos, etc. ricana a través de la familia Buendía.
o cualquier otro medio visual. Clave: C
11) Correcta: El universo referencial es el conjunto
de símbolos y abstracciones que el hombre crea 58. El autor de las obras: "Metamorfosis" y "El Pro-
para poder estudiar su medio ambiente. ceso" es el checo Franz Kafka.
111) Falsa: La función de pretender expresar senti- Clave: D
mientos o emociones corresponde a la función ex-
presiva del lenguaje.
Clave: B HISTORIA DEL PERÚ Y ACTUALIDAD
52. 59. "La Comisión de la Verdad y Reconciliación" en
1: Falsa: La proposición es un elemento dependiente su informe que presentó el año 2 003 concluyó:
y además no tiene autonomía sintáctica.
Las víctimas ascendieron a más de 60 000 personas, sien-
11: Correcta: Las conjunciones y proposiciones sir- do la mayor parte campesinos quechua hablantes o te-
va para unir palabras, proposiciones, su función nían otra lengua originaria o nativa. En el informe tam-
es relacionante. bién se diee que el movimiento insurgente "Sendero
111: Falsa: La oración compuesta subordinada es la Luminoso" controló a algunas aldeas selváticas como
que está compuesta por una o más proposiciones los Ashaninkas, a quienes lo tenían en calidad de escla-
dependientes de otra principal. vos.
Clave: E También se dio a conocer que aún los peruanos margi-
nan a las comunidades nativas.
53. Las palabras que carecen de significado por si
Clave: B
solas son: la proposición y la conjunción.
Clave: C 60. La guerra del Pacífico empezó con el ataque de
Chile al puerto de Antofagasta y la zona salitrera de
54. El hiato es la secuencia de vocales que pertene- Atacama.
cen a diferentes sílabas.
Cuando chile ya había invadido el Perú, algunos puc-
Hay presencia de hiato en: blos andinos liderados por Cáceres hicieron una bue-
1) El mo-ho no permite . na oposición; pero a pesar de todo los hacendados
11) El le-ón no siempre . liderados por Miguel Iglesias proclamó la derrota.

Clave: A La guerra terminó con el "Tratado de Ancón", donde


una de las tantas perdidas que se acepto fue que Chi le
55. En la expresión: ¿Cuántos murieron hoy? La
administraba la venta del guano de la Isla.
palabra "cuántos" por regla general no debería tildarse,
pues es una palabra grave o llana; pero se tilda por que Son correctas I y 11.
se necesita darle énfasis como pregunta. Clave: E
Clave: D
~=:~::l UNI 2004-1 APT. ACADÉMICA Y CULTURA GENERAL I':~:;::::;:;:::;:::i::::~~;'~
61. La "Federación de los Andes" concebida por Bo- 66. Las regiones naturales del Perú son:
lívar fue para conformar una Federación de Estado en
¡.
Sudamérica conformado en ese entonces (1826) por la ALTITUD
REGiÓN
Gran Colombia, Perú y Bolivia. Este proyecto que fra- < M.S.N.M.
casó involucraba a los territorios actuales de Panamá,
CHALA 0-500
Venezuela, Colombia, Ecuador, Perú y Bolivia.
'YUNGA 500 - 2500
Clave: O 'QUECHUA 2500-3500
• SUNI 3500-4100
62. "A inicios del Perú Republicano, Jaenque perte-
PUNA 4100-4800
necía a la Audiencia de Quito, por libre determinación
decide formar parte de El Perú. JANCA 4800 - 6 768
RUPARUPA 400 - 1500
Clave: A
OMAGUA 80 -400
63. La confederación Perú - Boliviana fue un pro- Son correctas ly 11.
yecto político económico creado por Perú y Bolivia. Clave: E
El Perú por ese entonces sufría de problemas sociales
67. Ordenados de Norte a Sur los ríos:
internas, razón por la cual se le llamó y nombró como
protector de la Confederación a Santa Cruz. RIO DEPARTAMENTO

Como el proyecto de la Confederación era tener el


PIURA PIURA
control del comercio en el pacífico sur, Chile se opuso,
SANTA ANCASH
declaró la guerra, que se terminó con la batalla de RIMAC LIMA
Yungay, donde Perú perdió y se acabó el Proyecto CAÑETE LIMA
Con federativo. MAJES AREQUIPA

Clave: O
Clave: A
64. La corona española creó en I 511 el Real y Su- 68. El Perú limita con Brasil con'Íl'cs departamentos,
premo Consejo de las Indias, organismo que tenía como Loreto, Ucayali y Madre de Dios.
una de sus funciones realizar el Juicio de Residencia a
El Perú y Brasil recientemente han suscrito acuerdos
todos los virreyes que terminaban su gobierno. Este
económicos; entre el Presidente Alejandro Toledo del
juicio consistía en haccrle un balance económico de
Perú y Lula Da Silva de Brasil.
sus cuentas para luego darle, si cra posible un eventual
Actualmente se está construyendo la carretera
sanción.
tansamazónica que parte de 110 aRio Branco.
Clave: C
Son correctas 1, 11, 111.
Clave: C

GEOGRAFíA 69. En Sudamérica los grandes bloques económicos


son: El CAN y MERCOSUR.
65. El mar peruano por encontrarse en zona tropical
debería tener sus aguas cál idas; pero este tiene sus aguas En 1991 se firmó el acuerdo de Asunción, dcnomi nado
frias y además verdosas debido a la presencia del "Mercado Común del Sur" (MERCOSUR). Los inte-
fitoplancton. grantes son: Brasil, Argentina, Uruguay, Paraguay.
Además integran como países asociados: Chile, 1301i-
El fitoplancton en nuestro mar es el primer eslabón en
via y Perú.
la cadena alimenticia.
En I 969 también se firmó el acuerdo de Cartagcna,
Son correctos I y III actualmente se le conoce como "Comunidad Andina
Clave: B de Naciones" (CAN). Sus integrantes son: Venezuela,
Colombia, Ecuador, Perú y Bolivia.
Clave: D
§~~' _1 UN12004-1 APT. ACADÉMICA Y CULTURA GENERAL'

tecnología, pues a efectuarse estas inversiones gene-


HISTORIA UNIVERSAL
ralrncnte se recurre a introducir tecnologías de última
70. En Estados Unidos I 947 se crea la ENLAC la generación.
primera computadora digital de la historia, que funcio- La lEO no debe impedir que se promueva la inversión
naba con tubos de vacío creados en la Universidad de nacional, de lo contrario impediría el .progreso econó-
Pensylvania. mico de los verdaderos peruanos.
Clave: D Clave: E
71. La Reforma de la Iglesia Católica en el siglo XVI
76. En el Perú, según la constitución el organismo
fue impulsada por la burguesía, con el objetivo de dis-
encargado de emitir las monedas y billetes es el Banco
minuir el poder de Roma. También tuvo bastante in-
Central de Reserva.
fluencia el pensamiento humanista que criticó durante
el dogmatismo católico y de la Biblia.
Clave: B

Clave: E
PSICOLOGíA
72. El Tribunal Penal Internacional (TPI) se creó en
Roma el 2 002. Una de sus funciones es investigar y 77. El proceso de socialización de un individuo es un
sancionar los delitos de LESA humanidad y los delitos proceso constante a lo largo de todo su ciclo de vida.
de derechos humanos. Este debe ir asimilando todo los cambios que ocurre en
su medio ambiente.
Los países que se sometieron a este tribunal son aque-
Clave: B
llos que ratificaron su creación entre los cuales se en-
cuentra Perú. Estados Unidos no acepta que sus tropas 78. Las personas según la experiencia quc hallan te-
militares se sometan a este tribunal porque según el nido tienden a tomar una actitud negativa o positiva
atenta contra el principio de soberanía, razón por la en su desenvolvimiento personal,
cual ha suscrito convenio de inmunidad para sus tro-
Clave: A
pas con 67 países miembros del TPI.
Clave: B
FILOSOFíA Y LÓGICA
ECONOMíA POLíTICA 79. La ciencia para su estudio se divide en:
73. El Producto Bruto Interno PBI es el valor mone- Ciencias Formales: estudia entes abstractos, ejemplo
taria que se le da a todos los bienes y servicios finales la matemática y la lógico"
producidos en un país en un determinado tiempo, ge- Ciencias Fácticas: estudia entes concretos, ejemplo la
neralmente es durante el periodo de un año. economía, sociología, Biología, etc.
Clave: C Clave: E
74. En todo préstamo existe: 80. El conjunto de animales: A
Servicio del capital, es el capital que se da como prés- El conjunto de los minerales: M
tamo a cambio de un interés.
El conjunto de los paquidcrmos: P
Amortización, es la devolución por cuotas de capital
De las primeras:
dado en préstamo.
Ningún animal es mineral:
Intereses, es un porcentaje del capital que se da como
retribución del préstamo. Este responde a una tasa fi- Act.M ... (1)
jada a un tiempo determinado. Todo paquidcrmo es animal:
Servicio de Capital = Amortización + Intereses PeA .... (2)
En la pregunta, el servicio de la deuda externa es de $ Oe(l)y(2):
800 millones. PcAa.M
Clave: E ==>Pct.M
75. La inversión extranjera directa (lEO) no genera Ningún paquiderrno es mineral.
endeudamiento, mas bien facilita la transferencia de -Clave: C
-=::=::::"';"""a::'Yw;"" J UNI 2004-11 APT. ACADÉMICA Y CULTURA GENERAL I :ili
Q!J11iQ
G~¡'ieZ

~~[J~~~~
APTITUD ACADÉMICA Y CULTURA GENERAL

APTITUD ACADEMICA
OPERADORES MATEMÁTICOS

1 . Se define los operadores B = (a + b)2 Y


2 2
8=a -b

Hallar el número de valores que puede tomar la


expresión ~ -T 8 ~~~
Cuando: a = 2b Y b « (1 , 9).

A) 1 B) 3 C) 7 D) 8 E) 9 ~~
D) E)

2. Sean ® y ó dos operadores definidos por 5. Indique la alternativa que completa la serie.
a®b=b-a y n~m=n+{1®[1;(1®2)]},

calcule 1ó 1.
3 3

A) 1 B) 1 C) 1 D) 3 E) 9
9 3

3. Se define &
que representa el número de
diferentes parejas de enteros positivos cuyo pro-
A) B) C) D) E)

6. Indique la alternativa que completa la serie mostrada.


ducto esy.
Determine el número x.

x=
VA+2~
~-1
A) B) C) D) E)
A) 1 B) 2 C) 3 D)4 E) 5
7. Indique la alternativa que continúa en la serie.
RAZONAMIENTO ABSTRACTO
4. La figura muestra el desarrollo de la superficie
de una caja. Indique la alternativa que correspon-
de a dicho desarrollo.
====::-':·1 UNI 2004-11 APT ACADÉMICA Y CULTURA GENERAL I
1. a , b yc son números naturales, además b yc
son impares.
n. a, b, y e son números naturales, además b es
mayor que c.
A) La información I es suficiente.
8. Indique la alternativa que se debe ubicar en el
B) La información Il es suficiente.
quinto lugar de la siguiente sucesión de figuras.
C) Es necesario emplear ambas informaciones a
la vez.
D) Cada una de las informaciones, por separado,
es suficiente.
E) La información no es suficiente.

RAZONAMIENTO LÓGICO
A) B) C) D) E) 12. Respecto de: "Si gana Perú, no voy a estudiar".
Indique la alternativa que se puede concluir:
SUFICIENCIA DE DATOS A) Si estudié, ganó Perú.
B) Si no ganó Perú, estudié.
9. Considerando la siguiente información:
C) Si no estudié, ganó Perú
1. X es un número primo.
D) Si fui a estudiar, no ganó Perú.
n. X < 13 E) Nunca estudio porque siempre gana Perú.
Determine el número X, si 7 < X < 17
13. Si ninguna persona que toma mate toma café
A) La información I es suficiente. y algunas personas que toman té toman café, en-
B) La información Il es suficiente. tonces:
C) Es necesario emplear ambas informaciones a la vez. A) Ninguna persona que toma té toma mate.
D) Cada una de las informaciones, por separado, B) Todas las personas que toman mate toman té.
es suficiente.
C) Algunas personas que toman mate toman café.
E) La información es insuficiente.
D) Algunas personas que toman mate no toman té.
10. De cuatro números enteros el mayor es "w" y el E) Todas las personas que toman café toman té.
menor es "x" siendo ambos impares y los dos restan-
14. Se tiene acceso a las siguientes proposiciones:
tes 'Y' y "z" son pares, comprendidos entre 2 y 10.
- Todos los docentes sorrpersonas cultas.
Considerando la siguiente información:
- Algunos docentesno son Ingenieros.
1. w+x=14 Por lo tanto, se puede concluir que:
n. y > 5 Y Y - z = 2 A) Los Ingenieros son cultos.
Se desea determinar los números pares. B) Todos los Ingenieros son docentes.
A) La información I es suficiente. C) Todas las personas cultas son docentes.
B) La información 11 es suficiente. D) Algunas personas cultas no son Ingenieros.
E) Los que no son Ingenieros no son cultos.
C) Es necesario emplear ambas informaciones a
la vez.
15. Si la proposición: "Todos los insectos son in-
D) Cada una de las informaciones, por separado, vertebrados", es verdadera.
es suficiente.
Determine cuál o cuáles de las siguientes proposi-
E) La información no es suficiente. ciones son correctas:
1
11. Determine si: [( 2a + l)b + be es par o impar: 1. Es verdad que ningún insecto es invertebrado.
Considerando la siguiente información: 11. Es cierto que algún insecto es invertebrado.
,
UNI 2004-11 APT. ACADÉMICA Y CULTURA GENERAL l"""-\%'2::;.
<S>,;;;, .~!':;';I 'í!¡;,

UJillL;
Gonl~Z

III. Es falso que algunos insectos no son inverte- gando 5 minutos antes. Si el colegio se encuentra
brados. ubicado a 600 111 de su casa, determine la exprc-
A) Sólo I B) Sólo II C) I Y II sión para calcular la velocidad normal de Renzo.
O) 1 Y 111 E) 11 Y 111 A) 2t + 10 B) t + 5 C) 2t + S

O) 2t-10 El 2t - 5
SUCESIONES NUMÉRICAS
23. Una vaca produce 20 litros de leche los días
16_ Determine el valor de: A soleados y sólo 16 litros los días nublados, además
B
cuando el campesino le canta, la vaca produce 2
litros más, sea día soleado o nublado. Si en 30 días
recolecta 590 litros, a pesar de que hubo 10 días
nublados, Cuántos días le cantó el campesino a I;¡
é

A) 1 B) 1 C) 3 D) 9 E) 11
vaca?
3
A) 5 B) 10 C) 15 D) 20 E) 25
17. Indique la alternativa que pertenece a la su-
cesión mostrada: 24. Una bacteria se reproduce de manera que cada
1; 9; 19; 33; 53; 81; ? 30 segundos el número de ellas se duplica.

A) 109 B) 119 C) 139 D) 169 E) 199 Si la observación de una población de bacterias se


inicia a la 9 h 00 horas y a las 9 h 59 la población es
18. Indique el número que no pertenece a la su- de 50 millones. Qué cantidad de millones de bac-
é

cesión mostrada: terias se tendrá a las 10hOO horas?

1 ; 2 ; 4 ; 8 ; 16 ; 26 ; 42 ; 64 ; 93 A) 100 B) 150 C) 200 D) 250 E) 300

A) 4 B) 8 C) 16 D) 26 E) 42 25. Juan y Pedro corren alrededor de un parque,


dando una vuelta en 5 minutos y 450 segundos,
19. En la sucesión mostrada, indique el valor de respectivamente.
X + y.
Si corren durante 30 minutos, determine la dilc-
1 ; 2 ; 4 ; 8 ; 7 ; 14; 10 ; X; Y rencia entre el número de vueltas que dan ambas
A) 7 B) 13 C) 20 D) 30 E) 33 personas.
A) 0,5 B) 1,0 C) 1,5 D) 2,0 E) 2,5
RAZONAMIENTO MATEMÁTICO
20. Carlos viaja de un punto a otro y sale con una
rapidez de 40 Km/h. Cuando aún le falta recorrer RAZONAMIENTO VERBAL
4/5 de-su camino, duplica su rapidez lo que le per-
mite llegar a su destino 2 horas antes. Hallar su ORACIONES INCOMPLETAS
recorrido, en Km.
Elija la alternativa que al insertarse en los espacios
A) 195 B) 200 C) 205 D) 210 E) 215 en blanco, complete adecuadamente el significado
de la oración.
21. Un reloj se adelanta cuatro minutos cada seis
horas. Cuánto tiempo debe transcurrir para que
é
26. Las culturas que son y poco inu-
dicho reloj marque la hora correcta? gradas encierran muchas .
Expresar la respuesta en días, A) homogéneas· similitudes
B) extrañas . peculiaridades
A) 30 B) 35 C) 40 D) 45 E) 50
C) ancestrales . vivencias
22. Cierto día, Renzo incrementa su velocidad nor- D) hererogéneas - contradicciones
mal en 10 m/min para ir de su casa al colegio, lle- E) modernas - carencias
~~
;g¡;

GomeZ
I UNI 2004-11 APT. ACADÉMICA Y CULTURA GENERAL I
del conocimiento debe tener cimientos inconmovi-
27. Muchos dicen que las novelas , pero
bles y, para ello, es necesario abandonar todos los
ésa es sólo una parte de la historia; la otra es que
supuestos y afirmar únicamente aquello que es tan
así una curiosa verdad.
evidente y simple que no puede ser objeto de duda."
A) delatan - traicionan
Señale el enunciado que no es concordante con el
B) mienten - expresan contenido del texto.
C) interpretan - encubren
A) Descartes se fundamenta en las conjeturas para
D) engañan - ocultan investigar.
E) callan - limitan B) Descartes opta por abandonar todos los supuestos.
28. La ante el sufrimiento y la . C) Según Descartes, el conocimiento tiene una
hacia actos injustos han sido diligentes . base evidente.
de las más terribles tragedias humanas. D) La certeza es para Descartes la base de su
investigación.
A) justicia - obstinación - guardianes
E) Para Descartes, la investigación no debe ser
B) indiferencia - tolerancia - aliados
objeto de duda.
C) fuerza - valentía - armas
D) cobardía - tenacidad - estratagemas 31. "Es evidente que la televisión influye decisiva-
E) vacilación - venganza - compañeros mente en la opinión que se forman los niños sobre la
realidad social. Y esta influencia se basa en buena
COMPRENSiÓN DE LECTURA parte en los estereotipos que utilizá la televisión al
representarla. Así podemos comprobar cómo se di-
29. "Algunas políticas universitarias, han puesto en
funden y reproducen los estereotipos sociales sobre
tela de juicio que un universitario con título no pue-
las diferencias en función de sexo, raza, clases so-
de convertirse a ser puramente un técnico. Dado que
la sociedad les concederá cierto poder y determina- ciales o el origen geográfico. Ello ocurre, por ejem-
das funciones, deben ser capaces de examinar con plo, en el mito de Tarzán que aparentemente es un
cierto rigor cuestiones que no son de su propia téc- héroe natural y neutral. Nada más lejos de la reali-
nica. Se trata de una elección política y ética, y ello, dad. Tarzán nace en lajungla, pero es blanco y nun-
porque las autoridades universitarias han juzgado ca se siente atraído por una mujer negra."
que sería irresponsable formar "científicos" sin dar- Considerando el contenido del texto, si la TV difun-
les una cierta formación en el terreno humano". diera programas donde las niñas asumen tareas que
Del contenido del texto se deduce que las universi- tradicionalmente realizan los varones, entonces
dades A) serían programas que transmiten una ideolo-
A) son indiferentes al desarrollo social del país. gía racista. -
B) dan una formación integral a los futuros pro- B) la TV permitiría deshacer estereotipos de ca-
fesionales. rácter social.
C) deben formar profesionales para funciones so- C) la TV tendría como propósito sólo un interés
ciales. económico.
D) examinan sin rigor científico el estatus del téc- D) Tarzán sería un héroe natural que ampara a los
nico. niños.
E) hacen caso omiso a la formación científica del E) los estereotipos difundidos por la TV serían sen-
estudiante. soriales.

30. "Descartes propone un modelo de investiga- COHERENCIA Y COHESiÓN TEXTUAL


ción basado en lo indudable, en lo absolutamente
firme. Es decir, no partir de lo probable, sino de lo Tomando como referencia el título, elija la alternati-
seguro; no de una hipótesis, sino de una tesis; no va cuyos enunciados permitan mantener una relación
de una conjetura, sino de una certeza. El edificio coherente y cohesiva al sentido global del texto.
<S> I UN12004-11 APT. ACADÉMICA Y CULTURA GENERAL I JlI:
,~
GorneZ

32. lA ROBÓTICA go, también el nombre latino que ahora va


I. La construcción de estas máquinas automáti- rían según los países.
cas es consecuencia de la madurez tecnológi- IV. Los astrónomos antiguos estudiaron el movi-
ca alcanzada. miento del Sol, la Luna y los planetas de las
II. Los usuarios de esta industria han influido mu- constelaciones.
cho en el desarrollo de los robots de soldadura V. Los árabes fueron los primeros en estudiar de
y de pintura. forma sistemática el firmamento.
III. En los años sesenta, los robots fueron concebi- A) IV - V - II - III - I
dos para asignarles casi cualquier tarea. B) V - IV - II - I - III
N. La industria del automóvil ha sido el principal C) V - I - III - IV - II
usuario de robots.
D) IV - V - 1 - III - II
V. La robótica se ocupa del diseño y la construc-
E) I - IV - V - II - III
ción de robots, ingenios electrónicos que rea-
lizan diversas actividades.
35. LOS PODERES DEL ESTADO
A) III - V - 1 - IV - II
I. Los poderes de Estado son órganos que diri-
B) V - I - III - IV -II gen el destino de los habitantes de un país,
C)IV-II-V-I-III siguiendo las normas señaladas en la Consti-
D) V - I - III - II - IV tución.
E) III - IV -V - I - II II. El Poder Legislativo está a cargo de la elabora-
ción de las leyes y la fiscalización de los actos
33. EL RADAR realizados por el poder anteriormente mencio-
I. El radar de impulsos se emplea en misiones nado.
de reconocimiento y exploración. III. El Poder Ejecutivo, constituido por el Presiden-
II. El radar es un dispositivo para detectar la pre- te y los Ministros de Estado, están encargados
sencia de objetos. de aplicar la política gubernamental correspon-
diente.
III. El tipo de radar más corriente conocido es el
radar de impulsos. N. Existe, por último, otro en cuya administra-
N. El llamado radar de exploración tiene una an-
ción descansa la esperanza de la población: el
Poder Judicial.
tena giratoria que emite impulsos en todas las
direcciones para abarcar el área circundante. V. Este Poder está constituido por congresistas ele-
gidos mediante el sistema del voto preferen-
V. Este dispositivo permite determinar la direc-
cial, por el mismo período establecido para el
ción y distancia a que se encuentran, utilizan-
Presidente.
do ondas de radio.
A) I - II - III - V - IV
A) II - V - III - I - IV
B) III - V - II - I - IV B) III - I - II - V - IV
C) III - 1 - V - II - IV C) I - III - 11- V - IV
D) II - V - III - IV - I D) 11- III - 1 - IV - V
E) 11- III - V - IV - 1 E) I - III - V - 11- IV

34. EL ESTUDIO DE lAS ESTRELlAS


ELIMINACiÓN DE ORACIONES
I. Por esa razón, prácticamente todos los nom-
bres más anLiguos de las estrellas proceden de Elija la oración que es redundante o no tiene rela-
esta cultura. ción con el tema desarrollado en el texto.
11.Así, por ejemplo, la Osa Mayor CUrsa Mayor) 36.
también se denomina popularmente Carro. I. Una aleación es la fusión de metales con la
III. Algunos de las constelaciones tienen, desde lue- que se obtiene una especie de metal artificial.
·1 UN12004-11 APT. ACADÉMICA Y CULTURA GENERAL I <8>
IL Elbronce es una aleación, resultado de la fu- A) I B) 11 C) III D) IV E) V
sión del cobre y del zinc.
39.
111. El latón se obtiene de la fusión del cobre y del
zinc. 1. En la cultura occidental, la inteligencia ha re-
IV El acero se usa desde el siglo VIII, cuando se presentado la capacidad por la que el hombre
elaboró el acero damasquino. ha intentado ubicarse en la escala de los seres
racionales.
V El acero es una aleación que surge de la incor-
poración del carbono al hierro. 11. Para la psicología, el estudio de la inteligencia
no es sino el análisis de cierto número de acti-
A) I B) 11 C) III D) IV E) V vidades, comurunente considerados "inteligen-
tes".
37_
III. Las distintas escuelas psicológicas, sin embar-
L La mayólica es un tipo de loza que recibió su go, no están de acuerdo en si estas actividades
nombre por alusión a la isla de Mallorca de dependen de una sola capacidad o si son fun-
donde procede. ciones de capacidades independientes.
11. La loza mayólica es una cerámica de pasta po-
IV Así, la psicología conductista intenta explicar
rosa habitualmente recubierta por un barniz la inteligencia en términos de habituación.
vítreo de estaño.
V. Para la psicología cognitiva, liderad a por
111. Los ingredientes que se utilizaban para prepa- Piaget, la inteligencia es una acción
rar la pasta son arcillas calcáreas mezcladas interiorizada que realiza una transformación
con arena fina. para pasar de un estado de partida a un esta-
IV Los antiguos egipcios fabricaban ya una espe- do finaL
cie de mayólica hacia el año 3300 a.C., que
A) I B) II C) III D) IV E) V
luego alcanzaría su apogeo unos dos mil años
más tarde.
CONECTORES LÓGICO - TEXTUALES
V Ese arte de fabricar la mayólica se extendió a
Persia y, desde allí, a través de los árabes, lle-
40 la protección del medio ambiente es
gó a España.
tarea de todos, muchas industrias son indi-
A) I B) 11 C) III D) IV E) V ferentes generan gases tóxicos, .
así ocasionan mayor daño a la naturaleza.
38.
A) Aunque - no obstante - porque - y
1. La primera bicicleta fue la que se conoció como
B) Si bien - y - puesto que - aunque
draisiana, en honor a su inventor, el barón Kart
Van Drais (1816). - C) Si - entonces - o - ergo
11.Pasaron más de 20 años hasta la construcción D) Puesto que - pero - aunque - es decir
de la bicicleta de impulsión mecánica inventa- E) Porque - pero - dado que - ni
da por Kirkpatrick MacMillan.
41. Para la ingeniería, el desarrollo no es un mito
III. En 1816, se colocó con éxito en el mercado
una bicicleta impulsada por dos pedales fija- .......... un conjunto de indicadores crecientes,
dos a la rueda delantera, ideada en Francia ............. una tarea diaria donde las obras constitu-
por Pierre Michaux, yen la base del desarrollo; el docente debe
desarrollar la creatividad en los estudiantes de esta
IV En 1888, John Dunlop introdujo la idea de in-
área.
flar un tuvo de goma con aire para la amorti-
guación y con cuya innovación la bicicleta se A) o - ya que - pues
convertió en un vehículo de gran aceptación. B) ni - sino - por eso
V En la segunda mitad del siglo XX, las bicicletas C) si bien - ni - aunque
aumentaron su difusión debido al interés de D) sino - mas - entonces
las personas de responder a la moda y cuidar
E) y - Y - dado que
su estado físico.
~;=::;:=jl ~NI2004-11 APT. ACADÉMICA Y CULTURA GENERAL I;::;;:;::i:~~;:l~:::;::;:i~;;~t~~'
\
42 el prestigio es un elemento subjeti-
vo intangible del esta tus social, sus TÉRMINO EXCLUIDO
miembros son clara universalmente re-
conocidos. Elija el término que no guarda relación de signifi-
cado común con la palabra escrita en mayúsculas.
A) Si bien - o - o
B) Aunque - e - y 47. CORTO
C) Si - es decir - pero
A) escueto B) breve C) nimio
D) Porque - e - mas
D) conciso E) lacónico
E) Puesto que - además - aunque
48. GENEROSO
43 el nivel de comprensión lectora de
los niños en nuestro país es crítico, muchas autori- A) filántropo B) altruista C) dadivoso
dades educativas no se preocupan; es D) acendrado E) bondadoso
momento de tomar alternativas de solución .
ello genere mayores gastos al Estado. 49. PETULANTE
A) Porque - pues - y A) arrogante B) soberbio C) obstinado
B) Si - y - a pesar de que
D) ufano E) presuntuoso
C) A pesar de que - no obstante - y
D) Pese a que - por eso - aunque 50. MANIFIESTO
E) Puesto que - es decir - o A) palmario B) ostensible C) compacto
D) paladino E) patente
ANALOGíAS
Teniendo como referencia la relación que mantie-
ne el par base escrito en mayúsculas, elija la alter- CULTURA GENERAL
nativa que contenga dicha relación analógica.
HISTORIA UNIVERSAL
44. EXTROVERTIDO: INTROVERTIDO
51. Señale la alternativa que completa correcta-
A) entretenido : locuaz mente el siguiente enunciado:
B) expresivo : tímido
" Las viviendas construidas mayormente de made-
C) fanático : contestatario
ra, levantadas en plataforma sobre lagos, lagunas
D) artista : cantante o suelos húmedos, y que caracterizan el periodo
E) insensato : prudente ................ , se conocen con el nombre de
45_ INFARTO: CORAZÓN
A) Paleolítico - domus.
A) patología : artrosis
B) saliva : boca B) Neolíaco - palafitos
C) espalda : vértigo C) Merolítico - megarones.
D) depresión : soledad D) Medioevo - bastidas.
E) cirrosis : hígado E) Magalítico - Dólmenes

46. DIVULGAR: TRANSMITIR 52. Dados los siguientes factores, señale cuáles
de ellos contribuyen principalmente a la conforma-
A) propalar : difundir
ción de la cultura Medieval.
B) enseñar : producir
C) cobijar : exiliar A) Greco - Latino - Bizantino - Cristiano.
D) retener : encerrar B) Mesopotámico - Cristiano - Germánico.
E) captar : entender C) Cristiano -' Greco - Latino - Germánico.
D) Bizantino - Greca - Latino - Árabe. 58. Lo que sigues es la parte final de un oficio de
E) Renacentista - Germánico - Cristiano. invitación. Señale cuántos errores de redacción se
aprecia en él:
53. Señale cuáles de la afirmaciones siguientes son
Con la seguridad de que esta nuestra invitación ten-
principios establecidos en la Carta de la ONU (1945). drá su favorable acogida en usted, y, a la espera de su
I. Todos los Estados miembros tienen igualdad respuesta, me le valgo de la oportunidad para reiterar-
soberana. le los sentimientos de mi mayor consideración".
11. Si un Estado se siente amenazado tiene dere- A) cuatro B) tres C) dos
cho a tomar medidas preventivas. D) uno E) ninguno
III. Los Estados deben abstenerse de usar la fuer-
za o amenazar con ello a otro Estado.
GEOGRAFíA
A) Sólo I B) Sólo II C) Sólo III
59. La Cordillera de los Andes divide el Perú en
D) I Y n E) I Y III regiones hidrográficas. Señale las afirmaciones co-
rrectas.
COMUNICACiÓN Y LENGUAJE I. La vertiente del Pacífico tiene como divisoria
de aguas la cadena occidental de los Andes.
54. Indique cuantos diptongos (sin considerar los
que se repiten) contiene el siguiente párrafo: n. La Hoya del Titicaca incluye los ríos que des-
aguan en ese lago.
"La ciudadanía disfrutaba de una de las más bri-
III. La Región Hidrográfica del Amazonas está for-
llantes obras teatrales realizadas a campo abierto;
mada por los ríos que vierten sus aguas en el
mejor regalo no podía recibir en el día del aniver-
río Amazonas y finalmente en el Océano Atlán-
sario patrio".
tico.
A) 3 B)4 C) 5 D) 6 E) 8 A) Sólo 1 y III B) Sólo I y II C) Sólo n y I!I
55. Señale la alternativa que presenta el mayor D)I,I1ylll E) Sólo III
número de hiatos:
60. Más del 70% de la población peruana vive en
A) Elias vió a mi abuelo ayer. zonas urbanas, pero hay departamentos donde la ma-
B) Raúl no oía lo que le decía. yoría todavía vive en zonas rurales. Uno de ellos es:
C) Cuestionaron esa nueva teoría.
A) Ucayali B) San Martín C) Paseo
D) Su reacción fue muy tardía.
D) Junín E) Huancavelica
E) Nadie quiso guiar a la nuera.
61. El geógrafo Javier Pulgar Vidal clasifica ocho
56. Indique la alternativa que completa adecua-
regiones naturales en el Perú en base a:
damente la frase:
A) Clima B) Suelo C) Altitud
"Ayer me con su buen , pero la
verdad es que aún no me a ir de paseo". D) Latitud E) Recursos naturales

A) animó - animo - animó 62. Complete la siguiente información:


B) animo - anímo - animo
Las tres grandes llanuras de América del Sur son:
C) animó - ánimo - anímo llanura Amazónica, y llanos del
D) animo - ánimo - anímo Orinoco.
E) animó - ánimo - animo
A) Llanura del Po.
57. En la frase "Él es un peruano ejemplar" tene- B) La Meseta del Callao.
mos un ejemplo de acentuación gráfica . C) Llanura Indogangética.
A) diacrítica B) diagráfica D) enfática D) Llanura del Golfo.
D) tónica E) de ruptura E) Las Pampas y el Gran Chaco
~~===:=:I UN12004-11 APT. ACADÉMICA Y CULTURA GENERAL ~'W%':mtY¡F""

A) Observaciones D) Leyes
63. Entre 1885 y 1929 la región arnazónica se con-
virtió en un lugar de especial interés económico y B) Teorías E) Explicaciones
geoestratégico para los países de la cuenca del Ama- C) hipótesis
zonas debido a:
A) La explotación del caucho y la shiringa. HISTORIA DEL PERÚ (Y ACTUALIDAD)
B) Los conflictos entre pueblos nativos. 68. Señale el orden cronológico correcto entre
C) La explotación de la madera. las culturas Preincaicas siguientes:
D) La explotación de la quinina. A) Tiahuanaco - Chimú - Chavín
E) La explotación petrolera. B) Chimú - Tiahuanaco - Paracas
C) Tiahuanaco - Chimú - Paracas
PSICOLOGíA D) Chavín - Tiahuanaco - Chirnú
64. Según la teoría psicoanalítica, el conjunto de E) Paracas - Chavín - Nazca
pensamientos, fantasías impulsívas y contradicto-
rias que pugnan por ser satisfechas de inmediato 69. ¿Quiénes fueron los dos Incas que más contri-
se denomina: buyeron al desarrollo y consolidación del imperio
del Tahuantinsuyo?
A) pensamiento preconsciente.
A) Mayta Capac y Pachacútec.
B) pensamiento consciente.
B) Pachacútec y Tupac Yupanqui.
C) pensamiento inconsciente.
C) Pachacútec y Huayna Capac,
D) pensamiento primitivo.
D) Huiracocha y Pachacutec.
E) pensamiento mágico.
E) Lloque Yupanquí y Capac Yupanqui,
65. Señale qué habilidad incluye el auto dominio,
70. Qué proyectos integracionistas impulsó
la persistencia y la capacidad de motivarse a uno
é

Simón Bolivar?
mismo.
A) Inteligencia dinámica. l. La Confederación Peruano-Boliviana y el Con-
B) Inteligencia emocional. greso de Panamá.
C) Instinto social. 11. El Congreso de Panamá y la Federación de los
- Andes.
D) Motivación flexible.
III. El Congreso de Panamá y la Unión Panameri-
E) Conducta reactiva.
cana.
A) Sólo 1 B) Sólo II C) Sólo III
FILOSOFíA Y LÓGICA
D) 1 Y II E) 1 Y III
66. Indique el tipo de falacia que se comete en la
siguiente expresión: 71. Señale cuál fue el cultivo de la costa norte
peruana que se incrementó notablemente durante
"No aprobé el examen anterior, porque los miérco-
el periodo de la Reconstrucción Nacional,
les todo me sale mal".
convertiéndose en uno de los principales produc-
A) Anfibología. tos de exportación.
B) Argumento por ignorancia. A) Caña de azúcar D) Arroz
C) Argumento de fuerza. B) Algarrobo E) Vid.
D) Causa falsa. C) Trigo
E) Énfasis.
72. Considerando las últimas seis decádas, indi-
67. Cómo se denomina en el método científico a que en qué años las crisis políticas y económicas
é

las conjeturas formuladas para dar razón de los propiciaron la instauración de regímenes autorita-
hechos. rios en el Perú?
A) 1945 - 1968 - 1990. A) I Y JI[ B) [ y II C) Sólo 1
B) 1948 - 1963 - 1995. 1)) II Y III E) Sólo 1"
C) 1948 - 1968 - 1992.
D) 1952 - 1963 - 1992. 77. Señale cuál es la alternativa que corresponde
a la siguiente definición: "Total de bienes y servi-
E) 1945 - 1980 - 1992.
cios finales producidos dentro .de un país, en un
73. cQué medidas corresponden al gobierno del periodo determinado, valorizados a precios de
General Juan Velasco Alvado? mercado".

1. La creación del Seguro Obrero, la Reforma A) Ingreso Nacional.


. Agraria y la Nacionalización del Petróleo. B) Ingreso Nacional Real.
Il. La reforma Agraria, la Nacionalización del Pe- C) Producto Nacional Bruto.
tróleo y la Proclamación de las 200 Millas del
D) Producto Nacional Neto.
Mar TerritoriaL
E) Producto Bruto Interno.
1II. La Reforma Agraria, la Estatización de los Me-
dios de Comunicación y la Nacionalización del
LITERATURA
Petróleo.
78. Entre la siguientes célebres citas señale cuá-
A) Sólo I B) Sólo II C) Sólo IIJ
les están correctamente atribuidas:
D) [ Y 11 E) II Y lJl
1. "En un lugar de la Mancha ... " Miguel de
ECONOMíA POLíTICA Cervantes Saavedra.
II. "¿Qué es poesía? ¿y tú me lo preguntas?". Gus-
74. En nuestro país, la instancia que aprueba el
tavo Adolfo Bécquer.
Presupuesto Nacional, es:
111. "cQué es la vida? un frenesí". Pedro Calderón
A) El Ministerio de Economía y Finanzas. de la Barca.
B) La Conualoría General de la República,
A) Sólo I B) I Y 11 C) 11 Y III
C) El Congreso de la República.
D) 1," Y 1I1 E)lyl"
D) La Comisión Nacional de Presupuesto.
E) La Dirección Nacional de Presupuesto Público. 79. Señale las afirmaciones correctas:

75. El documento que registra todas las transac- 1. El harawi de la Literatura lnca es el canto de
ciones económicas de un país con el resto del mun- amor y tristeza.
do se denomina: H. El amor doliente es el tema central de los
yaravíes de Mariano Melgar.
A) Balanza comercial.
11I. José María Arguedas en su prosa tierna y poé-
B) Balanza de pagos.
tica, eleva a categoría literaria el Castellano
e) Balanza en cuenta corriente.
Andino.
D) Cuenta financiera.
A) I , 11 Y 11I B) I Y 11 C) Sólo 1
E) Balanza cambiaria.
D) I Y JI[ E) J[ Y III
76. Señale las afirmaciones correctas.
1. En condiciones de libre mercado, el aumento 80. Señale la relación correcta entre obras y autores.
de la oferta de un bien conduce a una dismi- 1. Jorge Luís Borges a. La mala hora
nución de su precio. H. Grabiel García Márquez b. Odas elementales
II. En condiciones de libre mercado, el precio de 111. Pablo Neruda c. Historia de la eternidad
un bien garantiza márgenes de ganancia para
sus productores. A) la - IIb - Illc D) Ila - lb - lile

111. En condiciones de monopolio u oligopolio en B) lb - IIc - lIla E) lila - llb - k


la producción de un bien, su precio tiende a C) le - Ha - lIIb
crecer.
SOLUCIONARIO
APTITUD ACADÉMICA t+{l®~}
1/3
OPERADORES MATEMÁTICOS
1 . Del enunciado De (1): t+(~r
1/3
~=(a+b)2 =3
B=a 2
_b2
Clave: O

Resolviendo la expresión:
3. El operador matemático ffi
B (a+b)(a+b) a+b
define el úrnero

B (a - b)(a +b) a-b ... (1)


de diferentes parejas de enteros positivos, cuyo pro-
ducto esy.
Operando en cada una de las expresiones:
Por dato: a=2b y bE(1;9)
En (1):
a+b
a - b
2b+b
2b - b
~;: ~14
18 x 3
parejas

3b 9x6

Nota: La expresión
=3
b

toma un sólo valor cuando


M.: 47x1} 1 pareja

a = 2b.
Clave: A
&: 16X1)
8x 2 3 p a rejas

2. Del enunciado:
4x4
Luego reemplazamos el número de parejas de cada
... (1) uno de los operadores matemáticos antriores en la
expresión:
n+{1®[1+(1®2)]}
n 6. m = --'---"--------'---'-='- ... (2)
m
Resolviendo la expresión pedida: ~+2M.
x=

De (2):
1 1
36.3=
t+{1®[1+(1®2)]}
1/3 &-1
14 + 2(1)
t+{1®[1+(T1)J} x = -(7: C7)-_.'::-1-'-
3
De (1):
- 1/3
_ 2+ 2
--2-
t+{1®[1+1J} =2
1/3 Clave: B
~ª~~"~",:"l
UN12004-11 AP ACADÉMICA Y CULTURA GENERAL I
7. Si ubicamos la primera gráfica de la serie en
RAZONAMIENTO ABSTRACTO siste na coordenada.

4. En el desarrollo (despliegue) de la caja: z


Círculo negro - •.

rCuadrado
~
y

~-(
Círculo negro: De una secuencia a otra, gira 90°
en sentido horario en el plano ZY.
Clave: D
Cuadrado: De una secuencia a otra, gira 90° en sen-
5. En la secuencia: tido horario (observa desde arriba), en el planoXY.
Círculo blanco: Va girando 90° en sentido
antihorario (observando de la derecha), en el pla-

~~~~§j noXZ.
Clave: A

~~~~ 8. En la primera gráfica de la sucesión:


Clave: B

6. En la serie gráfica:
~.
Círcul{~/ea sombreada
Tra~~CUIO

Tríangulo '--v--J '--v--J


I II

&€~OO~EmOO
(?)
La región 1 está dividida en dos zonas.
La región 11está dividida en tres zonas 0, 2 Y 3).
De izquierda a derecha, la figura completa va gi-
Trama: al avanzar una secuencia de la serie se
rando 90° en sentido horario; pero el círculo va
desplaza verticalmente un lugar hacia abajo. Cuan- alternando entre las dos zonas de la región 1; y el
do llega al extremo inferior empieza nuevamente "área sombreada" va trasladándose en las zonas de
por el extremo superior. la región 11,siguiendo el orden: 1 - 2 - 3 - 1 - 2 - ...
Círculo: De una secuencia a otra avanza vertical-
mente hacia arriba, cuando llega al extremo supe-
rior, nuevamente empieza en el extremo inferior.
Triángulo: De una secuencia a otra gira 90° en
sentido horario; pero manteniéndose en su mismo
Clave: E
casillero.
Clave: D
Se deduce que tanto ')r" como "z" pueden asumir
hasta dos valores, por tanto, la "información no
SUFICIENCIA DE DATOS
es suficiente" para determinar con precisión es-
9. Considerando: tos valores.
I) X es un número primo. Clave: E
II) X<13
Para determinar X, tal que: 11. Del enunciado:

7<X<17 [(2a+l)b+bc] ... C')


Aplicando (I): Además:
X = 11 ; 13 (no se puede determinar). I) a ; b y e E N b Y e son impares
Aplicando (II): 11) a ; b y e E N b >e
XE (-00; 13) (no se puede determinar). En (*) y aplicando (I):
Aplicando la información (I) y (II), se deduce que: • Factorizando b:
X = 11
Clave: C

10. Enunciado: w; x ; y ; Z E Z b
y
impar r2a+1+
---
~ e
"
impar 1
Impar
w : número mayor (impar) '------v-----'
par
x: número menor (impar)
par
y : número par 2 < Y < 10
• Facrorizando b y aplicando (II):
Z : número par 2< Z < 10
De donde se deduce que w > x y además y, Z
pueden asumir los valores:
y; z=4;6;8 ... (1)
~
parb o
,Impar
r2a+1+
'--.,--'
impar
~
pare o
impar
1
Además se tiene la información:
¡No se puede determinar'
I) w+x=14
Luego, la información (I) es suficiente.
lI)y>S Y y-z=2
~ '---v--' Clave: A
a b

Para determinar los números pares, valor de ')r" y I

"z", O) no nos proporcionará ninguna información, RAZONAMIENTO LÓGICO


por lo tanto sólo nos queda por aplicar (II). 12. Según el enunciado:
De (1) y (IOa:
Si gana Per ú , 110 vaya estudiar
"-----v-----' '---v---' '-----v---'
y=4;6;8 /\ y c- S P q
obtenemos:
p --'> - q
y = 6 ; 8 ... (2)
Aplicando la propiedad de transposición
Aplicando (2) en (II)b:
y-z=2
"Si estudio es por que no ganó Perú" ; ó "Si fui a
6-4=2 estudiar; no ganó Perú".
8-6=2
Clave: B
"'"
4J,~
GomeZ
.:;:::.=:::::;;~;;.
. .;;;;¡,ifil UNI 2004-11 APT, ACADÉMICA Y CULTURA GENERAL ~"'1L~:74U~" <8>
13. Aplicando teoría de conjuntos: Lo cual 'es falso según el gráfico estructurado del
enunciado.
Del enunciado :
Clave: O
• "Ninguna persona que toma mate toma café":
MnC=0· 14_ De las proposiciones, y por la teoría de con-
• "Algunas personas que toman té toman café": juntos:
Personas cultas :C
n(CnT)=y
Docentes' :D
Además, como no dan información, se puede asu- Ingenieros :1
mir que:
Luego en las proposiciones:
n(MnT)=x
• 'Todos los docentes son personas cultas", entonces:
Graficando:
DcC
• "Algunos docentes no son ingenieros", entonces:

n(D n 1) = x

Además no se especifica que todos los ingenieros


sean cultos.
Con las proposiciones anteriores construimos el gr.i-
Analizando las alternativas, con ayuda del gráfico: fico:
C
A) "Ninguna persona que toma té toma mate".
FALSO
Pues según el enunciado:
n(MnT)=x
B) "Todas las personas que toman mate toman
té". FALSO. Analizando las alternativas y comparando con los
Para que esta proposición se cumpla, debe cum- datos que nos proporciona el gráfico:
plirse: A) "Los ingenieros son cultos". FALSO
MnT=T ó McT Se entiende que todos los ingenieros son cul-
tos, lo cual es falso, pues:
C) "Algunas personas que toman mate toman
café". FALSO n(CnI)=x+y
Del gráfico: B) "Todos los ingenieros son docentes". FALSO.
n(MnC)=0 Del gráfico:

D) "Algunas personas que toman mate no toman té". n(I)=x+y+i


VERDADERO Los ingenieros docentes:
Del gráfico: n(ID)=x
n(M-T)=m
C) "Todas las personas cultas son docentes". FALSO
E) "Todas las personas que toman café toman té". Del gráfico:
FALSO.
n(C)=c+d+x+y
Para que esta proposición sea correcta, debe
n(D)=d+x
cumplirse que:
CnT=T ó CcT No se cumple la proposición: Ce D
~~;=-:::=:I UN12004-11 APT. ACADÉMICA Y CULTURA GENERAL 1;r:-!~:;!:~;:::'({;';l=j!J.I~'u
D) "Algunas personas cultas no son ingenieros".
VERDADERO SUCESIONES NUMÉRICAS
Del gráfico: n(C) = e+d+ x +y 16. Calculando los valores de A y B:
n(I)=x+y+i
E) "Los que no son ingenieros no son cultos". FALSO
Para que se cumpla esta proposición, también
debe cumplirse:
InC=0
En el gráfico se observa que: A=81x3=243
B=27x3=81
n(I n C) = x + y
Calculando la expresión:
Clave: D
A = 243 = 3
15. En la proposición: "Todos los insectos son in- B 81
vertebrados" Clave: C
Invertebrados: V 17.
Insectos: 1
1 9 19; 33 ; 53 ; 81 , x
'-------""
<c.:> <c.::« <c:> <c:> <c:>
+8 +10 +14 +20 +28 +38
<c.:> '-------""
<c:> <c.:> <c.:>
+2 +4 +6 +8 +10

x=81+38
= 119
Analizando cada una de las poposiciones y compa-
rándolas con la proposición de la premisa. Clave: B
I) "Es verdad que ningún insecto es invertebrado". 18. Analizando la sucesión:
INCORRECTA
J1,
Según esta proposición:
InV=0 (Contradicción) 1 . 2 . 4 . 8 . (i'6( 26 . 42 . 64 . 93
~~~~'~~~~

Según el gráfico: +1 +2 +4 +7 +11 +16 +22 +29


~~~~~~~
InV=I +1 +2 +3 +4 +5 +6 +7

11) "Es cierto que algún insecto es invertebrado". El quinto número de la sucesión (16) debe ser re-
emplazado por (15) para que ésta tenga la secuen-
CORRECTA
cia que se indica.
Según esta proposición:
Clave: C
n(I n V) of. 0 (correcta)
19. En la sucesión:
III) "Es falso que algunos insectos no son inverte-
+3 +3 +3 +3
brados". CORRECTO ~~~,..-----......,
Esta proposición puede expresarse así: "Es cier- 1 ; 2 ; 4; 8; 7 ; 14; 10 ; X ; Y
to que todos los insectos son invertebrados". ~~~
+6 +6 +6
Según esta última proposición: 1 e V .Ia cual
puede verificar en el gráfico que es correcta. X=14+6=20
Clave: E y = 10 + 3+ = 13
m: e;:~=r:=::<;;;,;!;=1 UN12004-11APT. ACADÉMICA Y CULTURA GENERAL ,=:=¡::~:~
En la expresión: Reemplazando t B = 2 en (3):
X+Y=20+13
= 33 ~X = 2( 40 )( 2)
Clave: E
~ x=200(km)

RAZONAMIENTO MATEMÁTICO Clave: B


20. Rapidez.- Se define como la distancia reco-
21. Las agujas de un reloj reinciden su encuentro
rrida en una determinada unidad de tiempo, es cada doce horas, es decir:
decir:
Rapidez = v = 4.t ~ d = vt ... C') 12 horas = 12 h (60~~n)
= 720 min ... (1)
Del enunciado, si Carlos mantiene la rapidez "v" en
todo su recorrido, entonces emplea un tiempo t1 . Por condición del problema se sabe que el reloj se
adelanta 4 min cada 6 horas, entonces:
__ -=__ -"Ati ~'I

6h ~ 4min

A~.----L:{~-.-v--------~~lB
----x
f-I -----.11
24h ~ x

(24h)x(4min) 6 .
Dato1: v = 40 (km/h) x= 6h =1 mln

De (*): x=vt1=40t1 ... O) ~ 24h=ldía <> 16min


Además del enunciado se sabe que: Como el reloj necesita 720 minutos para coincidir,
t2 entonces:
A
r '1
16min ~ 1día
:~~ t{l ,:
I v ~ 2V I 720min ~ x
~. -- -. .... iB

.,
b-f~~X~ x =
720min xl día
16 min

Dato 2: tI - t2 = 2 (horas) = 4S días

Por (*): Clave: O

... (2)
22. Sea "v" la velocidad normal de Renzo y "t" el
tiempo que emplea para recorrer la distancia
... (3)
d = 600m para llegar a su casa.
x = v( t A + 2t B ) ___ ~_-"At ~,I

Por dato 1:

De O):
x=40(tA+2tB)

40t1 = 40(tA + 2tB)


f~ :
----- d -----
t1=tA+2tB
d = vt ... e")
Del gráfico:
Por condición del problema, un día incrementa su
velocidad en 10 mlmin y llega a su destino S mi-
Por dato 2: nutos antes, es decir:
A;:==rr::W:=1
~
UNI 2004-11 APT. ACADÉMICA Y CULTURA GENERAL 1::;~=:;~:~:1::r GmeZ
CTi

d = (v + 10)( t - 5 ) 24. Por la condición del problema, las bacterias


se duplican cada 30 segundos y a las 9:59 h existen
De (*): (vt) = (v + 10)(t - 5) 50 millones. A las 10:00 h existirá.
vt=vt-5v+10t-50 30 s 30 s
=> v=2t-10 ~~
9:59 h 9:59:30 h 10:00 h
Clave: O t~t~t
50 millones 100 millones 200 millones
23. La vaca produce leche: (50x2) (100x2)

20· litros! día soleado Clave: C


16 litros! día nublado 25. El tiempo que emplean Juan y Pedro en cada
Si le canta a la vaca produce 2 litros más por día, vuelta:
ya sea día soleado o nublado. 5 min
Juan:
vuelta
Por condición del problema se sabe que durante 30
días se recolecta 590 litros, con 10 días nublados y Pedro: 450-_s- = 45 ~
20 soleados; luego por condición: vuelta 6 vuelta
En los días soleados produce (sin cantarle): El número de vueltas que habrá dado cada uno de
ellos en 30 minutos:
Ps = 20días( 20 la) 30min
VJ = . = 6 vueltas
= 400L 5~
vuelta
En los días nublados produce (Sin cantarle):
V = 30 min = 4 vueltas
p 45 min
PN = 10días( 16 d7a) ""6 vuelta
= 160L La diferencia de vueltas en los 30 minutos:
Su producción en 30 días (sin cantarle) VJ - Vp = (6 - 4 )vueltas
P = Ps + PN = 2 vueltas

= 400L + 160L Clave: O


= 560L
La produccion de la vaca durante los días que se le RAZONAMIENTO VERBAL
canta:

Producción real = 590 L ORACIONES INCOMPLETAS


P=560L} (-) 26. "Las culturas que son heterogéneas y poco in-
tegradas encierran muchas contradicciones".
Días cantados = 3OL
Clave: O
Como la vaca produce dos litros más pordía que le
cante: 27. "Muchos dicen que las novelas mienten, pero
ésa es sólo una parte de la historia; la otra es que
Días cantados 30 L
así expresan una curiosa verdad".
L
2día Clave: B
= 15días
28. "La indiferencia ante el sufrimiento y la tole-
Clave: C rancia hacia actos injustos han sido diligentes alin-
dos de las más terribles tragedias humanas".
?~~
GomeZ
~§~~~=I UNI 2004-11 ApT ACADÉMICA Y CULTURA GENERAL I
La "indiferencia" al sufrimiento de los pueblos, y 34. El ESTUDIO DE LAS ESTRELLAS
con un entorno de injusticia se tendrá como con- IV) F<rudio de astrónomos antiguos· Genérico
secuencia reacciones de violencia social.
V) Estudio de cultura árabe - particulariza
Clave: 8 I) Consecuencia: nombres árabes de las estrellas
Ill) También existen estrellas con nombres latinos
COMPRENSiÓN DE LECTURA II) Ejemplo de nombre Latino
29. Se cuestiona la formación de un egresado de Clave: D
la universidad dentro del contexto social. Los pro-
fesionales deben tener una formación humanistica. 35. LOS PODERES DEL ESTADO
I) Definición de todos los Poderes del Estado.
Clave: C
III) Define el Poder Ejecutivo - Funciones

30. El enunciado que no corresponde a la lectura: II) Define el Poder Legislativo· Su función es fis-
calizar al anterior (Poder Ejecutivo).
"Descartes se fundamenta en las conjeturas para
V) Elementos del Poder Legislativo.
investigar" .
IV) Por último se da ha conocer la existencia del
La lectura afirma que Descartes propone una in- Poder Judicial.
vestigación a base de certezas y no de conjeturas.
Clave: C
Clave: A

31. La lectura afirma que la TV influye en la for- ELIMINACIÓN DE ORACIONES


mación integral de los niños, dándoles una imagen 36. El tema es "Aleación de metales":
definida como son los valores de la sociedad. Si en
La oración 1define la aleación, y la Il, III yV nos da
la TV se difundiera que las niñas asuman las tareas
ejemplos de aleaciones. En cambio, la oración IV
de los varones, entonces se estaría cambiando el
no trata de aleaciones, si no sobre la historia del
modelo de vida social (esteriotipo social).
uso de un metal (acero).
Clave: 8 Clave: O

COHERENCIA Y COHESiÓN TEXTUAL 37. El tema sobre el que tratan las oraciones es
sobre la "loza mayólica".
32. LA ROBÓTICA
La oración IlI: "Los ingredientes que se utilizaban
V) Define el tema para preparar la pasta son arcillas calcáreas mez-
I) Complementa la definición cladas con arena fina"; si bien es cierto que trata
Ill) Cuando comenzó a usarse la robótica sobre los ingredientes para preparar la loza mayó-
IV) El principal usuario de la robótica lica, ésta no hace mención sobre ésta.
Clave: C
II) Consecuencias
Clave: 8 38. El tema que trata las oraciones es sobre la
evolución de la bicicleta, desde su aparición en el
33. EL RADAR siglo XIX hasta el siglo XX.
II)Definición La oración V: "En la segunda mitad del siglo XX, las
V) Aplicación bicicletas aumentaron su difusión debido al inte-
III)Principal modelo - de impulsos rés de las personas de responder a la moda y cui-
dar su estado físico". Esta oración, a pesar que tra-
I)Empleo de principal modelo- Tipos: reconoci-
ta sobre la bicicleta no corresponde al grupo gené-
miento y exploración
rico, pues mientras en ésta se ocupa de ella duran-
IV) Descripción del radar de exploración. te el siglo XX, las otras lo hacen en el siglo XIX.
Clave: A
Clave: E
I UN12004-11 APT. ACADÉMICA Y CULTURA GENERAL l
39. El tema que trata las oraciones es sobre el 45. INFARTO, enfermedad que se caracteriza por
estudio de la inteligencia desde la perspectiva de la degeneración de un tejido por obstrucción
la Psicología. de la circulación sanguínea. Frecuentemente
La oración (I) no tiene relación con el tema men- ataca al corazón.
cionado porque ésta trata sobre "el significado de CORAZÓ ,órgano del cuerpo humano.
la inteligencia para la cultura occidental". La relación que existe es de nombrar una enferme-
Clave: A dad (infarto) de un órgano (corazón), y en las al-
ternativas la que mantiene una similar relación es
cirrosis enfermedad del hígado (órgano).
CONECTORES LÓGICOS
Clave: E
40. Aunque la protección del medio ambiente es
tarea de todos, no obstante muchas industrias 46. DIVULGAR: Publicar una cosa que estaba ig-
son indiferentes porque generan gases tóxicos, y norada. Hacer público un secreto.
así ocasionan mayor daño a la naturaleza.
TRANSMITIR: Transferir comunicar
Clave: A
Estas palabras mantienen una relación de sinoni-
41. Para la ingeniería, el desarrollo no es un mito mia, y en las alternativas, las que guardan ésta
ni un conjunto de indicadores crecientes, sino relación son la A, D Y E, de éstas, la primera es la
una tarea diaria donde las obras constituyen la base que contiene el par de palabras con una significa-
del desarrollo; por eso el docente debe desarro- ción más directa: propalar: difundir.
llar la creatividad en los estudiantes de esta área. Clave: A

Clave: B
TERMINO EXCLUIDO
42. Aunque el prestigio es un elemento subjeti- 47. CORTO: De poca extensión o duración
vo e intangible del estatus social, sus miembros son
clara y universalmente reconocidos. En las alternativas la palabra que no guarda una
relación significado es nimio, cuyo significado es
Clave: B excesivo abundante.
43. Pese a que el nivel de comprensión lectora Clave: C
de los niños en nuestro país es crítico, muchas au- 48_ GENEROSO: Persona humanitaria, bondado-
toridades educativas no se preocupan; por eso es so, etc.
momento de tomar alternativas de solución aun-
que ello genere mayores gastos al Estado. La palabra que no guarda relación de significado
es acendrado, cuyo significado es puro, sin man-
Clave: D chas ni defectos.
Clave: D
ANALOGíAS
49. PETULANTE: Vana y ridícula presunción. In-
44. EXTROVERTIDO, persona que exterioriza sus solencia.
pensamiento.
La palabra que no guarda relación de significado
INTROVERTIDO, persona que pasa sumida en es obstinado que significa terco, o persona que
sus pensamientos. porfía con necedad y pertinencia.
Las palabras bases en su significado, gu-ardan una Clave: C
relación de antonomía. En las alternativas existen
dos pares de palabras que guardan esta relación, la 50. MANIFIESTO: Declarar, dar a conocer.
correspondiente a la alternativa E, y la B que es la La palabra que no mantiene la relación de signifi-
que sus palabras guardan un significación más di- cado es compacto, pues su significado es denso,
recta: expresivo y tímido. apretado.
Clave: B Clave: C

_1
~0~~~~::~::!1
V~~
UNI 2004-11 APT. ACADÉMICA Y CULTURA GENERAL l;:i~!=~=:::=~~
~

COMUNICACiÓN Y LENGUAJE
CULTURA GENERAL 54. Diptongo: Unión de dos vocales diferentes
en una misma sílaba.
HISTORIA UNIVERSAL En el texto:
51. "Las viviendas construidasmayormente de ma- "La ciudadanía disfrutaba de una de las más bri-
dera, levantadas en plataforma sobre lagos, lagu- llantes obras teatrales realizadas a campo abierto;
nas o suelos húmedos, y que caracterizan el perio- mejor regalo no podía recibir en el día del aniveg-
do Neolítico, se conocen con el nombre de sario patrio".
palafitos "..
Existen cuatro diptongos.
Estas viviendas se construyeron, en el periodo Clave: A
Neolítico, en el Norte de Europa, Bélgica y Suiza.
Su construcción contribuyó a que el hombre dejara 55. Hiato: Encuentro de dos vocales que se pro-
de ser un nómade, dedicándose a la agricultura; nuncian en sílabas distintas.
razón por la cual se le conoce como la "Revolución En las alternativas, la que tiene mayor cantidad de
Neolítica". hiatos:
Clave: B "Ra-úl no o-í-a lo que se le decí-a"
Clave: B
52. Los factores que contribuyeron a la cultura
Medieval fueron: 56. "Ayer me animó con su buen ánimo, pero
la verdad es que aún no me animo a ir de paseo".
Cristianismo: Esta religión, lo impusieron y ex-
tendieron en las Parroquias y Universidades. Animó: Del verbo animar, tercera persona, tiempo
pasado.
Greco y Latino (Roma): El pensamiento de
Ánimo: Sustantivo.
Platón y Aristóteles definieron la filosofía de esta
cultura. Animo: Del verbo animar primera persona, tiempo
presente.
Germánico: Las costumbres de los caballeros, la
fidelidad hacia los amos, etc. Clave: B
Clave: C
57. La acentuación gráfica diacrítica se usa para
diferenciar palabras de igual escritura pero de di-
53. La Organización de la Naciones Unidas (ONU)
ferente función gramatical. Ejemplo:
en su carta Constitutiva de 1945; en el artículo 2
del capítulo 1 afirma: "Él es un peruano ejemplar".

"La organización está basada en el principio de la Si fuera un artículo se escribiría sin. acentuación
igualdad soberana de todos sus miembros". En el gráfica (El); pero es este caso está haciendo el pa-
apartado 4to. del mismo artículo 2 también men- pe~ de pronombre (Él).
ciona: " Los miembros de la organización, en sus Clave: A
relaciones internacionales; se abstendrán en recu-
58. "Con la seguridad de que (esta) nuestra invi-
rrir a la amenaza o al uso de la fuerza contra la
tación tendrá (su) favorable acogida en usted, y, a
integridad territorial o la independencia política de
la espera de su respuesta, me (le) valgo de la opor-
cualquier Estado". tunidad para reiterarle los sentimientos de mi ma-
En ninguno de sus principios menciona sobre las yor consideración".
"medidas preventivas". Las palabras: esta, su y le, deben ser borrados
Clave: E Clave: B
;~~~~.'~ll'Z~KI UNI 2004-11 APT. ACADÉMICA Y CULTURA GENERAL l ~I~
'hi.

ConleZ

63. Entre 1885 y 1929, la región Amazónica se


GEOGRAFíA convertió en un lugar de especial interés económi-
co y geoestratégico, debido a la gran demanda del
59. La cordillera de los Andes, cadena de monta- caucho y la shiringa con látex en la industria auto-
ñas, genera cuencas hidrográficas en toda América movilística en EE.UU de Norteamérica y Europa.
del Sur. En el Perú crea tres regiones hidrográficas:
La del Pacífico, la del Lago Titicaca y la del Ama- Clave: A
zonas.
I) La vertiente
del Pacífico tiene como divisoria
PSICOLOGíA
de aguas la cadena occidental de los Andes.
CORRECTO 64. Según el Psicoanalista Freud, el aparato psí-
II) La Hoya del Titicaca incluye los ríos que des- quico tiene tres instancias:
aguan en ese lago. CORRECTO Consciente: Es la que nos permite mantenernos
IlI) La Región hidrográfica del Amazonas está for- en contacto con la realidad.
mada por los ríos que vierten sus aguas en el Pre-consciente: Es la que alberga la información
río Amazonas y finalmente en el Océano Pací- que fácilmente se puede recordar.
fico. CORRECTO.
Inconsciente: Alberga el conjunto de pensamien-
Clave: D tos, fantasías impulsivas y contradictorias, eventos
traúrnaticos y desagradables que pugnan por ser li-
60. Si bien es cierto en el Perú el 70% de la berados y satisfechos de inmediato de alguna forma.
población vive en zonas urbanas, todavía existen
algunos departamentos que existe predominio de Clave: C
zonas rurales como Cajamarca, Piura y
65. Según D. Goleman, uno de los mejores expo-
Huancavelica.
nentes, la inteligencia emocional nos facilita la
Clave: E habilidad de tener el autodominio, la persistencia y
la capacidad de motivamos a uno mismo; así como
61. Las ocho regiones naturales que clasifica Ja-
también la facilidad de relacionamos con los demás
vier Pulgar Vidal esta basada en la Altitud. a partir del reconocimiento de si mismos.
6770
(Altitud en metros) Clave: B

FILOSOFíA Y LÓGICA
66. La falacia es un razonamiento con error en su
estructura o en su coherencia lógica, generando
como consecuencia, argumentos engañosos que ira-
o tan de convencer sobre una situación determinada.

Clave: C En la expresión:
"No aprobé el examen anterior, porque los miérco-
62_ Las tres grandes llanuras del América del Sur les todo me sale mal".
son:
Este tipo de falacia se llama Falacia de Causa por
Llanura Amazónica: Abarca la gran mayoría de estar fundamentado es una superstición o creencia.
los países de Sudamérica (Selva Amazónica).
Clave: D
Pampas y el Gran Chaco: Abarca los países del
Paraguay y Argentina. 67. En el método científico, a las Conjeturas for-
Llanos del Orinoco: Se encuentra en Venezuela. muladas para dar razón de los hechos se le deno-
mina Hipótesis.
Clave: E
Clave: C
• En 1 948, donde Manuel Odría derroca al go-
bierno de Bustamante y Rivero.
HISTORIA DEL PERÚ ( Y ACTUALIDAD)
• En 1 968, Juan Velasco Alvarado depone al go-
68. Las culturas Pre-incas, Jhon Rowe los clasifi- bierno de Fernando Belaunde Terry.
có en:
• En 1992, Alberto Fujimori sea da su autogolpe.
Horizontes: Proceso de centralización política.
Clave: C
Intermedio: Proceso de desarrollos regionales.
73. Las medidas que se tomaron durante el go-
Cronológicamente las culturas existieron:
bierno de Juan Velasco Alvarado fueron:
Chavín, Paracas, Vicus: Horizonte Temprano
• Nacionalización del petroleo, creando la em-
Nazca, Moche, Lima: Intermedio Temprano. presa estatal Petroperú.
Tiahuanaco, Wari: Horizonte Medio • Se Promulgó la ley de Reforma Agraria, bene-
ficiando por lo menos al 30% de los campesi-
Chimú, Chincha, Chancay, Chachapoyas, etc. In-
nos peruanos.
termedio Tardío.
• Otras de las medidas fue la apropiarse de los
Clave: D medios de comunicación con el fin de contro-
lar las masas.
69. El Imperio incaico se instauró después de que
Clave: C
derrotaron a los chancas, siendo el Inca Cusi
Chupanqui (Pachacutec), que además de iniciar la
ECONOMíA POLíTICA
reorganización social y política, se expandió hasta
la meseta del Collao y norte del Cuzco. A su muer- 74. En el Perú, la instancia que aprueba el Presu-
te, su hijo, Tupac Yupanqui extendió las fronteras puesto Nacional es el Congreso de la Repúbli-
del imperio por el norte hasta Quito, por el sur has- ca, pero éste es elaborado por el poder Ejecutivo y
ta Maule. debe presentarlo a mas tardar el 30 de agosto de
cada año.
Clave: B Clave: C
70. Bolivar desarrollo un proyecto integracionista 75. El documento que registra todas las transac-
en respuesta al intervencionismo de las potencias ciones económicas de un país con el resto del mun-
de las épocas, proyecto que se intento materializar do se denomina Balanza de Pagos, la cual es
en el Congreso de Panamá (1824), luego en elaborada por el Banco Central de Reserva (BCR).
Tacubaya (México). Además se venía planteando
la Federación de los Andes entre los países que Clave: B
él había independizado (La Gran Colombia, Perú y
Bolivia), basándose en la Constitución Vitalicia. 76. En las afirmaciones:
I) "En condiciones de libre mercado, el aumento
Clave: B
de la oferta de un bien conduce a una dismi-
71. En el periodo de la Reconstrucción Nacional nución de su precio". VERDADERA
(Después de la Guerra con Chile) en la costa norte En un libre mercado, un aumento de la oferta
se incrementa el cultivo de la caña de azúcar, implica mayor abundancia del bien y como con-
dando una re activación a la agro-industria y propi- secuencia de ello la disminución de su precio.
ciando la aparición de los ''Varones del azúcar" como II) "En condiciones de libre mercado, el precio de
los Gildemmeister, Larco, Grace, etc. que monopo- un bien garantiza márgenes de ganancia para
lizaron y lograron tomar el control del país. sus productores". FALSA
Clave: A En un libre mercado, los precios de un bien
deben ser competitivos, y muchas veces estos
72. En las últimas seís décadas, debido a las crisis pueden estar por debajo de los costos de pro-
políticas y económicas, en el Perú se instauraron ducción, generando pérdidas para los produc-
los regímenes autoritarios: tores.
IlI) "En condiciones de monopolio y oligopolio en la Esta cita corresponde del segundo monólogo
producción de un bien, su precio tiende a cre- de Sigismundo, personaje de la obra: "La vida
cer". VERDADERA es un sueño".
La producción de un bien, dentro de un am- Clave: O
biente monopólico y oligopólico, tiende a cre-
cer debido a que no tiene la libre competencia. 79. En las alternativas
Clave: A 1. "El harawi de la Literatura Inca es el canto de
amor y tristeza". CORRECTO
77. El "Total de bienes y servicios finales produci-
El harawi es una composición quechua de ori-
das dentro de un país, en un periodo determinado,
gen popular que refleja el amor a los anima-
valorizados a precios de mercado" se mide mediante
les, tierra, seres queridos, etc.
el Producto Bruto Interno (PBI), el cual se calcula
así: II. "El amor doliente es el tema central de los
yaravíes de Mariano Melgar". CORRECTO
PBI=C+I+G+(X-M) Los Yaravies de Maríano Melgar son deriva-
Donde: ciones del harawi, conservando su esencia de
tristeza y amor.
C : Consumo.
III. "José María Arguedas en su prosa tierna y poé-
I : Inversión.
tica, eleva a categoría literaria el Castellano
G : Gastos del gobierno. Andino" CORRECTO.
: Exportaciones.
X
Este autor utiliza el castellano andino en sus
M : Importaciones. diferentes obras en un estilo tierno y lírico,
X - M : Exportaciones netas como en Los ríos profundos.
El PBI es un agregado económico que muestra las Clave: A
principales actividades económicas de un país.
80. Las obras de los escritores hispanoamericanos:
Clave: E
1.José Luís Borges: Fervor de Buenos (Poe-
sía), El libro de arena, Ficciones, El aleph, (e)
LITERATURA Historia de la eternidad, etc.
78. En las alternativas: II. Grabiel García Márquez: (a) La mala
1. "En un lugar de la Mancha ... " Miguel de hora, Ojos de perro azul, Los funerales de la
Cervantes Saavedra. CORRECTO Mama Grande, Cien años de soledad, El otoño
Esta expresión corresponde a la novela titula- del patriarca, etc.
da: "El Ingenioso Hidalgo Don Quijote de la III. Pablo Neruda: Veinte poemas de amor y una
Mancha" canción desesperada, Residencia en la tierra,
II. "é Qué es poesía? ¿y tú me lo preguntas?". Gus- Los versos del capital, (b) Odas elementa-
tavo Adolfo Bécquer. CORRECTO les, Cien sonetos de amor, etc.
La expresión corresponde a una de sus in- Clave: C
mortales "Rimas".
III. "é Qué es la vida? un frenesí". Pedro Calderón
de la Barca. CORRECTO
. 1

1;'J~PTITUD
...•
ti'
~~[JtlWª~~ . i .....•

ACADÉMICA y CULTURA GENÉiRA


Indique cuál o cuáles de las afirmaciones son ver-
APTITUD ACADÉMICA daderas.
A) Sólo n B) Sólo III C) II Y III
RAZONAMIENTO MATEMÁTICO

v~
D) 1 Y n E) 1 Y III
1. Si:
4. Identifique la alternativa con la figura que com-
pleta la serie.
I 2m+n-<j I y

determine
ITJ = 3b-1
el valor de:
EJ~~D
~[j~~WA) B) C) D) E)

5. Cuál de las alternativas tiene la figura que debe


é

ocupar el casíllero en blanco?

A) 35 B) 39

2. Si ~(x - 3) = x + 7
C) 41 D) 44 E) 47
[i][rzJ][ili]D
Determine el valor de: E = ~(x - 2) - ~(x + 6)

A) -18 B) -10 C) -8 D) 12 E) 16 ~[Q]~~GJ


A) B) C) D) E)
3. En el conjunto de los números enteros (Z) se
define el operador * por: 6. Indique la alternativa con la figura que debe ocupar
la posición 9.
a * b = mín {a - b , b - a}
Sobre este operador * se afirma:
1. Es asociativo si
a*(b*c)=(a*b)*c ; Va,b,ceZ pocisión 1 pocisión 2 pocisión 3 pocisión 4 pocisión 5
Il. Es conrnutatívo si
a*b=.b*a; Va,beZ

1II. (-1)*(1*3)=-1
A) B) C) D) E)
7. Indique la alternativa con la figura que falta A) La información I es suficiente.
para que los pares guarden la misma relación. B) La información Il es suficiente.

1@1@11@171
C) Es necesario emplear ambas informaciones a
la vez.
O) Cada una de las informaciones, por separado,

@@@@@ es suficiente.
E) La información brindada es insuficiente.

11. En una tubería de 25 mm de diámetro inte-


A) B) C) O) E) rior, fluye agua, de manera que la tubería se en-
cuentra completamente llena.
8. Indique la alternativa con la figura que debe
Se desea determinar el caudal que circula por la
ocupar la posición x en la siguiente serie: tubería.
Información:
I. Se conoce el área de la sección transversal de
la tubería.

~B~~~
A) B) C) O) E)
Il. La velocidad del agua dentro de la tubería.
Para resolver el problema:
A) La información I es suficiente.
9. Enrique y Saúl estaban realizando una obra y
despúes de trabajar juntos durante 5 días, Enrique B) La información n es suficiente
se enfermó. C) Es necesario emplear ambas informaciones a
Si Enrique hubiese trabajado sólo cen cuántos días
la vez.
hubiese culminado la obra? O) Cada una de las informaciones, por separado,
es suficiente.
Información:
E) La información brindada es insuficiente.
I. Trabajando juntos la hubiesen acabado en 18
días. 12. La negación de "todos los rectángulos son
n. Trabajando juntos la hubiesen acabado en 18 paralelogramos", es:
días, pero como se enfermó Enrique, Saúl la A) Todos los rectángulos no son paralelogramos.
acabó en 45 días. B) Todos los no rectángulos no son paralelogra-
Para resolver el problema: mos.
A) La información I es suficiente. C) Algunos rectángulos no son paralelogramos.
B) La información Il es suficiente. O) Algunos rectángulos son paralelogramos.
C) Es necesario emplear ambas informaciones a E) Todos los no rectángulos son paralelogramos.
la vez.
13. Fico está al este de Daniel, Daniel está al nor-
O) Cada una de las informaciones, por separado,
te de Pedro, y Pedro está al sur de Toño.
es suficiente.
E) La información brindada es insuficiente. Carlos está al este de Daniel, Marco está al oeste de
Pedro, y Daniel está al norte de Toño. Entonces,
10. Juan vendió dos computadoras, cada una en podemos afirmar que.
$ 800. Se desea saber si Juan perdió o ganó en el
A) Carlos está al oeste de Daniel,
negocio.
B) Fico está al noreste de Toño.
Información:
C) Pedro está al norte de Fico.
I. En la primera computadora ganó el 25%
O) Marco está al norte de Toño.
n. En la segunda computadora perdió el 25%
Para resolver el problema: E) Carlos está al noroeste de Pedro.
14. Tres alumnos Ornar, Roxana y Xiomara, res- 18. Determine el valor de x + y .
ponden una evaluación con tres preguntas que tie-
nen las alternativas verdadera (V) o falsa (F). Sus
respuestas fueron reportadas en el cuadro adjunto.

Pregunta Omar Roxana Xiomara ,


1 V V F A) 12 B) 14 C) 16 D) 18 E) 20

2 V F F
19. Indique la alternativa que pertenece a la su-
3 F F V
cesión.
Se sabe además que uno de los alumnos contestó 2, S, 17, 71, ?
correctamente todas las preguntas, otro se equivo-
A) 189 B) 213 C) 288
có en todas sus respuestas y el restante falló sólo
en una pregunta. Indique el orden de mérito de D) 359 E) 393
dichos alumnos.
20. Un cuadro con su marco cuesta SI. 240. El
A) Roxana, Xiomara, Ornar
mismo cuadro con un marco que cuesta la mitad
B) Ornar, Roxana, Xiomara
del anterior, tiene un costo de SI. 180.
C) Xiomara, Ornar, Roxana
é Cuál es el costo, en soles, del cuadro sin marco?
D) Xiomara, Roxana, Ornar
E) Ornar, Xiomara, Roxana A) 80 B) 100 C) 120
D) 130 E) 160
15. Seis amigas, Ana, Bety, Celia, Doris, Eva y Lili
viven en un edificio de seis pisos, cada una en un 21. Si un kilogramo es la masa de 6 a 8 membri-
piso diferente. Si se sabe que: llos, écuál es la mayor masa, en kilogramos, que
- Eva vive entre Bety y Doris pueden tener 4 docenas de membrillos?
- Lili no vive en el último piso A) 6 B) 8 C) 10 D) 12 E) 16
- El cuarto piso está ocupado por Ana
- Ana vive entre Doris y Lili 22. Una hoja de papel de 1 S cm x 3 O cm se corta
Se puede afirmar que: en tiras de 1 mm de ancho. Cuál es la longitud, en
é

metros, que se obtendría al unir estas tiras en una


1. Celia vive en el sexto piso
sola tira de 1 mm de ancho?
Il. Bery vive en el tercer piso
III. Doris no vive en el tercer piso
A) 45 B) SO C) ss D) 75 E) 95

IV Bety vive en el primer piso


23. Federico vende 3 naranjas por un sol y Mi-
A) Sólo 1 D) Sólo I y IV guel, que tiene la misma cantidad de naranjas, las
B) Sólo I y n E) Sólo 1, n y III vende a 2 por un sol. Para evitar la competencia
C) Sólo 1 y III acuerdan asociarse y deciden vender las naranjas a
un precio que les reporte los mismos ingresos que
16. Determine el valor de z - x , en: si estuvieran separados. Por tanto, venderán.
2,7 , 6 ,9,12,13, x, z A) S naranjas por dos soles
A)7 B) 8 C) 9 D) 10 E) 11 B) 6 naranjas por tres soles
C) 7 naranjas por once soles
17. Indique la alternativa que pertenece a la su-

*'~'
cesión. D) 10 naranjas por dos soles
E) 12 naranjas por cinco soles
S , 13 , 30 , ?
A) SS B) 65 C) 67 D) 78 E) 81
• 158 ;;;;o¡. ; "'iPf5'ff?r:1 UNI 2005-1 APT. ACADÉMICA Y CULTURA GENERAL 1==J'~ GOllleZ

24. Se divide una viga de madera en cuatro par- e) irresistible - alcohólicas


tes iguales. Luego cada parte se divide a la vez en D) opcional - importadas
tres partes iguales y después cada parte se divide E) social - preferidas
en dos partes iguales.
Si la diferencia entre una de las primeras partes y COMPRENSiÓN DE LECTURA
una de las últimas partes es de 5 metros, écuál es la
29. "Desde su aparición, Patio de Letras, revista
longitud total de la viga en metros?
semestral de Investigaciones Humanísticas, se si-
A) 32 B) 30 e) 28 D) 25 E) 24 tuó no vale decir en la primera línea de las publica-
ciones nacionales de su género, que apenas si las
25. En un examen las respuestas a las cinco prime- hay, sino de las comprendidas en la denominación,
ras preguntas son: a, b ,e, d ,e. A las siguientes 10, más extensa, de revistas culturales ... Patio de Le-
son: a ,a ,b ,b ,e ,e ,d ,d ,e, e, a las siguientes 15, tras, nombre indesligable de la Universidad Nacio-
son: a ,a, a , b , b , b ,e ,e, e , d ,d ,d ,e, e ,e , y así su- nal Mayor de San Marcos, es vista equivocadamen-
te como un órgano oficial de esa casa de estudios.
cesivamente. Hallar la respuesta a la pregunta 90.
No es así ... se trata de una publicación "indepen-
A) a B) b e) e D) d E) e diente y plural", como ella misma se califica, pro-
ducida con el esfuerzo no sólo intelectual sino eco-
nómico de sus editores".

RAZONAMIENTO VERBAL Según el contenido del texto, Patio de Letras.


A) no es una revista de Investigaciones Humanís-
ORACIONES INCOMPLETAS ticas.
Elija la alternativa que completa adecuadamente B) es una revista cultural de primera línea.
el significado de la oración. e) es el órgano oficial de la UNMSM.
D) no es una revista independiente ni plural.
26. El es una actitud positiva o negativa E) requiere de un esfuerzo económico más que
hacia un grupo de personas por sus características intelectual.
sociales o .
A) amor - étnicas - físicas 30. "No parece coincidencia que el crimen cobar-
de y alevoso de tres policías - atribuible en princi-
B) desdén - latentes - existentes
pio a grupúsculos senderistas - se haya producido
e) escarnio - pasadas - presentes a pocos días de una operación policial que destru-
D} cariño - pacíficas - violentas yó 20 pozas de maceración. Tampoco que luego,
un sector de cocaleros persistiera en una huelga
E) prejuicio - reales - imaginarias
para protestar por varios asuntos, desde la supues-
ta depredación del ambiente (debido a las accio-
27. La autoridad es el que tiene una nes de erradicación de coca ilegal por parte del
persona sobre otra que le está . Estado) hasta la oposición ciega y sorda al '¡'Le con
A) amparo - juzgada Estados Unidos, que es un socio importante en la
lucha antinarcóticos".
B) carisma - admirada
De la lectura del texto, se infiere que el asesinato
e) estilo - encomendada
de los policías.
D) poder - subordinada
A) es una señal de una política militar del Estado,
E) recurso· fijada
B) generó un rechazo tetal por parte de los
cocaleros.
28. La dipsomanía es la necesidad de to-
e) condujo a la operación militar destinada a to-
mar bebidas .
mar pozos de maceración,
A) destructiva - gaseosas D) tuvo implicancias favorables para la fi-rna del
B) irracional - heladas tratado del TLC.
~::;-I UN12005-1 APT. ACADÉMICA Y CULTURA GENERAL I~;F::;::~
E) obedece a la alianza narcoterrorista existente 33. EL PAPA ENFERMO
en la selva. 1. Desde su último internamiento sólo se han di-
31 . El postmodernismo postula la naturaleza esen- vulgado tres partes médicos, el viernes 25, el
cialmente híbrida del mundo, rechazando la posibi- lunes 28 de febrero, y el viernes 3 de marzo.
lidad de categorías puras de ninguna clase. Es un II. El médico personal decidió, l4 días luego del
mundo de matrimonios mixtos: entre las palabras y inicio del problema, que se le someta a una
las cosas, el poder y la imaginación, la realidad ma- traquetomía.
terial y la construcción lingüística. Llevada al límite, III. Las complicaciones respiratorias empezaron el
la deconstrucción expresa nuestro sentido de la na- 1de febrero con un cuadro de laringotraqueitis
turaleza discontínua, fragmentada y fracturada de aguda.
la realidad cuya condición incierta queda represen- IV Los partes no los firman los médicos del
tada por el uso persistente de las comillas. policlínico, sino el portavoz papal Joaquín Na-
Señale el enunciado no compatible con el conteni- . varro Valls, quien es además médico.
do del texto. V. Lo volvieron a internar, para la intervención,
A) Para el postmodernismo, la realidad es en el policlínico Gemelli de Roma el 24 de fe-
discontinua y fragmentada. brero.
B) Se vive una época de distinción patente entre A) 1 - IV - III - II - V
las palabras y las cosas. B) II - IV - I - III - V
e) El postmodernismo postula la naturaleza e) III - II - V - I - IV
híbrida del mundo.
D)IV-I-III-I1-V
D) Las categorías puras son inexistente s para el
postmodernismo E) V - I - IV - TI - III
E) Según el postmodernismo, se vive una época
de incertidumbre. 34. LA TEORlA DEL BIG BANG
1. No obstante estas explicaciones detalladas, la
teoría del Big Bang contiene algunos datos con-
COHERENCIA Y COHESiÓN TEXTUAL trovertidos.
Tomando como referencia el título, elija la alterna- Ir. La expansión se originó a partir de un estado
tiva que presenta una secuencia adecuada de las primordial extremadamente denso y energético.
oraciones para que la estructura global del texto
III. Big Bang (gran explosión) es la teoría sobre el
resulte coherente y cohesiva.
. origen del Universo hoy aceptada.
32. ASESINAN A 8 EN TRUJILLO IV El estado espeso de energía logró alcanzar las
dimensiones y temperaturas que tiene hoy.
1. Domingo 27 de febrero, 22 horas, llega una
camioneta amarilla. V. Según ésta, desde hace unos 15 mil millones de
años, el Universo ha venido expandiéndose.
II. Entran al bar y disparan a 9 parroquianos en
una mesa. A) II - III - V - 1 - IV
III. Bajan unos 15 sujetos encapuchados y armados. B) III - V - I - IV - II
IV Bar "El Barro", distrito de la Esperanza, Trujillo. e) III - V - II - IV - I
V. Mueren 8 personas y una queda herida. D) IV - II - III - V - I
A) I - IV - II - III - V E) V - III - II - I - IV
B) II - V - I - III - IV
35. EL MOTOR TURBORREAeTOR
e) III - IV - II - I - V
1. La propulsión se produce por la fuerza de los
D) IV - I - III - II - V -gases de escape al salir por la parte posterior.
E) V - I - II - III - IV Ir. El aire entra por el frente y pasa a través de los
compresores.
16::-0
~
g;:===:1 UNI 2005-1 APT. ACADÉMICA Y CULTURA GENERAL I~~~:=~~~i~ G~m.Z

III. Un motor turborreactor es algo más que un tubo 38.


hueco. I. La idea de que "lo único constante es el cam-
IV Éste se mezcla con el combustible en la cámara bio" aún se remonta a los tiempos de Heráclito.
de combustión y luego se inflama. II. Hoy, algunos gerentes nos hablan de transfor-
V. Los motores de turbina de reacción son común- mación para referirse a las iniciativas de cam-
mente conocidos como motores turborreacto- bio organizacional como la de General Electric.
res. Ill. En el mundo de los negocios y las finanzas,
A) V - III - II - 1 - IV "Cambio" significa varias cosas, a veces
contradictoras.
B) IU - V - II - 1 - IV
IV A veces se refiere a los cambios externos en
C) V - III - II - IV - 1
tecnología, competencia de mercado, inclusi-
D) III - V - 1 - IV - II ve al ambiente socio político.
E) II - IV - 1- V - III V. "Cambio" también hace referencia a los cam-
bios internos y a la preocupación en si éstos se
mantendrán al ritmo de los cambios externos.
ELIMINACiÓN DE ORACIONES A) 1 B) II C) III D) IV E) V

Elija la oración que es redundante o no tiene rela- 39.


ción con el tema desarrollado en el texto.
I. Las primeras civilizaciones se enfrentaron con
36. problemas de cantidades, números y relacio-
I. La lengua es un conjunto de signos arbitrarios
nes matemáticas que resolvieron en forma
que utiliza una comunidad para comunicarse.
empírica.
II. Emisor es la persona o lugar desde donde par-
II. El surgimiento de las matemáticas es una con-
te la información.
secuencia de la necesidad de solucionar pro-
Ill. Receptor es quien recibe la información. blemas diversos de carácter pragmático.
IV Canal es el medio físico a través del cual se Ill. Con el transcurso del tiempo, los egipcios re-
envía el mensaje.
solvían este tipo de operaciones mediante re-
V. Contexto o situación 'son las circunstancias del soluciones numéricas razonadas y exactas, ba-
lugar y momento en que se produce la comuni- sadas en un sistema denumeración decimal.
cación.
IV En la Mesopotamia antigua, la civilización
A) 1 B) II C) III D) IV E) V babilónica creó un sistema sexagesimal y con
el tiempo inventaron tablas de multiplicar, de
37. dividir y de otros cálculos.
I. El carbón es producto de una lenta descompo-
sición de los vegetales durante millones de V. Los griegos se basaron en las ideas de los egip-
años. cios y babilonios, pero su aporte más impor-
tante fue la creación de una lógica que permi-
II. El carbón es una roca sedimentaria de color
negro compuesta principalmente de carbono. tió el enunciado de axiomas y demostraciones.

Ill. El carbón ha sido desplazado por el petróleo A) 1 B) II C) III D) IV E) V


como combustible fósil.
IV Las variedades de carbón difieren en el por- CONECTORES LÓGICOS - TEXTUALES
centaje de carbono que contienen. Marque la alternativa que, al insertarse en los es-
V. El carbón más puro, el grafito, contiene 95% pacios en blanco, dé un significado adecuado al
a 98% de carbono. enunciado.
A) 1 B) II C) III D) IV E) V 40 los turistas sólo van al cuzco,
el Gobierno ha creado un programa cultural
~~,di~~~;!0":,?'Mi::':::;=1 UN12005-1
ep........ ·:ti:¡:.dlmtmtttfUmMt~>~~~';::;~~&M::«:::
:t,
(;nmeZ
APT. ACADÉMICA Y CULTURA GENERAL.~&:w::;:m:~M~:'~:'~
:;:;- 161':
..J~m~:::::L,AA%~:::::~,ill'~N~ ,..:':::

................. difundir otras zonas turísticas;


................... , ellos podrían apreciar la 'riqueza cul- ANALOGíAS
tural del país mejorarían la condición
socioeconómica de sus pobladores. Teniendo como referencia la relación existente en-
A) Debido a que - para - así - y tre las palabras escritas en mayúsculas, elija la al,
ternativa que contenga dicha relación analógica, o
B) Porque - con el fin de - entonces -o
que continúe dicha secuencia.
C) A pesar de que - por - entonces - pues
D) Si bien - y - así - o 44. ACRISOLAR : ENSUCIAR

E) Pues - para - en tal caso - ya que A) oscurecer malograr


B) mezclar purificar
41 el resultado del último encuentro C) acendrar macular
no fue el esperado, la gente abriga espéranzas para
D) alterar limpiar
el próximo partido, en el fútbol, como
en otras actividades, los hechos resultan imprede- E) erosionar depurar
cibles la clasificación para el mundial
no deja de ser una posibilidad. 45. ARQUEOLOGÍA: RUINAS; BOTÁNICA:
A) A pesar de que - ya que - por lo que VEGETALES; CONTABILIDAD: COSTOS;
B) Dado que - y - si
C) Si bien - es decir - y A) biblioteca : literatura
D) en vista de que - aunque - o B) deontología : dientes
E) Aunque - pues - pero C) derecho : leyes
D) lenguaje : caligrafía
42. Expresar una posición política resulta hoy no
E) números : matemática
bien vista el hombre público la ha co-
rrompido; ello no debe asombramos,
46. ZURDO: DIESTRO; VANIDAD·: HUMILDAD;
.............. dicha animadversión es sólo coyuntural.
RUDEZA: CORTESÍA; .
A) y - pero - si bien
A) compensar : remediar
B) pues - entonces - ergo
B) empatar : igualar
C) porque - mas - dado que
C) hablar : parlar
D) vale decir - por eso - pero
D) fino : suave
E) empero - aunque - pues
E) nada : todo

43. Las víctimas hacían cola aprobar un


control médico y postular a un empleo público, TÉRMINO EXCLUIDO
............. : quedaron paralizadas por la explosión. Elija la palabra que no guarda relación de signifi-
A) o - por lo que cado común con las otras y con la palabra base.
B) para - cuando 47. MALO
C) por, pero A) malvado D) ruin
D) sin - entonces B) haragán E) bribón
E) y - Y C) perverso
~~~===~[I ~U~N[I
~20~0~5~-I=A~PT~.~A~C~A~D~E~·
M~I~C~A~Y~C~U~L~TU~R~A~G~E~N~E~R~A~L~r~

48. CULTO 53. Señale la alternativa que considera dos re-


A) adoración D) liturgia presentantes del progreso científico en los inicios
de la Edad Moderna:
B) devoción E) veneración
A) Descartes - Bacon.
C) ermita
B) Copérnico - Galileo.
C) Kant - Hegel.
49. GRATUITO D) Einstein - Russell.
A) arbitrario D) libre E) Newton - Owen.
B) caprichoso E) pueril
COMUNICACiÓN y LENGUAJE
C) infundado
54. ¿Cuál es el adverbio que se utiliza para expre-
sar simultaneidad?
50. BALANCEAR
A) Al igual que
A) bambolear D) equilibrar
B) Luego que
B) mecer E) ondular C) Tan pronto
C) oscilar D) inmediatamente que
E) Ya que
OULTURA GENERAL 55. La palabra subrayada que cumple la función
de sustantivo está en la alternativa:
HISTORIA UNIVERSAL A) Vino puntualmente a la universidad.
51. Señale la alternativa que completa adecuada- B) El incómodo vehículo llegaba tarde.
mente el enunciado siguiente: C) El jefe se incomodó por su tardanza.
El primer período de la Edad se denomina D) No incomodo a usted con mi presencia. cver-
.......... , y se entiende aproximadamente hasta el dad?
10000 a. c. El término viene de las voces griegas E) Para degustar vino, viaja a lea en marzo.
" " que significa antiguo, y "lithos", que quie-
re decir " ". 56. Señale el enunciado redactado correctamente.

A) Antigua - Grecolatino - Hellas - latino. A) Narrados en una lengua exuberante, sus nove-
las y relatos ofrecen una combinación de lo
B) Media - Bizantino - Bizancio - durable. real y lo fantástico definida como "realismo
C) de Piedra - Paleolítico - palaios - piedra. mágico".
D) de Bronce - Neolítico - neo - bronce. B) Narradas en una lengua exuberante, sus nove-
E) de los Metales - Mesopotamia - meso - desierto. las y relatos ofrecen una combinación de lo
real y lo fantástico, definida como "realismo
52. El Humanismo de los siglos XIV y XV se dis- mágico".
tingue por: C) Narradas en una lengua exuberante, sus nove-
I. El retorno a la cultura grecolatina en lo artísti- las y relatos ofrece una combinación de lo real
co y literario. y lo fantástico definidos como "realismo mági-
co".
n. La concepción de que la ciencia es el factor
fundamental del progreso. D) Narrado en una lengua exuberante, sus novelas
IlI. La reacción contra las formas verticales y dog- y relatos ofrece una combinación de lo real y lo
máticas en el campo religioso. fantástico definido como "realismo mágico".
E) Narrada en una lengua exuberante, sus novelas
A) Sólo 1 B) Sólo n C) Sólo III
y relatos ofrecen una combinación de lo real y
D) 1 Y III E) n y III lo fantástico definido como "realismo mágico".
~:~~~~·l UN12005-1 APT. ACADÉMICA Y CULTURA GENERAL I:=~~~;'~
57 _ Cuántos vicios o liSOS incorrectos encuentra
é C) Su población.
en las expresiones siguientes? D) Sus recursos pecuarios y pesqueros.
- No tuvimos otra solución que declararlo inapto. F.) Su ubicación estratégica a medio camino en-
El pobre no tenía ni genio ni cacumen. tre Europa y Asia.
- Tomó su carné y corrió como un gamo, los de-
62. En la costa peruana se distinguen los siguien-
más salieron tras él.
tes climas:
A) Ninguno B) Uno. C) Dos.
A) Semitropical y Subtropical.
D) Tres. E) Cuatro.
B) Yunga y Suni.
58. En qué oración se hace uso de la coma expli-
é C) Rupa-Rupa y Omagua.
cativa? D) Suni y Rupa- Rupa.
A) El ambiente, desde una visión holística, es el E) Yunga y Rupa-Rupa,
aire, las plantas, los animales, las personas, los
ríos, etc. 63. Los volcanes del mundo se agrupan en dos
B) En cambio, a los que piensan en forma super- zonas completamente definidas, una de ellas es el
. ficialles preocupa la piedra que cae. Círculo de Fuego del Pacífico y la otra zona es:
C) La reflexión en voz alta, siempre ejercito. A) El Cinturón de América del Sur.
D) Si nos aprecian, que nos inviten. B) El Cinturón Alpino - Caucásico - Himalayo.
E) El ambiente incluye a los edificios, las calles, C) El Cono Volcánico de Asia.
los puentes y muchas otras cosas más.
D) El Círculo de Fuego del Atlántico.
E) El Círculo Volcánico de Africa.
GEOGRAFíA
59. Indique a cuál de los siguientes grupos de or-
ganismos económicos internacionales pertenece el PSICOLOGíA
Perú. 64. La socialización es un proceso de adaptación
A) OEA, CAN, ALCA, NAFfA. progresiva y constante del individuo a su medio
B) ALADI, ALCA, MERCO SUR. socio cultural, que se caracteriza fundamentalmen-
te por:
C) CAN, ALADI,.APEC, ONUDI.
D) NAFfA, CAN, OEA, ONUDI. A) El aprendizaje de costumbres y valores en la
infancia.
E) NAFfA, CAN, ONUDI, MERCOSUR.
B) La progresiva asimilación de conocimientos en
60. En Lima, según el censo de 1993, residía la formación escolar y profesional.
aproximadamente: C) El aprendizaje de normas y creencias en la fa-
A) el 40% de la población del país. milia.
B) el 32% de la población del país. D) La integración de los individuos a la comuni-
dad.
C) el 20% de la población del país.
D) el 15% de la población del país. E) Ser permanente debido a la situación cambiante
del medio.
E) el 12% de la población del país.
65. Indique el autor según el cual la personalidad
61. Los recursos Ó características naturales que tiene tres componentes básicos: el ello, el yo y el
constituyen las principales ventajas competitivas del
super-yo
Perú son:
A) Hipócrates. D) Krestschmer.
A) Sus recursos mineros y pesqueros.
B) Freud. E) Pavlov.
B) Su megadiversidad biológica, rnicro-climas, y
recursos minero-energéticos. C) Jung.
~~~=~I UNI 2005-1 APT. ACADÉMICA Y CULTURA GENERAL 1,;:~:~~=;i:;:::;:;~¡~Z'
C) Paloma de papel.
FILOSOFíA Y LÓGICA D) Piratas en el Callao.
66. Dadas las siguientes afirmaciones sobre la ver- E) No se lo digas a nadie.
dad, señale cuál es correcta.
70. Nombre del actual Ministro de Trabajo:
I. Según el pragmatismo, una proposición es ver-
dadera si es útil. A) José Ortiz.
Il. Según el positivismo, una proposición es ver- B) Juan Sheput.
dadera si corresponde a los hechos. C) Javier Neves,
IlI. Según el escepticismo una proposición es ver- D) Alfonso Velásquez.
dadera si se puede comprobar.
E) Javier Sota.
A) Sólo I B) Sólo n C) Sólo III
D) I Y n E) n y III 71 . Se denomina Taki Onqoy:
A) El calendario inca, basado en la observación
67. Si se define que el significado de una palabra del Sol y la Luna.
es determinado por sus usuarios, señale la afirma-
ción verdadera: B) El movimiento de resistencia a la conquista
española, encabezado en la decáda de 1560
I. El significado de una palabra es uno y el mis- por sacerdotes andinos.
mo a través del tiempo.
C) El "mundo de arriba", en donde residen los
Il, El significado de una palabra varía de acuerdo
astros, según la cosmovisión andina.
al contexto.
IlI. El significado de una palabra varía a través del D) La ceremonia de invocación a las lluvias en el
tiempo. Imperio Incaico.
E) La fiesta por el reparto de la tierra en el Impe-
A)I Y n B) I Y III" C) 1, 11, III
rio Incaico.
D) Sólo I E) n y III
72. A mediados del siglo XIX la ocupación del
espacio amazónico se basaba en el supuesto de que:
HISTORIA DEL PERÚ (Y ACTUALIDAD)
A) Era un espacio desocupado.
68. ¿Con qué finalidad se estableció durante la B) Se reconocerían los derechos de los pueblos
Colonia los "Colegios de Caciques"? nativos.
I. Preparar altos funcionarios para las Audien- C) Era necesaria una operación militar.
cias y Capitanías en los nuevos territorios con- D) Era una región de fácil acceso.
quistados.
E) era una zona de proyección minera y comercial.
n. Preparar intermediarios políticos y culturales
para garantizar la continuidad del aparato de
73. Entre los siguientes elija el sitio religioso más
dominación colonial. .
antiguo de la cultura andina:
IlI. Difundir la educación entre la población indí-
A) La Huaca del Sol.
gena de ciertas regiones.
B) El Templo de Kalassasaya.
A) Sólo I B) Sólo n C) Sólo III·
C) El Templo de Chavín.
D) I Y n E) II Y III
D) El Templo de Kotosh.
69 . .Nombre de la película nacional de gran éxito
E) Pachacamac.
de taquilla en el mercado limeño actual:
A) Dina Páucar.
B) El aviador.
A) Amortización de la deuda externa.
ECONOMIA POLlTICA B) Desembolsos de préstamos del extranjero.
74. Dada una situación de equilibrio en el merca- C) Exportaciones de servicios.
do, si se observara un aumento sólo en la cantidad D) Inversión extranjera directa.
demanda de un producto sin que varíe ningún otro
factor de la demanda, habría ocurrido: E) Inversión extranjera en cartera.

A) Un desplazamiento expansivo de la demanda.


B) En desplazamiento contractivo de la demanda. LITERATURA
C) Un desplazamiento contractivo de la oferta. 78. Una de las siguientes novelas relata las des-
venturas de un niño mestizo llevado por su padre a
D) Un desplazamiento expansivo de la oferta.
una ciudad de los Andes a estudiar al colegio, don-
E) La estabilización de la oferta. de conoce la exclusión y opresión de los indios por
los gamonal es.
75. Una devaluación del Nuevo Sol (aumento de
la cantidad de nuevos soles a pagar por cada uni- A) Herencia
dad monetaria extranjera, sea dólares, euros o B) Yawar fiesta
yenes) ocasionaría en la economía peruana: C) Aves sin nido
A) Un aumento de las importaciones y una dismi- D) Los ríos profundos
nución de las exportaciones.
E) El mundo es ancho y ajeno
B) Un aumento de nuestras exportaciones y una
disminución de las importaciones.
79. Complete: "El Ingenioso Hidalgo Don Quijote
C) Un incremento en el déficit comercial externo de la Mancha" de Miguel Cervantes y Saavedra, ha
del Perú. cumplido .... años y tiene como tema principal ....
D) Una disminución de los ingresos de los A) 400 años satirizar las novelas de caballería.
exportadores peruanos.
B) 400 años combatir el idealismo.
E) Un aumento de los ingresos de los importadores
peruanos. C) 500 años mostrar los dos lados del espíritu hu-
mano, idealismo y materialismo.
76. La previsión del déficit fiscal para el 2005 es D) 500 años mostrar los peligros de una imagina-
de 1% del PBL Esto quiere decir que: ción excesiva.
A) El PBr 2005 será 99% de lo que sería sin el E) 500 años acabar con la literatura de caballerías.
déficit.
B) El monto no cubierto del gasto de gobierno, 80. Señale la correspondencia correcta entre no-
llegará a una cifra 1% menor que los ingresos velas y autores.
fiscales. A) Paradiso - Ciro Alegría.
C) El gobierno tomará el 1% del PBI para finan- B) Pedro Páramo - Juan Rulfo.
ciar el déficit.
C) El mundo es ancho y ajeno - José María
D) El gasto gubernamental no cubierto por ingre- Arguedas.
sos fiscales llegará a una cifra equivalente al
1% del PBL D) La muerte de Artemio Cruz - Grabiel García
Márquez.
E) El PBI llegará a ser 101 % de lo previsto.
E) Un mundo para Julius - Julio Ramón Ribeyro.
77. é Cuál de los siguientes rubros se registra en
la balanza en cuenta corriente?
SOLUCIONARIO
APTITUD ACADÉMICA 3. Se define el operador:
a~'b=mill{a-b; b-a}
RAZONAMIENTO MATEMATlCO
Ejemplo:
1.
2"5=min(2-5 ;5-2)
~
y = 12m+n-q I ,.. (1)
= min( -3 ;,3)
= -3
En las expresiones:
[}] =3b-1 .. ' (2)
1) Es asociativo si:
Aplicando el operador (1) para resolver: a*(b':'c)=(a':'b)"c ; \;fa,b,cEZ ¡Falso!
Ejemplo:
~ = 12(-2)+(-2)-(-2) 1 3"(4*5)=3':'[min(4-5; 5-4)1

\Y =3':'(-1)

=8] =min[3-(-1) ;(-1)-31


= -4
De (2): = 3( -4)-1
(3"'4)"5=[min(3-4; 4-3)]"5
=-13 ." (3)
=(-1)*5
Aplicando lo obtenido en (3) en el ejercicio:

\:7-1
=mim[(-1)-5; 5-(-1)J
"V = -6
-2 _2-1
~ 3*(4':5);.:(3':'4)':'5
-13
-2
11) Es conmutativo si:
De (1): = 12(2)+(-1)-(-13)1 a'" b = b" a ; \;fa, b , C E Z ¡Correcto!
Ejemplos:
=[}I] • 3"4=min(3-4 4-3)
De (2): =3(16)-1 =min(-l;l)
= 47 = -1
Clave: E
• 4"3=min(4-3; 3-4)
2. Del enunciado, si:
=min(l; -1)
t>(x-3)=x+7
= -1
= (x - 3) + 10 .., (*) III) (-1)*(1*3)=-1 ¡Correcto!
En la expresión: (-1)'" (min(l- 3 ; 3 -1)) =-1
E=t>(x-2)-t>(x+6) (-1) ,: ( -2) = -1

De (*): = [( x - 2) + 10 ]- [( x + 6) + 10] min(-1-(-2);-2-(-1))=-1

= -8 min(l; -1)=-1
Clave: C -1 = -1 (Cumple)
I UNI 2005-1 APT. ACADÉMICA
~~ª¿- .~' Y CULTURA GENERAL I:;:iª~E..;.<8>.
Sobre este operador se puede afirmar que II y III Este ángulo corresponde a dos vueltas completas
son correctos. (2 x 3 60u) ; por tanto la posición del gráfico será
Clave: C igual que la primera.
Clave: C
4. En la serie gráfica:
7. En el primer par de figuras:

En la primera gráfica muestra cero (O) triángulos,


la segunda un (1) triángulo, la tercera dos (2), Yla
que sigue debe mostrar tres (3) triángulos:

Para obtener la figura de la derecha la región externa


ha girado 90° en sentido antihorario, y la región inter-
na ha girado 90° grados en sentido horario.
Clave: C Para obtener el otro par de figuras se debe ejecu-
tar las mismas operaciones de rotación:
5. En la serie gráfica:

Externamente la figura geométrica va aumentan-


do un lado, y en su interior, se muestran inicial-
Clave: E
mente 4 segmentos de líneas (forman un cuadra-
do), que luego van disminuyendo de 1 en l.
8. Completando los cuadros en blanco:
Considerandolo anterior, en el casille- I r\ I
ro en blanco debe ir la figura: CJ
Clave: O

(1) (2) (3) (4) (5) (6) (7) (8) (9)


6.
(1) (2) (3) (4) (5) Pasol.- Para pasar de la secuencia (1) a la (2), el
"triángulo y el círculo intercambian posi-
ción y color.

~~~~~ Paso 2.- Para pasar a la secuencia (3), la figura


geométrica (triángulo o círculo), que se
~~t0~ ubica a la izquierda o arriba, avanza una
posición en sentido horario.
Para obtener la figura de la posición 9:
Paso 3.- Para pasar a la secuencia (4), la otra fi-
2da. =(2-1)90°=90° gura geométrica, que se ubica a la dere-
3ra. =(3-1)90°=180° cha o abajo, avanza una posición en sen-
tido horario.
4ta. = (4 -1)90° = 270°
Paso 4: Se sigue el paso 1 y subsiguientes.
Clave: E
~~::;~:=-¡¡¡¡¡.
r,¡¡¡.·1 UN12005-1 APT. ACADÉMICA Y CULTURA GENERAL I
9. Para saber en cuantos días terminará la obra Para determinar el caudal que circula par la tubería,
Enrique, si trabaja sólo, se tiene la información: se necesita conocer el área de la sección transversal
(A) y la velocidad (v) con la que fluye el líquido. La
I) Trabajando juntos lo acabarían en 18 días
información (1) es repetitiva, pues se dá en el enun-
i o dá ninguna información para Enrique sólo!
ciado, sólo se necesita la información (lI).
11) Trabajando juntos lo acabarían en 18 días, pero Clave: B
como Enrique se enfermó, Saúlla acabó en 45
días. 12. "Todas los rectángulas (R) san paralelogramos"
Esta información no da el rendimiento de los (P).
dos con respecto al total de la obra, y además
el rendimiento por si sólo de Saúl. El rendi-
miento de Enrique se puede deducir a partir
de este dato:
Clave: B

10. Datos:
Precio de venta de computadora 1: PVj = $ 800
Ningún
Precia de venta de computadora 2: PV2 = $ 800 elemento
Para saber si ganó o perdió se debe tener la ganan-
cia a pérdida en cada una de las computadoras. Su negación:

GI = PVj - Pc! ... (1)

G2 = PV2 - Pe2 ... (2)

Infor .•nación:
1) En la primera computadora, se ganó 25%
Gl = O,25Pc¡
Al menos un
11) En la segunda computadora se perdió el 25% elemento

G2 = -0,25 PC2 "Algunos rectángulos no. son paralelograrnos".

Para calcular las ganancias y ganancias en (1) y Clave: C


(2), se necesita la información de 1 y II.
13. Si consideramos:
Clave: C
Fico (F), Daniel (D), Pedro (P), Toño (n,
11. El caudal de un líquida se define como: Carlos (C), Marca (M).
Del enunciada:
Caudal = Volumen
Tiempo. • "Fico está al este de Daniel"
a también: N
Q=t'-
T D F
AL
T
0----+----'" E
= Av
Donde: A: Área de la sección transversal donde
fluye el líquido
v: Velocidad con que fluye el líquido. s
•.._1':.~ UN12005-1 APT. ACADÉMICA Y CULTURA GENERAL ::: =-~
• "Daniel está al norte de Pedro" • "Daniel está al norte de Toña"

N
N

e F e
--K
D.._--+--_ F D
,
0--'-:, --+----- o E
E T+,
, M--p,
~P ,
S
s Luego se puede afirmar que: "Fico está al noreste
de Toña"
• "Pedro está al sur de toña"
Clave: B
N
T~
, 14. Por condición del problema se sabe que uno
Dt---+-- ...• de ellos contestó todas las preguntas correctamen-
F
te, y el otro contestó todas incorrectamente, por
O.-~----r------- E tanto sus respuestas son opuestas. En el cuadro:
T* I
Pregunta Ornar Roxana Xiomara
p.
I
I 1 V V F
s 2 V F F
3 F· F V
• "Carlos está al este de Daniel"
~
N Respuestas opuestas
T* I
Ornar o Xiomara contestaron correctamente, y
e F
--x
e Roxana necesariamente debe haber contestado dos
D
respuestas correctas (1 y 3), que deben coincidir
o E con el que contestó todas correctamente:
T* I
Del cuadro se deduce el orden de mérito de los tres
p.
I alumnos será: 1) Ornar, 2) Roxana ,3) Xiomara.
I

S Clave: B

• "Marco está al oeste de Pedro" 15. Del enunciado:

N • "Eva (E) vive entre IBED


T~ Bety (B) y Doris (D)"
e F e DB
D --K

• "Lili (L) no vive en el último piso"; "El cuarto piso


o E
T* I
está ocupado por Ana (A)"
M_p
I
I

S
~ ~~I UN12Ó05-1 APT. ACADÉMICA Y CULTURA GENERAL I~~:'
• "Ana (A) vive entre Doris (D) y Lili (L)
18. Primera relación:
6
L S [li]5l [2I1J.[ilil ~
A 4
~ ~~ ~
D 3 -BD 9-6=3 10-6=4 8-3=5 13-Y =6

E 2 _E ~Y=7

B 1 - DB Segunda relación:

Como el sexto piso queda libre, entonces éste debe [illSJ ITDJ ~ [Z!¿J
estar ocupado por Cecilia:
[TIIJ ~ ITTIJ [21ill
e 6 6+5=11 6+1=7 3+5 =8 7+4=X
~x=11
L S
Finalmente:
A 4
D 3 x+y=1l+7
E 2 = 18
B 1 Clave: O

De las proposiciones se puede afirmar que: 19.


I) Cecilia vive en el sexto piso (Verdadero) 2 ; 5 ; 17 , 71 ; 359
Il) Bety vive en el tercer piso (Falso)
III) Doris no vive en el tercer piso (Falso) bd~117x4+l~
IV) Bety vive en el primer piso (Verdadero)
Clave: O
Clave: O
20. Datos:
16. Costo del cuadro sin marco: e
x3 x2 x1 Costo del marco sólo :m
,...-----...... ,...-----...... ,...-----......
Del enunciado, el costo de cuadro y su marco:
2 . 7 . 6· 9 . 12· 13 . x . y
'~~~ e+m = 240 (soles)
+2 +4 +6
~~ m = 240 - e ... (")
+2 +2
Con un marco a mitad de precio del anterior, el
x=12x1=12 cuadro costará 180 soles, es decir:
z=13+6=19
e + ~m = 180
Luego: z-x=19-12
=7 De (*): C+~(240-C)=180
Clave: A ==} e = 120 (soles)

Clave: C
17.
21. Según enunciado:
1
4
. ~
' 2'
. 5 .
,
13 .
,
30'
)
65
6 menbrillos {MaYOr peso

1 ¡x2+1 f hx 2+2 f} 5x2+3 {}13X2+4{}30X2+5{

Clave: B
5( 1 kg
kg ururano

8 menbrillos {Meno~ peso


kg umtarIo,
G léZ
UN12005-1 APT. ACADÉMICA Y CULTURA GENERAL ::~::::iid:;~;i:~
En 4 docenas de membrillos (4 x 12 = 48) , se Federico: 6 naranjas ---7 2 soles
obtendrá el mayor peso cuando se toma los mem-
Miguel: 6 naranjas ---7 3 soles
brillos de mayor peso unitario (6 menbrillos/kg),
Total de e/ venta: 12 naranjas ---7 5 soles

M = 48 membrillos Clave: E
mayor menbrillos
6 kg 24. Según el enunciado:
= 8 kg Longitud de la viga: L
Clave: 8 La viga se divide en 4 partes iguales: 1..
4
22. Del enunciado graficamos:
Una de las cuatro partes se divide en 3 partes iguales:
1 tira
--H- 1crn .1.+3=_L_
4 4(3)

1 30cm
Una de estas partes se divide en 2 partes:

_L_+2=
4( 3)
.i:
24
La diferencia entre una de las primeras partes y 1<1
última parte mide 5 m, entonces:

~J
1---15 crn=150 rnrn---j
y
150 tiras
L(1.
4
l.-~=sm
4
_l)
24
24
= sm

L=24m
Clave: E
El total de tiras de 1 mm de ancho que se obtiene
del papel será de 150, y su largo de cada una de 25. Del enunciado:
ellas es de 30 cm.
Las 5 (5 x l ) primeras preguntas:
La longitud que se obtiene al unir a lo largo cada
1 a f- 1 clave
una de estas tiras, equivale a sumar cada una de
longitudes de las tiras. 2 b f- 1 clave
L=ls0x30cm 3 c f- 1 clave
4 d f- 1 clave
=4s00cm
5 e f- 1 clave
= 45 m
Clave: A Las siguientes 10 (5 x 2) preguntas:
6
23. Del enunciado a} 2 claves iguales
7 a
Federico: 3 naranjas ---7 1 sol 8
Miguel: 2 naranjas ---7 1 sol 9 bb} 2 claves iguales

Al asociarse, como los dos tienen la misma canti-


14
dad de naranjas, éstas también deben colocarse en :} 2 claves igual es
la misma cantidad en cada venta que ejecuten, en- 15
tonces:
0r·w me .••,..;1 UN12005-1 APT. ACAQÉMICA y CULTURA GENERAL I~:=::=::·w._=
'-=='-'-
Las siguientes is (S x 3) preguntas:

~~ : 13
18 • a
claves iguales
RAZONAMIENTO VERBAL
ORACIONES INCOMPLETAS
19 • b
26. "El prejuicio es una actitud positiva o nega-
tiva hacia un grupo de personas por sus caracterís-
28 ticas sociales reales o imaginarias.
29
30 :1 3 claves iguales
Nos da una definición del prejuicio hacia un grupo
sociaL
Las siguientes 20 (S x 4) preguntas: Clave: E
31
32 27. "La autoridad es el poder que tiene una per-
4 claves iguales
33
34
;) sona sobre otra que le está subordinada".
Nos da una definición de "La autoridad", como re-
lación entre dos personas.

47 Clave: O
48
49
SO
¡) 4 claves iguales 28. La dipsomanía es la necesidad
de tomar bebidas alcohólicas.
irresistible

Define la Dipsomanía recurriendo a la etimología,


Las siguientes 25 (S x S) preguntas: Dipso: sed; Manía: deseo irresistible.
51 Dipsómano: Persona que consume licor en exceso
52
53
54
SS

75
~)
e
S claves iguales

COMPRENSiÓN

letras es una publicación


DE LECTURA
Clave: C

29. Del texto se deduce que la revista Patio de


de investigaciones
humanísticas, "independiente y plural", y que ade-
Las siguientes 30 (S x 6) preguntas: más por su extenso contenido cultural se convier-
76 a te en una revista de primera línea.
77 a Clave: 8
78 a
6 claves iguales 30. Según el texto, los sucesos que se dieron obe-
79 a
80 a decen a una acción de los cocaleros en alian-
81 a za con los narcotraficantes, que existen en .
la selva, pues la muerte de los tres policías se dió
82
en vísperas de la operación policial que destruyó

87 :¡ 6 claves iguales 20 pozas de maceración; el inicio de una huelga


protestando por varios asuntos, entre ellos la su-
88 puesta depredación del medio ambiente por erra-
89 dicación de la coca; y por último la oposición terca
90
:1 6 claves iguales al tratado del TLC con los EE.UU, que es un socio
importante de la lucha contra el narcotráfico.
Clave: C Clave: E
~.,
GmZ
MFWW?d:::1
W••••• UNI 2005-1 APT. ACADÉMICA Y CULTURA GENERAL I~~~~ ~

31. De las cinco alternativas la que no es compa- IV) Proceso 2 de funcionamiento - aire de mezcla
tible con el texto: "Se vive una época de distinción combustible y luego se inflama.
patente entre las palabras y las cosas". El conteni- J) Se produce la propulsión.
do del texto afirma todo lo contrario, se vive una Clave: C
mixtura de cosas y palabras, poder e imaginación,
realidad material y construcción lingüística.
ELIMINACiÓN DE ORACIONES
Clave: B
36. El tema que trata en cuatro de las alternativas
es sobre la comunicación, donde se define y se da
COHERENCIA Y COHESiÓN TEXTUAL
sus elementos. La alternativa que no tiene relación:
32. ASESINAN A 8 EN TRUJILLO
1) "La lengua es un conjunto de signos arbitrarios
IV) Lugar de los acontecimientos - Bar "El Barro", que utiliza una comunidad para comunicarse".
distrito La Esperanza.
Como puede deducirse, en esta última oración, el
1) Fecha de acontecimiento y llegada de camio- tema central es la lengua y no la comunicación.
neta con delincuentes.
Clave: A
III) Los delincuentes bajan de la camioneta - 15 su-
jetos 37. El Tema que se trata en las oraciones es el
11) Los delincuení'es entran al bar y disparan a 9 conocimiento de la composición o estructura del
personas. carbón.
V) A consecuencia de los disparos mueren 8 per- La oración que no guarda la misma relación:
sonas y una queda herida, 111) "El carbón ha sido desplazado por el petróleo
Clave: D como combustible fósil".
33. EL PAPAENFERMO Esta oración trata sobre el carbón; pero ocupándo-
111) Inicio de sus problemas de salud - 1 de febrero se del uso como combustible en la sociedad.

11) Después de 14 días, médico personal pide Clave: C


traquetomía
38. El tema que se ventila en las oraciones es so-
V) Lo vuelven a internar para la intervención - 24 bre el "cambio" en las diferentes áreas del ámbito
de febrero.
económicos empresarial; a excepción de la oración:
1) Después del último internamiento sólo existen
1) "La idea de que "lo único constante es el cam-
tres partes médicos: 25/02 ; 28/02 Y 03/03
bio" aún se remonta a los tiempos de Heraclito".
IV) Los partes médicos los firma el portavoz papal.
Esta oración nos habla sobre el cambio pero dentro
Clave: C de un contexto histórico.
34. LA TEORIA DEL BIG BANG Clave: A
I1I) Definición del Big Bang
39. El tema que se ocupa las oraciones es sobre
V) Inicio del fenomeno la evolución de la matemática, desde un nivel
11) Causas que lo originan impírico hasta el nivel abstracto y científico.
IV) Resultados
La oración que debe eliminarse:
1) Algunas controversias sobre esta teoría.
11) "El surgimiento de las matemáticas es una con-
Clave: C secuencia de la necesidad de solucionar pro-
35. EL MOTOR TURBORREACTOR blemas diversos de caracter pragmático".
V) Definición o denominación Esta última oración se ocupa del surgimiento de la
matemática.
III) Complementa definición
II) Proceso 1 de funcionamiento - aire entra a Clave: B
compresores.
'LA
UJII!Id
GlI"ni.Z

CONECTORES LÓGICOS - TEXTUALES ANALOGíAS


40. El texto trata sobre la preferencia de los turis- 44. ACRISOLAR: Significa purificar; reducir una
tas por visitar el cuzco, causa por la cual el gobier- cosa al estado de pureza separando lo impuro
no plantea promocionar otras zonas turísticas y con o extraño.
ello mejorar la condiciones sociales y económicas ENSUCIAR: Manchar, poner impurezas en una
de las pobladores de estas zonas. El texto se com- cosa.
pleta así: Estas dos palabras mantienen una relación de
"Debido a que los turistas sólo van al cuzco, el antonomía, por lo tanto en las alternativas debe-
Gobierno ha creado un programa cultural para mos seleccionar el par de palabras:
difundir otras zonas turísticas; así, ellos podrían Acendrar: Depurar, purificar, dejar sin mancha ni
apreciar la riqueza cultural del país y mejorarían defecto.
la condición socio económica de sus pobladores".
Macular: Ensuciar, manchar una cosa.
Clave: A
Clave: C
41. El contenido del texto trata sobre el resultado
45. En cada par de palabras: ARGUEOLOGÍA:
adverso de un encuentro de fútbol, pero que la gente
RUINAS; BOTÁNICA: VEGETALES; CONTABILI-
abriga la esperanza de un buen resultado en el
DAD: COSTOS; se observa que existe una relación
próximo encuentro, e incluso conseguir la clasifi-
de una ciencia o disciplina y su elemento que estu-
cación. El texto se completa así:
dia, manteniendo éste orden.
"Apesar de que el resultado del último encuen-
En las alternativas, el par de palabras que mantie-
tro no fue el esperado, la gente abriga esperanzas
ne la misma relación mencionada, es: derecho:
para el próximo partido, ya que en el fútbol,
leyes
como en otras actividades, los hechos resultan im-
predecibles por lo que la clasificación para el Clave: C
mundial no deja de ser una posibilidad.
46. En los pares de palabras: ZURDO: DIESTRO;
Clave: A VANIDAD: HUMILDAD; RUDEZA: CORTESÍA; exis-
ten una relación de antonomía. En las alternativas
42. El texto trata sobre lo negativo que podría ser el par de palabras que mantiene esta relación es
el tomar una posición política, porque ésta se en- nada y todo.
cuentra corrupta, pero que esta realidad es sólo cir-
cunstancial. El texto se completa así. Clave: E

" Expresar una posición política resulta hoy no bien


vista porque el hombre público la ha corrompido; TÉRMINO EXCLUIDO
mas ello no debe asombramos, dado que dicha 47. MALO: Que muestra una actitud negativa, o
animadversión es sólo conyuntural". carece de bondad.
Clave: C De las alternativas la que no guarda una similitud.
de significado es haragan, pues esta significa que
43. El texto trata sobre un grupo de personas que rehuye al trabajo.
se encontraban esperando por aprobar su exámen Clave: B
médico, pero quedaron paralizados por un explo-·
sión. El texto se completa así: 48. CULTO: Honor que se tributa religiosamente
"Las víctimas hacían cola para aprobar un control a lo que se considera divino o sagrado.
médico y postular a un empleo público, cuando En las alternativas la palabra que no guarda rela-
quedaron paralizadas por la explosión". ción de significado común es ermita; pues ésta se
refiere a un templo pequeño.
Clave: B
Clave: C
;% me';:::;;::::;:;r:~1 UNI 2005-1 APT. ACADÉMICA Y CULTURA GENERAL I:=':;=::~
49. GRATUITO: Arbitrario, sin fundamento, ca- n. La concepción de que la ciencia es el factor
pricho. fundamental del progreso. FALSO
De las alternativas, la palabra que no guarda simi- III. La reacción contra las forma~ verticales y dog-
litud en su significado es la palabra libre, cuyo sig- máticas en el campo religioso. VERDADERO
nificado es que tiene la facultad para obrar o no Clave: D
obrar, o que está suelto.
. 53. En los inicios de la edad moderna destacaron
Clave: D
dos científicos:
50. BALANCEAR. Que se mueve haciaun lado y Nicolás Copérnico: De origen polaco quien con
luego hacia el otro. sus observaciones sobre las tradicionales concep-
En las alternativas, el conjunto de las palabras nos ciones sobre el universo, se opuso a la iglesia. Me-
dan una idea común de movimiento de vaivén, a diante la observación y el cálculo explicó el
excepción de la palabra equilibran, la cual nos dá heliocentrismo.
la idea de algo que se encuentra en condiciones de Galileo Galilei: De origen Italiano fue uno de los
igualdad. pioneros del método experimental, demostrando
ser un gran 'matemático, físico y astrónomo, cons-
Clave: D
truyó un telescopio con el cual estudió la luna, yel
sol. También estudió el centro de gravedad de los
sólidos.
Clave: B
CULTURA GENERAL
COMUNICACIÓN Y LENGUAJE
HISTORIA UNIVERSAL
54. El adverbio se define como la parte invaria-
51 . La edad de piedra se caracteriza por que los ble de la oración cuya función consiste en comple-
pobladores de la pre-historia usaron la piedra para mentar la significación del verbo, de un objeto o de
preparar sus armas y herramientas. En el primer otro adverbio. El adverbio que se utiliza para ex-
periodo llamado "Paleolítico" se dedicaron a la caza; presar la simultaneidad es ya que.
y, en el segundo periodo llamado "Neolítico" cono- Ejemplo: Compra una revista ya que estas en la li-
cieron la agricultura. brería.
En el enunciado: Clave: E
El primer período de la Edad de piedra se deno-
55. La palabra subrayada que cumple la función
mina Paleolítico, y se entiende aproximadamen-
de sustantivo:
te hasta el 10000 a. c. El término viene de las voces
griegas "palaios" que significa antiguo, y "lithos'', A) Vino puntualmente a la universidad. (verbo)
que quiere decir "piedra". B) El incómodo vehículo llegaba tarde. (adjeti-
Clave: C vo)
C).El jefe se incomodó por su tardanza. (atribu-
52. El Humanismo, que apareció en Italia en el
to)
siglo XIv, fue un movimiento ideológico liderados
por la burguesía y el capitalismo mercantilista que D) No incomodo a usted con mi presencia. ¿ver-
emergía. Este se distinguió por: dad? (Verbo)
E) Para degustar vino, viaja a lea en marzo. (sustan-
- Retorno a la cultura Greca - Latina tivo) .
- Rechazo al dogmatismo de los religiosos. Clave: E
- Le dio gran importancia a la literatura y filosofía, etc.
56. El texto de la alternativa A:
En las alternativas:
Narrados en una lengua exuberante, sus novelas y
I. El retorno a la cultura grecolatina en lo artísti- relatos ofrecen una combinación de lo real y lo
co y literario. VERDADERO fantástico definida como "realismo mágico".
~:=:::=;=I,-U_N_12_0_0_5_-I ...•
Expresado en otra forma: 61. Las cuatro regiones naturales (200 millas de
"Sus novelas y relatos narrados en una lengua mar, costa, sierra y selva), y los microclima que se
exuberante, ofrecen una combinación de lo real y generan en cada uno de ellas, han hecho que en el
lo fantástico definida como realismo mágico". Perú se cuente con una gran diversidad en su fau-
na y flora, al punto que actualmente estamos au-
Clave: A mentando las exportaciones. También es importante
resaltarel gas de camisea que además de solucio-
57. En el texto:
nar los problemasenergéticos internos, se exporta-
"No tuvimos otra solución que declararlo inapto. rá a algunos países.
El pobre no tenía ni genio ni cacumen".
La megadiversidad biológica, microclimas
"Tomó su carné y corrió como gamo, los demás y recursos minero-energéticos, mencionados
salieron tras él". anteriormente, constituyen ciertas ventajas compe-
La palabra inapto no está reconocida, la palabra titivas para el Perú.
que debería usarse es inepto que significa no apto. Clave: B
Este uso incorrecto se llama barbarismo.
Clave: B 62. En la costa peruana se distinguen dos climas:
Clima semitropical: La temperatura media
58. Las comas explicativas se utilizan al inicio y anual es de 24° C y con lluvias en los meses de
final de un texto aclaratorio del tema central. Las verano. Este clima corresponde a los departamen-
comas explicativas podrían reemplazarse, algunas tos de Piura y Tumbes.
veces, por paréntesis. Clima subtropical: Presenta una temperatura
Las comas explicativas se usan en el texto: media anual de 18° C y con presencia de lloviznas
y neblina la mayor parte del año. Este clima co-
A) "El ambiente, desde una visión holística, es el aire,
rresponde él los demás departamentos de la costa.
las plantas, los animales, las personas, los ríos, etc.
Clave: A
Clave: A
63. Los volcanes del mundo se agrupan en dos
GEOGRAFíA zonas completamente definidas, una de ellos es el
59. El Perú pertenece a los siguientes organismos círculo de fuego del Pacífico y la otra es el Cintu-
económicos internacionales: rón Alpino - Caucásico - Himalayo, cuyo nom-
Comunidad Andina de Naciones (CAN). Se bre deriva de las tres cadenas montañosas.Los Al-
funda en 1969 y lo integran: Venezuela, Colombia, pes, los montes Caucaso y los montes Himalayas.
Ecuador, Perú y Bolivia. Clave: B
Asociación Latinoamericana de integración
(ALADI). Esta conformado por 12 paises, entre PSICOLOGíA
ellos el Perú. 64. La socialización es un proceso de adaptación
Foro de Cooperación Económica Asia Pací- progresiva y constante del individuo a su medio
fico (APEC). Se inician en 1989, pero el Perú re- ambiente sociocultural, que se caracteriza funda-
cién lo integra en 1998. mentalmente, por ser permanente debido a la si-
Organización de la Naciones Unidas para tuación cambiante del medio. La sociedad va carn-
el Desarrollo Industrial (ONUDI). Organis- biando día a día debido a los grandes adelantos de
mo que pertenece a la ONU, y que se especializa en la ciencia, y el individuo debe someterse a un apren-
promover el desarrollo industrial de los países en dizaje permanente.
desarrollo. Clave: E
Clave: C
65. Los tres componentes básicos de la personali-
60. Según el censo de 1993 la población que resi- dad: El ello (id), elyo (ego), el super yo (super ego),
día en Lima fue de aproximadamente el 32% de la son sustentados en el modelo psicoanalítico de la
población de todo el Perú. personalidad de Sigrrrun Freud.
Clave: B Clave: B
· l&mmJmmWf:1g&%,.~,...6..
UNI 2005-1 APT. ACADEMICA y CULTURA GENERAL;!@ifdJt!li!iifb@li!iMV~

rurales para garantizar la continuidad del aparato


FILOSOFíA Y LÓGICA de dominación colonial. Una de sus funciones era
66. la imponer la educación moral de España.
I. "Según el pragmatismo, una proposición es ver- Clave: B
dadera si es útil". CORRECTO.
69. El nombre de la película nacional de gran éxi-
Pragmatismo: Esta filosofía afirma que el
to de taquilla en el mercado limeño actual es Pira-
único criterio para juzgar la verdad de cual-
tas del Callao, con una asistencia de mas de
quier doctrina se ha de fundar en sus efectos cien mil espectadores. Es la primera película ani-
prácticos. Esta teoría es sustentada por Willian
mada por computadora en América Latina, se basa
James. sobre un cuento creado por Hernán Garrido Lecca.
II. "Según el positivismo, una proposición es ver-
Clave: D
dadera si corresponde a los hechos". CORREC-
TO.
Positivismo: Según este pensamiento filo- 70. El nombre del actual ministro de trabajo
(2005) es Juan Sheput, quien es miembro del par-
sófico, el espíritu humano no debe renunciar
a conocer el ser mismo de las cosas, y conten- tido Perú Posible.
tarse con verdades obtenidas de la observación Clave: B
y de la experiencia. Esta corriente filosófica
fue sustentada por Agusto Comte. 71. Se denomina Taki Ongoy al movimiento de
resistencia a la conq uistu, española, encabezado en
III. "Según el escepticismo una proposición es ver-
la década de I 560 por sacerdotes a ndinos, cuyo ob-
dadera si se puede comprobar". FALSA.
jetivo era la del retorno al culto andino, rechazan-
Escepticismo: Afirma que la verdad no exis- do los cultos cristianos.
te, o si existiera, el hombre es incapaz de co-
Esta rebelión se dio a consecuencia de la disminu-
nocerla.
ción de la población indígena y fue liderada por
Clave: D Juan Chocne en las zonas de Ayacucho, Apurimac
67. y Cuzco.
I. "El significado de una palabra es uno y el mis- Clave: B
mo a través del tiempo". (FALSA)
El significado de una palabra puede variar con .72. A mediados del siglo XIX la ocupación del
el transcurso del tiempo, tal es el caso de la espacio amazónico se basaba en el supuesto de
idea sobre la palabra "Tierra". que era una zona de proyección minera y comer-
Il. "El significado de una palabra varía de acuer- cial. En esta época el gobierno peruano se preocu-
do al contexto". (VERDADERO) pó de conocer la flora, fauna y recursos naturales,
Una palabra asume un significado dependien- con el propósito de preveer la continuidad de la
do de la función gramatical que esté ejecutan- balanza económica que se tenía por la explotación
do, también del tema que esté tratando. del guano de las islas.

Ejemplos: Él se encuentra en el lago. Clave: E


III. El significado de una palabra varía a través del
73. El sitio religioso mas antiguo de la cultura
tiempo. (VERDADERO)
andina es el templo Ceremonial del Fuego Sagra-
Clave: E do, llamado Templo Kotosh, considerado el más
antiguo de América (2500 a.e): se ubica en lo que
ahora es el departamento de Huánuco. Los descu-
i HISTORIA DEL PERÚ (Y ACTUALIDAD)
'" bridores de este templo fueron los historiadores:
68. Durante la Colonia establecida en América por Seiichi Izumi y Kasuo Terada.
parte de España, se creó los "Colegios de Caciques"
con el fin de preparar intermediarios políticos y cul- Clave: D
~===~=J UNI 2005-1 APT. ACADÉMICA Y CULTURA GENERAL li:;=:i:~:~i~:i::;:
- Flujo de Bienes (Balanza Comercial)
meZ

ECONOMíA POLÍTICA - Flujo de Rentas (Interés, ganancias, dividen-


.74. Dada una situación de equilibrio en el merca- dos, etc)
do, si se observa un aumento sólo en la entidad de - Flujo de servicios no financieros (Balanza de
un producto sin que varie ningún otro factor de la servicios)
demanda, entonces habría ocurrido un desplaza- - Flujo de donaciones (Entradas y salidas)
miento expansivo de la oferta, tal como se muestra Por lo expuesto, las exportaciones de servicios
en el gráfico. se registran en la balanza de cuenta corriente.
Precios(P) Clave: C

LITERATURA
78. En la novela Los ríos profundos (1956)
Po
José María Arguedas relata las desventuras de un
p¡ -------- niño mestizo llevado por su padre a una ciudad de
los Andes a estudiar al colegio, donde conocen la
exclusión y opresión de los indios por los
gamonales. En esta novela el autor narra sus pro-
pias vivencias.
Cantidad de
productos (Q) Clave: D

o: Oferta; D: Demanda 79. "El Ingenioso Hidalgo Don Quijote de la Man-


Clave: D cha" de Miguel de Cervantes y Saavedra, ha cum-
plido 400 arios y tiene como tema principal la de
75. La devaluación de la moneda nacional ocasio- satirizar las novelas de caballería.
naría en.la economía peruana un aumento de la ex- Clave: A
portaciones y una disminución de las importaciones;
ya que el exportador estaría más estimulado con sus 80. En cuanto a novelas y autores:
ganacias, inclusive podría bajar los precios de sus pro- A) Paradiso - Ciro Alegría. Falso
ductos en el extranjero y mejoraría su competividad.
Lo contrario le sucedería al importador, pues a éste La novela Paradiso corresponde al escritor cu-
le sería mas caro la compra en dólares, puesto que bano José Lezama Lima.
éstos han subido de precio. B) Pedro Páramo - Juan Rulfo. Verdadero
Clave: 8 Juan Rulfo es un autor mexicano.
C) El mundo es ancho y ajeno - José María
76. Si la previsión del déficit fiscal para el 2005
Arquedas. Falso.
es eje 1% del PBI; esto quiere decir que el gasto
gubernamental no cubierto por ingresos fiscales lle- La novela El mundo es ancho y ajeno es obra
gará a una cifra equivalente al 1% del PBI. del autor peruano Ciro Alegría.

El déficit fiscal se da cuando los gastos del gobier- D) La muerte de Artemio Cruz - Grabiel García
no supera a sus ingresos y para medir su implican- Márquez. Falso
cia en la economía nacional, se mide porcentual- Esta novela corresponde al autor mexicano Car-
mente con respecto al PB!. los Fuentes.
Clave: D E) Un mundo para Julius- Julio Ramón Ribeyro.
Falso.
77. La balanza en cuenta corriente registra las
transacciones económicas y comerciales que reali- Esta novela también corresponde al mexicano
Juan Rulfo.
za internacionalmente un país. Esta registra las si-
guientes operaciones: Clave: 8
UNI 2005-11 APT. ACADÉMICA Y CULTURA GENERAL =~~::~:!::~

'"ACADÉMICA y CULr·

APTITUD ACADÉMICA
RAZONAMIENTO MATEMÁTICO A) B)
\ ¿~
C) D) E)

ni a+3 rnl 5. Indique la alternativa que no guarda relación con


1 • Si ~ = @] y t:J = 3a - 1. las demás.

Determinar el valor de "r'' ,en: W =7


A) 1. B) 1. C) 3 D) 7 E) 11
S 2
A) B) C) D) E)
2. Si se cumple que:
6. Indique la alternativa que debe ocupar la posi-
O ,si A + B > 10 ciónNo.7.
A®B= { 100 , si A + B ::; 10

Determine el número de valores que puede tomar R,


para
xe{S;6;7; ... ;lS}, y={4;S;6; ... ;lS};
mIIH~!m
posición1 posición2 posición3
?
posición4 ... posición7

x ey son números enteros.

A) 12 B) 14
R = x ®(y + 1)

C) 16
x
D) 18 E) 20
mEEmE
A) B) C) D) E)

7. Identifique la alternativa con la figura que com-


3. Si mq)11=Sm-2@ y ® =10-2b pleta la serie.
Determine el valor de "k" en: k <p 4 = 11
A) O B) 1 C) 2 D)3 E)4

4. Si las figuras de los recuadros I y II tienen la


misma relación analógica, determine la figura que
debe ocupar el casillero Z.

RECUADRO I RECUADRO I

También podría gustarte